Sei sulla pagina 1di 672

For Civil Services

Preliminary Examination

Indian Polity
Question Bank

9789353161484.indd 1 8/7/2018 7:52:32 PM


About the Editor
 inay G B: Vinay G B did his schooling at Jawahar Navodaya Vidyalaya (JNV) Chitradurga. Years spent
V
at JNV shaped his personality and outlook towards life. After completion of graduation in Genetics, he
decided to become an IAS officer where he saw an opportunity to serve more people and at the same time
enhance his knowledge through self-study. He soon resigned from his job and started his online initiative
to provide right guidance to thousands of
students across India free of cost and thereby produce finest
leaders—whose primary motto will be to serve the society—to the nation.
Today, more than three thousand students are being guided by InsightsIAS under the mentorship of
Vinay G B in Bengaluru. These students include graduates from India’s top institutions such as IITs, IIM
and other top medical, law, engineering, science and arts colleges and universities. This book is a product
of Insights team effort guided and edited by Vinay G B. Hope it helps IAS aspirants to fine tune their
UPSC exam preparation.

9789353161484.indd 2 8/7/2018 7:52:32 PM


For Civil Services
Preliminary Examination

Indian Polity
Question Bank
Edited by:
Vinay G B

McGraw Hill Education (India) Private Limited


Chennai
McGraw Hill Education Offices
Chennai New York St Louis San Francisco Auckland Bogotá Caracas
Kuala Lumpur Lisbon London Madrid Mexico City Milan Montreal
San Juan Santiago Singapore Sydney Tokyo Toronto
Indian Polity Question Bank P.iii

9789353161484.indd 3 8/7/2018 7:52:32 PM


McGraw Hill Education (India) Private Limited
Published by McGraw Hill Education (India) Private Limited
444/1, Sri Ekambara Naicker Industrial Estate, Alapakkam, Porur, Chennai 600 116

Indian Polity Question Bank

Copyright © 2018 by McGraw Hill Education (India) Private Limited.


No part of this publication may be reproduced or distributed in any form or by any means, electronic, mechanical, photocopying,
recording, or otherwise or stored in a database or retrieval system without the prior written permission of the publishers. The program
listings (if any) may be entered, stored and executed in a computer system, but they may not be reproduced for publication.

This edition can be exported from India only by the publishers,


McGraw Hill Education (India) Private Limited.

 1 2 3 4 5 6 7 8 9        22 21 20 19 18
Printed and bound in India.
Print Edition
ISBN (13): 978-93-5316-148-4
ISBN (10): 93-5316-148-7

Information contained in this work has been obtained by McGraw Hill Education (India), from sources believed to be reliable. However,
neither McGraw Hill Education (India) nor its authors guarantee the accuracy or completeness of any information published herein, and
neither McGraw Hill Education (India) nor its authors shall be responsible for any errors, omissions, or damages arising out of use of this
information. This work is published with the understanding that McGraw Hill Education (India) and its authors are supplying information
but are not attempting to render engineering or other professional services. If such services are required, the assistance of an appropriate
professional should be sought.

Typeset at NuWave eSolutions Pvt. Ltd., New Delhi-110019 and printed at

Cover Design: Creative Designer


Visit us at: www.mheducation.co.in
Write to us at: info.india@mheducation.com
CIN: U22200TN1970PTC111531
Toll Free Number: 1800 103 5875

9789353161484.indd 4 8/7/2018 7:52:33 PM


Preface

InsightsIAS is proud to present the first edition of Indian Polity-Prelims Mock Test questions module.
It has the most commonly asked questions in competitive exams on the polity subject. It is a must-
read book for aspirants appearing in various competitive examinations, especially the civil services
examinations.
The book has been compiled with the objective of helping an aspirant to sail through the difficult
journey of UPSC. It contains quality objective questions on the Indian Constitution and International
relations, segregated from Insights Prelims Test Series previous years papers, the credibility of which is
acknowledged by many UPSC toppers, along with the detailed explanations for the questions.
The questions are of UPSC standards and have been framed from all possible angles so that they
help an aspirant to fine tune his/her preparation through which he/she can test his/her understanding of
concepts and facts. Questions have also been categorized topic-wise so that it aids in revision.
Salient Features:
1.  Separate sections on different topics of the subject.
2.  UPSC questions on Indian Polity for Preliminary exam.
3.  Practice questions on Indian Polity for Preliminary exam.
4.  2000+ quality questions.
5.  Detailed explanations.
6.  Topic-wise question categories in sync with the standard textbook.
7.  Last 15 years (from 2000 to 2016) UPSC prelims questions.
The secret of success in UPSC exam is to read limited resources and revise it over and over again.
Time management is very important for this exam and when you have a friend who can reduce your
burden by giving you the UPSC Standard Question Bank to study, your work is made smart and bit easy.
This book is that friend for you.
We hope our efforts will help you strengthen your base in current affairs and prepare you well for
the exam.

‘Your Success is found in Your Daily Routine’


All The Best !

P.v

9789353161484.indd 5 8/7/2018 7:52:33 PM


9789353161484.indd 6 8/7/2018 7:52:33 PM
Table of Contents
Prefacev Sovereignty36
Socialist37
1 HISTORICAL BACKGROUND 1 Secular38
The Regulating Act of 1773 1 Republic41
Amending Act (1781)—Act of Settlement 2 Democratic41
Pitt’s India Act of 1784 2 Justice48
Charter Act of 1793 & Charter Act of 1813 2 Equality48
Charter Act of 1833 3
5 UNION AND TERRITORY – PART 1 49
Charter Act of 1853 5
Government of India Act 1858 6 State Reorganization 50
Indian Councils Act 1861 6 Evolution of States & Union Territories 52
Indian Councils Act of 1892 7
6 CITIZENSHIP – PART II –
Indian Councils Act of 1909 7
ARTICLEs 5–11 53
Government of India Act 1919 9
Rowlatt Act 11
7 FUNDAMENTAL RIGHTS –
Government of India Act of 1935 12
PART III – ARTICLEs 12–35 55
Indian Independence Act 14
Interim Government (1946) 14 International Covenant 58
Miscelleneous15 FR Available to Citizens & Aliens 59
FR Available to Citizens 59
2 MAKING OF THE CONSTITUTION 16 Definition of State 62
Demand for Constituent Assembly 16 Article 13 63
Composition of Constituent Assembly 17 Right to equality – Articles 14,
Working of the Constituent Assembly 19 15, 16, 17 & 18 63
Objective Resolution 21 Right to Freedom Granted under
Committees of Constituent Assembly 22 Articles 19 to 22 67
Changes by the Independence Act 23 Right Against Exploitation 72
Other Functions Performed 23 Right to Freedom of
Miscelleneous24 Religion – Articles 25–28 73
Right of Minorities – Articles 29 & 30 74
3 SALIENT FEATURES OF THE Article 32 – Right to Constitutional
CONSTITUTION25 Remedies77
Importance of Constitution 26 Article 34 – Martial Law 83
Features of Indian Constitution 27 Right to Property 84
Schedule30
8 DIRECTIVE PRINCIPLES OF
Sources31
STATE POLICY – PART IV –
Switzerland Constitution 32
ARTICLEs 36–51 84
4 PREAMBLE 33 Features of the Directive Principles 85
Ingredients of Preamble 34 Directive Principles of State Policy 88
Preamble as Part of Constitution and DPSP versus Fundamental rights92
it’s Amendability 35 Implementation of DPSP 93

P.vii

9789353161484.indd 7 8/7/2018 7:52:33 PM


9 FUNDAMENTAL DUTIES — CENTRAL GOVERNMENT
PART IVA – ARTICLE 51A 93 (PART V) (ARTICLE 51–151) 141
Features of Fundamental Duties 95
16 PRESIDENT – ARTICLE 52–78 141
10 AMENDMENT OF CONSTITUTION — Presidential Elections 142
PART XX – ARTICLE 368 96 Qualification 145
Procedure for Amendment 96 Oath145
Simple Majority 101 Conditions of Office 146
Special Majority + State Consent 102 Impeachment  147
Basic Structure of the Constitution 103 Vacancy in President’s Office 148
Few Amendments 105 Powers and Functions of the President 148
Veto Power 160
Miscellaneous163
SYSTEM OF GOVERNMENT 107
17 VICE-PRESIDENT OF INDIA –
11 PARLIAMENTARY SYSTEM 107
ARTICLE 63–71 164
Difference between Britain &
Elections164
Indian Parliamentary System 111
Qualification 165
Presidential System 112
Term of Office 166
12 FEDERAL Government  112 Vacancy Occurred in Office 166
Powers and Functions 167
Federal System 112
Indian and American Vice President
Indian Model of Federalism 115
Miscellaneous167
Unitary System 120
18 PRIME MINISTER 168
13 CENTRE-STATE RELATION –
PART XI & PART XII 121 Appointment168
Oath170
Legislative Relations 121
Powers and Functions 171
Executive Relations 125
Miscellaneous174
Financial Relations 126
19 COUNCIL OF MINISTERS 176
14 INTERSTATE RELATIONS 128
Article 74 – COM to Aid and
Inter-State Water Disputes 129
Advice President 177
Inter-State Council – Article 263 130
Article 75 – Collective Responsibility
Freedom of Trade, Commerce
and Other Provisions 178
and Intercourse – Part XIII –
Composition of COM 180
Article 301 to 307 131
Cabinet181
Zonal Councils 132
20 CABINET COMMITTEES 182
15 EMERGENCY PROVISIONS –
PART XVIII – ARTICLE 352 TO 360 133
21 CABINET SECRETARIAT 184
National Emergency 133
President’s Rule 136 22 PARLIAMENT 185
Financial Emergency 140
Organisation of Parliament
Composition of Two Houses 185

 P.viii For Civil Services Preliminary Examination

9789353161484.indd 8 8/7/2018 7:52:33 PM


System of Election Lok Sabha 188 27 CHIEF SECRETARY OF
Duration of Two Houses 190 THE STATE 279
Membership of Parliament 192
Presiding Officers of Parliament 195
SPECIAL STATUS FOR
Leaders in Parliament 203
Sessions of Parliament 204
STATES – PART XXI –
Devices of Parliament 208 ARTICLE 370 and 371 279
Legislative Procedure 218 28 SPECIAL STATUS OF JAMMU
Joint Sitting of Two Houses 221 AND KASHMIR – PART XXI –
Budget in Parliament 223 ARTICLE 370 279
Funds226
Multifunctional Role of Parliament 230 29 SPECIAL PROVISION FOR
Position of Rajya Sabha 234 SOME STATES  284
Office of Profit–Article 102 238
Parliamentary Privileges 239 Article 371–Part xxi 284
Sovereignty of Parliament 240 Special Category States (SCSs) 286
Committees of Parliament 242
Secretariat249 UNION TERRITORIES AND
Parliamentary forums250 SPECIAL AREAS 286
Miscellaneous250
30 UNION TERRITORIES—
PART VIII – ARTICLE 239–242 286
STATE GOVERNMENT –
PART VI – ARTICLE 152 – 237 254 National Capital Territory—
69th Constitutional Amendment 290
23 GOVERNOR – PART VI  254
31 SCHEDULE & TRIBAL AREAS 292
Appointment  255
Terms of Office and Removal  256 Fifth Schedule – Scheduled Areas 292
Condition of Office 257 Sixth Schedule  293
Powers and Functions 258
Constitutional Position 264 JUDICIARY296

24 CHIEF MINISTER 266 32 JUDICIARY IN INDIA 296


Appointment266
Powers and Functions 267 33 CRIMINAL JUSTICE SYSTEM
IN INDIA 299
25 STATE COUNCIL OF MINISTERS 268
34 SUPREME COURT 302
26 STATE LEGISLATURE 268 Appointment302
Legislative Assemblies 269 Qualification 303
Legislative Councils 270 Removal of Judges 303
Qualification and Disqualification Independence of Supreme Court 305
of Members 273 Jurisdiction and Power of
Presiding Officers of the House 274 Supreme Court 307
Legislative Process 274 American versus Indian Judiciary 314
Privileges278 Some of Cases 315
Miscellaneous278 Miscellaneous318

Indian Polity Question Bank P.ix

9789353161484.indd 9 8/7/2018 7:52:33 PM


35 HIGH COURT 320 CONSTITUTIONAL BODIES 358
Appointment & Transfer 321
44 ELECTION COMMISSION –
Tenure  322
PART XV—ARTICLE 324 358
Removal323
Independence of High Court 323 Composition358
Jurisdiction and Power of High Court 325 Independence360
Miscellaneous327 Powers and Functions 362
Miscellaneous365
36 SUBORDINATE COURT –
PART VI – ARTICLES 233 to 237 329 45 Union Public Service
Commission UPSC) 366
37 OTHER DISPUTE RESOLUTION
MECHANISMS331 46 STATE PUBLIC SERVICE
COMMISSIONS369
National Legal Services Authority 331
Lok adalats331
47 TRIBUNALS – PART XIVA –
Gram Nyayalaya 332
ARTICLES 323A AND 323B 369
Alternative Dispute Resolution (ADR)  333
Panchayat Adalats 334
48 FINANCE COMMISSION –
Nyaya Panchayat 334
PART XII—ARTICLE 280 372
38 JUDICIAL REVIEW AND Functions374
ACTIVISM334
49 STATE FINANCE COMMISSIONS
Judicial Review 334
SFCS) – PART IX & IXA —
Judicial Activism 336
ARTICLES 243I & 243Y 375
39 PUBLIC INTEREST
LITIGATION PIL) 336 50 NATIONAL COMMISSION FOR
SCHEDULED CASTES (SCs) –
PART XVI—ARTICLE 338 376
LOCAL GOVERNMENT 338
Powers376
40 PANCHAYATI RAJ – PART IX – Functions377
ARTICLE 243 TO 243-O 338
51 NATIONAL COMMISSION
73rd Constitutional Amendment 340 FOR STS – PART XVI—
Miscellaneous350 ARTICLE 338A 378
41 MUNICIPALITIES – PART IXA – Functions & Powers 379
ARTICLE 243-P TO 243ZG 350
52 SPECIAL OFFICER FOR
Types of Urban Government 352 LINGUISTIC MINORITIES –
PART XVII – ARTICLE 350B 379
42 RURAL & DISTRICT
ADMINISTRATION IN INDIA 354
53 CAG – PART V –
ARTICLE 148 TO 151 380
43 COOPERATIVES – PART IX-B –
ARTICLES 243ZH TO 243ZT 356 Appointment  380
Independence  382

 P.x For Civil Services Preliminary Examination

9789353161484.indd 10 8/7/2018 7:52:33 PM


Functions383 62 CBI – CENTRAL BUREAU
Role384 OF INVESTIGATION 404
Appointment404
54 ATTORNEY GENERAL (AG) –
Functions405
PART V – ARTICLE 76 386
Rights & Limitations of Auditor 63 LOKPAL & LOKAYUKTA 406
General of India 388 Lokpal406
Solicitor General of India 389 Lokayukta408
Miscellaneaus409
55 ADVOCATE GENERAL IN
THE STATE – PART VI –
ARTICLE 165 389 OTHER CONSTITUTIONAL
DIMENSIONS410
NON-CONSTITUTIONAL 64 OFFICIAL LANGUAGE –
BODIES390 PART XVII –
ARTICLES 343 to 351 410
56 NITI AYOG 390
Official Languages of Union 410
Official Languages of State 411
57 NHRC – NATIONAL HUMAN
Special Directives 412
RIGHTS COMMISSION 391
Languages of Judiciary 414
Appointment  392 Eighth Schedule 415
Composition392
Removal393 65 PUBLIC SERVICES – PART XIV –
Powers & Functions 393 ARTICLES 308 TO 323 416
Human Rights Court  396 Classification of Services 417
Constitutional Provisions 419
58 SHRC – STATE HUMAN RIGHTS
COMMISSION396 66 RIGHTS AND LIABILITIES
OF THE GOVERNMENT 421
59 CIC – CENTRAL INFORMATION
COMMISSION396 67 AUTHORITATIVE TEXT OF
Appointment397 THE CONSTITUTION IN
Removal398 HINDI – PART XXII 421
Powers and Functions 398
Powers and Functions 400 68 SPECIAL PROVISIONS
RELATING TO CERTAIN
60 SIC – STATE INFORMATION CLASSES – PART XVI –
COMMISSION400 ARTICLE 330 TO 342 422
Scheduled Castes and Scheduled Tribes 422
61 CVC – CENTRAL VIGILANCE
Other Backward Communities (OBC) 423
COMMISSION401
Minorities424
Appointment402 Anglo-Indians425
Removal402
Powers & Functions 402

Indian Polity Question Bank P.xi

9789353161484.indd 11 8/7/2018 7:52:33 PM


POLITICAL DYNAMICS 426 Act, 1989 469
Armed Forces Special Powers Act
69 RIGHT TO VOTE 426 (AFSPA) 1958 469
Panchayats (Extension to the
70 POLITICAL PARTIES 429 Scheduled Areas) Act (PESA), 1996 470

71 ELECTORAL SYSTEM 432 84 SOME OF THE GOVERNMENT


BODIES472
72 DELIMITATION OF Constitutional Body 472
CONSTITUENCIES435 Non-Statutory472
Statutory Bodies 473
73 MODEL CODE OF CONDUCT 438
85 COMMITTEES & COMMISSIONS 490
74 ELECTIONS 440 Committee490
Election Officers 444 Commissions492
Polls445
86 NATIONAL POLICIES 494
75 ELECTORAL REFORMS 446
87 MISCELLANEOUS 496
76 ANTI-DEFECTION LAW 448 Constitutional Provisions 507
Sectarian Ideologies – Communalism,
77 PRESSURE GROUPS 452 Regionalism, Casteism etc.  515
Good Governance 517
78 NATIONAL INTEGRATION  453 Media519
Consumer Rights 520
OTHER ASPECTS 453 Public Health 521

79 LIST 453 PREVIOUS YEAR QUESTION


PAPERS523
80 TABLE OF PRECEDENCE 455 2000523
2001525
81 NATIONAL SYMBOLS 457 2002527
National Flag 457 2003530
National Song – Vande Mataram 458 2004533
2005536
82 MINISTRIES AND DEPARTMENTS 459 2006538
2007539
83 SOME IMPORTANT ACTS 464 2008540
Right to Information Act, 2005 466 2009541
Essential Services Maintenance Act 2010543
(ESMA)467 2011543
Forest Rights Act 2006 467 2012545
Scheduled Castes and the Scheduled 2013549
Tribes (Prevention of Atrocities) 2014552

 P.xii For Civil Services Preliminary Examination

9789353161484.indd 12 8/7/2018 7:52:33 PM


2015554
2016556

INDIAN SOCIETY 558


Tribes558
Women563
Child Welfare Committees 565
International Issues 567
International Declarations 572
Cold War 574
Non-Alignment Movement 578
Soviet Union and Disintegration 579
Military Operations 581
India’s Foreign Policy 582
Bilateral Relations 586
Joint Military Exercises 591
International Organisations 591
(5) Trade Blocks 618

References657

Indian Polity Question Bank P.xiii

9789353161484.indd 13 8/7/2018 7:52:33 PM


9789353161484.indd 14 8/7/2018 7:52:33 PM
(1) HISTORICAL BACKGROUND by requiring the Court of Directors
(governing body of the Company) to
report on its revenue, civil, and military
The Regulating Act of 1773 affairs in India.
Q.1 What is the importance of the Regulating Act Q.2 The Regulating Act of 1773 is said to be
of 1773 in the Indian colonial administrative of great constitutional importance in India.
setup? Which of the following can be the reason(s)
1. It was the first step taken by the British for it?
Government to control and regulate the 1. It created a federal architecture to govern
affairs of the East India Company in India. India decentralizing the administrative
2. It designated the Governor of Bengal system.
as the ‘Governor-General of India’ and 2. It transferred control of India from
vested in him major executive powers. commercial companies to the British
3. It provided for the establishment of a government representative in India.
Supreme Court at Bombay presidency. Which of the above is/are correct?
Select the correct answer using the codes (a)  1 only (b)  2 only
below. (c)  Both 1 and 2 (d)  None
(a)  1 and 2 only (b)  1 and 3 only Solution: (d)
(c)  2 and 3 only (d)  1 only
Q.3 Under which of the following Acts, a new
Solution: (d)
Supreme Court and a court of appeal –
Justification: Statement 2: It designated
the Sadar Nizamat Adalat – was set up at
the Governor of Bengal as the ‘Governor-
Calcutta?
General of Bengal’ and created an Executive
(a)  The Regulating Act of 1773
Council of four members to assist him.
(b)  The Regulating Act of 1813
The first such Governor-General was Lord
(c)  The Regulating Act of 1883
Warren Hastings.
(d)  The Government of India Act 1858
Statement 3: It was established at Calcutta
Solution: (a)
(not Bombay) in 1774.
In India, from 1772, a new system of justice
Learning: This act is of great
was established. Each district was to have
constitutional importance as
two courts – a criminal court (faujdari adalat)
• it was the first step taken by the British
and a civil court (diwani adalat). Maulvis
Government to control and regulate the
and Hindu pandits interpreted Indian laws for
affairs of the East India Company in India.
the European district collectors who presided
• it recognised, for the first time, the
over civil courts. The criminal courts were
political and administrative functions of
still under a qazi and a mufti but under the
the Company
supervision of the collectors.
• It laid the foundations of central
A major problem was that the Brahman
administration in India.  It made the
pandits gave different interpretations of
governors of Bombay and Madras
local laws based on different schools of the
presidencies subordinate to the governor-
dharmashastra. To bring about uniformity, in
general of Bengal, unlike earlier, when
1775 eleven pandits were asked to compile
the three presidencies were independent
a digest of Hindu laws. Nathaniel Brassey
of one another.
Halhed translated this digest into English.
• It prohibited the servants of the Company
By 1778 a code of Muslim laws was also
from engaging in any private trade or
compiled for the benefit of European judges.
accepting presents or bribes from the
Under the Regulating Act of 1773, a new
‘natives’.
Supreme Court was established, while a
• It strengthened the control of the
court of appeal – the Sadar Nizamat Adalat –
British Government over the Company
was also set up at Calcutta.

Indian Polity Question Bank P.1

01-Indian Polity_Q1-412.indd 1 8/7/2018 7:39:01 PM


Amending Act (1781)—Act of (c)  Pitt’s India Act of 1784
(d) All of the above are incorrect as they
Settlement were legislated by the Central Legislative
Q.4 The Amending Act (1781) to the Regulating Assembly (CLA) and not the British
Act of 1773 had which of the following Parliament.
provisions? Solution: (c)
1. Increase in the jurisdiction of Supreme Learning: It was the Pitt’s India Act of 1784,
Court established by the Regulating but it was passed by the British Parliament,
Act 1773 not in the Central Legislative Assembly
2. Recognition of the appellate jurisdiction (CLA) of colonial India. Hence, D is
of the Governor-General-in- Council incorrect.
3. Enactment of a temporary Uniform In a bid to rectify the defects of the
Civil Code Regulating Act of 1773, the British
Select the correct answer using the codes Parliament passed the Amending Act of 1781,
below. also known as the Act of Settlement. The next
(a)  1 only (b)  2 and 3 only important act was the Pitt’s India Act.
(c)  1 and 3 only (d)  2 only 1. It distinguished between the commercial
Solution: (d) and political functions of the Company.
Justification: Statement 1: It was actually 2. It allowed the Court of Directors to manage
the opposite. The act significantly reduced the commercial affairs but created a new
the powers of the Supreme Court at Calcutta. body called Board of Control to manage
The actions of the servants of the Company the political affairs. Thus, it established a
in their official capacity were exempted from system of double government.
the jurisdiction of the Supreme Court. 3. It empowered the Board of Control to
It also separated the Governor-General- supervise and direct all operations of the
in-Council and revenue matters from the civil and military government or revenues
Court`s jurisdiction. of the British possessions in India.
Statement 2: The Act recognized the appellate Thus, the act was significant for two
jurisdiction of the Governor- General-in- reasons:
Council. It empowered the Governor-General • the Company’s territories in India were
and Council to convene as a Court of Record for the first time called the ‘British
to hear appeals from the Provincial Courts possessions in India’and
on civil cases. It meant that appeal could • British Government was given the
be taken from the provincial courts to the supreme control over Company’s affairs
Governor General & Council and that was to and its administration in India.
be the final court of appeal.
Statement 3: The Act asserted that Charter Act of 1793 and Charter
Mohammedan cases should be determined
by Mohammedan law and Hindu law
Act of 1813
applied in Hindu cases. So, the number 3 is Q.6 Consider the following statements.
clearly wrong. 1. The Charter Act of 1793 gave powers
to the Governor General to override his
Pitt’s India Act of 1784 council.
2. The Charter Act of 1813 allowed the
Q.5 Which of these enactments by the British British subjects access to Indian shore
Parliament provided the British Government with their ships.
ultimate control over Company’s affairs and Which of the above is/are correct?
its administration in India? (a)  1 only (b)  2 only
(a)  Indian Councils Act 1861 (c)  Both 1 and 2 (d)  None
(b)  Charter Act of 1853 Solution: (c)

 P.2 For Civil Services Preliminary Examination

01-Indian Polity_Q1-412.indd 2 8/7/2018 7:39:01 PM


Justification: Statement 1: It also exclusive legislative powers for the entire
empowered him to exercise effective control British India.
over the Presi­dencies. Through this Act the The laws made under the previous acts
British introduced the concept of a civil law were called as Regulations while laws
enacted by a secular human agency, i.e., the made under this act were called as Acts.
government and applied universally in place 3. It ended the activities of the East India
of the personal rule of the past rulers. Company as a commercial body, which
In 1786, the Presidencies were divided became a purely administrative body.
into districts and collectors were appointed. It provided that the company’s territories
Statement 2: The most important development in India were held by it ‘in trust for His
that came as a consequence was the deprival Majesty, His heirs and successors’.
of the monopoly of trade and commerce of 4. The Charter Act of 1833 attempted to
the British East India Company except tea introduce a system of open competition
and throwing open the trade of India to all for selection of civil servants, and
the British citizens. stated that the Indians should not be
debarred from holding any place, office
Charter Act of 1833 and employment under the Company.
However, this provision was negated after
Q.7 Consider the following statements about the opposition from the Court of Directors.
Charter Act of 1833.
Q.8 Consider the following statements about the
1. The act created, for the first time, a
administrative situation in India before the
Government of India having authority
enactment of the Charter Act 1833.
over the entire territorial area possessed
1. The Governor-General was appointed by
by the British in India.
the Court of Directors of the East India
2. It ended the monopoly of the East India
Company (EIC).
Company as a commercial body.
2. The Saint Helena Act temporarily
3. It separated, for the first time, the
abolished the offices of Governor-General
legislative and executive functions of the
and Viceroy to promote decentralization
Governor-General’s council.
in India.
Choose the correct answer using the codes
Which of the above is/are correct?
below.
(a)  1 only (b)  2 only
(a)  1 and 2 only (b)  2 and 3 only
(c)  Both 1 and 2 (d)  None
(c)  1 and 3 only (d)  All of the above
Solution: (a)
Solution: (a)
Justification: The Regulating Act of 1773
Charter Act of 1833
created the office with the title of Governor-
This Act was the final step towards
General of the Presidency of Fort William, or
centralisation in British India. The features
Governor-General of Bengal to be appointed
are
by the Court of Directors of the East India
1. It made the Governor-General of Bengal
Company (EIC).
as the Governor-General of India and
• The Court of Directors assigned a Council
vested in him all civil and military powers.
of Four (based in India) to assist The
Thus, the act created, for the first time, a
Governor General, and decision of council
Government of India having authority
was binding on Governor General during
over the entire territorial area possessed
1773–1784.
by the British in India.
The Saint Helena Act 1833 (or
Lord William Bentinck was the first
Government of India Act 1833) re-
governor-general of India.
designated the office with the title of
2. It deprived the governor of Bombay and
Governor-General of India. So, the
Madras of their legislative powers. The
Governor-General of India was given number 2 is incorrect.

Indian Polity Question Bank P.3

01-Indian Polity_Q1-412.indd 3 8/7/2018 7:39:01 PM


• After the Indian Rebellion of 1857, the Via this act, the charter was renewed for
company rule was brought to an end, and 20 years but the East India Company was
the British India along with princely states deprived of its commercial privileges which
came under the direct rule of the Crown. it enjoyed so far.
The Government of India Act 1858 created This act made the Governor General of
the office of Secretary of State for India in Bengal the Governor General of British India
1858 to oversee the affairs of India, which and all financial and administrative powers
was advised by a new Council of India. were centralized in the hands of Governor
Q.9 The Governor-General of India was given General-in-Council.
exclusive legislative powers for the entire Q.12 Consider the following about the Charter Act
British India by this Act, and in effect the of 1833 which was an important step towards
act created for the first time, a Government centralisation and administrative reforms in
of India having authority over the entire British India.
territorial area possessed by the British in 1. It deprived the Governor-General of India
India. The Act referred here was? of his legislative powers and vested it in a
(a)  Government of India Act 1935 nominated assembly.
(b)  Act of 1919 2. It provided for reservation of Indians in
(c)  Act of 1909 the covenanted civil services.
(d)  Charter Act of 1833 Which of the above is/are correct?
Solution: (d) (a)  1 only (b)  2 only
Q.10 The Governor-General of Bengal became the (c)  Both 1 and 2 (d)  None
Governor- General of India by: Solution: (d)
(a)  Government of India Act of 1858 Q.13 Consider the following statements.
(b)  Indian Councils act of 1861 1. It made the Governor-General of Bengal
(c)  Pitts India Act of 1784 as the Governor-General of India and
(d)  Charter Act of 1833 vested all civil and military powers in him.
Solution: (d) 2. The Governor-General of India was given
Q.11 Which among the following was NOT a exclusive legislative powers for the entire
provision of Charter act of 1833? British India.
(a) An Indian should be appointed as The provisions above were made in which of
defence member in the Governor General the following Acts?
in Council. (a)  Charter Act of 1833
(b) Trading activities of the East India (b)  Charter Act of 1853
Company were to be abolished. (c)  Government of India Act of 1858
(c) The designation of the supreme authority (d)  Pitt’s India Act of 1784
was to be changed as the Governor Solution: (a)
General of India in Council. Q.14 Ministry of Law and Justice is the oldest limb
(d) All law making powers were to be of the Government of India dating back to the
conferred on Governor General in (a)  Charter Act of 1833
council. (b)  Government of India Act 1935
Solution: (a) (c) First Constitutional Amendment, 1952
Justification: Macaulay was Secretary to the (d)  Regulating Act 1773
Board of Control from 1832 until 1833. He Solution: (a)
could not have provided for the association Learning: This was enacted by the British
of Indians directly with the British Parliament. The said Act vested for the first
administration. So, the answer can be easily time legislative power in a single authority,
arrived at as (a). namely the Governor General of Council.
However, the act opened civil services • By virtue of this authority and the
competition to Indians also. authority vested under him under the

 P.4 For Civil Services Preliminary Examination

01-Indian Polity_Q1-412.indd 4 8/7/2018 7:39:01 PM


Indian Councils Act, 1861 the Governor (a)  Charter Act of 1853
General in Council enacted laws for the (b)  Indian Councils Act of 1861
country from 1834 to 1920. (c)  Indian Councils Act of 1892
• After the commencement of the (d)  Charter Act of 1833
Government of India Act, 1919 the Solution: (a)
legislative power was exercised by the Features of the 1853 Act
Indian Legislature constituted there under. 1. It separated, for the first time, the
• This shows the early importance of the legislative and executive functions of the
Ministry of Law and Justice in shaping Governor- General’s council. It provided
India’s legal regime. for addition of six new members called
Q.15 The first Law Commission was established, legislative councillors to the council.
under the Chairmanship of Lord Macaulay In other words, it established a separate
which recommended codification of the Governor-General’s legislative council
Penal Code and the Criminal Procedure which came to be known as the Indian
Code, by the (Central) Legislative Council. This
(a)  Government of India Act 1935 legislative wing of the council functioned
(b)  Charter Act of 1833 as a mini-Parliament, adopting the same
(c)  Law Commission Act, 1967 procedures as the British Parliament.
(d)  Act of 1919 Thus, legislation, for the first time,
Solution: (b) was treated as a special function of the
Learning: Law Reform in the ancient period government, requiring special machinery
was ad hoc and not institutionalised. and special process.
However, since the third decade of the 2. It introduced an open competition system
nineteenth century, Law Commissions were of selection and recruitment of civil
constituted by the Government from time to servants. The covenanted civil service
time. was thus thrown open to the Indians also.
• The first law commission was established Accordingly, the Macaulay Committee
in 1834 under the Charter Act of 1833. (the Committee on the Indian Civil
• Thereafter, the second, third and fourth Service) was appointed in 1854.
Law Commissions were constituted in (Before this the Charter Act of 1833
1853, 1861 and 1879 respectively helped attempted to introduce a system of open
adapt English Laws to Indian conditions. competition for selection of civil servants,
• The Indian Code of Civil Procedure, the and stated that the Indians should not be
Indian Contract Act, the Indian Evidence debarred from holding any place, office
and employment under the Company.
Act, and the Transfer of Property Act etc.
However, this provision was negated after
are products of the labour of the first four
opposition from the Court of Directors.)
Law Commissions.
3. It extended the Company’s rule and
• The First Law Commission of Independent
allowed it to retain the possession of
India in 1955 with the then Attorney-
Indian territories on trust for the British
General of India, Mr. M. C. Setalvad, as
Crown. But, it did not specify any
its Chairman.
particular period, unlike the previous
Charters. This was a clear indication that
Charter Act of 1853 the Company’s rule could be terminated at
Q.16 Which of the Acts passed by the British any time the Parliament liked.
Parliament introduced an open competition 4. It introduced, for the first time, local
system of selection and recruitment of civil representation in the Indian (Central)
servants in India before independence and a Legislative Council. Of the six new
separate legislative wing for the Governor- legislative members of the governor-
General? general’s council, four members were

Indian Polity Question Bank P.5

01-Indian Polity_Q1-412.indd 5 8/7/2018 7:39:01 PM


appointed by the local (provincial) 5. It constituted the secretary of state-in-
governments of Madras, Bombay, Bengal council as a body corporate, capable
and Agra. of suing and being sued in India and in
England.
Government of India Act 1858 The Act of 1858 was, however, largely
confined to the improvement of the
Q.17 Consider the following statements about the administrative machinery by which the
Government of India Act 1858. Indian Government was to be supervised
1. It abolished the East India Company. and controlled in England. It did not alter
2. It changed the entire structure of in any substantial way the system of
government in India prevailing earlier. government that prevailed in India.
3. As per the Act, Indian Government was to
Q.18 Which of the following is/are correct about
be supervised and controlled in England.
the Government of India Act 1858?
Choose the correct answer using the codes
1. It did not alter in any substantial way
below.
the system of government prevailing
(a)  1 and 2 only (b)  2 and 3 only
in India.
(c)  1 and 3 only (d)  All of the above
2. The Indian government after this Act
Solution: (c)
was to be supervised and controlled in
This significant Act was enacted in the wake
England.
of the Revolt of 1857—also known as the
Choose the correct answer using the codes
First War of Independence or the ‘sepoy
below.
mutiny’. The act known as the Act for the
(a)  Only 1 (b)  Only 2
Good Government of India, abolished
(c)  Both (d)  None
the East India Company, and transferred
Solution: (c)
the powers of government, territories and
revenues to the British Crown.
Features of the Act Indian Councils Act 1861
1. It provided that India henceforth was to Q.19 Consider the following statements.
be governed by, and in the name of, Her 1. Before the adoption of the portfolio system
Majesty. It changed the designation of in the Government of India, governmental
the Governor-General of India to that of business was disposed of by the Secretary
Viceroy of India. He (viceroy) was the of State from Britain.
direct representative of the British Crown 2. The Councils Act of 1861 led to the
in India. introduction of the portfolio system and
Lord Canning thus became the first the inception of the Executive Council of
Viceroy of India. the Governor-General.
2. It ended the system of double Which of the above is/are correct?
government by abolishing the Board of (a)  1 only (b)  2 only
Control and Court of Directors. (c)  Both 1 and 2 (d)  None
3. It created a new office, Secretary of State Solution: (b)
for India, vested with complete authority Justification: Statement 1: They were
and control over Indian administration. disposed by the Governor-General-in council.
The secretary of state was a member of The Council functioned as a joint consultative
the British cabinet and was responsible board. But, as the amount and complexity
ultimately to the British Parliament. of business of the Government increased,
4. It established a 15-member Council the work of the various departments was
of India to assist the secretary of state distributed amongst the members of the
for India. The council was an advisory Council only the more important cases being
body. The secretary of state was made the dealt with by the Governor-General or the
chairman of the council. Council collectively.

 P.6 For Civil Services Preliminary Examination

01-Indian Polity_Q1-412.indd 6 8/7/2018 7:39:01 PM


Statement 2: This procedure was legalised 3. The act made a limited and indirect
by the Councils Act of 1861 during the time of provision for the use of election in filling
Lord Canning, leading to the introduction of up some of the nonofficial seats both in
the portfolio system and the inception of the the Central and provincial legislative
Executive Council of the Governor-General. councils. The word “election” was,
• The Secretariat of the Executive Council however, not used in the act.
was headed by the Private Secretary to the The process was described as
Viceroy, but he did not attend the Council nomination made on the recommendation
meetings. of certain bodies. It provided for the
• Lord Willingdon first started the practice nomination of some non-official members
of having his Private Secretary by his of the
side at these meetings. Later, this practice • Central Legislative Council by the
continued and in November, 1935, the viceroy on the recommendation of the
Viceroy’s Private Secretary was given the provincial legislative councils and the
additional designation of Secretary to the Bengal Chamber of Commerce, and
Executive Council. • that of the Provincial legislative
• The constitution of the Interim councils by the Governors on the
Government in September 1946 brought recommendation of the district boards,
a change in the name, though little in municipalities, universities, trade
functions, of this Office. associations, zamindars and chambers.
• The Executive Council’s Secretariat was Q.21 Which of the following acts introduced the
then designated as Cabinet Secretariat. principle of elections in India?
The act made a beginning of representative (a)  Indian Councils act of 1861
institutions by associating Indians with the (b)  Indian Councils act of 1892
law-making process. It thus provided that (c)  Morley Minto Act 1909
the viceroy should nominate some Indians (d)  Government of India Act 1919
as non-official members of his expanded Solution: (b)
council. In 1862, Lord Canning, the then
Q.22 Which of the following was a salient feature
viceroy, nominated three Indians to his
of the Act of 1892?
legislative council—the Raja of Benares, the
(a) It increased the functions of legislative
Maharaja of Patiala and Sir Dinkar Rao.
councils and gave them the power of
discussing the budget.
Indian Councils Act of 1892 (b) It introduced responsible governments in
Q.20 Which of the following acts gave legislative provinces where governor was required
councils the power to discuss the budget? to act with the advice of ministers.
(a)  Government of India Act 1909 (c) It abolished diarchy in the provinces and
(b)  Government of India Act 1919 established it at the Centre.
(c)  Government of India Act 1935 (d) It introduced bicameralism in many
(d)  Indian councils Act of 1892 eleven provinces.
Solution: (d) Solution: (a)
Explanation and Learning: The Act of Justification: All options other than A are
1892 provided for the following of the Government of India Act 1935 Only
1. It increased the number of additional Option A was provided for in Act of 1892.
(non-official) members in the Central
and provincial legislative councils, but Indian Councils Act of 1909
maintained the official majority in them.
Q.23 Consider the following statements about the
2. It increased the functions of legislative
Act of 1909 which is also known as Morley-
councils and gave them the power of
Minto Reforms.
discussing the budget and addressing
questions to the executive.

Indian Polity Question Bank P.7

01-Indian Polity_Q1-412.indd 7 8/7/2018 7:39:01 PM


1. It introduced a system of communal 1. The Indian Councils Act of 1909 increased
representation. the number of elected members in both
2. It provided for the association of Indians the imperial Legislative Council and the
with the executive Councils of the Viceroy provincial legislative councils.
and Governors. 2. According to the Act an Indian was to
3. It curtailed the deliberative functions of be appointed a member of the Governor-
the legislative councils at the provincial General’s Executive Council.
level but increased the same at Central 3. The Act allowed for Voting on separate
level. budget items.
4. It relaxed the central control over the Which of the above statements is/are true?
provinces by separating the central and (a)  Only 1 & 3 (b)  Only 1
provincial subjects. (c)  Only 2 & 3 (d)  All
Select the correct answer using the codes Solution: (d)
below. Q.25 It is argued by some scholars that the
(a)  1 and 2 only (b)  3 and 4 only separate electorates for Muslims, created by
(c)  1, 2 and 4 only (d)  1, 2, 3 and 4 the colonial government crucially shaped
Solution: (a) the nature of communal politics in India. In
Justification: This Act is also known as relation to the Separate electorates, consider
Morley-Minto Reforms (Lord Morley was the following:
the then Secretary of State for India and Lord 1. With the Poona pact of 1916, the
Minto was the then Viceroy of India). Congress and the Muslim League reached
Statement 1: It introduced a system of an agreement whereby the Congress
communal representation for Muslims by accepted separate electorates.
accepting the concept of ‘separate electorate’. 2. The Indian Councils Act of 1909 provided
Under this, the Muslim members were to be separate electorates for the Muslims.
elected only by Muslim voters. Thus, the Act Which of the above statement/s is/are correct?
‘legalised communalism’ and Lord Minto (a)  1 only (b)  2 only
came to be known as the Father of Communal (c)  Both (d)  None
Electorate. Solution: (b)
Statement 2: It empowered the Governor The Lucknow Pact of December 1916 was
General to nominate one Indian member an understanding between the Congress
to the Executive Council leading to the and the Muslim League (controlled by the
appointment of Satyendra Sinha (law UP-based “Young Party”) whereby the
member) as the first Indian member. The Congress accepted separate electorates. The
Government of India Act 1919 increased the Poona Pact was signed in 1932 between
number of Indians in the council to three. Dr. Babasaheb Ambedkar and Mahatma
Statement 3: It enlarged the deliberative Gandhi by which seats were reserved for the
functions of the legislative councils at both Depressed Classes out of general electorate.
the levels. For example, members were
Q.26 Which of these acts, for the first time,
allowed to ask supplementary questions,
provided for the association of Indians
move resolutions on the budget, and so on.
with the executive Councils of the Viceroy
In addition, it also provided for the separate
and introduced the system of communal
representation of presidency corporations,
representation?
chambers of commerce, universities and
(a)  Act of 1909
zamindars.
(b)  Act of 1919
Statement 4: This was done in the Act of
(c)  Government of India Act 1935
1919.
(d)  Councils Act 1891
Q.24 Consider the following statements regarding Solution:(a)
the Indian Councils Act of 1909:

 P.8 For Civil Services Preliminary Examination

01-Indian Polity_Q1-412.indd 8 8/7/2018 7:39:01 PM


Q.27 The first attempt at introducing a The majority of members of both the
representative and popular element in Houses were chosen by direct election.
administration was made by : 4. It required that the three of the six members
(a)  Indian Councils Act, 1900 of the Viceroy’s executive Council (other
(b)  Indian Councils Act, 1909 than the commander-in-chief) were to be
(c)  Government of India Act, 1935 Indian.
(d)  Indian Councils Act, 1919 5. It extended the principle of communal
Solution: (b) representation by providing separate
electorates for Sikhs, Indian Christians,
Government of India Act 1919 Anglo-Indians and Europeans.
6. It provided for the establishment of a
Q.28 Which of the following were introduced for public service commission. Hence, a
the first time in India by the Act of 1919? Central Public Service Commission was
1. Bicameralism set up in 1926 for recruiting civil servants.
2. Direct elections
Q.29 The Government of India Act, 1919 popularly
3. Diarchy
known as Montague
Choose the correct answer using the codes
Chelmsford Act was significant in which of
below.
the following ways?
(a)  1 and 2 only (b)  2 and 3 only
1. Established a responsible Parliamentary
(c)  1 and 3 only (d)  All of the above
system of Government in the country
Solution: (d)
2. The final decision on all important
Features of the Act
questions was no more in the hands of the
1. It relaxed the central control over the
Viceroy
provinces by demarcating and separating
3. It relaxed previously stringent control of
the central and provincial subjects.
centre on provinces.
The central and provincial legislatures
Which of the above is/are true?
were authorised to make laws on their
(a)  1 and 2 only (b)  2 and 3 only
respective list of subjects. However, the
(c)  1 and 3 only (d)  3 only
structure of government continued to be
Solution: (d)
centralised and unitary.
The Morley-Minto Reforms failed
2. It further divided the provincial subjects
to satisfy the aspirations of the nationalists
into two parts—transferred and reserved.
in India inasmuch as professedly, the
The transferred subjects were to be
Reforms did not aim at the establishment of
administered by the governor with the aid
a Parliamentary system of government in the
of ministers responsible to the legislative
country and provide for the retention of the
Council. The reserved subjects, on the
final decision on all questions in the hands of
other hand, were to be administered by
the irresponsible Executive.
the governor and his executive council
II. Relaxation of Central control over
without being responsible to the legislative
the Provinces. As stated already, the Rules
Council. This dual scheme of governance
made under the Government of India Act,
was known as ‘DIARCHY’—a term
1919, known as the Devolution Rules, made
derived from the Greek word di-arche
a separation of the subjects of administration
which means double rule. However, this
into two categories—Central and Provincial.
experiment was largely unsuccessful.
Broadly speaking, subjects of all-India
3. It introduced, for the first time,
importance were brought under the category
BICAMERALISM and DIRECT
‘Central’, while matters primarily relating
ELECTIONS in the country. Thus, the
to the administration of the provinces were
Indian Legislative Council was replaced
classified as ‘Provincial’. This meant a
by a bicameral legislature consisting of
relaxation of the previous Central control
an Upper House (Council of State) and
over the provinces not only in administrative
a Lower House (Legislative Assembly).

Indian Polity Question Bank P.9

01-Indian Polity_Q1-412.indd 9 8/7/2018 7:39:01 PM


but also in legislative and financial matters. central and provincial legislatures and
Even the sources of revenue were divided widened the franchise considerably.
into two categories so that the Provinces Q.32 Which of the following were introduced or
could run the administration with the aid of provided for by the Government of India Act
revenue raised by the Provinces themselves of 1919?
and for this purpose, the provincial budgets 1. Bicameralism
were separated from the Government of 2. Direct Elections
India and the Provincial Legislature was 3. Central Public Service Commission
empowered to present its own budget and 4. Autonomy to provincial legislatures to
levy its own taxes relating to the provincial enact their budgets
sources of revenue. Select the correct answer using the codes
below.
Q.30 Which of the following statements is/are
(a)  1 and 2 only (b)  1, 2 and 3 only
true concerning the Government of India Act
(c)  3 and 4 only (d)  1, 2, 3 and 4 only
1919?
Solution: (d)
1. It introduced diarchy in the provinces.
2. It introduced for the first time the system Q.33 The Institution of Speaker and Deputy
of direct elections in the country. Speaker in India originated under the
3. It extended the principle of communal provisions of
representation to include other minority (a)  Charter Act 1891
communities too. (b)  Morley-Minto Act, 1909
4. It authorized the provinces to enact their (c)  Montague Chelmsford Act, 1919
budgets there by separating their budgets (d)  Independence of India Act, 1947
from the central budgets. Solution: (c)
Choose the correct answer using the codes The institutions of Speaker and Deputy
below Speaker originated in India in 1921 under the
(a)  1 and 3 (b)  All of the above provisions of the Government of India Act
(c)  2, 3 and 4 (d)  1,2 and 4 of 1919 (Montague–Chelmsford Reforms).
Solution: (b) At that time, the Speaker and the Deputy
Speaker were called the President and
Q.31 In the Government of India Act, 1919
Deputy President respectively and the same
1. Principle of diarchy was introduced
nomenclature continued till 1947.
2. Local bodies and provincial legislatures
Before 1921, the Governor- General of
were given autonomy.
India used to preside over the meetings of
3. Franchise base of Central as well as
the Central Legislative Council. In 1921,
provincial legislatures was increased.
the Frederick Whyte and Sachidanand Sinha
Choose the correct answer using the codes
were appointed by the Governor-General
below.
of India as the first Speaker and the first
(a)  1 and 2 only (b)  1 and 3 only
Deputy Speaker (respectively) of the central
(c)  2 and 3 only (d)  All of the above
legislative assembly.
Solution: (b)
In 1925, Vithalbhai J. Patel became the
The British themselves adopted a “carrot
first Indian and the first elected Speaker
and stick” approach in recognition of India’s
of the central legislative assembly. The
support during the war and in response to
Government of India Act of 1935 changed
renewed nationalist demands. In August 1917,
the nomenclatures of President and Deputy
the means of achieving the proposed measure
President of the Central Legislative
were later enshrined in the Government
Assembly to the Speaker and Deputy Speaker
of India Act 1919, which introduced the
respectively.
principle of a dual mode of administration,
or diarchy, in which both elected Indian Q.34 The origin of the second Chamber - the
legislators and appointed British officials ‘Council of States’ which is also known as
shared power. The act also expanded the Rajya Sabha – can be traced to

 P.10 For Civil Services Preliminary Examination

01-Indian Polity_Q1-412.indd 10 8/7/2018 7:39:01 PM


(a)  Charter Act 1833 Q.37 The Government of India Act of 1919, also
(b)  Act of 1861 known as Montagu-Chelmsford Reforms
(c)  Morley-Minto Act 1909 provided for the system of diarchy. This
(d)  Government of India Act, 1919 meant that
Solution: (d) (a) The legislative sphere of central and
Learning:  It can be traced to the Montague- provincial government was clearly
Chelmsford Report of 1918. demarcated
The Government of India Act, 1919 (b) Instead of a single legislative chamber,
provided for the creation of a ‘Council two councils (bicameralism) will take
of State’ as a second chamber of the then decisions
legislature with a restricted franchise which (c) A part of provincial legislation will be
actually came into existence in 1921. handled by the Governor and the rest by
The Governor-General was the ex-officio the legislative council
President of the then Council of State. (d)  None of the above
The Government of India Act, 1935, Solution: (c)
hardly made any changes in its composition.
Q.35 The Government of India Act 1919 led to the Rowlatt Act
adoption of an important tradition into India.
Q.38 Consider the following about Rowlatt Act.
This is
1. It allowed the British government to
(a) the finance minister presenting the whole
imprison people without due trial.
budget by reading
2. The Act banned all Indian owned presses
(b) President’s address to both houses of
from circulating news.
Parliament
3. It was repealed following the Jallianwala
(c) Use of English language in legislatures
Bagh massacre.
(d)  All of the above
Select the correct answer using the codes
Solution: (b)
below.
Provision for Address by the Head of State to
(a)  1 and 2 only (b)  2 and 3 only
Parliament goes back to the year 1921 when
(c)  1 only (d)  1 and 3 only
the Central Legislature was set up for the
Solution: (c)
first time under the Government of India Act,
Justification:  Statement 1: The Act clearly
1919. Today this can be seen when
violated the rule of law, which the British
1. the house assembles for the first time
often used to justify their administration in
2. when a new government is sworn in
India. Indian nationalists including Mahatma
3. before the budget is presented etc.
Gandhi were vehement in their opposition
Q.36 The system of ‘diarchy’ introduced by the to the Rowlatt bills due to such arbitrary
Government of India Act of 1919 meant that provisions.
(a) Both Centre and provincial legislatures Statement 2: This was not a provision.
had the power of legislate in their own Only the nationalist newspapers were
spheres. prevented from printing news. Seditious acts
(b) Both the British Parliament and the were definitely penalized by the act though.
Central Legislature had the right to make Some restrictive provisions on Press were
laws for India made by the Vernacular Press Act, 1878.
(c) Certain subjects were devolved to the Statement 3: Despite the large number of
‘Indian’ members of the Viceroy’s protests, the Rowlatt Act came into effect on
executive council and certain others to 10 March 1919.
the ‘British’ members of the executive But was repealed only in 1922.
council Learning: In Punjab, protests against this
(d) None of the above Act continued quite actively and on April 10
Solution: (d) two leaders of the movement, Dr. Satyapal
and Dr. Saifuddin Kitchlew were arrested.

Indian Polity Question Bank P.11

01-Indian Polity_Q1-412.indd 11 8/7/2018 7:39:01 PM


• To protest these arrests, a public meeting (c)  A is incorrect, but R is correct.
was held on 13 April at Jallianwala Bagh (d)  Both A and R are incorrect.
in Amritsar. Solution: (d)
• General Dyer entered the park and ordered Justification: The GoI Act 1935 did envisage
the troops to fire. states as autonomous units of governance.
• Several hundreds of people died in 1. The division of legislative powers,
this gunfire. This incident is known as between the Centre and the Provinces is of
Jallianwala Bagh massacre. special interest in view of the fact that the
However, Hunter commission was appointed division made in the Indian Constitution
to enquire into the incident. But the findings of between the Union and the States proceeds
the commission didn’t satisfy the nationalist largely on the same lines.
demands. 2. It was not a mere delegation of power by
the Centre to the Provinces as by Rules
Government of India Act of 1935 made under the Government of India Act,
1919.
Q.39 Which one of the following is not a feature of 3. As already pointed out, the Government
the Government of India Act of 1935? of India Act of 1935 itself divided the
(a) Diarchy at the Centre as well as in the legislative powers between the Central
provinces and Provincial Legislatures and, subject to
(b) A bicameral legislature the provisions mentioned below, neither
(c) Provincial autonomy Legislature could transgress the powers
(d)  An All-India Federation assigned to the other.
Solution: (a) 4. A three-fold division was made in the Act:
• The Government of India Act 1935 There was a Federal List over which the
introduced the provincial autonomy and Federal Legislature had exclusive powers
provided for an all India federation. of legislation and so forth.
• This act introduced diarchy at the central Q.41 The legislative powers of both the Central
level. It made a provision for establishment and Provincial Legislatures were subject to
of a Federal court. various limitations under the Government of
• The franchisee was extended. India Act 1935 and neither could be said to
• It divided the subjects in 3 lists. have possessed the features of a sovereign
• The Indian council was abolished and an Legislature. What were the limitations?
advisory body was introduced. 1. Apart from the Governor-General’s power
• Burma was separated from India, and of veto, a Bill passed by the Central
Aden was surrendered to British Colonial Legislature was also subject to veto by the
office. Crown
Q.40 Consider the following statements. 2. Apart from the power to promulgate
Assertion (A): The Government of Ordinances the Governor-General had
India Act 1935 never envisaged states as independent powers of legislation,
autonomous units of governance. concurrently with those of the Legislature.
Reason (R): Power was never divided Which of the above is/are correct?
between the Centre and States, it was only (a)  1 only (b)  2 only
delegated to the states by the Centre. (c)  Both 1 and 2 (d)  None
In the context of the above, which of these Solution: (c)
is correct? Learning: There were other limitations too:
(a) A is correct, and R is an appropriate The Governor-General might suspend the
explanation of A. proceedings in regard to any Bill if he was
(b) A is correct, but R is not an appropriate satisfied that it would affect the discharge of
explanation of A. his special responsibilities.

 P.12 For Civil Services Preliminary Examination

01-Indian Polity_Q1-412.indd 12 8/7/2018 7:39:01 PM


No bill or amendment could be introduced The structural part of the Constitution is, to
in the Legislature without the Governor- a large extent, derived from the Government
General’s previous sanction, with respect to of India Act of 1935. The philosophical part
certain matters, e.g., if the Bill or amendment of the Constitution (the Fundamental Rights
sought to repeal or amend or was repugnant and the Directive Principles of State Policy)
to any law of the British Parliament extending derives their inspiration from the American
to India or any Governor-General’s or and Irish Constitutions respectively. The
Governor’s Act, or if it sought to affect political part of the Constitution (the principle
matters as respects which the Governor- of Cabinet Government and the relations
General was required to act in his discretion. between the executive and the legislature)
Q.42 The Government of India (GoI) Act of 1935 has been largely drawn from the British
provided for a threefold enumeration, viz., Constitution.
federal, provincial and concurrent. The present In one of the explanations above, refer
Constitution follows the scheme of this act but to the provision of the GoI act, 1935 to
with some difference(s), which are understand how these provisions affected the
1. In the GoI Act 1935, residuary powers Indian constitution.
were given to the governor-general of Q.44 Consider the following statements.
India, which now vests with the Centre. Assertion (A): The single system of courts
2. In the GoI Act 1935, Centre had no right that enforces both Central laws as well as the
to legislate in the concurrent list without state laws has been adopted in India from the
the consent from the concerned state. Government of India Act of 1935.
Which of the above is/are correct? Reason (R): The Government of India
(a)  1 only (b)  2 only Act of 1935 established the Federal Court of
(c)  Both 1 and 2 (d)  None India with original, appellate and advisory
Solution: (a) jurisdiction.
Justification: The present Constitution In the context of the above, which of these
follows the scheme of this act but with one is correct?
difference, that is, under this 1935 act, the (a) A is correct, and R can be an appropriate
residuary powers were given neither to explanation of A.
the federal legislature nor to the provincial (b) A is correct, but R cannot be an
legislature but to the governor-general of appropriate explanation of A.
India. As of now, it vests with the Centre. The (c)  A is correct, but R is incorrect.
provision with regard to concurrent list was (d)  A is incorrect, but R is correct.
same as earlier. So, the number 2 is incorrect. Solution: (a)
Q.43 The structural part of the Constitution is, to a Justification: The Federal Court of India
large extent, derived from the Government of was a judicial body, established in India in
India Act of 1935. This argument is supported 1937 under the provisions of the Government
by which of the following features/provisions of India Act 1935, with original, appellate
of the constitution? and advisory jurisdiction. It functioned until
1. Public Service Commission of Union and 1950, when the Supreme Court of India was
States established.
2. Federal polity The Federal Court had exclusive original
3. Division of powers in lists between Centre jurisdiction in any dispute between the
and states Central Government and the Provinces.
4. Bicameralism in state assemblies Initially, it was empowered to hear appeals
Choose the correct answer using the codes from the High Courts of the provinces in the
below. cases which involved the interpretation of
(a)  1 and 2 only (b)  2 and 3 only any Section of the Government of India Act,
(c)  1 and 4 only (d)  All of the above 1935.
Solution: (d)

Indian Polity Question Bank P.13

01-Indian Polity_Q1-412.indd 13 8/7/2018 7:39:02 PM


Q.45 The Government of India Act, 1935 provided (c)  Charter Act, 1831
for (d)  Government of India Act, 1935
1. Diarchy at both Centre and Provinces Solution: (d)
2. Prohibition of direct elections for
legislatures Indian Independence Act
3. Abolition of the system of communal
representation Q.48 Consider the following statements about the
4. Bicameralism in several provinces provisions of the Indian Independence Act:
Select the correct answer using the codes 1. According to it, the princely states were
below. free to join either India or Pakistan or to
(a)  1, 2 and 3 only (b)  2 and 3 only remain independent.
(c)  4 only (d)  1, 3 and 4 only 2. The boundary between the two nations was
Solution: (c) to be decided by a boundary commission.
3. The Governor-general of both dominions,
Q.46 The Government of India Act of 1935 marked
India and Pakistan, were to be appointed
a major milestone towards establishing a
by the British Crown.
‘responsible government’ in India. What do
Which of these is/are true?
you understand by ‘responsible government’
(a)  1 and 2 (b)  2 and 3
in this context?
(c)  1 and 3 (d)  All of the above
(a) The Governor was required to act with
Solution: (d)
the advice of ministers responsible to the
These were the major provisions of Act,
provincial legislature.
other than partitioning India. It also provided
(b) Government of India took major steps
a list of provinces which were to be a part of
in promoting the welfare of vulnerable
Pakistan.
sections of Indian population
(c) The Viceroy, Governor and Ministers
would conduct themselves strictly based Interim Government (1946)
on laws Q.49 Consider the following about the Interim
(d) Bureaucracy will act as per the Government (1946) of India, whose members
instructions of the elected representatives were the members of the Viceroy’s Executive
instead of the Governor and Viceroy Council.
Solution: (a) 1. The defence portfolio was held by the
Learning: Responsible government is a President of the Council.
system which embodies the principle of 2. There was no Health portfolio in the
parliamentary accountability, such as in India. interim government.
It is also the foundation of the Westminster 3. External Affairs portfolio was managed
system of parliamentary democracy. by the Home Minister.
Government is responsible to the Select the correct answer using the codes
parliament rather than to the monarch, below.
or, in a colonial context, to the imperial (a)  1 only (b)  2 and 3 only
government. If the parliament is bicameral, (c)  1 and 3 only (d)  None of the above
then the government is responsible first to Solution: (d)
the parliament’s lower house, which is more Justification: The Viceroy’s Executive
numerous, directly elected and thus more Council became the executive branch of
representative than the upper house. the interim government. Originally headed
Q.47 Which one of the following aimed at by the Viceroy of India, it was transformed
providing a federal structure for India? into a council of ministers, with the powers
(a)  Indian Council Act, 1909 of a prime minister bestowed on the vice-
(b) Montague-Chelmsford Reforms Act, president of the  Council, a position held by
1919 Jawaharlal Nehru.

 P.14 For Civil Services Preliminary Examination

01-Indian Polity_Q1-412.indd 14 8/7/2018 7:39:02 PM


Statement 1: Defence portfolio was held • The judgments of the council till today the
by Sardar Baldev Singh who was only a fountain-source of law on many points in
member of the council. India.
Statement 2: Health portfolio existed and Q.51 Consider the following statements.
was held by Ghazanfar Ali Khan. 1. The Indian Councils Act 1861 transformed
Statement 3: External Affairs & the executive council of Viceroy into a
Commonwealth Relations was held by Pandit cabinet run on the portfolio system.
Nehru, who was the Vice-President of the 2. The members of the interim government
Council. Home, Information & Broadcasting (1946) were members of the Viceroy’s
portfolio was held by Sardar Vallabhbhai Executive Council.
Patel. Which of the above is/are correct?
(a)  1 only (b)  2 only
Miscellaneous (c)  Both 1 and 2 (d)  None
Solution: (c)
Q.50 The Judicial Committee of Privy Council, an
Justification: Statement 1: The Viceroy’s
institution under the British regime, served
Executive Council was the cabinet of the
as a bridge between the Indian and the
government of British India headed by the
English legal system, over which legal ideas
Viceroy of India. It was transformed from
travelled from England to India. Consider the
an advisory council into a cabinet run by the
following about it.
portfolio system by the Indian Councils Act
1. Before the establishment of the Supreme
1861.
Court, it was the highest forum to
Statement 2: The interim government of
entertain appeals from the orders passed
India was formed in 1946 from the newly
by the courts in India.
elected Constituent Assembly.
2. Before the enactment of Charter Act of
The members of the interim government
1891, all legislative and judicial powers
also include the Viceroy and Commander-in-
were delegated to this council by the
Chief of Britain and other members from All-
Crown.
India Muslim League. The members of the
Which of the above is/are correct?
interim government were also the members
(a)  1 only (b)  2 only
of the Viceroy’s Executive Council.
(c)  Both 1 and 2 (d)  None
Solution: (a) Q.52 India’s Constitution was drawn up under very
Justification: The Privy Council acted as difficult circumstances. With reference to the
a channel, through which English concepts making of the constitution of India, consider
came to be assimilated with Indian laws. the following statements.
• It was through this body, that the common 1. In 1928, Motilal Nehru and other Congress
law of England was introduced in India leaders drafted a Constitution for India.
under the British regime, as the base of its 2. In 1931, the resolution at Lahore Session
legal system. of the Indian Nation Congress dwelt on
• During the British regime, the King how independent India’s constitution
in Council, or Privy Council as it was should look like.
generally called, was the highest forum to 3. Both the documents (mentioned above)
entertain appeals from courts. were committed to the inclusion of
• On enactment of the Judicial Committee universal adult franchise and protection of
Act 1833, it came to be called the Judicial rights of minorities.
Committee of Privy Council. Which of the statement is/are incorrect?
• The decisions of the Judicial Committee (a)  1 Only (b)  2 Only
used to be couched in advisory form, (c)  2 and 3 Only (d)  1 and 3 Only
though, in practice, the Crown always Solution: (b)
accepted its advice, and it was unthinkable As far back as in 1928, Motilal Nehru and
that its report will not be given effect to. eight other Congress leaders drafted a

Indian Polity Question Bank P.15

01-Indian Polity_Q1-412.indd 15 8/7/2018 7:39:02 PM


constitution for India. In 1931, the resolution Learning: MN Roy had put forth the idea
at the Karachi session of the Indian National of a Constituent assembly of India in 1934.
Congress dwelt on how Independent India’s Later the INC demanded it.
constitution should look like. Both these During the Second World War, this
documents were committed to the inclusion assertion for an independent Constituent
of universal adult franchise, right to freedom Assembly formed only of Indians gained
and equality and to protecting the rights of momentum and this was convened in
minorities in the constitution of independent December 1946.
India. Between December 1946 and November
Q.53 Which of these authorities appointed the last 1949, the Constituent Assembly drafted a
Governor General of free India? constitution for independent India.
(a) Prime Minister of the Interim government Q.56 The demand for a Constituent Assembly was
(b) Constituent Assembly first made
(c)  British Crown (a) By the Indian national Congress in 1934
(d)  Predecessor Viceroy (b) At Karachi Session of the Indian National
Solution: (c) Congress
Learning: Even though India was a free (c) After the Non-cooperation movement
country after 1947, it was still a British was withdrawn
dominion, and became a republic only after (d) After the Simon Commission reached
the enactment of the constitution. India
Till this point, the British government Solution: (a)
(even though not sovereign in India) played Explanation:You can eliminate Karachi
an important role in managing Indian affairs. Session, non-cooperation movement and
C. Rajagopalachari was appointed as the Simon Commission, as all of them were
last Governor General of free India by the happened before the demand for a Constituent
British crown. Assembly was made by INC.
Q.54 Which of the following acts are concerned Q.57 The demand for a constituent assembly was
with changes to the legislative councils in made officially by Indian National Congress
British India? for the first time at
1. Act of 1909    2. Act of 1892 (a) Second Round table conference
3. Act of 1861 (b) After the departure of Simon Commission
Choose the correct answer using the codes (c)  the time of Gandhi-Irwin Pact
below. (d)  None of the above
(a)  1 and 2 only (b)  2 and 3 only Solution: (d)
(c)  1 and 3 only (d)  All of the above The failure of the Simon commission and
Solution: (d) the Round table conference which led to the
Government of India Act, 1935, to satisfy the
Indian aspirations accentuated the demand
(2) MAKING OF THE CONSTITUTION for a constitution of India made by people of
India without outside interference. This was
Demand for Constituent Assembly officially asserted by the INC in 1935.
Q.55 The Indian National Congress (INC) made This was reiterated at the working
the demand for a Constituent Assembly only committee meeting of INC in Ludhiana,
during 1939. Before this period, there were demands
(a)  The Second World War either only to review the constitution or
(b)  Quit India movement reform the administrative structures – not to
(c)  Non-cooperation movement have a constituent assembly.
(d)  None of the above
Solution: (d)

 P.16 For Civil Services Preliminary Examination

01-Indian Polity_Q1-412.indd 16 8/7/2018 7:39:02 PM


Composition of Constituent Assembly principal communities—Muslims, Sikhs
and general (all except Muslims and
Q.58 Consider the following statements about the Sikhs), in proportion to their population.
Constituent assembly of India. 4. The representatives of each community
1. Its constitution was finally subject to the were to be elected by members of that
approval of the British Parliament. community in the provincial legislative
2. The selection of representatives of various assembly and voting was to be by the
Indian states was flexible and decided by method of proportional representation by
consultation. means of single transferable vote.
3. Members of the Constituent assembly 5. The representatives of princely states
from Provinces were elected directly from were to be nominated by the heads of
the people by a very limited franchise. the princely states. It is thus clear that
Which of the above is/are true? the Constituent Assembly was to be
(a)  1 and 2 only (b)  2 and 3 only a partly elected and partly nominated
(c)  1 and 3 only (d)  2 only body. Moreover, the members were to be
Solution: (d) indirectly elected by the members of the
In 1942, Sir Stafford Cripps, a member of the provincial assemblies, who themselves
cabinet, came to India with a draft proposal of were elected on a limited franchise.
the British Government on the framing of an It is thus clear that the Constituent Assembly
independent Constitution to be adopted after was to be a partly elected and partly
the World War II. The Cripps Proposals were nominated body.
rejected by the Muslim League which wanted The elections to the Constituent Assembly
India to be divided into two autonomous (for 296 seats allotted to the British Indian
states with two separate Constituent Provinces) were held in July–August 1946.
Assemblies. Finally, a Cabinet Mission was The Indian National Congress won 208 seats,
sent to India. While it rejected the idea of two the Muslim League in 73 seats and the small
Constituent Assemblies, it put forth a scheme groups and independents got the remaining
for the Constituent Assembly which more or 15 seats.
less satisfied the Muslim League. However, the 93 seats allotted to the
The Constituent Assembly was constituted princely states were not filled as they decided
in November 1946 under the scheme to stay away from the Constituent Assembly.
formulated by the Cabinet Mission Plan. Although the Constituent Assembly was
The features of the scheme were: not directly elected by the people of India
1. The total strength of the Constituent on the basis of adult franchise, the Assembly
Assembly was to be 389. Of these, 296 comprised representatives of all sections of
seats were to be allotted to British India Indian Society—Hindus, Muslims, Sikhs,
and 93 seats to the Princely States. Out Parsis, Anglo–Indians, Indian Christians,
of 296 seats allotted to the British India, SCs, STs including women of all these
292 members were to be drawn from the sections. The Assembly included all
eleven governors’ provinces and four from important personalities of India at that time,
the four chief commissioners’ provinces, with the exception of Mahatma Gandhi and
one from each M.A. Jinnah.
2. Each province and princely state (or
Q.59 The Constituent Assembly was constituted
group of states in case of small states)
in 1946 under the scheme formulated by the
was to be allotted seats in proportion to
Cabinet Mission Plan. As per the Plan
their respective population. Roughly, one
1. Each province and princely state was to
seat was to be allotted for every million
be allotted seats in proportion to their
population.
respective population.
3. Seats allocated to each British province
2. Princely states could veto the resolutions
were to be decided among the three
passed in the Constituent assembly.

Indian Polity Question Bank P.17

01-Indian Polity_Q1-412.indd 17 8/7/2018 7:39:02 PM


3. The representatives of princely states Justification: Statement 1: It was constituted
were to be nominated by the heads of the as per the Cabinet Mission plan.
princely states. Statement 2: It only considered the views
4. Community based representation was to of public. None were allowed to vote in its
be abolished in the constituent assembly. proceedings.
Select the correct answer using the codes Q.62 Consider the following statements about the
below. constituent assembly (CA).
(a)  1 and 3 only (b)  1, 2 and 4 only 1. Each Province and each Princely
(c)  3 only (d)  1, 3 and 4 only State or group of States were allotted
Solution: (a) seats proportional to their respective
Q.60 Representatives from most of the princely population.
states joined the Constituent assembly (CA) 2. Seats in each province (that elected the
(a) immediately after the formation of the CA) were divided proportionally between
CA major communities like Muslims, Sikhs
(b) after Cabinet Mission Plan was accepted etc.
by Indian National Congress Which of these is/are true?
(c) after the acceptance of Mountbatten Plan (a)  1 only (b)  2 only
of 1947 (c)  Both 1 and 2 (d)  None
(d) after the constitution was adopted by the Solution: (c)
CA Q.63 The Constituent Assembly was constituted in
Solution: (c) November 1946 under the scheme formulated
93 seats allotted to the princely states were not by the
filled as they decided to stay away from the (a)  Cabinet Mission Plan
Constituent Assembly. The representatives of (b)  August Offer
the princely states, who had stayed away from (c)  Cripps Proposals
the Constituent Assembly, gradually joined (d)  Wellington Commission Report
it. On April 28, 1947, representatives of the Solution: (a)
six states were part of the Assembly. After the
Q.64 The Constituent Assembly was constituted in
acceptance of the Mountbatten Plan of June
November 1946 under the scheme formulated
3, 1947 for a partition of the country, the
by the Cabinet Mission Plan. What was/were
representatives of most of the other princely
the salient features of the assembly?
states took their seats in the Assembly. The
1. The assembly had representation from
members of the Muslim League from the
both British India and princely states.
Indian Dominion also entered the Assembly.
2. The system of proportional representation
Q.61 Consider the following statements about the was adopted.
Constituent Assembly? 3. There was no communal representation in
1. It was constituted as per the provisions of the assembly.
the Indian Independence Act 1945. 4. The Assembly included important
2. It allowed members to the public to attend ministers of the British Cabinet as ex-
and vote in its proceedings several times. officio members.
3. Some of its members were directly elected Select the correct answer using the codes
on the basis of universal adult franchise. below.
4. Certain number of seats was reserved for (a)  1 and 2 only (b)  1 and 3 only
women and minorities. (c)  2 and 4 only (d)  1, 2 and 4 only
Select the correct answer using the codes Solution: (a)
below. Statement 4: There was no such
(a)  1, 2 and 3 only (b)  4 only representation.
(c)  3 and 4 only (d)  None of the above
Q.65 Consider the following statements about the
Solution: (d)
constituent assembly (CA).

 P.18 For Civil Services Preliminary Examination

01-Indian Polity_Q1-412.indd 18 8/7/2018 7:39:02 PM


1. It was based on a scheme suggested by the Q.69 Consider the following statements about the
Cabinet mission. Constituent Assembly formed in 1946:
2. All provinces and princely state were to be 1. The members of the Constituent Assembly
represented in the CA as per the decided were elected in 1946 directly on the basis
scheme. of universal franchise.
3. There were to be no nominated members 2. The Constituent Assembly that came into
in the CA as per the decided scheme. being was dominated by the Congress
Choose the correct answer using the codes party.
below. 3. The Congress party members of the
(a)  1 and 2 only (b)  2 and 3 only Constituent Assembly were unanimous in
(c)  1 and 3 only (d)  All of the above their opinion on all the critical issues.
Solution: (a) Which of the above is/are incorrect?
Q.66 Which of the following statements is/are (a)  1 only (b)  1 and 2 only
true about the composition of the constituent (c)  1 and 3 only (d)  1, 2 and 3
assembly of India? Solution. (c)
1. Each province and princely state was The Constituent Assembly that came into
generally allotted seats in proportion to being was dominated by one party: the
their respective population. Congress. The Congress however was not a
2. The representatives of each principal party with one voice. Its members differed
community were to be elected indirectly in their opinion on critical issues. Some
through the provincial legislatures. members were inspired by socialism while
3. It was a partly elected and party nominated others were defenders of landlordism. Some
body. were close to communal parties while others
Choose the correct answer using the codes were assertively secular.
below: Q.70 Which of the following statements regarding
(a)  1 and 2 (b)  2 and 3 the’ Constituent Assembly are true?
(c)  1 and 3 (d)  All of the above 1 It was not based on Adult Franchise.
Solution: (d) 2. It resulted from direct elections.
Q.67 Consider the following statements 3. It was a multi-party body.
1. The members of the Constituent Assembly 4. It worked through several Committees.
were chosen on the basis of the provincial (a)  1 and 2 (b)  2 and 3
elections of 1946. (c)  3 and 4 (d)  1, 2, 3 and 4
2. The Assembly did not have any Solution: (c)
representatives from the Princely States
Which of the above statements is/are Working of the Constituent Assembly
incorrect?
Q.71 Consider the following statements:
(a)  1 Only (b)  2 Only
1. The Indian Constitution does not make
(c)  Both (d)  None
ethnic identity a criterion for citizenship.
Solution: (b)
2. The members of the Indian Constituent
Q.68 Consider the about the proceedings and Assembly that was formed pre-
powers of the constituent assembly? Independence were indirectly elected.
1. The Muslim league withdrew from the 3. The Constituent Assembly was composed
assembly roughly along the lines suggested by the
2. The assembly was also a legislative body Cripps Mission plan.
in addition to being a constituent assembly. 4. The introduction of universal suffrage was
Which of the following statements is/are the only provision that was passed without
true? virtually any debate in the Constituent
(a)  Only 1 (b)  Only 2 Assembly.
(c)  Both (d)  None
Solution: (c)

Indian Polity Question Bank P.19

01-Indian Polity_Q1-412.indd 19 8/7/2018 7:39:02 PM


Which of the statements given above is/are and ideas. Continuation of separate electorates
correct? for Muslims was one such issue. Consider the
(a)  1 and 2 Only (b)  2, 3 and 4 Only following statements related to it:
(c)  1, 2 and 4 Only (d)  1, 2, 3 and 4 1. all the Muslim members of the Constituent
Solution: (c) Assembly supported the demand for
Most nations are an amalgamation of a separate electorates which was rejected
complex set of historical traditions; they by others.
weave together the diverse groups that reside 2. It was Begum Qudsia Aizaz Rasul,
within the nation in different ways. For the lone Muslim woman member of
example, German identity was constituted the Constituent Assembly who made a
by being ethnically German. The constitution powerful plea for continuing separate
gave expression to this identity. The Indian electorates.
Constitution, on the other hand, does not make Which of the above is/are correct?
ethnic identity a criterion for citizenship. (a)  1 Only  (b)  2 Only  (c)  Both  (d)  None
Formally, the Constitution was made by Solution: (d)
the Constituent Assembly which had been On 27 August 1947, B. Pocker Bahadur
elected for undivided India. It held its first from Madras made a powerful plea for
sitting on 9 December1946 and re-assembled continuing separate electorates. Not all
as Constituent Assembly for divided India on Muslims supported the demand for separate
14 August 1947. Its members were elected electorates. Begum Aizaz Rasul, for instance,
by indirect election by the members of the felt that separate electorates were self
Provisional Legislative Assemblies that had destructive since they isolated the minorities
been established in 1935. from the majority.
The Constituent Assembly was composed Which of the following about the
roughly along the lines suggested by the plan Constituent Assembly is incorrect?
proposed by the committee of the British (a) Rajendra Prasad was the President of
cabinet, known as the Cabinet Mission. the Constituent Assembly and Dr. B.R.
There were legitimate differences Ambedkar served as the Chairman of the
of principle. And the differences were Drafting Committee of the Constitution.
many: should India adopt a centralised or (b) The Constituent Assembly never debated
decentralised system of government? What the important issues of cultural rights
should be the relations between the States and social justice on the floor of the
and the centre? What should be the powers Assembly.
of the judiciary? Should the Constitution (c) The discussions within the Constituent
protect property rights? Almost every issue Assembly were also influenced by the
that lies at the foundation of a modern state opinions expressed by the public.
was discussed with great sophistication. (d) The Constituent Assembly that came into
Only one provision of the Constitution was being was dominated by congress with
passed without virtually any debate: the more than 80 per cent of the members
introduction of universal suffrage (meaning of the Constituent Assembly being
that all citizens reaching a certain age, Congress members.
would be entitled to be voters irrespective of Solution: (b)
religion, caste, education, gender or income). Q.73 The Constituent Assembly arrived at
So, while the members felt no need at all to decisions on the various provisions of the
discuss the issue of who should have the right Constitution:
to vote, every other matter was seriously (a)  by a majority vote
discussed and debated. (b)  by a two-thirds majority
Q.72 The Constituent Assembly deliberated on (c)  by consensus
a number of important issues with many (d)  unanimously
differences among the members in their views Solution: (c)

 P.20 For Civil Services Preliminary Examination

01-Indian Polity_Q1-412.indd 20 8/7/2018 7:39:02 PM


Objective Resolution On the basis of the Objectives Resolution,
India’s Constitution gave institutional
Q.74 The “Objectives Resolution” introduced in expression to the fundamental commitments:
the Constituent Assembly on 13 December equality, liberty, democracy, sovereignty and
1946 was a momentous resolution that a cosmopolitan identity.
outlined the defining ideals of the Constitution
Q.76 The historic ‘Objectives Resolution’ was
of Independent India, and provided the
passed in
framework within which the work of
(a) Lahore Session of the Indian National
constitution-making was to proceed. consider
Congress
the following statements with respect to the
(b) Constituent Assembly in 1946
“Objectives Resolution”:
(c) All Parties Conference called after the
1. It was introduced by Sardar Patel in the
Cabinet Mission Plan
Constituent Assembly.
(d) A Meeting which declared the
2. It proclaimed India to be an “Independent
Mountbatten Plan.
Sovereign, Socialist and Secular
Solution: (b)
Republic”.
Learning: In December, 1946, Jawaharlal
Which of the above is/are correct?
Nehru moved the historic ‘Objectives
(a)  1 Only (b)  2 Only
Resolution’ in the Assembly. It laid down
(c)  Both (d)  None
the fundamentals and philosophy of the
Solution: (d)
constitutional structure. It read:
Q.75 “Objectives Resolution” was introduced by It included the major values and ideals
Pandit Nehru in the Constituent Assembly, of sovereignty, republic, fundamental rights,
It proclaimed India to be an “Independent directive principles, non-interference etc.
Sovereign Republic”. Did not include the It sought to secure to ideals mentioned in
words Socialist and Secular. the Preamble.
Consider the following about the famous It provided for adequate safeguards for
“Objectives Resolution” proposed by minorities, backward and tribal areas, and
Jawaharlal Nehru. depressed and other backward classes.
1. It was proposed to the Constituent This Resolution was unanimously adopted
Assembly of India before the enactment by the Assembly on January 22, 1947.
of the Constitution. It influenced the eventual shaping of the
2. It contained clear provisions to abolish constitution through all its subsequent stages.
Privy Purse from India. Its modified version forms the Preamble of
3. It became the basis for adding the the present Constitution.
words “Socialist” and “Secular” to the
Q.77 The Objectives resolution which formed the
Constitution in 1950.
bedrock of the Indian constitution guaranteed
Select the correct answer using the codes
which of the following:
below.
1. Social justice
(a)  1 only (b)  2 and 3 only
2. Economic justice
(c)  3 only (d)  None of the above
3. Safeguards for minorities including the
Solution: (a)
tribals
Justification: Statement 1: It was proposed
4. Separate status for Jammu and Kashmir
in 1946, and later passed by the Constituent
Choose the correct answer using the codes
assembly shaping the Preamble of the
below
constitution.
(a)  1 and 2 (b)  2, 3 and 4
Statement 2: There was no such provision.
(c)  1 and 3 (d)  All of the above
Statement 3:  It was added later by the
Solution: (d)
42nd amendment to the Indian constitution.
This resolution enshrined the aspirations
and values behind the Constitution making.

Indian Polity Question Bank P.21

01-Indian Polity_Q1-412.indd 21 8/7/2018 7:39:02 PM


Committees of Constituent Assembly Q.80 It was this committee that was entrusted
with the task of preparing a draft of the new
Q.78 Which of the following was/were committees Constitution. It consisted of seven members.
associated with the Constituent Assembly of They were:
India? • Dr. B. R. Ambedkar (Chairman)
1. Welfare State Ideals committee • N. Gopalaswamy Ayyangar
2. A States Committee for Negotiating with • Alladi Krishnaswamy Ayyar
the States • Dr. K. M. Munshi
3. A provincial Constitution Committee • Syed Mohammad Saadullah
Select the correct answer using the codes • N. Madhava Rau (He replaced B.  L.
below. Mitter who resigned due to ill-health)
(a)  1 and 2 only (b)  2 only • T. T. Krishnamachari (He replaced D.  P.
(c)  1 and 3 only (d)  2 and 3 only Khaitan who died in 1948)
Solution: (d) The Drafting Committee, after taking into
Learning: The Constituent Assembly consideration the proposals of the various
appointed a number of committees to deal committees, prepared the first draft of the
with different tasks of constitution-making. Constitution of India, which was published
Out of these, eight were major committees in February 1948. The people of India were
and the others were minor committees. given eight months to discuss the draft and
The names of these committees and their propose amendments. In the light of the
chairmen are given below: public comments, criticisms and suggestions,
Major Committees as constructed: the Drafting Committee prepared a second
1. Union Powers Committee – Jawaharlal draft, which was published in October 1948.
Nehru
Q.81 Who among the following was the
2. Union Constitution Committee –
Constitutional adviser to the Constituent
Jawaharlal Nehru
Assembly of India?
3. States Committee (Committee for
(a)  Dr. B.N. Rao (b)  Dr. B.R. Ambedkar
Negotiating with States) – Jawaharlal Nehru
(c)  K.M. Munshi (d)  M.C. Setalvad
4. Provincial Constitution Committee  –
Solution: (a)
Sardar Patel
5. Advisory Committee on Fundamental Q.82 Who presided over the inaugural meeting of
Rights, Minorities and Tribal and the Constituent Assembly of India?
Excluded Areas – Sardar Patel. (a)  Sachidananda Sinha
6. Drafting Committee – Dr. B.R. Ambedkar (b)  B. R. Ambedkar
7. Rules of Procedure Committee – Dr. (c)  Dr. Rajendra Prasad
Rajendra Prasad (d)  P. Upendra
8. Steering Committee – Dr. Rajendra Prasad Solution: (a)
The Constituent Assembly held its first
Q.79 Among all the committees of the Constituent
meeting on 9 December 1946. The Muslim
Assembly, the most important committee
League boycotted the meeting and insisted
was the Drafting Committee. Who among
on a separate state of Pakistan. The meeting
the following were NOT the members of the
was thus attended by only 211 members.
committee?
Dr. Sachchidanand Sinha, the oldest member,
1. N Gopalaswamy Ayyangar
was elected as the temporary President of the
2. Syed Mohammad Saadullah
Assembly, following the French practice.
3. T. T. Krishnamachari
Later, Dr. Rajendra Prasad was elected as
4. Dr. K. M. Munshi
the President of the Assembly. Similarly, both
Choose the correct answer using the codes
H.C. Mukherjee and V.T. Krishnamachari
below.
were elected as the Vice-Presidents of the
(a)  4 only (b)  2 and 3 only
Assembly. In other words, the Assembly had
(c)  1 only (d)  All were the members
two Vice-Presidents.
Solution: (d)

 P.22 For Civil Services Preliminary Examination

01-Indian Polity_Q1-412.indd 22 8/7/2018 7:39:02 PM


Changes by the Independence Act 4. Was restricted in its powers by an order of
the British government
Q.83 Consider the following statements as per the 5. Supervised the functioning of the Supreme
Independence Act of 1947 Court and other subordinate courts
1. the constituent assembly was to become Choose the correct answer using the codes
the dominion legislature. below.
2. The constituent assembly was not a fully (a)  1, 2 and 4 only (b)  2, 3 and 5 only
sovereign body. (c)  3 only (d)  All of the above
Which of the above is/are true? Solution: (c)
(a)  1 only (b)  2 only The Assembly was made a fully sovereign
(c)  Both 1 and 2 (d)  None body, which could frame any Constitution it
Solution: (a) pleased. The independence act empowered the
After the acceptance of the Mountbatten Plan Assembly to abrogate or alter any law made
of 3 June 1947 for a partition of the country, the by the British Parliament in relation to India.
representatives of most of the other princely The Assembly also became a legislative body
states took their seats in the Assembly. The i.e. it enacted ordinary laws for the country
members of the Muslim League from the The Prime Minister was appointed by
Indian Dominion also entered the Assembly. the party in majority, not by the Constituent
The Indian Independence Act of 1947 made Assembly.
the following three changes in the position of
the Assembly: Other Functions Performed
1. The Assembly was made a fully sovereign
body, which could frame any Constitution Q.85 Apart from framing the Constitution and
it pleased. The act empowered the enacting ordinary laws, the Constituent
Assembly to abrogate or alter any law Assembly also
made by the British Parliament in relation 1. Ratified India’s membership of the
to India. Commonwealth
2. The Assembly also became a legislative 2. Adopted the national flag and anthem
body. In other words, two separate 3. Elected the first President of India
functions were assigned to the Assembly Select the correct answer using the codes
that is, making of a constitution for free below.
India and enacting of ordinary laws for (a) 1 and 2 only (b)  2 and 3 only
the country. These two tasks were to be (c) 1 and 3 only (d)  1, 2 and 3
performed on separate days. Solution: (d)
Thus, the Assembly became the first Learning: The Constituent Assembly also
Parliament of free India (Dominion performed the following functions:
Legislature). Whenever the Assembly met 1. It ratified the India’s membership of the
as the Constituent body it was chaired by Commonwealth in May 1949.
Dr. Rajendra Prasad and when it met as the 2. It adopted the national flag on July  22,
legislative body, it was chaired by G. V. 1947.
Mavlankar. These two functions continued 3. It adopted the national anthem on January
till 26 November 1949, when the task of 24, 1950.
making the Constitution was over. 4. It adopted the national song on January
Q.84 The constituent assembly of India 24, 1950.
1. Was elected based on universal adult 5. It elected Dr. Rajendra Prasad as the first
franchise President of India on January 24, 1950.
2. Elected and appointed the Prime Minister Q.86 Match the following:
and other ministers in the first government A.  22 July 1947 1. Adoption of
3. Enacted laws until the first official National Song by
Parliament was elected the Constituent
Assembly

Indian Polity Question Bank P.23

01-Indian Polity_Q1-412.indd 23 8/7/2018 7:39:02 PM


B.  24 January 1950 2. Adoption of The Constitution drew enormous legitimacy
National Emblem from the fact that it was drawn up by people
by the Government who enjoyed immense public credibility, who
C.  26 January 1950 3. Adoption of had the capacity to negotiate and command
National Calendar the respect of a wide cross-section of society,
by the Government and who were able to convince the people that
D.  22 March 1957 4. Adoption of the constitution was not an instrument for the
National Flag by aggrandisement of their personal power. The
the Constituent final document reflected the broad national
Assembly consensus at the time.
ABCD Some countries have subjected their
(a)  4 1 2 3 (b)  3 1 2 4 constitution to a full-fledged referendum,
(c) 4 2 1 3 (d)  2 1 3 4 where all the people vote on the desirability
Solution: (a) of a constitution. The Indian Constitution
was never subject to such a referendum,
Miscellaneous but nevertheless carried enormous public
authority, because it had the consensus and
Q.87 Which of the following erstwhile provinces backing of leaders who were themselves
had the largest membership in the Constituent popular. Although the Constitution itself was
Assembly of India? not subjected to a referendum, the people
(a)  United Provinces (b)  Madras adopted it as their own by abiding by its
(c)  West Bengal (d)  Bombay provisions. Therefore, the authority of people
Solution: (a) who enact the constitution helps determine in
Learning: Both the Indian provinces and part its prospects for success.
Princely states were the members of constituent
Q.89 Consider the following statements:
assembly of India. Former taken together were
1. Leaders of the freedom movement had an
greater in number than the princely states.
open mind whether the country should be
1. United Provinces – 55 members
democratic or not after independence.
2. Madras – 49
2. Members of the Constituent Assembly of
3. West Bengal – 19
India held the same views on all provisions
4. Bombay – 21.
of the Constitution.
Out of all princely states, Travancore had
3. A country that has a constitution must be a
the highest membership of 6. Other princely
democracy.
states had 1-4 members in the constituent
4. Constitution cannot be amended because it
assembly of India.
is the supreme law of a country.
Q.88 Which of the following make the Indian Which of the statements given above is/are
constitution acceptable, legitimate and correct?
authoritative? (a)  1 and 2  (b)  1 and 3  (c)  1 and 4  (d)  None
1. Citizens of India participated directly or Solution: (d)
indirectly by way of notes, debates, and The leaders of the freedom movement were
suggestions in the constituent assembly. clear in their mind that their country should
2. The Constituent assembly was not entirely be a democracy. The different members had
nominated by the British. different views on different points, but
3. The Constitution was adopted after a ultimately they resolved their differences.
referendum. All countries that have a constitution are not
Choose the correct answer using the codes necessarily democratic but all countries which
below: are democratic must have their constitutions.
(a)  1 and 2 (b)  2 and 3 The Constitution is amended by two-third
(c)  1 and 3 (d)  All of the above majority of the members of the Parliament if
Solution: (a) any change was required.

 P.24 For Civil Services Preliminary Examination

01-Indian Polity_Q1-412.indd 24 8/7/2018 7:39:02 PM


Q.90 The Indian Constitution was never subject to Choose the correct answer using the codes
such a referendum, but nevertheless carried below:
enormous public authority. This is because (a)  1 and 2 (b)  2 and 3
1. It had the consensus and backing of (c)  1 and 3 (d)  All of the above
leaders who were themselves popular Solution: (c)
2. It was drawn up by people who enjoyed It was debated in the CA whether India
immense public credibility should be a Parliamentary or a Presidential
3. It was drawn by a  completely elected democracy. Only that we are to be a
body of representatives democracy giving fundamental rights to
Which of the above statements is/are correct? people was agreed upon earlier.
(a)  1 and 3 Only (b)  2 and 3 Only Q.92 Who was the first Foreign Minister of free
(c)  1 and 2 Only (d)  All India? [NDA 2008]
Solution: (c) (a)  Jawaharlal Nehru
Mode of promulgation (b)  Gulzari Lal Nanda
This refers to how a constitution comes (c)  Lal Bahadur Shastri
into being. Who crafted the constitution (d)  John Mathai
and how much authority did they have? In Solution: (a)
many countries constitutions remain defunct
because they are crafted by military leaders
or leaders who are not popular and do not (3) SALIENT FEATURES OF THE
have the ability to carry the people with CONSTITUTION
them. The most successful constitutions, like
India, South Africa and the United States, Q.93 What is/are the major difference/differences
are constitutions which were created in the between a written and an unwritten
aftermath of popular national movements. Constitution?
Although India’s Constitution was 1. A written Constitution is the formal source
formally created by a Constituent Assembly of all Constitutional Laws in the country
between December 1946 and November and the unwritten constitution is not the
1949, it drew upon a long history of the formal source.
nationalist movement that had a remarkable 2. A written Constitution is entirely codified
ability to take along different sections of whereas an unwritten Constitution is not.
Indian society together. The Constitution Select the correct answer using the codes
drew enormous legitimacy from the fact given below.
that it was drawn up by people who enjoyed (a)  Only 1 (b)  Only 2
immense public credibility, who had the (c)  Both 1 and 2 (d)  Neither 1 nor 2
capacity to negotiate and command the Solution: (b)
respect of a wide cross-section of society, and Unwritten constitution is a constitution not
who were able to convince the people that embodied in a single document but based
the constitution was not an instrument for the chiefly on custom and precedent as expressed
aggrandisement of their personal power. The in statutes and judicial decision that forms
final document reflected the broad national base for all formal decision making in the
consensus at the time. country. ex: Britain
Q.91 Which of the following principles enshrined/ Q.94 “Constitution is not an end in itself, but
implied in the constitution were based on a means to pursue the end”, hence consider
broad consensus among the national leaders these statements and pick out the correct
even before the constituent assembly started ones.
functioning? 1. One of the needs of a constitution is to
1. India as a welfare state. restrict the exercise of power.
2. India as a Parliamentary democracy. 2. Constitutions are documents capable of
3. India as a secular state. social transformation.

Indian Polity Question Bank P.25

01-Indian Polity_Q1-412.indd 25 8/7/2018 7:39:02 PM


3. Constitutions cannot empower those who 1. It specifies the basic allocation of power
are traditionally kept out of power. in a society. It decides who gets to decide
Code what the laws will be.
(a)  1 and 2 only (b)  1 and 3 only 2. It decides how the government will be
(c)  2 and 3 only (d)  All of these constituted.
Solution: (a) 3. It sets some limits on what a government
Self-explanatory can impose on its citizens.
4. Some constitutions enable and empower
Importance of Constitution the government to take positive measures
to overcome forms of inequality or
Q.95 What purposes does the constitution of a deprivation.
nation necessarily serve? Which of the statements given above is/are
1. It may allow better coordination in the correct?
nation amongst individuals. (a)  1 and 2 Only (b)  2 and 3 Only
2. It helps define the formal decision-making (c)  3 and 4 Only (d)  All
process in a society. Solution: (d)
3. It allows people to ban autocracy and
Q.97 A constitution serves which of following
adopt democracy.
purposes?
Choose the correct answer using the codes
1. It is the supreme law which determines
below:
the relationship among citizens living in a
(a)  1 and 2 (b)  2 and 3
territory.
(c)  1 and 3 (d)  All of the above
2. It limits the powers of government.
Solution: (a)
Which of the above is/are correct?
The function of a constitution is to
(a) 1 only (b)  2 only
1. provide a set of basic rules that allow for
(c) Both 1 and 2 (d)  None
minimal coordination amongst members
Solution: (c)
of a society.
2. specify who has the power to make Q.98 The constitution of a nation
decisions in a society. It decides how the 1. is the supreme law of the land
government will be constituted. 2. determines the rights of citizens
3. set some limits on what a government can 3. determines the powers of the government
impose on its citizens. These limits are 4. lays down the way of functioning of the
fundamental in the sense that government government in a broad manner
may never trespass them. Which of the following would be correct?
4. enable the government to fulfil the (a)  1, 2 and 3 only (b)  1 and 4 only
aspirations of a society and create (c)  2 and 3 only (d)  All of the above
conditions for a just society. Solution: (d)
5. ensure that a dominant group does not In a democracy the rulers are not free to do
use its power against other less powerful what they like. There are certain basic rules
people or groups. Every society is prone that the citizens and the government have
to this tyranny of the majority. The to follow. All such rules together are called
Constitution usually contains rules that constitution. As the supreme law of the
ensure that minorities are not excluded country, the constitution determines the rights
from anything that is routinely available of citizens, the powers of the government and
to the majority how the government should function.
Q.96 A constitution does not always specify for a Q.99 Which of the following purposes is served by
democracy. Syrian constitution is an example. a democratic Constitution?
Regarding a constitution of a country, (a) Defining the way power will be exercised
consider the following statements in the country’s political system
(b) Guards against the misuse of power by
the authorities

 P.26 For Civil Services Preliminary Examination

01-Indian Polity_Q1-412.indd 26 8/7/2018 7:39:02 PM


(c) Protects the minority against the tyranny 2. It limits the powers of the government
of the majority 3. It determines how a government should
(d) All of the above govern the country
Solution: (d) 4 It determines the powers of the
Justification and Learning: An important government
purpose of a Constitution is to define the Select the correct answer using the codes
nature of a country’s political system. For given below
example, Nepal’s earlier Constitution stated (a)  1 and 4 (b)  1, 3, and 4
that the country was to be ruled by the King (c)  1, 2 and 4 (d)  1, 2, 3 and 4
and his council of ministers. So, A is correct. Solution: (d)
• In democratic societies, the Constitution The constitution is the supreme law of the
often lays down rules that guard against land, it not only determines the powers of
this misuse of power by our political the government but also lays down certain
leaders. In the case of the Indian limitations on the powers of the government
Constitution many of these laws are by providing Fundamental rights to the
contained in the section on Fundamental citizens of India. It determines how a
Rights. So, B is correct. government should function and generates
Q.100 Consider the following statements: a degree of trust and coordination that is
1. Having a constitution guarantees that the necessary for different kind of people to live
concerned country will be democratic. together.
2. Not having a written constitution Q.103 Consider the following statements about
guarantees that the concerned country will Indian Constitution.
be autocratic. 1. Indian Constitution envisaged
Which of these is/are true? decentralized idea of national unity.
(a)  1 only (b)  2 only 2. Indian constitution comprehensively
(c)  Both (d)  None addressed all issues of gender justice.
Solution: (d) Which of the following statements are
Britain does have a written constitution. It is correct?
still a democratic monarchy. (a)  Only 1 (b)  Only 2
Syria has a written constitution. It is an (c)  Both 1 and 2 (d)  None of these
autocracy. Solution: (d)
A constitution is the mere embodiment of Indian constitution has envisaged the
the political philosophy of a nation. centralized idea of national unity. The unitary
Q.101 Consider the following statements: bias visible in indian constitution is intended
1. All democracies have written to preserve and promote national unity.
constitutions. Q.104 The Indian Constitution is regarded as:
2. All countries that have written (a) federal
constitutions are democracies. (b)  unitary
Which of these is/are true? (c)  parliamentary
(a)  1 only (b)  2 only (d) federal in form and unitary in spirit
(c)  Both 1 and 2 (d)  None Solution: (d)
Solution: (d) Constitution of India provides for the
distribution of powers between state and
Features of Indian Constitution centre. Hence it’s federal in nature. But
Article 1 describes India as union of states
Q.102 Which of the following statements is/are and there are provisions like emergency,
correct with respect to the Constitution of residuary powers in centre etc. which are
India? unitary in nature. It was to protect the security
1. It generates a degree of trust among and unity of india.
people of India

Indian Polity Question Bank P.27

01-Indian Polity_Q1-412.indd 27 8/7/2018 7:39:02 PM


Q.105 The Indian constitution is a written one unlike Originally (1949), the Constitution
in some of the other democracies. What does contained a Preamble, 395 Articles (divided
it imply? into 22 Parts) and 8 Schedules. Presently
1. The form of government in India has (2013), it consists of a Preamble, about
been codified in the constitution to reduce 465 Articles (divided into 25 Parts) and
political and administrative conflicts. 12 Schedules. The various amendments
2. All the laws made by Parliament are to be carried out since 1951 have deleted about 20
written down as a part of the constitution. Articles and one Part (VII) and added about
3. Only because of a written constitution, 85 Articles, four Parts (IVA, IXA, IXB and
citizens are able to enjoy fundamental XIVA) and four Schedules (9, 10, 11 and 12).
rights. No other Constitution in the world has so
Choose the correct answer using the codes many Articles and Schedules.
below. Centre-state relations have been given
(a)  1 and 2 only (b)  2 and 3 only in so much detail containing even the least
(c)  1 and 3 only (d)  1 only of provisions of taxation, administration
Solution: (d) and finances. For e.g. which taxes will be
Constitution specifies the structure, appropriated by either centre or state has
organisation, powers and functions of both the been given in the constitution. Similarly, fifth
Central and state governments and prescribes and sixth schedule areas have not only been
the limits within which they must operate. defined and demarcated by the constitution,
Thus, it avoids the misunderstandings and but also several administrative provisions like
disagreements between the two. All the laws tribal advisory council; power of Gram sabhas
made in India are codified separate from the in those areas etc. have also been provided.
constitution and maintained in a law book. Four factors have contributed to the
They need not be a part of the constitution. elephantine size of our Constitution. They
Even in the UK where there is no written are:
constitution, people enjoy several fundamental (a) Geographical factors, that is, the vastness
rights. However, only because our fundamental of the country and its diversity.
rights are written in the constitution, it is (b) Historical factors, e.g., the influence of
difficult to amend and change them as per the the Government of India Act of 1935,
wishes of the political executive. which was bulky.
Q.106 The Constitution contains not only the (c) Single Constitution for both the Centre
fundamental principles of governance but and the states except Jammu and
also detailed administrative provisions. Kashmir.
This fact can be supported by looking up (d) Dominance of legal luminaries in the
in which of the following provisions of the Constituent Assembly.
constitution? Q.107 Which of the following features of the
1. Fifth and Sixth schedule Indian Constitution deal with maintaining
2. Centre-state relations the social fabric, moral values and national
3. Appointment and functions of several consciousness?
constitutional bodies 1. Fundamental Rights
Choose the correct answer using the codes 2. Fundamental Duties
below. 3. Directive Principles of State Policy
(a)  1 and 2 only (b)  2 and 3 only Choose the correct answer using the codes
(c)  1 and 3 only (d)  All of the above below.
Solution: (d) (a)  1 and 2 only (b)  2 and 3 only
The Constitution of India is the lengthiest of (c)  1 and 3 only (d)  All of the above
all the written constitutions of the world. It is Solution: (d)
a very comprehensive, elaborate and detailed A constitution is not a frozen and unalterable
document. document. It is a document made by human

 P.28 For Civil Services Preliminary Examination

01-Indian Polity_Q1-412.indd 28 8/7/2018 7:39:02 PM


beings and may need revisions, changes and Q.110 In the context of India, which of these is a
re-examination. provision that a democratic constitution does
It is true that the constitution reflects the not have?
dreams and aspirations of the concerned 1. Powers of the legislature
society. It must also be kept in mind that 2. Name of the country
the constitution is a framework for the 3. Powers of the head of the state
democratic governance of the society. In this 4. Name of the head of the state
sense, it is an instrument that societies create Answer the question using the codes given
for themselves. below
For e.g. Fundamental rights clearly (a)  2 Only (b)  4 Only
establish a secular, equitable, just and non- (c)  2, 3 and 4 (d)  2 and 4
exploitative social fabric in India. Solution: (b)
Fundamental duties provide a secular, The Constitution of India has provisions for
fraternistic and a patriotic aspect to the Powers of the legislature, Name of the country,
Indian citizens by making them respect the Powers of the head of the state but not the
constitution, national flag, asking them to Name of the head of the state
render service to the nation and mankind etc. Q.111 Indian our Constitution is a living document.
DPSP does so by providing for a more What do you mean by that? Consider the
equitable work culture in men and women; following statements
minimizing inequalities in social and 1. Like a living being, the Constitution
economic status; establishing the ideal of a responds to experience.
welfare society etc. 2. Even after so many changes in the society,
Q.108 Which of the following are enshrined in the the Constitution continues to work
Indian constitution? effectively because of this ability to be
1. Societal ideals dynamic, to be open to interpretations
2. Nature of Indian society and the ability to respond to the changing
3. Nature of country’s political system situation
Choose the correct answer using the codes 3. Almost like a living being, this document
below: keeps responding to the situations and
(a)  1 and 2 (b)  2 and 3 circumstances arising from time to time.
(c)  1 and 3 (d)  All of the above 4. Like a living being, our constitution is
Solution: (d) also prone to destruction and it’s not
Societal ideals like fraternity, harmony, permanent
justice etc. are enshrined in the Preamble and Which of the above aptly describe the
Directive Principles of State Policy (DPSP). statement in the question in the context of
Nature of society is to be secular, liberal Indian constitution?
and socialist. It is mentioned in the Preamble, (a)  1, 2 and 4 (b)  2 and 4
Directive Principles and Fundamental Rights. (c)  1, 2 and 3 (d)  All
Political system is republic democratic Solution: (c)
mentioned in the constitution in the Preamble. Almost like a living being, this document
Q.109 Which of the following are the aspects of the keeps responding to the situations and
Indian constitution? circumstances arising from time to time. Like
1. Separation of powers a living being, the Constitution responds to
2. Federalism experience. In fact that is the answer to the
3. Fundamental Rights riddle we mentioned at the beginning about
Choose the correct answer using the codes the durability of the Constitution.
below: Even after so many changes in the
(a)  1 and 2 (b)  2 and 3 society, the Constitution continues to
(c)  1 and 3 (d)  All of the above work effectively because of this ability to
Solution: (d) be dynamic, to be open to interpretations

Indian Polity Question Bank P.29

01-Indian Polity_Q1-412.indd 29 8/7/2018 7:39:03 PM


and the ability to respond to the changing subjects (originally 66) and the concurrent
situation. This is a hallmark of a democratic list contains 52 subjects (originally 47)
constitution. In a democracy, practices and • Eighth Schedule – Languages recognized
ideas keep evolving over time and the society by the Constitution.
engages in experiments according to these. • Ninth schedule – Acts & regulation of
A constitution, which protects democracy states dealing with the land reforms &
and yet allows for evolution of new practices abolition of zamindari
becomes not only durable but also the object • Tenth schedule–Anti-defection provisions
of respect from the citizens. • Eleventh schedule – powers, authority
Indian constitution, as of now seems and responsibilities of Panchayat
to be indestructible because of the strong • Twelfth schedule – powers, authority and
democracy and presence of checks and responsibilities of Municipalities
balance. Q.113 Which of the following matters is/are dealt
by the Third Schedule of the Constitution?
Schedule 1. Allocation of seats in the Rajya Sabha to
the states and the union territories
Q.112 Match these schedules of Constitution to
2. Provisions for elections to the Parliament
what they contain.
and state legislatures
1. First Schedule – List of names of All
3. Provisions relating to the privileges of
States and Union Territories
dignitaries
2. Second Schedule – Powers of President,
4. Forms of Oaths or Affirmations for
Governor and Judges
constitutional functionaries
3. Fourth Schedule – Allocation of seats in
5. Provisions relating to disqualification
Rajya Sabha
of the members of Parliament and State
4. Seventh Schedule – Division of powers
Legislatures
between Legislative, Executive and
Select the correct answer using the codes
Judiciary
below.
Choose the correct answer using the codes
(a)  1 and 5 only (b)  3 and 4 only
below.
(c)  4 only (d)  2 and 5 only
(a)  1, 2 and 4 only (b)  2 and 4 only
Solution: (c)
(c)  1 and 3 only (d)  All of the above
Solution: (c) Q.114 Which of the following schedules of the
• Third Schedule – Forms of Oaths constitution gives a clear constitutional
or Affirmations for MPs, Ministers, recognition to the existence of the system of
Constitutional functionaries etc. “political parties” in India?
• Fourth Schedule – Allocation of seats (a)  Seventh Schedule
in the Rajya Sabha to the states and the (b)  Tenth Schedule
union territories. (c)  Third Schedule
• Fifth Schedule – Provisions relating to the (d)  Ninth Schedule
administration and control of scheduled Solution: (b)
areas and scheduled tribes. Justification & Learning: The Tenth
• Sixth Schedule – Provisions relating to Schedule of the Constitution (which embodies
the administration of tribal areas in the the anti-defection law) is designed to prevent
states of Assam, Meghalaya, Tripura and the evil or mischief of political defections
Mizoram. motivated by the lure of office or material
• Seventh Schedule – Division of powers benefits or other similar considerations.
between the Union and the States in terms Since the anti-defection provisions clearly
of List I (Union List), List II (State List) mention the existence of political parties (and
and List III (Concurrent List). Presently, members being disqualified on leaving its
the Union List contains 100 subjects membership), the Tenth schedule recognizes
(originally 97), the state list contains 61 the system of political parties.

 P.30 For Civil Services Preliminary Examination

01-Indian Polity_Q1-412.indd 30 8/7/2018 7:39:03 PM


Q.115 Which of the following schedules of the of the legislative institutions proved to be
Constitution have a bearing on the governance very useful for the country in setting up it
of tribals in India? Statement 1: It’s the socialistic revolution
1. Fifth schedule 2.  Sixth Schedule of Russia that inspired our nationalist leaders
3. Seventh Schedule 4.  Tenth Schedule Statement 2: The parliamentary system in
Select the correct answer using the codes India is adopted from the Britain and US has
below: presidential system of government
(a)  1 and 2 only (b)  2 and 3 only Statement 3: The Government of India
(c)  1 and 4 only (d)  1, 2 and 3 only act, 1935 was formed on basis of Indian
Solution: (d) government structure as anticipated during
5th and 6th schedule directly concern post-independence period
themselves with tribal governance as all Q.117 Which of the following can be said to be
the special provisions made under the the sources of the core values of the Indian
constitution fall under it. For e.g. constituting constitution?
a tribal advisory council; giving tribals 1. Karachi session of Congress, 1931
control over land and forests etc. 2. Cabinet Mission Plan 1946
7th schedule is concerned with the 3. 1916 Lucknow Pact
division of legislative power between Centre Choose the correct answer using the codes
and States. It concerns with forests; welfare; below:
mining and minerals which have a direct (a)  3 only (b)  1 and 3
bearing on the tribal community in India. (c)  1 only (d)  All of the above
10th schedule is about anti-defection. Solution: (c)
Even as India fought for its independence
Sources from British colonialism a vision of what
Indian democracy ought to look like emerged.
Q.116 With regard to making of the constitution,
As far back as in 1928, Motilal Nehru and
consider the following statements:
eight other Congress leaders drafted a
1. Our leaders were inspired by socialist
constitution for India.
revolution in French
In 1931, the resolution at the Karachi
2. Our leaders were inspired by the
session of the Indian National Congress dwelt
parliamentary democracy in U.S.
on how independent India’s constitution
3. The Indian Constitution adopted many
should look like. The Karachi Resolution
institutional details and procedures from
reflects a vision of democracy that meant
Government of India Act, 1935
not just formal holding of elections but
4. Much of the consensus regarding the
a substantive reworking of the Indian
democratic setup had evolved during the
social structure in order to have a genuine
freedom struggle
democratic society. The Karachi Resolution
Which of the statement is/are correct?
clearly spells out the vision of democracy
(a)  1 and 3 (b)  3 and 4
that the nationalist movement in India had. It
(c)  1, 3 and 4 (d)  1, 2 and 4
articulates the values that were further given
Solution: (b)
full expression in the Indian Constitution.
The familiarity with political institutions
Cabinet mission plan was about the
of colonial rule also helped develop an
post-independent political and territorial
agreement over the institutional design. The
reorganization of India.
British rule had given voting rights only to a
Lucknow pact was the merger of the
few. On that basis the British had introduced
league and Congress and the acceptance of
very weak legislatures. Elections were
separate electorates (which is not at all a core
held in 1937 to Provincial Legislatures and
value of the constitution).
Ministries all over British India. These were
not fully democratic governments. But the Q.118 Consider the following features that are
experience gained by Indians in the working borrowed from constitutions of other nations.

Indian Polity Question Bank P.31

01-Indian Polity_Q1-412.indd 31 8/7/2018 7:39:03 PM


1. Quasi-federal form of government – (a)  Cabinet Mission Plan
Canada (b) Lahore Session of the Indian National
2. Charter of Fundamental Rights – UK Congress (INC)
constitution (c)  Regulating Act of 1858
3. Directive Principles of State Policy – (d)  Government of India Act of 1935
German Constitution Solution: (d)
Choose the correct matches from the codes Justification: Provincial autonomy,
given below: administrative structure, the establishment of a
(a)  1 and 2 (b)  2 and 3 Federal Court, provision for the establishment
(c)  1 and 3 (d)  1 only of a “Federation of India”, a representative
Solution: (d) government and many more features were
Q.119 The values of liberty, equality and fraternity borrowed directly from the GoI Act 1935.
that influenced the Indian National Movement Q.122 The amendment procedure laid down in the
as well as our constitution come from Constitution of India is on the pattern of?
(a)  French revolution (a)  Government of India Act, 1935
(b)  Russian Socialist Policies (b)  Government of India Act, 1947
(c)  Events leading to Balkanization (c)  Constitution of South Africa
(d)  Vietnam War (d)  Constitution of UK
Solution: (a) Solution: (c)
*Learning*: is The French Revolution Q.123 Under the Constitution of India Article
regarded as one of the most important 20 providing for Security against double
movements in history. jeopardy and ‘ex post facto laws,’ and Article
As inspired by liberal and radical ideas, the 21 providing for Right to life and liberty have
Revolution profoundly altered the course of their inspirational counter part in
modern history, triggering the global decline (a)  South African (b)  Candedian
of absolute monarchies while replacing them (c) U.S. Constitution (d) Irish
with republics and liberal democracies. Solution: (c)
Q.120 Consider the following sources of the Q.124 Which one of the following is the common
Constitution and the features borrowed from feature of Indian and Swiss federalism 2
them. (a)  Judicial Review
1. Canadian Constitution: Parliamentary (b) Equal representation of the Units in the
privileges and bicameralism Upper House
2. Australian Constitution: Concurrent List (c) Right of the Units in all constitutional
and joint sitting of Parliament amendments
3. Weimar Constitution of Germany: (d) Division of powers between the centre
Fundamental duties and the units
4. Irish Constitution: Directive Principles of Solution: (d)
State Policy
Q.125 In which one of the following federations the
Select the correct answer using the codes
executive is collegiate ?
below.
(a)  U.S.A (b)  Switzerland
(a)  1, 2 and 4 only (b)  2 and 3 only
(c)  Australia (d)  Nigeria
(c)  1, 3 and 4 only (d)  2 and 4 only
Solution: (b)
Solution: (d)
Justification: Statement 1: They were
borrowed from British constitution. Switzerland Constitution
Statement 3: They were borrowed from Q.126 Which one of the following is the common
Soviet Constitution. feature of Indian and Swiss federalism ?
Q.121 The structural part of the Constitution is (a) Judicial Review
largely derived from the (b) Equal representation of the Units in the
Upper House

 P.32 For Civil Services Preliminary Examination

01-Indian Polity_Q1-412.indd 32 8/7/2018 7:39:03 PM


(c) Right of the Units in all constitutional Learning: ‘Preamble’refers to the introduction
amendment. or preface to the Constitution. It contains the
(d) Division of powers between the centre summary or essence of the Constitution. The
and the units . Preamble to the Indian Constitution is based
on the ‘Objectives Resolution’, drafted and
moved by Pandit Nehru, and adopted by the
(4) PREAMBLE Constituent Assembly.
Q.127 Consider the following statements with Q.129 Which of the following does NOT find
reference to the Preamble of the Constitution? mention in the Preamble to India’s
1. Taking inspiration from the American Constitution?
model, India has chosen to begin its 1. Social justice 2.  Liberty of worship
constitution with a preamble. 3. Equality of status 4.  Liberty of religion
2. Values that inspired and guided the Choose the correct answer using the codes
freedom struggle are embedded in the below:
Preamble of the Indian Constitution. (a)  2 Only (b)  3 Only
3. It is the soul of the Indian Constitution. (c)  2 and 4 Only (d)  4 Only
4. It provides a standard to examine Solution: (d)
and evaluate any law and action of These are the opening words of the
government. preamble to the Indian Constitution
Which of the statement is/are correct? “WE, THE PEOPLE OF INDIA, having
(a)  2 and 4 (b)  2 and 3 solemnly resolved to constitute India into
(c)  2, 3 and 4 (d)  1, 2, 3 and 4 a SOVEREIGN SOCIALIST SECULAR
Solution: (d) DEMOCRATIC REPUBLIC and to secure to
Values that inspired and guided the freedom all its citizens:
struggle and were in turn nurtured by it, JUSTICE, social, economic and political;
formed the foundation for India’s democracy. LIBERTY of thought, expression, belief,
These values are embedded in the Preamble faith and worship; EQUALITY of status and
of the Indian Constitution. They guide all the of opportunity; and to promote among them
articles of the Indian Constitution. all FRATERNITY assuring the dignity of
Taking inspiration from American model, the individual and the unity and integrity
most countries in the contemporary world of the Nation; IN OUR CONSTITUENT
have chosen to begin their constitutions with ASSEMBLY this twenty-sixth day of
a preamble. November, 1949, do HEREBY ADOPT,
It contains the philosophy on which the ENACT AND GIVE TO OURSELVES
entire Constitution has been built. It provides THIS CONSTITUTION.
a standard to examine and evaluate any law Q.130 Which of the following terms does NOT find
and action of government, to find out whether a mention in the Preamble to the constitution
it is good or bad. It is the soul of the Indian of India?
Constitution. (a)  Democratic Republic
Q.128 The Preamble to the Indian Constitution is (b)  Unity and Integrity of the Nation
based on the (c)  Welfare State
(a) Preamble of the Nehru Report produced (d) Secular
in 1928 Solution: (c)
(b) Prelude of the Government of India Act, Q.131 You will not find which of the following
1935 ‘ideals’ or ‘values’ in the Preamble of the
(c) ‘Objectives Resolution’ moved in the Constitution?
Constituent Assembly in 1946 (a)  Liberty (b)  Fraternity
(d) ‘Outcome Paper’ of the Lahore Session (c)  Rationality (d)  Justice
of the Indian National Congress, 1930. Solution: (c)
Solution: (c)

Indian Polity Question Bank P.33

01-Indian Polity_Q1-412.indd 33 8/7/2018 7:39:03 PM


Rationality and scientific temper are hinted at Justification: Statement 1: The Preamble
in the Fundamental Duties in Part IV of the of the Constitution like the preamble of any
constitution. statute furnish the key to open the mind of
Q.132 Which of the following words are attached to the makers of the Constitution more so
the term “Republic” in the Preamble? because the Constituent Assembly toiled
1. Sovereign 2.  Parliamentary hard in formulating it so that it may reflect
3. Secular 4.  Federal the essential features and basic objectives of
Select the correct answer using the codes below. the Constitution.
(a)  1 and 2 only (b)  1 and 3 only Statement 2: The Preamble does not grant
(c)  2, 3 and 4 only (d)  1, 2, 3 and 4 any power but it gives direction and purpose
Solution: (b) to the Constitution .It outlines the objective
of the whole Constitution. The Preamble
contains the fundamental of constitution. It
Ingredients of Preamble serves several important purposes like
Q.133 Consider the following statements about the 1. It contains the enacting clause which
Preamble of the Constitution. brings the Constitution into force.
1. The Preamble is the source of authority of 2. It declares the basic type of government
the Constitution. and polity which is sought to be
2. It declares the nature of the Indian state. established in the country.
3. It envisages political as well as social 3. It declares the great rights and freedom
ideals. which the people of India intended to
Choose the correct answer using the codes secure to its entire citizen
below. Q.135 Consider the following statements.
(a)  1 and 2 only (b)  2 and 3 only Assertion (A): The Constitution of India
(c)  1 and 3 only (d)  All of the above derives its authority from the Parliament of
Solution: (b) India.
The Preamble reveals four ingredients or Reason (R): It was enacted by the constituent
components: assembly of India which had legal and
1. Source of authority of the Constitution: The democratic authority.
Preamble states that the Constitution derives In the context of the statements above, which
its authority from the people of India. of these is true?
2. Nature of Indian State: It declares India (a) A and R both are true, and R is the correct
to be of a sovereign, socialist, secular explanation for A.
democratic and republican polity. (b) A and R both are true, and R is the NOT
3. Objectives of the Constitution: It specifies the correct explanation for A.
justice, liberty, equality and fraternity as (c) A is correct, R is incorrect.
the objectives. (d)  A is incorrect, R is correct.
4. Date of adoption of the Constitution: It Solution: (d)
stipulates November 26, 1949 as the date. The Preamble clearly tells that the source
Q.134 The Preamble is used by the Courts in of authority of the constitution is the people
constitutional interpretation because of India. A democratic polity, as stipulated
1. It contains the objective and philosophy in the Preamble, is based on the doctrine
of the Constitution makers. of popular sovereignty, that is, possession
2. It is the source of all constitutional of supreme power by the people. Universal
powers and limitations on authority of the adult franchise, periodic elections, rule of
government. law, independence of judiciary, and absence
Which of the above is/are correct? of discrimination on certain grounds are the
(a)  1 only (b)  2 only manifestations of the democratic character of
(c)  Both 1 and 2 (d)  None the Indian polity
Solution: (a)

 P.34 For Civil Services Preliminary Examination

01-Indian Polity_Q1-412.indd 34 8/7/2018 7:39:03 PM


Preamble as Part of Constitution and the opinion of the founding fathers of the
Constitution.
It’s Amendability However, two things should be noted:
1. The Preamble is neither a source of power
Q.136 The status of the Preamble of the Constitution
to legislature nor a prohibition upon the
has always been a question of debate in the
powers of legislature.
country. The present opinion of the Supreme
2. It is non-justiciable, that is, its provisions
Court about the status of the Preamble is that
are not enforceable in courts of law.
(a) It is a part of the constitution but cannot
be amended. Q.137 Which of the following statements is correct
(b) It is a not part of the constitution and about the Preamble of the constitution?
cannot be amended. (a) It was enacted by the constituent
(c) It is a part of the constitution and can be assembly.
amended. (b) It is a source of power to legislature.
(d) It is not a part of the constitution but can (c) Its provisions are enforceable in courts
be amended. of law.
Solution: (c) (d) The Supreme Court has never considered
One of the controversy about preamble is Preamble as a part of the constitution.
whether it’s part of constitution or not. Solution: (a)
In Berubari Union Case, the Supreme Refer previous explanation.
Court held that preamble is not part of Q.138 The 42nd Amendment to the Constitution
constitution. amended the Preamble to add the words
But in historic case of Kesavananda (a)  Sovereign and democratic
Bharati v. State of Kerala, (1973) 4 SCC (b)  Socialist and Secular
225, Supreme Court, however, held that (c)  Republic and Integral
the Preamble is a part of the Constitution. (d)  Liberty and Equality
The Court stated that the Preamble can be Solution: (b)
amended, subject to the condition that no Justification: It changed the description of
amendment is done to the ‘basic features’. India from “sovereign democratic republic”
The Preamble has been amended only once to a “sovereign, socialist secular democratic
so far, in 1976, by the 42nd Constitutional republic”, and also changed the words “unity of
Amendment Act, which has added three new the nation” to “unity and integrity of the nation”.
words—Socialist, Secular and Integrity—to Learning: The 42nd amendment was a
the Preamble. This amendment was held to major as well as controversial amendment to
be valid the constitution.
Like any other part of the Constitution, In Minerva Mills v. UOI, AIR 1980
the Preamble was also enacted by the SC 1789, the Supreme Court declared
Constituent Assembly, but, after the rest of unconstitutional two provisions of the 42nd
the Constitution was already enacted. The Amendment which prevent any constitutional
reason for inserting the Preamble at the end amendment from being “called in question
was to ensure that it was in conformity with in any Court on any ground” and accord
the Constitution as adopted by the Constituent precedence to the Directive Principles of
Assembly. State Policy over the Fundamental Rights of
While forwarding the Preamble for votes, individuals respectively.
the president of the Constituent Assembly Q.139 Consider the following statements about the
said, ‘The question is that Preamble stands Preamble:
part of the Constitution’. The motion was 1. It is a part of the constitution.
then adopted. 2. It grants power to the legislature to enact
Hence, the current opinion held by the laws that are in conformity with its values.
Supreme Court that the Preamble is a part 3. It can be selectively amended by the
of the Constitution, is in consonance with legislature.

Indian Polity Question Bank P.35

01-Indian Polity_Q1-412.indd 35 8/7/2018 7:39:03 PM


Select the correct answer using the codes Supreme Court of India in Beru Bari case
below therefore it is not enforceable in a court of
(a)  1 and 2 (b)  Only 1 law. However, Supreme Court of India has,
(c)  1 and 3 (d)  Only 3 in the Kesavananda case, recognised that the
Solution: (c) preamble may be used to interpret ambiguous
Refer previous explanation areas of the constitution where differing
Q.140 Consider the following statements. interpretations present themselves. In UOI v.
Assertion (A): Preamble of the Constitution LIC of India, 1995 also the Supreme Court
cannot be amended by the Parliament. has once again held that Preamble is the
Reason (R): Preamble is not considered as integral part of the Constitution. Democracy
a part of the constitution. stands much superior to any other form
In the context of the above, which of these of government in promoting dignity and
is correct? freedom of the individual. Every individual
(a) A is correct, and R is an appropriate wants toreceive respect from fellow beings.
explanation of A. Often conflicts arise among individuals
(b) A is correct, but R is not an appropriate because some feel that they are not treated
explanation of A. with due respect. The passion for respect and
(c) A is correct, but R is incorrect. freedom are the basis of democracy.
(d)  Both A and R are incorrect.
Solution: (d) Sovereignty
Refer previous explanation
Q.142 The “Sovereignty” of India means
Q.141 Consider the following statements: 1. No external power can dictate the
1. The Constitution of India ensures equality government of India.
of citizens within religious communities. 2. Citizens cannot be discriminated against
2. A democratic government is the most on any grounds.
legitimate government 3. There is absolute freedom of speech and
3. Preamble is not an integral part of the expression for Indian  citizens.
Constitution 4. All citizens have equal economic rights.
4. Democracy helps in promoting dignity Select the correct answer using the codes below.
and freedom of the individual (a)  1, 2 and 3 only (b) 1 only
Which of the following statements is/are (c)  3 and 4 only (d) 2 and 4 only
correct? Solution: (b)
(a)  1 Only (b)  1 and 4 Justification: Statement 1: Sovereignty
(c)  1, 2 and 4 (d)  1, 2, 3 and 4 simply implies that India is a state that takes
Solution: (c) its own decisions ultimately guided by the
The Constitution of India ensures equality people. No external agency dictates terms to
of citizens within religious communities India. So, 1 is correct.
(as provided under Fundamental Rights – Statement 2: However, the notion of
Articles 25 to 28). sovereignty does not seep into the framework
There is one respect in which democratic of all democratic rights. For example, a
government is certainly better than its sovereign state may very well discriminate
alternatives: democratic government is between its citizens, like in Pakistan where it
legitimate government. It may be slow, did with all non-Muslims.
less efficient, not always very responsive Statement 3: Same applies for freedom of
or clean. But a democratic government is speech and expression. A sovereign state may
people’s own government. That is why there very well restrict freedom of speech, as it is
is an overwhelming support for the idea of not an absolute right, even though it is vital
democracy all over the world. to a democracy.
The preamble is not an integral part of the Statement 4: This is only possible in an
Indian constitution was first decided by the economic democracy.

 P.36 For Civil Services Preliminary Examination

01-Indian Polity_Q1-412.indd 36 8/7/2018 7:39:03 PM


Q.143 In 1949, India declared the continuation of a democracy, political authority flows from
her full membership of the Commonwealth the people to the government—not from
of the Nations. But, this extra-constitutional government to the People. Popular sovereignty
declaration did not affect India’s sovereignty means that the government can only exercise
in any manner as authority if it has been given permission
1. The position of Head of Commonwealth to do so by the People. Therefore, popular
rotates among member nations on a sovereignty limits the powers of government.
regular basis. Q.145 In the Indian Polity, ‘Sovereignty’ ultimately
2. The Commonwealth is a voluntary lies with
association of independent nations. (a)  The elected Government
Which of the above is/are correct? (b)  The Supreme Court
(a)  1 only (b) 2 only (c)  The Parliament
(c)  Both 1 and 2 (d) None (d)  The People of India
Solution: (b) Solution: (d)
Justification: Statement 1: Under the formula Justification: It is clear from the Preamble
of the London Declaration, Queen Elizabeth that the sovereignty lies ultimately with the
II is the Head of the Commonwealth. So, 1 people of India when it declares that the
is wrong. “We the people of India” have created this
Statement 2: Being a member of republic.
commonwealth also does not affect India’s Option A: The elected government derive
republican character as India neither pays their legitimacy from the people.
final allegiance to the British Crown nor Option B: The SC acts as a guardian of the
the latter has any functions to discharge Constitution, it is not a sovereign entity.
in relation to India since it is a voluntary Option C: Parliament is not sovereign
organization. since it is governed by the Constitution,
Q.144 “Popular sovereignty” implies and its power is ultimately derived from the
1. People are the ultimate source of the people.
authority of their government.
2. A region can be declared sovereign by the Socialist
wish of the people.
Which of the above is/are correct? Q.146 The word ‘Socialist’ was added to the
(a)  1 only (b)  2 only Preamble, its main aim is to
(c)  Both 1 and 2 (d)  None (a) eliminate inequality in economic and
Solution: (a) political status
Justification: Popular sovereignty is a (b) eliminate inequality in political and
basic idea of democracy. In a democracy, religious affairs
political authority flows from the people to (c) eliminate inequality in income and status
the government—not from government to the and standards of life
People. (d) eliminate class based society
Popular sovereignty means that the people Solution: (a)
are the ultimate source of the authority of their Q.147 Avadi session of Indian National Congress
government. Popular sovereignty means that was famously known for
democratic government is by the people and 1. Adopting a resolution to establish a
for the people—for the benefit of the people, ‘socialistic pattern of society’
not for the benefit of those who govern in 2. Publishing the ‘Magna carta’ of a
their name. Government in a democracy is libertarian democratic framework for India
the servant of the people; it is not their master. Which of the above is/are correct?
Therefore, those who govern are public (a)  1 only (b)  2 only
servants—they hold public office only to (c)  Both 1 and 2 (d)  None
serve the people, not to serve themselves. In Solution: (a)

Indian Polity Question Bank P.37

01-Indian Polity_Q1-412.indd 37 8/7/2018 7:39:03 PM


Justification: The famous session of the 2. The government decides how goods are
Indian National Congress was held at Avadi to be produced and how they should be
in January 1955. distributed.
This historical meet emphasised the 3. Strictly, a socialist society has no private
importance of socialism and its impact on property since everything is owned by the
social development. state.
Jawaharlal Nehru with Morarji Desai Which of the statements given above is/are
and other Congress leaders at the All India correct?
Congress Committee (AICC) session (a)  1 and 2 Only (b)  2 Only
declared that a socialistic pattern of society (c)  2 and 3 Only (d)  All
was the goal of the Congress. Solution: (d)
This was the basic philosophy of the INC A socialist society answers the three questions
as “It is not possible to pursue a policy of in a totally different manner.
laissez-faire in industry.... It is incompatible In a socialist society the government
with any planning. It has long been Congress decides what goods are to be produced in
policy that basic industries should be owned accordance with the needs of society. It is
or controlled by the state.” assumed that the government knows what is
“This policy holds and must be good for the people of the country and so the
progressively given effect to. State trading desires of individual consumers are not given
should be undertaken wherever the balance much importance.
of advantage lies in favour of such a course. The government decides how goods are to
A large field for private enterprise is, be produced and how they should be distributed.
however, left over. Thus our economy will
In principle, distribution under socialism is
have a public sector as well as a private
sector. But the private sector must accept supposed to be based on what people need and
the objective of the National Plan and fit not on what they can afford to purchase.
into it.” Unlike under capitalism, for example, a
Q.148 Consider the following statements: socialist nation provides free health care to
1. The kind of socialistic practice that is the citizens who need it. Strictly, a socialist
enshrined in the Preamble is that of state society has no private property since
socialism. everything is owned.
2. Indian socialism is a blend of Marxism
and Leninism. Secular
Which of these is/are true? Q.150 The Indian Constitution mandates that the
(a)  Only 1 (b)  Only 2 Indian States be secular. The objective behind
(c)  Both (d)  None of the above such a mandate is:
Solution: (d) 1.  To ensure some members do not dominate
The kind of socialistic practice that is other members of the same religious
enshrined in the Preamble is that of democratic community
socialism not state socialism. State socialism 2.  To ensure one religious community does
is basically communism, while Indian setup not dominate another
is more of a balance. Indian socialism is a 3.  To ensure that State does not enforce any
blend of Marxism and Gandhism leaning particular religion
heavily towards Gandhism. 4.  To ensure that State does not take away
Q.149 Consider the following statements about the religious freedom of individuals.
the response of government in a socialist Which of the statements given above is/are
society correct?
1. In a socialist society the government (a)  2 and 3 (b)  2 Only
decides what goods are to be produced in (c)  2, 3 and 4 (d)  1, 2, 3 and 4
accordance with the needs of society. Solution: (d)

 P.38 For Civil Services Preliminary Examination

01-Indian Polity_Q1-412.indd 38 8/7/2018 7:39:03 PM


Q.151 The version of Indian secularism means that the status of Buddhism in Sri Lanka, that of
the state can NOT Islam in Pakistan and that of Christianity in
1. Preach any religion England, our Constitution does not give a
2. Give any religious education special status to any religion.
3. Give any help to any religion. Statement 2: The Constitution provides to
Which of the following options make the all individuals and communities freedom to
question statement correct and complete? profess, practice and propagate any religion,
(a)  1 and 2 (b)  2 and 3 or not to follow any. The Constitution also
(c)  1 and 3 (d)  All of the above prohibits discrimination on grounds of religion.
Solution: (a) At the same time, the Constitution allows
Being a country which is home to several the state to intervene in the matters of religion
religions, it is necessary that the government in order to ensure equality within religious
must extend equal treatment to different communities.
religions. Negatively, it means that For example, it bans untouchability
government will not favour any particular (a practice within the Hindu Varna system).
religion. Statement 3: Perhaps the best examples
India does not have any official religion. are the government control of Vaishno Devi
We don’t have to belong to any particular Shrine Board in J&K. The state does this in
religion in order to be a prime minister or order to curb private monopoly in religious
president or judge or any other public official. dominations and the equitable use of revenue
We have also seen that under the right to earned there from.
equality, there is a guarantee that government Q.153 In the Indian context, which of the following
will not discriminate on the basis of religion practices of the government would NOT be
in giving employment. considered ‘Secular’?
The institutions run by the state will not 1. Allowing the Punjabi community not to
preach any religion or give religious education wear helmets while driving
nor will they favour persons of any religion. 2. Acquiring religions trusts
The objective of these provisions is to sustain 3. Giving subsidies for Haz (to Mecca)
and nurture the principle of secularism. Choose the correct answer using the codes
While the government can help a religion, below:
for e.g. it used to give Haj subsidies, it has to (a)  1 and 2 only
be within the limits so as to not diminish any (b)  3 only
religion or at the expense of any religion. (c)  2 and 3 only
Q.152 Which of the following is/are the key features (d)  None of the above.
of Indian Secularism? Solution: (d)
1. The Constitution does not give a special One kind of difficulty is created by the
status to any religion. tension between the western sense of the
2. The Constitution allows the state to state maintaining a distance from all religions
intervene in the religious matters in and the Indian sense of the state giving equal
order to ensure equality within religious respect to all religions. Supporters of each
communities. sense are upset by whatever the state does
3. The state can regulate and even control to uphold the other sense. Should a secular
religious institutions. state provide subsidies for the Haj pilgrimage,
Select the correct answer using the codes or manage the Tirupati-Tirumala temple
below. complex, or support pilgrimages to Himalayan
(a)  1 and 2 only (b)  2 and 3 only holy places? Should all religious holidays be
(c)  1 and 3 only (d)  1, 2 and 3 abolished, leaving only Independence Day,
Solution: (d) Republic Day, Gandhi Jayanti and Ambedkar
Justification: Statement 1: There is no Jayanti for example? Should a secular state
official religion for the Indian state. Unlike ban cow slaughter because cows are holy for a

Indian Polity Question Bank P.39

01-Indian Polity_Q1-412.indd 39 8/7/2018 7:39:03 PM


particular religion? If it does so, should it also 1. The state not promoting officially any
ban pig slaughter because another religion particular religion.
prohibits the eating of pork? If Sikh soldiers 2. There is no discrimination at all by the
in the army are allowed to have long hair and state, whether legal, administrative, etc.
wear turbans, should Hindu soldiers also be among religions.
allowed to shave their heads or Muslim soldiers 3. The state does not intervene at all in
allowed to have long beards? Questions of this matters of religion.
sort lead to passionate disagreements that are Choose the correct answer using the codes
hard to settle. below:
Some of the above mentioned dilemmas (a)  1 and 2 (b)  2 and 3
are solved by the Indian model of Secularism (c)  1 and 3 (d)  Only 1
by positive discrimination and maintaining a Solution: (d)
‘principled distance from religion’. The state does discriminate among religions.
Q.154 Which of the following is/are the desirable The jains were given minority status
characteristics of a secular society in India? nationally. They will be eligible for several
1. One religious community does not incentives which Hindus may not get officially.
dominate another. Moreover, there is a different set of laws for
2. Some members do not dominate other almost every religion. Sardars are allowed to
members of the same religious community. carry kirpans; and allowed not to wear helmet
3. The State does not interfere at all in while driving. Normal citizens are not.
religious activities. Q.156 Which of the following is NOT correct as per
Select the correct answer using the codes the Indian practice of Secularism?
below. (a) State maintains a principled distance
(a)  1 and 2 only (b)  2 and 3 only from religion.
(c)  1 and 3 only (d)  1, 2 and 3 (b) State does not interfere in any religious
Solution: (a) activity.
Justification:  Statement 1: If religion X is (c) All religious communities can
privileged, when compared to religion Y, in co-exist peacefully without negative
gaining access to public offices, paying taxes, discrimination.
enjoying public services etc., it cannot be (d) Individual has freedom to interpret
called a secular society. religious teachings differently and
Moreover, if all the wealth and political choose her/his religion.
power is amassed by religion X, it certainly Solution: (b)
is not a desirable characteristic of a secular Q.157 The value of secularism can be found in which
society.  of the following parts of the constitution?
Statement 2: For e.g. if a particular 1. Preamble
section of Hindu society is ostracized, either 2. Directive Principles of State Policy
due to their backward caste status or based 3. Fundamental Duties
on customs, it goes against the ideal of 4. Fundamental Rights
secularism. Select the correct answer using the codes
Statement 3: India is a secular state, and the below.
State interferes in religious activities for reasons (a)  1 and 2 only (b)  3 and 4 only
of public welfare and reducing exploitation. (c)  1, 3 and 4 only (d)  1, 2, 3 and 4
For e.g. it may ban religious practices that Solution: (d)
promote superstition, black magic, etc. Justification: Statement 1: The Preamble
At the same time, the state maintains a secures to all citizens of India liberty of
“principled distance” from all the religions. belief, faith and worship.
Q.155 Which of the following can be considered Statement 2: The State shall endeavour
valid under the Indian concept of to secure for all the citizens a Uniform Civil
“Secularism”? Code (Article 44 of Directive Principles).

 P.40 For Civil Services Preliminary Examination

01-Indian Polity_Q1-412.indd 40 8/7/2018 7:39:03 PM


Statement 3: Article 51A mentions (c) The constitution ensures fundamental
this as one of the fundamental duties: “to human rights.
promote harmony and the spirit of common (d)  There is an elected Parliament.
brotherhood amongst all the people of Solution: (b)
India transcending religious, linguistic Learning: A democratic polity can be
and regional or sectional diversities and to classified into two categories— monarchy
renounce practices derogatory to the dignity and republic.
of women.” • In a monarchy, the head of the state
Statement 4: (usually king or queen) enjoys a hereditary
• The State shall not deny to any person position, that is, he comes into office
equality before the law or equal protection through succession, eg, Britain.
of the laws (Article 14). • In a republic, on the other hand, the head
• The State shall not discriminate against of the state is always elected directly or
any citizen on the ground of religion indirectly for a fixed period, eg, USA.
(Article 15). • Therefore, the term ‘republic’ in our
• Equality of opportunity for all citizens in Preamble indicates that India has an
matters of public employment (Article 16). elected head called the president.
• All persons are equally entitled to • He is elected indirectly for a fixed period
freedom of conscience and the right to of five years. A republic also means two
freely profess, practice and propagate any more things: one, vesting of political
religion (Article 25). sovereignty in the people and not in a
• Every religious denomination or any of its single individual like a king; second, the
section shall have the right to manage its absence of any privileged class and hence
religious affairs (Article 26). all public offices being opened to every
• Similarly Articles 27-30 also uphold citizen without any discrimination.
values of secularism. Q.160 India is a democratic republic, because:
• The Constitution of India stands for a (a)  there is independence of judiciary
secular state. Hence, it does not uphold (b) the Head of the State is elected by the
any particular religion as the official people
religion of the Indian State. (c) there is distribution of powers between
Q.158 Which of the following articles of the the Centre and the States
Constitution reveal the secular character of (d)  there is Parliamentary supremacy
the Indian State? Solution: (b)
1. Article 14 2.  Article 15 and 16
3. Article 30 4.  Article 44 Democratic
Choose the correct answer using the codes
below. Q.161 The Indian constitution explicitly contains
(a)  1 and 4 only (b)  2 and 3 only which of the following provisions for
(c)  1, 2 and 3 only (d)  All of the above exercising direct democracy?
Solution: (d) 1. Referendum 2.  Plebiscite
The term ‘secular’ was added to the Preamble 3. Recall 4.  Initiative
of the Indian Constitution by the 42nd Choose the correct answer using the codes
Constitutional Amendment Act of 1976. below.
(a)  1 and 2 only (b)  1 only
(c)  3 and 4 only (d)  None of the above
Republic Solution: (d)
Q.159 India is a republican polity because Democracy is of two types—direct and
(a)  There is Universal Adult Franchise. indirect. In direct democracy, the people
(b) There are no hereditary elements in our exercise their supreme power directly as
polity. is the case in Switzerland. There are four

Indian Polity Question Bank P.41

01-Indian Polity_Q1-412.indd 41 8/7/2018 7:39:03 PM


devices of direct democracy, namely, For example, recently in Greece, the
Referendum, Initiative, Recall and Plebiscite. government asked the voter to choose an
In indirect democracy, on the other hand, the austerity package for Greece or choose to
representatives elected by the people exercise exit from European Union.
the supreme power and thus carry on the Q.163 Which of the following are closes to direct
government and make the laws. This type democracy?
of democracy, also known as representative 1. Decisions taken by the Gram Sabha
democracy, is of two kinds—parliamentary 2. Decisions taken by the legislative
and presidential. Assembly of a state
The Indian Constitution provides for 3. Decisions taken in the Board of Directors
representative parliamentary democracy of a corporation
under which the executive is responsible to Choose the correct answer using the codes
the legislature for all its policies and actions. below:
1. Referendum is a procedure whereby a (a)  1 and 2 (b)  2 and 3
proposed legislation is referred to the (c)  1 and 3 (d)  Only 1
electorate for settlement by their direct Solution: (d)
votes. The Board of directors consist of either
2. Initiative is a method by means of which direct representatives or nominees. So it is an
the people can propose a bill to the example of indirect democracy. Same can be
legislature for enactment. said of the assembly.
3. Recall is a method by means of which the In the case of Gram Sabha, it is the most
voters can remove a representative or an direct decision-making directly by the people.
officer before the expiry of his term, when
Q.164 The concept of ‘Participatory democracy’ in
he fails to discharge his duties properly.
play can be seen in which of the following
4. Plebiscite is a method of obtaining the
bodies?
opinion of people on any issue of public
1. Gram Sabha
importance. It is generally used to solve
2. State Legislative assembly
the territorial disputes.
3. District Planning Committee
Q.162 Plebiscite refers to the process where Choose the correct answer using the codes
(a) Citizens of a country can propose below:
legislation for adoption by the legislature (a)  3 only (b)  1 and 3
(b) Local citizen bodies monitor important (c)  1 only (d)  All of the above
government decisions Solution: (c)
(c) Citizens vote to accept or reject a Both the concepts of participatory democracy
proposal and decentralised governance have thus
(d) Citizens can amend the constitution of a become popular. Participatory democracy is
nation by a special procedure a system of democracy in which the members
Solution: (c) of a group or community participate
Justification: Option (a) is the like the collectively in the taking of major decisions.
practice of “initiative” present in some Panchayati Raj is a major example where
democratic nations like Canada. participatory democracy is being practiced.
Option (b) is akin to Panchayati Raj For e.g. in MGNREGA major project
Institutions (PRIs) and Urban Local Bodies decisions, approvals, social audit etc. are
(ULBs). done by the local community. In other bodies
Learning: A referendum (in some countries like State LA or DPC, it is an indirect or
synonymous with plebiscite — or a vote on representative democracy.
a ballot question) is a direct vote in which
Q.165 Consider the following statements.
an entire electorate is asked to vote on a
Assertion (A): The system of monarchy
particular proposal. This may result in the
cannot exist in a Parliamentary democracy.
adoption of a new law.

 P.42 For Civil Services Preliminary Examination

01-Indian Polity_Q1-412.indd 42 8/7/2018 7:39:03 PM


Reason (R): In a democracy representatives government which derives its right to
are elected based on popular vote. govern from their consent.
In the context of the above, which of these 2. Majority rule and minority rights:
is correct? Although “the majority rules,” the
1. A is correct, and R is an appropriate fundamental rights of individuals in the
explanation of A. minority are protected.
2. A is correct, but R is not an appropriate 3. Limited government: The powers of
explanation of A. government are limited by law and a
3. A is incorrect, but R is correct. written or unwritten constitution which
4. Both A and R are incorrect. those in power obey.
Solution: (c) 4. Institutional and procedural limitations on
Justification: Democracy is of two types – powers: There are certain institutional and
republic (where the head of state is elected, procedural devices which limit the powers
like India), and monarchy (where the head of of government.
state is hereditary). Q.168 Which of the following are the manifestations
UK practices Monarchy (crown) and yet it of or the proof of the democratic character of
is a Parliamentary democracy. the Indian polity?
Q.166 Consider the following statements about 1. The executive is responsible to the Indian
autocratic and Democratic form of Parliament.
government: 2. Ban on unreasonable discrimination.
1. The autocrat is not responsible to the people. 3. Governors are nominated by the Centre.
2. Autocracy is also termed as Monarchy 4. India has a written constitution.
since decision-making style is same in Select the correct answer using the codes
both. below
3. No other form of democracy except (a)  1, 2 and 4 (b)  2 and 4
representative democracies exist till date. (c)  Only 2 (d)  1, 3 and 4
Which of the above statements is/ are true? Solution: (c)
(a)  1 and 2 (b)  2 and 3 The executive is responsible only to the Lok
(c)  1 and 3 (d)  Only 1 Sabha, not the whole Parliament, which
Solution: (d) includes Rajya Sabha too.
Autocracy and Monarchy are two different Governors hold the highest post in a
political systems. An autocrat is an absolute state government. They being nominated
dictator for e.g. in Syria. characterizes undemocratic tendencies.
A democracy is termed as monarchy when The fact that India has a written
the head of state is chosen on the hereditary constitution nowhere proves that it is a
basis. Republic form of democracy is when democratic nation. Several autocratic nations
the head of state is either directly or indirectly like Syria have written constitutions, whereas
elected. democratic nations like the U.K. do not have
Q.167 A democratic constitution aims at a written constitution.
establishing a Q.169 Which of these continents had the least
(a)  Rules-based society number of democratic countries till 1950?
(b)  Strong bureaucracy (a)  Africa (b)  South America
(c)  Powerful Rule (c) Europe (d) Asia
(d) Completely economically equal society Solution: (a)
Solution: (a) Q.170 Which of the following ideally should NOT
Learning: Constitutional Democracy is the be features of a democratic polity?
antithesis of arbitrary rule. It is democracy 1. Censorship of the press
characterized by: 2. Military can take over civilian institutions
1. Popular sovereignty: The people are the if they fail to deliver on democratic hopes
ultimate source of the authority of the

Indian Polity Question Bank P.43

01-Indian Polity_Q1-412.indd 43 8/7/2018 7:39:03 PM


3. The Government should not be run by Solution: (d)
non-elected individuals. Answer Justification:
Choose the correct answer using the codes Justification: Option A: Rule of law ensures
below: that democracy doesn’t turn into Rule of
(a)  1 and 2 (b)  2 and 3 Men.
(c)  1 and 3 (d)  All of the above Option B: Fundamental rights guard the
Solution: (a) citizens from the tyranny and despotism of
Democracy is a form of government that the rulers.
allows people to choose their rulers. In a Option C: If a certain class of citizens
democracy: only leaders elected by people are banned (without any intelligible criteria)
should rule the country, and people have from participating in elections or running
the freedom to express views, freedom to for public offices, a democracy may not be
organise and freedom to protest. Option 1 sustained.
and 2 go against these two basic tenets. Option D: Many democracies practice
Q.171 Which of the following is NOT essential to indirect election of representatives, for e.g.
have and sustain a democracy? President of USA is indirectly elected. Voters
(a) Protection of the fundamental rights of elect the Electoral College, which then elects
all citizens the President.
(b) Freedom to express views, freedom to In India, Rajya Sabha MPs are
organise and freedom to protest indirectly elected. So, direct election of all
(c)  Political equality of citizens representatives is not crucial for a democracy.
(d)  A written constitution Q.173 The famous historical ‘Magna Carta’ had laid
Solution: (d) the first steps towards
Explanation: A democracy cannot exist 1. Parliamentary Democracy
without giving some fundamental rights of 2. Rule of Law
citizens and to safeguard them via institutional 3. Liberty and rights
mechanisms. Option (a) is essential. Choose the correct answer using the codes
Without freedom to opine, express, below.
organize and protest a government can turn (a)  1 and 2 only (b)  2 and 3 only
tyrannical and authoritarian. It acts as a check (c)  1 and 3 only (d)  All of the above
and balance. Hence, option (b) is essential. Solution: (a)
Political equality of citizens refers to United Kingdom (UK) recently celebrated
equal voting rights, equal eligibility for 800 years of signing of Magna Carta on
public offices etc. In a sense, it is one man, 15 June 2015. The celebrations were for
one vote as per BR Ambedkar. Option (c) is the 8 century year old historic document
the heart of democracy. which had laid foundation for modern
A democracy can exist without a written democratic society. The document had
constitution. Britain is the best example. written promises between King John and his
Moreover, nations having a written subjects mentioning that the king will govern
constitution may not be democratic, e.g, Syria. England and deal with its people according
However, it is essential to have laws/rules that to the customs of feudal law. Magna Carta
protect citizen’s rights. Option (d) is correct. had laid the first steps towards parliamentary
Q.172 Which of the following is NOT vital for the democracy and the principle supremacy of
establishment or success of a democracy? law i.e. no one is above the law. It also laid
(a)  Presence of Rule of Law. the principle of balance of power between the
(b) Granting fundamental rights to the governed and government. Its influence can
population. be seen in other documents across the world
(c) Political equality of citizens to run for including the UN Universal Declaration of
public offices. Human Rights and on US Constitution and
(d) Direct elections of all representatives. Bill of Rights. In case of India, its influence

 P.44 For Civil Services Preliminary Examination

01-Indian Polity_Q1-412.indd 44 8/7/2018 7:39:03 PM


also can be seen on Part III of constitution Option (c): A bicameral legislature is required
which contains fundamental rights, described for a federal structure, not necessarily
as the magna carta of India which has been a Parliamentary democracy. Collective
derived inspiration from Bill of Rights. responsibility of the government (council
Q.174 How is a democracy helpful in ensuring of Ministers) to the legislature is the central
equal rights for women? feature of Parliamentary government.
(a) Women are by-default given equal degree Q.176 Consider the following examples:
of political representation in legislatures 1. King of Bhutan has declared that in future
in democracies. he will be guided by the advice given to
(b) Women in democracies cannot be him by elected representatives—Major
subjected to sex discrimination in any decisions by elected leaders
aspect of social life by the very design of 2. The Indian Supreme Court held that
democratic institutions. the dissolution of Bihar assembly was
(c) The principle of Rule of Law in most unconstitutional—Free and fair electoral
democracies reduces discrimination competition.
against women. 3. Political parties in Bangladesh have
(d) Women are given reservations in agreed that a neutral government should
political institutions by the design of any rule the country at the time of elections—
democracy. Rule of law
Solution: (c) Which of these examples is/are matched
Justification: Rule of Law is Critical to correctly with the features of a democracy?
Women’s Empowerment and Sustainable (a)  1 and 2 (b)  2 and 3
Development. There is nothing in the default (c)  Only 1 (d)  All of the above
design of a democratic institution that can Solution: (c)
prevent discrimination against women. For Options 2 and 3 should be interchanged.
e.g. many women were denied the right to The ruling of Supreme Court denotes rule of
vote in many democracies in the 1950s. law and non-arbitrariness is the functioning
But, rule of law ensures that what applies of democracy.
to men also applies to women. It depends A neutral government during elections
on the progressive ideals of a state to enact would promote free and fair electoral
progressive programs for women, and help competition.
them with positive discrimination. Q.177 Match the following examples correctly with
Q.175 A Parliamentary Democracy cannot exist the democratic principle involved.
without 1. The Dutch and the French community
(a) Direct election of the Members of share power – Community government
Parliament 2. The Panchayats and the state government
(b) A Strong Centre in India share powers – Federal
(c) Bicameral Legislature government
(d) Accountability of the government to 3. The Department of Personnel and
Parliament Training and the Central administrative
Solution: (d) tribunal (CAT) share powers - separation
Justification: Option (a): Even if the MPs of powers.
are indirectly elected, a Parliament can be Choose the correct answer using the codes
constituted, and government can be held below:
accountable to the Parliament. (a)  1 and 2 (b)  2 and 3
Option (b): Whether the Centre is strong, or (c)  1 and 3 (d)  All of the above
states are completely autonomous, it does Solution: (d)
not make a difference for a Parliamentary Q.178 Explanation will again be the same as the
democracy. However, it does affect the previous question as we are dealing with the
balance of power in the federation or Union.

Indian Polity Question Bank P.45

01-Indian Polity_Q1-412.indd 45 8/7/2018 7:39:04 PM


same concept that has been properly explained Choose the correct answer using the codes
previously. DoPT is an administrative agency below:
(executive); CAT is a judicial body. Both deal (a)  1 and 2 (b)  2 and 3
with personnel matters. Hence the separation (c)  1 and 3 (d)  2 only
of powers. Solution: (d)
Q.179 In 2005, the Supreme Court has declared Evidence across history and countries, shows
the Presidential Proclamation dissolving the that democracies have generally have lower
Bihar Assembly as unconstitutional. Among rates of economic growth than dictatorships.
the following, which democratic feature of This is because of their collective decision-
Indian polity does this case safeguards? making structures and the number of filters,
(a)  Respect for Rights and tests that these decisions have to pass.
(b)  Rule of Law Moreover, democracies often follow a middle
(c)  Right to Public office path between the road to economic growth
(d)  Universal Adult Franchise and environmental sustainability. This is to
Solution: (b) achieve economic equity and mete out justice
Explanation: There is no ‘right to public to all sections of society.
office’ in a democracy. It is equality of Q.182 Consider the following statements:
opportunity in matters of public employment. Assertion (A): Democracy remains
So, option (c) is wrong. democracy only as long as every citizen has
There are no fundamental rights involved a chance of being in majority at some point
in adjudicating such cases. Option (a) is of time.
wrong. Reason (R): If someone is barred from
Universal adult franchise was not affected being in majority at all points of time, on
in any way by the dissolution and the order of the basis of birth, caste, religion etc. then the
SC. Option (d) is wrong. democratic rule ceases to be accommodative
The SC held that dissolution was not as for that person or group.
per the provisions of the constitution and Q.183 In the context of the statements above, which
illegal. Hence, safeguarding the assembly of these is correct?
was the protection of rule of law. Option (b) (a) A and R both are correct, and R is the
is correct. correct explanation for A.
Q.180 A Democracy’s success can be measured by (b) A and R both are correct, and R is NOT
which of the following criteria? the correct explanation for A.
1. Improving the quality of decision-making (c) A is correct, R is incorrect.
for the country. (d) A is incorrect, R is correct.
2. Promoting equality among citizens. Solution: (a)
3. Providing arenas and methods to resolve An example of this can be Sri Lanka.
conflicts. Even though it is a democratic state, the
4. Enhancing the dignity of the individual majoritarianism by the Sinhalese community
Choose the correct answer using the codes has been threatening the very existence of
below: a total integrated Sri Lankan state. The Sri
(a)  All of the above (b)  2, 3 and 4 only Lankan tamils have been raising their voice both
(c)  1 and 2 only (d)  3 and 4 only violently and non-violently for a long period of
Solution: (a) time. This has also resulted in a civil war. If they
Q.181 In contrast to autocracy and authoritarianism, will be always denied a share in political power,
democracy generally tends to lead/result in SL can no longer be called a democracy.
which of the following? Q.184 Which of the following can be said to be an
1. High rate of economic growth upcoming challenge for the Indian democracy?
2. Economic equity 1. Ensuring greater powers and
3. Sectarianism responsibilities for local governments

 P.46 For Civil Services Preliminary Examination

01-Indian Polity_Q1-412.indd 46 8/7/2018 7:39:04 PM


2. Inclusion of women and minority groups Q.187 While dealing with social divisions, several
in political institutions adjustments have to be made in a democracy.
3. Extension of the federal principle to all In this context, which of the following
units of federation statements is NOT true about democracy?
Choose the correct answer using the codes 1. Democracy always leads to disintegration
below: of society on the basis of social divisions.
(a)  1 and 2 (b)  2 and 3 2. In a democracy it is possible for
(c)  1 and 3 (d)  All of the above communities to voice their grievances in
Solution: (d) a peaceful manner.
A democracy needs to deepen and widen 3. Social divisions get reflected in politics
in order to (a) include more left out groups of a country because of the political
in decision-making process; and (b) competition in that country’s democracy.
devolve more power to self-govern via Choose the correct answer using the codes
decentralization of decision-making process. below:
These two ensure greater democracy in (a)  1 and 2 (b)  2 and 3
the country. And this is the challenge most (c)  1 only (d)  None of the above
successful democracies will face in the Solution: (c)
coming years including India and U.S. Social divisions can be best accommodated
Extension of the federal principle means in a democracy because every community
nothing but giving the local government has a say in the final outcome of a policy,
similar autonomy and authority as that of the law, rule, division of resources. Besides, it is
state governments in India. all done through thoughtful, transparent and
Q.185 Power sharing is the very spirit of democracy. broad-based political negotiation.
It is generally shared in the following ways. While it is true that the politics of the
1. Horizontally between different organs of day reflects these social divisions, it by no
the government way means that it also exacerbates it. It can
2. Vertically among governments at different actually act as a binder and bring cohesion in
levels divided social interests by acting as a bridge
3. Among different social groups between them.
Which of the above is practiced in the Indian Q.188 Which of the following would definitely
democracy? threaten the democracy of a socially and
(a)  1 and 2 only (b)  2 and 3 only culturally diverse nation?
(c)  1 and 3 only (d)  All of the above 1. Existence of social differences.
Solution: (d) 2. Political expression of social differences.
Refer to the section on Power sharing in 3. Economic inequality
the 1st chapter 10th Polity NCERT. The Choose the correct answer using the codes
explanation is very long and it is suitable to below:
refer to the NCERT directly. (a)  1 and 2 (b)  2 and 3
Q.186 What can be the potential implications of (c)  2 only (d)  None of the above
power sharing in a democracy? Solution: (d)
1. Promotes people’s participation in the Existence of stark social divisions, coupled
government with very high economic inequality and
2. delays decision making process heavy politicization of the same, may
3. decreases the possibility of arbitrariness threaten democracy. Mere existence of social
Choose the correct answer using the codes differences or economic inequality does not
below: threaten democracy. They are in fact part
(a)  1 and 2 (b)  2 and 3 and parcel of any democratic nation’s life. In
(c)  1 and 3 (d)  All of the above fact, political expressions of social divisions
Solution: (d) like gender etc. help fight such divisions
collectively.

Indian Polity Question Bank P.47

01-Indian Polity_Q1-412.indd 47 8/7/2018 7:39:04 PM


Justice 1. Preamble
2. First Schedule
Q.189 Social Justice is one of the ideals enshrined 3. Third Schedule
in the preamble of the Constitution. Which 4. Fundamental Rights
of the following are correct with regard to the 5. Directive Principles of State Policy
social differences that occur in a society? Select the correct answer using the codes
1. All kinds of social differences are based below.
on accident of birth. (a)  1 and 5 only
2. Some of the differences are based on our (b)  2, 3 and 4 only
choices. (c)  1, 4 and 5 only
3. Every social difference does not lead to (d)  1, 2 and 5 only
social division. Solution: (c)
4. Democracy always leads to disintegration Justification: Statement 1: The words as
of society on the basis of social divisions. enumerated “JUSTICE, social, economic
Select the correct answer using the codes and political; EQUALITY of status and of
below opportunity; and to promote among them all
(a)  2 and 3 (b)  2, 3 and 4 FRATERNITY assuring the dignity of the
(c)  1, 3 and 4 (d)  1, 2 and 4 individual and the unity and integrity of the
Solution: (a) Nation”.
These social differences are mostly based on Statement 2: It talks about Indian states and
accident of birth. Normally we don’t choose Indian territory.
to belong to our community. We belong to Statement 3: It contains forms of Oaths and
it simply because we were born into it. We affirmations for office holders.
all experience social differences based on Statement 4:
accident of birth in our everyday lives. People • A 23. Prohibition of traffic in human
around us are male or female, they are tall and beings and forced labour
short, have different kinds of complexions, or • A 24. Prohibition of employment of
have different physical abilities or disabilities. children in factories, etc.
But all kinds of social differences are Statement 5:
not based on accident of birth. Some of the • A38. State to secure a social order for the
differences are based on our choices. For promotion of welfare of the people
example, some people are atheists. They don’t • A39 (only first part) – The State shall,
believe in God or any religion. Some people in particular, direct its policy towards
choose to follow a religion other than the one securing – (a) that the citizens, men and
in which they were born. Most of us choose women equally, have the right to an
what to study, which occupation to take up and adequate means of livelihood.
which games or cultural activities to take part
in. All these lead to formation of social groups Equality
that are based on our choices.
Every social difference does not lead to Q.191 Which of the following are NOT secured by
social division. Social differences divide the preamble of the Indian constitution to its
similar people from one another, but they citizens?
also unite very different people. Democracy 1. Social and economic equality
does ‘not always’ leads to disintegration of 2. Political equality
society on the basis of social divisions. 3. Equality of opportunity
Actually democracy is one of the best way to Select the correct answer using the codes
accommodate social diversity (as in India). below
Q.190 Which of the following from the Constitution (a)  Only 2 (b)  1 and 2
of India is/are specially relevant to Social (c)  Only 1 (d)  None of the above
Justice and Empowerment? Solution: (b)

 P.48 For Civil Services Preliminary Examination

01-Indian Polity_Q1-412.indd 48 8/7/2018 7:39:04 PM


The preamble does not state that all Indian electoral rolls on grounds of religion,
citizens will be socially, economically and race, caste or sex (Article 325).
politically equal. It merely states that there will 2. elections to the Lok Sabha and the
be justice. And justice does not mean equality. state assemblies to be on the basis of
It simply means what is fair and reasonable. adult suffrage (Article 326).
Equality of opportunity is mentioned in
preamble.
(5) UNION AND TERRITORY – PART 1
Q.192 Which of the following provisions of the
constitution ensure civic equality? Q.194 The Constitution of India, in Article 1,
1. Equality of opportunity in matters of describes India as a
public employment (a)  Federation
2. Abolition of titles (b)  Cooperative State
3. Freedom of speech and expression (c)  Union of States
Choose the correct answer using the codes (d)  Quasi-Federal Entity
below Solution: (c)
(a)  2 and 3 (b)  1 and 2 Justification: Option A: The term
(c)  Only 1 (d)  All of the above ‘Federation’ has nowhere been used in the
Solution: (b) Constitution. So, A is clearly wrong.
Freedom of speech and expression is not a In S. R. Bommai v. UOI, 1994 AIR 1918,
measure of civic equality but that of individual however, the Supreme Court laid down that
liberty. The other two ensure that citizens are the Constitution is federal and characterised
treated without any special privileges. federalism as its ‘basic feature’.
Q.193 Which of the following provisions of the 1. Option C: Article 1, on the other hand,
chapter on Fundamental Rights in the describes India as a ‘Union of States’ which
Constitution ensure civic equality? implies two things: Indian Federation is
1. Article 13 2.  Article 14 not the result of an agreement by the states
3. Article 16 4.  Article 19 2. no state has the right to secede from the
Choose the correct answer using the codes federation.
below. Hence, the Indian Constitution has been
(a)  2 and 3 only (b)  2 and 4 only variously described as ‘federal in form but
(c)  1, 3 and 4 only (d)  1, 2 and 3 only unitary in spirit’, ‘quasi-federal’, ‘bargaining
Solution: (a) federalism’, ‘co-operative federalism’, etc.
The Preamble secures to all citizens of Q.195 Consider the following statements.
India equality of status and opportunity. 1. ‘Territory of India’ is a wider expression
This provision embraces three dimensions than the ‘Union of India’ as the later
of equality—civic, political and economic. includes only states while the former
The following provisions of the chapter on includes not only the states but also union
Fundamental Rights ensure civic equality: territories and territories that may be
(a) Equality before the law (Article 14). acquired by the Government of India at
(b) Prohibition of discrimination on grounds any future time
of religion, race, caste, sex or place of 2. India can acquire foreign territories
birth (Article 15). according to the modes recognised by
(c) Equality of opportunity in matters of international law.
public employment (Article 16). 3. Parliament can establish new states that
(d) Abolition of untouchability (Article 17). were not a part of the Union of India.
(e) Abolition of titles (Article 18). Choose the correct answer using the codes
1. There are two provisions in the below.
Constitution that seek to achieve (a)  1 and 2 only (b)  2 and 3 only
political equality. No person is to be (c)  1 and 3 only (d)  All of the above
declared ineligible for inclusion in Solution: (d)

Indian Polity Question Bank P.49

01-Indian Polity_Q1-412.indd 49 8/7/2018 7:39:04 PM


Notably, the ‘Territory of India’ is a wider 2. The Indian constitution permits the
expression than the ‘Union of India’ because Indian government to acquire any foreign
the latter includes only states while the territory by even conquest or subjugation.
former includes not only the states but also Which of these is/are correct?
union territories and territories that may be (a)  Only 1 (b)  Only 2
acquired by the Government of India at any (c)  Both (d)  None of the above
future time. The states are the members of Solution: (b)
the federal system and share a distribution of Jammu and Kashmir is an exception to this
powers with the Centre. The union territories rule. It has its own constitution and several laws
and the acquired territories, on the other and constitutional provisions are applicable to
hand, are directly administered by the Central J&K only by the assent of the state.
government. India is a sovereign state. So it can acquire
Being a sovereign state, India can acquire a foreign territory using any method, even
foreign territories according to the modes tyrannical ones. Had it been a dominion, it
recognised by international law. could do so only by the permission of the
1. cession (following treaty, purchase, gift, ruling nation.
lease or plebiscite)
2. occupation (hitherto unoccupied by a State Reorganization
recognised ruler)
3. conquest or subjugation. Q.198 Article 3 of the Constitution authorizes the
For example, India acquired several foreign Parliament to
territories such as Dadra and Nagar Haveli; 1. Increase or diminish the area of any state
Goa, Daman and Diu; Puducherry; and 2. Merge a state and Union Territory to form
Sikkim since the commencement of the a new state
Constitution. 3. Grant special status to the newly created
Notably, Article 2 relates to the admission weak states
or establishment of new states that are not Choose the correct answer using the codes
part of the Union of India. below.
Article 3, on the other hand, relates to the (a)  1 and 2 only (b)  2 and 3 only
formation of or changes in the existing states (c)  1 and 3 only (d)  All of the above
of the Union of India. In other words, Article Solution: (a)
3 deals with the internal re-adjustment inter Article 3 authorizes the Parliament to:
se of the territories of the constituent states of (a) form a new state by separation of territory
the Union of India. from any state or by uniting two or more
states or parts of states or by uniting any
Q.196 Consider the following statements:
territory to a part of any state,
1. The constitution does not contain any
(b) Increase the area of any state,
separate provisions for the administration
(c) Diminish the area of any state,
of acquired territories.
(d) Alter the boundaries of any state, and
2. There is no uniformity in the administrative
(e) Alter the name of any state
system of the Union Territories even while
they belong to a common category. Q.199 Which of the following is the competent
Which of these is/are true? authority to change the name of any State of
(a)  Only 1 (b)  Only 2 India?
(c)  Both (d)  None of the above (a)  Concerned State Legislature
Solution: (c) (b)  Governor concerned
(c) Parliament
Q.197 Consider the following statements:
(d)  Union Home Minister
1. The provisions of the constitution do
Solution: (c)
not allow the Union government to
Learning: The Constitution authorizes the
discriminate between states.
Parliament to form new states or alter the

 P.50 For Civil Services Preliminary Examination

01-Indian Polity_Q1-412.indd 50 8/7/2018 7:39:04 PM


areas, boundaries or names of the existing Select the correct answer using the codes
states without their consent. below.
The United Provinces was the first state (a)  1 only (b)  1 and 2 only
to have a new name. It was renamed ‘Uttar (c)  2 and 3 only (d)  None of the above
Pradesh’ in 1950. In 1969, Madras was Solution: (a)
renamed ‘Tamil Nadu’. Justification: Statement 1: It is mentioned as
Q.200 Consider the following statements about the a condition in Article 3 of the constitution.
process of creation of new states. Statement 2: Before recommending the bill,
1. A bill which intends to form a new state the President has to refer the same to the state
can be introduced in the Parliament only legislature concerned for expressing its views
with the prior recommendation of the within a specified period.
President. Statement 3: There is no such provision.
2. Before the bill is approved by the The division of Andhra Pradesh took place
Parliament, it refers the same to the state despite the opposition of the state legislature.
legislature concerned for expressing its Q.202 The Parliament can redraw the political map
views within a specified period. of India according to its will. Which of these
Which of the above is/are correct? arguments or statements support this view?
(a)  1 only (b)  2 only 1. Parliament is not bound by the views of
(c) Both 1 and 2 (d)  None the state legislature and may either accept
Solution: (a) or reject them.
Article 3 lays down two conditions in this 2. The constitution must be amended under
regard: Article 368 to accommodate new states, for
1. a bill contemplating the above changes which states do not play any decisive role.
can be introduced in the Parliament only Which of the above is/are correct?
with the prior recommendation of the (a)  1 only (b)  2 only
President (c)  Both 1 and 2 (d)  None
2. before recommending the bill, the Solution: (a)
President has to refer the same to the state Q.203 Which one of the following statements
legislature concerned for expressing its regarding the process of creation of new
views within a specified period. states by the Parliament is NOT correct?
The President (or Parliament) is not (a) The Parliament may by law form a new
bound by the views of the state legislature State and alter the boundaries or names
and may either accept or reject them, even of existing States.
if the views are received in time. (b) A Bill to this effect cannot be introduced
Further, it is not necessary to make in the Parliament except on the
a fresh reference to the state legislature recommendation of the President.
every time an amendment to the bill is (c) A Bill to this effect may be referred by
moved and accepted in Parliament. the President to the Legislature of the
Q.201 Consider the following about the Parliament’s affected State.
power to reorganise the States? (d) Such a law will fall under the purview of
1. A State reorganization bill can be Article 368 meaning amendment of the
introduced in the Parliament only with the constitution.
prior recommendation of the President. Solution: (d)
2. The Home Minister must send the Justification: Creating a new state, increase
bill to the Governor concerned for his in the area of any State; diminishing the area
ratification. of any State; altering the boundaries of any
3. If a state legislature unanimously opposes State; alter the name of any State: all of these
reorganization of the state concerned, the would give effect to some changes in the
bill is sent to the Supreme Court for a various entries of the constitution, yet, none of
review. them is called a Constitution Amendment Act.

Indian Polity Question Bank P.51

01-Indian Polity_Q1-412.indd 51 8/7/2018 7:39:04 PM


This means that just to increase, decrease different regions, particularly South India, for
areas, create a new state and alter name reorganisation of states on linguistic basis.
and boundaries is not required to bring a • Accordingly, in June 1948, the
constitution amendment bill. Government of India appointed the
These changes are statutory changes Linguistic Provinces Commission under
affected by a law enacted by the parliament. the chairmanship of S. K. Dhar to examine
For example, Telangana was created by the feasibility of this.
the Andhra Pradesh Reorganisation Bill, • The commission submitted its report in
2014 (Telangana). December 1948 and recommended the
Q.204 Consider the following statements about reorganisation of states on the basis of
the Indian Parliament’s power to reorganize administrative convenience rather than
states: linguistic factor.
1. Any bill for reorganization of the states • This created much resentment and led
can only be introduced with prior to the appointment of another Linguistic
Presidential recommendation. Provinces Committee by the Congress
2. The bill for reorganization of state(s) must in December 1948 itself to examine the
be referred to the concerned state(s) every whole question afresh.
time it is amended. • As discussed earlier that three of
3. The Parliament can destroy any Indian contemporary stalwarts of Indian Politics
state by diminishing its territorial area to were associated with this committee.
zero. • It submitted its report in April 1949 and
Which of these is/are true? formally rejected language as the basis for
(a)  1 and 2 (b)  1, 2 and 3 reorganisation of states.
(c)  Only 1 (d)  None of the above • However, in October 1953, the
Solution: (c) Government of India was forced to create
The Indian parliament can destroy any state the first linguistic state, known as Andhra
except the J&K. Even to alter its name or area state, by separating the Telugu speaking
it would need the consent of the J&K state areas from the Madras state.
legislature. Q.206 Which of the following regions were
Referring the bill to the state legislature integrated by referendum in India?
more than once is a discretionary power of 1. Hyderabad 2.  Junagarh
the President. He may or may not do it every 3. Sikkim 4.  Nagaland
time the bill is amended. Select the correct answer using the codes
below.
Evolution of States & Union Territories (a)  2 and 3 only (b)  1 and 4 only
(c)  1, 2 and 3 only (d)  3 and 4 only
Q.205 JVP committee (Jawaharlal Nehru– Solution: (a)
Vallahbhai Patel–Pattabhi Sitaramayya) is Justification: Hyderabad was integrated by
associated with means of police action whereas Junagarh and
(a) Integration of princely states with India Sikkim (Sikkim later in 1974) by means of
(b) Reorganization of states on linguistic referendum. Kashmir was integrated by an
basis instrument of accession. Nagaland was part
(c) Reservation concerns for minority of Indian Union later separated from Assam.
communities
Q.207 Consider the following statements with
(d) Regressive religious provisions in
reference to Goa
colonial Indian laws
1. In January 1967, the Central Government
Solution: (b)
held a special ‘opinion poll’ in Goa asking
Learning: The integration of princely states
people to decide if they wanted to be part
with the rest of India has purely an ad hoc
of Maharashtra or remain separate.
arrangement. There was a demand from

 P.52 For Civil Services Preliminary Examination

01-Indian Polity_Q1-412.indd 52 8/7/2018 7:39:04 PM


2. This was the only time in independent Q.208 Consider the following statements.
India that a referendum-like procedure 1. Manipur and Tripura were Union Territories
was used to ascertain people’s wishes on a (UTs) that later gained statehood.
subject. 2. Meghalaya was initially declared as an
3. In 1967, Goa became a State of the Indian ‘autonomous state’ by a constitutional
Union. amendment, which later gained full
Which of the statements given above is/are statehood.
incorrect? 3. Sikkim gained Indian statehood before
(a)  3 Only (b)  1 and 2 Only Nagaland and Arunachal Pradesh gained
(c)  2 and 3 Only (d)  None statehood.
Solution: (a) Select the correct answer using the codes
Goa’s liberation below.
Although the British empire in India (a)  1 and 2 only (b)  2 and 3 only
came to an end in 1947, Portugal refused to (c)  3 only (d)  1, 2 and 3
withdraw from the territories of Goa, Diu and Solution: (a)
Daman which were under its colonial rule Justification: Statement 1: In 1963, the State
since the sixteenth century. During their long of Nagaland was formed by taking the Naga
rule, the Portuguese suppressed the people of Hills and Tuensang area out of the state of
Goa, denied them civil rights, and carried out Assam. This was done to satisfy the movement
forced religious conversions. After India’s of the hostile Nagas. In 1972, the political map
Independence, the Indian government tried of Northeast India underwent a major change.
very patiently to persuade the Portuguese 1. two Union Territories of Manipur and
government to withdraw. There was also Tripura
a strong popular movement within Goa for 2. Sub- State of Meghalaya got statehood
freedom. They were strengthened by socialist 3. the two union territories of Mizoram and
satyagrahis from Maharashtra. Finally, in Arunachal Pradesh (originally known as
December 1961, the Government of India North-East Frontier Agency—NEFA)
sent the army which liberated these territories came into being.
after barely two days of action. Goa, Diu and Statement 2: Initially, the 22nd
Daman became Union Territory. Constitutional Amendment Act (1969)
Another complication arose soon. Led created Meghalaya as an ‘autonomous
by the Maharashtrawadi Gomanatak Party state’ or ‘sub-state’ within the state of
(MGP) one section desired that Goa, as a Assam with its own legislature and
Marathi speaking area should merge with council of ministers.
Maharashtra. However, many Goans were Statement 3: In 1974, Sikkim expressed
keen to retain a separate Goan identity and its desire for greater association with India.
culture, particularly the Konkani language. Accordingly, the 35 th Constitutional
They were led by the United Goan Party Amendment Act (1974) was enacted by
(UGP). In January 1967, the Central the parliament. So, it gained statehood
Government held a special ‘opinion poll’ in after Nagaland and Arunachal Pradesh,
Goa asking people to decide if they wanted statement 3 is wrong.
to be part of Maharashtra or remain separate.
This was the only time in independent India
that a referendum-like procedure was used
(6) CITIZENSHIP – PART II —
to ascertain people’s wishes on a subject. ARTICLE 5–11
The majority voted in favour of remaining Q.209 In India, the concept of single citizenship is
outside of Maharashtra. Thus, Goa continued adopted from
as a Union Territory. Finally, in 1987, Goa (a)  England (b)  U.S.A.
became a State of the Indian Union. (c)  Canada (d)  France
Solution: (a)

Indian Polity Question Bank P.53

01-Indian Polity_Q1-412.indd 53 8/7/2018 7:39:04 PM


Q.210 Consider the following statements. (b) applies to all Indian diplomats serving
Assertion (A): The constitution does not outside India
contain any provisions with regard to (c)  applies to minors of Indian diplomats
citizenship of India. born outside India
Reason (R): The issue of citizenship was (d)  does not exist as Indian laws and
considered by Parliament for the first time constitution provides for only single
only after the Indo-China war 1962. citizenship
In the context of the above, which of these Solution: (c)
is correct? Although Indian constitution and law do
(a) A is correct, and R is an appropriate not provide for double citizenship normally,
explanation of A. the provision of dual citizenship is only
(b) A is correct, but R is not an appropriate applicable to those children of diplomats who
explanation of A. were born outside India.
(c) A is correct, but R is incorrect. Children of foreign diplomats, who are
(d)  Both A and R are incorrect. born in India, are also given dual citizenship
Solution: (d) till the period of their parent’s service in India.
Justification: The Constitution deals with the Q.213 Consider the following statements:
citizenship from Articles 5 to 11 under Part II. 1. A person who was born on 26th January,
However, it contains neither any permanent 1951 in Rangoon, whose father was a
nor any elaborate provisions in this regard. citizen of India by birth at the time of his
Bu, it identifies the persons who became birth is deemed to be an Indian citizen by
citizens of India at its commencement (i.e., descent.
on January 26, 1950). So, A is incorrect. 2. A person who was born on 1st·July, 1988
However, it does not deal with the in Itanagar, whose mother is a citizen of
problem of acquisition or loss of citizenship India at the time of his birth but the father
subsequent to its commencement. It was not, is deemed to be a citizen of India
empowers the Parliament to enact a law to by birth.
provide for such matters and any other matter Which of the statements given above is/are
relating to citizenship. correct? [CDS 2009]
Accordingly, the Parliament has enacted (a)  I only (b)  II only
the Citizenship Act, 1955, which has been (c)  Both I and II (d)  Neither I nor II
amended in 1986, 1992, 2003 and 2005. So, Solution: (c)
clearly R is incorrect.
Q.214 Consider the following statements.
Q.211 Consider the following statements about 1. In India citizens by birth as well as a
Citizenship in India: naturalised citizen are eligible for the
1. The criteria for Citizenship can be office of President.
amended by a simple majority in the 2. Foreigners staying in India do not have to
Parliament. oblige to the fundamental duties.
2. An Indian citizen can voluntary relinquish 3. The Constitution does not prescribe any
citizenship. qualifications for citizenship.
Which of these is/are true? Choose the correct answer using the codes
(a)  Only 1 (b)  Only 2 below.
(c)  Both (d)  None of the above (a)  1 and 2 only (b)  2 and 3 only
Solution: (c) (c)  1 and 3 only (d)  All of the above
The criteria for citizenship are mentioned Solution: (a)
both in the constitution and in the laws In India both a citizen by birth as well as a
enacted by the parliament. naturalised citizen are eligible for the office
Q.212 In India the provision of double citizenship of President while in USA, only a citizen by
(a) is provided to a select class of foreign birth and not a naturalised citizen is eligible
investors on strategic grounds for the office of President.

 P.54 For Civil Services Preliminary Examination

01-Indian Polity_Q1-412.indd 54 8/7/2018 7:39:04 PM


Q.215 According to the Citizenship Act, 1955 (7) FUNDAMENTAL RIGHTS –
the PIOs will get which of the following
benefits? PART III – ARTICLE 12–35
1. Exemption from appearing before the
Q.218 Which of the following is the most appropriate
local police station after every visit
description of Fundamental Rights?
2. Life-long Indian visa facility
(a) All rights that an individual should have
3. Acquire property in India except
to lead a perfect life.
agricultural/plantations.
(b) All the rights given to citizens by law.
Choose the correct answer using the codes
(c) The rights given and protected by the
below:
Constitution.
(a)  1 and 2 only (b)  3 only
(d) The rights given by the Constitution that
(c)  1 and 3 only (d)  All of the above
cannot be restricted on any ground.
Solution: (d)
Solution: (c)
Key amendments
Explanation: No option given here is an
• Citizenship Act will give them benefits
exact definition of fundamental rights. You
like life-long visa and exemption from
should use elimination in such cases.
appearing before the local police station
Option (a) is incorrect as fundamental
on every visit etc.
rights do not envisage a perfect life; Directive
Q.216 With reference to Indian citizenship Principles may do so in a sense.
and related laws, consider the following Option (b) is also incorrect, as fundamental
statements rights are generally given by the Constitution.
1. At present two year continuous stay in Option (d) is incorrect because
India is mandatory for Indian Citizenship fundamental rights can be restricted on
2. The Indian Citizenship Act, 1955 provides grounds of national security, moral order,
for acquisition, termination, deprivation, safety, etc.
determination of Indian Citizenship and Most appropriate here is option (c).
other related aspects Learning: The Fundamental Rights are
3. Under the Citizenship act, a minor, whose defined as basic human freedoms that every
parents are Indian Citizens, can register as Indian citizen has the right to enjoy for a
Overseas Citizen of India (OCI) proper and harmonious development of
Which of the above statements is/are correct? personality. These rights universally apply
(a)  1 Only (b)  1 and 2 Only to all citizens, irrespective of race, place of
(c)  1 and 3 Only (d)  2 and 3 Only birth, religion, caste or gender.
Solution: (d) These include individual rights common
Q.217 Indian citizenship of a person can be to most liberal democracies, such as
terminated if: equality before law, freedom of speech
1. a person voluntarily acquires the and expression, and peaceful assembly,
citizenship of some other country. freedom to practice religion, and the right to
2. a person who has become a citizen through constitutional remedies for the protection of
registration is sentenced to imprisonment civil rights by means of writs such as habeas
for not less than 10 years within five years corpus. Violation of these rights result in
of his registration. punishments as prescribed in the Indian
3. the Government of India is satisfied that Penal Code or other special laws, subject to
citizenship was obtained by fraud. discretion of the judiciary.
4. a person who is a citizen by birth indulges Q.219 Which of the following statements is true
in trade with an enemy country during concerning the fundamental rights in India?
war. 1. They promote the idea of political
(a)  I and III (b)  I, II and III democracy
(c)  I, III and IV (d)  I, II, III and IV 2. They operate as limitation on the powers
Solution: (c) of the state.

Indian Polity Question Bank P.55

01-Indian Polity_Q1-412.indd 55 8/7/2018 7:39:04 PM


Choose the correct answer using the codes Learning: The Fundamental Rights are meant
below for promoting the ideal of political democracy.
(a)  Only 1 (b)  Only 2 They operate as limitations on the tyranny
(c)  Both (d)  None of the executive and arbitrary laws of the
Solution: (c) legislature. In short, they aim at establishing
Q.220 The Fundamental Rights promote the idea of ‘a government of laws and not of men’.
political democracy as The Fundamental Rights are named so
1. They limit the authority of the Central because they are guaranteed and protected by
government. the Constitution, which is the fundamental
2. They secure vital political rights to the law of the land. They are ‘fundamental’ also
citizens of India. in the sense that they are most essential for the
Which of the above is/are correct? allround development (material, intellectual,
(a)  1 only (b)  2 only moral and spiritual) of the individuals.
(c)  Both 1 and 2 (d)  None Q.222 The Fundamental Rights under the Indian
Solution: (c) Constitution have often been referred to as
Justification: They operate as limitations the ‘conscience’ of the Indian Constitution.
on the tyranny of the executive and arbitrary They help guard
laws of the legislature. They are justiciable 1. Against the misuse of State power
in nature, that is, they are enforceable by the 2. The rights of minorities against the
courts for their violation. majority
Statement 2: Rights such as equality to Which of the above is/are correct?
contest for political office, right against (a)  1 only (b)  2 only
discrimination, etc. show the political and (c)  Both 1 and 2 (d)  None
social equality of citizens. Solution: (c)
Learning: The aggrieved person can directly Justification: Statement 1: Fundamental
go to the Supreme Court which can issue Rights protect citizens against the arbitrary
the writs of habeas corpus, mandamus, and absolute exercise of power by the State.
prohibition, certiorari and quo warranto for For example, the right to free speech also
the restoration of his rights. includes the right to criticize the government;
However, the Fundamental Rights are the right to life and liberty means the state
not absolute and subject to reasonable cannot take away one life without a valid law.
restrictions. Further, they are not sacrosanct Statement 2: For example, Articles 29-30
and can be curtailed or repealed by the provide minorities with a right to conserve
Parliament through a constitutional their religion and cultural heritage. In
amendment act. They can also be suspended Niyamgiri Hills in Odisha (a  tribal region),
during the operation of a National Emergency SC declared Vedanta mining as ultra vires the
except the rights guaranteed by Articles 20 constitution since it polluted the hill which
and 21. was considered as sacred by the tribals. So,
Q.221 Which of the following prevent the option 2 is correct.
establishment of an authoritarian and despotic Q.223 Fundamental rights form an integral part of
rule in the country, operate as limitations Constitution of India. Which of the following
on the arbitrary laws of the legislature and is/are correct?
protect the liberties of the people against the 1. Fundamental Rights are referred to as the
invasion by the State? conscience of the Indian Constitution.
(a) Directive Principles of State Policy 2. Fundamental Rights protect citizens
(b)  Fundamental Duties against the arbitrary and absolute exercise
(c)  Federal Structure of India of power by the State.
(d) Fundamental Rights enshrined in the 3. The Constitution guarantees the rights of
Constitution the individual against the State but not
Solution: (d) against Other individuals.

 P.56 For Civil Services Preliminary Examination

01-Indian Polity_Q1-412.indd 56 8/7/2018 7:39:04 PM


(a)  1 and 2 (b)  1 only 1. The State
(c)  1 and 3 (d)  all the above 2. Private individuals in some cases
Solution: (a) 3. Societies and associations
The Constitution of India guarantees the 4. International agencies
rights of the individual against the State as Choose the correct answer using the codes
well as against other individuals. below.
Q.224 Which of the following articles (provisions) of (a)  2, 3 and 4 only (b)  1, 2 and 3 only
the Indian Constitution provides a safeguard (c)  1 and 2 only (d)  1 only
against the excesses of the legislature and the Solution: (b)
executive? Q.227 The guarantor and defender of the
1. Article 13 2.  Article 19 fundamental rights of the Indian citizens is
3. Article 21 4.  Article 32 the
Choose the correct answer using the codes (a)  President of India
below. (b)  Parliament
(a)  1 and 4 only (b)  2 and 4 only (c)  Supreme Court of India
(c)  1, 2 and 3 only (d)  All of the above (d)  National Human Rights Commission
Solution: (d) Solution: (c)
Article 13 gives Judiciary the power to review Learning: The Supreme Court is a federal
legislative acts and administrative rules. court, the highest court of appeal, the
Article 19 gives citizens the freedom guarantor of the fundamental rights of the
of free speech, expression, etc. which can citizens and the guardian of the Constitution.
be used to exposethe wrongdoings of the Under Article 32 of the constitution, a citizen
government. can approach the Supreme Court in case of
Article 21 safeguards the lives of the infringement of fundamental rights.
citizens against arbitrary executive action. The SC can enforce the fundamental right
Article 32 gives judiciary the power to by issuing orders to the concerned authority.
correct excesses that have led to the violation NHRC only investigates cases of human
of the fundamental rights of a citizen by the rights violations; it is not the guarantor of
state. fundamental rights.
Q.225 Consider the following about fundamental Q.228 Which of the following institutions can
rights. expand and increase the scope of fundamental
1. They are not available against the action rights in the constitution?
of private individuals. 1. President of India 2.  Supreme Court
2. They can be available only to citizens and 3. High Court 4.  Parliament
not to legal or commercial entities. 5. National Human Rights Commission
Which of the above is/are correct? Select the correct answer using the codes
(a)  1 only (b)  2 only below.
(c)  Both 1 and 2 (d)  None (a)  2 and 4 only (b)  2, 3 and 4 only
Solution: (d) (c)  1, 2, 4 and 5 only (d)  1 and 4 only
Justification: Statement 1: They are available Solution: (b)
against the actions of private individuals, Explanation: In matters of constitutional
for e.g. right against untouchability. So, 1 is amendment (e.g. amending fundamental
clearly wrong. right), the President is a mere rubber
Statement 2: It is wrong. For e.g. stamp. He cannot disagree or send back any
media enjoys the right against government constitutional amendment. He must sign it.
censorship as a part of Article 19 – Freedom Statement 1 is this incorrect.
of Speech and Expression. Alongwith SC, HCs also interpret and
Q.226 Fundamental rights in the constitution are expand fundamental rights. For e.g. Delhi HC
available against the arbitrary action of gave a judgment on Lesbian, Gay, Bisexual,

Indian Polity Question Bank P.57

01-Indian Polity_Q1-412.indd 57 8/7/2018 7:39:04 PM


and Transgender (LGBT) community, by then whenever the Committee requests
declaring section 377 IPC as unconstitutional. (usually every four years).
SC has expanded the meaning of Right to • India is a party to the convention.
life and liberty (Article 21) which now also Q.230 The minimum age at which an Indian citizen
covers Right to a safe environment, Right becomes eligible to join a trade Union is
to Dignity etc – all this without any formal (a)  15 years of age or above
amendment of the constitution. Statement 2 (b) Adulthood reached at 18 years or above
and 3 are thus correct. (c)  At least 21 years of age
Parliament can directly amend the (d)  At least 35 years of age
constitution and expand the scope of Solution: (a)
fundamental rights. So, statement 4 is also Learning: The right to form trade unions
correct. and become its members are available to all
National Human Rights Commission Indian citizens who are legally eligible to
(NHRC) does not have such powers. At the work, i.e. children above 15 years of age.
most it may recommend the government Since constitution does not bar their
some steps to improve the situation of human participation in work, it doesn’t ban their
rights in the country. So, statement 5 is participation in trade unions as well.
incorrect. Statistics on Trade Unions are collected
annually by the Labour Bureau of the
International Covenant Ministry of Labour, Government of India.
Q.229 International Covenant on Civil and Political Q.231 Which of the following fundamental rights
Rights (ICCPR) does NOT cover which of can be enjoyed without any restrictions
these rights? imposed by the state?
(a)  Freedom of speech 1. Right to move the courts for issuance of
(b) Right to livelihood and employment writs
(c)  Freedom of religion 2. Protection against forced or bonded
(d)  Right to a fair trial labour
Solution: (b) 3. Prohibition of employment of children
Learning: It is a multilateral treaty adopted under 14 years in hazardous industries
by the UNGA in 1966. Choose the correct matches from the codes
• It commits its parties to respect the given below:
civil and political rights of individuals, (a)  1 and 2 (b)  2 and 3
including the right to life, freedom of (c)  1 only (d)  1 and 3
religion, freedom of speech, freedom of Solution: (b)
assembly, electoral rights and rights to Right to move the courts can be suspended
due process and a fair trial. during an emergency by the state. The
• The ICCPR is part of the International other two are enjoyed unqualified in every
Bill of Human Rights, along with the circumstance.
International Covenant on Economic, Q.232 Which of the following fundamental rights
Social and Cultural Rights (ICESCR) are denied to foreigners staying in India, but
and the Universal Declaration of Human granted to Indian citizens?
Rights (UDHR). 1. Freedom of speech and expression.
• The ICCPR is monitored by the United 2. Right against racial discrimination.
Nations Human Rights Committee (a 3. Equality before law.
separate body to the United Nations 4. Right to elementary education
Human Rights Council), which reviews Select the correct answer using the codes
regular reports of States parties on how below
the rights are being implemented. (a)  3 and 4 (b)  1 and 2
• States must report initially one year (c)  2 and 4 (d)  All of the above
after acceding to the Covenant and Solution: (b)

 P.58 For Civil Services Preliminary Examination

01-Indian Polity_Q1-412.indd 58 8/7/2018 7:39:04 PM


In the eyes of the law, the foreigner and the Select the correct answer using the codes
Indian is alike. However, the offence for below.
which both can be convicted and tried can be (a)  2 and 3 only (b)  1 and 3 only
different. For e.g. a foreigner is not entitled (c)  2 only (d)  1, 2 and 3
to free speech or protection against racial Solution: (c)
discrimination. Q.235 Which of the following fundamental rights
are conferred to Indian citizens but not
FR Available to Citizens & Aliens foreign citizens living in India?
1. Right against discrimination on grounds
Q.233 The Constitution confers which of the
of religion
following rights and privileges on the citizens
2. Right to conserve one’s culture
of India, and denies the same to aliens?
3. Right to Life and Liberty
1. Cultural and educational rights
4. Right to freedom of speech and expression
2. Right to freedom
Select the correct answer using the codes
3. Right against exploitation
below.
4. Right to equality of opportunity
(a)  2 only (b)  1, 2 and 4 only
Choose the correct answer using the codes
(c)  1 and 2 only (d)  3 and 4 only
below.
Solution: (b)
(a)  1 and 2 only (b)  2 and 3 only
(c)  3 and 4 only (d)  1, 2 and 4 only
Solution: (d) FR Available to Citizens
The following are denied to foreign citizens, Q.236 In the context of India, the Fundamental
also called aliens: Rights include:
1. Right against discrimination on grounds 1. Right against forced labour
of religion, race, caste, sex or place of 2. Protection of linguistic rights of the
birth (Article 15) minorities only.
2. Right to equality of opportunity in 3. Protection of religious rights of the
the matter of public employment minorities only.
(Article 16) 4. Right to constitutional remedies
3. Right to freedom of speech and expression, Which of the statements is/are correct?
assembly, association, movement, (a)  1 and 4 (b)  4 Only
residence and profession (Article 19) (c)  1, 3 and 4 (d)  1, 2 and 4
4. Cultural and educational rights (Articles Solution: (a)
29 and 30)
Q.237 Consider the following instances with regard
5. Right to vote in elections to the Lok Sabha
to exercise of a fundamental right?
and state legislative assembly
1. Men and women under Mahatma Gandhi
6. Right to contest for the membership of the
National Rural Employment Guarantee
Parliament and the state legislature
Act (MGNREGA) program get the same
7. Eligibility to hold certain public offices,
salary
that is, President of India, Vice-President
2. Parents property is inherited by their
of India, judges of the Supreme Court
children
and the high courts, governor of states,
3. Workers from Bihar go to the Maharashtra
attorney general of India and advocate
in search of job
general of states.
4. Christian missions set up a chain of
Q.234 The Constitution confers which of the missionary schools and allure other
following rights and privileges on the citizens persons to convert to Christianity.
of India but denies the same to aliens? Which of the statements is/are correct?
1. Right to education (a)  2 Only (b)  1 and 3
2. Right to freedom of speech and expression (c)  1,3 and 4 (d)  3 Only
3. Right against exploitation Solution: (b)

Indian Polity Question Bank P.59

01-Indian Polity_Q1-412.indd 59 8/7/2018 7:39:04 PM


Men and women under MGNREGA program Statement 2 violates the right to equal
get the same salary- Right to Equality, the opportunity to public employment.
government shall not discriminate against Understand that the movement of the
any citizen on grounds only of religion, race, labourers was not restricted, only their
caste, sex or place of birth. All citizens have employment was restricted.
equality of opportunity in matters relating to Statement 3 violates the Right to Freedom
employment or appointment to any position of Speech and Expression. Press censorship
in the government. No citizen shall be goes against it.
discriminated against or made ineligible for Q.239 Which of the following are Fundamental
appointment. Rights under Part III of the constitution of
Parents property inherited by their India?
children is not a fundamental right. 1. Right to vote
Workers from Bihar go to the Maharashtra 2.  Right to Education
in search of job comes under right to freedom. 3. Right to information
As citizens we have the freedom to travel to Choose the correct answer using the codes
any part of the country. We are free to reside below:
and settle in any party of the territory of India. (a)  1 and 2 (b)  2 and 3
Christian missions can set up a chain of (c)  1 and 3 (d)  All of the above
missionary schools (this comes under Right Solution: (b)
to Freedom of Religion) but cannot allure Right to vote is a constitutional right
other persons to convert to Christianity. mentioned in Article 326 of the constitution.
Every person has a right to profess, practice This is outside part III of the constitution.
and propagate the religion he or she believes Right To Information and has been given
in. Every religious group or sect is free the status of a fundamental right under
to manage its religious affairs. A right to Article 19(1) of the Constitution. Article 19
propagate one’s religion, however, does not (1) under which every citizen has freedom of
mean that a person has right to compel another speech and expression and have the right to
person to convert into his religion by means know how the government works, what role
of force, fraud, inducement or allurement. Of does it play, what are its functions and so on.
course, a person is free to change religion on
Q.240 Which of the following is/are fundamental
his or her own will.
rights provided by the Constitution of India?
Q.238 Consider the following matches of situations 1. No person shall be deprived of his
and the fundamental rights that they violate. property save by the authority of law.
1. A 9 year old girl child is working in a 2. All linguistic and religious minorities shall
firecracker factory—Right to equality have the right to establish and administer
2. If a politician in one state decides to educational institutions of their choice.
not allow labourers from other states to Which of the above is/are correct?
work in his state—Right to freedom of (a)  1 only (b)  2 only
movement (c)  Both 1 and 2 (d)  None
3. If the government censors a newspaper Solution: (b)
unreasonably—Right to equality before Justification: Statement 1: It is a legal
law right only and was stripped of its status as a
Select the correct matches from the above. constitutional right long back.
(a)  1 only (b)  2 and 3 only Statement 2: These are minority rights under
(c)  2 only (d)  None of the above the broader framework of Articles 25-30
Solution: (d) that protect religious rights, safeguards
Explanation & Learning: Statement 1 minorities, their culture and heritage.
violates the Right against exploitation, as it
Q.241 Which of the following fundamental rights
involves child labour, whether of a girl or a
are found under the Indian constitution as
boy.
well as the UN acknowledged human rights?

 P.60 For Civil Services Preliminary Examination

01-Indian Polity_Q1-412.indd 60 8/7/2018 7:39:04 PM


1. All are equal before the law and are Right to demonstration or picketing can be
entitled without any discrimination to a Fundamental Right but not right to strike.
equal protection of the law. Reference : Page 35 (Indian Constitution
2. No one shall be subjected to arbitrary at Work Class XI)
interference with his privacy. Q.243 Below are the instances where government
3. Right to freedom of thought, conscience has taken certain steps or the individuals
and religion. or the society have taken certain actions.
Choose the correct answer using the codes Which of the following is/are violation of
below: Fundamental Rights?
(a)  1 and 2 (b)  2 and 3 1. The district court has banned the use of
(c)  1 and 3 (d)  All of the above loudspeakers by the temples, mosques,
Solution: (d) and gurudwaras after 10 pm.
Refer to http://www.un.org/en/universal- 2. Karnataka society runs Kannada medium
declaration-human-rights/index.html schools outside Karnataka.
Q.242 Consider the following statements with 3. A director makes a documentary film that
reference to Fundamental Rights provided in criticises the policies of the government.
Constitution of India: Select the correct answer using the codes
1. The Motilal Nehru committee had below:
demanded a bill of rights as far back as (a)  1 Only (b)  2 and 3 Only
in 1928. (c)  1 and 3 Only (d)  None
2. Sometimes a person can be arrested Solution: (d)
simply out of an apprehension that he or Banning of loudspeaker after 10 pm is in
she is likely to engage in unlawful activity. the interest of the larger public good, else
3. Right to Strike is a Fundamental Right. this would lead to sound pollution (loud
4. The government can impose restrictions speakers also disrupt people’s peace and
in certain areas declaring the assembly of concentration). Hence this ban by the district
five or more persons as unlawful. court does not violate fundamental right.
Which of the statements given above is/are All linguistic minorities can set up their
correct? own educational institutions to preserve
(a)  3 Only (b)  1 and 3 Only and develop their own culture. Thus the
(c)  1, 2 and 4 Only (d)  1, 2, 3 and 4 Karnataka Society has every right to run
Solution: (c) Kannada medium schools outside Karnataka.
Ordinarily, a person would be arrested after he A director makes a documentary film that
or she has reportedly committed some offence. criticises the policies of the government. In
However there are exceptions to this. this situation the freedom of expression is
Sometimes a person can be arrested being used.
simply out of an apprehension that he or she Q.244 Which of the following can NOT be forced
is likely to engage in unlawful activity and on Indian citizens?
imprisoned for some time without following 1. Singing national anthem
the above mentioned procedure. This is 2. Showing allegiance to all national sports
known as preventive detention. It means that teams
if the government feels that a person can be 3. Healthy habits like regulated diets,
a threat to law and order or to the peace and exercise and Yoga
security of the nation, it can detain or arrest Choose the correct answer using the codes
that person. below.
The government can impose restrictions (a)  1 only (b)  2 and 3 only
in certain areas declaring the assembly of (c)  3 only (d)  All the above
five or more persons as unlawful. Freedom Solution: (d)
to assemble too is to be exercised peacefully This issue was in news because of
and without arms. government planning to introduce Yoga in

Indian Polity Question Bank P.61

01-Indian Polity_Q1-412.indd 61 8/7/2018 7:39:04 PM


school curriculums; and some Indian cricket FRs can be suspended during the operation
fans being booked for sedition for supporting of a National Emergency except the rights
Pakistani cricket team. We enjoy Right to guaranteed by Articles 20 and 21. Further,
freedom of Speech, Expression, Movement, the six rights guaranteed by Article 19 can be
belief and so on. All of the activities, if suspended only when emergency is declared
forced, in some way or the other violate these on the grounds of war or external aggression
fundamental rights. A law that makes these (i.e., external emergency) and not on the
compulsory can be challenged in Supreme ground of armed rebellion (i.e., internal
Court or High courts. emergency).
Q.245 In India, all citizens irrespective of the state Their application can be restricted while
in which they are born or reside enjoy the martial law is in force in any area. Martial
same political and civil rights of citizenship law means ‘military rule’ imposed under
all over the country and no discrimination is abnormal circumstances to restore order
made between them excepting in few cases in (Article 34). It is different from the imposition
1. Tribal areas of national emergency.
2. State of J&K
3. Areas mentioned under Article 371 of the Definition of State
Constitution
Q.247 The term ‘State’ has been used in different
Choose the correct answer using the codes
provisions concerning the fundamental rights
below.
against which the citizens are protected.
(a)  1 and 2 only (b)  2 and 3 only
‘State’ can include which of the following?
(c)  1 and 3 only (d)  All of the above
1. A Public Sector Undertaking
Solution: (d)
2. District Planning Committee
In tribal areas, fifth and sixth schedule give
3. A Private party under a Public Private
them greater autonomy, more political rights
Partnership (PPP) contract with the
and a different political structure alongwith
government
regular special incentives from the Union and
Choose the correct answer using the codes
State governments. In J&K, acts like AFSPA,
below.
different constitution, special category status,
(a)  1 and 2 only (b)  2 and 3 only
Article 370 of the constitution, etc. create a
(c)  1 and 3 only (d)  All of the above
difference in the status of citizens.
Solution: (d)
Article 371 contains special provisions
Article 12 has defined the term for the
for Maharashtra and Gujarat. Then follow
purposes of Part III. According to it, the State
a stream of special provisions, including
includes the following:
Articles 371A (for Nagaland), 371B (Assam),
1. Government and Parliament of India, that
371C (Manipur), 371D and E (Andhra
is, executive and legislative organs of the
Pradesh), 371F (Sikkim), 371G (Mizoram),
Union government.
371H (Arunachal Pradesh), and 371(I) (Goa).
2. Government and legislature of states, that
Q.246 The application of fundamental rights to is, executive and legislative organs of
the citizens can be restricted in which of the state government.
following situations? 3. All local authorities, that is, municipalities,
1. President’s rule panchayats, district boards, improvement
2. Financial emergency trusts, etc.
3. Martial Law 4. All other authorities, that is, statutory
4. Internal emergency or non-statutory authorities like LIC,
Choose the correct answer using the codes ONGC, SAIL, etc.
below. Thus, State has been defined in a wider
(a)  1, 2 and 4 only (b)  3 and 4 only sense so as to include all its agencies. It is
(c)  3 only (d)  1 and 3 only the actions of these agencies that can be
Solution: (b)

 P.62 For Civil Services Preliminary Examination

01-Indian Polity_Q1-412.indd 62 8/7/2018 7:39:05 PM


challenged in the courts as violating the activities of the government. They can not be
Fundamental Rights. a part of the state government.
According to the Supreme Court, even
a private body or an agency working as Article 13
an instrument of the State falls within the
meaning of the ‘State’ under Article 12. Q.250 The Source of power of “judicial review” in
the constitution flows from
Q.248 The term ‘State’, when used in the context of
(a) The doctrine of separation of power
India, refers most appropriately to which of
between legislature and judiciary
the following?
(b) The authority of the Supreme Court and
(a) Government of India at all federal levels
High courts to issue writs (Article 32 and
(b) Territory of India including Union
226)
territories and other occupied areas
(c) Article 13 of the Constitution
(c) The Political institution that represents
(d) The Basic structure of the Constitution
sovereign people of India occupying a
Solution: (c)
definite territory
Article 13 declares that all laws that are
(d) Any institution that ultimately works
inconsistent with or in derogation of any
towards public welfare
of the fundamental rights shall be void. In
Solution: (c)
other words, it expressively provides for the
Learning: The term ‘State’ as such does not
doctrine of judicial review. This “power”
refer to state governments.
(not source of power, read carefully) has
• Rather when we use State, we are trying to
been conferred on the Supreme Court
distinguish it from ‘government’.
(Article 32) and the high courts (Article
• ‘Government’ is responsible for
226) that can declare a law unconstitutional
administering and enforcing laws. The
and invalid on the ground of contravention
government can change with elections.
of any of the Fundamental Rights. Thus,
• The State on the other hand refers to
not only a legislation but any of the above
a political institution that represents a
can be challenged in the courts as violating
sovereign people who occupy a definite
a Fundamental Right and hence, can be
territory. We can, thus, speak of the Indian
declared as void.
State, the Nepali State, etc.
• The Indian State has a democratic form
of government. The government (or the Right to Equality – Articles 14, 15, 16,
executive) is one part of the State. The State 17 and 18
refers to more than just the government
and cannot be used interchangeably Q.251 Which of the following come under the ambit
with it. of the Right to Equality in the Constitution?
1. Abolition of titles
Q.249 The term ‘state government’ does NOT refer to
2. Abolition of economic inequality
1. The State High courts
3. Protection of language and culture of the
2. State Human Rights Commission
Minorities
3. State legislature
Choose the correct matches from the codes
Choose the correct answer using the codes
given below:
below:
(a)  1 and 2 (b)  2 and 3
(a)  1 and 2 (b)  2 and 3
(c)  1 only (d)  1 and 3
(c)  1 and 3 (d)  All of the above
Solution: (c)
Solution: (d)
Abolition of economic inequality nowhere
Government is formed from the legislature.
mentioned in the constitution. At most in the
So the whole legislature can NOT be called
Directive Principles, it says that economic
the governemt. State judiciary and human
inequalities should be minimized.
rights commission keep a check on the

Indian Polity Question Bank P.63

01-Indian Polity_Q1-412.indd 63 8/7/2018 7:39:05 PM


Q.252 The Right to Equality provided in the (c) those sections of the society that are
Constitution covers which of the following considered advanced according to the
fundamental rights? Karpuri Thakur formula
1. All persons are equal before the law. (d) all sections of the upper castes of the
2. The State cannot discriminate arbitrarily society
in matters of employment. Solution: (b)
3. All citizens are entitled to move freely Creamy layer is a term used in Indian politics
anywhere in the Indian Territory. to refer to the relatively forward and better
Select the correct answer using the codes educated members of the Other Backward
below. Classes (OBCs) who are not eligible for
(a)  1 and 2 only (b)  1 only government-sponsored educational and
(c)  2 and 3 only (d)  1, 2 and 3 professional benefit programs. The term was
Solution: (a) introduced by the Sattanathan Commission in
Justification: Statement 3 comes under the 1971, which directed that the “creamy layer”
Right to Freedom (Article 19). should be excluded from the reservations
The Right to Equality (Articles 14-18) (quotas) of civil posts.
covers several fundamental rights. Q.255 Rule of law in Article 14 of the Constitution
• This right means that all persons shall implies that
be equally protected by the laws of the 1. The state cannot treat citizens differently
country. with respect to the same law.
• It also states that no citizen can be 2. The state cannot punish citizens except
discriminated against on the basis of their for violating the law.
religion, caste or sex. 3. The state cannot confer any special socio-
• Every person has access to all public economic privilege on any individual.
places including playgrounds, hotels, Choose the correct answer using the codes
shops etc. below.
• The practice of untouchability has also (a)  1 and 2 only (b)  2 and 3 only
been abolished by the Right to Equality. (c)  2 only (d)  1 only
• The State cannot discriminate against Solution: (c)
anyone in matters of employment. But The concept of ‘equality before law’ is an
there are exceptions to this ex: taking element of the concept of ‘Rule of Law’,
claims of SC and ST propounded by A.V. Dicey, the British jurist.
Q.253 If an Indian citizen is denied a public office His concept has the following three elements
because of his religion, which of the following or aspects:
Fundamental Rights is denied to him? 1. Absence of arbitrary power, that is, no
(a)  Right to Freedom man can be punished except for a breach
(b)  Right to Equality of law.
(c)  Right to Freedom of Religion 2. Equality before the law, that is, equal
(d)  Right against Exploitation subjection of all citizens (rich or poor,
Solution: (b) high or low, official or non-official) to the
Q.254 The expression ‘Creamy layer’ used in the ordinary law of the land administered by
judgement of the Supreme Court relating to the ordinary law courts.
the case regarding reservations refers to: 3. The primacy of the rights of the individual,
(a) those sections of the society which pay that is, the constitution is the result of the
income tax rights of the individual as defined and
(b) those sections of socially and enforced by the courts of law rather than
educationally backward classes of the the constitution being the source of the
society that are developed individual rights. The first and the second
elements are applicable to the Indian
System and not the third one. In the Indian

 P.64 For Civil Services Preliminary Examination

01-Indian Polity_Q1-412.indd 64 8/7/2018 7:39:05 PM


System, the constitution is the source of of them aim at establishing equality of legal
the individual rights. status, opportunity and justice.
The Supreme Court held that where
(1) Article 14 equals and unequals are treated differently,
Article 14 does not apply.
Q.256 ‘Equal Protection of Laws’ granted under Equality of opportunity is a different
Article 14 of the Constitution may imply fundamental right under Article 16.
(a) No person is above the law.
Q.258 Article 14 of the Constitution forbids class
(b) Equal treatment of equals under the law.
legislation. What is implied by this?
(c) Absence of arbitrary power with the state.
1. Laws that discriminate between people
(d)  The society operates on laws.
based on unreasonable classification stand
Solution: (b)
void.
Justification: Article 14 has two parts:
2. Class legislation applies only to private
equality before law and equal protection of
individuals.
laws. The first concept basically means law is
Which of the above is/are correct?
for everyone whoever it might be.
(a)  1 only (b)  2 only
The second concept means law will apply
(c)  Both 1 and 2 (d)  None
equally to equally situated people. For e.g.
Solution: (a)
if an adult receives a punishment of 3 years
Justification: Statement 2: Article 14 says
imprisonment, another adult for the same
that the State shall not deny to any person
crime and same circumstances should also
equality before the law or the equal protection
get 3 years of imprisonment.
of the laws within the territory of India.
Q.257 Article 14 of the constitution says that the This provision confers rights on all persons
State shall not deny to any person ―equality whether citizens or foreigners.
before the law|| or the ―equal protection of Moreover, the word ‘person’ includes
the laws|| within the territory of India. Which legal persons, viz., statutory corporations,
of the following would come under ―equal companies, registered societies or any other
protection of the laws||? type of legal person. So, 2 is wrong.
1. The equal subjection of all persons to the Statement 1: The Supreme Court held
ordinary law of the land administered by that where equals and unequals are treated
ordinary law courts. differently, Article 14 does not apply.
2. Similar application of the same laws to all While Article 14 forbids class legislation, it
persons who are similarly situated permits reasonable classification of persons,
3. Equality of opportunity to all under the subjects and transactions by the law. But the
laws made by the Parliament classification should not be arbitrary, artificial
Choose the correct answer using the codes or evasive. Rather, it should be based on
below. an intelligible differential and substantial
(a)  2 only (b)  2 and 3 only distinction.
(c)  1 and 3 only (d)  1 and 2 only For e.g. a special law can be made for
Solution: (a) transgender as this classification is reasonable
The term connotes: and just, but classification made solely on
1. the equality of treatment under equal grounds of financial or economic status may
circumstances, both in the privileges be challenged in the courts for being based in
conferred and liabilities imposed by the laws unreasonable classification.
2. the similar application of the same laws to
all persons who are similarly situated
3. the like should be treated alike without
(2) Articles 15 and 16
any discrimination. Q.259 Which of the following is implied by the
Thus, the former is a negative concept while Right to Equality given under the Indian
the latter is a positive concept. However, both Constitution?

Indian Polity Question Bank P.65

01-Indian Polity_Q1-412.indd 65 8/7/2018 7:39:05 PM


1. All citizens will be treated alike by the state (a)  1 and 2 (b)  2 and 3
irrespective of their social circumstances. (c)  1 and 3 (d)  All of the above
2. There will be no unreasonable Solution: (d)
discrimination for citizens in participation Article 15, 16 and 17 abolish untouchability;
for public offices. provide for equal opportunity in matters
3. Same legal provisions apply on all citizens of public employment; and prohibit
irrespective of their social or political discrimination on grounds of caste, religion,
stature. sex or place of birth respectively.
Select the correct answer using the codes All three stand in consonance with the
below. above provisions.
(a)  1 only (b)  2 and 3 only Q.261 “Nothing in this article shall prevent the
(c)  1 and 2 only (d)  2 only State from making any provision for the
Solution: (d) reservation of appointments or posts in favour
Explanation: All citizens are not treated of any backward class of citizens which, in
alike by the state. For e.g. disabled and the opinion of the State, is not adequately
transgender are provided reservation in represented in the services under the State.”
public jobs and social security. This is much This is a provision found in
greater than that provided to normal public. (a) Directive Principles of State Policy
There are numerous examples of positive (b)  Fundamental Rights
discrimination (SCs, STs, women etc.). (c) Special Constitutional Rights under
Therefore, statement 1 is incorrect. Part VII
The state can discriminate reasonably (d) National Commission for Backward
when it comes to public employment. For e.g. Classes (NCBC) Act, 1993
a state government can prescribe domicile Solution: (b)
requirement for local jobs. But, unreasonable Learning: This is a provision given under
discrimination (e.g. on the basis of religion) Article 16 (4) of the Constitution.
is not allowed. So, statement 2 is correct. Article 16 provides for equality of
Same legal provisions do not apply to all opportunity for all citizens in matters relating
citizens. For e.g. women cannot be called to to employment or appointment to any office
police station for inquiry in night; men can be. under the State. It prohibits discrimination
Punishment for sexual abuse is more severe on grounds other than those mentioned in the
for a civil servant than for a normal citizen. article itself.
Take another example, civil servants cannot be The NCBC Act, 1993 established the
dismissed without giving them a chance to be National Commission for Backward Classes.
heard. The same is not true for all government The commission was the outcome of the
employees. So, statement 3 is incorrect. direction of the Supreme Court in the Mandal
Therefore, Right to equality does not case judgement.
mean complete equality. It simply means
absence of unreasonable discrimination.
(3) Abolition of untouchability –
Q.260 Consider the following provisions.
1. Reservation of SCs and STs in government
Article 17
jobs Q.262 ‘Abolition of Untouchability’ is a component
2. SC & ST (Prevention of Atrocities) Act of which of the following categories of
1989 Fundamental Rights under the constitution?
3. Employment of Manual Scavengers and (a)  Right against Exploitation
Construction of Dry Latrines (Prohibition) (b)  Right to Equality
Act. (c)  Right to Freedom
Which of these have been enacted in the (d)  Right to Religion
spirit of Article 15, 16 and 17 of the Indian Solution: (b)
constitution?

 P.66 For Civil Services Preliminary Examination

01-Indian Polity_Q1-412.indd 66 8/7/2018 7:39:05 PM


Learning: Right to equality is an important (1) Article 19
right provided for in Articles 14, 15, 16, 17
and 18 of the constitution. It is the principal Q.264 Which of the following statements violates
foundation of all other rights and liberties the Right to Freedom guaranteed by the
Article 17 of the constitution abolishes Constitution of India?
the practice of untouchability. Practice of 1. Indian citizens need permission to visit
untouchability is an offence and anyone doing some border areas of the country.
so is punishable by law under Protection of 2. Outsiders are not allowed to buy property
Civil Rights Act. in some areas to protect the interest of the
local population.
Right to Freedom Granted under 3. The government bans the publication of a
book that can go against the ruling party in
Articles 19 to 22 the next elections.
Select the correct answer using the codes
Q.263 Right to freedom granted under Articles 19 to
given below
22 cover which of these rights?
(a)  1 and 3 (b)  2 Only
1. Right to freedom of speech, movement
(c)  3 Only (d)  2 and 3
and expression
Solution: (c)
2. Protection against arrest and detention in
Every citizen has the right to freedom
certain cases
mentioned under Article 19. But one cannot
3. Prohibition of traffic in human beings
exercise his/her freedom in such amanner
4. Right to freedom against bonded labour
that it violates others’ right to freedom. Your
Select the correct answer using the codes
freedoms should not cause public nuisance or
below.
disorder.You are free to do everything which
(a)  1 and 2 only (b)  1, 2 and 4 only
injures no one else. Freedom is not unlimited
(c)  2, 3 and 4 only (d)  1, 2, 3 and 4
license to do what one wants. Accordingly, the
Solution: (a)
government can impose certain reasonable
Justification: It consists of
restrictions on our freedoms in the larger
1. Protection of six rights regarding freedom
interests of the society.
of: (Article 19).
All citizens of India enjoy the freedom
(i)  speech and expression
to move freely throughout the country.
(ii)  assembly
But under certain circumstances, due to
(iii) association
security reasons the freedom can restricted.
(iv) movement
For instance, entry in border areas or army
(v) residence
cantonment is restricted to ordinary citizens
(vi) profession
as it can be dangerous for the security of
2. Protection in respect of conviction for
India. Outsiders are not allowed to buy
offences (Article 20).
property in some areas to protect the interest
3. Protection of life and personal liberty
of the local population.
(Article 21).
According to article 370, outsiders are
4. Right to elementary education (Article
not allowed to buy property and settle
21A).
permanently in the State of Jammu &
5. Protection against arrest and detention in
Kashmir and also in tribal areas it is restricted
certain cases (Article 22).
inorder to protect the ethnic and cultural
6. Right against exploitation (Articles 23–
identity of the tribal people.
24) cover
The government banning publication
7. Prohibition of traffic in human beings and
of book motivated by political reasons
forced labour (Article 23).
is a violation of freedom of expression
8. Prohibition of employment of children in
(Article 19(1)).
factories, etc. (Article 24).

Indian Polity Question Bank P.67

01-Indian Polity_Q1-412.indd 67 8/7/2018 7:39:05 PM


(a) Article 19(1)(a)—Freedom of Speech Q.266 Which statements is/are true?
1 The American Constitution does not
and Expression expressly mention the liberty of the press
Q.265 The Supreme Court held that the freedom of 2. Article 19(1) (a) of Indian Constitution
speech and expression includes does not expressly mention the liberty of
1. Right against tapping of telephonic the press.
conversation 3. it is settled law that the right to freedom of
2. Right against strike called by a political speech and expression in Article 19(1)(a)
party or organisation includes the liberty of the press.
3. Freedom of commercial advertisements Code
Select the correct answer using the codes (a)  1 and 2 (b)  1, 2 and 3
below. (c)  2 and 3 (d)  1 and 3
(a)  1 and 2 only (b)  2 and 3 only Solution: (c)
(c)  1 and 3 only (d)  1, 2 and 3 Q.267 As per the Supreme Court, which of the
Solution: (d) following are included in the Freedom of
Learning: Freedom of Speech and Expression Speech and expression?
It implies that every citizen has the right 1. Freedom of silence
to express his views, opinions, belief and 2. Right to know about government
convictions freely by word of mouth, writing, activities.
printing, picturing or in any other manner. 3. Right to propagate one’s views as well as
The Supreme Court held that the freedom of views of others
speech and expression includes the following: Choose the correct answer using the codes
1. Right to propagate one’s views as well as below.
views of others. (a)  All of the above (b)  2 and 3 only
2. Freedom of the press. (c)  1 and 3 only (d)  1 and 2 only
3. Freedom of commercial advertisements. Solution: (a)
4. Right against tapping of telephonic
Q.268 The ban on pre-censorship of newspapers by
conversation.
the country in the country is based on the
5. Right to telecast, that is, government has
(a) Special provisions made under Article 19
no monopoly on electronic media.
of the constitution
6. Right against bundh called by a political
(b) Rule of law in the constitution of India
party or organisation.
(c) Explicit freedom given to media or any
7. Right to know about government
people’s forum in the constitution
activities.
(d) A ruling of Supreme Court which
8. Freedom of silence.
interpreted Article 19 of the constitution
9. Right against imposition of pre-censorship
in a wider sense
on a newspaper.
Solution: (d)
10. Right to demonstration or picketing but
not right to strike. Q.269 Article 19(a) guarantees the Right to
The State can impose reasonable restrictions freedom of speech and expression. As per the
on the exercise of the freedom of speech and interpretations of the Supreme Court, which of
expression on the grounds of the following rights are covered in its ambit?
1. sovereignty and integrity of India, 1. Right to protests and strike.
2. security of the state 2. Right to be offended.
3. friendly relations with foreign states 3. Right to demonstration or picketing.
4. public order, Select the correct answer using the codes
5. decency or morality below
6. contempt of court (a)  1 and 3 (b)  1 and 2
7. defamation (c)  2 and 3 (d)  Only 3
8. incitement to an offence. Solution: (d)
We have a right to protest, not to strike.

 P.68 For Civil Services Preliminary Examination

01-Indian Polity_Q1-412.indd 68 8/7/2018 7:39:05 PM


Q.270 Right to be offended is an opinion of leading (b) The legislation is valid by virtue of
intellectuals in the country to be included in Article 31 B
the freedom of speech – in the case of the book (c) The legislation is invalid as it imposes
by Wendy Doniger – Penguin Publications. unreasonable restrictions under Article
Which of the following hypothetical cases 19(2) of the Constitution
would qualify for a restriction on the right to (d) The legislation is valid as the Press is
freedom of speech and expression of Mr. A? not a citizen under Articles 19 of the
1. Mr. A is defaming Mr. B publicly and Mr. Constitution
B moves the court of law. Solution: (c)
2. Mr. A’s views on religious matters have Q.273 Censorship of the press:
the potential of instigating a local riot. (a) is prohibited by the Constitution
3. Country X has very cordial relations (b) has to be judged by the test of
with India. Mr. A’s publicly stated views reasonableness
on country X have the potential to affect (c) is a restriction on the freedom of the
the relations between these two countries press mentioned in Article 19
negatively. (d) is specified in Article 31 of the
Select the correct answer using the codes Constitution
below Solution: (b)
(a)  2 and 3 (b)  1 and 2 The reasonable restrictions were listed in
(c)  All of the above (d)  None of the above 19(2)
Solution: (c)
Q.274 Which of the following can impose
The criteria of restricting freedom of speech
reasonable restrictions on the Fundamental
are: defamation; relations with foreign states;
Rights of the Indian citizens?
public order etc. All thus qualify under it.
(a)  Supreme Court
M.S. Dhoni had moved to court for
(b)  Parliament
restricting the false reports against him that
(c) President on the advice of the Council of
were defaming him.
Ministers
Q.271 Apart from sovereignty and integrity of (d) None of these; the restrictions have
India, the state can restrict fundamental rights already been included in the Constitution
on which of the following grounds? Solution: (b)
1. Public order, decency or morality Laws in Fundamental rights are made by the
2. Incitement to an offence parliament.
3. To maintain cordial relations with foreign
nations. (b) Article 19(1)(b) – Right to Assemble
4. Contempt of court.
Choose the correct answer using the codes Peacefully Without Arms
below: Q.275 Section 141 and 144 of Criminal Procedure
(a)  1, 2 and 3 (b)  1, 3 and 4 Code (1973) that are often heard in news
(c)  1, 2 and 4 (d)  All of the above deal with
Solution: (d) 1. Reasonable restrictions on freedom of
Q.272 Suppose a legislation was passed by the assembly
Parliament imposing certain restrictions on 2. Powers of civilian administration to
newspapers. These included page ceiling, impose martial law
price and advertisements. The legislation Which of the above is/are correct?
is included in the Ninth Schedule to the (a)  1 only (b)  2 only
Constitution of India. In this context, which (c)  Both 1 and 2 (d)  None
one among the following statements is Solution: (a)
correct? Justification: Under Sec 144, a magistrate
(a) The legislation is invalid as it violates the can restrain an assembly, meeting or
freedom of Press procession if there is a risk of obstruction,

Indian Polity Question Bank P.69

01-Indian Polity_Q1-412.indd 69 8/7/2018 7:39:05 PM


annoyance or danger to human life, health what he is not legally bound to do, or to
or safety or a disturbance of the public omit to do what he is legally entitled to do.
tranquillity or a riot or any affray. The District magistrate can order the
Under Section 141 of the Indian Penal assembly to disperse.
Code, as assembly of five or more persons Explanation: An assembly which was
becomes unlawful if the object is (a) to resist not unlawful when it assembled, may
the execution of any law or legal process; subsequently become an unlawful assembly.
(b) to forcibly occupy the property of some
person; (c) to commit any mischief or criminal (2) Article 20 – Protection in Respect
trespass; (d) to force some person to do an
illegal act; and (e) to threaten the government
of Conviction of Offences
or its officials on exercising lawful powers. Q.277 Which one of the following are Fundamental
Q.276 As per Section 141 of the Indian Penal Code Rights?
(IPC), an assembly of five or more persons 1. The Right to be presented before a
is designated an “unlawful assembly”, if the magistrate within 24 hours of arrest.
common object of the persons composing 2. The Right not to be ill-treated or tortured
that assembly is during arrest or in custody.
1. To protest non-violently against the 3. Confessions made in police custody cannot
government or its policies be used as evidence against the accused.
2. To resist the execution of any law 4. A boy under 15 years of age and women
3. To forcefully take the possession of any cannot be called to the police station only
property for questioning.
4. To gather large masses of people in open Answer the questions using the codes given
public spaces below .
Select the correct answer using the codes below. (a)  1 and 2 Only (b)  2 and 3 Only
(a)  1 and 2 only (b)  2 and 3 only (c)  1,2 and 4 Only (d)  1,2,3 and 4
(c)  1 and 4 only (d)  1, 2, 3 and 4 Solution: (d)
Solution: (b) All are fundamental rights
Justification: The provisions (as per Bare
Act) are: (3) Article 21 – Right to Life
a. To overawe by criminal force, or show
Q.278 Right to fair trial, Right to hearing, Right
of criminal force, [the Central or any
to speedy trial and Right to free legal aid
State Government or Parliament or the
are provided to citizens under which of the
Legislature of any State], or any public
following fundamental rights?
servant in the exercise of the lawful power
(a) Equality before law and equal protection
of such public servant; or
of laws
b. To resist the execution of any law, or of
(b)  Right to Life and Liberty
any legal process; or
(c) Right against arbitrary arrest and
c. To commit any mischief or criminal
detention
trespass, or other offence; or
(d)  Right against exploitation
d. By means of criminal force, or show of
Solution: (b)
criminal force, to any person, to take or
The Supreme Court has reaffirmed its
obtain possession of any property, or to
judgement in the Maneka Gandhi v. UOI,
deprive any person of the enjoyment of
1978 SCR (2) 621, in the subsequent cases.
a right of way, or of the use of water or
It has declared the following rights as part of
other incorporeal right of which he is in
Article 21:
possession or enjoyment, or to enforce
• Right to live with human dignity.
any right or supposed right; or
• Right to decent environment including
e. By means of criminal force, or show of
pollution free water and air and protection
criminal force, to compel any person to do
against

 P.70 For Civil Services Preliminary Examination

01-Indian Polity_Q1-412.indd 70 8/7/2018 7:39:05 PM


• Hazardous industries. held that the right to safe drinking water is a
• Right to livelihood. Fundamental Right.
• Right to privacy. Q.280 Which of the following rights come under
• Right to shelter. the Right to Life and Liberty under Article
• Right to health. 21 of the Indian Constitution as per the
• Right to free education up to 14 years of interpretations of the Supreme Court?
age. 1. Right to livelihood
• Right to free legal aid. 2. Right to shelter
• Right against solitary confinement. 3. Eradication of preventable diseases like
• Right to speedy trial. polio.
• Right against handcuffing. 4. Right to speedy trial
• Right against inhuman treatment. 5. Right to travel abroad
• Right against delayed execution. 6. Right of women to be treated with decency
• Right to travel abroad. and dignity
• Right against bonded labour. Select the correct answer using the codes
• Right against custodial harassment. below
• Right to emergency medical aid. (a)  4 and 6 (b)  All of the above
• Right to timely medical treatment in (c)  1, 2, 3 and 4 (d)  1,2, 3 and 6
government hospital. Solution: (b)
• Right not to be driven out of a state.
Q.281 The Fundamental Right to Clean Environment
• Right to fair trial.
has been established by
• Right of prisoner to have necessities of
(a) An informal understanding between the
life.
legislature and executive
• Right of women to be treated with decency
(b) The Supreme Court in the case Subhas
and dignity.
• Right against public hanging. Kumar v. State of Bihar, (1991) 1 SCC 598
• Right to hearing. (c) A constitutional amendment of Article 21
• Right to information. (d) An executive order by the Government
• Right to reputation. of India
Solution: (b)
Q.279 Which of the following rights have been Learning: Article 21 of the Constitution is
recognized under the Right to Life under a fundamental right which reads as follows:
Article 21 by the courts? “No person shall be deprived of his life
1. Right to safe drinking water or personal liberty except according to
2. Right to food procedure established by law.”
3. Right to a speedy trial Though this Article does not explicitly
Choose the correct answer using the codes mention the environment, the Supreme Court
below: and the various High Courts of the country
(a)  1 and 2 (b)  2 and 3 have given a wider interpretation to the word
(c)  1 and 3 (d)  All of the above “life” in this Article.
Solution: (d) The Supreme Court in Subhas Kumar v.
The Constitution of India recognises the State of Bihar, AIR 1991 SC 420, held that
right to water as being a part of the Right right to environment is a fundamental right
to Life under Article 21. This means that it of every citizen of India and is included in the
is the right of every person, whether rich or “right to life” guaranteed under Article 21 of
poor, to have sufficient amounts of water to the Constitution of India.
fulfil his/her daily needs at a price that he/ A Public Interest Litigation (PIL) is
she can afford. In other words, there should maintainable in the High Court or Supreme
be universal access to water. There have Court at the instance of affected persons
been several court cases in which both the or even by a group of social workers or
High Courts and the Supreme Court have journalists for prevention of pollution.

Indian Polity Question Bank P.71

01-Indian Polity_Q1-412.indd 71 8/7/2018 7:39:05 PM


(4) Article 22 – Protection against The expression ‘traffic in human beings’
include
Arrest and Detention 1. selling and buying of men, women and
Q.282 Consider the following statements. children like goods
1. Right to Equality before law is not 2. immoral traffic in women and children,
effective in India as citizens do not  have including prostitution devadasis
a Fundamental Right to be defended by a 3. slavery
lawyer. To punish these acts, the Parliament has made
2. It is the constitutional duty of the state the Immoral Traffic (Prevention) Act, 1956.
to provide a lawyer to any citizen who is The term ‘begar’ means compulsory work
unable to engage one due to poverty or without remuneration. It was a peculiar Indian
other disability. system under which the local zamindars
Which of the above is/are correct? sometimes used to force their tenants to
(a)  1 only (b)  2 only render services without any payment.
(c)  Both 1 and 2 (d)  None In addition to begar, the Article 23 prohibits
Solution: (b) other similar forms of forced labour like
Justification: Statement 1: According to ‘bonded labour’. The term ‘forced labour’
Article 22 of the Constitution, every person means compelling a person to work against
has a Fundamental Right to be defended by his will. The word ‘force’ includes not
a lawyer. only physical or legal force but also force
Statement 2: Article 39A of the Constitution arising from the compulsion of economic
places a duty upon the State to provide circumstances, that is, working for less
a lawyer to any citizen who is unable than the minimum wage. In this regard, the
to engage one due to poverty or other Bonded Labour System (Abolition) Act,
disability. 1976; the Minimum Wages Act, 1948; the
National Legal Services Authority as Contract Labour Act, 1970 and the Equal
established to provide legal help to such Remuneration Act, 1976 were made
citizens.
We will be covering questions on such (2) Article 24 – Prohibition of Child
bodies in coming tests. Labour
Right Against Exploitation Q.284 With reference to child labour problem in
India, consider the following statements
1. At present there is complete ban on child
(1) Article 23 – Prohibition of Traffic labour under the age of 14 in all types
in Human Beings, Forced Labour of occupation except farm and domestic
Q.283 Article 23 of the Constitution prohibits traffic work after school hours
in human beings, forced labour and other 2. The Child Labour (Prohibition &
similar forms of forced labour. It is enforced Regulation) Act is being implemented by
by which of the following laws made by the the Ministry of Women and Child Welfare
Parliament? Which of the above statements is/are
1. Minimum Wages Act, 1948 INCORRECT?
2. Contract Labour Act, 1970 (a)  1 Only (b)  2 Only
3. Equal Remuneration Act, 1976 (c)  Both (d)  None
Choose the correct answer using the codes Solution: (c)
below. The scheme is being implemented by the
(a)  All of the above (b)  2 and 3 only Ministry of Labour and Employment First
(c)  1 and 3 only (d)  1 and 2 only statement is wrong as it is still a proposal put
Solution: (a) forward recently by the new government at
the centre.

 P.72 For Civil Services Preliminary Examination

01-Indian Polity_Q1-412.indd 72 8/7/2018 7:39:05 PM


Right to Freedom of Religion – 3. The government cannot take over the
administration of a religious shrine.
Article 25–28 Which of the following options make the
question statement correct and complete?
Q.285 Although India is a secular country, the state
(a)  1 and 2 (b)  2 and 3
can take some steps to ensure public order,
(c)  1 and 3 (d)  All of the above
morality and the like. Which of the following
Solution: (d)
can be done by the state constitutionally?
In India, everyone is free to choose a religion
1. Regulate a religious institution
and practice that religion. Freedom of religion
2. Throw open Hindu religious institutions
also includes the freedom of conscience. This
of a public character to all classes and
means that a person may choose any religion
sections of Hindus.
or may choose not to follow any religion.
3. Restrict a religious practice
Freedom of religion includes the freedom to
Choose the correct answer using the codes
profess, follow and propagate any religion.
given below.
Freedom of religion is subject to certain
(a)  1 and 2 only (b)  2 and 3 only
limitations. The government can impose
(c)  1 and 3 only (d)  All of the above
restrictions on the practice of freedom of
Solution: (d)
religion in order to protect public order,
An example of restricting a religious practice
morality and health. This means that the
would be the Supreme Court banning
freedom of religion is not an unlimited right.
Jallikattu in TN.
The government can interfere in religious
Article 25 covers not only religious
matters for rooting out certain social evils.
beliefs (doctrines) but also religious practices
For example in the past, the government
(rituals). Moreover, these rights are available
has taken steps banning practices like sati,
to all persons—citizens as well as non-
bigamy or human sacrifice. Such restrictions
citizens.
cannot be opposed in the name of interference
However, these rights are subject to public
in right to freedom of religion.
order, morality, health and other provisions
relating to fundamental rights. Further, the Q.287 Consider that you belong to the religion
State is permitted to: Zoroastrianism. You are outside a Hindu
(a) Regulate or restrict any economic, temple managed by the state and want to
financial, political or other secular enter. But somewhere at the entry you find
activity associated with religious this written ―Only Hindus are allowed. Now
practice; and consider the following courses of action.
(b) Provide for social welfare and reform or 1. Complain to the local police authority
throw open Hindu religious institutions about this discrimination and violation of
of a public character to all classes and your fundamental right of free movement
sections of Hindus. 2. Approach the state High court of this
violation of fundamental right
Q.286 Article 25 also contains two explanations
3. Approach the Supreme Court with a
1. wearing and carrying of kirpans is to be
Public Interest Litigation (PIL) on the
included in the profession of the Sikh
same issue
religion
Which of the above options are legally and
2. the Hindus, in this context, include Sikhs,
constitutionally appropriate?
Jains and Buddhists.
(a)  2 only
Freedom of religion as a fundamental right
(b)  3 only
does NOT mean that
(c)  All of the above
1. The government cannot intervene in
(d)  None of the above
religious affairs
Solution: (d)
2. The Government cannot ban a religious
This case is not a violation of fundamental
practice.
right. A hindu temple managed by the state

Indian Polity Question Bank P.73

01-Indian Polity_Q1-412.indd 73 8/7/2018 7:39:05 PM


is not a public place where free movement Which of the above is/are correct?
is unrestricted. It is a place which belongs (a)  1 only (b)  2 only
to a certain religious community. It has (c)  Both 1 and 2 (d)  None
all the constitutional rights to block the Solution: (c)
entry of people belonging to other religious Justification: Statement 1: This is why in
communities. state-run schools, religious texts are not
Hence, none of the above options would included in the curriculum. Moreover, the
be appropriate as this case is not a violation dress code and other symbolic elements are
of fundamental right. designed in such a way that they do not reflect
Q.288 Consider the following statements about any particular religious biases of such schools.
the Right to Freedom from taxation for Statement 2: All minorities, religious or
Promotion of religion. linguistic, can set up their own educational
1. The state cannot use public tax money for institutions. By doing so, they can preserve
the promotion of any religion. and develop their own culture. The
2. The state cannot impose a fee on any government will not, while granting aid to
religious activities. educational institutions, discriminate against
Which of the above is/are true? any educational institution on the basis that
(a)  1 only (b)  2 only it is under the management of minority
(c)  Both 1 and 2 (d)  None community.
Solution: (d) But, while granting aid to an educational
Article 27 expresses that no person shall be institution, the government will not
compelled to pay any taxes for the promotion discriminate against any such institution on
or maintenance of any particular religion or the basis that it is under the management of
religious denomination. In other words, the minority community.
State should not spend the public money
collected by way of tax for the promotion or Right of Minorities – Articles 29 and 30
maintenance of any particular religion. This
Q.290 If a group of minorities is denied permission
provision prohibits the State from favouring,
for opening a minority-based educational
patronising and supporting one religion over
institution in Kerala, it violates which of the
the other. This means that the taxes can be
following Fundamental Rights?
used for the promotion or maintenance of
(a)  Right to Life
all religions.
(b)  Rights given under Articles 29-30
This provision prohibits only levy of a tax
(c)  Right to Freedom of movement
and not a fee. This is because the purpose of
(d)  Right against exploitation
a fee is to control secular administration of
Solution: (b)
religious institutions and not to promote or
Learning: Option A is incorrect, because the
maintain religion. Thus, a fee can be levied
violation is specific to Articles 29-30 that aim
on pilgrims to provide them some special
at safeguarding the cultural and educational
service or safety measures. Similarly, a fee
rights of minorities.
can be levied on religious endowments for
For e.g. tribals can preserve their cultural
meeting the regulation expenditure.
heritage against the dominance of the
Q.289 Consider the following statements. majority mainstream culture.
1. The institutions run by the state will not
Q.291 Cultural and educational rights given under
give any religious education to ensure the
Articles 29 and 30 of the Indian constitution
neutrality of State in matters of religion.
imply
2. The government will not discriminate
1. Minorities have the right to conserve their
between educational institutions, when
language and script.
granting aid, on the basis of their
2. Minorities have a fundamental right
management being under particular
to receive financial support from the
minority communities.
government to preserve their traditions.

 P.74 For Civil Services Preliminary Examination

01-Indian Polity_Q1-412.indd 74 8/7/2018 7:39:05 PM


Which of the above is/are correct? provisions of UP Arms Act in 1948, and was
(a)  1 only (b)  2 only decided by the Allahabad High Court in 1952.
(c)  Both 1 and 2 (d)  None Dhyan Singh had been convicted under the
Solution: (a) Arms Act for wearing and carrying more than
Justification: Statement 1: In the recent case one kirpan without licence. When the Appeal
of Niyamgiri Hills in Odisha, the SC ordered came to the High Court, the Constitution of
the Ministry of Environment & Forests India had come into operation, giving right
(MoEF) to reconsider its decision to grant to the Sikhs to wear and carry kirpans under
bauxite mining license to a mining company Article 25. The question to be decided before
as it was affecting the local tribe’s religious the High Court was: Whether a Sikh can wear
and cultural heritage. So, 1 is correct. and carry more than one kirpan as the plural
Statement 2: Generally the government word kirpans has been used in Explanation
does support minorities. For e.g. Jains being I of the Article 25. After referring to the
a minority now receive land concessions religious literature of the Sikhs, the High
from the government. However, obtaining Court had held that a Sikh is entitled to wear
such support is not their fundamental right. and carry only one kirpan, as Guru Gobind
So, 2 is wrong. Singh had ordained the Sikhs to wear always
Q.292 With regard to rights of minorities provided five K’s, i.e., kesh, kangha, kirpan, kachh and
in India Constitution, consider the following kara, which indicates only one of these signs.
statements: Right to minorities includes both linguistic
1. Right to minorities includes both linguistic and cultural minorities.
and cultural minorities. Q.293 Which of the following is a correct
2. All minorities can set up their own interpretation of the Cultural and Educational
educational institutions. Rights?
3. Members of the Sikh community have the (a) Exposing the children of the minority
right to assemble at a public place with group to their belief and culture is the
their kirpans. Government’s responsibility
4. Minority educational institutions have (b) Children belonging to the minority group
the right to impart education in their own that has opened educational institution
language. only can study there
Select the correct answer using the codes (c) Reservations can be made in the schools
below: opened by linguistic and religious
(a)  2 Only (b)  1, and 2 Only minorities for their children
(c)  1, 2, and 4 Only (d)  1, 2, 3 and 4 Only (d) It can demanded by the minorities that
Solution: (d) their children must not study in any
Our Constitution believes that diversity is our educational institution except those
strength. Therefore, one of the fundamental managed by their own community.
rights is the right of the minorities to Solution: (c)
maintain their culture. This minority status is Q.294 Who have a fundamental right to set up
not dependent only upon religion. Linguistic their own educational institutions in order to
and cultural minorities are also included in preserve and develop their own culture?
this provision. All minorities, religious or (a)  Linguistic minorities
linguistic, can set up their own educational (b)  Religious minorities
institutions. By doing so, they can preserve (c) All minorities as defined by the Central
and develop their own culture. Educational and State governments
institutions set up by a minority are open all (d) Only tribals belonging to the fifth and
and not only to those children who belong to sixth schedule areas
that minority. Solution: (c)
The first case: “Rex versus Dhyan Singh” Explanation & Learning: The expression
(AIR 1952 Allahabad, 53) arose under the minority under Article 29-30 (Cultural and

Indian Polity Question Bank P.75

01-Indian Polity_Q1-412.indd 75 8/7/2018 7:39:05 PM


Education rights) covers all minorities, 1. To protect the rich cultural heritage
linguistic and religious. 2. To ensure social justice
The constitution does not define the 3. To prevent majority authoritarianism
word minority. It is left for the Central and on issues related to political power and
state governments to decide this. For e.g. access to resources
recently Jain community was recognized as Choose the correct answer using the codes
a minority. below:
They will have rights to set up their (a)  1 and 2 (b)  2 and 3
own educational institutions, where they (c)  1 and 3 (d)  All of the above
can regulate any admission of outside Solution: (d)
communitis. In recognition of this, the constitution
Tribals also fall under minority. But, the provides religious and cultural rights to the
option (c) is more wide and appropriate than minorities and provides for provisions which
option (d) discriminate positively in favour of these
Q.295 On what grounds do the National and State minorities. For e.g. minority scholarships,
government have control over religious govt. non-interference in minority run
education imparted in minority institutions? institutions etc.
(a) On grounds of protecting the culture of Q.298 Consider the following statements.
minorities 1. Linguistic minorities in India are
(b) On grounds of promoting scientific determined on a state-wise basis.
temper in society 2. Religious Minorities in India must be
(c) On grounds of national unity and integrity determined on national basis only.
(d) The government cannot control Which of the above is/are correct?
religious education imparted in minority (a)  1 only (b)  2 only
institutions. (c)  Both 1 and 2 (d)  None
Solution: (d) Solution: (a)
Q.296 The Indian constitution protects the rights of Justification: A linguistic minority is a group
minorities because of people whose mother tongue is different
1. In an electoral democracy, the will of the from that of the majority in the state or part
majority prevails over the minority. of a state. Thus, the linguistic minorities
2. All minority classes are socioeconomically are determined on a statewise basis. The
vulnerable India. same is applicable for religious minorities
Which of the above is/are true? as well.
(a)  1 only (b)  2 only • In the 2002 judgment, in T.M.A. Pai
(c)  Both 1 and 2 (d)  None Foundation & Others v. the State of
Solution: (a) Karnataka, the Supreme Court deliberated
Article 29 provides that any section of the on the various contentions that the Centre,
citizens residing in any part of India having State, or a particular region within a State
a distinct language, script or culture of its may be considered as the basic unit for
own, shall have the right to conserve the protection of the right of minorities.
same. Further, no citizen shall be denied • The Court has set out the principle that
admission into any educational institution minority status should be determined in
maintained by the State or receiving aid out relation to the population of the State and
of State funds on grounds only of religion, not to India as a whole.
race, caste, or language. Their rights are to be • It ruled that as the reorganisation of
protected because their future in a democracy the States in India had been effected
is vulnerable to the will of the majority. on linguistic lines, for the purpose of
determining a minority, the unit would be
Q.297 Safeguarding the rights of minorities in
the State and not the whole of India.
India is important for which of the following
reasons?

 P.76 For Civil Services Preliminary Examination

01-Indian Polity_Q1-412.indd 76 8/7/2018 7:39:06 PM


• Thus, religious and linguistic minorities, right of the citizen. According to Article 32,
who have been placed on a par in Article Indian citizens can stand up and fight for their
30, have to be considered in terms of the fundamental rights if they are breached.
State concerned. Q.301 Consider the following statements about the
Q.299 Consider a situation where a minority tribal Right to Constitutional Remedies.
group worships the local forests and treats 1. Any court in India can be moved for
them as their god. A private company wishes enforcing this right.
to acquire the same patch of forest. Imagining 2. The right to move to courts can never be
you are one of the members of the tribal suspended under this right.
group, which of the following constitutional 3. A citizen cannot go directly to the
articles can you cite that will stop the private Supreme Court to get this right enforced.
company from acquiring the patch of sacred He has to go by way of appeal to higher
forest? courts.
(a)  Articles 14-16 Choose the correct answer using the codes
(b)  Articles 25-28 given below.
(c)  Article 29-30 (a)  1 and 2 only (b)  2 and 3 only
(d) Article 39 (b) & (c) – Directive Principles (c)  1 and 3 only (d)  None of the above
of State Policy Solution: (d)
Solution: (c) The Supreme Court has ruled that Article 32
Articles 14-18 talk about non-discrimination is a basic feature of the Constitution. Hence,
and social and economic justice it cannot be abridged or taken away even by
Articles 25-28 give religious rights. way of an amendment to the Constitution. It
Articles 29-30 give the minorities a right to contains the following four provisions:
safeguard their cultural heritage- the forest in (a) The right to move the Supreme Court
this case. The Odisha Niyamgiri tribals also by appropriate proceedings for the
saved their sacred hills using this provision enforcement of the Fundamental Rights
of the constitution. is guaranteed.
(b) The Supreme Court shall have power to
Article 32 – Right to Constitutional issue directions or orders or writs for the
enforcement of any of the fundamental
Remedies rights. The writs issued may include
Q.300 The Right to constitutional remedies is a habeas corpus, mandamus, prohibition,
(a)  Legal Right certiorari and quo-warranto.
(b)  Right by convention (c) Parliament can empower any other court
(c)  Fundamental right to issue directions, orders and writs of all
(d)  Moral right kinds.
Solution: (c) However, this can be done without
Learning: It is referred to as the ‘conscience’ prejudice to the above powers conferred
of the constitution because without this right on the Supreme Court. Any other court
all rights stand without merit. here does not include high courts because
In case of any one of the fundamental Article 226 has already conferred these
rights being deprived or denied to the powers on the high courts.
resident of the country, the individual or the (d) The right to move the Supreme Court shall
party has the right to present their case in a not be suspended except as otherwise
court. In this case, the court has the flexibility provided for by the Constitution. Thus
to assign writs to the public in the form of the Constitution provides that the
habeas corpus, mandamus, prohibition, quo President can suspend the right to move
warranto and certiorari. any court for the enforcement of the
In the case of a national emergency, the fundamental rights during a national
government has the flexibility to append the emergency (Article 359).

Indian Polity Question Bank P.77

01-Indian Polity_Q1-412.indd 77 8/7/2018 7:39:06 PM


It is thus clear that the Supreme Court Only the High Courts have the right to
has been constituted as the defender and issue orders for the enforcement of both
guarantor of the fundamental rights of the Fundamental Rights and Legal rights,
citizens. It has been vested with the ‘original’ Supreme Court can issue orders or writs only
and ‘wide’ powers for that purpose. Original, for the enforcement of the Fundamental right
because an aggrieved citizen can directly go and not other rights.
to the Supreme Court, not necessarily by way Q.303 Consider the following statements regarding
of appeal. Wide, because its power is not the Right to Constitutional remedies under
restricted to issuing of orders or directions Article 32 of the Constitution:
but also writs of all kinds. 1. Unconstitutional executive actions can be
Q.302 Consider the following Statements: questioned under Article 32.
1. In India, citizens have the right to seek 2. Even statutory rights can be enforced
enforcement of the Fundamental Rights under Article 32.
2. The Right to Constitutional Remedies is Which of these is/are correct?
not a Fundamental Right. (a)  Only 1 (b)  Only 2
3. In case of violation of a Fundamental (c)  Both (d)  None of the above
Right the citizen can directly approach Solution: (d)
Supreme Court or High Court. Article 32 can be resorted only when the
4. Only the High Courts have the right to fundamental rights are violated.
issue orders for the enforcement of both Unconstitutional executive actions can
Fundamental Rights and Legal rights. be questioned under Article 32 only if they
5. The Courts can also enforce the violate fundamental rights.
Fundamental Rights against private Statutory rights can not be enforced under
individuals and bodies. Article 32. Ordinary legal route needs to be
Which of the statements is/are correct? adopted if that is the case.
(a)  1 and 3 (b)  1, 3 and 5 Q.304 According to the “Right to Constitutional
(c)  1, 3, 4 and 5 (d)  1, 2, 3 and 5 Remedies”
Solution: (c) 1. Aggrieved citizens can move the court in
The Fundamental Rights in the constitution case of violation of fundamental rights.
are important because they are enforceable. 2. The Central Government can move the
We have a right to seek the enforcement of Court if the State government refuses to
the above mentioned rights. This is called the follow its orders.
Right to Constitutional Remedies. This itself Which of the above is/are correct?
is a Fundamental Right. This right makes (a)  1 only (b)  2 only
other rights effective. (c)  Both 1 and 2 (d)  None
It is possible that sometimes our rights Solution: (a)
may be violated by fellow citizens, private Justification: Statement 1: The court issues
bodies or by the government. When any of a writ to address the violation of fundamental
our rights are violated we can seek remedy rights. Only the Supreme Court and High
through courts. Courts are authorized to issue writs.
If it is a Fundamental Right we can We will be covering more on this topic in
directly approach the Supreme Court or the coming tests.
High Court of a state. Courts also enforce Statement 2: The government doesn’t
the Fundamental Rights against private approach the court in such cases. It can
individuals and bodies. impose President’s rule in the State if the
The Supreme Court and High Courts a State continuously denies following
have the power to issue directions, orders or Centre’s executive directions and subverts
writs for the enforcement of the Fundamental the constitutional scheme of division
Rights. They can also award compensation to of powers.
the victims and punishment to the violators.

 P.78 For Civil Services Preliminary Examination

01-Indian Polity_Q1-412.indd 78 8/7/2018 7:39:06 PM


Q.305 If there was no ‘Right to constitutional 1. Right to education.
remedies’ in the Constitution, which of the 2. Right to a pollution free environment
following would follow? 3. Right to safeguard cultural heritage and
(a) A citizen would not be able to move religious beliefs of minorities
court at all. Choose the correct answer using the codes
(b) There would be no rule of law in the below:
country. (a)  1 and 2 (b)  2 and 3
(c) Separation of powers between legislature (c)  1 and 3 (d)  All of the above
and executive would be completely Solution: (d)
diluted. The violation of fundamental rights is the
(d) A democratic government may turn basis for moving to the supreme court
authoritarian. under Article 32. Here all the options are
Solution: (d) fundamental rights provided under the
Provision: Dr. Ambedkar considered the constitution.
right to constitutional remedies as ‘heart and Q.307 Which of the following courts in India
soul of the constitution’. It is so because this can issues writs for the enforcement of
right gives a citizen the right to approach a fundamental rights?
High Court or the Supreme Court to get any 1. Supreme Court
of the fundamental rights restored in case of 2. High Court
their violation. 3. Specially constituted courts
The Supreme Court and the High Courts Choose the correct answer using the codes
can issue orders and give directives to the below:
government for the enforcement of rights. (a)  1 and 2 (b)  2 and 3
The courts can issue various special orders (c)  1 and 3 (d)  All of the above
known as writs. Solution: (a)
Justification: Option A: A citizen will not Right to constitutional remedies is the
be able to move court only for getting his means through which this is to be achieved.
constitutional rights enforced. He can move Dr. Ambedkar considered the right to
courts for other purposes such as resolving a constitutional remedies as ‘heart and soul of
legal dispute. So, A is wrong. the constitution’. It is so because this right
Option B: Rule of law emanates from gives a citizen the right to approach a High
Article 14 which essentially considers Court or the Supreme Court to get any of the
everyone equal before law. Even if the fundamental rights restored in case of their
constitutional remedies cease to exist, rule of violation. The Supreme Court and the High
law would continue to operate. Citizens may Courts can issue orders and give directives to
still be able to enjoy fundamental rights, but the government for the enforcement of rights.
they may not be able to complaint against its Any other court, if specified by the
infringement. Moreover, all other legal rights Parliament can issue writs
and civil, criminal laws will be operational.
Q.308 Apart from Supreme Court, writs to enforce
So, B is wrong.
fundamental rights can be issued by
Option C: It is an absurd statement.
1. District Courts
Option D: If citizens cannot get their
2. National Police Commission
fundamental rights enforced, governments
3. Lok Adalats
may very well breach these rights, such as
Select the correct answer using the codes
censoring newspapers, imposing curfews and
below.
emergency like situations. So, D is correct.
(a)  1, 2 and 3 only (b)  1 and 3 only
Q.306 The Right to constitutional remedies is referred (c)  2 only (d)  None of the above
to as the “conscience‟ of the constitution. Solution: (d)
A citizen can move the Supreme Court for Explanation: The Parliament is authorized
enforcing which of these rights below? to extend the power to issue writs to any

Indian Polity Question Bank P.79

01-Indian Polity_Q1-412.indd 79 8/7/2018 7:39:06 PM


other court other than the SC and the HCs. Choose the correct answer using the codes
But it has not extended the same to any other below:
body in India. So, all statements are wrong. (a)  1 and 2 (b)  2 and 3
Lok adalat and District courts thus (c)  1 and 3 (d)  All of the above
only solve disputes and do not interfere Solution: (d)
pro-actively in matter of violations of Legislators can be issued Quo warranto for
fundamental rights. example, if a legislator holds some office
National Police Commissions are which makes him ineligible for become
constituted by government to reform the an MP.
police system in the country. So, statement 2 Executive can be issues mandamus or
is also wrong. Habeas corpus.
Q.309 Consider the following statements about writ Judiciary can be issues certiorari, or
jurisdictions of Supreme Court and High prohibition.
Courts: Q.312 Supreme Court is regarded as the guardian of
1. High court can issue writs even for Fundamental Rights. For the enforcement of
enforcing a statutory right. fundamental rights the Supreme Court issues
2. A High court cannot issue writs outside writs. Which of the following statements is/
the borders of the particular state. are correct?
3. For the High courts issuing writs is a 1. Quo warranto is issued when the court
discretionary power unlike the Supreme finds that a particular office holder is not
Court. doing legal duty and thereby is infringing
Which of these is/are correct? on the right of an individual.
(a)  1 and 2 (b)  2 and 3 2. Certiorari is issued by a higher court
(c)  1 and 3 (d)  All of the above when a lower court has considered a case
Solution: (c) going beyond its jurisdiction.
The writ jurisdiction of high courts is wider 3. Habeas corpus means that the court
than that of the Supreme Court as it can orders that the arrested persons should
enforce legal rights too. be presented before it. It can also order to
Q.310 The right to constitutional remedies in India set free an arrested person if the manner
is a vailable to : or grounds of arrest are not lawful or
(a)  citizens of India only satisfactory.
(b) all persons in case of infringement of any 4. Mandamus is issued when the court finds
fundamental right that a person is holding office but is not
(c) any person for enforcing any of the entitled to hold that office, it issues the
fundamental rights conferred on all writ of mandamus and restricts that person
persons from acting as an office holder.
(d)  an aggrieved individual alone 5. Prohibition means the court orders a lower
Solution: (b) court or another authority to transfer a
matter pending before it to the higher
authority or court.
Writs Select the correct answer using the codes
Q.311 Consider the following statements about below:
writs issued for enforcing fundamental (a)  3 Only (b)  2, 3 and 4 Only
rights. (c)  1, 3 and 4 Only (d)  1, 2, 3, 4 and 5
1. It can be issued by the Judiciary to the Solution: (d)
Legislators.
2. It can be issued by the Judiciary to the (1) Habaes Corpus
Executive.
3. It can be issued by the Judiciary to the Q.313 Consider the following statements about the
Judiciary. write of Habaes Corpus.

 P.80 For Civil Services Preliminary Examination

01-Indian Polity_Q1-412.indd 80 8/7/2018 7:39:06 PM


1. It can be issued by even District courts in The writ of mandamus cannot be issued
India. • against a private individual or body
2. It can be issued against private individuals • to enforce departmental instruction that
too. does not possess statutory force
3. It cannot be issued when the detention of • when the duty is discretionary and not
the person is lawful. mandatory
Choose the correct answer using the codes • to enforce a contractual obligation
given below. • against the president of India or the state
(a)  1 and 2 only (b)  2 and 3 only governors
(c)  1 and 3 only (d)  All of the above • against the chief justice of a high court
Solution: (b) acting in judicial capacity
It is a Latin term which literally means ‘to Q.315 Consider the following about the writ of
have the body of’. It is an order issued by the Mandamus.
court to a person who has detained another 1. It is used to prevent usurpation of public
person, to produce the body of the latter offices by ineligible individuals.
before it. The court then examines the cause 2. It cannot be issued against public
and legality of detention. It would set the executive authorities.
detained person free, if the detention is found 3. It cannot be issued by the lower courts.
to be illegal. Thus, this writ is a bulwark of Select the correct answer using the codes below.
individual liberty against arbitrary detention. (a)  1 and 2 only (b)  3 only
The writ of habeas corpus can be issued (c)  2 and 3 only (d)  1 only
against both public authorities as well as Solution: (b)
private individuals. The writ, on the other Justification: Statement 1 & 2: That is the
hand, is not issued where the writ of Quo Warranto, not Mandamus.
(a)  detention is lawful Statement 3: only supreme court and high
(b) the proceeding is for contempt of a court can issue the writs. Lower courts aren’t
legislature or a court authorisedby the parliament to issue writs.
(c)  detention is by a competent court
Q.316 Under which of the following writs a court
(d) detention is outside the jurisdiction of
issue command to a public official asking
the court.
him to perform his official duties that he has
failed or refused to perform?
(2) Mandamus (a)  Certiorari (b)  Mandamus
Q.314 Writ of Mandamus is a command issued by (c)  Prohibition (d)  Quo-warranto
the court. It can be issued to Solution: (b)
1. Governors Q.317 The writ of Mandamus CANNOT be issued
2. Tribunals by the High Court against
3. Inferior courts 1. A private individual
Choose the correct answer using the codes 2. Any government official
given below. 3. A subordinate court
(a)  1 and 2 only (b)  2 and 3 only Select the correct answer using the codes
(c)  1 and 3 only (d)  All of the above below.
Solution: (b) (a)  1 and 2 only (b)  1 and 3 only
It literally means ‘we command’. It is a (c)  1 only (d)  2 and 3 only
command issued by the court to a public Solution: (c)
official asking him to perform his official
duties that he has failed or refused to perform. (3) Certiorari
It can also be issued against any public body,
a corporation, an inferior court, a tribunal or Q.318 The writ of Certiorari is issued by
government for the same purpose. (a) A higher court to a lower court or tribunal
(b)  Judiciary to the executive

Indian Polity Question Bank P.81

01-Indian Polity_Q1-412.indd 81 8/7/2018 7:39:06 PM


(c)  Judiciary to the legislature Also, there must be a contravention
(d)  A higher court to its officers of statues or constitutional provisions in
Solution: (a) appointing such persons to the office. So, 3
Learning: It is issued by a higher court to a is wrong.
lower court or tribunal either to transfer a case A writ of quo warranto is never issued as
pending with the latter to itself or to squash a matter of course and it is always within the
the order of the latter in a case. It is issued discretion of the Court to decide.
on the grounds of excess of jurisdiction or The Court may refuse to grant a writ of
lack of jurisdiction. The order could also be quo warranto if it is vexatious or where the
against the principles of natural justice or the petitioner is guilty of larches, or where he
order could contain an error of judgment in has acquiesced or concurred in the very act
appreciating the facts of the case. against which he complains or where the
Thus, unlike prohibition, which is only motive of the relater is suspicious.
preventive, certiorari is both preventive as As to the question that can apply for writ
well as curative. to quo warranto, it can be stated that any
Certiorari is not available against private person can file a petition for this writ,
legislative bodies and private individuals although he is not personally aggrieved in or
or bodies. interested in the matter.
Ordinarily, delay and lashes would be no
(4) Quo warranto ground for a writ of quo warranto unless the
delay in question is inordinate.
Q.319 Whenever any private person wrongfully
usurps an office, he is prevented by the writ
of quo warranto from continuing in that
Miscellaneous
office. Which among the following is/are the Q.320 Consider the following statements about
basic conditions necessary for the issue of the writs issued by the Supreme court and
writ of quo warranto? High Courts:
1. The office must be public and must be 1. “Habeas Corpus‟ can not be issued against
created by a statute or by the constitution private individuals.
itself. 2. “Prohibition‟ can be issued only against
2. The office must be a substantive one and judicial or quasi-judicial authorities.
not merely the function or employment of a 3. “Mandamus‟ can be issued against private
servant at will by a government employee. entities.
3. There must have been a contravention of Which of these is/are correct?
prevailing conventions or meritocracy in (a)  1 and 2 (b)  2 and 3
appointing such persons to the office. (c)  1 and 3 (d)  Only 2
Select the correct answer using the code Solution: (d)
given below. Q.321 Habeas Corpus can be issued against private
(a)  1 only (b)  1 and 2 only individuals too as it is concerned with
(c)  2 and 3 only (d)  1, 2 and 3 protection of human rights.
Solution: (b) Mandamus can only be issued against
Justification: The term quo warranto means public authorities
“by what authority you are holding the office” Consider the following statements about
The basic conditions for the issue of the writ various writs mentioned in the Constitution
are that the office must be public, it must have of India.
been created by statute or Constitution itself, 1. Mandamus is issued when the court finds
it must be of a substantive character and the that a particular office holder is not doing
holder of the office must not be legally qualified legal duty and thereby is infringing on the
to hold the office or to remain in the office or right of an individual.
he has been appointed in accordance with law.

 P.82 For Civil Services Preliminary Examination

01-Indian Polity_Q1-412.indd 82 8/7/2018 7:39:06 PM


2. Certiorari is issued when the executive Which of these is/are correct?
has prorogued a House Session before the (a)  Only 1 (b)  Only 2
due date on unreasonable grounds. (c)  Both (d)  None of the above
Which of the above is/are correct? Solution: (b)
(a)  1 only (b)  2 only Habeas Corpus can be issued against private
(c)  Both 1 and 2 (d)  None individuals too as it is concerned with
Solution: (a) protection of human rights.
Q.322 Which of the following is/are the difference(s) Consider the following statements.
between the writs of “prohibition” and 1. The writ of Prohibition can be issued
“certiorari”? only against judicial and administrative
1. While “prohibition” is available during authorities.
the pendency of proceedings, “certiorari” 2. The writ of Certiorari is different from
is applicable only after the order has Prohibition in that the former can be
been passed. applied even to legislative bodies that
2. While “prohibition” is available against exceed their jurisdiction.
judicial authorities only, “certiorari” is Which of the above is/are correct?
applicable to administrative authorities (a)  1 only (b)  2 only
only. (c)  Both 1 and 2 (d)  None
Which of the above is/are correct? Solution: (d)
(a)  1 only (b)  2 only Q.324 Which one of the following pairs is correctly
(c)  Both 1 and 2 (d)  None matched?
Solution: (a) (a) Writ of Habeas Corpus - available against
Justification & Learning: For certiorari to private individual as well
apply, the order should have been already (b) Writ of Quo Warranto - available against
passed by the inferior court, which can subordinate courts only
later be quashed by the SC/HC. It applies to (c) Writ of Prohibition - available against
decisions of officers/tribunals having legal autonomous bodies only
authority. Both the writs are issued against (d) Writ of Certiorari - available against
legal bodies. public servants only
So, 2 is wrong. Solution: (d)
Prohibition
The Writ of prohibition means to forbid Article 34 – Martial Law
or to stop and it is popularly known as Stay
Order. Q.325 Consider the following statements about
This is issued when a lower court or a Martial law:
body tries to transgress the limits or powers 1. Martial law has explicit constitutional
vested in it. backing.
The writ of prohibition is issued by any 2. Martial law does not affect Centre-state
High Court or the SC to any inferior court, or relations.
quasi-judicial body prohibiting the latter from 3. While it is in operation, ordinary law
continuing the proceedings in a particular courts are suspended.
case, where it has no jurisdiction to try. Which of these is/are correct?
(a)  All of the above (b)  2 and 3
Q.323 Consider the following statements:
(c)  1 and 3 (d)  Only 2
1. ‘Habeas Corpus’ and ‘Mandamus’
Solution: (b)
both cannot be issued against private
Martial law is implicit in the constitution. It
individuals.
is imposed in an extraordinary situation and
2. ‘Prohibition’ can be issued only against
all the law courts are suspended while it is in
judicial or quasi-judicial authorities.
operation.

Indian Polity Question Bank P.83

01-Indian Polity_Q1-412.indd 83 8/7/2018 7:39:06 PM


It however does not affect centre-state Century. If you were only aware of this fact,
relations even while it suspends the civilian other options (after year 2000) could be
government in the specific areas where it is easily eliminated.
imposed. 86th amendment was about Right to
Education, 91st was about Anti-Defection,
Right to Property and 119th was about India-Bangladesh Land
Boundary Agreement (LBA).
Q.326 Which of these fundamental rights is NOT
Q.328 The right to property is a legal right and not
guaranteed by the Constitution of India?
a fundamental right. This has which of the
(a) Freedom to form cooperatives and other
following implications?
associations
1. There is no guaranteed right to
(b) Freedom against unreasonable
compensation in case of acquisition of a
censorship of thought and speech
private property by the state.
(c) Freedom to own, acquire and dispose of
2. It can be regulated without a constitutional
property anywhere in the country
amendment by an ordinary law of the
(d) Freedom from bonded labour and
Parliament.
inhuman exploitation
Which of the above is/are correct?
Solution: (c)
(a)  1 only (b)  2 only
Learning: Right to property was a
(c)  Both 1 and 2 (d)  None
fundamental right, but it was made a legal
Solution: (c)
right by 44th constitutional amendment act,
Justification: Statement 1: For e.g. the
1978. The act repealed article19(1)(f) and
earlier Land Acquisition Act did not provide
article 31 from part III. It added article 300A
for resettlement and rehabilitation after the
in part XII. It provided no person be deprived
acquisition of private land by the state.
of his property except by the authority of law.
Statement 2: Since it is a legal right, it
Now the state can acquire the property
can be curtailed by the Parliament through an
of the individual and it need not pay
ordinary law. Other implications are:
compensation except in when state acquires
1. It protects private property against
property
executive action but not against legislative
1. of minority educational institution
action.
2. land held by a person under his personal
2. In case of violation, the aggrieved person
cultivation and the land is within the
cannot directly move the Supreme Court
statutory ceiling limit
under Article 32 (right to constitutional
Q.327 Consider the following provisions. remedies including writs) for its
1. It curtailed the fundamental right to enforcement. He can move the High Court
property in India. under Article 226.
2. Any law made to give effect to the Directive
Principles in Article 39 (b) or (c) cannot
be challenged on violation of certain (8) DIRECTIVE PRINCIPLES OF STATE
fundamental rights. POLICY — PART IV – ARTICLES 36–51
Which of the following constitutional
amendment the above refers to? Q.329 What was known as the “Instrument of
(a) 25th Amendment Act, 1971 Instructions” contained in the Government
(b) 119th Constitutional Amendment, 2013 of India Act 1935 was incorporated in the
(c)  91st Amendment Act, 2003 Constitution of India as
(d)  86th Amendment Act, 2002 (a) Writs exercised by the Supreme Court
Solution: (a) (b) Rules of business of the Government of
Justification: The clear choice has to (a), India
because the Right to property was curtailed (c) Directive Principles of State Policy
in India much before, than the present 21st (d)  Mandate of the Cabinet Secretariat
Solution: (c)

 P.84 For Civil Services Preliminary Examination

01-Indian Polity_Q1-412.indd 84 8/7/2018 7:39:06 PM


Learning: The Directive Principles resemble make it clear that ‗these principles are
the ‘Instrument of Instructions’ enumerated fundamental in the governance of the country
in the Government of India Act of 1935. and it shall be the duty of the state to apply
In the words of Dr B R Ambedkar, ‘the these principles in making laws’. Thus,
Directive Principles are like the instrument they impose a moral obligation on the state
of instructions, which were issued to the authorities for their application, but the real
Governor-General and to the Governors of the force behind them is political, that is, public
colonies of India by the British Government opinion.
under the Government of India Act of 1935. As observed by Alladi Krishna Swamy
What is called Directive Principles is Ayyar, ‘no ministry responsible to the people
merely another name for the instrument of can afford lightheartedly to ignore the
instructions. The only difference is that they provisions in Part IV of the Constitution’.
are instructions to the legislature and the Similarly, Dr B R Ambedkar said in the
executive’. Constituent Assembly that ‗a government
Q.330 In the colonial legacy of government in India, which rests on popular vote can hardly ignore
the Directive Principles in the present Indian the Directive Principles while shaping its
Constitution resemble which of the following policy. If any government ignores them, it
colonial enactments? will certainly have to answer for that before
(a)  „Codes‟ enacted under Act of 1919 the electorate at the election time.
(b) „Instrument of Instructions‟ enacted Q.333 The purpose of the inclusion of Directive
under Independence Act, 1947 Principles of State Policy in the Indian
(c) „Instrument of Instructions‟ enumerated Constitution is to establish
in the Government of India Act of 1935 (a)  A market economy
(d) „Duties of State‟ enumerated in the (b) A stable Federation of States and the
Morley-Minto Act of 1909 Union
Solution: (c) (c)  Strong Bureaucracy
Q.331 The enforcement of Directive Principles (d)  Social and Economic Democracy
depends most on : Solution: (d)
(a)  the Courts Learning: The principles laid down therein
(b) an effective opposition in Parliament are considered fundamental in the governance
(c) resources available to the Government of the country, making it the duty of the State
(d)  public cooperation to apply these principles in making laws to
Solution: (c) establish a just society in the country.
They were made non-justifiable keeping in
view that the state may not have resources to
Features of the Directive Principles implement them. DPSPs are not enforceable
Q.332 As per the Constitution, which of the in a court of law.
following is/are “fundamental in the Q.334 What do you understand by the statement,
governance‟ of the country? “The Directiveprinciples of State Policy
1. Fundamental Rights (DPSP) in the constitution are non-
2. Fundamental Duties justiciablein nature”?
3. Directive Principles of State Policy (a) The courts cannot recognize the DPSP in
4. Preamble their judgments.
Choose the correct answer using the codes (b)  DPSP cannot be enforced by law.
given below. (c) They are not enforceable by the courts
(a)  All of the above (b)  1, 2 and 3 only for their violation.
(c)  1 and 2 only (d)  3 only (d)  All of (a), (b) and (c)
Solution: (d) Solution: (c)
Though the Directive Principles are non- Justification: Option (a): They do recognize.
justiciable, the Constitution (Article 37) For e.g. certain laws that violate Article

Indian Polity Question Bank P.85

01-Indian Polity_Q1-412.indd 85 8/7/2018 7:39:06 PM


14 of the constitution can be declared Q.336 Which of the following is/are true concerning
legitimate if they fulfil DPSP under parts of the Directive Principles of state policy(DPSP)
Article 39. enshrined in the Indian constitution?
In the Minerva Mills case (1980), 1. They are fundamental to the governance
the Supreme Court held that ‘the Indian of the country.
Constitution is founded on the bedrock of the 2. They are non-justiciable in nature.
balance between the Fundamental Rights and 3. The Indian constitution is founded on
the Directive Principles’. the bedrock of the balance between the
Option (b): DPSP like organizing village Fundamental rights and DPSP.
panchayats, cheap legal remedies for the poor Choose the correct answer using the codes
have already been implemented by law. below
Option (c): For e.g. if adequate steps are (a)  1 and 2 (b)  2 and 3
not taken to stop circulation of intoxicants (c)  1 and 3 (d)  All of the above
like liquors (according to the Directive Solution: (d)
Principles), the courts cannot dictate the Q.337 Directive Principles of State Policy help in
government to do so. 1. Amplifying the Preamble
Q.335 Even though the DPSP is non-justiciable, it 2. Guiding courts in Judicial review of
carries a lot of moral authority. Consider the legislative and executive acts
following statements: 3. Supplementing fundamental rights of
1. Parliament cannot make any law that is citizens
inconsistent with the DPSP. Choose the correct answer using the codes
2. Parliament cannot make any law that is below.
based on any principle outside that of (a)  1 and 2 only (b)  2 and 3 only
DPSP. (c)  1 and 3 only (d)  All of the above
3. The Supreme Court can review laws in the Solution: (d)
backdrop of DPSP and allow them even The Directive Principles, although confer no
if they violate some specific fundamental legal rights and creates no legal remedies, are
rights. significant and useful in the following ways:
Choose the correct answer using the codes • They are like an ‘Instrument of
below: Instructions’ or general recommendations
(a)  1 and 2 (b)  2 and 3 addressed to all authorities in the Indian
(c)  1 and 3 (d)  Only 3 Union. They remind them of the basic
Solution: (d) principles of the new social and economic
Parliament can make laws or take actions that order, which the Constitution aims at
are outside or inconsistent with DPSP. DPSP building.
is not a legally binding code. For example, • They have served as useful beacon-lights
even though the DPSP provides for a ban on to the courts. They have helped the courts
liquors and intoxicants, they are still sold and in exercising their power of judicial
regulated by laws in India. review, that is, the power to determine the
The SC or the HCs may approve of a law constitutional validity of a law.
that violates the FRs under Article 14 and • They form the dominating background to
19, if it implements Article 39 of the DPSP all State action, legislative or executive
(minimizing socio-economic inequality). and also a guide to the courts in some
Read the section in Chapter 2 of NCERT respects.
11th polity - RELATIONSHIP BETWEEN • They amplify the Preamble, which
FUNDAMENTAL RIGHTS AND solemnly resolves to secure to all citizens
DIRECTIVE PRINCIPLES – for greater of India justice, liberty, equality and
clarity. fraternity.

 P.86 For Civil Services Preliminary Examination

01-Indian Polity_Q1-412.indd 86 8/7/2018 7:39:06 PM


Q.338 Consider the following statements. 1. The goals and objectives that we as a
Assertion (A): The constitution classifies the society should adopt
Directive Principles of State Policy (DPSP) 2. Certain rights that individuals should
into three broad categories, socialistic, enjoy apart from the Fundamental Rights
Gandhian and liberal– intellectual. 3. Some policies that the government should
Reason (R): The values of the Indian adopt to further citizen welfare
National Movement have a bearing on the Select the correct answer using the codes
Indian constitution. below.
In the context of the above, which of these (a)  1 and 2 only (b)  2 and 3 only
is correct? (c)  3 only (d)  1, 2 and 3
(a) A is correct, and R is an appropriate Solution: (d)
explanation of A. Justification: We get some idea of the vision
(b) A is correct, but R is not an appropriate of makers of our Constitution by looking at
explanation of A. some of the Directive Principles
(c) A is correct, but R is incorrect. The governments from time to time tried
(d)  A is incorrect, but R is correct. to give effect to some Directive Principles of
Solution: (d) State Policy. They passed several zamindari
Justification: The Constitution does not abolition bills, nationalized banks, enacted
contain any classification of Directive numerous factory laws, fixed minimum
Principles. However, on the basis of their wages, cottage and small industries were
content and direction, they can be classified promoted and provisions for reservation
into three broad categories, viz, socialistic, for the uplift of the scheduled castes and
Gandhian and liberal–intellectual. scheduled tribes were made.
Gandhian principles in DPSP represent Statement 1: All these clearly reflect the
the programme of reconstruction enunciated social goals that we must follow like equity
by Gandhi during the national movement. and justice. So, Statement 1 is correct.
The constitution imports values from all over Statement 2: Rights such as equal pay for
the World, e.g. socialism from USSR, liberal equal work, child nutrition, old age security
values from the West etc. are some of the rights that DPSP mandates the
Q.339 Consider the following statements about the government to implement. So, 2 is correct.
Directive Principles of State Policy (DPSP). Statement 3: Statement 3 logically follows
1. The inheritance of DPSP in the Indian from Statements 1 and 2.
constitution is British colonial legacy. Q.341 The “Directive Principles of State Policy”
2. Legislations or rules are always required denote the ideals that the State should keep in
for implementing DPSP. mind while formulating policies and enacting
Which of the above is/are correct? laws. These principles are applicable to
(a)  1 only (b) 2 only which of the following organs?
(c)  Both 1 and 2 (d)  None 1. Legislative
Solution: (c) 2. Executive
Justification: DPSP is not self-enforceable. 3. Local authorities
It requires the parliament to make laws to 4. Public Sector enterprises (PSUs)
implement them. Choose the correct answer using the codes
They were inherited from the British given below.
Instrument of instructions given to the (a)  All of the above (b)  1, 2 and 3 only
governors and governor-generals under the (c)  1 only (d)  2 and 3 only
Government of India Act 1935. Solution: (a)
Q.340 The Directive Principles lists in Part IV of These are the constitutional instructions or
the Constitution essentially imply which of recommendations to the State in legislative,
the following? executive and administrative matters.

Indian Polity Question Bank P.87

01-Indian Polity_Q1-412.indd 87 8/7/2018 7:39:06 PM


According to Article 36, the term ‘State’ Solution: (a)
in Part IV has the same meaning as in Part III The Right against Exploitation, contained in
dealing with Fundamental Rights. Therefore, Articles 23–24, lays down certain provisions
it includes the legislative and executive to prevent exploitation of the weaker sections
organs of the central and state governments, of the society by individuals or the State.
all local authorities and all other public 1. Article 23 provides prohibits human
authorities in the country trafficking, making it an offence punishable
Q.342 Consider the following statements: by law, and also prohibits forced labour or
1. Fundamental rights are fundamental to the any act of compelling a person to work
governance of the nation as they embody without wages where he was legally entitled
the idea of a political democracy. not to work or to receive remuneration for
2. A Panchayat samiti is also morally it. However, it permits the State to impose
supposed to implement the Directive compulsory service for public purposes,
Principles of State Policy (DPSP). including conscription and community
Which of these is/are true? service. The Bonded Labour system
(a)  Only 1 (b)  Only 2 (Abolition) Act, 1976 has been enacted by
(c)  Both (d)  None Parliament to give effect to this Article.
Solution: (b) 2. Article 24 prohibits the employment
Statement 1: It’s DPSP that is fundamental to of children below the age of 14 years in
the governance of the nation, not fundamental factories, mines and other hazardous jobs.
rights as the vision of the nation is embodied Parliament has enacted the Child Labour
in DPSP. (Prohibition and Regulation) Act, 1986,
Statement 2: Panchayat samitis at the providing regulations for the abolition of,
block level are local bodies under the 73rd and penalties for employing, child labour,
amendment to the Indian constitution, and as well as provisions for rehabilitation of
are therefore covered under the definition of former child labourers.
‘State’ under Article 12 of the constitution. 3. Article 39 lays down certain principles
The ‘State’ is supposed to implement the of policy to be followed by the State,
DPSP. including providing an adequate means
of livelihood for all citizens, equal pay
for equal work for men and women,
Directive Principles of State Policy proper working conditions, reduction of
Q.343 With reference to some of the provisions the concentration of wealth and means of
of the Constitution of India related to the production from the hands of a few, and
Fundamental rights and Directive principles distribution of community resources to
of state policy (DPSPs), consider the “subserve the common good”.
following statements Despite being non-justiciable, the Directive
1. Prohibiting all forms of forced labour, Principles act as a check on the State;
child labour and human trafficking is theorised as a yardstick in the hands of the
enshrined in Directive Principles of State electorate and the opposition to measure
Policy the performanceof a government at the
2. The equal pay for equal work for men and time of an election. While stating that the
women is a fundamental right in India Directive Principles are not enforceable in
3. According to the Constitution of India, the any court of law, Article 37 declares them
DPSPs are fundamental to the governance to be “fundamental to the governance of
of the country the country” and imposes an obligation
Which of the above statements is/are on the State to apply them in matters of
INCORRECT? legislation.
(a)  1 and 2 Only (b)  2 Only
(c)  3 Only (d)  2 and 3 Only

 P.88 For Civil Services Preliminary Examination

01-Indian Polity_Q1-412.indd 88 8/7/2018 7:39:06 PM


Q.344 As per Directive Principles in Part IV of • To secure a living wage, a decent
the Constitution, the State shall direct its standard of life and social and cultural
policy towards securing that the ownership opportunities for all workers (Article 43).
and control of the material resources of the • To take steps to secure the participation
community so that
of workers in the management of
1. Concentration of wealth is avoided
2. Means of production are nationalized
industries (Article 43 A).
3. Private ownership is gradually • To raise the level of nutrition and the
discouraged standard of living of people and to
Select the correct answer using the codes improve public health (Article 47).
below. Q.346 Which of the following is/are implicitly/
(a)  1 and 2 only (b)  2 only explicitly both a fundamental Right and a
(c)  1 only (d)  2 and 3 only Directive principle of state policy (DPSP)?
Solution: (c) 1. Protecting children from forcible abuse.
Justification: Article 39 (b) says—The State 2. The right to adequate means to livelihood.
shall direct its policy towards securing that 3. Early childhood care
the ownership and control of the material Choose the correct answer using the codes
resources of the community are so distributed below
as best to subserve the common good. (a)  All of the above (b)  Only 1
Article 39 (c) says—“The state shall direct (c)  1 and 2 (d)  2 and 3
its policy towards securing that the operation Solution: (c)
of the economic system does not result Statement 1: Article 23 forbids the child
in the concentration of wealth and means labour. Where as Article 39(e) calls for the
of production to the common detriment preservation of the health and strength of
(it doesn’t imply either nationalization or workers and children against forcible abuse
shunning of private properties).” So, both 2 Statement 2: Article 19–freedom of
and 3 will be wrong. profession
Q.345 Which of the following is NOT a ‘Directive Article 39(a)–To secure the right to
Principle of State Policy‟? adequate means of livelihood for all citizens
1. Free legal aid to poor Statement 3: Early childhood care is not
2. Public assistance in cases of a fundamental right even though we have
unemployment a right of life. It does not cover this right.
3. Secure the participation of workers in the Subsequent legislations although have
management of industries ensured it as it is a DPSP.
Choose the correct answer using the codes Q.347 Article 39A of the Constitution places a duty
below. upon the State to provide a lawyer to any
(a)  3 only (b)  2 and 3 only citizen who is
(c)  1 only (d)  All are Directive Principles 1. Unable to engage a lawyer due to
Solution: (d) economic conditions
Some of the Socialistic Principles are: 2. Illiterate
• To promote equal justice and to provide 3. Unable to engage a lawyer due to disability
free legal aid to the poor (Article 39 A). 4. Harmed by the actions of the ‘State’
• To secure the right to work, to education Select the correct answer using the codes
and to public assistance in cases of below.
unemployment, old age, sickness and (a)  1 and 3 only (b)  1, 3 and 4 only
(c)  1, 2 and 3 only (d)  2, 3 and only
disablement (Article 41).
Solution: (a)
• To make provision for just and humane Explanation: As far as statement 4 is
conditions for work and maternity relief concerned, a court may order the government
(Article 42).

Indian Polity Question Bank P.89

01-Indian Polity_Q1-412.indd 89 8/7/2018 7:39:06 PM


to pay the litigation expenditure to the affected. So the correct hierarchy is minimum wage
The state does not have a duty to provide < Fair wage < Living wage.
a lawyer to them as such. So, statement 4 Q.350 “To secure for all citizens a uniform civil
is wrong. code throughout the country” is one of the
A person may be wealthy, but illiterate. Directive Principles. It means that
He is economically capable to hire a lawyer. (a) All citizens will be under the same laws
The state does not have a ‘duty’ to provide irrespective of their caste
a lawyer to them. So, statement 2 is also (b) All citizens will be under the same laws
wrong. irrespective of their religion
Learning: According to Article 22 of the (c) All citizens will be under the same laws
Constitution, every person has a Fundamental irrespective of their political authority
Right to be defended by a lawyer. To achieve and status
the objectives of Article 39A, government (d) All citizens will be under the same laws
has established the National Legal Services irrespective of them residing or not
Authority (NALSA) has been constituted residing in India
under the Legal Services Authorities Act, Solution: (b)
1987 to provide free Legal Services to The Uniform Civil Code (UCC) in India
the weaker sections of the society and to as depicted in Article 44 proposes to replace
organize Lok Adalats for amicable settlement the personal laws based on the scriptures and
of disputes. customs of each major religious community
Q.348 To get just and humane conditions for work in the country with a common set governing
and maternity relief is a every citizen.
(a)  Fundamental Right The implementation is the tough task in
(b)  Legal Right country like India. There are pros as well
(c)  Directive Principle of State Policy cons of it.
(d)  Executive Decree Pros
Solution: (c) To provide equal status to all citizens
Learning: To make provision for just and To promote gender parity
humane conditions for work and maternity To accommodate the aspirations of the young
relief is a DPSP under Article 42. It is related population
to labour rights. To support the national integration
Q.349 The DPSP provides for securing a living To bypass the contentious issue of reform of
wage to all workers. What is the correct existing personal laws
ascending hierarchy of the following wages Cons
that are given in the country? Practical difficulties due to diversity in India
1. Living wage Perception of UCC as encroachment on
2. Minimum wage religious freedom
3. Fair wage Interference of state in personal matters
Choose the correct answer using the codes Q.351 “To secure for all citizens a uniform civil
below code throughout the country” is one of the
(a)  3, 2, 1 (b)  1, 2, 3 Directive Principles of state policy in our
(c)  2, 1, 3 (d)  2, 3, 1 constitution. If this is implemented, it will
Solution: (d) lead to
1. Minimum wage includes the bare needs of 1. common laws for all religious
life like food, shelter and clothing. communities in India
2. Living wage contains the cost of education, 2. common laws for both men and women
health etc. other than the minimum wage 3. centralization of justice delivery system in
3. Fair wage is a mean between minimum courts rather than local bodies
wage and living wage.

 P.90 For Civil Services Preliminary Examination

01-Indian Polity_Q1-412.indd 90 8/7/2018 7:39:06 PM


Choose the correct answer using the codes Q.354 Which of the following Directive Principles
below. did NOT exist in the unamended original
(a)  1 and 2 only (b)  2 and 3 only constitution?
(c)  1 only (d)  All of the above 1. To protect and improve the environment
Solution: (c) and to safeguard forests and wild life
Uniform civil code in India is the debate 2. To secure for all citizens a uniform civil
to replace the personal laws based on code throughout the country
the scriptures and customs of each major 3. To secure the right to adequate means of
religious community in the country with a livelihood for all citizens
common set governing every citizen. These Select the correct answer using the codes
laws are distinguished from public law below.
and cover marriage, divorce, inheritance, (a)  1 and 2 only (b)  2 and 3 only
adoption and maintenance. (c)  1 only (d) 1, 2 and 3
The laws already apply similarly for both Solution: (c)
men and women. In specific cases of property Learning: The 42nd Amendment Act of
inheritance etc. a uniform civil code will have 1976 added four new Directive Principles to
no effect. However, there will be a common the original list. They require the State:
law for all the communities in India. 1. To secure opportunities for healthy
Q.352 Separation of the Judiciary from the development of children (Article 39).
Executive is enjoined by : 2. To promote equal justice and to provide
(a)  Preamble free legal aid to the poor (Article 39 A).
(b)  Directive Principle 3. To take steps to secure the participation of
(c)  Seventh Schedule workers in the management of industries
(d)  Judicial Decision (Article 43 A).
Solution: (b) 4. To protect and improve the environment
Article 50 says “To separate the judiciary and to safeguard forests and wild life
from the executive in the public services of (Article 48 A).
the State”. The 97th Amendment Act of 2011 added
a new Directive Principle relating to
Q.353 The Directive Principles of State Policy
co-operative societies. It requires the state to
(DPSP) concern with which of the following
promote voluntary formation, autonomous
matters?
functioning, democratic control and
1. Conservation of environment
professional management of co-operative
2. Protection of National Flag and honour
societies (Article 43B).
3. Labour rights
Select the correct answer using the codes Q.355 Consider the following statements about the
below: Directive principles outside of Part IV of the
(a)  1 and 2 only (b)  2 and 3 only constitution:
(c)  1 and 3 only (d)  All of the above 1. Promoting Hindi language is the duty of
Solution: (c) the Union government.
Protection of national flag and honour is a 2. The Panchayats should endeavour to
part of Fundamental rights for which laws provide adequate facilities for instruction
have been enacted. in mother tongue to linguistic minority
Labour rights have been covered by primary school children.
mentioning equal wage for equal work for Which of these is/are true?
men and women; ensuring a living wage (a)  Only 1 (b)  Only 2
for labourers; their participation in the (c)  Both (d)  None
management of industries etc. Solution: (c)
To protect and improve the environment
and to safeguard forests and wild life
(Article 48A).

Indian Polity Question Bank P.91

01-Indian Polity_Q1-412.indd 91 8/7/2018 7:39:07 PM


DPSP versus Fundamental rights 7. The Courts are bound 7. The court cannot
to declare a law declare a law
Q.356 Consider the following statements.
violative of any of the violative of any of the
1. Fundamental Rights are subordinate
Fundamental Rights Directive Principles
to Directive Principles of State Policy
as unconstitutional as unconstitutional
(DPSP).
and invalid. and invalid. However,
2. DPSP is not enforced automatically by
they can upload the
merely being a part of the Constitution; it
validity of a law on
requires a legislation to enforce it.
the ground that it was
Which of the above is/are correct?
enacted to give effect
(a)  1 only (b)  2 only
to a directive.
(c)  Both (d)  None
Solution: (b) Q.357 As per the Supreme Court of India, if a
Distinction Between Fundamental Rights law may transgress certain fundamental
and Directive Principles rights, but seeks to give effect to a Directive
Distiction Between Fundamental Rights and Directive Principle, the law may be saved from
Principles unconstitutionality. Which of the fundamental
rights may be transgressed by law this way?
Fundamental Rights Directive Principles 1. Article 14 2.  Article 19
1. These are negative as 1. These are positive as 3. Article 21 4.  Article 30
they prohibit the State they require the State Choose the correct answer using the codes
from doing certain to do certain things. given below.
things. (a)  All of the above (b)  1, 2 and 3 only
2. These are justiciable, 2. These are non- (c)  1 and 2 only (d)  3 and 4 only
that is, they are justiciable, that is, Solution: (c)
legally enforceable by they are not legally The Directive Principles are non-justiciable
the Courts in case of enforceable by the in nature, that is, they are not legally
their violation. Courts for their enforceable by the courts for their violation.
violation. Though non-justiciable in nature, they help
the courts in examining and determining
3. They aim at 3. They aim at the constitutional validity of a law. The
establishing political establishing social and Supreme Court has ruled many a times that
democracy in the economic democracy in determining the constitutionality of any
country in the country. law, if a court finds that the law in question
4. These have legal 4. These have moral and seeks to give effect to a Directive Principle, it
sanctions. political sanctions. may consider such law to be ‘reasonable’ in
5. They provide the 5. They provide the relation to Article 14 (equality before law) or
welfare of the welfare of the Article 19 (six freedoms) and thus save such
individual. Hence, community. Hence, law from unconstitutionality.
they are personal and they are Socretarian Q.358 Consider the following statements:
individualistic. and socialistic. 1. Any law or rule that violates the
6. They do not 6. They require fundamental right under Article 19 (Right
require any legislation for their to freedom) will be termed void by the
legislation for their implementation. They Supreme Court.
implementation. They are not automatically 2. The judiciary can not question the laws
are automatically enforced. that are made for the welfare of people of
enforced. India.

 P.92 For Civil Services Preliminary Examination

01-Indian Polity_Q1-412.indd 92 8/7/2018 7:39:07 PM


Which of these is/are true? 1. Establishment of Central Administrative
(a)  Only 1 (b)  Only 2 Tribunal
(c)  Both (d)  None 2. Enactment of Legal Services Authorities
Solution: (d) Act
Statement 1: Laws and rules will not be termed 3. 73rd Amendment Act to the Constitution
void by the SC if they intend to implement 4. Implementation of land reforms
some of the directive principles even if they Select the correct answer using the codes
violate the FRs under Articles 14 and 19. below.
Statement 2: Judicial review is a basic (a)  1, 3 and 4 only (b)  2, 3 and 4 only
feature. Any law can be questioned. (c)  1 and 4 only (d)  1, 2, 3 and 4
Q.359 Consider the following statements: Solution: (b)
1. DPSP is subordinate to the Fundamental Learning: Statement 1: It addresses
rights except Article 14 and 19 (Right to the service related grievances of central
freedom). government employees. It does not
2. The subordination of DPSP to fundamental implement any DPSP.
rights is mentioned in the constitution of Statement 2: The Legal Services Authorities
India. Act (1987) has established a nation-wide
Which of these is/are true? network to provide free and competent legal
(a)  Only 1 (b)  Only 2 aid to the poor and to organise lok adalats for
(c)  Both (d)  None promoting equal justice. – Article 39A
Solution: (a) Statement 3: Three-tier panchayati raj system
Statement 1: is true subsequent to the (at village, taluka and zila levels) has been
judgment of the SC in the Minerva Mills introduced to translate into reality Gandhiji’s
case (1980). It has not been mentioned in dream of every village being a republic. (As
the constitution and this is why the court mentioned in Article 40).
was moved in the first place to ascertain the Statement 4: It reduces inequality and leads to
status of DPSP versus FRs. welfare of weaker sections of the population
Statement 2: There is no such provision in (As mentioned in Article 39(b))
the constitution. Q.362 The enforcement of Directive Principles
Q.360 That the ―Indian Constitution is founded depends most on:
on the bedrock of the balance between (a)  the Courts
the Fundamental Rights and the Directive (b) an effective opposition in Parliament
Principles has been (c) resources available to the Government
(a)  Ruled by the Supreme Court (d)  public cooperation
(b) Provided for in the Constitution of India Solution: (c)
(c) Affirmed by a resolution of the Parliament Insufficient financial resources was one
(d) Affirmed by a resolution of the of the reason for making DPSP non
Constituent assembly of India enforceable.
Solution: (a)
In the Minerva Mills Ltd v. UOI, AIR 1980 (9) FUNDAMENTAL DUTIES —
SC 1789, the Supreme Court held that ‘the
Indian Constitution is founded on the bedrock
PART IVA – ARTICLE 51A
of the balance between the Fundamental Q.363 Which of the following is/are NOT a
Rights and the Directive Principles”. Fundamental Duty as enshrined in the
Constitution of India?
Implementation of DPSP 1. To value and preserve the rich heritage of
our composite culture
Q.361 Which of the following show the 2. To safeguard public property and to abjure
implementation of Directive Principles of violence
State Policy?

Indian Polity Question Bank P.93

01-Indian Polity_Q1-412.indd 93 8/7/2018 7:39:07 PM


3. Who is a parent or guardian to provide Q.364 Which of the following is a fundamental
opportunities for education to his child or duty mentioned in Article 51-A of the
ward, as the case may be, between the age Constitution?
of six and fourteen years 1. To develop a spirit of inquiry
4. To uphold and protect the sovereignty, 2. To respect the National Flag and National
unity and integrity of India Anthem
5. To respect women and uphold India‟s rich 3. To safeguard Public property
culture 4. To do charitable activities which promote
Choose the correct answer using the codes social and economic equity
below Choose the correct answer using the codes
(a)  3 and 5 Only (b)  3 Only given below.
(c)  5 Only (d)  1 and 5 (a)  All of the above (b)  1, 2 and 3 only
Solution: (c) (c)  1 and 2 only (d)  3 and 4 only
Except 5th option, all are fundamental duties Solution: (b)
The Fundamental Duties noted in the Q.365 Which of the following fundamental duties
constitution are as follows: are mentioned implicitly/explicitly under the
It shall be the duty of every citizen of India constitution of India?
1. to abide by the Constitution and respect its 1. To strive towards excellence in all spheres
ideals and institutions, the National Flag of individual and collective activity.
and the National Anthem; 2. To safeguard public property.
2. to cherish and follow the noble ideals which 3. To pay taxes.
inspired our national struggle for freedom; 4. Not to entertain superstition and
3. to uphold and protect the sovereignty, superstitious beliefs.
unity and integrity of India; Choose the correct answer using the codes
4. to defend the country and render national below
service when called upon to do so; (a)  1 and 2 (b)  1 and 3
5. to promote harmony and the spirit of (c)  2 and 4 (d)  1, 2 and 4
common brotherhood amongst all the Solution: (d)
people of India transcending religious, One of our fundamental duties asks us to
linguistic and regional or sectional develop scientific temper and develop a
diversities; to renounce practices spirit of enquiry. It means rejecting all that
derogatory to the dignity of women; is not based on reason and enquiry including
6. to value and preserve the rich heritage of superstitious beliefs. Scientific temper does
our composite culture; not only mean appreciating science and
7. to protect and improve the natural engaging in science related activities, but
environment including forests, lakes, also living rationally.
rivers and wild life, and to have There is no such duty to pay taxes. It
compassion for living creatures; however is a punishable offence not to pay
8. to develop the scientific temper, humanism taxes.
and the spirit of inquiry and reform;
Q.366 Which of these is/are NOT Fundamental
9. to safeguard public property and to abjure
Duties of an Indian citizen?
violence;
1. To properly nourish his/her children.
10. to strive towards excellence in all spheres
2. To safeguard national property.
of individual and collective activity so that
3. To defend the country and render national
the nation constantly rises to higher levels
service when called upon.
of endeavour and achievement;
Select the correct answer using the codes
11. who is a parent or guardian to provide
below.
opportunities for education to his child or
(a)  1 and 2 only (b)  3 only
ward, as the case may be, between the age
(c)  1 only (d)  2 and 3 only
of six and fourteen years
Solution: (c)

 P.94 For Civil Services Preliminary Examination

01-Indian Polity_Q1-412.indd 94 8/7/2018 7:39:07 PM


Justification: To provide opportunities for 3. The fundamental duties are non-justiciable,
education to his child or ward between the but fundamental rights are.
age of six and fourteen years is a fundamental Select the correct answer using the codes
duty. Nourishment, health, happiness etc below.
don’t come in it. So, 1 is wrong. (a)  1 only (b)  2 and 3 only
If there was an option like respect the (c)  1 and 3 only (d)  2 only
elderly, you can tick mark it right, since valuing Solution: (c)
and preserving the rich heritage of the country’s Justification: Statement 1: For e.g.
composite culture is a fundamental duty foreigners cannot be mandated to cherish the
Q.367 Which of the following is NOT a fundamental ideals that led to our freedom struggle, even
duty under Part IVA of the Constitution? if they are granted certain fundamental rights
(a)  To pay taxes on time during their stay in India, such as Article 21,
(b)  To respect the National Flag right to life.
(c)  To protect the environment Statement 2: Both can be enforced. For
(d) To develop a scientific and rational example, the flag code of India enforces
temper the fundamental duty of respecting national
Solution: (a) symbols.
Learning: Option (a) was recommended Statement 3: The Constitution does not
by Swaran Singh Committee but was not provide for their direct enforcement by the
accepted by the Government. courts. Moreover, there is not legal sanction
Rest all are enshrined in article 51A against their violation
Q.368 For a citizen of India, the duty to pay taxes Q.370 Consider the following statements about
is a: Fundamental duties.
(a)  Fundamental Duty 1. They are enforceable by law.
(b)  Legal obligation 2. They can be used by the courts for
(c)  Constitutional obligation determining the constitutionality of any
(d)  Moral obligation law that transgresses certain fundamental
Solution: (b) rights.
As per the Swaran Singh Committee (that Which of the above is/are true?
suggested inclusion of fundamental rights); (a)  1 only (b)  2 only
including duty to pay taxes should have been a (c)  Both (d)  None
fundamental duty, but it was not agreed upon. Solution: (c)
So as per the Direct taxation laws, They are enforceable by law. Hence, the
income and other kinds of taxes are a legal Parliament can provide for the imposition of
obligation for an Indian citizen as he uses appropriate penalty or punishment for failure
public services, receives social and political to fulfil any of them.
security from the state etc. Fundamental Duties help the courts
in examining and determining the
constitutional validity of a law. In 1992, the
Features of Fundamental Duties Supreme Court ruled that in determining the
Q.369 Which of these differentiate between constitutionality of any law, if a court finds
Fundamental Duties and Fundamental Rights that the law in question seeks to give effect
as enshrined in the constitution? to a fundamental duty, it may consider such
1. Unlike fundamental rights, the law to be ‘reasonable’ in relation to Article
fundamental duties are confined to citizens 14 (equality before law) or Article 19 (six
only and do not extend to foreigners. freedoms) and thus save such law from
2. While fundamental rights can be enforced unconstitutionality.
by a legislation, fundamental duties
cannot be.

Indian Polity Question Bank P.95

01-Indian Polity_Q1-412.indd 95 8/7/2018 7:39:07 PM


Q.371 Consider the following statements; 2. This method is more difficult: it requires
Assertion (A): Fundamental duties do not special majority of the Parliament and
extend to foreigners staying in India. consent of half of the State legislatures.
Reason (R): Since they do not enjoy Note that all amendments to the Constitution
Fundamental rights in India, they are under are initiated only in the Parliament. Besides
no obligation to follow fundamental duties. the special majority in the Parliament
In the context of the statements above, which no outside agency—like a constitution
of these is true? commission or a separate body—is required
(a)  Both A and R are true and R is the correct for amending the Constitution.
explanation for A. Similarly, after the passage in the
(b)  Both A and R are true and R is not the Parliament and in some cases, in State
correct explanation for A. legislatures, no referendum is required for
(c)  A is incorrect but R is correct. ratification of the amendment.
(d)  A is correct but R is incorrect Only elected representatives of the people
Solution: (d) are empowered to consider and take final
There are several fundamental rights enjoyed decisions on the question of amendments.
by the foreigners like Right to life, Right Thus, Sovereignty of elected representatives
against exploitation etc. But, as they are not (parliamentary sovereignty) is the basis of
the citizens of India, they are not supposed to the amendment procedure.
follow the duties Q.373 As per Article 368 of the Constitution, the
amendment of the Indian constitution can be
(10) AMENDMENT OF CONSTITUTION initiated in
1. Lok Sabha
— PART XX – ARTICLE 368 2. Rajya Sabha
3. The office of The President
Procedure for Amendment 4. State Legislatures
Q.372 Article 368 provides for the procedure for 5. Council of Ministers
amendment of the Constitution. With regard Choose the correct answer using the codes
to amendment, consider the following: below.
1. All amendments to the Constitution are (a)  1, 3 and 5 only (b)  1 and 2 only
initiated only in the Parliament. (c)  2, 3 and 4 only (d)  1, 2 and 5 only
2. The Constitution Commission is required Solution: (b)
to amend the Constitution. An amendment of the Constitution can be
3. After the passage of the amendment bill in initiated only by the introduction of a bill for
the Parliament and in some cases, in State the purpose in either House of Parliament
legislatures, referendum is required for and not in the state legislatures.
ratification of the amendment. It is true that the proposal for amendment
4. Sovereignty of elected representatives is may come from any of the options mentioned,
the basis of the amendment procedure. but only after the executive clears it can it
Which of the above statements is/are correct? be put forth the Parliament. From here the
(a)  1 and 4 Only (b)  1 and 2 Only formal process starts.
(c)  2 and 3 Only (d)  1, 2 and 4 Only Q.374 Consider the following statements about
Solution: (a) Constitutional amendment in India.
In this article (Article-368), there are two 1. A private member of the Parliament cannot
methods of amending the Constitution and introduce a constitutional amendment bill
they apply to two different sets of articles of 2. The prior permission of the President
the Constitution is required for the introduction of every
1. amendment can be made by special constitutional amendment bill
majority of the two houses of the 3. Special days are reserved for introducing
Parliament constitutional amendment bills

 P.96 For Civil Services Preliminary Examination

01-Indian Polity_Q1-412.indd 96 8/7/2018 7:39:07 PM


Choose the correct answer using the codes 1. A constitutional amendment bill requires
below the prior permission of the President.
(a)  1 and 2 only (b)  2 only 2. It can only be introduced in the Lok
(c)  1 and 3 only (d) None of the above sabha first as it is the house of the people
Solution: (d) from which the constitution derives its
The bill can be introduced either by a authority.
minister or by a private member and does not Which of these is/are true?
require prior permission of the president. A (a)  Only 1 (b)  Only 2
private member here means a MP who does (c)  Both (d)  None
not belong to the ruling coalition. Solution: (d)
Special days in the Parliament are reserved The prior permission of the President is
for Private member bills – not constitutional required only in the case of money bills and
amendment bills. other bills like the one which seeks to divide
Q.375 Article 368 of the Constitutions puts which a state. It can be introduced in any house.
of the following caveats for bills suggesting Q.378 Consider the following statements about a
constitutional amendment? Constitutional amendment bill.
1. They can be introduced only by the Leader 1. Each House of Parliament must pass the
of the House. bill separately.
2. They must receive prior consent of the 2. The bill must be introduced first in Lok
President before admission in Parliament. Sabha.
Which of the above is/are correct? Which of the above is/are correct?
(a)  1 only (b)  2 only (a)  1 only (b)  2 only
(c)  Both 1 and 2 (d)  None (c)  Both 1 and 2 (d)  None
Solution: (d) Solution: (a)
Q.376 A constitutional amendment bill Justification: Statement 1: Each House
1. Can only be initiated by the President of must pass the bill separately. In case of
India a disagreement between the two Houses,
2. Must be introduced in the Lok Sabha first there is no provision for holding a joint
3. Is subject to the casting vote of the sitting of the two Houses for the purpose of
Chairman of the house deliberation and passage of the bill. The bill
4. Cannot be introduced by private MPs must be passed in each House by a special
Select the correct answer using the codes majority.
below. Statement 2: An amendment of the
(a)  1, 2 and 4 only (b)  1, 2 and 3 only Constitution can be initiated only by the
(c)  4 only (d)  None of the above introduction of a bill for the purpose in either
Solution: (d) House of Parliament and not in the state
Justification: Statement 1: Only certain legislatures.
category of CA bills need the prior permission Q.379 A constitutional amendment bill must be
of the President, for example major tax passed in each house by a combination of
reforms. which of these conditions
Statement 2: It can be introduced in either 1. Two-thirds of total membership of each
house. house
Statement 3: Speaker has the casting vote 2. Majority of those present and voting in
only in case of a tie or indecisive situations. each house
Otherwise, if the MPs clear the bill, it is sent 3. Two-thirds of those present and voting in
to the President. each house
Statement 4: Any MP can introduce a CA 4. Majority of total membership of each
bill. house
Q.377 Consider the following statements about the
procedure to amend the Indian constitution:

Indian Polity Question Bank P.97

01-Indian Polity_Q1-412.indd 97 8/7/2018 7:39:07 PM


Choose the correct answer using the codes (d) None of the above is correct in this
below. regard.
(a)  1 and 2 (b)  3 and 4 Solution: (d)
(c)  1 only (d) Either option (b) OR (c) Justification: Option A: It should be ratified
Solution: (d) by the legislatures of half of the states by
The bill must be passed in each House a simple majority, that is, a majority of the
by a special majority, that is, a majority members of the House present and voting.
(that is, more than 50 per cent) of the total No special majority or two-thirds states are
membership of the House and a majority needed. So, A is incorrect.
of two-thirds of the members of the House Option B: After duly passed by both the
present and voting. Houses of Parliament and ratified by the
In case of two-third of total membership state legislatures, where necessary, the bill is
approving the bill, the second condition presented to the president for assent. No prior
automatically gets fulfilled. consent is required.
Q.380 The following four numbers represent the The power to initiate an amendment to the
attendance in the Lok Sabha on a particular Constitution lies with the Parliament.
day of a session. In which of the following Q.382 A constitutional amendment bill that amends
case(s) can a constitutional amendment bill major federal features can be passed by
never be passed which seeks to expand the (a) Special majority in each house of the
list of Fundamental duties? Parliament
1.  173 2.  274 (b) Absolute Majority in any one house of
3.  50 4.  358 the Parliament
Choose the correct answer using the codes (c) Simple majority in both house of
below Parliament as well as in majority of State
(a)  Only 3 Legislatures
(b)  All of the above (d)  None of the above is correct.
(c)  1 and 3 Solution: (d)
(d)  It can be passed in all the cases. Learning: Ordinarily, the bill must be passed
Solution: (c) in each House by a special majority, that is,
A constitutional amendment bill in this case a majority (that is, more than 50 per cent)
can only be passed with both a 2/3rd majority of the total membership of the House and a
of members present and voting and an majority majority of two-thirds of the members of the
of the total membership in the Parliament House present and voting.
which is 545. Each House must pass the bill separately.
In case 1 – 173 members, the condition of In case of a disagreement between the two
absolute majority can not be fulfilled. Houses, there is no provision for holding a
In case 3 – 50 members, the quorum is not joint sitting of the two Houses for the purpose
fulfilled. The house would not even proceed. of deliberation and passage of the bill.
Q.381 If a bill seeks to amend the federal provisions If the bill seeks to amend the federal
of the Constitution provisions of the Constitution, it must also be
(a) It must also be ratified by the legislatures ratified by the legislatures of half of the states
of at least two-thirds of the states by a by a simple majority, that is, a majority of the
special majority members of the House present and voting.
(b) It must be introduced in the Parliament So, (d) is the only correct answer.
with prior consent from the President Q.383 In case of a disagreement between the
(c) It must receive the approval of a two Houses regarding a constitutional
constitutional bench of Supreme Court amendment bill, which of the following can
under Article 143 before introduction in be recourse(s) to resolve the deadlock?
the Parliament 1. A joint sitting of both the houses
2. Vote by majority of state legislatures

 P.98 For Civil Services Preliminary Examination

01-Indian Polity_Q1-412.indd 98 8/7/2018 7:39:07 PM


3. If the bill is passed again by the Lok Sabha 1. Withhold the bill
alone with a higher majority 2. Return the bill for reconsideration of the
Choose the correct answer using the codes Parliament
below. 3. End the bill
(a)  1 and 2 only (b)  2 only Choose the correct answer using the codes
(c)  1 and 3 only (d)  None of the above below.
Solution: (d) (a)  1 and 2 only (b)  2 only
Q.384 Consider the following statements: (c)  1 and 3 only (d) None of the above
1. A constitutional amendment bill ends in Solution: (d)
case of a disagreement between the two After duly passed by both the Houses
houses. of Parliament and ratified by the state
2. The president can not return a legislatures, where necessary, the bill is
constitutional amendment bill to the presented to the president for assent.
Parliament for reconsideration. The president must give his assent to the
Which of these is/are true? bill. He can neither withhold his assent to the
(a)  Only 1 (b)  Only 2 bill nor return the bill for reconsideration of
(c)  Both (d)  None the Parliament.
Solution: (c) After the president’s assent, the bill
becomes an Act (i.e., a constitutional
Q.385 Consider the following statements.
amendment act) and the Constitution stands
Assertion (A): The president must give her
amended in accordance with the terms of
assent to a Constitutional amendment bill
the Act.
passed by due process.
Reason (R): The Constitution makers did Q.387 A constitutional amendment bill may need
not give the office of the President any the consent of
responsibility with regard to protecting the 1. Lok Sabha
constitution. 2. Rajya Sabha
In the context of the above, which of these 3. President of India
is correct? 4. State Legislative assemblies
(a) A is correct, and R is an appropriate 5. State legislative councils
explanation of A. Choose the correct answer using the codes
(b) A is correct, but R is not an appropriate below.
explanation of A. (a)  1, 2 and 3 only (b)  1, 2, 3 and 4 only
(c)  A is incorrect, but R is correct. (c)  4 and 5 only (d)  All of the above
(d)  A is correct, but R is incorrect. Solution: (b)
Solution: (d) The majority of the provisions in the
Justification: The president must give his Constitution need to be amended by a special
assent to the bill. He can neither withhold majority of the Parliament, that is, a majority
his assent to the bill nor return the bill for (that is, more than 50 per cent) of the total
reconsideration of the Parliament. membership of each House and a majority
The 24th Constitutional Amendment of two-thirds of the members of each House
Act of 1971 made it obligatory for the present and voting.
President to give his assent to a constitutional Those provisions of the Constitution
Amendment Bill. which are related to the federal structure
Before this he could treat a constitutional of the polity can be amended by a special
amendment bill just as an ordinary bill and majority of the Parliament and also with the
could use his veto powers. So, R is wrong consent of half of the state legislatures by a
simple majority.
Q.386 When a constitutional amendment bill is
produced before the President, what are the Q.388 Consider the following statements about the
options available to him? position of states concerning constitutional
amendments:

Indian Polity Question Bank P.99

01-Indian Polity_Q1-412.indd 99 8/7/2018 7:39:07 PM


1. A constitutional amendment bill can be (b) Both A and R are true but R is not a
initiated by the states in the USA unlike in correct explanation of A.
India. (c) A is true, but R is false
2. The constitution does not provide a time (d)  A is false, but R is true.
limit for getting the required amendment Solution: (d)
cleared by the states in India. Women’s reservation bill would bring major
Which of these is/are true? changes in the composition of the Parliament
(a)  Only 1 (b)  Only 2 and would thus require a constitutional
(c)  Both (d)  None amendment. But Political parties are non-
Solution: (c) constitutional bodies. They can internally
The USA being a truly federal nation gives allocate 33% seats to women without the
power to its states to initiate amendments need for any such amendment. However, their
to the constitution. It is not the case in India party constitution may need to be amended.
which is a quasi-federal nation. Q.391 Providing for reservation of women in which
Q.389 Consider the following statements. of the following bodies will NOT require a
1. The Judiciary cannot initiate the process constitutional amendment?
of constitutional amendment but can 1. Lok Sabha
effectively change the Constitution by 2. Rajya Sabha
interpreting it differently. 3. State Legislative Assembly
2. Elected representatives alone have the 4. State Legislative Council
power to amend the Constitution. 5. Political parties
Which of the above is true? Choose the correct answer using the codes
(a)  1 only (b)  2 only below.
(c)  Both 1 and 2 (d) None of the above (a)  All of the above (b)  3 and 4 only
Solution: (c) (c)  5 only (d)  None of the above
First statement can be seen in cases like Solution: (c)
Kesavananda Bharati Sripadagalvaru v. Any provision that changes the composition
State of Kerala (1973) 4 SCC 225: Basic of the legislative bodies will require a
structure Doctrine; in other cases which constitutional amendment. It is a major
enhanced the scope of Article 21 etc. In all change and hence will require a higher
other cases of constitutional amendment, it is majority in Parliament. A political party
the Parliament and the state legislatures that is a self-governing voluntary body and is
are involved in constitutional amendment via not a legislative or executive body in that
Article 368 of the constitution. sense. A change in Societies Act will do for
Q.390 Given below are two statements, one labelled the political parties. Hence, this is another
as Assertion (A) and the other labelled as solution advocated by parties to break
Reason (R): the deadlock in Parliament over women
Assertion (A): The reservation of 33% reservation.
of seats for women in Parliament and Q.392 The following are some cases of constitutional
Parliament does not require constitutional amendment. Which of these is/are correctly
amendment. matched with respect to its requirement?
Reason (R): Political parties contesting 1. Change in the jurisdiction of the Supreme
elections can allocate 33% of seats they Court – Special majority and Consent of
contest to women candidates without any states.
constitutional amendment. 2. Sixth schedule (administration of tribal
In the context of the above two statements, areas) – Special majority
which one of the following is correct? 3. Election of the Vice- President - Special
(a) Both A and R are true and R is the correct majority and Consent of states
explanation of A.

P.100 For Civil Services Preliminary Examination

01-Indian Polity_Q1-412.indd 100 8/7/2018 7:39:07 PM


4. Citizenship (acquisition and termination) • Privileges of the Parliament, its members
– Special majority and its committees.
Choose the correct answer using the codes • Use of English language in Parliament.
below • Number of puisne judges in the Supreme
(a)  1 and 3 only (b)  Only 1 Court.
(c)  1, 2 and 4 (d)  None of the above • Conferment of more jurisdictions on the
Solution: (d) Supreme Court.
Expansion in the SC’s mandate requires • Use of official language.
only a simple majority while a reduction in • Citizenship—acquisition and termination.
mandate requires a special majority with the • Elections to Parliament and state
consent of states. Change can mean both. legislatures.
The case of changing the procedure of • Delimitation of constituencies.
Election of only President requires special • Union territories.
majority with consent of the states, not • Fifth Schedule—administration of
Vice- President. For, the state assemblies are scheduled areas and scheduled tribes.
involved in electing the president unlike in • Sixth Schedule—administration of tribal
the case of the vice-president. areas.
Q.394 Which of the following provisions of the
Simple Majority Constitution can be amended by a simple
majority in the Parliament?
Q.393 A number of provisions in the Constitution
1. Conferment of more jurisdictions on the
can be amended by a simple majority of the
Supreme Court
two Houses of Parliament outside the scope
2. Delimitation of constituencies
of Article 368. These provisions include,
3. Directive Principles of State Policy
inter alia
4. Election of the President and its manner
1. Citizenship clauses
Select the correct answer using the codes
2. Abolition or creation of legislative
below.
councils in states
(a)  1 and 2 only (b)  3 and 4 only
3. Election of the President and its manner
(c)  1 only (d)  1, 2, 3 and 4
Choose the correct answer using the codes
Solution: (a)
below.
Justification: Statement 3 requires special
(a)  1 and 2 only (b)  2 only
majority.
(c)  1 and 3 only (d) None of the above
Statement 4 requires Special Majority of
Solution: (a)
Parliament and Consent of States.
These matters include:
Other provisions that can be changed by
• Admission or establishment of new states.
simple majority of Parliament are:
• Formation of new states and alteration
of areas, boundaries or names of existing Q.395 Changing the number of puisne (regular)
states. judges in the Supreme Court or High Court
• Abolition or creation of legislative requires legislation by the Parliament with
councils in states. (a)  Ordinary majority
• Second Schedule—emoluments, (b)  Special majority
allowances, privileges and so on of the (c)  Absolute majority
president, the governors, the Speakers, (d) Special majority with consent of majority
judges, etc. of states
• Quorum in Parliament. Solution: (a)
• Salaries and allowances of the members For changing powers, special majority is
of Parliament. required; for number of judges it is not. The
• Rules of procedure in Parliament. original Constitution of 1950 envisaged a

Indian Polity Question Bank P.101

01-Indian Polity_Q1-412.indd 101 8/7/2018 7:39:07 PM


Supreme Court with a Chief Justice and 5. Any of the lists in the Seventh Schedule.
seven puisne Judges - leaving it to Parliament 6. Representation of states in Parliament.
to increase this number. In the early years, all 7. Power of Parliament to amend the
the Judges of the Supreme Court sat together Constitution and its procedure
to hear the cases presented before them. As Any of the Lists in the 7th Schedule
the work of the Court increased and arrears Q.397 Constitutional amendments in which of
of cases began to cumulate, Parliament the following provisions would require the
increased the number of Judges from 8 in consent of the majority of the states?
1950 to 11 in 1956, 14 in 1960, 18 in 1978 1. Qualifications and appointment of the
and 26 in 1986. As the number of the Judges Governor of the state as depicted in Article
has increased, they sit in smaller Benches 157.
of two and three - coming together in larger 2. Powers of the President to promulgate
Benches of 5 and more only when required ordinances on the subject mentioned in
to do so or to settle a difference of opinion or the Concurrent list – Article 123.
controversy. 3. Term of the office of President –
Article 56.
Special Majority + State Consent Choose the correct answer using the codes
below:
Q.396 Which of the following provisions of the
(a)  1 and 3 (b)  1 and 2
Constitutions of India need the ratification by
(c)  2 and 3 (d) None of the above
the legislatures of not less than one-half of
Solution: (d)
the States to effect amendment?
Election of the President and its manner only
1. Extent of the legislative powers of the
are to ratified by the states, not his term in
Union and the States
the office.
2. Powers of the Supreme Court and
Although Governors are the highest
High Courts
authority in the states, the consent of the
3. Acquisition of a new territory by the
states is not needed in amending provisions
Government of India
relating to the Governor.
Select the correct answer using the codes
Change in the Ordinance making powers
below.
with respect to the state list may needed to be
(a)  1 and 2 only (b)  2 and 3 only
ratified by the states.
(c)  3 only (d)  2 only
Solution: (a) Q.398 Which of the following category of
Justification: Those provisions of the constitutional amendments would require the
Constitution which are related to the federal ratification of the states in India?
structure of the polity can be amended by 1. Citizenship clauses
a special majority of the Parliament and 2. Changes in the Concurrent List under
also with the consent of half of the state Seventh Schedule
legislatures by a simple majority. 3. Changes in the state boundaries
There is no time limit within which the 4. Changing the number of puisine judges in
states should give their consent to the bill. the Supreme Court
The following provisions can be amended in Choose the correct answer using the codes
this way: below.
1. Election of the President and its manner. (a)  1 and 4 only (b)  2 and 3 only
2. Extent of the executive power of the (c)  1, 2 and 3 only (d)  2 only
Union and the states. Solution: (d)
3. Supreme Court and high courts. Citizenship – simple majority only in the
4. Distribution of legislative powers between Parliament.
the Union and the states. changing the boundary of states – simple
majority only in the Parliament.

P.102 For Civil Services Preliminary Examination

01-Indian Polity_Q1-412.indd 102 8/7/2018 7:39:07 PM


changing the number of judges in the feature would be determined by the Court in
SC – simple majority only in the Parliament. each case that comes before it.
Statement 3: During judicial review if the law
Basic Structure of the Constitution is found to be violative of the basic structure
of constitution then it can be declared void
Q.399 The Parliament cannot amend those and null
provisions which form the ‘basic structure’
Q.401 In Kesavananda Bharati Sripadagalvaru
of the Constitution. This was ruled by the
v. State of Kerala, (1973) 4 SCC 225, the
Supreme Court in the
Supreme Court came out with the ‘basic
(a) Olga Tellis v. Bombay Municipal
structure doctrine’. What are the implications
Corporation, AIR 1986 SC 18
of this doctrine for the constitution and the
(b) Bhikaji Jagannath Waghdhare v. UOI,
powers of the Parliament?
decided on 13 August, 2009
1. Not every provision of the constitution
(c) Kesavananda Bharati Sripadagalvaru v.
can be amended by the Parliament.
State of Kerala, (1973) 4 SCC 225
2. No constitutional body established
(d) Shankari Prasad v. UOI, AIR 1951 SC 455
immediately after the commencement of
Solution: (c)
the constitution can be scrapped.
Q.400 Consider the following statements about 3. The powers of Parliament and courts can
the ‘Basic Structure’ of the Constitution neither be expanded nor contracted.
of India. Choose the correct answer using the codes
1. It is mentioned in Part III of the below.
Constitution. (a)  1 only (b)  1 and 3 only
2. The Supreme Court has interpreted the (c)  2 and 3 only (d)  All of the above
Basic Structure in its various judgements Solution: (a)
giving also the exhaustive constituents of The reasoning behind the court’s judgment
the ‘basic structure’. was: “Since the Constitution had conferred a
3. A law or constitutional amendment can limited amending power on the Parliament,
be termed void if it violates the ‘Basic the Parliament cannot under the exercise
Structure’. of that limited power enlarge that very
Choose the correct answer using the codes power into an absolute power. Indeed, a
below: limited amending power is one of the basic
(a)  1 and 2 (b)  2 and 3 features of the Constitution and, therefore,
(c)  1 and 3 (d)  3 only the limitations on that power cannot be
Solution: (d) destroyed.
Statement 1: Basic structure is the judicial In other words, Parliament cannot, under
innovation and not mentioned in constitution. article 368, expand its amending power so
It restricts the power of the parliament to as to acquire for itself the right to repeal or
amend the constitution. It embodies those abrogate the Constitution or to destroy its
features which if amended will negate the basic features. The doctrine of a limited
very essence of our constitution. power cannot by the exercise of that power
Statement 2: The basic features of the convert the limited power into an unlimited
Constitution have not been explicitly defined one.”
by the Judiciary. At least, 20 features have
Q.402 The Parliament can amend the Constitution
been described as “basic” or “essential” by
subjected to the limit of
the Courts in numerous cases, and have been
(a) Altering the division of power between
incorporated in the basic structure.
Centre and States
In Indira Nehru Gandhi v. Raj Naraian
(b)  Basic Structure of the Constitution
and also in the Minerva Mills case, it was
(c)  Text of the Preamble
observed that the claim of any particular
(d)  Directive Principles of State Policy
feature of the Constitution to be a “basic”
Solution: (b)

Indian Polity Question Bank P.103

01-Indian Polity_Q1-412.indd 103 8/7/2018 7:39:07 PM


Learning: In the Kesavananda Bharati 1. Power of Supreme Court to issue writs in
Sripadagalvaru v. State of Kerala, (1973) case of violation of Fundamental Rights
4 SCC 225, the Supreme Court ruled that 2. Effective access to justice
the constituent power of Parliament under 3. Parliamentary system
Article 368 does not enable it to alter the Choose the correct answer using the codes
‘basic structure’ of the Constitution. below.
Options (a), (c) and (d) do not come (a)  1 and 2 only (b)  2 and 3 only
in the basic structure. For e.g. Centre can (c)  1 and 3 only (d)  All of the above
reduce the taxation powers of states under, Solution: (d)
let’s say GST, which is possible through The following are part of the ‘basic structure’
the Parliament. as has become clear from several rulings of
Preamble has already been amended once, Supreme Court:
and the DPSP too has been extended. 1. Supremacy of the Constitution
Q.403 With regard to the theory of basic structure 2. Sovereign, democratic and republican
of the Constitution, consider the following: nature of the Indian polity
1. Parliament has powers to amend all 3. Secular character of the Constitution
parts of the Constitution except the basic 4. Separation of powers between the
structure. legislature, the executive and the judiciary
2. Constitution specifies the basic structure 5. Federal character of the Constitution
of the Constitution 6. Unity and integrity of the nation
3. It found its first expression in the Golaknath 7. Welfare state (socio-economic justice)
v State of Punjab, (1967 AIR 1643). 8. Judicial review
4. It has increased the powers of the 9. Freedom and dignity of the individual
judiciary. 10.  Parliamentary system
5. Federal system, fundamental duties, 11.  Rule of law
social justice form the basic structure of 12. Harmony and balance between
the Constitution Fundamental Rights and Directive
Which of the above statements is/are Principles
incorrect? 13.  Principle of equality
(a)  1, 2 and 3 (b)  1 and 4 Only 14.  Free and fair elections
(c)  2 and 3 Only (d)  2, 3 and 5 Only 15.  Independence of Judiciary
Solution: (d) 16. Limited power of Parliament to amend the
Judiciary has defined which aspects of the Constitution
Constitution can be termed as the basic 17.  Effective access to justice
structure and which cannot. 18.  Principle of reasonableness
This theory found its first expression in 19. Powers of the Supreme Court under
the Kesavananda Bharati Sripadagalvaru v. Articles 32, 136, 141 and 142
State of Kerala, (1973) 4 SCC 225 and has Q.405 As per the interpretation of the Supreme
been discussed in subsequent judgments. It Court, which of the following does NOT
has increased the powers of the judiciary and come under the ‘basic structure’ of the
has come to be accepted by different political constitution?
parties and the government. Though Federal (a)  Panchayati Raj
system and social justice forms the basic (b)  Judicial Review
structure of the Constitution, fundamental (c) Freedom and dignity of the individual
duties does not. (d) Secularism
Q.404 As per a ruling of the Supreme Court, the Solution: (a)
“basic structure‟ of the constitution can Q.406 From the various judgements of the Supreme
not be amended by the Parliament. Which Court, which of the following have emerged
of the following come under the “basic as elements of ‘basic structure’ of the
structure”? Constitution?

P.104 For Civil Services Preliminary Examination

01-Indian Polity_Q1-412.indd 104 8/7/2018 7:39:07 PM


1. Separation of powers between the The First Amendment of the Constitution
legislature, the executive and the judiciary of India, enacted in 1951, made several
2. Harmony and balance between changes to the Fundamental Rights provisions
Fundamental Rights and Directive of the constitution. It provided against
Principles abuse of freedom of speech and expression,
3. Supremacy of the Constitution validation of zamindari abolition laws, and
4. Principle of reasonableness clarified that the right to equality does not bar
5. Welfare state and ideals of Socio- the enactment of laws which provide “special
economic Justice consideration” for weaker sections of society.
Select the correct answer using the codes The formal title of the amendment is the
below. Constitution (First Amendment) Act, 1951.
(a)  1, 2 and 3 only (b)  1, 3 and 5 only It was moved by the then Prime Minister of
(c)  2, 4 and 5 only (d)  1, 2, 3, 4 and 5 India, Jawaharlal Nehru, on 10 May 1951
Solution: (d) and enacted by Parliament on 18 June 1951.
Q.407 In which of the following cases of This amendment set the precedent of
constitutional amendment can the law be amending the Constitution to overcome
possibly declared void by the Supreme judicial judgements impeding fulfilment of
Court for violating the basic structure of the the government’s perceived responsibilities
constitution? to particular policies and programmes. The
1. An amendment which makes India a amendment’s language giving it retrospective
police state instead of a welfare state. as well as prospective effect was used
2. An amendment which gives the Parliament by Prime Minister Indira Gandhi during
substantial judicial powers. the Emergency, to render constitutional,
3. An amendment which declares the actions that had been both illegal and
Parliament to be supreme above the unconstitutional.
Judiciary. Q.409 First Amendment Act, 1951 to the
Choose the correct answer using the codes Constitution of India
below 1. Added more restrictions on Freedom of
(a)  2 and 3 (b)  1 and 3 Speech and Expression in India
(c)  1 and 2 (d)  All of the above 2. Added Ninth Schedule to protect the land
Solution: (d) reform and other laws included in it from
the judicial review
Few Amendments 3. Readjusted the scale of representation in
the Lok Sabha
Q.408 Consider the following statements 4. Empowered the president to fix the time-
1. The First Constitutional amendment set the limit for the state legislatures to express
precedent of amending the Constitution to their views on the proposed Central
overcome judicial judgements impeding legislation
fulfilment of the government’s perceived Select the correct answer using the codes
responsibilities to particular policies and below.
programmes. (a)  1 and 2 only (b)  2, 3 and 4 only
2. In the First Constitutional amendment, (c)  1 and 4 only (d)  1, 2, 3 and 4
article 31B was introduced to validate 13 Solution: (a)
enactments relating to zamindari abolition Justification: Statement 1 and 2: Important
Which of the above is/are incorrect? elements were:
(a)  1 Only (b)  2 Only • Empowered the state to make special
(c)  Both (d)  None provisions for the advancement of socially
Solution: (d) and economically backward classes.
Both are correct.

Indian Polity Question Bank P.105

01-Indian Polity_Q1-412.indd 105 8/7/2018 7:39:07 PM


• Provided for the saving of laws providing Q.412 Put in chronological order the following
for acquisition of estates, etc. Amendments to the Constitution.
• Added Ninth Schedule to protect the land 1. Addition of Ninth Schedule to the
reform and other laws included in it from Constitution.
the judicial review 2. Abolition of privy purses and privileges
• Added three more grounds of restrictions of former princes.
on freedom of speech and expression, 3. Insertion of XI Schedule
viz., public order, friendly relations with 4. Removal of Right to Property from
foreign states and incitement to an offence. Constitutional Rights.
Also, made the restrictions ‘reasonable’ (a)  I, II, III, IV (b)  II, I, IV, III
and thus, justiciable in nature. (c)  I, II, IV, III (d)  IV, I, II, III
Statement 3: It was added by the 2nd CA. Solution: (c)
Statement 4: Added by the 3rd CA.
Q.410 Which of these Amendments and their subject
matter is/are incorrectly matched?
1. 26th Amendment - Abolition of titles
and privileges of former rules of princely
states
2. 21st Amendment - Curtailed the right to
property
3. 51st Amendment - Curbed political
defections
4. 61st Amendment - Reduced voting age.
(a)  I, II and IV (b)  II, III and IV
(c)  I and IV (d)  II and III
Solution: (d)
Q.411 Match the following:
A. Fifty-eighth 1. Delhi to be called
  Amendment as National Capital
Territory of Delhi
B. Sixty-first 2. Reduced voting
  Amendment age from 21 years
to 18 years
C. Sixty-ninth 3. An authoritative
  Amendment text of the
Constitution in Hindi
D. Seventy-first 4. Included Konkani,
  Amendment Manipuri Nepali
 languages in the
Eighth Schedule
  A  B  C  D
(a)  3 2 1 4 (b)  3 1 2 4
(c)  2 1 3 4 (d)  4 2 1 3
Solution: (a)

P.106 For Civil Services Preliminary Examination

01-Indian Polity_Q1-412.indd 106 8/7/2018 7:39:07 PM


SYSTEM OF GOVERNMENT Reason (R): It is not practical for all
ministers to meet regularly and discuss
everything; hence the decisions are taken in
(11) PARLIAMENTARY SYSTEM Cabinet meetings
Q.413 Which of the following possibilities can be Which of the statements is correct?
found in a Parliamentary Democracy? (a) Both A and R are True and R is the correct
1. It can be a constitutional monarchy. explanation of A
2. President can be the head of the (b) Both A and R are True and R is not the
government. correct explanation of A
3. The Prime Minister may belong to the (c)  A is True and R is False
party without majority in the legislature. (d)  A is False and R is True
Choose the correct answer using the codes Solution: (a)
below. Cabinet Ministers are usually top-level
(a)  1 and 2 (b)  2 and 3 leaders of the ruling party or parties who are
(c)  1 and 3 (d)  All of the above in charge of the major ministries. Usually
Solution: (c) the Cabinet Ministers meet to take decisions
Statement 1 & 2: Parliamentary democracy in the name of the Council of Ministers.
can be monarchy where head of the Cabinet is thus the inner ring of the Council
government is the hereditary position or of Ministers. Since it is not practical for
republic that has elected head of the state all ministers to meet regularly and discuss
Statement 3: One of the features of the everything, the decisions are taken in
parliamentary democracy is majority party rule. Cabinet meetings. That is why parliamentary
Prime Minister is the leader chosen by party democracy in most countries is often known
that has majority support in the parliament. as the Cabinet form of government.
Q.414 Collective Responsibility of the council of Q.416 The Indian model of government is also
ministers to the Parliament is the bedrock called as the “Westminster‟ model of
principle of parliamentary government. government because
It implies that (a) Indian constitution was made with the
(a) The Parliament can remove the council assistance of the British
of ministers from office by passing a vote (b) Indian constitution was modelled
of no confidence very closely on the lines of the British
(b) The council of Ministers is appointed constitution
and dismissed by the Parliament (c) India follows parliamentary form of
(c) All important decisions of the council government
of Ministers have to be approved by the (d) The popular parliamentary house is
Parliament elected
(d)  None of the above Solution: (c)
Solution: (a) Westminster is a place in London where the
The ministers are collectively responsible to British Parliament is located. It is often used
the Parliament in general and to the Lok Sabha as a symbol of the British Parliament.
in particular (Article 75). They act as a team, The Parliament is the legislative organ
and swim and sink together. The principle of the Union government. It occupies a pre-
of collective responsibility implies that the eminent and central position in the Indian
Lok Sabha (not Parliament) can remove the democratic political system due to adoption
ministry (i.e., council of ministers headed by of the parliamentary form of government, also
the prime minister) from office by passing a known as ‘Westminster’ model of government.
vote of no confidence. Indian constitution departs on a number
of principles from the British constitution
Q.415 Assertion (A): Parliamentary democracy in
namely – federalism v/s unitary government;
most countries is often known as the Cabinet
monarchy v/s republic; judicial review etc.
form of government.

Indian Polity Question Bank P.107

02-Indian Polity_Q413-927.indd 107 8/7/2018 7:40:32 PM


Q.417 In a Parliamentary form of Government (c) Doctrine of separation of powers between
(a) The Legislature is responsible to the the legislature and judicial organs
Judiciary (d) Doctrine of separation of powers between
(b) The Executive controls the Judiciary the executive and judicial organs
(c) The Judiciary is responsible to the Solution: (b)
Executive Q.420 The parliamentary form of government
(d) The Executive is responsible to the emphasises on the
Legislature (a) Separation of powers between the
Solution: (d) legislature and executive
Learning: The relationship between the (b) System of Universal Adult Franchise
executive and the legislature in a parliamentary (c) Interdependence between the legislative
system is called responsible government. and executive organs
Here the executive branch derives its (d) Democratic election system administered
democratic legitimacy from its ability to by an independent body
command the confidence of the legislative Solution: (c)
branch. Legislature also holds the executive Justification: option (a) is valid in the
accountable. Presidential system as in USA. A Parliament
Q.418 Consider the following statements about the may not be based on democratic elections
forms of government in a democracy: or even adult franchise. It can be an entirely
Assertion (A): The Presidential form nominated body as it largely used to be in the
is a non-responsible government, whereas colonial rule. So, (b) and (d) are incorrect.
the Parliamentary form of government is a In India, executive is a part of the
responsible government. legislature, and the legislature is led by the
Reason (R): The legislature is responsible executive. So, (c) is the most appropriate
to the people in the Parliamentary system, option.
unlike in the Presidential system. Q.421 Which of the following is NOT a pillar of the
In the context of the above two statements, Parliamentary democracy in India?
which one of these is true? (a)  Universal adult suffrage
(a) Both A and R is true and R is the correct (b) Political representatives are accountable
explanation of A. to the people
(b) Both A and R is true but R is not a correct (c) All eligible citizens are allowed to contest
explanation of A. elections without discrimination.
(c)  A is true, but R is false. (d)  Separation of powers
(d)  A is false, but R is true Solution: (d)
Solution: (c) Explanation & Learning: Options (a), (b),
The legislature is accountable to the people (c) should be clear, since without universal
in both forms of government. It is in the participation and accountability, there cannot
Parliamentary form only that the executive be any Parliamentary government.
or the government is directly accountable to Indian parliamentary democracy does
the parliament. It establishes a responsible not rely on separation of powers. It is
government as the legislature can question instead based on cooperation, fusion and
the government on its conduct. In the coordination between the legislature and
presidential form, this is not possible. executive. Separation of powers is a feature
Q.419 India has adopted the parliamentary system of the Presidential form of democracy, as
of Government. It is based on the principle of in USA.
(a) Doctrine of separation of powers between Q.422 The Constitution establishes the
the legislature and executive organs ‘parliamentary system’ not only at the Centre
(b) Cooperation and co-ordination between but also in the states. This is evident by
the legislative and executive organs 1. Membership of the ministers in the
legislature

P.108 For Civil Services Preliminary Examination

02-Indian Polity_Q413-927.indd 108 8/7/2018 7:40:33 PM


2. Collective responsibility of the executive Parliaments all over the world exercise
to the legislature some control over those who run the
3. The provision of dissolution of the lower government. In some countries like India this
House control is direct and full. Those who run the
Choose the correct answer using the codes government can take decisions only so long
below. as they enjoy support of the Parliament.
(a)  1 and 2 only (b)  2 and 3 only Parliament is the highest forum of
(c)  1 and 3 only (d)  All of the above discussion and debate on public issues and
Solution: (d) national policy in any country
The parliamentary system is based on the Parliament can seek information about any
principle of cooperation and co-ordination matter but it does not enjoy absolute power
between the legislative and executive organs unlike Britain. The Parliament of India is
while the presidential system is based on the bound by the provisions of the Constitution.
doctrine of separation of powers between the Q.424 The features of Indian parliamentary system
two organs. The features of parliamentary are?
government in India are: 1. A Written Constitution
• Presence of nominal and real executives; 2. Presence of de jure and de facto executives
• Majority party rule, 3. Independent judiciary
• Collective responsibility of the executive 4. Collective responsibility of the executive
to the legislature, to the legislature
• Membership of the ministers in the 5. Individual responsibility of the executive
legislature, to the legislature
• Leadership of the prime minister or the Choose the correct answer using the codes
chief minister, below:
• Dissolution of the lower House (Lok (a)  2, 3 and 4 (b)  1, 2 and 4
Sabha or Assembly). (c)  2, 4 and 5 (d)  1, 2, 4 and 5
Q.423 According the parliamentary form of Solution: (c)
Government adopted in India, which of the It is a classical question. It has been asked
following statements are correct? several times in UPSC Prelims in one form
1. The Parliament is the final authority for or the other. A written constitution is not a
making laws feature of the Indian parliamentary system;
2. The Parliament control over those who it is a feature of our federal system or more
run the government is direct and full. broadly the democratic polity. Same holds
3. The Parliament is the highest forum of true for the independence of Indian judiciary
discussion and debate on public issues Q.425 The major features of parliamentary
and national policy. government in India is/are?
4. The Parliament of India enjoys absolute 1. Collective responsibility of the executive
power and can seek information about to the legislature
any matter. 2. Leadership of the prime minister
Select the correct answer using the codes 3. Complete separation of the legislature and
given below the executive organs
(a)  1 and 3 (b)  3 Only Select the correct answer using the codes below.
(c)  1, 2 and 3 (d)  1, 2, 3 and 4 (a)  1 and 2 only (b)  2 and 3 only
Solution: (c) (c)  1 and 3 only (d)  1, 2 and 3
Parliament is the final authority for making Solution: (a)
laws in any country. This task of law making or Justification: Statement 3: The parliamentary
legislation is so crucial that these assemblies system is based on the principle of cooperation
are called legislatures. Parliaments all and co-ordination between the legislative and
over the world can make new laws, change executive organs while the presidential system
existing laws, or abolish existing laws and is based on the doctrine of separation of powers
make new ones in their place. between the two organs. So, option 3 is incorrect.
Indian Polity Question Bank P.109

02-Indian Polity_Q413-927.indd 109 8/7/2018 7:40:33 PM


Learning: Other major features are: in some cases like investigation of scams and
(a) Presence of nominal and real executives; election related disputes, Judiciary resolves
(b) Majority party rule, (c) Collective the disputes.
responsibility of the executive to the Q.427 The Head of the State is an integral part of the
legislature, (d) Membership of the ministers Parliament in which of the following nations:
in the legislature, (e) Leadership of the prime 1. India
minister or the chief minister, (f) Dissolution 2. United States of America (USA)
of the lower House (Lok Sabha or Assembly). 3. United Kingdom (UK)
Q.426 The Constitution establishes the Choose the correct answer using the codes
parliamentary system not only at the Centre below:
but also in the states. Which of the following (a)  1 and 2 (b)  2 and 3
is/are the major features of parliamentary (c)  1 and 3 (d)  All of the above
government in India? Solution: (c)
1. Sovereignty of the Indian Parliament In the USA, while the President does have
2. Collective responsibility of the executive power over legislations passed by the
to the legislature and Judiciary congress and the senate, he is not a part of the
3. Membership of the ministers in the Parliament. In India and UK, the President
legislature and the monarch respectively are a part of
4. Presence of nominal and real executives the Parliament.
5. Resolution of all Parliamentary disputes Q.428 In the parliamentary system of Government,
by the Judiciary India has adopted the principle of checks and
Select the correct answer using the codes balances. Consider the following statements:
below. 1. Even though ministers and government
(a)  1, 2 and 4 only (b)  2, 3 and 4 only officials exercise power, they are
(c)  3 and 5 only (d)  1, 2, 3, 4 and 5 responsible to the Parliament or State
Solution: (b) Assemblies.
Justification: Statement 1: The Indian 2. Powers shared among governments at
Parliament is not a sovereign body even different levels
though it is based on British Parliamentary 3. Although the judges are appointed by the
model where the Parliament is sovereign. executive, they can check the functioning of
Statement 2: The parliamentary system executive or laws made by the legislatures
is based on the principle of cooperation 4. Power shared among different social
and co-ordination between the legislative groups, such as the religious and
and executive organs and the collective linguistic groups.
responsibility of the latter to the former. It Which of the statements is/are incorrect?
is not accountable to the Judiciary. Judiciary (a)  2 and 4 Only (b)  1 and 3 Only
only reviews the actions of the executive (c)  1, 2 and 3 Only (d)  1, 2, 3 and 4
when challenged. Solution: (a)
Statement 3: Ministers are mostly chosen Power is shared among different organs of
from the legislature. They act as the members government, such as the legislature, executive
of the particular house. and judiciary. This horizontal distribution
Statement 4: President and Governor of power because it allows different organs
are nominal executives, whereas the Prime of government placed at the same level to
Minister and the Chief Ministers of the exercise different powers. Such a separation
states are real executives wielding de facto ensures that none of the organs can exercise
powers. unlimited
Statement 5: The Chairman of the house
Q.429 Importance of the Parliamentary system in
resolves all major disputes within the
the A context of India lies in that
Parliament including the charges of political
1. It provides effective leadership in
defection under anti-defection law. However,
emergencies.

P.110 For Civil Services Preliminary Examination

02-Indian Polity_Q413-927.indd 110 8/7/2018 7:40:33 PM


2. It gives more responsibility and the federal system, judicial review and
assessment of the responsibility of the fundamental rights.
executive is both daily and periodic 3. In Britain, the prime minister should be
3. It ensures harmony between the Executive a member of the Lower House (House of
and the Legislature. Commons) of the Parliament. In India, the
4. It is a system with which the country had prime minister may be a member of any of
grown familiar. the two Houses of Parliament.
5. It is more efficiently workable when many 4. Usually, the members of Parliament alone
political parties in the form a coalition are appointed as ministers in Britain.
government. In India, a person who is not a member
Code of Parliament can also be appointed as
(a)  Only 4 (b)  2 and 4 minister, but for a maximum period of
(c)  2, 3 and 4 (d)  All of the above six months.
Solution: (d) Q.431 Even though the Indian Parliamentary System
Self-explainatory is largely based on the British pattern, there
are some fundamental differences between
Difference between Britain & Indian the two such as
1. The Indian Parliament is a sovereign body
Parliamentary Systems unlike the British Parliament which is
Q.430 Consider the following statements about the subordinate to its constitution.
Parliamentary system in India. 2. The Indian state is a republic where
1. Parliament is the supreme authority in the Head of state is elected unlike the
India. British states which is a monarchy with a
2. In India, the Prime Minister can be from hereditary Head of State.
either house of the Parliament. Which of the above is/are correct?
3. A person who is not a Member of (a)  1 only (b)  2 only
Parliament is also eligible to become (c)  Both 1 and 2 (d)  None
a Minister. Solution: (b)
Choose the correct answer using the codes Q.432 The parliamentary system of government
below. in India is largely based on the British
(a)  1 and 2 only (b)  2 and 3 only parliamentary system. However the Indian
(c)  1 and 3 only (d)  All of the above system differs from the British in which of
Solution: (b) the following?
The parliamentary system of government 1. India is a Republic whereas Britain is a
in India is largely based on the British constitutional Monarchy.
parliamentary system. 2. Citizens other than MPs cannot be appointed
However, it never became a replica of the as Ministers in India unlike in Britain.
British system and differs in the following 3. In Britain the prime minister should be a
respects: member of the Lower House unlike in India
1. India has a republican system in place where he can be a member of any house.
of British monarchical system. In other Select the correct answer using the codes
words, the Head of the State in India (that below.
is, President) is elected, while the Head (a)  1 and 2 only (b)  2 and 3 only
of the State in Britain (that is, King or (c)  1 and 3 only (d)  1, 2 and 3 only
Queen) enjoys a hereditary position. Solution: (c)
2. The British system is based on the Justification: In Britain, the prime minister
doctrine of the sovereignty of Parliament, should be a member of the Lower House
while the Parliament is not supreme in (House of Commons) of the Parliament. In
India and enjoys limited and restricted India, the prime minister may be a member of
powers due to a written Constitution, any of the two Houses of Parliament.

Indian Polity Question Bank P.111

02-Indian Polity_Q413-927.indd 111 8/7/2018 7:40:33 PM


Usually, the members of Parliament alone 1. There are four levels (or tiers) of
are appointed as ministers in Britain. In India, government.
a person who is not a member of Parliament 2. The fundamental provisions of the
can also be appointed as minister, but for a constitution can be unilaterally changed
maximum period of six months. by one level of government.
3. Courts have the power to interpret the
Presidential System constitution and the powers of different
levels of government.
Q.433 The feature which is present in the Presidential 4. Sources of revenue for each level of
System but lacking the Parliamentary system, government are clearly specified to ensure
is the its financial autonomy.
(a) distinction between the real and the Select the correct answer using the codes
nominal Executive below
(b)  fixed tenure of the Chief Executive (a)  3 and 4 (b)  1, 3 and 4
(c) formation of the Executive from the (c)  1 and 3 (d)  1, 2, 3 and 4
members of legislature Solution: (b)
(d) responsibility of the Executive to the Federalism is a system of government in which
Legislature the power is divided between a central authority
Solution: (b) and various constituent units of the country.
In presidential system president cannot Key features of federalism:
be removed by the Congress except by 1 There are two or more levels (or tiers) of
impeachment for a grave unconstitutional act. government.
Rest are features of parliamentary 2 Different tiers of government govern
government. the same citizens, but each tier has its
Q.434 The essence of the Presidential system of own jurisdiction in specific matters of
government is that, in such a system legislation, taxation and administration.
(a) the legislative, executive and judicial 3 The jurisdictions of the respective levels
powers of the government are separated or tiers of government are specified in
and vested in the three independent the constitution. So the existence and
organs of the government. authority of each tier of government is
(b) the President is elected directly by the constitutionally guaranteed.
people 4 The fundamental provisions of the
(c)  it has written and rigid constitution constitution cannot be unilaterally
(d) the Supreme Court has power of judicial changed by one level of government.
review Such changes require the consent of both
Solution: (a) the levels of government.
The doctrine of separation of powers is the 5 Courts have the power to interpret the
basis of the American presidential system. constitution and the powers of different
The President is elected by an electoral levels of government. The highest court
college for a fixed tenure. Hence option b acts as an umpire if disputes arise between
is wrong. different levels of government in the
Option c & d are there in parliamentary exercise of their respective powers.
system also 6 Sources of revenue for each level of
government are clearly specified to ensure
its financial autonomy.
(12) FEDERAL SYSTEM
Q.436 Which of the following is/are the key features
Federal System of Federalism?
Q.435 Country ‘A’ is said to be a federal country. 1. Different tiers of government govern the
Which of the following are the features of a same citizens, but each tier has its own
federal government? jurisdiction.

P.112 For Civil Services Preliminary Examination

02-Indian Polity_Q413-927.indd 112 8/7/2018 7:40:33 PM


2. Each tier of government must draw all • Essentially, federalism is an institutional
its financial resources independent of the mechanism to accommodate two sets
other tier. of polities—one at the regional level
3. The existence and authority of each tier of and the other at the national level. Each
government generally is constitutionally government is autonomous in its own
guaranteed. sphere. In some federal countries, there
4. Division of powers between State units is even a system of dual citizenship. India
cannot be arbitrarily manipulated by any has only a single citizenship.
one tier of government alone. • The people likewise, have two sets of
Select the correct answer using the codes identities and loyalties—they belong
below. to the region as well as the nation,
(a)  1, 2 and 3 only (b)  1, 3 and 4 only for example we are Gujaratis or
(c)  3 and 4 only (d)  1 and 2 only Jharkhandis as well as Indians. Each
Solution: (b) level of the polity has distinct powers
Justification: Statement 1: Each tier has and responsibilities and has a separate
its own jurisdiction in specific matters of system of government.
legislation, taxation and administration; • The details of this dual system of
for e.g. in India states legislate in matters government are generally spelt out
of police, and Centre legislates in areas of in a written constitution, which is
national security as a whole. considered to be supreme and which
Statement 2: Tiers may be dependent upon is also the source of the power of both
each other; however not completely, else it sets of government. Certain subjects,
erodes autonomy. So, 2 is incorrect. which concern the nation as a whole,
Statement 3: Existence of each tier of for example, defence or currency, are
government is constitutionally guaranteed the responsibility of the union or central
Statement 4: It is provided as check against government. Regional or local matters
arbitrary action of central government. are the responsibility of the regional or
Q.437 In a federation State government.
1. No more than two autonomous tiers of • To prevent conflicts between the centre
government can exist. and the State, there is an independent
2. All tiers derive their authority from the judiciary to settle disputes. The judiciary
higher tier and the constitution. has the powers to resolve disputes
3. Each tier has distinct set of powers and between the central government and the
responsibilities. States on legal matters about the division
Choose the correct answer using the codes of power.
below: Q.438 In a federal system of Government, there
(a)  1 and 2 (b)  2 and 3 must be
(c)  1 and 3 (d)  3 only 1. A Written Constitution
Solution: (d) 2. Independent Judiciary
Federalism does not consist of a set of fixed 3. Bicameral legislature
principles, which are applied, to different 4. Local governments
historical situations. Rather, federalism Choose the correct answer using the codes
as a principle of government has evolved given below.
differently in different situations. American (a)  All of the above
federalism – one of the first major attempts (b)  1, 2 and 3 only
to build a federal polity – is different from (c)  1 and 2 only
German or Indian federalism. But there are (d)  3 and 4 only
also a few key ideas and concepts associated Solution: (b)
with federalism.

Indian Polity Question Bank P.113

02-Indian Polity_Q413-927.indd 113 8/7/2018 7:40:33 PM


FEDERAL UNITARY Political scientists have classified
GOVERNMENT GOVERNMENT governments into unitary and federal on
the basis of the nature of relations between
1. Dual Government 1. Single government, the national government and the regional
(that is, national that is, the national governments. By definition, a unitary
government and government which government is one in which all the powers
regional government) may create regional are vested in the national government and
government the regional governments, if at all exist,
2. Written Constitution 2. Constitution may be derive their authority from the national
written (France) or government.
unwritten (Britain) A federal government, on the other hand,
3. Division of powers 3. No division of powers. is one in which powers are divided between
between the national All powers are the national government and the regional
and regional vested in the national governments by the Constitution itself and
government. government both operate in their respective jurisdictions
independently. In a federal model, the
4. Supremacy of the 4. Constitution may be national government is known as the Federal
Constitution supreme(Japan) or government or the Central government or
may not be supreme the Union government and the regional
(Britain) government is known as the state government
5. Rigid Constitution 5. Constitution may or the provincial government.
be rigid (France) or Britain, France, Japan, China, Italy,
flexible (Britain) Belgium, Norway, Sweden, Spain and many
6. Independent Judiciary 6. Judiciary may be others have the unitary model of government
independent or may while the US, Switzerland, Australia, Canada,
not be independent. Russia, Brazil, Argentina and so on have the
federal model of government.
7. Bicameral legislature 7. Legislature may be
bicameral (Britain) or Q.440 Which of the following could be considered
unicameral (china) as a precondition for the establishment of
federal government?
Q.439 Federal government means a government (a) A desire among a group of communities
in which or states to form a regional association so
(a) all the powers are vested in the as to augment their bargaining power with
national government and the regional the neighbouring bigger states, without
governments derive their authority from sacrificing any of their autonomous
the national government. powers to a higher authority.
(b) powers are divided between the (b) A desire among a group of communities
national government and the regional or states to leave all the major functions of
governments by the Constitution and both government to a central and independent
operate in their respective jurisdictions authority retaining with them only some
independently welfare work.
(c) there is division of powers between the (c) A desire among a group of communities or
Central and state governments; and also states to form an association of themselves
between the federal and state judiciaries to face some problems relating to
(d) a large number of powers are vested in communications and defence without any
the national government and the regional desire to establish a single independent
governments, with some independent government above them on a lasting basis
powers, derive their authority from the (d) A desire among a group of communities
national government or states to function under a single
Solution: (b) independent government for some

P.114 For Civil Services Preliminary Examination

02-Indian Polity_Q413-927.indd 114 8/7/2018 7:40:33 PM


purposes on a lasting basis, without Indian Model of Federalism
surrendering their autonomy in other
areas. Q.444 Indian model of federation is closer to that
Solution: (d) of which of the following nations/nation-
Refer above explanations states?
1. USSR
Q.441 Consider the following statements.
2. Canada
1. In the Unitary model of government,
3. USA
there cannot be more than one level of
Choose the correct answer using the codes
government.
below:
2. In federal model of government, each tier
(a)  1 and 2 (b)  2 and 3
of government’s existence and authority is
(c)  1 and 3 (d)  2 only
constitutionally determined.
Solution: (d)
3. In a federal model of government, the
US model is based on very strong states.
constituent units are not sub-ordinate to
USSR was based on an authoritarian centre.
the Central authority.
Canada is a federal country with a Union bias
Choose the correct answer using the codes
same as India.
below:
(a)  1 and 2 (b)  2 and 3 Q.445 Which one of the following statements
(c)  1 and 3 (d)  All of the above correctly depicts the true nature of the Indian
Solution: (b) Federal System?
Federations are contrasted with unitary (a) The Indian Federation is more a “functional”
governments. than an “institutional” Concept.
Under the unitary system, either there is only (b)  There is necessary incompatibility
one level of government or the sub-units are between one dominant party regime and
subordinate to the central government. The the principle of federalism in India.
central government can pass on orders to the (c) The Indian Federation is not the result of
provincial or the local government. an agreement by the units.
(d) India is a unitary state and subsidiary
Q.442 Consider the following statements:
federal features rather than a federal state
1. Sri Lanka is a federation because the
with subsidiary unitary features
country is divided into provinces.
Solution: (c)
2. A country can no longer remain a
Option a: Indian federation is both
federation if some powers of the
functional & institutional as we have vertical
States have been devolved to the local
distribution of power between different levels
government bodies.
of government.
Which of the statements given above are
Option d: federal features are not subsidiary
correct?
in India.
(a)  1 only (b)  2 only
Option b: Today we have coalition party
(c)  Both 1 and 2 (d)  None
regime rather the one dominant party.
Solution: (d)
Option c: Indian federation is not result of
A country cannot be federal just because
agreement between states. Hence states can’t
it has several provinces. Even Britain has
secede from the union.
provinces for that matter, but is not federal.
Q.446 Consider the following statements.
Q.443 Which of these countries practice federalism?
1. The term ‘Federation’ has no where been
1. Pakistan
used in the Constitution.
2. Russia
2. The state and local governments derive
3. Brazil
their authority from the constitution rather
Choose the correct answer here.
than from the Union government.
(a)  1 and 2 (b)  2 and 3
(c)  1 and 3 (d)  All of the above
Solution: (d)

Indian Polity Question Bank P.115

02-Indian Polity_Q413-927.indd 115 8/7/2018 7:40:33 PM


Which of the above is/are true? There are some units of the Indian Union
(a)  1 only (b)  2 only which enjoy very little power. These are areas
(c)  Both 1 and 2 (d)  None which are too small to become an independent
Solution: (c) State but which could not be merged with
The Constitution of India establishes a any of the existing States. These areas, like
federal system of government. It contains Chandigarh, or Lakshadweep or the capital city
all the usual features of a federation, viz., of Delhi, are called Union Territories. These
two government, division of powers, written territories do not have the powers of a State.
Constitution, supremacy of Constitution, The Central Government has special powers in
rigidity of Constitution, independent running these areas. Therefore, option 3 has no
judiciary and bicameralism. connection with the Principle of federalism.
However, the Indian Constitution also Rest two options ensure that the state
contains a large number of unitary or non- government is not subordinated by the Union
federal features, viz., a strong Centre, single government
Constitution, single citizenship, flexibility Q.448 Consider the following about Federalism
of Constitution, integrated judiciary, in India.
appointment of state governor by the Centre, 1. The Constitution does not mention
all-India services, emergency provisions, the sources of finance for each tier of
and so on. Government and leaves it for the executive
Moreover, the term ‘Federation’ has to decide.
nowhere been used in the Constitution. 2. The states are not merely agents of
Article 1, on the other hand, describes India the Central government but draw their
as a ‘Union of States’ which implies two authority from the Constitution.
things: one, Indian Federation is not the Which of the above is/are correct?
result of an agreement by the states; and (a)  1 only (b)  2 only
two, no state has the right to secede from the (c)  Both 1 and 2 (d)  None
federation. Solution: (b)
Hence, the Indian Constitution has been Justification and Learning: While each
variously described as ‘federal in form but state in India enjoys autonomy in exercising
unitary in spirit’,‘quasi-federal’ by K C powers on certain issues, subjects of national
Wheare, ‘bargaining federalism’ by Morris concern require that all of these states follow
Jones, ‘co-operative federalism’ by Granville the laws of the central government.
Austin, ‘federation with a centralising • The Constitution contains lists that detail
tendency’ by Ivor Jennings, and so on. the issues that each tier of government can
Q.447 Which of the following ensures that the make laws on.
principle of federalism is upheld in the • In addition, the Constitution also specifies
Indian polity? where each tier of government can get the
1. Any change to the constitution that affects money from for the work that it does. So,
the power of the states must be ratified by 1 is wrong.
a majority of state assemblies. • Under federalism, the states are not merely
2. The Judiciary plays an important role agents of the federal government but draw
in overseeing the implementation and their authority from the Constitution as
amendment of constitutional provisions. well. All persons in India are governed by
3. Union Territories (UTs) are administered laws and policies made by each of these
by the Centre. levels of government.
Choose the correct answer using the codes Q.449 The constitution of India provides for
below: federal system of Government. Consider the
(a)  1 and 2 (b)  2 and 3 following statements:
(c)  1 and 3 (d)  All of the above 1. The states are merely agents of the Central
Solution: (a) government.

P.116 For Civil Services Preliminary Examination

02-Indian Polity_Q413-927.indd 116 8/7/2018 7:40:33 PM


2. The states draw their authority from the 1. Division of powers between Central and
Constitution Regional governments
Which of the statements is/are correct? 2. Flexibility of the Constitution
(a)  1 Only (b)  2 Only 3. Independent Judiciary
(c)  Both 1 and 2 (d)  Neither 1 nor 2 4. All-India Services
Solution: (b) Select the correct answer using the codes
Under federalism, the states are not merely below.
agents of the federal government but draw (a)  1 and 2 only
their authority from the Constitution as well. (b)  2 and 4 only
Q.450 Consider the following about the federal (c)  1 and 3 only
structure of India. (d)  1, 2 and 3 only
1. The Indian federal structure is the result of Solution: (c)
an agreement between the states. Justification: The usual features of a
2. All states have a legitimate right to secede federation, viz., two government, division
from the federation by legislative means. of powers, written Constitution, supremacy
Which of the above is/are correct? of Constitution, rigidity of Constitution,
(a)  1 only (b)  2 only independent judiciary and bicameralism.
(c)  Both 1 and 2 (d)  None However, the Indian Constitution also
Solution: (d) contains a large number of unitary or non-
Justification: The term ‘Federation’ has federal features, viz. a strong Centre, single
nowhere been used in the Constitution. Constitution, single citizenship, flexibility of
Article 1 describes India as a ‘Union of Constitution, integrated judiciary, appointment
States’. This implies two things: one, Indian of state governor by the Centre, all-India
Federation is not the result of an agreement services, emergency provisions, and so on.
by the states; and two, no state has the right to Q.453 Which of the following is/are the federal
secede from the federation. So, a secessionist features of the Constitution?
movement by say the state of Nagaland is not 1. The Constitution is written and not easily
legitimate and constitutional. amendable.
Both statements 1 and 2 are wrong. Hence, 2. State governments derive authority from
the Indian Constitution has been variously the Centre.
described as ‘federal in form but unitary 3. Equal representation of all states in
in spirit. Rajya Sabha
Q.451 Federalism is a key feature of the Indian Select the correct answer using the codes
constitution. Which of the following below.
statements concerning this is INCORRECT? (a)  1 only (b)  2 and 3 only
(a) Both Central and state governments have (c)  1 and 3 only (d)  1 and 2 only
been established by the constitution. Solution: (a)
(b) Both governments derive their power Learning: The division of powers
from the constitution. established by the Constitution as well
(c) Since the State governments are as the supremacy of the Constitution can
autonomous political units, the Central be maintained only if the method of its
government cannot enforce any order amendment is rigid. Hence, the Constitution
on them. is rigid to the extent that those provisions
(d) Indian citizens are governed by laws which are concerned with the federal
and rules made by both Central and state structure. So, 1 is correct.
governments at the same time. State governments derive their authority
Solution: (c) from the constitution, not the Centre. So, 2
is wrong.
Q.452 The Constitution of India establishes a
The states are given representation in
federal system of government. It contains all
the Rajya Sabha on the basis of population.
the usual features of a federation which are?

Indian Polity Question Bank P.117

02-Indian Polity_Q413-927.indd 117 8/7/2018 7:40:33 PM


Hence, the membership varies from 1 to 31. Q.456 Which of the following strengthens the
So, 3 is wrong. practice of “Federalism” in India?
Q.454 Which of the following trends in the 1. An independent judiciary
working of Indian political system reflects its 2. Principle of Subsidiarity
federal spirit? Which of the above is/are correct?
1. Territorial disputes between states (a)  1 only (b)  2 only
2. Demand for creation of new states (c)  Both 1 and 2 (d)  None
3. Demand of the states for more financial Solution: (c)
grants from the Centre to meet their Justification: Statement 1: To prevent
developmental needs conflicts between the centre and the State,
4. Emergence of Regional parties and their there is an independent judiciary to settle
contribution in national politics disputes. The more independent the judiciary
5. Opposition of the State to the Goods and is, the more effectively it will be able to settle
Services Tax (GST) the disputes and enforce rule of law rather
Select the correct answer using the codes than the domination of the Centre.
below. The judiciary has the powers to resolve
(a)  1, 2 and 3 only disputes between the central government and
(b)  2, 4 and 5 only the States on legal matters about the division
(c)  3, 4 and 5 only of power.
(d)  1, 2, 3, 4 and 5 Statement 2: Certain subjects, which
Solution: (d) concern the nation as a whole, for example,
Learning: Although the Constitution of India defence or currency, are the responsibility of
has created a strong Central government, it the union or central government. Regional
has not made the state governments weak or local matters are the responsibility of the
and has not reduced them to the level of regional or State government.
administrative agencies for the execution of Appropriate division of responsibility
policies of the Central government. at the appropriate level (for example, local
Indian federation has been described as sanitation at the PRIs level and national
“a new kind of federation to meet India’s defence at the Central level) is called
peculiar needs”. principle of subsidiarity.
The state governments are autonomous Q.457 Which of the following factors have possibly
units working under Constitutional strengthened federalism in India?
framework in tandem with the Centre to 1. Rise of coalition politics at the Centre
achieve common goals. 2. Rise of regional parties
Q.455 The features of the Indian federal system are? 3. More financial and functional autonomy
1. Division of powers to the states.
2. Separation of powers between three Choose the correct answer using the codes
organs of the state below:
3. Independent judiciary (a)  1 and 2 (b)  2 and 3
4. Leadership of the Prime Minister and (c)  1 and 3 (d)  All of the above
cabinet Solution: (d)
5. A written constitution Over the last three decades, the number and
Choose the correct answer using the codes strength of these parties has expanded. This
below: made the Parliament of India politically more
(a)  2, 3 and 5 (b)  1, 4 and 5 and more diverse. No one national party is
(c)  1, 2 and 5 (d)  1, 3 and 5 able to secure on its own a majority in Lok
Solution: (d) Sabha (except the last general elections 2014).
Separation of powers and the leadership of the As a result, the national parties are compelled
Prime Minister are the salient features of our to form alliances with State parties. Since
democratic polity, not the federal structure. 1996, nearly every one of the State parties

P.118 For Civil Services Preliminary Examination

02-Indian Polity_Q413-927.indd 118 8/7/2018 7:40:33 PM


has got an opportunity to be a part of one or circumstances.
the other national level coalition government. (b) 
where the general government
This has contributed to the strengthening of is dependent upon the regional
federalism and democracy in our country. governments.
Q.458 The practice of “Fiscal Federalism” in India (c) 
where the regional governments are
involves subordinated to the general government.
(a) Distribution of financial powers between (d) 
where both general government and
Centre and States regional governments are coordinate and
(b) Setting up of Finance Commission every independent bodies
five years Solution: (d)
(c) Devolution of central pool to states Cooperative federalism, also known as
(d) All of the above marble-cake federalism, is a concept of
Solution: (d) federalism in which national, state, and
Learning: Federalism refers to the division local governments interact cooperatively
of responsibilities and functions between the and collectively to solve common problems,
Centre and States. Fiscal federalism is an rather than making policies separately
important component of federalism. It refers but more or less equally (such as the dual
to all financial matters conducted between the federalism of the nineteenth-century United
Centre and states with a view of a political States) or clashing over a policy in a system
federal structure in mind. dominated by the national government.
For e.g. the 14th FC has radically
enhanced the share of the states in the central Unitary Bias in Indian Constitution
divisible pool from the current 32 percent to
Q.461 Which of the following factors do NOT
42 per cent which is the biggest ever increase
justify a strong central government in the
in vertical tax devolution. This is supposed to
Indian federal structure?
strengthen fiscal federalism.
1. Balanced regional development
Q.459 Consider the following statements: 2. Territorial integrity of India
Assertion (A): All states In India have the 3. Growing financial needs of states
same number of assembly constituencies. 4. Increasing volume and funds allotted to
Reason (R): Indian political system is Social sector schemes
federal in nature. Choose the correct answer using the codes
In the context of the statements above, below.
which of these is correct? (a)  1 and 3 only (b)  1, 3 and 4 only
(a) Both A and R are correct and R is a (c)  3 and 4 only (d)  1, 2 and 4 only
correct explanation for A. Solution: (c)
(b) Both A and R are correct and R is NOT a Besides the concern for unity, the makers of
correct explanation for A. the Constitution also believed that the socio-
(c) A is correct but R is incorrect. economic problems of the country needed to
(d) A is incorrect but R is correct. be handled by a strong central government
Solution: (d) in cooperation with the States. Poverty,
All states do not have the same number of illiteracy and inequalities of wealth were
constituencies. India being a federal nation some of the problems that required planning
has nothing to with the number of assembly and coordination. Thus, the concerns for
seats in a state. The federal character of india unity and development prompted the makers
has more to do with the representation of of the Constitution to create a strong central
states in the Rajya Sabha. government.
Q.460 The idea of co-operative federalism indicates The central government has very effective
a constitution. financial powers and responsibilities. In the
(a) which would be both federal and unitary first pace, items generating revenue are under
according to requirements of time and the control of the central government. Thus,

Indian Polity Question Bank P.119

02-Indian Polity_Q413-927.indd 119 8/7/2018 7:40:33 PM


the central government has many revenue Q.463 Which of the following provisions of the
sources and the States are mostly dependent constitution point towards the Central bias in
on the grants and financial assistance from the Indian federation?
the centre. 1. Emergency provisions
Secondly, India adopted planning as the 2. The Position of Governor
instrument of rapid economic progress and 3. Constitution provides ‘Planning’ under
development after independence. Planning the Union List.
led to considerable centralisation of economic Choose the correct answer using the codes
decision making. below:
But as the needs of states grow, a central (a)  1 and 2 (b)  2 and 3
control will become obsolete. Instead Indian (c)  1 and 3 (d)  All of the above
federalism will have to give more space and Solution: (a)
autonomy for the states. The very existence of a State including
Q.462 The Constitution of India establishes a its territorial integrity is in the hands of
federal system of government. However, Parliament.
which of these are Unitary (Central) biases in The Constitution has certain very powerful
the federal system? emergency provisions, which can turn our
1. A written Constitution federal polity into a highly centralised system
2. Rigidity of Constitution once emergency is declared.
3. Independent Judiciary Even during normal circumstances,
4. All-India services the central government has very effective
Select the correct answer using the codes financial powers and responsibilities.
below. Governor has certain powers to
(a)  1 and 2 only (b)  3 and 4 only recommend dismissal of the State government
(c)  1 and 4 only (d)  4 only and the dissolution of the Assembly.
Solution: (d) Planning is under the Concurrent List –
Justification: Statement 1: A written you can find it out because each State has a
constitution actually shows federal features separate Planning Department.
of a constitution since both units (central and Q.464 Which of the following is/are “Unitary”
state) are bound by a common framework. features of the Indian constitution?
Statement 2: A rigid constitution is 1. Single Constitution
essential to a federal system so that the Centre 2. Flexibility of constitution
cannot unilaterally amend the constitution. 3. Integrated judiciary
Statement 3: An independent judiciary is 4. Position and appointment of the governor
crucial to maintain rule of law and ensure 5. Emergency provisions
harmonious and unbiased dispute redressal Choose the correct answer using the codes
between federal units. below
Statement 4: The All-India services are (a)  2, 4 and 5 (b)  1,3,4 and 5
controlled jointly by the State and Centre. (c)  All of the above (d)  1,2 and 3
Ultimate control lies with the Centre and Solution: (c)
only immediate control lies with states. These
civil servants severely limit the autonomy of Unitary System
the federal units. So, 4 is correct.
Other such features are a strong Centre, Q.465 Consider the following statements about
single Constitution, single citizenship, Unitary model of governments:
flexibility of Constitution, integrated 1. In a Unitary model of government, the
judiciary, appointment of state governor by constitution of the nation is always
the Centre, emergency provisions, and so on. supreme.
2. Legislatures are not bicameral in such a
model as all the power is vested in the
Central government.

P.120 For Civil Services Preliminary Examination

02-Indian Polity_Q413-927.indd 120 8/7/2018 7:40:33 PM


Which of these is/are true? Choose the correct answer using the codes
(a)  Only 1 (b)  Only 2 below:
(c)  Both (d) None of the above (a)  All of the above (b)  1, 3, 4 and 5
Solution: (d) (c)  1, 3 and 4 (d)  1 and 5
The constitution may or may not be supreme – Solution: (d)
not in case of Britain. Finance commission is a constitutional body.
In Britain legislature is bicameral. But it There is no council named Regional
is not as powerful as the lower house because Development Council. However, there are
all the power is technically vested in the several councils that are regional in nature
central government like North-Eastern Council etc.
Q.466 Consider the following statements about the National Advisory panel makes policies at
Unitary model of government: the national level. States do not have a say in it.
Assertion (A): It is possible to create University Grant Commission (UDC)
governments at multiple (regional etc.) coordinates the centre and the states in matters
levels in such a model even as the central of university education.
government is all powerful.
Reason (R): The regional governments Legislative Relations
derive their authority from the constitution of
Q.468 The legislative matters on which uniformity
that nation.
of legislation throughout the country is
In the context of two statements above,
desirable but not essential are enumerated in
according to you, which one of these is true?
the (Constitution)
(a) Both A and R is true and R is the correct
(a)  Residuary List
explanation of A.
(b)  Concurrent List
(b) Both A and R is true but R is not a correct
(c)  Fifth and Sixth Schedule
explanation of A.
(d)  Directive Principles of State Policy
(c) A is true, but R is false.
Solution: (b)
(d)  A is false, but R is true.
The matters of national importance and
Solution: (c)
the matters which require uniformity of
Regional governments can be created by
legislation nationwide are included in the
the Central government. But they do not
Union List. The matters of regional and local
derive their authority from the constitution.
importance and the matters which permits
They derive it from the Central government.
diversity of interest are specified in the
And it can be taken back anytime by that
State List. The matters on which uniformity
central government too. They will not be
of legislation throughout the country is
autonomous units.
desirable but not essential are enumerated in
the concurrent list. Thus, it permits diversity
(13) CENTRE-STATE RELATION –  along with uniformity.
PART XI & PART XII The power to make laws with respect to
residuary subjects (i.e., the matters which
Q.467 Which of the following non-constitutional are not enumerated in any of the three lists)
mechanisms promote coordination between the is vested in the Parliament. This residuary
centre and states? power of legislation includes the power to
1. National Integration council levy residuary taxes.
2. Finance Commission Q.469 Who can make laws on matters not included
3. Regional Development Council in the state List or Concurrent List?
4. National Advisory Council (a)  Parliament alone.
5. University Grants Commission (b)  State Legislature alone.
(c) Both Parliament and State Legislature.
(d)  Council of Ministers.
Solution: (a)

Indian Polity Question Bank P.121

02-Indian Polity_Q413-927.indd 121 8/7/2018 7:40:33 PM


Explanation: The Parliament of India is kind of transformation is not found in any
competent to legislate on all matters that other federation.
are enumerated in the Union List and the The governor is empowered to reserve
Concurrent List of the Constitution. In the certain types of bills passed by the state
Concurrent List, the Parliament and the State legislature for the consideration of the
Legislatures have joint jurisdiction. However, President. The President can withhold his
in case of conflict over any law made under assent to such bills not only in the first
the Concurrent List, the Union Law will instance but also in the second instance.
prevail upon the State Law provided the Thus, the President enjoys absolute veto (and
State Law has not received the earlier assent not suspensive veto) over state bills. But in
of the President. Parliament alone has power US and Australia, the states are autonomous
to make laws on matters not included in the within their fields and there is no provision for
state list or concurrent list. any such reservation.
Q.470 Which of the following features given in the
Constitution show that the division of powers Parliamentary Legislation
in the Constitution is in favour of the Centre? in the State List
1. Centre has overriding authority over the
Concurrent List Q.471 Under which of the following circumstances
2. States in India have no right to territorial can the Parliament legislate on the subjects in
integrity. the State list?
3. Veto of Parliament over State bills 1. Rajya Sabha passed a resolution to that
4. Financial emergency effect.
Choose the correct answer using the codes 2. Financial emergency
given below. 3. President’s Rule.
(a)  All of the above Choose the correct answer using the codes
(b)  1, 2 and 3 only below:
(c)  1, 2 and 4 only (a)  1 and 2 (b)  2 and 3
(d)  3 and 4 only (c)  1 and 3 (d)  All of the above
Solution: (c) Solution: (c)
The division of powers is in favour of the There are 5 conditions under which it can be
Centre and highly inequitable from the done:
federal angle. (a) To give effect to an international treaty to
1. Union List contains more subjects than which India is a signatory
the State List (b)  Under President’s rule
2. more important subjects have been (c)  Under national emergency
included in the Union List. (d) When two or more states request
3. The centre has overriding authority over Parliament to do so (applies to only those
the Concurrent List states)
4. residuary powers have also been left with (e) When Rajya Sabha passes a resolution
the Centre, but in the US, they are vested calling for Parliament to legislate in the
in the states. state list.
Thus, the Constitution has made the Centre Q.472 The Constitution empowers the Parliament
very strong. to make laws on any matter enumerated in
The Constitution stipulates three types of the State List under which of the following
emergencies—national, state and financial. extraordinary circumstances?
During an emergency, the Central government 1. When the President exhorts the Council of
becomes all powerful and the states go into Ministers to do so in national interest
the total control of the Centre. It converts the 2. When the lower house passes a resolution
federal structure into a unitary one without a to this effect by special majority
formal amendment of the Constitution. This

P.122 For Civil Services Preliminary Examination

02-Indian Polity_Q413-927.indd 122 8/7/2018 7:40:33 PM


3. When a National Emergency is in If the Rajya Sabha declares that it is necessary
operation in the national interest that Parliament should
4. When the National Security Council make laws on a matter in the State List, then
recommends so in the interest of national the Parliament becomes competent to make
security laws on that matter. Such a resolution must
Select the correct answer using the codes be supported by two-thirds of the members
below. present and voting. The resolution remains
(a)  1 and 2 only (b)  3 only in force for one year; it can be renewed any
(c)  3 and 4 only (d)  1, 2, 3 and 4 number of times but not exceeding one year
Solution: (b) at a time. The laws cease to have effect on the
Learning: If the Rajya Sabha declares that expiration of six months after the resolution
it is necessary in the national interest that has ceased to be in force.
Parliament should make laws on a matter in When the legislatures of two or more
the State List, then the Parliament becomes states pass resolutions requesting the
competent to make laws on that matter. This Parliament to enact laws on a matter in the
cannot be declared by the President, Lok State List, then the Parliament can make laws
Sabha or the National Security Council. So, for regulating that matter. A law so enacted
all statements except 3 are wrong. applies only to those states which have passed
The Parliament can make laws on any the resolutions. However, any other state may
matter in the State List for implementing adopt it afterwards by passing a resolution to
the international treaties, agreements or that effect in its legislature. Such a law can be
conventions amended or repealed only by the Parliament
Q.473 The Parliament can make any law for the and not by the legislatures of the concerned
whole or any part of India for implementing states.
International treaties Q.475 If the Rajya Sabha declares that it is necessary
(a)  with the consent of all the States in the national interest that Parliament should
(b) with the consent of the majority of States make laws on a matter in the State List, then
(c) with the consent of the States concerned the Parliament becomes competent to make
(d)  without the consent of any State laws on that matter. Such a resolution must
Solution: (d) be supported by
This power of the Parliament is derived from (a) two-third of members present and voting
the sovereign nature of India. Sovereign (b) two-third of members present and voting,
deals need not be ratified by the states. and majority of the membership of the
Q.474 The Constitution empowers the Parliament house
to make laws on any matter enumerated in (c)  simple majority of the house
the State List under which of the following (d) majority of the membership of the house
extraordinary circumstances? Solution: (a)
1. When states make a request Such a resolution must be supported by two-
2. To implement International agreements thirds of the members present and voting.
3. When Rajya Sabha passes a resolution to The resolution remains in force for one year;
that effect it can be renewed any number of times but
4. President’s order in the Extraordinary not exceeding one year at a time. The laws
gazette of India cease to have effect on the expiration of six
Choose the correct answer using the codes months after the resolution has ceased to be
given below. in force.
(a)  All of the above This provision does not restrict the power
(b)  1, 2 and 3 only of a state legislature to make laws on the
(c)  1, 2 and 4 only same matter. But, in case of inconsistency
(d)  3 and 4 only between a state law and a parliamentary law,
Solution: (b) the latter is to prevail.

Indian Polity Question Bank P.123

02-Indian Polity_Q413-927.indd 123 8/7/2018 7:40:33 PM


Q.476 Consider the following statements. to any of the matters enumerated in the
1. When the legislatures of two or more Concurrent List even if a Central law exists.
states pass resolutions requesting the But, laws made by state should not contravene
Parliament to enact laws on a matter in the Central laws.
State List, then the Parliament can make Q.478 In case of a conflict between the Central law
laws for regulating that matter. and the state law on a subject enumerated in
2. The Parliament can make laws on any the Concurrent List, which of the following
matter in the State List for implementing is possible?
obligations arising out of international 1. Central law prevails over the state law.
treaties, agreements or conventions. 2. State law prevails if it has received
Which of the above is/are correct? Presidential assent
(a)  1 only (b)  2 only 3. State law prevails of Governors of two
(c)  Both 1 and 2 (d)  None or more states have approved the same
Solution: (c) legislation
Justification: Statement 1: A law so enacted Choose the correct answer using the codes
applies only to those states which have passed below.
the resolutions. However, any other state may (a)  1 and 2 only
adopt it afterwards by passing a resolution to (b)  2 and 3 only
that effect in its legislature. (c)  1 and 3 only
Such a law can be amended or repealed only (d)  All of the above
by the Parliament and not by the legislatures of Solution: (a)
the concerned states. The Constitution expressly secures the
Statement 2: Some examples of laws predominance of the Union List over the
enacted under the above provision are United State List and the Concurrent List and that
Nations (Privileges and Immunities) Act, of the Concurrent List over the State List.
1947; Geneva Convention Act, 1960; Anti- Thus, in case of overlapping between the
Hijacking Act, 1982 and legislations relating Union List and the State List, the former
to environment and TRIPS. should prevail. In case of overlapping
between the Union List and the Concurrent
State Laws and Central Laws List, it is again the former which should
in the Concurrent List prevail. Where there is a conflict between
the Concurrent List and the State List, it is
Q.477 Consider the following statements. the former that should prevail.
1. State Legislatures can make laws on In case of a conflict between the Central
matters enumerated in the Residuary list law and the state law on a subject enumerated
in special circumstances. in the Concurrent List, the Central law
2. State Legislatures cannot make laws prevails over the state law. But, there is an
on those matters enumerated in the exception. If the state law has been reserved
Concurrent List on which a Central for the consideration of the president and has
legislation already exists. received his assent, then the state law prevails
Which of the above is/are correct? in that state. But, it would still be competent
(a)  1 only (b)  2 only for the Parliament to override such a law
(c)  Both 1 and 2 (d)  None by subsequently making a law on the same
Solution: (d) matter.
Justification: Statement 1: The Parliament
Q.479 Consider the following statements about state
has exclusive powers to make laws with
laws made in the concurrent list:
respect to any of the matters enumerated in
1. State laws in the concurrent list that
the Union List and Residuary list.
conflict with the Central law can still
Statement 2: Both, the Parliament and
prevail if it has received the President’s
state legislature can make laws with respect
assent.

P.124 For Civil Services Preliminary Examination

02-Indian Polity_Q413-927.indd 124 8/7/2018 7:40:33 PM


2. Even after the assent of the President, the States. Its purpose is to protect the powers of
Parliament can override such a law by a the States.
subsequent legislation on the same matter. In matters of concurrent list, both the
Which of these is/are correct? Parliament and State Legislatures can make
(a)  Only 1 (b)  Only 2 laws.
(c)  Both (d)  None of the above Therefore, any matter that affects the
Solution: (c) States must be referred to Rajya Sabha for its
Parliament has overriding powers over state consent and approval.
legislation normally too. This shows that the Thus, if the Union Parliament wishes to
India is a quasi-federal polity, not a federal remove a matter from the State list (over
polity. which only the State Legislature can make
Q.480 If State laws on subjects mentioned in the law) or to either the Union List or Concurrent
Concurrent List conflict with a Central Law, List in the interest of the nation, the approval
which of these follows? of the Rajya Sabha is necessary.
(a) The state law prevails over the Central law The same is true when matter need to be
if the State legislature passes it again with moved from Concurrent List to the Union
special majority. list.
(b) The Central law prevails over the State law. This provision adds to the strength of the
(c) The matter is moved to the Supreme Rajya Sabha.
Court which decides on the validity of
the respective laws in the larger national Executive Relations
interest.
Q.482 Consider the following statements about the
(d) The matter is referred to the Governor
administrative relations between the centre
whose decision shall be final and binding
and the states:
in this regard.
1. President’s rule can be imposed in the state
Solution: (b)
which has wilfully denied complying with
Justification: The Constitution provides a
the centre’s directions.
scheme for demarcation of powers through
2. The duty to execute the laws made under
three ‘lists’ in the seventh schedule.
the concurrent list by the Parliament lies
But, the Constitution also provides
with the centre unless provided otherwise.
primacy to Parliament on concurrent list
Which of these is/are correct?
items: if there is a conflict, a central law will
(a)  Only 1 (b)  Only 2
override a state law, i.e. the provisions of the
(c)  Both (d)  None of the above
state law will not take effect.
Solution: (a)
Q.481 If the Union Parliament wishes to move a Article 356 empowers the centre to impose
matter from Concurrent List to the Union President’s rule in case a state has failed to
List, which of these follows? comply with its directions. But if the state is
(a) It must obtain the consent of all State incapable to do the same on some genuine
Legislatures for the same. grounds, it can not be imposed. If done so,
(b) The Rajya Sabha must pass a resolution the courts can intervene.
to this effect approving the action of the The legislations under the concurrent list
Lok Sabha. enacted by the Parliament should be executed
(c) The President must consult the Governors by the states unless provided otherwise
of a majority of States and act according
Q.483 The Central Government can assign any
to their advice.
function to the States:
(d) The Cabinet must pass an executive fiat
(a)  on the directive of the President
to this effect.
(b) on the recommendation of Parliament
Solution: (b)
(c)  any time it wishes to do so
Learning: The Rajya Sabha is an institutional
(d) with the consent of the State Government
mechanism to provide representation to the

Indian Polity Question Bank P.125

02-Indian Polity_Q413-927.indd 125 8/7/2018 7:40:34 PM


Solution: (d) (a) A is correct, and R is an appropriate
The distribution of executive power in general explanation of A.
follows the distribution of legislative powers. (b) A is correct, but R is not an appropriate
But, such a rigid division in the executive explanation of A.
sphere may lead to occasional conflicts (c)  A is incorrect, but R is correct.
between the two. Hence, the Constitution (d)  Both A and R are incorrect.
provides for inter-government delegation Solution: (c)
of executive functions in order to mitigate Justification: Major ports are generally owned
rigidity and avoid a situation of deadlock. and managed by the Central government;
1. the President may, with the consent of minor ports, by the State government. So, A is
the state government, entrust to that anyways incorrect.
government any of the executive functions All major ports, except Ennore Port are
of the Centre. government administered. This also shows
2. Conversely, the governor of a state that A is incorrect.
may, with the consent of the Central Private sector participation in ports has
government, entrust to that government increased substantially. Many shipping
any of the executive functions of the companies are privately owned.
state. Entries 31 and 32 of the 7th Schedule
This mutual delegation of administrative (Concurrent List) read:
functions may be conditional or • Ports other than those declared by or
unconditional. under law made by Parliament or existing
The Constitution also makes a provision law to be major ports.
for the entrustment of the executive • Shipping and navigation on inland
functions of the Centre to a state without the waterways as regards with mechanically
consent of that state. But, in this case, the propelled vessels, and the rule of the road
delegation is by the Parliament and not by on such waterways, and the carriage of
the president. passengers and goods on inland waterways
Thus, a law made by the Parliament on a subject to the provisions of List I with
subject of the Union List can confer powers respect to national waterways.
and impose duties on a state, or authorise the • The above can be managed by both
conferring of powers and imposition of duties Central and State governments.
by the Centre upon a state (irrespective of
the consent of the state concerned). Notably, Financial Relations
the same thing cannot be done by the state
legislature. Q.485 The States in India can borrow from the
Thus mutual delegation of functions market
between the Centre and the state can take (a)  at their discretion
place either under an agreement or by the (b)  only through the Centre
legislation. While the Centre can use both the (c)  only with the consent of the Centre
methods, a state can use only the first method. (d)  under no circumstance
Solution: (c)
Q.484 Consider the following statements.
The Constitution makes the following
Assertion (A): All major ports in India
provisions with regard to the borrowing
are presently being administered or managed
powers of the Centre and the states:
by either the Central government or State
• The Central government can borrow either
governments.
within India or outside upon the security
Reason (R): As per the constitution,
of the Consolidated Fund of India or can
maritime transport can be administered by
give guarantees, but both within the limits
both the Central and the State governments.
fixed by the Parliament. So far, no such
In the context of the above, which of these
law has been enacted by the Parliament.
is correct?

P.126 For Civil Services Preliminary Examination

02-Indian Polity_Q413-927.indd 126 8/7/2018 7:40:34 PM


• Similarly, a state government can borrow Choose the correct answer using the
within India (and not abroad) upon the following codes:
security of the Consolidated Fund of the (a)  1 and 3 (b)  Only 2
State or can give guarantees, but both (c)  2 and 3 (d)  All of the above
within the limits fixed by the legislature Solution: (d)
of that state. Finance commission recommends the
• The Central government can make loans statutory grants and grants-in-aid to the
to any state or give guarantees in respect states. The role of states is limited in getting
of loans raised by any state. Any sums the desired grants sanctioned.
required for the purpose of making The CAG is integrated audit machinery
such loans are to be charged on the auditing both centre and states. CAG does
Consolidated Fund of India. not merely audits the accounts for legality
• A state cannot raise any loan without but also for performance of the government
the consent of the Centre, if there is still expenditure. The states have no power over
outstanding any part of a loan made to the the election and removal of CAG.
state by the Centre or in respect of which During the financial emergency the centre
a guarantee has been given by the Centre. becomes all powerful that constrains the
Q.486 A bill which imposes or varies any tax or duty fiscal space of states
in which states are interested can be introduced Q.488 Consider the following statements.
(a) Only on the recommendation of the Assertion (A): A state legislature cannot
President impose any taxes on the sale or purchase of
(b) Only with the consent of two or more states goods on its own.
(c) Only after Rajya Sabha passes a Reason (R): A state legislature needs the
resolution to that effect approval of the President for imposing any tax.
(d) Only on the recommendation of the In the context of the above, which of these
Finance Commission is correct?
Solution: (a) (a) A is correct, and R is an appropriate
To protect the interest of states in the explanation of A.
financial matters, the Constitution lays down (b) A is correct, but R is not an appropriate
that the following bills can be introduced in explanation of A.
the Parliament only on the recommendation (c)  A is incorrect, but R is correct.
of the President: (d)  Both A and R are incorrect.
1. A bill which imposes or varies any tax or Solution: (d)
duty in which states are interested; Justification: A state legislature can impose
2. A bill which varies the meaning of the taxes on professions, trades, callings and
expression ‘agricultural income’ as employments, sale or purchase of goods (other
defined for the purposes of the enactments than newspapers) etc. However, a tax imposed
relating to Indian income tax; on the sale or purchase of goods declared by
3. A bill which affects the principles on Parliament to be of special importance in
which moneys are or may be distributable inter-state trade and commerce is subject to
to states; and the restrictions and conditions specified by the
4. A bill which imposes any surcharge on Parliament. Presidential assent is not generally
any specified tax or duty for the purpose required for introduction of taxes by a state.
of the Centre. Q.489 The Constitution provides for specific grants
Q.487 Which of the following constitutional for promoting the welfare of the scheduled
mechanisms/provisions restrict the financial tribes in a state or for raising the level
autonomy of the states? of administration of the scheduled areas.
1. Finance Commission It comes under
2. The office of CAG (a)  Statutory grants
3. Financial emergency (b)  Discretionary Grants

Indian Polity Question Bank P.127

02-Indian Polity_Q413-927.indd 127 8/7/2018 7:40:34 PM


(c)  Backward area development grant Q.491 Grants-in-aid given to States are meant:
(d) Grants-in-aid (a)  to show favour to backward States
Solution: (a) (b) for use in centrally-sponsored schemes
Article 275 empowers the Parliament to (c) to cover gaps on revenue account so that
make grants to the states which are in need States can undertake beneficial activities
of financial assistance and not to every (d)  for funding the State plan
state. Also, different sums may be fixed for Solution: (c)
different states. These sums are charged on A grant-in-aid is money coming from
the Consolidated Fund of India every year. central government for a specific project.
Apart from this general provision, the This kind of funding is usually used when the
Constitution also provides for specific grants government and parliament have decided that
for promoting the welfare of the scheduled the recipient should be publicly funded but
tribes in a state or for raising the level of operated with reasonable independence from
administration of the scheduled areas in a the state. A grant-in-aid is funds allocated by
state including the State of Assam. one level of government to another level of
The statutory grants under Article 275 government to be used for specific purposes
(both general and specific) are given to the Q.492 The Constitution provides for a division of
states on the recommendation of the Finance taxation powers between Centre and States.
Commission. The Constitution also provided Among the several taxes, service tax is
for a third type of grants-in-aid (other than (a) Levied by the Centre but Collected and
Statutory and Discretionary), but for a Appropriated by the States
temporary period. Thus, a provision was (b) Levied by the Centre but Collected and
made for grants in lieu of export duties on Appropriated by the Centre and the States
jute and jute products to the States of Assam, (c) Levied, collected and retained by the Centre
Bihar, Orissa and West Bengal. These grants (d) Levied, collected and retained by the States
were to be given for a period of ten years from Solution: (b)
the commencement of the Constitution. These Service Tax Levied by the Centre but
sums were charged on the Consolidated Fund Collected and Appropriated by the Centre and
of India and were made to the states on the the States (Article 268-A): Taxes on services
recommendation of the Finance Commission. are levied by the Centre. But, their proceeds
Q.490 Grants from the Centre to the States under the are collected as well as appropriated by both
recommendations of Finance Commission the Centre and the states. The principles
are known as of their collection and appropriation are
(a)  Plan grants formulated by the Parliament.
(b)  Development assistance Taxes Levied and Collected by the Centre
(c)  Statutory grants but Assigned to the States (Article 269): The
(d)  Discretionary grants following taxes fall under this category:
Solution: (c) (i) Taxes on the sale or purchase of goods
Learning: Vertical and horizontal imbalances (other than newspapers) in the course of
are common features of most federations. The inter-state trade or commerce.
Constitution assigned taxes with a nation- (ii) Taxes on the consignment of goods in the
wide base to the Union to make the country course of inter-state trade or commerce
one common economic space unhindered by
internal barriers to the extent possible. States
being closer to people and more sensitive to
(14) INTERSTATE RELATIONS
the local needs have been assigned functional Q.493 The Constitution deals with which of the
responsibilities involving expenditure following?
disproportionate to their assigned sources of 1. Adjudication of inter-state water disputes
revenue resulting in vertical imbalances. The 2. Coordination between Centre and State
devolution of taxes from the centre helps them through inter-state councils
fulfil these responsibilities.
P.128 For Civil Services Preliminary Examination

02-Indian Polity_Q413-927.indd 128 8/7/2018 7:40:34 PM


3. Freedom of inter-state trade, commerce • Planning Commission of India (Now
and intercourse abolished) – NITI ayog
Select the correct answer using the codes • National Development Council
below. • National Integration Council
(a)  1 and 2 only (b)  3 only • Central Advisory Board of Education
(c)  2 and 3 only (d)  1, 2 and 3 • Central Council for Research in Ayurveda
Solution: (d) & Siddha
Justification: Statement 1: Article 262 of the • Central Council for Research in
Constitution provides for the adjudication Homoeopathy
of inter-state water disputes. It makes the • Central Council for Research in Yoga &
provision that Parliament may by law Naturopathy
provide for the adjudication of any dispute or • Labour Conference
complaint with respect to the use, distribution
and control of waters of any inter-state river Inter-State Water Disputes
and river valley.
Statement 2: Article 263 contemplates Q.495 Article 262 of the Constitution provides for
the establishment of an Inter-State Council the adjudication of inter-state water disputes
to effect coordination between the states and by a separate tribunal. The need for an extra
between Centre and states. judicial machinery to settle inter-state water
Statement 3: Articles 301 to 307 in disputes is because
Part XIII of the Constitution deal with the (a) The courts being overburdened with
trade, commerce and intercourse within the litigations and adjudicating slowly are
territory of India. incapable of adjudicating sensitive water
Article 301 declares that trade, commerce disputes
and intercourse throughout the territory of (b) water resources are not private property
India shall be free. and so rule of law is applied by ordinary
courts is not appropriate to deal with its
Q.494 Which of the following are extra-constitutional
distribution
devices to promote cooperation and
(c) Division and distribution of any natural
coordination between the Centre and the states?
resources is out of judicial scrutiny in
1. Zonal Councils
India
2. Inter-State Councils
(d) The courts cannot employ technical
3. North-Eastern Council
committees to ascertain the distribution
4. Central Council of Indian Medicine
of water resources
Select the correct answer using the codes below.
Solution: (b)
(a)  1 and 3 only
The Inter-State Water Disputes Act empowers
(b)  2 and 4 only
the Central government to set up an ad hoc
(c)  1, 3 and 4 only
tribunal for the adjudication of a dispute
(d)  1, 2, 3 and 4 only
between two or more states in relation to the
Solution: (c)
waters of an inter-state river or river valley.
Learning: Subject specific Inter-State
The decision of the tribunal would be final
Councils set up under Article 263 of the
and binding on the parties to the dispute.
Constitution of India
Neither the Supreme Court nor any other
• Central Council of Health
court is to have jurisdiction in respect of any
• Central Council for Local Government
water dispute which may be referred to such
and Urban Development
a tribunal under this Act.
• Regional Councils for Sales Tax and State
The need for an extra judicial machinery to
Excise Duties
settle inter-state water disputes is as follows:
Inter-State Coordination Mechanism set up
―The Supreme Court would indeed have
outside the framework of Article 263 of the
jurisdiction to decide any dispute between
Constitution of India
states in connection with water supplies, if

Indian Polity Question Bank P.129

02-Indian Polity_Q413-927.indd 129 8/7/2018 7:40:34 PM


legal rights or interests are concerned; but even the Parliament is not supreme or
the experience of most countries has shown sovereign here. Its authority is restricted by
that rules of law based upon the analogy of Fundamental rights and Judicial oversight.
private proprietary interests in water do not
afford a satisfactory basis for settling disputes Inter-State Council – Article 263
between the states where the interests of the
public at large in the proper use of water Q.498 With reference to the Inter State Councils,
supplies are involved. consider the following statements
1. A provision in the Constitution of India
Q.496 Consider these major inter-state water
governing formation of Inter State
disputes with some of their party states.
Councils was added later through an
1. Krishna Dispute: Tamil Nadu and Andhra
amendment
Pradesh
2. The genesis of the article can be traced
2. Cauvery Dispute: Karnataka and Tamil
directly to Section 135 of the Govt. of
Nadu
India Act, 1935 provided for establishment
3. Narmada Dispute: Rajasthan and Madhya
of Inter-Provincial Council with duties
Pradesh
identical with those of the Inter-State
Which of the above is\are correct matches?
Council
(a)  1 and 2 only (b)  2 and 3 only
Which of the above statements is/are correct?
(c)  1 and 3 only (d)  1, 2 and 3
(a)  1 Only (b)  2 Only
Solution: (b)
(c) Both (d) None
Learning: These are the major Inter-State
Solution: (b)
water disputes tribunal and the parties
concerned. Q.499 Consider the following about Inter-
1. Godavari Water Disputes Tribunal: State Council as under Article 263 of the
Maharashtra, Andhra Pradesh, Karnataka, constitution.
Madhya Pradesh and Odisha Assertion (A): It is not a permanent
2. Krishna Water Disputes Tribunal–I: constitutional body for coordination between
Maharashtra, Andhra Pradesh, Karnataka the States of the Union.
3. Narmada Water Disputes Tribunal: Reason (R): It is established by the
Rajasthan, Madhya Pradesh, Gujarat and President.
Maharashtra In the context of the above, which of these
4. Ravi and Beas Water Tribunal: Punjab, is correct?
Haryana and Rajasthan (a) A is correct, and R is an appropriate
5. Cauvery Water Disputes Tribunal: Kerala, explanation of A.
Karnataka, Tamil Nadu and Puducherry (b) A is correct, but R is not an appropriate
6. Krishna Water Disputes Tribunal–II: explanation of A.
Karnataka, Telangana, Andhra Pradesh (c)  A is correct, but R is incorrect.
and Maharashtra (d)  A is incorrect, but R is correct.
Solution: (b)
Q.497 The manner in which the Cauvery river water
Justification & Learning: As the article 263
dispute between Tamilnadu and Karnataka is
makes it clear, the Inter-State Council is not a
being resolved shows the
permanent constitutional body for coordination
(a)  Democratic nature of Indian state
between the States of the Union. It can be
(b)  Republican nature of Indian state
established ‘at any time’ if it appears to the
(c) Parliamentary sovereignty in the Indian
President that the public interests would be
state
served by the establishment of such a Council.
(d) Judicial sovereignty in the Indian state
The provision of article 263 of the
Solution: (a)
Constitution was invoked for the first time
There is no term like Judicial sovereignty. In
in 1952 when President by a notification
India no organ is supreme. There is separation
established the Central Council of Health
of powers between different organs. So

P.130 For Civil Services Preliminary Examination

02-Indian Polity_Q413-927.indd 130 8/7/2018 7:40:34 PM


under the Chairmanship of the Union Article 131 to decide a legal controversy
Minister of Health and Family Planning. between the governments.
Q.500 Consider the following about the Inter-State The Council can deal with any controversy
Council. whether legal or non-legal, but its function is
1. It is not a permanent constitutional body. advisory unlike that of the court which gives a
2. It is chaired by the Prime Minister. binding decision.
3. It does not include the representatives of Q.502 Consider the following about the inter-state
the Union Territories (UTs). council.
4. It includes the Presiding officers of both 1. Chief Ministers of all the states are its
houses of Parliament. members.
Select the correct answer using the codes 2. Five Ministers nominated by the Prime
below. Minister are permanent invitees to the
(a)  1 and 2 only (b)  2 and 4 only Council.
(c)  1 and 3 only (d)  2, 3 and 4 only 3. The council is a recommendatory body.
Solution: (a) 4. The Council meets once biannually.
Justification: It is a recommendatory body Select the correct answer using the codes
to investigate and discuss subjects, in which below.
some or all of the states or the Central (a)  1, 2 and 3 only (b)  2 and 4 only
government have a common interest. It is set (c)  1, 3 and 4 only (d)  1, 2, 3 and 4 only
up on the basis of provisions in Article 263 of Solution: (a)
the Constitution of India. Learning: The council is a recommendatory
Statement 1: The Inter-state council is body on issues relating to inter-state, Centre–
not a permanent constitutional body for state and Centre–union territories relations. It
coordination between the states and Central aims at promoting coordination between them
government. Rather, President can establish by examining, discussing and deliberating on
it at any time if it appears to him that the such issues. Its duties, in detail, are as follows:
public interests would be served by the • Investigating and discussing such subjects
establishment of such a council. in which the states or the centre have a
Statement 2, 3 and 4: It facilitates common interest;
coordination between states and the centre. • making recommendations upon any such
The Inter State Council composes of the subject for the better coordination of
following members: policy and action on it; and
(a)  Prime Minister, Chairman. • Deliberating upon such other matters of
(b) Chief Ministers of states and union general interest to the states as may be
territories. referred to it by the chairman.
(c)  Administrators of union territories. The Council may meet at least thrice in a
(d)  Six members of Cabinet rank. year. Its meetings are held in camera and all
Q.501 Who among the following is/are member(s) questions are decided by consensus.
of the Inter-State Council?
(a)  President of India Freedom of Trade, Commerce
(b)  Six members of Cabinet rank
(c) Governors of states and union territories
and Intercourse – Part XIII –
(d)  Chief Justice of India Articles 301 to 307
Solution: (b)
Justification: composition of the council is Q.503 Consider the following with reference to the
given above. Freedom of trade, commerce and intercourse
Learning: Being a constitutional body, within India.
the council’s function to enquire and advice 1. The constitution gives Parliament the
upon inter-state disputes is complementary power to restrict free trade between and
to the Supreme Court’s jurisdiction under within states on grounds of public interest.

Indian Polity Question Bank P.131

02-Indian Polity_Q413-927.indd 131 8/7/2018 7:40:34 PM


2. Ordinarily, no taxes can be applied to the five zonal councils viz. Northern, Western,
goods originating in another state that are Eastern, Central and Southern.
also not applied on goods produced within The north eastern states are not included
a state. in these five zonal Councils. Their special
Which of the above is/are correct? problems are looked after by North Eastern
(a)  1 only (b)  2 only Council, set up under the North Eastern
(c)  Both 1 and 2 (d)  None Council Act, 1972. It is also headed by
Solution: (c) Union Home Minister. The North Eastern
Justification: The constitution states that Council includes Assam, Arunachal Pradesh,
“Subject to the other provisions, trade, Manipur, Tripura, Mizoram, Meghalaya and
commerce and intercourse throughout the Nagaland. Sikkim was included in the North
territory of India shall be free.” Eastern Council in 2002 and excluded from
Articles 302-304 both qualify and the eastern council.
elaborate on that principle. Learning: The Zonal Councils are mandated
• Article 302 gives Parliament the power to discuss and make recommendations on any
to restrict free trade between and within matter of common interest in field of social
states on grounds of public interest. and economic planning, linguistic minorities,
• Article 303 (a) then imposes a most- border disputes or inter-State transport etc.
favoured nation type obligation on both Thus, the Zonal Councils are regional
Parliament and state legislatures; that is forums of cooperative endeavour for States
no law or Regulation powers of the Union linked with each other economically,
and of the States with regard to trade and culturally and politically.
commerce can give arbitrary preference to Q.505 Consider the following about Zonal Councils.
one state over another. 1. They were established by States
• Article 304 (a) then imposes a national Reorganisation Act.
treatment-type obligation on state 2. Each zonal council is headed by the Chief
legislatures (apparently not on Parliament); Minister of concerned states in rotation.
that is, no taxes can be applied to the goods 3. Zonal councils are statutorily authorized
originating in another state that are also not to approve development plans prepared
applied on goods produced within a state. by Gram Sabhas and Urban Local bodies.
• This Article refers only to taxes and not to Select the correct answer using the codes
regulations more broadly. below.
(a)  1 only (b)  2 and 3 only
Zonal Councils (c)  1 and 3 only (d)  1, 2 and 3
Solution: (a)
Q.504 Consider the following about Zonal Councils.
Justification: Statement 1: The act divided
1. They are Constitutional bodies as setup
the country into five zones (Northern,
under Schedule VII of the Constitution.
Central, Eastern, Western and Southern) and
2. The north eastern states are not included
provided a zonal council for each zone.
in these zonal Councils.
Statement 2: The home minister of Central
Which of the above is/are correct?
government is the common chairman of the
(a)  1 only (b)  2 only
five zonal councils. Each chief minister acts
(c)  Both 1 and 2 (d)  None
as a vice-chairman of the council by rotation,
Solution: (b)
holding office for a period of one year at
Justification: Statement 1: They were set
a time.
up under the States Reorganization Act,
Statement 3: They sort out coordination
1956. So, they are statutory bodies, not
issues between states and plan for holistic
constitutional bodies.
development of these zones which shares
Statement 2: They are setup to foster
many factors in common such as the natural
Inter-State co-operation and co-ordination
divisions, means of communication, the
among the States. Currently, there are total

P.132 For Civil Services Preliminary Examination

02-Indian Polity_Q413-927.indd 132 8/7/2018 7:40:34 PM


cultural and linguistic affinity and the 3. A Parliamentary resolution approving
requirements of economic development, National Emergency needs to be passed
security and law and order. They, however, by a simple majority, whereas a special
don’t approve bottom-up plans prepared by majority in case of President’s rule.
PRIs and ULBs. So, option 3 is wrong. Select the correct answer using the codes
Learning: Each zonal council consists of the below.
following members: (a)  1 only (b)  2 and 3 only
• home minister of Central government (c)  1 and 3 only (d)  2 only
• chief ministers of all the States in the zone Solution: (a)
• Two other ministers from each state in the
zone National Emergency
• Administrator of each union territory in
the zone. Q.508 Under Article 352, the President can declare
a national emergency when
Q.506 Which of these bodies is/are chaired by the
(a) The states fail to obey Centre’s directions.
Chief Ministers of member states on rotation
(b) There is large scale corruption and mal-
basis?
governance.
1. Inter-State Council
(c) The security of India or a part of is
2. Zonal Councils
threatened by an imminent danger.
3. NITI Aayog Governing Council
(d) She can declare it in all of the above
Select the correct answer using the codes
situations.
below.
Solution: (c)
(a)  1 and 2 only (b)  2 only
Justification: Option (a) refers to President’s
(c)  1 and 3 only (d)  None of the above
rule, not National Emergency. Even
Solution: (d)
President’s rule cannot be applied in case
Justification: Statement 1: He is a member
of (b), as per a SC judgment. The President
of the Inter-State Council chaired by the
can declare a national emergency when the
prime minister.
security of India or a part of it is threatened by
Statement 2: He acts as a “vice-chairman”
war or external aggression or armed rebellion.
(not Chairman) of the concerned zonal
The president can declare a national
council by rotation, holding office for a period
emergency even before the actual occurrence
of one year at a time. Union home minister is
of war or external aggression or armed
the chairman of all the zonal councils.
rebellion, if he is satisfied that there is an
Statement 3: The Governing council too is
imminent danger.
chaired by the Prime Minister, and not CMs
on a rotation basis. Q.509 Consider the following statements.
1. National emergency can be declared even
if security of India is not in threat, but
(15) EMERGENCY PROVISIONS –  there is a case of imminent danger.
PART XVIII – ARTICLE 352 TO 360 2. The operation of National Emergency
always applies to the whole of Indian
Q.507 Which of these is/are major points of Territory.
differences between National emergency and Which of the above is/are correct?
President’s rule? (a)  1 only (b)  2 only
1. State governments are automatically (c)  Both 1 and 2 (d)  None
dismissed on application of President’s Solution: (a)
Rule, which may not be the case in a Under Article 352, the President can declare
National Emergency. a national emergency when the security of
2. There is no maximum period prescribed India or a part of it is threatened by war or
for the operations of President’s rule, external aggression or armed rebellion. It
unlike National emergency which ceases may be noted that the president can declare
after an year.

Indian Polity Question Bank P.133

02-Indian Polity_Q413-927.indd 133 8/7/2018 7:40:34 PM


a national emergency even before the actual Solution: (d)
occurrence of war or external aggression or The President can proclaim a national
armed rebellion, if he is satisfied that there is emergency only after receiving a written
an imminent danger. recommendation from the cabinet. This
A proclamation of national emergency means that the emergency can be declared
may be applicable to the entire country or only on the concurrence of the cabinet and not
only a part of it. The 42nd Amendment Act merely on the advice of the prime minister. In
of 1976 enabled the president to limit the 1975, the then Prime Minister, Indira Gandhi
operation of a National Emergency to a advised the president to proclaim emergency
specified part of India. without consulting her cabinet. The cabinet
Q.510 The President of India can proclaim National was informed of the proclamation after it was
Emergency under which of the following made, as a fait accompli. The 44th Amendment
conditions? Act of 1978 introduced this safeguard to
(a) Report of rampant corruption in States eliminate any possibility of the prime minister
(b) Armed invasion by another nation in alone taking a decision in this regard.
India Approval of the Parliament is required
(c) Loss of lives due to a natural calamity later to decide whether to continue the
(d) Failure to form a government at the emergency or not.
Centre Q.512 The President can proclaim a national
Solution: (b) emergency when
Learning: National emergency can be (a) Cabinet gives a written recommendation
declared under Article 352 on the basis of to the President
external aggression or armed rebellion in the (b) The Parliament passes a resolution by
whole of India or a part of its territory. Such simple majority to this effect
an emergency was declared in India in 1962 (c) The President is individually satisfied
(Indo-China war), 1971 (Indo-Pakistan war), that it is in the best national interest
and 1975 (declared by Indira Gandhi). (d) The National Security Council
The President can declare such an recommends it to the President
emergency only on the basis of a written Solution: (a)
request by the Council of Ministers headed Learning: The emergency can be declared
by the Prime Minister. Such a proclamation only on the concurrence of the cabinet
must be laid before both houses of and not merely even on the advice of the
Parliament, and the state of emergency prime minister or Home Minister. This
expires after one month unless approved safeguardwas added by 44th constitutional
within that time by both houses sitting and amendment.
voting separately. The proclamation of Emergency should be
It is not declared in case of epidemics, approved by both the Houses of Parliament
natural calamities or failure to form within one month from the date of its issue.
government at the Centre which makes all So, (b)  is wrong.
options other than (b) wrong. Q.513 The 44th constitutional amendment brought
Q.511 The President of India can declare a National some changes in the powers of Prime
emergency only after the Minister in declaring emergency. What was
1. Approval of the Cabinet the change brought?
2. Approval by the Parliament (a) A national emergency can be declared by
3. Approval by the State legislatures the President only on the concurrence of
involved the cabinet and not merely on the advice
Choose the correct answer using the codes of the prime minister
below. (b) The President can declare national
(a)  1 and 2 only (b)  2 and 3 only emergency only after the concurrence of
(c)  1 and 3 only (d)  1 only the cabinet and Parliament

P.134 For Civil Services Preliminary Examination

02-Indian Polity_Q413-927.indd 134 8/7/2018 7:40:34 PM


(c) If a house was not in session, only then must be passed by either House of Parliament
the President could declare national by a special majority, that is,
emergency on the advice of the Prime 1. The majority of the total membership of
Minister that house
(d) The Prime Minister could only advice 2. The majority of not less than two-thirds
declaring internal emergency to the of the members of that house present and
President; for external emergency the voting.
concurrence of the Parliament was needed A proclamation of emergency may be revoked
Solution: (a) by the President at any time by a subsequent
Q.514 The President can proclaim a national proclamation. Such a proclamation does not
emergency when require the parliamentary approval.
(a) Cabinet gives a written recommendation Q.516 National Emergency has been declared so
to the President far:
(b) The Parliament passes a resolution by (a) once (b) twice
simple majority to this effect (c) thrice (d) four times
(c) The President is individually satisfied that Solution: (c)
it is in the best national interest
(d) The National Security Council Effects of National Emergency on
recommends it to the President
Solution: (a)
Centre-State Relations
Learning: The emergency can be declared Q.517 During an emergency, our political structure
only on the concurrence of the cabinet and undergoes a transformation from being
not merely even on the advice of the prime “Federal‟ to
minister or Home Minister. This safeguardwas (a) Military
added by 44th constitutional amendment. (b) Quasi-federal
The proclamation of Emergency should be (c) Unitary
approved by both the Houses of Parliament (d)  Being only a Single Government
within one month from the date of its issue. Solution: (c)
So, (b)  is wrong. During an Emergency, the Central government
Q.515 Consider the following statements. becomes all powerful and the states go into
Assertion (A): Resolutions approving the the total control of the Centre. It converts the
Proclamation of Emergency are passed only federal structure into a unitary one without a
by the Lok Sabha. formal amendment of the Constitution. This
Reason (R): The Rajya Sabha does not kind of transformation of the political system
represent directly elected representatives. from federal during normal times to unitary
In the context of the above, which of these during Emergency is a unique feature of the
is correct? Indian Constitution.
(a) A is correct, and R is an appropriate Since the state governments do exist even
explanation of A. at the time of emergency, we can not call it―
(b) A is correct, but R is not an appropriate single government.
explanation of A. Q.518 During a national emergency
(c)  A is incorrect, but R is correct. 1. State governments are suspended and
(d)  Both A and R are incorrect. Centre takes the control of the states.
Solution: (c) 2. Parliament becomes empowered to make
Justification: The proclamation of laws on any subject mentioned in the State
Emergency must be approved by both the List.
Houses of Parliament within one month from Which of the above is/are correct?
the date of its issue. (a)  1 only (b)  2 only
Every resolution approving the (c)  Both 1 and 2 (d)  None
proclamation of emergency or its continuance Solution: (b)

Indian Polity Question Bank P.135

02-Indian Polity_Q413-927.indd 135 8/7/2018 7:40:34 PM


Justification: Statement 1: The Centre (b) A is correct, but R is not an appropriate
becomes entitled to give executive directions explanation of A.
to a state on ‘any’ matter. Thus, the state (c)  A is correct, but R is incorrect.
governments are brought under the complete (d)  A is incorrect, but R is correct.
control of the Centre, though they are not Solution: (c)
suspended. Justification: Article 359 authorises the
Statement 2: Although the legislative President to suspend the right to constitutional
power of a state legislature is not suspended, remedies for the enforcement of Fundamental
it becomes subject to the overriding power of Rights during a National Emergency.
the Parliament. Thus, the normal distribution This means that under Article 359,
of the legislative powers between the Centre the Fundamental Rights as such are not
and states is suspended, though the state suspended, but only their enforcement. The
legislatures are not suspended. In brief, the rights are theoretically alive but the right to
Constitution becomes unitary rather than seek remedy is suspended.
federal. The suspension of enforcement relates
Q.519 Consider the following statements: to only those Fundamental Rights that are
Assertion (A): A national emergency specified in the Presidential Order. It does not
converts the federal structure into a unitary apply to all fundamental rights. So, R is wrong
one without a formal amendment of the Q.521 Which of the following rights would be
Constitution. suspended if a National Emergency is
Reason (R): During an Emergency, the proclaimed by the Government of India?
Central government becomes all powerful 1. Right to freedom of speech and expression
and the states go into the total control of the 2. Right to equality of opportunity in matters
Centre. of public employment.
In the context of the statements above, 3. Right to protection in respect of conviction
which of these is true? for offences.
(a) A and R both are true, and R is the correct Select the correct answer using the codes
explanation for A. below
(b) A and R both are true, and R is the NOT (a)  Only 1 (b)  1 and 2
the correct explanation for A. (c)  2 and 3 (d)  None of the above
(c)  A is correct, R is incorrect. Solution: (b)
(d)  A and R both are incorrect. Rights under Article 20 and 21 remain
Solution: (a) unaffected by the imposition of national
emergency. Other rights may or may not be
Effect of National Emergency on suspended depending on the decision of the
government.
Fundamental Rights
Q.520 Consider the following statements. President’s Rule
Assertion (A): The constitution authorises
the President to suspend the right to move Q.522 During President’s rule in a state
any court for the enforcement of certain 1. State council of ministers is dissolved
Fundamental Rights during a National 2. Central government takes over the
Emergency. executive machinery of the state
Reason (R): All fundamental rights are 3. The President becomes empowered to
automatically suspended with a proclamation make laws with respect to any matter in
of National Emergency. state list
In the context of the above, which of these Choose the correct answer using the codes
is correct? below.
(a) A is correct, and R is an appropriate (a)  1 and 2 only (b)  2 and 3 only
explanation of A. (c)  1 and 3 only (d)  1 only
Solution: (a)

P.136 For Civil Services Preliminary Examination

02-Indian Polity_Q413-927.indd 136 8/7/2018 7:40:34 PM


When the President’s rule is imposed in a Select the correct answer using the codes
state, the Parliament becomes empowered below.
to make laws with respect to any matter in (a)  1 and 2 only (b)  2 only
the State List in relation to that state. A law (c)  1 and 3 only (d)  3 only
made so by the Parliament continues to be Solution: (b)
operative even after the president’s rule. Justification: Statement 1: The assembly is
All the decisions concerning the state immediately dissolved, and the President takes
emanate from the President based on the the administrative charge of the state. A panel of
advice given by Council of Ministers. advisors is appointed to advise the Governor in
The state legislature is also either the discharge of his day to day duties.
suspended or dissolved. Statement 2: A law made so by the
Q.523 Consider the following about President’s Parliament continues to be operative even
rule that is imposed under Article 356 of the after the president’s rule.
Constitution. But, such a law can be repealed or altered
1. It cannot be imposed without the written or re-enacted by the state legislature.
recommendation of the Governor of the Statement 3: There is no such provision.
concerned state. Q.525 State emergency or President’s rule can be
2. Every proclamation of President’s rule applied to an Indian state on which of the
must be approved by both the houses of following grounds?
Parliament within a stipulated time. 1. Failure to comply with Centre’s directions
Which of the above is/are correct? 2. Breakdown of constitutional machinery in
(a)  1 only (b)  2 only the state
(c)  Both 1 and 2 (d)  None 3. Threat to the financial stability of the state
Solution: (b) 4. Internal aggression in the state
Learning: Statement 1: Article 356 empowers Choose the correct answer using the codes
the President to issue a proclamation, if he is below.
satisfied that a situation has arisen in which (a)  1 and 2 only (b)  2 and 4 only
the government of a state cannot be carried (c)  1, 3 and 4 only (d)  1, 2 and 3 only
on in accordance with the provisions of the Solution: (a)
Constitution. The Indian Constitution contains elaborate
Notably, the president can act either on a emergency provisions to enable the
report of the governor of the state or otherwise President to meet any extraordinary
too (i.e., even without the governor’s report). situation effectively. The rationality behind
Statement 2: A proclamation imposing the incorporation of these provisions is to
President’s Rule must be approved by both safeguard the sovereignty, unity, integrity
the Houses of Parliament within two months and security of the country, the democratic
from the date of its issue. If approved by both political system and the Constitution.
the Houses of Parliament, the President’s The Constitution envisages three types of
Rule continues for six months emergencies, namely:
Q.524 Consider the following statements about the (a) National emergency on the ground of war
effects of President’s Rule in states. or external aggression or armed rebellion
1. The state legislative assembly continues to (Article 352);
hold sessions but the civilian government (b) State emergency (President’s Rule) on
loses all authority. the ground of failure of Constitutional
2. The Parliament becomes empowered to machinery in the states (Article 356) or
make laws with respect to any matter in failure to comply with the directions of
the State List in relation to that state. the Centre (Article 365); and
3. The state immediately loses all funding (c) Financial emergency on the ground of
support from the Union government. threat to the financial stability or credit of
India (Article 360).

Indian Polity Question Bank P.137

02-Indian Polity_Q413-927.indd 137 8/7/2018 7:40:34 PM


Q.526 Consider the following statements about review only when President acts against
President’s rule: the advice of the Council of Ministers.
1. There is no maximum time limit for both 3. The state legislative assembly should be
National emergency and President’s rule if dissolved only after the Parliament has
continuously approved by the Parliament. approved the presidential proclamation.
2. President’s rule can be imposed in a Select the correct answer using the codes
state which is proved to have engaged in below.
maladministration in the state. (a)  1 and 2 only (b)  2 and 3 only
Which of these is/are true? (c)  1 and 3 only (d)  1, 2 and 3
(a)  Only 1 (b)  Only 2 Solution: (c)
(c)  Both (d)  None of the above Justification: Statement 1: Secularism is one
Solution: (d) of the ‘basic features’ of the Constitution.
While National emergency has no time limit, Hence, a state government pursuing anti-
President’s rule has to be revoked after a secular politics is liable to action under
maximum of three years. Article 356.
Further the 44th Amendment Act of 1978 Statement 2: It can be subject to Judicial
introduced a new provision to put restraint review in any condition. Further, the
on the power of Parliament to extend a satisfaction of the President must be based on
proclamation of President’s Rule beyond one relevant material. The action of the president
year. Thus, it provided that, beyond one year, can be struck down by the court if it is based
the President’s Rule can be extended by six on irrelevant or extraneous grounds or if it
months at a time only when the following was found to be mala fide or perverse.
two conditions are fulfilled: Burden lies on the Centre to prove
1. A proclamation of National Emergency that relevant material exist to justify the
should be in operation in the whole of India, imposition of the President’s Rule.
or in the whole or any part of the state Statement 3: The state legislative assembly
2. Election Commission must certify that should be dissolved only after the Parliament
the general elections to the legislative has approved the presidential proclamation.
assembly of the concerned state cannot be Until such approval is given, the president
held on account of difficulties. can only suspend the assembly. In case the
A proclamation of President’s Rule may Parliament fails to approve the proclamation,
be revoked by the President at any time the assembly would get reactivated.
by a subsequent proclamation. Such If the court holds the presidential
a proclamation does not require the proclamation to be unconstitutional and
parliamentary approval. invalid, it has power to restore the dismissed
Proved maladministration or even state government and revive the state
corruption can not be a ground to impose legislative assembly if it was suspended or
president’s rule as per the guidelines laid dissolved.
down by the Supreme Court in the S. R.
Bommai v UOI, 1994 AIR 1918. Article 355
Q.527 In the S. R. Bommai v UOI, 1994 AIR 1918.,
Q.528 The Central government has a constitutional
several propositions have been laid down
duty to protect states against
by the Supreme Court on imposition of
1. External aggression and internal
President’s Rule in a state under Article 356.
disturbance
These include
2. Failure of Constitutional machinery
1. A state government pursuing anti-
Which of the above is/are correct?
secular politics is liable to action under
(a)  1 only (b)  2 only
Article 356.
(c)  Both 1 and 2 (d)  None
2. The presidential proclamation imposing
Solution: (c)
President’s Rule is subject to judicial

P.138 For Civil Services Preliminary Examination

02-Indian Polity_Q413-927.indd 138 8/7/2018 7:40:34 PM


Justification: Statement 1: This is a provision The Constitution contains the following
under Article 355 of the constitution. It says, other provisions which enable the Centre to
“It shall be the duty of the Union to protect exercise control over the state administration:
every State against external aggression and (i) 
Article 355 imposes two duties on the
internal disturbance and to ensure that the Centre: (a) to protect every state against
government of every State is carried on external aggression and internal disturbance;
in accordance with the provisions of this and (b) to ensure that the government of
Constitution.” every state is carried on in accordance with
Statement 2: President’s Rule is applied the provisions of the Constitution.
under this case under Article 356 of the (ii) The governor of a state is appointed by
Constitution. the president. He holds office during the
It can also be applied when the state pleasure of the President. In addition to
government has wilfully disobeyed the the Constitutional head of the state, the
directions of the Central government and governor acts as an agent of the Centre in
disturbed the constitutional distribution of the state. He submits periodical reports to
executive power. the Centre about the administrative affairs
Q.529 The provision that “It is the duty of the of the state.
Union to protect the states against external (iii) The state election commissioner, though
aggression and internal disturbance” is a appointed by the governor of the state,
(a)  Constitutional provision can be removed only by the President.
(b)  Supreme Court Judgment Q.531 Article 355 of the Constitution of India is an
(c)  Parliamentary Enactment important instrument that enables the Centre
(d) Convention followed in a federal polity to exercise control over State administration.
Solution: (a) It imposes which of the following duties on
Learning: As per Article 355, it is the duty the Centre?
of the Union to protect States against external 1. To protect every state against external
aggression and internal disturbance. aggression and internal disturbance
“It shall be the duty of the Union to protect 2. To protect the financial integrity of the
every State against external aggression and states
internal disturbance and to ensure that the 3. To ensure that the government of every
government of every State is carried on state is carried on in accordance with the
in accordance with the provisions of this provisions of the Constitution
Constitution” 4. To resolve conflicts between States
While Article 356 has been repeatedly through negotiations or binding executive
used and misused on flimsy grounds, the orders
Centre has seldom resorted to Article 355, Select the correct answer using the codes
because, though it serves as a precursor to below.
Article 356, it comes with an onerous burden. (a)  1 and 3 only (b)  2, 3 and 4 only
Q.530 Which of the following duties have been (c)  4 only (d)  1, 3 and 4 only
constitutionally prescribed for the Centre to Solution: (a)
exercise control over the state administration? Q.532 Article 355 of the Indian Constitution provides
1. To protect every state against external that ‘It shall be the duty of the Union to protect
aggression and internal disturbance every State against external aggression and
2. To ensure that corruption and internal disturbance’. This shows the
maladministration do not affect state (a)  Federal character of the Indian
governments constitution
Which of the above is/are true? (b) Quasi-federal Character of Indian polity
(a)  1 only (b)  2 only (c) Democratic character of Indian polity
(c)  Both 1 and 2 (d)  None (d) Unity and territorial integrity of India.
Solution: (a) Solution: (b)

Indian Polity Question Bank P.139

02-Indian Polity_Q413-927.indd 139 8/7/2018 7:40:34 PM


Article 355 shows that the Union government approved by the resolutions of both
has been given prominence over the state Houses of Parliament.
government. This shows the federal character 2. If any proclamation of financial emergency
with a Union bias i.e. quasi-federal nature is in operation. it is competent for the
of the Indian polity. The Union government President of India to issue directions for
has a responsibility to ensure the territorial the reduction of salaries and allowances
integrity and security of the nation. of all or any class of persons serving in
Q.533 Concerning the Central government, which connection with the affairs of the Union
of the following are constitutional mandates/ but excluding the Judges of the Supreme
duties? Court and the High Courts.
1. To devolve a share of income proceeds to Which of the statements given above is/are
states correct? [IAS 2007]
2. To protect states from constitutional (a)  1 only (b)  2 only
breakdown (c)  Both 1 and 2 (d)  Neither 1 nor 2
3. To protect states from external and Solution: (a)
internal aggression Q.535 Which of the following is/are the financial
Choose the correct answer using the codes powers of the President of India?
below. 1. Money bills can be introduced in
(a)  1 and 2 only (b)  2 and 3 only the Parliament only with his prior
(c)  1 and 3 only (d)  All of the above recommendation.
Solution: (d) 2. He causes to be laid before the Parliament
Article 355: Duty of the Union to protect the Union Budget.
States against external aggression and 3. No demand for a grant can be made except
internal disturbance - It shall be the duty on his recommendation.
of the Union to protect every State against 4. He can make advances out of the
external aggression and internal disturbance contingency fund of India to meet any
and to ensure that the government of every unforeseen expenditure.
State is carried on in accordance with the 5. He constitutes the Union finance
provisions of this Constitution. commission to recommend the distribution
Article 365: Article 365 says that where of revenues between the Centre and the
any state has failed to comply with (or to states.
give effect to) any directions given by the Select the correct answer using the codes
Centre, it will be lawful for the President to below.
hold that a situation has arisen in which the (a)  3 and 5 only (b)  1, 2 and 5 only
government of the state cannot be carried (c)  1, 2 and 4 only (d)  1, 2, 3, 4 and 5
on in accordance with the provisions of Solution: (d)
the Constitution. It means that, in such Justification: Statement 1 and 3: Both
a situation, the President’s rule can be statements are correct. More on these lines
imposed in the state under Article 356. will be covered in the coming tests.
Statement 2: President shall, in respect of
Financial Emergency every financial year, cause to be laid before
Parliament, Annual Financial Statement
Q.534 Consider the following statements in respect
as per Article 112 of the Constitution. The
of financial emergency under Article 360 of
Budget is presented to Parliament on such
the Constitution of India:
date as is fixed by the President.
1. A proclamation of financial emergency
Statement 4: The fund is held by the Finance
issued shall cease to operate at the
Secretary (Department of Economic Affairs)
expiration of 2 months, unless before
on behalf of the President of India and it can
the expiration of that period it has been
be operated by executive action.

P.140 For Civil Services Preliminary Examination

02-Indian Polity_Q413-927.indd 140 8/7/2018 7:40:34 PM


Q.536 When a financial emergency is proclaimed: CENTRAL GOVERNMENT
(a) the repayment of government debts will
stop (PART V) (ARTICLES 51–151)
(b) the payment of salaries to public servants
will be postponed (16) PRESIDENT – ARTICLES 52–78
(c) the salaries and allowances of any class
of employees may be reduced Q.538 Consider the following statements.
(d)  the Union Budget will not be presented 1. Parliament may, by law, regulate any
Solution: (b) matter relating to or connected with the
Q.537 The provisions regarding division of taxes election of a President or a Vice-President.
between Union and the States: 2. All doubts and disputes arising out of the
(a) can be suspended during National election of a President or Vice-President
Emergency shall be inquired into and decided by the
(b) can be suspended during Financial Election Commission of India whose
Emergency decision shall be final.
(c) can be suspended only with the consent Which of the above is/are correct?
of the majority of State legislatures (a)  1 only (b)  2 only
(d) cannot be suspended under’ any (c)  Both 1 and 2 (d)  None
circumstances Solution: (a)
Solution: (c) Justification: Statement 1: This is however
subject to the provisions of the constitution.
Presidential and Vice-Presidential Elections
Act, 1952 was enacted in this context.
Statement 2: As per Article 71 of the
constitution, it shall be inquired into and
decided by the Supreme Court whose
decision shall be final. The ECI may be called
for advice.
Q.539 The highest formal authority in India is
carried by
(a)  Prime Minister of India
(b)  President of India
(c)  Cabinet Secretary
(d) Heads of independent Constitutional
bodies
Solution: (b)
Explanation: President is the head of the
state and is the highest formal authority in
the country. All executive actions of the
government are taken in his name.
Prime Minister is the head of the
government and actually exercises all
governmental powers. He takes most of the
decisions in the Cabinet meetings. So, option
(b) is correct.
Learning: As per Article 53 of the
Constitution
(a) The executive power of the Union shall
be vested in the President and shall
be exercised by him either directly or

Indian Polity Question Bank P.141

02-Indian Polity_Q413-927.indd 141 8/7/2018 7:40:34 PM


through officers subordinates to him in Justification: Statement 1: Dr. Rajendra
accordance with this Constitution. Prasad has served for two terms (1950-1962),
(b) Without prejudice to the generality of and apart from him none has served more
the foregoing provision, the supreme than once. So, 1 is correct.
command of the Defence Forces of the Statement 2: So far two Presidents, Dr. Zakir
Union shall be vested in the President and Hussain and Fakhruddin Ali Ahmed, have
the exercise there of shall be regulated by died during their term of office. So, 2 is
law. incorrect.
Q.540 Consider the following with regard to the For example, when President Dr. Zakir
President of India. Hussain died in May 1969, the then Vice-
1. The President does not exercise his/her President, V.V. Giri was acting as the
discretion on the advice given by the President.
Council of Ministers. Soon after V.V. Giri resigned to contest
2. Only elected members of the Legislative the election of the President. Then the Chief
Assemblies and both the Houses of Justice of India, M Hidayatullah worked as
the Parliament take part in electing the the officiating President.
President.
3. The President can send the bill back to the Presidential Elections
Parliament asking it to reconsider the bill.
4. The President’s ordinance making power Electoral college
is not a discretionary power.
Which of the above statements is/are correct? Q.542 The President of India is elected by members
(a)  1 and 4 Only (b)  1, 2 and 3 Only of an electoral college consisting of
(c)  2 and 3 Only (d)  2, 3 and 4 Only 1. Elected members of Lok Sabha
Solution: (d) 2. Elected members of Legislative
The President can send back the advice given Assemblies of states
by the Council of Ministers and ask the 3. Elected members of Rajya Sabha
Council to reconsider the decision. In doing 4. Elected members of Legislative Councils
this, the President acts on his (or her) own of states
discretion. When the President thinks that Choose the correct answer using the codes
the advice has certain flaws or legal lacunae, below
or that it is not in the best interests of the (a)  1 and 2 Only (b)  1, 2 and 3 Only
country, the President can ask the Council (c)  1, 2 and 4 Only (d)  All the four
to reconsider the decision. Although, the Solution: (b)
Council can still send back the same advice Article 54 of the Constitution says:
and the President would then be bound by “The President shall be elected by the
that advice, such a request by the President to members of an electoral college consisting
reconsider the decision, would naturally carry of -
a lot of weight. So, this is one way in which (a) The elected members of both Houses of
the president can act in his own discretion. Parliament and
(b) The elected members of the Legislative
Q.541 Consider the following statements about the Assemblies of the States (including
office of President of India. National Capital Territory of Delhi and
1. No person has occupied the office for the Union Territory of Pondicherry vide
more than two complete terms. the Constitution 70th amendment Act,
2. Every President has served the full term of 1992).”
the office. Thus in the election of the President the
Which of the above is/are correct? citizens play no direct part and he is elected
(a)  1 only (b)  2 only indirectly by the representatives or the
(c)  Both 1 and 2 (d)  None people, like the American President but
Solution: (a)

P.142 For Civil Services Preliminary Examination

02-Indian Polity_Q413-927.indd 142 8/7/2018 7:40:34 PM


no special electoral college is elected, as Q.545 The Indian President, head of the State, is not
in the case of America. Another point of elected directly by the people of India. He is
difference that may be noted is that the elected by members of an electoral college
election of the President of India is by the consisting of
system of proportional representation, by 1. Former Presidents
the single transferable vote, as provided by 2. Bharat Ratna Awardees
Article 55(3) of the Constitution, while the 3. All Members of Parliament (MPs)
American President is elected by the straight 4. All Members of Legislative Assemblies of
vote system. all states (MLAs)
Q.543 Which of the following Union Territories 5. All Members of Legislative Councils of
participate in the election of the President? states where applicable (MLCs)
1. Delhi Select the correct answer using the codes
2. Puducherry below.
3. Chandigarh (a)  3 and 4 only (b)  1, 2 and 3 only
Choose the correct answer from the codes (c)  3, 4 and 5 only (d)  None of the above
below. Solution: (d)
(a)  1 and 2 only (b)  2 and 3 only Justification: Statements 1 and 2: They are
(c)  1 and 3 only (d)  None of the above not included in the collegium.
Solution: (a) Statement 3 and 4: Only elected members
The composition of electoral college is given participate. Nominated members do not.
in the previous question. The rationale is that nominated members are
chosen by the incumbent President. If they
Q.544 The nominated members of both Lok Sabha
participate in the election of the President, it
and Rajya Sabha do NOT participate in
confers an unfair advantage to the incumbent.
which of the following?
Statement 5: MLCs do not participate in
1. Election of President
the election.
2. Passing of Constitutional Amendment
Bill Q.546 Which of the following take part in the
3. Impeachment of President election of the President of India?
4. Election of Vice-President 1. All the members of Lok Sabha
Choose the correct answer from the codes 2. All the members of Rajya Sabha
below. 3. All the members of Legislative Assembly
(a)  1 and 4 only (b)  2 and 3 only 4. All the members of Legislative Councils
(c)  1 and 3 only (d)  1 only Choose the correct answer using the codes
Solution: (d) below:
The nominated members of both of Houses (a)  All of the above
of Parliament, the nominated members of (b)  None of the above
the state legislative assemblies, the members (c)  1 and 3 only
(both elected and nominated) of the state (d)  1, 2 and 3 only
legislative councils (in case of the bicameral Solution: (b)
legislature) and the nominated members Q.547 In which of the following case(s), the Chief
of the Legislative Assemblies of Delhi and Minister of a State in India is NOT eligible to
Puducherry do not participate in the election vote in the Presidential election?
of the President. Where an assembly is 1. If he himself is a candidate
dissolved, the members cease to be qualified 2. If he is yet to prove his majority on the
to vote in presidential election, even if fresh floor of the Lower House of the State
elections to the dissolved assembly are not legislature
held before the presidential election. 3. If he is a member of the Upper House of
While electing and impeaching the State legislature
Vice-President, nominated members 4. If he is a caretaker Chief Minister
participate.

Indian Polity Question Bank P.143

02-Indian Polity_Q413-927.indd 143 8/7/2018 7:40:35 PM


Choose the correct answer using the codes 1. Every elected member of the Legislative
below: Assembly of a state shall have as many
(a)  1 and 2 only (b)  1, 2 and 3 votes as there are multiples of one
(c)  Only 3 (d)  2 and 3 thousand in the quotient obtained by
Solution: (c) dividing the population of the state by the
The upper houses in the states do not take total number of the elected members of
part in Presidential election. Therefore, if the assembly. This can be expressed as:
the CM is a member of the upper house, he Value of the vote of an MLA
cannot vote.
Total population of state    1
= ×
Total number of elected     100
Election Method members in the state legislative assembly
Q.548 Consider the following statements.
2. Every elected member of either House
Assertion (A): All doubts and disputes in
of Parliament shall have such number of
connection with election of the President are
votes as may be obtained by dividing the
inquired into and decided by the Election
total number of votes assigned to members
Commission of India.
of legislative assemblies of the states by
Reason (R): Election Commission of India
the total number of the elected members
is entrusted with the responsibility of
of both the Houses of Parliament. This
conducting the Presidential elections.
can be expressed as:
In the context of the above, which of these
Value of the vote of an MP =
is correct?
(a) A is correct, and R is an appropriate Total value of voted of all MLAs
 of all the states
explanation of A.
= Total number of elected members
(b) A is correct, but R is not an appropriate
explanation of A. of Parliament
(c)  A is correct, but R is incorrect.
Q.550 Consider the following statements about the
(d)  A is incorrect, but R is correct.
election of the President of India:
Solution: (d)
1. The total values of the votes of all the
Justification: Supreme Court decides all
MLAs is equal to that of all the MPs.
the doubts and disputes in connection with
2. All doubts and disputes regarding the
election of the President even though ECI
election of the President is enquired into
organizes and conducts the elections. So, A is
by the Election Commission of India.
incorrect, and R is correct.
Which of these is /are true?
Moreover, ECI has only limited quasi-
(a)  Only 1 (b)  Only 2
judicial authority with regard to elections
(c) Both (d) None
of MPs or MLAs. We will be covering these
Solution: (d)
topics in the coming tests.
The total value of the votes of all the MLAs
Q.549 Consider the following statements regarding is equal to that of all the elected MPs only
the election of President. which excludes the nominated ones.
1. The value of vote of a MLA is equivalent to All doubts and disputes regarding the
that of a MLC casting vote in Presidential election of the President is enquired into by
election. the Supreme Court of India considering the
2. The value of vote of a MLA is equivalent high prestige of the office of President of
to that of a MP casting vote in Presidential India.
election.
Q.551 In the election of the President, every elected
Which of the above is/are true?
member of either House of Parliament shall
(a)  1 only (b)  2 only
have such number of votes as may be obtained
(c)  Both 1 and 2 (d)  None
by dividing the
Solution: (d)

P.144 For Civil Services Preliminary Examination

02-Indian Polity_Q413-927.indd 144 8/7/2018 7:40:35 PM


(a) Total number of votes assigned to MLAs • It does not include the members of the
of the states by the total number of the state legislative assemblies (in the case
elected MPs of President, the elected members of the
(b) Total population of India by the total state legislative assemblies are included).
number of seats in all the State legislative
assemblies Qualification
(c) Total votes assigned to the President by
the total number of seats in Parliament Q.554 Consider the following statements.
(d) Total strength of Parliament by the 1. In India, only a citizen by birth and not a
number of nominations filed for the naturalised citizen is eligible for the office
President’s office of President.
Solution: (a) 2. In USA, a citizen by birth as well as a
The President’s election is held in accordance naturalised citizen is eligible for the office
with the system of proportional representation of President.
and single transferrable vote. Which of the above is/are correct?
(a)  1 only (b)  2 only
Q.552 In the election of the President, each Member
(c)  Both 1 and 2 (d)  None
of the electoral college has:
Solution: (d)
(a)  one vote
Justification: Statement 1: A person not born
(b) as many votes as there are candidates
in India, but having acquired citizenship by
(c)  one vote with value attached to it
registration or by naturalisation can become
(d) one vote with value attached to it and he
the PM or the President of India. The Indian
can give as many preferences as there are
constitution does not differentiate on the basis
candidates
of the method of acquiring of citizenship.
Solution: (d)
Statement 2: This is however not the
Q.553 What is the difference in the electoral college case with the USA constitution, which
of the Vice-President and the President of categorically denies the right to President
India? Candidacy to naturalized citizens.
1. Both the houses take part in the election of
Q.555 Consider the following statements.
President, unlike the Vice-President who
Assertion (A): A person not born in India but
is elected by the Rajya Sabha alone.
having acquired citizenship by registration
2. State legislative assemblies do not take
or by naturalisation cannot become the
part in the election of the Vice-President,
President of India.
unlike that of the President.
Reason (R): President of India is the head
Which of the above is/are correct?
of the state and represents the Indian state
(a)  1 only (b)  2 only
symbolically.
(c)  Both 1 and 2 (d)  None
In the context of the above, which of these
Solution: (b)
is correct?
Justification: Statement 1: The Vice-
(a) A is correct, and R is an appropriate
President, like the president, is elected not
explanation of A.
directly by the people but by the method
(b) A is correct, but R is not an appropriate
of indirect election. He is elected by the
explanation of A.
members of an electoral college consisting of
(c)  A is correct, but R is incorrect.
the members of both Houses of Parliament.
(d)  A is incorrect, but R is correct.
Statement 2: His Electoral College is
Solution: (d)
different from the Electoral College for the
election of the President in following two
respects. Oath
• It consists of both elected and nominated Q.556 Which of the following forms part of the oath
members of the Parliament (in the case of of the President of India but not that of the
president, only elected members). Members of Parliament (MPs)?

Indian Polity Question Bank P.145

02-Indian Polity_Q413-927.indd 145 8/7/2018 7:40:35 PM


1. To preserve, protect and defend the Solution: (c)
constitution and the law. Learning: The oath of office to the President
2. To uphold the sovereignty and integrity of is administered by the Chief Justice of India
India. and in his absence, the senior most judge of
Choose the correct answer using the the Supreme Court available.
following codes: Any other person acting as President or
(a)  Only 1 (b)  Only 2 discharging the functions of the President also
(c)  Both (d)  None of the above undertakes the similar oath or affirmation.
Solution: (a) Q.559 Before entering upon his office, the President
In his oath, the President swears: has to make and subscribe to an oath or
• to faithfully execute the office; affirmation. Which of the following is/are
• to preserve, protect and defend the part of the oath or affirmation made by the
Constitution and the law; and President?
• to devote himself to the service and well- 1. To preserve, protect and defend the
being of the people of India. Constitution and the law
Where as the oath of MPs is 2. To uphold the sovereignty and integrity of
1. to bear true faith and allegiance to the India
Constitution of India; 3. Perform the duties of the office without
2. to uphold the sovereignty and integrity of fear or favour
India; and Select the correct answer using the codes
3. to faithfully discharge the duty upon below.
which he is about to enter. (a)  1 and 2 only (b)  2 and 3 only
Unless a member takes the oath, he cannot (c)  1 only (d)  1, 2 and 3
vote and participate in the proceedings of Solution: (c)
the House and does not become eligible to Oath in Statement 2 is taken by the judges
parliamentary privileges and immunities. of Supreme Court and High Courts, and Oath
Q.557 At the time of appointment, the oath to in Statement 3 is taken by the information
“preserve, protect and defend the constitution” commissioners, judges of Supreme Court and
is subscribed by which of the following: High Courts etc.
1. President.
2. Governor. Conditions of Office
3. Chief Justice of India
Choose the correct answer using the codes Q.560 During the tenure of the Indian President, he
below: is immune from which of the following?
(a)  1 and 2 (b)  1 and 3 1. Criminal proceedings
(c)  2 and 3 (d)  All of the above 2. Civil Proceedings
Solution: (a) 3. Arrest and Imprisonment
Only the president and governors are to take Choose the correct answer from the codes
the oath to defend and protect the constitution. below.
The Chief Justice of India and other (a)  1 and 2 only (b)  2 and 3 only
judges uphold the constitution and the laws. (c)  1 and 3 only (d)  All of the above
Solution: (d)
Q.558 Before entering upon his office, the President
The President is entitled to a number of
has to make and subscribe to an oath or
privileges and immunities. He enjoys personal
affirmation. This oath is administered to the
immunity from legal liability for his official
President by
acts. During his term of office, he is immune
(a)  Prime Minister of India
from any criminal proceedings, even in
(b) The predecessor President of India who
respect of his personal acts. He cannot be
immediately vacated office
arrested or imprisoned. However, after giving
(c)  Chief Justice of India
two months’ notice, civil proceedings can be
(d)  Vice-President of India

P.146 For Civil Services Preliminary Examination

02-Indian Polity_Q413-927.indd 146 8/7/2018 7:40:35 PM


instituted against him during his term of office 2. the elected members of the legislative
in respect of his personal acts. assemblies of states and the Union
Territories of Delhi and Puducherry do
Impeachment not participate in the impeachment of the
President though they participate in his
Q.561 Consider the following statements about the election
Impeachment of the Indian President. No President has so far been impeached.
1. Impeachment can only be initiated by Lok
Q.562 The “Violation of Constitution” as a
Sabha.
ground of removal is explicitly mentioned
2. Supreme Court is the authority that
for which of the following offices under the
investigates against the charges levelled
constitution of India?
against the President.
1. President 2.  Prime Minister
3. Representatives of Union Territories in
3. Governor 4.  Chief Justice of India
the Parliament do not participate in the
Choose the correct answer using the codes
impeachment process.
below:
Choose the correct answer from the codes
(a)  1, 2 and 3 (b)  1, 3 and 4
below.
(c)  Only 1 (d)  All of the above
(a)  2 only (b)  2 and 3 only
Solution: (c)
(c)  1 and 3 only (d)  None of the above
Impeachment of president is taken up by the
Solution: (d)
house in cases of violation of the constitution.
The President can be removed from office by
For others, grounds are either different or
a process of impeachment for ‘violation of
not mentioned explicitly as “violation of the
the Constitution’. However, the Constitution
constitution”.
does not define the meaning of the phrase
‘violation of the Constitution’. Q.563 Consider the following about the
The impeachment charges can be initiated impeachment of President of India.
by either House of Parliament. These charges 1. The impeachment charges can be initiated
should be signed by one-fourth members by either House of Parliament.
of the House (that framed the charges), and 2. The Parliament investigates the
a 14 days’ notice should be given to the impeachment charges.
President. After the impeachment resolution 3. No President has so far been impeached.
is passed by a majority of two-thirds of the Select the correct answer using the codes
total membership of that House, it is sent to below.
the other House, which should investigate (a)  1 and 2 only (b)  2 and 3 only
the charges. (c)  1 only (d)  1, 2 and 3
The President has the right to appear Solution: (d)
and to be represented at such investigation. Justification: Statement 1: Impeachment is
If the other House also sustains the charges a quasi-judicial procedure in the Parliament.
and passes the impeachment resolution These charges should be signed by one-
by a majority of two-thirds of the total fourth members of the House (that framed
membership, then the President stands the charges).
removed from his office from the date on Statement 2: After the impeachment
which the bill is so passed. resolution is passed by a majority of two-
Thus, an impeachment is a quasi-judicial thirds of the total membership of that House,
procedure in the Parliament. In this context, it is sent to the other House, which should
two things should be noted: investigate the charges.
1. the nominated members of either House The President has the right to appear and
of Parliament can participate in the to be represented at such investigation.
impeachment of the President though they If the other House also sustains the
do not participate in his election charges and passes the impeachment

Indian Polity Question Bank P.147

02-Indian Polity_Q413-927.indd 147 8/7/2018 7:40:35 PM


resolution by a majority of two-thirds of the and advise the President in the exercise of his
total membership, then the President stands functions. The advice so tendered is binding
removed from his office from the date on on the President. The unamended constitution
which the bill is so passed. had some scope of Presidential discretion
Q.564 Consider the following statements about the where he was not explicitly bound by the aid
impeachment of the President of India: and advice of the council of Ministers.
1. Impeachment proceedings can only be
started by Lok Sabha first. Powers and Functions of the President
2. Nominated members of Lok sabha
take part in the impeachment of the (1) Executive Powers
President.
3. The Supreme Court investigates the Q.567 All executive actions of the Government of
charges labelled against the President for India are formally taken in name of president
impeachment. of India. Consider the following with
Which of these is /are true? this reference.
(a)  Only 1 (b)  Only 2 1. He can make rules specifying the manner
(c)  1 and 3 (d)  Only 3 in which the orders and other instruments
Solution: (b) made and executed in his name shall be
All the MPs whether elected or nominated authenticated only after concurrence from
take part in the election of the President. Parliament.
Impeachment proceedings can be started 2. He can make rules for more convenient
by either house of the Parliament. The transaction of business of the Union
concerned house investigates the charges, not government, and for allocation of the said
the Supreme Court of India business among the ministers.
Which of the above is/are correct?
Q.565 If a resolution impeaching the President is (a)  1 only (b)  2 only
passed, the President is considered to have (c)  Both 1 and 2 (d)  None
been removed: Solution: (b)
(a) from the date on which the resolution is Justification: The above are some of the
passed executive powers and functions of the
(b) once the Chief Justice of India takes out President.
an order to the effect Statement 1: He does not require the
(c) as soon as the Gazette of India notifies it consent or concurrence of the Parliament in
(d)  once the new incumbent is elected making these rules, since these are executive
Solution: (a) fiats.
Statement 2: Orders and other instruments
Vacancy in President’s Office made and executed in the name of the
Q.566 The President is bound by the aid and advice President shall be authenticated in such
tendered by the Council of Ministers. This manner as may be specified in rules to be
provision is made by the President.
(a) Mandated by Representation of People of Some of the other powers are:
India Act • He appoints the prime minister and the
(b) Followed as a convention of other ministers. They hold office during
Parliamentary form of Government his pleasure.
(c) An executive precedent followed since • He appoints the attorney general of India
Independence and determines his remuneration. The
(d) Mandated by a Constitutional amendment attorney general holds office during the
Solution: (d) pleasure of the President.
Learning: Article 74 provides for a council of • He appoints the comptroller and auditor
ministers headed by the Prime Minister to aid general of India, the chief election

P.148 For Civil Services Preliminary Examination

02-Indian Polity_Q413-927.indd 148 8/7/2018 7:40:35 PM


commissioner and other election 3. The rules for the allocation of business of
commissioners, the chairman and the Government of India are made by the
members of the Union Public Service President.
Commission, the governors of states, Select the correct answer using the codes
the chairman and members of finance below.
commission, and so on. (a)  1 and 2 only (b)  3 only
• He can seek any information relating to (c)  2 and 3 only (d)  1, 2 and 3
the administration of affairs of the Union, Solution: (d)
and proposals for legislation from the Q.570 Consider the following statements about the
prime minister. President of India:
• He can require the Prime Minister to 1. All governmental activities take place in
submit, for consideration of the council of the name of the President.
ministers, any matter on which a decision 2. All laws and major policy decisions of
has been taken by a minister but, which the government are issued in the name of
has not been considered by the council. President.
Q.568 Merger of two Ministries/departments of the 3. All major appointments are made in the
Union Government can be done by name of the President.
(a)  A constitutional amendment only Choose the correct answer using the codes
(b)  A statutory provision only below:
(c) The President of India by an executive (a)  All of the above (b)  1 and 3 only
order (c)  1 and 2 only (d)  2 and 3 only
(d) The Cabinet Secretary without needing Solution: (a)
authorization from the Prime Minister These appointments include the Chief Justice
Solution: (c) of India, the Judges of the Supreme Court and
Learning: The Govt. of India (Allocation the High Courts of the states, the Governors
of Business) Rules, 1961 are made by the of the states, the Election Commissioners,
President of India under Article 77 of the ambassadors to other countries, etc.
Constitution for the allocation of business of All international treaties and agreements
the Govt. of India. The Ministries/Departments are made in the name of the President. The
of the Govt. are created (or merged, President is the supreme commander of the
dissolved)  by the President on the advice of defence forces of India.
the Prime Minister under these Rules. But we should remember that the President
The Cabinet Secretariat is under the exercises all these powers only on the advice
direct charge of the Prime Minister. The of the Council of Ministers.
administrative head of the Secretariat is the Q.571 The President places the report of which of
Cabinet Secretary who is also the ex-officio the following bodies before the Parliament?
Chairman of the Civil Services Board. 1. Union Public Service Commission
The Cabinet Secretariat is responsible for 2. Comptroller and Auditor General of India
the administration of the Govt. of India 3. National Commission for STs
Rules, 1961. 4. National Human Rights Commission
Q.569 Consider the following statements about Select the correct answer using the codes
the administrative setup of the Government below.
of India. (a)  1 and 2 only (b)  3 and 4 only
1. The Ministries/Departments of the (c)  1, 2 and 4 only (d)  1, 2, 3 and 4 only
Government are created by the President Solution: (d)
on the advice of the Prime Minister. Q.572 Who among the following are appointed by
2. Each of the Ministries is assigned to a the President of India ?
Minister by the President on the advice of 1. The Chairman, Finance Commission.
the Prime Minister. 2. The Auditor general of India
3. The Chief Minister of a Union Territory.

Indian Polity Question Bank P.149

02-Indian Polity_Q413-927.indd 149 8/7/2018 7:40:35 PM


Choose the correct answer from the codes 1. Governor
given below: 2. Chief Election Commissioner (CEC)
(a)  all (b)  1 and 2 only 3. Comptroller and Auditor General of India
(c)  1 and 3 only (d)  2 and 3 only (CAG)
Solution: (a) Select the correct answer using the codes
The CM of the states are appointed by the below.
Governors concerned. But in a UT, they are (a)  1 only (b)  2 and 3 only
appointed by the President. The Administrator (c)  1 and 2 only (d)  1, 2 and 3
of the UT forwards the recommendation of Solution: (a)
the appointment to the President. Justification: Statement 1: He can be
He appoints the prime minister and the removed by the President at any time without
other ministers. They hold office during citing reasons for the same. He is an agent of
his pleasure. the Centre and does not enjoy any security
He appoints the attorney general of of tenure.
India and determines his remuneration. The Statement 2: The chief election
attorney general holds office during the commissioner is provided with the security
pleasure of the President. of tenure. He cannot be removed from his
He appoints the comptroller and office except in same manner and on the same
auditor general of India, the chief grounds as a judge of the Supreme Court.
election commissioner and other election In other words, he can be removed by the
commissioners, the chairman and members president on the basis of a resolution passed to
of the Union Public Service Commission, the that effect by both the Houses of Parliament
governors of states, the chairman and members with special majority, either on the ground of
of finance commission, and so on. He can seek proved misbehaviour or incapacity.
any information relating to the administration Thus, he does not hold his office till
of affairs of the Union, and proposals for the pleasure of the president, though he is
legislation from the prime minister. appointed by him.
Q.573 The President appoints various Ministers and Statement 3: Same is true of CAG.
decides their portfolio on the advice of the
(a)  Prime Minister (2) Legislative Powers
(b) Vice-President
Q.575 The President of India is an integral part of
(c)  Speaker, Lok Sabha
the Parliament. One of the reasons for it is that
(d) He makes the decision independently.
(a) He has a constitutional right to attend the
Solution: (a)
meetings of the Parliament.
Learning: Formally, a leader who has the
(b) A bill passed by both the Houses of
support of the majority is appointed by the
Parliament cannot become law without
President as Prime Minister.
the President’s assent.
The Prime Minister then decides who will
(c) He has the constitutional authority to both
be the ministers in the Council of Ministers.
constitute and dissolve the Parliament.
The Prime Minister allocates ranks and
(d) He is the head of the executive branch
portfolios to the ministers. Depending upon
of the Government in our Parliamentary
the seniority and political importance, the
democracy.
ministers are given the ranks of cabinet
Solution: (b)
minister, minister of State or deputy
Though the President of India is not
minister.
a member of either House of Parliament
In the same manner, Chief Ministers of
and does not sit in the Parliament to attend
the States choose ministers from their own
its meetings, he is an integral part of the
party or coalition.
Parliament. This is because a bill passed by
Q.574 Which of these constitutional functionaries both the Houses of Parliament cannot become
holds office till the pleasure of the President? law without the President’s assent.

P.150 For Civil Services Preliminary Examination

02-Indian Polity_Q413-927.indd 150 8/7/2018 7:40:35 PM


He also performs certain functions Chairman of Rajya Sabha, thus statement
relating to the proceedings of the Parliament, 3 is also wrong. A bill is referred to a Joint
for example, he summons and prorogues Parliamentary Committee for examining
both the Houses, dissolves the Lok Sabha, controversial provisions in the bill or to
addresses both the Houses, issues ordinances give insights on the bill to make it better. Its
when they are not in session, and so on. recommendations are not binding. Statement
In this respect, the framers of the Indian 2 is wrong.
Constitution relied on the British pattern Learning: The Parliament consists of two
rather than the American pattern. In Britain, Houses and President of India. The two
the Parliament consists of the Crown (King or Houses are known as the Council of States
Queen), the House of Lords (Upper House) (Rajya Sabha) and the House of the People
and the House of Commons (Lower House). (Lok Sabha). The President of India is a
By contrast, the American president is not part of the Parliament, although she is not a
an integral part of the legislature. In USA, member of either House. That is why all laws
the legislature, which is known as Congress, made in the Houses come into force only
consists of the Senate (Upper House) and the after they receive the assent of the President.
House of Representatives (Lower House). Q.578 Consider the following statements.
Q.576 Consider the following statements. Assertion (A): The Prime Minister is obliged
Assertion (A): Though the President of to furnish all the information that the
India is not a member of either House of President may call for.
Parliament, he is an integral part of the Reason (R): Constitutionally, the President
institution of Parliament. has a right to be informed of all important
Reason (R): A bill passed by both the matters and deliberations of the Council of
Houses of Parliament cannot become law Ministers.
without the President’s assent. In the context of the above, which of these
In the context of the above, which of these is correct?
is correct? (a) A is correct, and R is an appropriate
(a) A is correct, and R is an appropriate explanation of A.
explanation of A. (b) A is correct, but R is not an appropriate
(b) A is correct, but R is not an appropriate explanation of A.
explanation of A. (c)  A is correct, but R is incorrect.
(c)  A is correct, but R is incorrect. (d)  Both A and R are incorrect.
(d)  A is incorrect, but R is correct. Solution: (a)
Solution: (a) Justification: The President often writes to
Q.577 To become a law, all bills passed by the Lok the Prime Minister and expresses his views
Sabha and Rajya Sabha must be approved by on matters confronting the country. Here, the
1. Speaker, Lok Sabha PM is bound to share the information that the
2. Joint Parliamentary Committee President has called for.
3. Vice-President In doing so, the President exercises his
4. President situational discretion.
Select the correct answer using the codes Learning: Moreover, the President can send
below. back the advice given by the Council of
(a)  1 and 3 only (b)  2 only Ministers and ask the Council to reconsider
(c)  4 only (d)  3 and 4 only the decision.
Solution: (c) • In doing this, the President acts on her
Explanation: Speaker of Lok Sabha only has own discretion. When the President
to endorse certain money bills when they are thinks that the advice has certain flaws or
sent to the President. He need not give assent legal lacunae, or that it is not in the best
to a bill to get it passed. Statement 1 is wrong. interests of the country, the President can
Same is true for Vice-President, ask the Council to reconsider the decision.

Indian Polity Question Bank P.151

02-Indian Polity_Q413-927.indd 151 8/7/2018 7:40:35 PM


• Although, the Council can still send back 2. He can address the Parliament at the
the same advice and the President would commencement of the first session after
then be bound by that advice, such a each general election and the first session
request by the President to reconsider of each year.
the decision, would naturally carry a lot 3. He can send messages to the Houses of
of weight. Parliament, whether with respect to a bill
Q.579 Consider the following statements. pending in the Parliament or otherwise.
Assertion (A): Our Constitution mandates 4. He can appoint any member of the Lok
that either the President or one of his official Sabha to preside over its proceedings
representatives must attend all Parliamentary when the offices of both the Speaker and
proceedings. the Deputy Speaker fall vacant. Similarly,
Reason (R): The President forms an integral he can also appoint any member of
part of the Indian Parliament. the Rajya Sabha to preside over its
In the context of the above, which of these proceedings when the offices of both
is correct? the Chairman and the Deputy Chairman
(a) A is correct, and R is an appropriate fall vacant.
explanation of A. Q.581 Which of the functions relating to the
(b) A is correct, but R is not an appropriate Parliament are performed by the President?
explanation of A. 1. Summoning the Houses
(c)  A is correct, but R is incorrect. 2. Proroguing the Houses
(d)  A is incorrect, but R is correct. 3. Adjournment Sine dine
Solution: (d) 4. Dissolving lower house
Justification: The Parliament consists of the 5. Addressing both houses
President, Lok Sabha and Rajya Sabha. Choose the correct answer using the codes
So, the President forms an integral part of below.
the Parliament, since no bill can become an (a)  1, 4 and 5 only
act without her consent. (b)  2 and 3 only
But, the President is not required to attend (c)  1, 2, 4 and 5 only
the sessions of Parliament at all. He only (d)  All of the above
presents welcome address which is passed as Solution: (c)
motion of thanks every time a new house is As explained in the previous question.
constituted. Q.582 As per the constitution, the Indian president
Q.580 The President is an integral part of the has the power to
Parliament of India. Which of the following 1. Ask for files relating to any matter from
are done by the President, with reference to Prime Minister that is in deliberation in
the sessions of the Parliament? the Council of Ministers
1. Summoning 2. Send the decision of an individual ministry
2. Proroguing for re-consideration
3. Dissolving Lok Sabha 3. Suspend the council of ministers if he
Choose the correct answer from the codes feels it is violating constitutional norms
below. Select the correct answer using the codes
(a)  1 and 2 only (b)  2 and 3 only below:
(c)  1 and 3 only (d)  All of the above (a)  1 and 2 only (b)  2 and 3 only
Solution: (d) (c)  1 and 3 only (d)  All of the above
He enjoys the following legislative powers. Solution: (a)
1. He can summon or prorogue the Parliament If an individual ministry has taken the
and dissolve the Lok Sabha. He can also decision, which has not been considered by
summon a joint sitting of both the Houses the cabinet, the president can send the file for
of Parliament, which is presided over by re-consideration.
the Speaker of the Lok Sabha.

P.152 For Civil Services Preliminary Examination

02-Indian Polity_Q413-927.indd 152 8/7/2018 7:40:35 PM


The 42nd Constitutional Amendment 3. The council of ministers must advice
Act of 1976 (enacted by the Indira Gandhi the President to do so.
Government) made the President bound by Q.584 At the commencement of the first session
the advice of the council of ministers headed after each general election to the House of
by the prime minister. the People and at the commencement of the
The 44th Constitutional Amendment first session of each year the President shall
Act of 1978 (enacted by the Janata Party address both Houses of Parliament assembled
Government headed by Shri Morarji Desai) together and inform Parliament of the causes
authorised the President to require the council of its summons.” This is a
of ministers to reconsider such advice either (a)  Constitutional provision
generally or otherwise. (b)  Statutory provisions
However, he ‘shall’ act in accordance (c)  Rules of Procedure of the House
with the advice tendered after such (d)  Parliamentary Convention
reconsideration. In other words, the President Solution: (a)
may return a matter once for reconsideration Learning: Article 87(1) of the Constitution
of his ministers, but the reconsidered advice provides for this. This is the case of the first
shall be binding. session after each general election to the
Also, the president can not just ask for the Lok Sabha, the President addresses both
files, but also ask details about the decisions Houses of Parliament assembled together
that are to be taken in the council of ministers. after the Members have made and subscribed
However, he cannot suspend it, if it enjoys the oath or affirmation and the Speaker has
confidence in the Lok Sabha. been elected.
Q.583 If Parliament has repeatedly failed to perform It takes generally 2 days to complete these
its duty, in the opinion of the President, then preliminaries. No other business is transacted
as per his constitutional mandate he can tilt the President has addressed both Houses of
1. Dissolve the Lok Sabha Parliament assembled together and informed
2. Dismiss the Council of Ministers Parliament of the Government’s agenda.
3. Promulgate ordinance to keep the Q.585 Consider the following about the powers of
legislative business on track the President of India.
Choose the correct answer using the codes 1. He can appoint an inter-state council
below. to promote Centre–state and inter-state
(a)  1 and 2 only (b)  2 and 3 only cooperation.
(c)  3 only (d)  None of the above 2. He directly administers the union
Solution: (d) territories through administrators
The President’s powers with respect to the appointed by him.
Parliament are restricted by: 3. He can declare an area as scheduled
• The council of Ministers can be dismissed area and has powers with respect to the
only when it has lost the confidence of the administration of scheduled areas and
house. tribal areas.
• Lok Sabha can be dissolved only when no Select the correct answer using the codes
government can be formed. below.
• And, there are only three grounds to issue (a)  1 and 2 only (b)  2 and 3 only
an ordinance: (c)  1 and 3 only (d)  1, 2 and 3
1. The parliament should not be in Solution: (d)
session, whether one or both the houses Learning: Apart from the above functions,
2. The subject must be very urgent that he also has the following powers:
any delay in passing such a bill may be • He can seek any information relating to
against public interest, like in the recent the administration of affairs of the Union,
case of Criminal law (amendment) and proposals for legislation from the
ordinance, 2013. prime minister.

Indian Polity Question Bank P.153

02-Indian Polity_Q413-927.indd 153 8/7/2018 7:40:35 PM


• He can require the Prime Minister to require the recommendation of the President
submit, for consideration of the council of for introduction.
ministers, any matter on which a decision Other bills are that require prior
has been taken by a minister but, which permission are
has not been considered by the council. 1. Money Bill (as per Article 110) and
• He can appoint a commission to investigate Finance Bill
into the conditions of SCs, STs and other 2. Any bill which affects the taxation in
backward classes. which the states are interested (Article
Q.586 The president nominates 12 members to the 274)
Rajya Sabha from 3. State Bills which impose restriction upon
(a) 
The persons recommended by the freedom of trade (Article 304).
National Integration Council 4. Constitutional amendment bills require
(b) 
People who have special knowledge or prior Presidential assent.
practical experience in art, literature, Q.588 Without the prior approval of the President
science and social service. of India, which of the following can NOT be
(c) 
People who have contributed to done?
immensely to Indian politics. 1. Abolition of a certain tax by the Parliament
(d) 
Eminent political scientists who have 2. Introduction of State reorganization bill in
never contested an election the Parliament
Solution: (b) 3. Introduction of money bill in the
Learning: The rationale behind this principle Parliament.
of nomination is to provide eminent persons Choose the correct answer using the codes
a place in the Rajya Sabha without going below.
through the process of election. (a)  1 and 2 only (b)  2 and 3 only
It should be noted here that the American (c)  1 and 3 only (d)  All of the above
Senate has no nominated members. Solution: (b)
Article 3 lays down two conditions in the
Bills Requiring Prior Permission of event of state’s reorganization: one, a bill
contemplating the above changes can be
President introduced in the Parliament only with the
Q.587 Some Bills require the prior permission/ prior recommendation of the President;
consent/recommendation of the President and two, before recommending the bill, the
to be introduced. However, such President has to refer the same to the state
recommendations are not is required in which legislature concerned for expressing its views
of the following legislative cases? within a specified period. The Constitution
(a) For changing the official language of the lays down a special procedure for the passing
Parliament of money bills in the Parliament. A money
(b) For alteration of areas of the existing bill can only be introduced in the Lok Sabha
States and that too on the recommendation of the
(c) Making provision for the reduction or president. Every such bill is considered to be
abolition of any tax a government bill and can be introduced only
(d) All the above cases require prior by a minister.
Presidential assent Q.589 The prior recommendation of the President is
Solution: (a) needed to introduce which of these bills in
Justification: According to Article 3 of the the Parliament?
Constitution of India, the bills relating to 1. A bill involving expenditure from the
Admission or establishment of new States, Consolidated Fund of India
formation of new States, and alteration of 2. A bill for the alteration of boundaries
areas, boundaries or names of existing States of states

P.154 For Civil Services Preliminary Examination

02-Indian Polity_Q413-927.indd 154 8/7/2018 7:40:35 PM


3. A private member bill that concerns empowers the Centre to exercise control over
minority welfare the state’s legislative matters in the above
4. Any ordinary bill in which the states may mentioned ways.
be interested For example, the bills imposing restrictions
Select the correct answer using the codes on the freedom of trade and commerce can be
below. introduced only after Presidential assent.
(a)  1 and 2 only (b)  3 and 4 only Q.591 In the matter of State legislation the President
(c)  1, 2 and 3 only (d)  1, 2 and 4 only may:
Solution: (a) (a) exercise only suspensive veto power
Justification: Statement 1: It becomes a (b) may withhold assent to any bill reserved
money bill and thus President must approve for his consideration except money bills
the introduction of the bill. (c) withhold his assent to any bill reserved
Statement 2: Under Article 3, the President for his consideration
must recommend the introduction of this bill (d) directly disallow any bill which he
as it has the potential to significantly affect considers anti national
the federal balance of power between centre Solution: (c)
and states. President can return the state bill for
Statement 3: The constitution does not reconsideration of state legislature. However
distinguish between Private or public bills president is not abided by the constitution to
when it comes to the requirement of prior give assent to the bill if it’s again passed by
Presidential assent. the state legislature.
Statement 4: It is only in certain cases,
such as alternation of state boundaries or Ordinance making power
introduction of GST, that the bill needs prior
permission of the President. In other cases it Q.592 Article 123 of the Constitution empowers the
is not mandatory. President to promulgate ordinances. Consider
the following in this regard.
Writ to State Bills 1. An ordinance made when both the Houses
are in session is valid if approved by the
Q.590 Consider the following statements. Council of Ministers and Parliament.
1. The governor can reserve certain types of 2. An ordinance cannot be used to amend the
bills passed by the state legislature for the constitution.
consideration of the President. Which of the above is/are correct?
2. Bills on certain matters enumerated in the (a)  1 only (b)  2 only
State List can be introduced in the state (c)  Both 1 and 2 (d)  None
legislature only with the previous sanction Solution: (b)
of the president. Justification: Statement 1: He can
3. The President can direct the states to promulgate an ordinance only when both the
reserve money bills and other financial Houses of Parliament are not in session or
bills passed by the state legislature for when either of the two Houses of Parliament
his consideration during a financial is not in session.
emergency. • An ordinance can also be issued when
Select the correct answer using the codes only one House is in session because a
below. law can be passed by both the Houses and
(a)  1 and 2 only (b)  1 and 3 only not by one House alone.
(c)  2 and 3 only (d)  1, 2 and 3 • An ordinance made when both the Houses
Solution: (d) are in session is void. Thus, the power of
Learning: Besides the Parliament’s power to the President to legislate by ordinance is
legislate directly on the state subjects under not a parallel power of legislation.
the exceptional situations, the Constitution

Indian Polity Question Bank P.155

02-Indian Polity_Q413-927.indd 155 8/7/2018 7:40:35 PM


Statement 2: This is because a constitutional Q.594 Which of the following can be a valid ground
amendment requires special majority in both to promulgate and ordinance by the President
houses of Parliament, unlike an ordinary on a subject of urgent importance?
legislation that can be approved by a 1. When only one house of the Parliament is
simple majority. in session.
Moreover, some amendment may require 2. When the Parliament has repeatedly failed
the approval of half of the states as well. to perform its duty.
Hence, option 2 is correct. 3. The council of Ministers must advice the
Q.593 Consider the following statements. president for issuing an ordinance.
1. An ordinance made when both the Houses Choose the correct answer using the codes
are in session is void. below:
2. Decision of the President to issue an (a)  1 and 2 (b)  2 and 3
ordinance can be questioned in a court. (c)  1 and 3 (d)  All of the above
Which of the above is/are true? Solution: (c)
(a)  1 only (b)  2 only As explained in previous question
(c)  Both 1 and 2 (d)  None Q.595 Consider the following about the ordinance-
Solution: (c) making Power of the President.
The ordinance-making power is the 1. The ordinances have the same force and
most important legislative power of the effect as an act of Parliament.
President. It has been vested in him to deal 2. An ordinance can be issued even if only
with unforeseen or urgent matters. But, one House of Parliament is in session.
the exercise of this power is subject to the 3. It can be issued to amend the Constitution.
following limitations: 4. It is subject to judicial review.
He can promulgate an ordinance only Select the correct answer using the codes
when both the Houses of Parliament are not below.
in session or when either of the two Houses (a)  1, 3 and 4 only (b)  1 only
of Parliament is not in session. An ordinance (c)  1, 2 and 4 only (d)  2 and 3 only
can also be issued when only one House is Solution: (c)
in session because a law can be passed by Justification: Statements 1 and 3: An
both the Houses and not by one House alone. ordinance like any other legislation, can
An ordinance made when both the Houses be retrospective, that is, it may come into
are in session is void. Thus, the power of the force from a back date. It may modify or
President to legislate by ordinance is not a repeal any act of Parliament or another
parallel power of legislation. ordinance. It can alter or amend a tax law
He can make an ordinance only when he also. However, it cannot be issued to amend
is satisfied that the circumstances exist that the Constitution.
render it necessary for him to take immediate Statement 2: An ordinance can also be
action. In R. C. Cooper v. UOI, AIR 1970 issued when only one House is in session
SC 564, the Supreme Court held that the because a law can be passed by both the
President’s satisfaction can be questioned in a Houses and not by one House alone. An
court on the ground of mala fide. This means ordinance made when both the Houses are in
that the decision of the President to issue an session is void.
ordinance can be questioned in a court on Statement 3: He can make an ordinance
the ground that the President has prorogued only when he is satisfied that the
one House or both Houses of Parliament circumstances exist that render it necessary
deliberately with a view to promulgate an for him to take immediate action. In R. C.
ordinance on a controversial subject, so as Cooper v. UOI, AIR 1970 SC 564, the
to bypass the parliamentary decision and Supreme Court held that the President’s
thereby circumventing the authority of the satisfaction can be questioned in a court on
Parliament. the ground of mala fide.

P.156 For Civil Services Preliminary Examination

02-Indian Polity_Q413-927.indd 156 8/7/2018 7:40:35 PM


Q.596 When can the President re-promulgate an Statement 2: Such recommendation is not
ordinance? required in all cases, but those for example
(a) when it is in public interest and any of the where an ordinance is likely to endanger
houses fail to pass the bill position of state high court or against
(b) when any of the houses has not DPSPs.
reassembled before the expiry of the
ordinance (3) Judicial Powers
(c) when in an emergency, it is required in
public interest Q.598 Which of the following comes under the
(d) when the bill is rejected by both the judicial powers of the President of India?
houses despite the government pushing 1. Appointment of the judges of Supreme
for the bill Court of India.
Solution: (b) 2. Pardoning a death sentence
Article 123 of the constitution gives 3. Deciding on whether a bill should be sent
legislative power to President. He can issue for re-consideration to the concerned state
ordinances when Parliament is in recess i.e. legislature.
not in session if there is urgent need to have Choose the correct answer using the codes
a law on some urgent public matter. The below:
promulgated ordinance has similar effect (a)  1 and 2 (b)  2 and 3
to an act of parliament. However, every (c)  1 and 3 (d)  All of the above
ordinance must be laid and approved by both Solution: (a)
houses of the parliament within 6 weeks from Deciding on a bill comes under the legislative
the reassembling. If not placed and approved powers of the President of India. Appointing
by both houses of the parliament after the judges and pardoning/remitting/
reassembling it becomes invalid or lapses. commuting/granting reprieve in sentences
comes under the judicial powers
Q.597 The executive can make laws via the
ordinance route as mentioned in the Article
123 of the constitution. Consider the Power of Pardon
following with this reference. Q.599 The pardoning power given to the President
1. Ordinances can only be made from of India under Article 72 can be exercised
subjects in the Union List by the Union (a) only after the trial and on the sentence of
executive. conviction
2. All ordinances made by the state executive (b) during or after trial but never before trial
are null and void if made without the prior (c) at any time before, during or after the
recommendation of the President. trial
Which of the above is/are correct? (d) either before or after the trial but never
(a)  1 only (b)  2 only during the trial of the case
(c)  Both 1 and 2 (d)  None Solution: (c)
Solution: (d) It is a well-established principle that a person
Justification: Ordinances are temporary can be sentenced or punished only when he
laws that are promulgated by the President has been convicted by the court. A person is
of India on the recommendation of the Union deemed to be innocent unless it is proved in
Cabinet. They can only be issued when the eyes of the law. Thus if a person has not
Parliament is not in session. They enable been given a chance of a fair trial or a proper
the Indian government to take immediate investigation has not been carried out against
legislative action. that person, then there is no reason why that
Statement 1: They can also be made on person should be given a pardon, because he
subjects in concurrent list, and those subjects is still innocent. Therefore, it is important to
in state list for which Parliament is making note that the pardoning power can be exercised
laws under special circumstances. only in the case of a convicted person only.

Indian Polity Question Bank P.157

02-Indian Polity_Q413-927.indd 157 8/7/2018 7:40:35 PM


However, in some of the cases the Court 3. Final decision of the President need not
has said that the pardon can be granted even rely on the evidences of the case.
before conviction or trial by a Court.This Choose the correct answer from the codes
principle was laid down in the case of In below.
Re Maddela Yerra Channugadu, AIR 1954 (a)  1 and 2 only (b)  2 and 3 only
Mad 911. (c)  1 and 3 only (d)  All of the above
Q.600 The power of pardon of the President of India Solution: (b)
extends to the laws made under which of the The Supreme Court examined the pardoning
following lists? power of the President under different cases
1. Union list 2.  State list and laid down the following principles.
3. Concurrent list 1. The petitioner for mercy has no right to an
Choose the correct answer using the codes oral hearing by the President.
below: 2. The President can examine the evidence
(a)  1 and 2 (b)  1 and 3 afresh and take a view different from the
(c)  Only 1 (d)  All of the above view taken by the court.
Solution: (d) 3. The power is to be exercised by the
The President can exercise his powers of President on the advice of the union
pardon on laws made under any list. The cabinet.
Governor, however, cannot do the same. He 4. The President is not bound to give reasons
is restricted to only the state laws. for his order.
5. The President can afford relief not only
Q.601 Mercy petition decided by the President
from a sentence that he regards as unduly
Article 72 of the Constitution of India are
harsh but also from an evident mistake.
disposed on the advice of the
6. There is no need for the Supreme Court
(a)  Prime Minister
to lay down specific guidelines for the
(b)  Ministry of Home affairs
exercise of power by the President.
(c) Parliamentary Committee on Legal
7. The exercise of power by the President is
affairs
not subject to judicial review except where
(d) Vice-President
the presidential decision is arbitrary,
Solution: (b)
irrational, mala fide or discriminatory.
Under the existing rules of procedure
8. Where the earlier petition for mercy has
governing mercy petitions, the view of the
been rejected by the President, stay cannot
Union Ministry of Home Affairs (MHA),
be obtained by filing another petition.
conveyed to the President in writing, is
taken as the view of the Cabinet, and Q.603 The Supreme Court has examined the
the President decides a mercy petition pardoning power of the President under
accordingly. different cases and laid down which of the
Once a convict has been finally awarded following principles?
the death sentence by the Supreme Court, 1. The petitioner for mercy has no right to an
anybody, including a foreign national, can oral hearing by the President.
send a mercy petition with regard to that 2. The power is to be exercised by the
person to the President’s Office or the MHA. President on the advice of the union
A mercy plea can also be sent to the Governor cabinet.
of the state concerned, who then forwards it 3. The President is bound to give reasons for
to the MHA for further action. his order.
4. The exercise of power by the President is
Q.602 Consider the following statements about the
not subject to judicial review.
pardoning power of the President.
Select the correct answer using the codes
1. He can exercise this power even without
below.
the advice of the cabinet.
(a)  1 and 2 only (b)  2 and 4 only
2. The President is not bound to give reasons
(c)  1 and 3 only (d)  2 and 3 only
for his order.

P.158 For Civil Services Preliminary Examination

02-Indian Polity_Q413-927.indd 158 8/7/2018 7:40:35 PM


Solution: (a) Which of the above is/are correct?
As explained in previous question. (a)  1 only (b)  2 only
Differences Between the Pardoning Powers (c)  Both 1 and 2 (d)  None
of President and Governor Solution: (a)
Justification: Statement 1: Even if a state
Q.604 The constitution grants both the President
law prescribes for death sentence, the power
and the Governor right to pardon
to grant pardon lies with the President and not
punishments granted by the Judiciary. What
the governor. But, the governor can suspend,
is/are the important differences between
remit or commute a death sentence.
their powers?
Statement 2: The President can grant
1. President can only grant pardon on the
pardon, reprieve, respite, suspension,
advice of Union Council of Ministers,
remission or commutation in respect to
whereas for the Governor this is purely a
punishment or sentence by a court-martial
discretionary matter.
(military court). Governor cannot.
2. President can grant pardon to a person
awarded death sentence, but Governor Q.606 Which one of the following powers can be
cannot do so. exercised by both the President and the
3. President can exercise this power only in Governor?
cases adjudicated by the Supreme Court, (a) Power to pardon a sentence by court
whereas Governor can do this only in martial
cases under High Courts. (b) Power to remit a sentence in an offence
Select the correct answer using the codes relating to a matter on the State List
below. (c) Power to commute a sentence of death in
(a)  1 only (b)  2 only certain circumstances
(c)  1 and 3 only (d)  2 and 3 only (d) Power to remit a sentence by court
Solution: (b) martial
Justification: Statement 1: The pardoning Solution: (c)
power of President is not absolute. It is
governed by the advice of the Council of (4) Military Powers
Ministers. Same applies for Governor as well.
Q.607 Who is the supreme commander of the
Statement 2: There are only three major
defence forces of India and in that capacity
differences:
appoints the chiefs of the Army, the Navy and
1. President has the right to pardon
the Air Force?
punishments of sentences given under
(a)  President of India
Court Martial, whereas the governor does
(b)  Prime Minister
not have this power.
(c)  Chief Justice of India
2. Resident can grant pardon to a person
(d)  Union Minister of Defence
awarded death sentence. But Governor of
Solution: (a)
State does not enjoy this power.
Justification: In the capacity of the supreme
3. Governor’s pardoning power only extends
commander of defence forces, the President
to those areas where the executive power
can also declare war or conclude peace,
of the concerned state extends. He cannot
subject to the approval of the Parliament.
pardon in case of offences committed under
Learning: Moreover, the international
Central laws. So, the option 3 is incorrect.
treaties and agreements are negotiated and
Q.605 Which of the following is/are the differences concluded on behalf of the President.
between the pardoning Powers of President However, they are subject to the approval
and Governor? of the Parliament. He represents India in
1. Governor cannot pardon a death sentence, international forums and affairs and sends
President can. and receives diplomats like ambassadors,
2. Governor can pardon a sentence of court high commissioners, and so on.
martial, President cannot.

Indian Polity Question Bank P.159

02-Indian Polity_Q413-927.indd 159 8/7/2018 7:40:36 PM


(5) Emergency Powers (d) Parliament should wait for 6 months
without taking any action on the returned
Q.608 Consider a situation where the Prime Minister bill, after which it will be deemed passed.
wants to impose “President’s rule” in one Solution: (b)
State because the State government has failed The President has the veto power over the
to effectively curb atrocities against the bills passed by the Parliament, that is, he can
Dalits in that State. The President does not withhold his assent to the bills. The object
agree wholeheartedly on signing the order. of conferring this power on the President is
Which of the following courses of action are two-fold—
available to the President? (a) to prevent hasty and ill-considered
1. Tell the Prime Minister that he will not legislation by the Parliament; and
sign on the order promulgating President’s (b) to prevent legislation which may be
rule. unconstitutional.
2. Make a press statement about how the The veto power enjoyed by the executive
Prime Minister is wrong. in modern states can be classified into the
3. Discuss the matter with the Prime Minister following four types.
and try to dissuade him from taking this 1. Absolute veto withholds of the assent to
action, but if he insists, agreeing to sign the the bill passed by the legislature.
said order. 2. Qualified veto, which can be overridden
Choose the correct answer using the codes by the legislature with a higher majority.
below. 3. Suspensive veto can be over ridden by the
(a)  1 only (b)  2 and 3 legislature with an ordinary majority.
(c)  1 and 3 (d)  3 only 4. Pocket veto takes no action on the bill
Solution: (d) passed by the legislature.
Here again Article 74(1) comes in the picture. Of the above four, the President of India is
In view of the controversy about the scope of vested with three—absolute veto, suspensive
the President’s powers, a specific mention was veto and pocket veto. There is no qualified
made in the Constitution by an amendment veto in the case of Indian President; it is
that the advice of the Council of Ministers possessed by the American President.
will be binding on the President. By another
Q.610 Every bill passed by the Parliament goes to
amendment made later, it was decided that
the President for his assent before it becomes
the President can ask the Council of Ministers
a law. Consider the following with reference
to reconsider its advice but, has to accept
to it.
the reconsidered advice of the Council of
1. The President can send the bill back to the
Ministers (the Prime Minister representing
Parliament asking it to reconsider the bill.
them in this case).
2. There is no time limit mentioned in the
constitution for the President to approve
Veto Power these bills.
Q.609 Consider a case where the Parliament has 3. The President is constitutionally
passed a legislation–sent to the President and authorized to send and introduce
the President returns it to the Parliament for legislative proposals in the Parliament.
reconsideration. For the bill to get passed Select the correct answer using the codes
now which of these should happen? below.
(a) Parliament should pass the bill again but (a)  1 and 2 only (b)  2 only
this time by a higher majority (c)  1 and 3 only (d)  1, 2 and 3 only
(b) Parliament should pass the bill again by Solution: (a)
an ordinary majority Justification: Statements 1 and 2: In doing
(c) A joint sitting should be convened to get so, the President exercises his situational
the bill passed discretion.

P.160 For Civil Services Preliminary Examination

02-Indian Polity_Q413-927.indd 160 8/7/2018 7:40:36 PM


The President has veto power by which decision with respect to a bill presented to
he can withhold or refuse to give assent to him for his assent.
Bills (other than Money Bill) passed by the • In USA, on the other hand, the President
Parliament. has to return the bill for reconsideration
The President can send the bill back to within 10 days. Hence, it is remarked that
the Parliament asking it to reconsider the the pocket of the Indian President is bigger
bill. This ‘veto’ power is limited because, than that of the American President.
if the Parliament passes the same bill again • In 1986, President Giani Zail Singh
and sends it back to the President, then, exercised the pocket veto with respect to
the President has to give assent to that the Indian Post Office (Amendment) Bill.
bill. However, there is no mention in the Q.612 Consider the following statements.
Constitution about the time limit within Assertion (A): The President of India has never
which the President must send the bill back exercised his Veto powers in India till date.
for reconsideration. Reason (R): As per 24th Constitutional
This means that the President can just amendment, the President is bound to give
keep the bill pending with him without his assent to any legislation passed by the
any time limit. This gives the President an Parliament.
informal power to use the veto in a very In the context of the above, which of these
effective manner. This is sometimes referred is correct?
to as ‘pocket veto’. (a) A is correct, and R is an appropriate
Statement 3: No such provision is found explanation of A.
in the Constitution. Even though President (b) A is correct, but R is not an appropriate
is an integral part of the Parliament, he explanation of A.
cannot initiate or participate in the legislative (c) A is correct, but R is incorrect.
proceedings of both the houses. (d) Both A and R are incorrect.
Q.611 It is often remarked that the “pocket of the Solution: (d)
Indian President is bigger than that of the Justification: In 1986, President Giani Zail
American President.” This is so because Singh exercised the pocket veto with respect to
(a) The salary and allowances of Indian the Indian Post Office (Amendment) Bill. The
President are far higher in nominal terms bill, passed by the Rajiv Gandhi Government,
than that of the American President. imposed restrictions on the freedom of press
(b) There is no time limit for the Indian and hence, was widely criticised.
President to exercise the power of pocket After three years, in 1989, the next
veto, unlike the American President who President R Venkataraman sent the bill back
is constrained by time. for reconsideration, but the new National
(c) Legislation in India cannot be passed Front Government decided to drop the bill.
without President’s seal, unlike in USA It should be noted here that the
where President is not involved in the President has no veto power in respect of
legislative process at all. a constitutional amendment bill. The 24th
(d)  All of the above Constitutional Amendment Act of 1971
Solution: (b) made it obligatory for the President to give
Learning: The President can neither ratify his assent to a constitutional amendment bill
nor reject nor return a bill, but simply keep it (not an ordinary bill).
pending for an indefinite period.
• This power of the President not to take (6) Discretionary Powers
any action (either positive or negative) on
the bill is known as the pocket veto. Q.613 In which of the following situations can the
• The President can exercise this veto power President exercise discretionary powers?
as the Constitution does not prescribe any 1. Asking the Council of Ministers to
time-limit within which he has to take the reconsider the advice tendered to him

Indian Polity Question Bank P.161

02-Indian Polity_Q413-927.indd 161 8/7/2018 7:40:36 PM


2. Pocket veto in case of Parliamentary veto in a very effective manner. This is
legislation sometimes referred to as ‘pocket veto’.
3. In appointing the Prime Minister when no 3. third kind of discretion arises more out
political party can prove a clear majority of political circumstances. Formally, the
Choose the correct answer using the codes President appoints the Prime Minister.
below: Normally, in the parliamentary system,
(a)  1 and 2 (b)  2 and 3 a leader who has the support of the
(c)  1 and 3 (d)  All of the above majority in the Lok Sabha would be
Solution: (d) appointed as Prime Minister and the
There are at least three situations where the question of discretion would not arise.
President can exercise the powers using his But imagine a situation when after an
or her own discretion. election, no leader has a clear majority
1. We have already noted that the President in the Lok Sabha. Imagine further that
can send back the advice given by the after attempts to forge alliances, two or
Council of Ministers and ask the Council three leaders are claiming that they have
to reconsider the decision. In doing this, the support of the majority in the house.
the President acts on his (or her) own Now, the President has to decide whom
discretion. When the President thinks to appoint as the Prime Minister. In such
that the advice has certain flaws or a situation, the President has to use his
legal lacunae, or that it is not in the best own discretion in judging who really
interests of the country, the President may have the support of the majority
can ask the Council to reconsider the or who can actually form and run the
decision. Although, the Council can government.
still send back the same advice and the Q.614 The President can NOT act without the
President would then be bound by that advice of the Council of Ministers in which
advice, such a request by the President to of the following cases?
reconsider the decision, would naturally (a) Appointment of Prime Minister when
carry a lot of weight. So, this is one way no party has a clear majority in the Lok
in which the president can act in his Sabha
own discretion. (b) Dismissal of the council of ministers
2. President also has veto power by which when it cannot prove the confidence of
he can withhold or refuse to give assent the Lok Sabha
to Bills (other than Money Bill) passed (c) Appointment of Governors in States
by the Parliament. Every bill passed by (d) Dissolution of the Lok Sabha if the
the Parliament goes to the President for council of ministers has lost its majority
his assent before it becomes a law. The Solution: (c)
President can send the bill back to the Learning: Though the President has no
Parliament asking it to reconsider the bill. constitutional discretion, hehas some
This ‘veto’ power is limited because, if situational discretion.
the Parliament passes the same bill again In other words, the President can act on his
and sends it back to the President, then, discretion (that is, without the advice of the
the President has to give assent to that ministers) under the following situations:
bill. However, there is no mention in the 1. When the Prime Minister in office
Constitution about the time limit within dies suddenly and there is no obvious
which the President must send the bill successor.
back for reconsideration. This means that 2. when it would be wrong for the Council
the President can just keep the bill pending of Ministers to advice the President,
with him without any time limit. This gives for example, after losing majority in
the President an informal power to use the Lok Sabha.

P.162 For Civil Services Preliminary Examination

02-Indian Polity_Q413-927.indd 162 8/7/2018 7:40:36 PM


Q.615 In which of the following matters can the First cabinet, Nehru appointed Mr. Ambedkar
President of India possibly act without the as a cabinet minister. He was from the
aid and advice of the Council of Ministers? opposition party. However, it does not usually
1. Sending a decision taken solely by a happen in contemporary politics.
Minister to the Council of Ministers for Q.617 Though the President has no constitutional
their re-consideration. discretion, he has some situational discretion
2. Appointing the Prime Minister from in which of the following cases?
multiple parties claiming majority in the 1. Appointment of Prime Minister when
Lok Sabha. no party has a clear majority in the Lok
3. Delaying approval of a bill passed by both Sabha.
the houses. 2. Dissolution of the Lok Sabha if the
Choose the correct answer using the codes council of ministers has lost its majority.
below: 3. Imposing President’s rule in States in case
(a)  2 and 3 (b)  1 and 3 of constitutional breakdown
(c)  1 and 2 (d)  All of the above Choose the correct answer from the codes
Solution: (d) below.
The President of India is generally bound by (a)  1 and 2 only (b)  2 and 3 only
the aid and advice of the Council of Ministers (c)  1 and 3 only (d)  All of the above
under Article 74 of the constitution of India. Solution: (a)
In some situations he has to act without The President can act on his discretion (that
their aid and advice. All the listed options is, without the advice of the ministers) under
count under these situations. Moreover, he the following situations:
can also exercise discretion in proroguing 1. Appointment of Prime Minister when
and convening sessions of the parliament. A no party has a clear majority in the Lok
special case is when a government has been Sabha or when the Prime Minister in
challenged under the no-confidence motion: office dies suddenly and there is no
here, the President has the responsibility to obvious successor.
summon a session of the Parliament. 2. Dismissal of the council of ministers
Q.616 Consider the following statements: when it cannot prove the confidence of the
1. The President has absolute discretion in Lok Sabha.
appointing the Prime Minister. 3. Dissolution of the Lok Sabha if the
2. The Prime Minister has absolute discretion council of ministers has lost its majority.
in appointing the Council of Ministers.
3. Ministers can be appointed only from the Miscellaneous
party/coalition which has a majority of
seats in the Lok Sabha. Q.618 Consider the following statements about
Choose the correct answer using the codes the position, powers, functions and
below: responsibilities of the President of India.
(a)  1 and 2 only (b)  2 and 3 only 1. He has a right to be informed of all
(c)  3 only (d)  None of the above important matters and deliberations of the
Solution: (d) Council of Ministers.
The President has only a situational discretion 2. The President need not accept the advice
in appointing the Prime Minister. Only the of Council of Ministers if the advice has
leader of the majority party/coalition can be been given for the first time.
appointed as the Prime Minister. The ministers 3. The only ground for his impeachment is
are appointed by the President, not the Prime the violation of the Constitution.
Minister, on the advice of the Prime Minister. 4. He scrutinizes the annual budget proposal
A minister can be any MP and not necessarily of the government before placing it to the
from the ruling party/coalition. For e.g. in the Parliament.

Indian Polity Question Bank P.163

02-Indian Polity_Q413-927.indd 163 8/7/2018 7:40:36 PM


Choose the correct answer using the codes Q.620 Acts of State done in the name of the President
below. of India are required to be countersigned by
(a)  1 and 2 (b)  1, 2 and 3 way of authentication by:
(c)  2 and 3 only (d)  All of the above (a)  a Minister
Solution: (b) (b)  the Prime Minister
The Constitution of India vests the executive (c)  the Speaker
power of the Union formally in the President. (d)  a Secretary to the Government
In reality, the President exercises these powers Solution: (d)
through the Council of Ministers headed by Q.621 The Constitution guarantees the following
the Prime Minister. The President is elected privileges to the President of India:
for a period of five years. But there is no (a) No criminal proceedings shall be
direct election by the people for the office of instituted against the President in any
President. The President is elected indirectly. court during his term of office
This means that the president is elected not (b) No process for the arrest or impeachment
by the ordinary citizens but by the elected of the President shall be issued from any
MLAs and MPs. This election takes place in court during his term of office
accordance with the principle of proportional (c) The President shall not be answerable
representation with single transferable vote. to any court for the exercise and
The President can be removed from performance of the powers and duties
office only by Parliament by following the of his office or for any act done or
procedure for impeachment. This procedure purporting to be done by him in exercise
requires a special majority as explained of those powers and duties
in the last chapter. The only ground for (d) No civil proceedings whatsoever shall
impeachment is violation of the Constitution. be instituted against the President in any
As per Article 74(1): There shall be a court during the term of his office
Council of Ministers with the Prime Minister Solution: (d)
at the head to aid and advise the President
Q.622 The Constitution
who shall in the exercise of his functions,
(a) is silent on the President’s re-election to
act in accordance with such advice. Provided
the office
that the President may require the Council of
(b) allows re-election of a person to the
Ministers to reconsider such advice, either
President’s post
generally or otherwise, and the President
(c) restricts a person to remain President for
shall act in accordance with the advice
only two terms
tendered after such reconsideration.
(d) has been amended to allow a person only
So he need not accept the advice if placed
one term as President
before him for the first time.
Solution: (b)
Q.619 Consider the following statements
1. The President of India is the constitutional
head of executive of the Union (17) VICE-PRESIDENT OF INDIA –
2. The Council of Ministers are collectively ARTICLE 63–71
responsible to both the houses of the
Parliament Elections
Which of the above statements is/are correct?
(a)  1 Only (b)  2 Only Q.623 How is the electoral college of the Vice-
(c) Both (d) None President different from that of the President?
Solution: (a) 1. Vice-President’s Electoral College
According to the PartV, Article-75 of the consists of both elected and nominated
Indian Constitution, the Council of Ministers members of the Parliament.
are collectively responsible to only to the
Lok Sabha.

P.164 For Civil Services Preliminary Examination

02-Indian Polity_Q413-927.indd 164 8/7/2018 7:40:36 PM


2. Vice-President’s Electoral College does Qualification
not include the members of the state
legislative assemblies. Q.625 To be eligible for election as Vice-President,
Which of the above is/are correct? a person should fulfil which of the following
(a)  1 only (b)  2 only qualifications?
(c)  Both 1 and 2 (d)  None 1. He should be qualified for election as a
Solution: (c) member of the Rajya Sabha.
Justification: Statement 1: In the case of 2. He should not be affiliated to a political
President, only elected members participate. party.
Statement 2: In the case of President 3. He should not have ever held the office of
the elected members of the state legislative the President.
assemblies are included. Select the correct answer using the codes
This is because Vice-President is only below.
a ceremonial head and does not wield any (a)  1 only (b)  2 and 3 only
executive power unless he assumes the office (c)  1 and 3 only (d)  1 and 2 only
of the President. Solution: (a)
Justification: Statement 1: Article 102 of
Q.624 Consider the following statements.
the Constitution lays down that a person
Assertion (A): State legislatures do not
shall be disqualified for being chosen as,
participate in the election of Vice-President.
and for being, a member of either House of
Reason (R): The Vice-President presides
Parliament, e.g. unsound mind, undischarged
over the Council of States.
insolvent and so on. So, the nominee to the
In the context of the above, which of these
office of the Vice-President must fulfil these
is correct?
conditions apart from others.
(a) A is correct, and R is an appropriate
Statement 2 and 3: He should not hold any
explanation of A.
office of profit under the Union government
(b) A is correct, but R is not an appropriate
or any state government or any local authority
explanation of A.
or any other public authority. Conditions in 2
(c)  A is correct, but R is incorrect.
and 3 do not prevent him/her to file nomination
(d)  Both A and R are incorrect.
for election to the office of the Vice-President.
Solution: (b)
Even a sitting President or Vice-President of the
Justification: In the constituent assembly
Union, the governor of any state and a minister
debates, it was pointed out that The President
for the Union or any state is not deemed to
is the head of the State and his power extends
hold any office of profit and hence qualified for
both to the administration by the Centre as
being a candidate for Vice-President.
well as to the states.
• Consequently, it is necessary that in his Q.626 Who among the following will be qualified
election, not only members of Parliament for being a candidate for the post of Vice-
should play their part, but the members of President?
the state legislatures should have a voice. 1. Governor of any state
• But, when we come to the Vice- 2. Union Cabinet Ministers
President, his normal functions are to 3. State cabinet Ministers
preside over the council of states. It is Choose the correct answer from the codes
only on a rare occasion, and that too below.
for a temporary period, that he may be vSolution: (d)
called upon to assume the duties of the To be eligible for election as Vice-President,
president. a person should fulfil the following
That being so, it does not seem necessary that qualifications:
the members of the state legislatures should 1. He should be a citizen of India.
also be invited to take part in the election of 2. He should have completed 35 years of
the Vice- President.” age.

Indian Polity Question Bank P.165

02-Indian Polity_Q413-927.indd 165 8/7/2018 7:40:36 PM


3. He should be qualified for election as a Which of these is /are true?
member of the Rajya Sabha. (a)  Only 1 (b)  Only 2
4. He should not hold any office of profit (c) Both (d) None
under the Union government or any state Solution: (c)
government or any local authority or any The vice-president is the chairman of the
other public authority. Rajya Sabha. He has to first impeached by
But, a sitting President or Vice-President the Rajya Sabha by an absolute majority
of the Union, the governor of any state and a and then by the Lok Sabha. The constitution
minister for the Union or any state is not deemed unlike in the case of the President, does not
to hold any office of profit and hence qualified mention any ground for his removal.
for being a candidate for Vice-President.
Vacancy Occurred in Office
Term of Office Q.629 When a vacancy occurs in the office of the
Q.627 The vice-President can be removed in which President due to his resignation, the Vice-
of the following ways? President acts as the President until a new
(a) By a resolution of the Rajya Sabha President is elected. Consider the following
passed by a majority and agreed to by the with regard to when the office of Vice-
Lok Sabha President is vacant.
(b) By a resolution of the Rajya Sabha Assertion (A): Speaker, Lok Sabha acts as the
passed by a special majority and agreed President of India in such a case.
to by the Lok Sabha Reason (R): Speaker, Lok Sabha is ranked
(c) By a resolution of the Rajya Sabha and immediately below the Vice- President and
Lok Sabha passed by a special majority Prime Minister in the Table of Precedence.
in both the houses In the context of the above, which of these
(d) By the order of the President after an is correct?
enquiry of the Supreme Court (a) A is correct, and R is an appropriate
Solution: (a) explanation of A.
The Vice President is elected for five years. (b) A is correct, but R is not an appropriate
His election method is similar to that of explanation of A.
the President, the only difference is that (c)  A is incorrect, but R is correct.
members of State legislatures are not part of (d)  Both A and R are incorrect.
the electoral college. Solution: (d)
The Vice President may be removed from Justification: When a vacancy occurs in the
his office by a resolution of the Rajya Sabha office of the President due to his resignation,
passed by a majority and agreed to by the removal, death or otherwise, the Vice-
Lok Sabha. The Vice President acts as the ex- President acts as the President until a new
officio Chairman of the Rajya Sabha and takes President is elected.
over the office of the President when there is Further, when the sitting President is
a vacancy by reasons of death, resignation, unable to discharge his functions due to
removal by impeachment or otherwise. absence, illness or any other cause, the Vice-
Q.628 Consider the following statements about the President discharges his functions until the
impeachment of the Vice-President: President resumes his office.
1. Only an absolute majority is needed in the In case the office of Vice-President is
Rajya Sabha followed by the approval of vacant, the Chief Justice of India (or if his office
the Lok Sabha for impeaching the Vice- is also vacant, the senior-most judge of the
President. Supreme Court available) acts as the President
2. No ground has been mentioned in the or discharges the functions of the President
constitution for the removal of the vice- Q.630 Consider the following statements regarding
president. the Vice-President of India:

P.166 For Civil Services Preliminary Examination

02-Indian Polity_Q413-927.indd 166 8/7/2018 7:40:36 PM


1. The Vice-President is elected by an presided over by the Speaker.
electoral college consisting of all Q.632 When the Vice-President is acting in the
the members of the both Houses of capacity of the President, then:
Parliament. (a) He ceases to be the Chairman of Rajya
2. The Constitution is silent about a person Sabha.
who is to discharge the duties of the Vice- (b) He resigns from the position of the
President during the period of vacancy. Chairman of Rajya Sabha.
Which of these statements is/are correct? (c) He continues to preside over the
(a)  1 only (b)  2 only proceedings of the Rajya Sabha.
(c)  Both 1 and 2 (d)  Neither 1 nor 2 (d) None of the above
Solution: (c) Solution: (d)
The Constitution is silent on who performs the He neither resigns, nor ceases to be the
duties of the Vice-President, when a vacancy chairman of the RS. He only stops presiding
occurs in the office of the Vice-President of over its proceedings. Preferably the Deputy
India, before the expiry of his term, or when Chairman takes his position temporarily
the Vice-President acts as the President of
Q.633 In case a President dies while in office, the
India. The only provision in the Constitution
Vice-President can act as President for a
is with regard to the Vice-President’s function
maximum period of:
as the Chairperson of the Council of States
(a)  2 years (b)  1 year
(Rajya Sabha), which is performed, during
(c)  3 months (d)  6 months
the period of such vacancy, by the Deputy
Solution: (d)
Chairperson of the Rajya Sabha, or any other
member of the Rajya Sabha authorised by the Q.634 Which of the following bodies is/are presided
President of India. by nonmembers?
1. Rajya Sabha
Powers and Functions 2. Vidhan Sabha
3. Vidhan Parishad
Q.631 The ‘Council of States’ is chaired by the Select the correct answer using the codes
(a)  President of India below.
(b)  Vice-President of India (a)  1 and 2 only (b)  1 only
(c)  Prime Minister of India (c)  2 and 3 only (d)  1 and 3 only
(d) Governors Solution: (b)
Explanation: Council of States is popularly Justification: The presiding officers of
known as Rajya Sabha. It should not be Vidhan Parishad are generally known as
confused with the Inter-state Council or any Chairman and Deputy Chairman. They
other such coordination mechanisms between are elected (from amongst the members of
the Centre and State. the house) and hold their offices like the
So, option (b) is correct. The point that is presiding officers of Vidhan Sabha.
to be noted is: The chairman of the Council Chairman of Rajya Sabha is the Vice-
of States is not elected solely by the Rajya President of India, who is not a member of
Sabha. In the case of Lok Sabha, the speaker the Parliament.
is solely elected by the Lok Sabha.
The Inter-state council is chaired by the Indian and American Vice-Presidents
Prime Minister.
Learning: The Parliament of India (Sansad) is Miscellaneous
the supreme law-making institution. It has two
Q.635 Which of the following statements with
Houses, the Rajya Sabha and the Lok Sabha.
reference to the Office of the Vice-President
Rajya Sabha (Council of States), with a total
of India is correct?
strength of 245 members, is chaired by the
(a) 
State legislatures are not part of the
Vice-President of India. Lok Sabha (House of
electoral college of the Vice-President.
the People), with a total membership of 545, is

Indian Polity Question Bank P.167

02-Indian Polity_Q413-927.indd 167 8/7/2018 7:40:36 PM


(b) The Vice-President may be removed Vice-President. The Vice-President is elected
from his office by a resolution of the indirectly by members of an electoral college
Rajya Sabha passed by a majority and consisting of the members of both Houses of
agreed to by the Lok Sabha. Parliament in accordance with the system of
(c) The Vice-President acts as the ex-officio Proportional Representation by means of the
Chairman of the Rajya Sabha and takes Single transferable vote and the voting is by
over the office of the President when secret ballot.
there is a vacancy. Q.637 Consider the following with regard to the
(d)  All of the above Vice President of India:
Solution: (d) 1. The Vice-President is elected by the
Learning: Option A: The Vice-President electoral college consisting of members
is elected for 5 years. His election method of the Parliament, State Legislatures.
is similar to that of the President, the only 2. The Vice President may be removed from
difference is that members of State legislatures his office by a resolution of the Lok Sabha
are not part of the electoral college. passed by a majority and agreed to by the
Option B: The impeachment of Vice- Rajya Sabha.
President is different from that of the 3. The Vice-President acts as the ex- officio
President. In the latter case, both houses must Chairman of the Rajya Sabha
pass the resolution by a special majority. 4. The decision of Supreme Court is final
But, the Vice-President may be removed with regard to any disputes in the election
from his office by a resolution of the Rajya of the Vice-President
Sabha passed by a majority and agreed to by Which of the above statements is/are correct?
the Lok Sabha. (a)  2 and 3 only (b)  3 and 4 only
Option C: The Vice-President acts as the (c)  1, 2 and 3 only (d)  1, 2, 3 and 4 only
President only until a new President is Solution: (b)
elected. B. D. Jatti acted as President on the All disputes arising in connection with the
death of Fakhruddin Ali Ahmed until a new election of the Vice-President are petitioned
President was elected. to the Supreme Court of India, which enquires
Q.636 With reference to the office of Vice-President into the matter. The petition is heard by a
of India, consider the following statements: five- member bench of the Supreme Court,
1. When he is acting as President, he still which decides on the matter. The decision of
performs the function of the Chairman of the Supreme Court is final.
the Rajya Sabha
2. The Vice-President is elected indirectly
by members of an electoral college
(18) PRIME MINISTER
consisting of the members of both Houses
of Parliament Appointment
Which of the above statements is/are correct? Q.638 The Constitution does not contain any specific
(a)  1 only (b)  2 only procedure for the selection and appointment
(c) Both (d) None of the Prime Minister. This means that
Solution: (b) 1. The Constitution does not mention the
The Vice President shall act as President in appointing authority for the office of
the absence of the President due to death, Prime Minister.
resignation, impeachment, or other situations. 2. The President is free to change the
The Vice-President of India is also ex officio selection and appointment procedure
Chairperson of the Rajya Sabha. He ceases to of Prime Minister by rules issued in the
perform as Chairman of Rajya Sabha while Gazette of India.
acting as the President. Which of the above is/are correct?
Article 66 of the Indian Constitution (a)  1 only (b)  2 only
states the manner of election of the (c)  Both 1 and 2 (d)  None

P.168 For Civil Services Preliminary Examination

02-Indian Polity_Q413-927.indd 168 8/7/2018 7:40:36 PM


Solution: (d) (b) The President may first appoint him the
Statement 1: Article 75 says only that the Prime Minister and then ask him to prove
Prime Minister shall be appointed by the his majority in the Lok Sabha within a
president. However, this does not imply that reasonable period.
the president is free to appoint any one as the (c) The Leader of Winning coalition/party
Prime Minister. first ensures majority in both houses of
Statement 2: In accordance with the Parliament by a motion and voting and
conventions of the parliamentary system of then sends appointment request to the
government, the President has to appoint the President
leader of the majority party in the Lok Sabha (d) The Prime Minister does not need to
as the Prime Minister. prove his majority on the house floor; a
But, when no party has a clear majority in written proof of support from required
the Lok Sabha, then the President may exercise number of MPs is sufficient.
his personal discretion in the selection and Solution: (b)
appointment of the Prime Minister. Learning: In 1980, the Delhi High Court
Justification: In such a situation, the held that the Constitution does not require
President usually appoints the leader of the that a person must prove his majority in
largest party or coalition in the Lok Sabha as the Lok Sabha before he is appointed as the
the Prime Minister and asks him to seek a vote Prime Minister.
of confidence in the House within a month. The President may first appoint him the
However, this is not guided by rules made Prime Minister and then ask him to prove his
by the President, and is based on established majority in the Lok Sabha within a reasonable
conventions. So, 2 is wrong. period.
Q.639 Consider the following statements. For example, Charan Singh (in 1979),
1. The Constitution does not contain any and after in a series, V.P. Singh (1989),
specific procedure for the selection and Chandrasekhar (1990), P.V. Narasimha Rao
appointment of the Prime Minister. (1991), A.B. Vajyapee (1996), Deve Gowda
2. The Constitution does not require that a (1996), I.K. Gujral (1997) and again A.B.
person must prove his majority in the Lok Vajpayee (1998) were appointed as Prime
Sabha before he is appointed as the Prime Ministers in this way.
Minister. Q.641 In 1997, the Supreme Court held that a
Which of the above is/are correct? person who is not a member of either House
(a)  1 only (b)  2 only of Parliament can be appointed as Prime
(c)  Both 1 and 2 (d)  None Minister for 6 months, within which, he
Solution: (c) should become a member of either House
Justification: Statement 1: As explained in of Parliament; otherwise, he ceases to be the
previous question Prime Minister.
Statement 2: In 1980, the Delhi High Court The Prime Minister and all the ministers
held that the Constitution does not require have to be members of the Parliament. If
that a person must prove his majority in someone becomes a minister or Prime Minister
the Lok Sabha before he is appointed as without being an MP, which of these follows?
the Prime Minister. The President may first (a) He must obtain a special approval from
appoint him the Prime Minister and then ask the Parliament to continue in the office.
him to prove his majority in the Lok Sabha (b) He should get prior Presidential consent
within a reasonable period. if he continues in the office for a period
Q.640 Which of the following statements is correct? of more than 6 months.
(a) The Constitution requires that a person (c) Such a person has to get elected to the
must prove his majority in the Lok Sabha Parliament within six months, failing
before he is appointed as the Prime which he stands removed from the office.
Minister. (d)  None of the above

Indian Polity Question Bank P.169

02-Indian Polity_Q413-927.indd 169 8/7/2018 7:40:36 PM


Solution: (c) Q.644 The Prime Minister, at the time of the
Learning: Our former Prime Minister appointment
Manmohan Singh was not a MP when he was 1. does need not necessarily be a member of
chosen to be as the Prime Minister. He was one of the Houses of Parliament but must
subsequently elected to a Rajya Sabha seat become a member of one of the Houses
from Assam within a period of 6 months. within 6 months.
There are many cases like this. The essential 2. does need not necessarily be a member of
idea is that in the parliamentary democracy. one of the. Houses of Parliament but must
Without being a part of the Parliament, no become the member of the Lok Sabha
one can become a part of the executive. within 6 months.
Q.642 Which of the following is/are the eligibility 3. must be either a nominated or elected
criteria for the office of the Prime Minister? member of one of the Houses of
1. He should not have been a former Parliament.
President of India. 4. must be an elected member of only Lok
2. He should have been directly elected by Sabha.
the people of India. (a)  I only (b)  I and III
3. He should be the leader of a political (c)  II only (d)  IV only
party. Solution: (a)
Choose the correct answer using the codes
below. Oath
(a)  1 and 2 only (b)  2 and 3 only
Q.645 Which of the following is/are common to
(c)  3 only (d)  None of the above
the oath taken by the Prime Minister and the
Solution: (d)
other Members of Parliament (MPs)?
Article 75 says only that the Prime Minister
1. To bear true faith and allegiance to the
shall be appointed by the president. President
Constitution of India
has to appoint the leader of majority party
2. To uphold the sovereignty and integrity of
as the Prime Minister. Though he/she is
India
not member of parliament at the time of
3. To do right to all manner of people in
appointment he/she has to acquire parliament
accordance with the Constitution and the
membership within 6 months. This means
law, without fear or favour, affection or ill
one has to be eligible to be a member of Lok
will.
Sabha or Rajya Sabha to be a Prime Minister.
Select the correct answer using the codes
Q.643 Consider the following statements. below.
1. The Constitution does not contain any (a)  1 and 2 only (b)  2 only
specific procedure for the selection and (c)  2 and 3 only (d)  3 only
appointment of the Prime Minister. Solution: (a)
2. A person must prove his majority in the Justification: Before the Prime Minister
Lok Sabha before he is appointed as the enters upon his office, the president
Prime Minister. administers to him the oaths of office and
3. The Prime Minister may be a member of secrecy. In his oath of office, the Prime
any of the two Houses of parliament. Minister swears:
Choose the correct answer from the codes • To bear true faith and allegiance to the
below. Constitution of India,
(a)  1 and 2 only (b)  2 and 3 only • To uphold the sovereignty and integrity of
(c)  1 and 3 only (d) All of the above India,
Solution: (c) • To faithfully and conscientiously discharge
Refer previous explanations the duties of his office, and
• To do right to all manner of people in
accordance with the Constitution and the

P.170 For Civil Services Preliminary Examination

02-Indian Polity_Q413-927.indd 170 8/7/2018 7:40:36 PM


law, without fear or favour, affection or 2. The Prime Minister can bring about the
ill will. collapse of the council of ministers by
The MP also takes a similar oath apart from resigning from office.
the oath of secrecy and the last point in the 3. The council of Ministers cannot function
oath of Prime Minister (or Union Ministers). when the Prime Minister resigns.
The oath of MPs is to Choose the correct answer from the codes
1. bear true faith and allegiance to the below.
Constitution of India; (a)  1 and 2 only (b)  2 and 3 only
2. uphold the sovereignty and integrity of (c)  1 and 3 only (d) All of the above
India; and Solution: (d)
3. faithfully discharge the duty upon which The Prime Minister enjoys the following
he is about to enter. powers as head of the Union council of
Unless a member takes the oath, he cannot ministers:
vote and participate in the proceedings of 1. He recommends persons who can
the House and does not become eligible to be appointed as ministers by the
parliamentary privileges and immunities. president. The President can appoint
Q.646 In his oath of secrecy, the Prime Minister only those persons as ministers who are
swears that recommended by the Prime Minister.
1. All government proceedings will be kept 2. He allocates and reshuffles various
secret. portfolios among the ministers.
2. No matter handled by the Prime Minister’s 3. He can ask a minister to resign or advise
Office will be disclosed to other ministries. the President to dismiss him in case of
Which of the above is/are correct? difference of opinion.
(a)  1 only (b)  2 only 4. He presides over the meeting of council of
(c)  Both 1 and 2 (d)  None ministers and influences its decisions.
Solution: (d) 5. He guides, directs, controls, and
Justification: The Prime Minister swears that coordinates the activities of all the
he will not directly or indirectly communicate ministers.
or reveal to any person or persons any 6. He can bring about the collapse of the
matter which shall be brought under his council of ministers by resigning from
consideration or shall become known to his office.
as prime minister for the Union except as Since the Prime Minister stands at the
may be required for the due discharge of his head of the council of ministers, the other
duties as such Minister. ministers cannot function when the Prime
This does not imply that all government Minister resigns or dies. In other words, the
proceedings will be secret or Prime Minister’s resignation or death of an incumbent Prime
office will work in confidentiality. So, both Minister automatically dissolves the council
statements are incorrect. of ministers and thereby generates a vacuum.
The resignation or death of any other
minister, on the other hand, merely creates
Powers and Functions a vacancy which the Prime Minister may or
may not like to fill.
(1) Writ to Council of Ministers
Q.648 In India, the Prime Minister enjoys a pre-
Q.647 Consider the following statements about eminent place in the government. Consider
the powers of the Prime Minister (Prime the following with reference to this.
Minister). 1. The Council of Ministers comes into
1. The President can appoint only existence only after the Prime Minister
those persons as ministers who are has taken the oath of office.
recommended by the Prime Minister.

Indian Polity Question Bank P.171

02-Indian Polity_Q413-927.indd 171 8/7/2018 7:40:36 PM


2. The resignation of the Prime Minister the council of ministers only if an alternative
automatically brings about the dissolution Prime Minister from the same party is
of the Council of Ministers. not present.
Which of the above is/are correct? Q.651 Consider the following statements about the
(a)  1 only (b)  2 only Prime Minister of India:
(c)  Both 1 and 2 (d)  None 1. His decisions are final in case
Solution: (c) disagreements arise between Departments.
Justification: Statement 1: This is because the 2. He also has the power to dismiss ministers.
Council of Ministers cannot exist without the 3. When the Prime Minister quits, the entire
Prime Minister. He advises the appointment ministry quits.
of all the ministers and heads the cabinet Choose the correct answer using the codes
meetings. He can also advise the President to below:
dismiss ministers from the CoMs. (a)  All of the above (b)  1 and 3 only
Statement 2: This is because the entire (c)  1 and 2 only (d)  2 and 3 only
Council of Minister is de facto appointed by Solution: (a)
the Prime Minister, and after his resignation a The Constitution does not say very much
new Council of Minister is constituted. about the powers of the Prime Minister or
Q.649 Consider the following statements about the the ministers or their relationship with each
position of the Prime Minister in the Council other. But as head of the government, the
of Ministers. Prime Minister has wide ranging powers.
1. His decision is binding on the Council of 1. He chairs Cabinet meetings. He coordinates
Ministers. the work of different Departments. His
2. His resignation may lead to the dissolution decisions are final in case disagreements
of the Council of Ministers. arise between Departments. He exercises
3. He can remove any Minister from the general supervision of different ministries.
Council without Parliamentary sanction. 2. All ministers work under his leadership.
Choose the correct answer using the codes The Prime Minister distributes and
below: redistributes work to the ministers. He
(a)  1 and 2 (b)  2 and 3 also has the power to dismiss ministers.
(c)  1 and 3 (d) All of the above 3. If the cabinet is the most powerful, inside
Solution: (d) it the PM is the most powerful.
As explained in previous question. Q.652 If there is frequent disagreement on major
Q.650 Consider the following statements about the decisions in the Council of Ministers due to
office of the Prime Minister (Prime Minister): just a few ministers, what option(s) does the
1. The Prime Minister can dismiss any Prime Minister has/have?
minister at his will. 1. Dismiss the Ministers who are causing
2. He can possibly bring the collapse of the disagreement
council of ministers by resigning from 2. Enforce his decision irrespective of the
office. view of the Council of Ministers
3. His resignation automatically leads to the 3. Convene a Parliamentary session to take
dissolution of the Lok Sabha. final view on the decision
Which of these is/are true? Choose the correct Choose the correct answer using the codes
answer using the codes below: below.
(a)  1 and 2 (b)  2 and 3 (a)  1 and 2 only (b)  2 and 3 only
(c)  1 and 3 (d)  All of the above (c)  1 and 3 only (d)  All of the above
Solution: (a) Solution: (a)
The resignation of the Prime Minister leads The Prime Minister plays the leadership
to the dissolution of the Lok Sabha only in role in this system of government. He is the
case another government can not be formed. leader of council of ministers, leader of the
But his resignation may bring the collapse of Parliament and leader of the party in power.

P.172 For Civil Services Preliminary Examination

02-Indian Polity_Q413-927.indd 172 8/7/2018 7:40:36 PM


In these capacities, he plays a significant (2) Writ to President of India
and highly crucial role in the functioning of
the government. Q.655 As per the constitution, it is the duty of the
Although all decisions are supposed to be Prime Minister to
taken based on consensus in the Council, he 1. Communicate to the President all
has the final say over all the decisions of the decisions of the council of ministers
Council of Ministers. To this effect, he can relating to the administration of the affairs
even dismiss a few ministers who are causing of the Union and proposals for legislation
frequent disagreement in the council. 2. Submit for the consideration of the
council of ministers any matter on
Q.653 The President can dismiss a Cabinet Minister,
which a decision has been taken by
who is also a MP, with
a minister but which has not been
(a)  Consent from the Speaker
considered by the council, if required by
(b) The recommendation of the Prime
the President.
Minister
Which of the above is/are correct?
(c) A show cause note issued to the Judiciary
(a)  1 only (b)  2 only
(d) The application of Doctrine of Pleasure
(c)  Both 1 and 2 (d)  None
of the President
Solution: (c)
Solution: (b)
Justification: These are the provisions as per
Learning: All of the executive authorities
Article 78 of the constitution.
vested in the President are, in practice,
It also requires the Prime Minister to
exercised by the Prime Minister with the help
furnish such information relating to the
of the Council of Ministers.
administration of the affairs of the Union and
So, to dismiss a minister, including
proposals for legislation as the President may
cabinet ministers, the President requires the
call for.
consent of the Prime Minister.
This article is one of the most important
Without Prime Minister’s advice, it would
ones in the constitution delineating the
be unconstitutional to remove a minister.
relationship between Prime Minister and
Q.654 Consider the following statements about the President.
role of the Prime Minister.
Q.656 The Prime Minister is the leader of the Lower
1. He allocates and reshuffles various
House. In this capacity, he enjoys which of
portfolios among the ministers.
the following powers?
2. He can bring about the collapse of the
1. He advises the President with regard
council of ministers by resigning from
to summoning and proroguing of the
office.
sessions of the Parliament.
Which of the above is/are correct?
2. He can recommend dissolution of the Lok
(a)  1 only (b)  2 only
Sabha to President at any time.
(c)  Both 1 and 2 (d)  None
3. He has the authority to adjourn the lower
Solution: (c)
house sine die.
Learning: Since the Prime Minister stands at
Select the correct answer using the codes
the head of the council of ministers, the other
below.
ministers cannot function when the Prime
(a)  1 and 2 only (b)  2 and 3 only
Minister resigns or dies.
(c)  1 and 3 only (d)  1, 2 and 3
In other words, the resignation or death of
Solution: (a)
an incumbent Prime Minister automatically
Justification: Statement 2: Both the executive
dissolves the council of ministers and thereby
and legislature in India have powers to shake
generates a vacuum.
each other’s existence. Executive (PM) can
The resignation or death of any other
recommend dissolution of lower house to the
minister, on the other hand, merely creates
President.
a vacancy which the Prime Minister may or
may not like to fill.

Indian Polity Question Bank P.173

02-Indian Polity_Q413-927.indd 173 8/7/2018 7:40:36 PM


On the other hand, legislature can pass no- Solution: (d)
confidence motion to remove the council of The first statement is false as the Prime
Ministers from office. Minister can only recommend the dissolution
Statement 3: Speaker, Lok Sabha only is of the Lok Sabha and not the whole of
authorized to do so. Parliament which also includes the Rajya
Sabha. He can do so by the virtue of being in
(3) Writ to Parliament the majority in the Lok Sabha.
Q.657 The Prime Minister enjoys which of the
following powers and responsibilities in
(4) Others
relation to the Parliament? Q.659 Prime Minister of India
1. He advises the President with regard 1. Heads the Public Accounts Committee of
to summoning and proroguing of the Parliament
sessions of the Parliament. 2. Chairs the NITI Aayog
2. He can recommend dissolution of the Lok 3. Is in direct charge of Ministry of Personnel,
Sabha to President at any time. Public Grievances and Pensions
3. He announces government policies on the Select the correct answer using the codes
floor of the House. below.
Select the correct answer using the codes (a)  1 and 2 only (b)  2 and 3 only
below. (c)  1 and 3 only (d)  1, 2 and 3
(a)  1 only (b)  2 and 3 only Solution: (b)
(c)  1 and 3 only (d)  1, 2 and 3 Justification and Learning: Some specific
Solution: (d) ministries/department are not allocated to
Justification: Statement 1: The President anyone in the cabinet but the prime minister
cannot arbitrarily summon or prorogue himself. The prime minister is usually always
proceedings of the house, but based on Prime in-charge/head of:
Minister’s advice. However, he is not bound • Appointments Committee of the Cabinet;
to accept the advice of the Prime Minister, • Ministry of Personnel, Public Grievances
when he is seeking a vote of confidence. and Pensions;
Statement 2: When he recommends • Ministry of Planning;
dissolution, he should not have been subject • Department of Atomic Energy; and
to a prior vote of confidence. • Department of Space.
Statement 3: The Prime Minister is the
leader of the Lower House. In this capacity, Miscellaneous
he enjoys these powers.
Q.660 The salary and perquisites of the Prime
Q.658 Consider the following statements about the
Minister of India are decided by the:
power of executive branch of government of
(a) Constitution (b) Cabinet
India:
(c) Parliament (d) President
Assertion (A): The Prime Minister can
Solution: (c)
recommend the dissolution of the Parliament.
Reason (R): The Prime Minister is the Q.661 Consider the following statements.
leader of the ruling majority party in India. Assertion (A): A serving Prime Minister
In the context of the above two statements, cannot be expelled by a house of Parliament
which one of these is true? for breach of its privilege.
(a) Both A and R is true and R is the correct Reason (R): If Prime Minister is a member
explanation of A. of a house of Parliament, he serves as the
(b) Both A and R is true but R is not a correct Leader of the House.
explanation of A. In the context of the above, which of these
(c)  A is true, but R is false. is correct?
(d)  A is false, but R is true (a) 
A is correct, and R is an appropriate
explanation of A.

P.174 For Civil Services Preliminary Examination

02-Indian Polity_Q413-927.indd 174 8/7/2018 7:40:36 PM


(b) A is correct, but R is not an appropriate 4. When the Prime Minister quits, the entire
explanation of A. ministry quits.
(c)  A is correct, but R is incorrect. 5. The rise of coalition politics has imposed
(d)  A is incorrect, but R is correct. certain constraints on the power of the
Solution: (d) Prime Minister.
Justification: In 1977, the sixth Lok Sabha Which of the statements is/are correct?
expelled Mrs. Indira Gandhi from its (a)  1, 2 and 5 (b)  2, 3 and 5
membership and sentenced her to jail for a (c)  1, 3, 4 and 5 (d)  2, 3, 4 and 5
week for committing the contempt of House Solution: (c )
while she was Prime Minister. So, A is The President appoints the Prime Minister,
incorrect. no direct election to the Prime Minister. But
But, the seventh Lok Sabha rescinded the the President cannot appoint anyone she
resolution expelling her by describing it as likes. The President appoints the leader of
politically motivated. the majority party or the coalition of parties
A Prime Minister need not be the Leader that commands a majority in the Lok Sabha,
of the Lok Sabha, but he is the Leader of the as Prime Minister. In case no single party
House where he is a member (for example or alliance gets a majority, the President
Manmohan Singh was leader of Rajya Sabha appoints the person most likely to secure a
and its member). So, R is correct. majority support.
Q.662 If the Prime Minister of India belonged to the The Prime Minister does not have a fixed
Upper House of Parliament tenure. He continues in power so long as he
(a) He can make statements only in the remains the leader of the majority party or
Upper House. coalition.
(b) He cannot speak on the Budget in the Sometimes, a person who is not a member
Lower House. of Parliament can also become a minister.
(c) He cannot vote in his coalition’s favour But such a person has to get elected to one
in a no-confidence motion. of the Houses of the Parliament within six
(d) He must become a member of the Lower months of appointment as minister.
House within 6 months after being When the Prime Minister quits, the entire
sworn-in as the Prime Minister. ministry quits.
Solution: (c) In recent years the rise of coalition politics
Justification: A no-confidence motion is has imposed certain constraints on the power
introduced and passed in the Lok Sabha. It is of the Prime Minister. The Prime Minister of
a test of the party in power. a coalition government cannot take decisions
Even though Prime Minister can attend as he likes. He has to accommodate different
the meetings of Lok Sabha, it is only the groups and factions in his party as well as
members of Lok Sabha who can vote on such among alliance partners. He also has to heed
a motion. Therefore, in his first tenure Shri to the views and positions of the coalition
Manmohan Singh could not have voted in partners and other parties, on whose support
case of a no-confidence motion. the survival of the government depends.
Q.663 Prime Minister is the most important political Q.664 The power wielded by the Prime Minister
institution in the country. Consider the flows from various sources. Which of the
following statements: following are such sources of power of the
1. There is no direct election to the post of Prime Minister?
the Prime Minister. 1. Control over the Council of Ministers
2. The Prime Minister has a fixed tenure. 2. Leadership of the Lok Sabha
3. A person who is not a member of 3. Command over the bureaucratic machine
Parliament can also become a minister 4. Access to media
provided he meets the required conditions. 5. Projection of personalities during elections
6. Projection as national leader during

Indian Polity Question Bank P.175

02-Indian Polity_Q413-927.indd 175 8/7/2018 7:40:36 PM


International summitry and foreign visits be enquired by any court. This provision
Select the correct answer using the codes emphasises the intimate and the confidential
below: relationship between the President and the
(a)  1, 2, 3 and 4 only ministers.
(b)  1, 2, 5 and 6 only Statement 3: In 1971, the Supreme Court
(c)  1, 2, 3, 5 and 6 only held that ‘even after the dissolution of the
(d)  1, 2, 3, 4, 5 and 6 only Lok Sabha, the council of ministers does not
Solution: (d) cease to hold office.
The Prime Minister is involved in all crucial • Article 74 is mandatory and, therefore, the
decisions of the government and decides president cannot exercise the executive
on the policies of the government. Thus, power without the aid and advise of the
the power wielded by the Prime Minister council of ministers.
flows from various sources: control over • Any exercise of executive power without
the Council of Ministers, leadership of the the aid and advice will be unconstitutional
Lok Sabha, command over the bureaucratic as being violative of Article 74.
machine, access to media, projection of • Again in 1974, the court held that
personalities during elections, projection as ‘wherever the Constitution requires
national leader during international summitry the satisfaction of the President, the
as well as foreign visits. satisfaction is not the personal satisfaction
of the President but it is the satisfaction
of the council of ministers with whose
(19) COUNCIL OF MINISTERS aid and on whose advice the President
Q.665 Consider the following with regard to the exercises his powers and functions’.
relationship between the President, Prime Q.666 The functions of the Indian Government
Minister and the Council of Ministers. is carried out by a number of Ministry
1. In normal circumstances, the advice of and Departments. Consider the following
the council of Ministers is constitutionally statements:
binding on the President. 1. No minister can openly criticise any
2. The nature of advice tendered by decision of the government, even if it is
ministers to the President can be enquired about another Ministry or Department.
by higher court to ascertain malpractices 2. Every ministry has secretaries, who are
in administration. civil servants.
3. The council of ministers ceases to hold 3. The secretaries provide the necessary
office immediately after the dissolution of background information to the ministers
the Lok Sabha by the President. to take decisions.
Select the correct answer using the codes 4. The Prime Ministers decisions are final
below. in case disagreements arise between
(a)  1 and 2 only (b)  1 only Departments.
(c)  2 and 3 only (d)  1 and 3 only Which of the statements is/are incorrect?
Solution: (b) (a)  1 and 4 (b)  1, 2 and 4
Justification: Statement 1: Article 74 (c)  1, 2, 3 and 4 (d)  None
provides for a council of ministers with Solution: (d)
the Prime Minister at the head to aid and Cabinet Ministers are usually top-level
advice the President in the exercise of his leaders of the ruling party or parties who are
functions. The 42nd and 44th Constitutional in charge of the major ministries. Since it is
Amendment Acts have made the advice not practical for all ministers to meet regularly
binding on the President. and discuss everything, the decisions are
Statement 2: Further, the nature of advice taken in Cabinet meetings. The cabinet works
tendered by ministers to the President cannot as a team. The ministers may have different

P.176 For Civil Services Preliminary Examination

02-Indian Polity_Q413-927.indd 176 8/7/2018 7:40:36 PM


views and opinions, but everyone has to advice of the council of ministers, thus
own up to every decision of the Cabinet. No it helps him.
minister can openly criticise any decision of Solution: (d)
the government, even if it is about another Justification: Option (a): This option may
Ministry or Department. Every ministry seem correct as in absence of council of
has secretaries, who are civil servants. The Ministers, bureaucracy becomes an important
secretaries provide the necessary background decisionmaking authority. But, such extreme
information to the ministers to take decisions. generalizations should not be chosen as the
answer. (d) is more appropriate.
Article 74 – Council of Ministers to Aid Option (b): Once the house is dissolved,
the very idea of accountability to the
and Advice President legislature dissolves for the time being. Only
Q.667 Consider the following statements. the President then issues orders.
1. The President may require the Council of Option (c): Once the house is dissolved,
Ministers to reconsider advice tendered the Counsil of Minister cannot prove their
by it. majority again. So, it is wrong.
2. The advice tendered by Ministers to the Option (d): Article 74 is mandatory and,
President shall not be inquired into in any therefore, the president cannot exercise the
court. executive power without the aid and advice
Which of the above is/are correct? of the council of ministers. Any exercise of
(a)  1 only (b)  2 only executive power without the aid and advice
(c)  Both 1 and 2 (d)  None will be unconstitutional as being violative of
Solution: (c) Article 74’.
Justification: As per Article 74—Council of Q.669 Consider the following statements:
Ministers to aid and advice President. 1. An ordinary citizen (not an MP) can also
There shall be a Council of Ministers become a minister for a period of at least
with the Prime Minister at the head to aid 6 months.
and advise the President who shall, in the 2. The council of ministers collapse as soon
exercise of his functions, act in accordance as the Lok Sabha is dissolved.
with such advice. Which of these is /are true?
However, the President may require (a)  Only 1 (b)  Only 2
the Council of Ministers to reconsider (c) Both (d) None
such advice and the President shall act in Solution: (a)
accordance with the advice tendered after
Q.670 Consider the following statements.
such reconsideration.
1. The advice tendered by Ministers to the
Q.668 In 1971, the Supreme Court held that even President shall not be inquired into by
after the dissolution of the Lok Sabha, the any court.
council of ministers does not immediately 2. After the dissolution of the Lok Sabha, the
cease to hold office. How does it help the council of ministers does not cease to hold
political system and administration in India? office with immediate effect.
(a) It prevents the bureaucracy from usurping 3. A minister who is a member of one House
political power. of Parliament has the right to speak and to
(b) It ensures the accountability of the take part in the proceedings of the other
executive to the legislature. House also.
(c) The Council of Ministers get an Choose the correct answer from the codes
opportunity to again prove their party below.
majority on the house floor. (a)  1 and 2 only (b)  2 and 3 only
(d) The president cannot exercise the (c)  1 and 3 only (d)  All of the above
executive power without the aid and Solution: (d)

Indian Polity Question Bank P.177

02-Indian Polity_Q413-927.indd 177 8/7/2018 7:40:36 PM


Q.671 The Ministers are individually responsible to Ministers, all ministers including the Prime
(a)  the President Minister, have to quit. The Rajya Sabha does
(b)  the House of the people not have this power.
(c)  the Prime Minister Q.674 The principle of collective responsibility
(d) the House of which they are members is the bedrock principle ofparliamentary
Solution: (a) government. The principle implies that the
1. Lok Sabha can remove the council of
Article 75 – Collective Responsibility ministers from office by passing a vote of
no confidence.
and Other Provisions 2. The Council of Ministers as a whole is
Q.672 In appointing Ministers, which of the bound by consensus.
following are followed by the President and Which of the above is/are correct?
the Governor? (a)  1 only (b)  2 only
1. Constitution of India (c)  Both 1 and 2 (d)  None
2. Laws made by Parliament Solution: (c)
3. Conventions Learning: The ministers are collectively
Select the correct answer using the codes responsible to the Parliament in general and
below: to the Lok Sabha in particular (Article 75).
(a)  1 and 2 only (b)  2 and 3 only They act as a team, and swim and
(c)  1 and 3 only (d)  All of the above sink together. The principle of collective
Solution: (d) responsibility implies that the Lok Sabha can
Constitution of India prescribes that Minister remove the ministry (i.e., council of ministers
will be appointed on the advice of Prime headed by the prime minister) from office by
Minister and CM respectively in Centre passing a vote of no confidence.
and States. The 91st amendment restricts Members of the council of ministers are
the Council of Ministers to 15% of the total bound by consensus. The government cannot
strength of Lok Sabha. Other conventions have two opinions on the same issue.
like Majority party leader; coalition leader Q.675 Consider the following statements.
etc. are also followed when appointing Assertion (A): A Ministry which loses
Ministers. confidence of the Rajya Sabha is obliged to
Q.673 The Council of Ministers is held accountable resign.
to and controlled by Reason (R): The Council of Ministers is
(a)  Both houses of Parliament collectively responsible to the Rajya Sabha.
(b)  Chairman, Rajya Sabha In the context of the above, which of these
(c)  Lok Sabha is correct?
(d)  President of India (a) A is correct, and R is an appropriate
Solution: (c) explanation of A.
Explanation: Parliament consists of both (b) A is correct, but R is not an appropriate
houses. Rajya Sabha is not an elected explanation of A.
chamber and thus cannot control the council (c)  A is correct, but R is incorrect.
of ministers. Option (a) is wrong. Hence, (d)  Both A and R are incorrect.
option (b)  is also wrong. Solution: (d)
President of India can only advise/ Justification: One of the most important
question the CoM on matters sent by it for features of a parliamentary democracy is that
consideration. So, the option (d) is wrong. the executive is routinely under the control
Learning: Only a person who enjoys the and supervision of the legislature.
support of the majority of the members in the • This provision means that a Ministry
Lok Sabha is appointed the Prime Minister. If which loses confidence of the Lok Sabha
the majority of the Lok Sabha members say (Rajya Sabha) is obliged to resign. The
they have ‘no confidence’ in the Council of principle indicates that the ministry is an

P.178 For Civil Services Preliminary Examination

02-Indian Polity_Q413-927.indd 178 8/7/2018 7:40:37 PM


executive committee of the Parliament Justification and Learning: The 91st
and it collectively governs on behalf of Amendment Act of 2003 has made the
the Parliament. following provisions to limit the size of
• Collective responsibility is based on the Council of Ministers:
principle of the solidarity of the cabinet. 1. The total number of ministers, including
It implies that a vote of no confidence the Prime Minister, in the Central Council
even against a single minister leads to of Ministers shall not exceed 15 per cent
the resignation of the entire Council of of the total strength of the Lok Sabha
Ministers. (Article 75).
• It also indicates that if a minister does 2. The total number of ministers, including
not agree with a policy or decision of the Chief Minister, in the Council of
the cabinet, he or she must either accept Ministers in a state shall not exceed 15 per
the decision or resign. It is binding on all cent of the total strength of the Legislative
ministers to pursue or agree to a policy for Assembly of that state. But, the number of
which there is collective responsibility. ministers, including the Chief Minister, in a
Q.676 Article 74 provides for a council of ministers state shall not be less than 12 (Article 164).
with the Prime Minister at the head to aid and Q.678 Before the 91st Amendment Act (2003),
advice the President. Which of the following the size of the Council of Ministers was
is/are correct with regard to council of determined by the
ministers? (a) Parliament
1. The size of the Council of Ministers (b)  Exigencies of time and situation
should not exceed 15 percent of total (c)  Total population size of the country
number of members of the House of (d) Number of constituencies in the country
People and this is also applicable to the Solution: (b)
Legislative Assembly in the States. Learning: This convention before 2003 led
2. The Council of Ministers is collectively to widespread abuse of the system.
responsible to both the Lok Sabha and It led to very large size of the Council of
Rajya Sabha. Ministers. Besides, when no party had a clear
3. Vote of no confidence even against a majority, there was a temptation to win over
single minister leads to the resignation of the support of the members of the Parliament
the entire Council of Ministers. by giving them ministerial positions as there
4. The death or resignation of the Prime was no restriction on the number of the
Minister automatically brings about the members of the Council of Ministers.
dissolution of the Council of Ministers. This was happening in many States also.
Select the correct answer using the codes Therefore, an amendment was made that the
below: Council of Ministers shall not exceed 15
(a)  1 Only (b)  1, 2 and 4 Only percent of total number of members of the
(c)  1, 3 and 4 Only (d)  1, 2, 3 and 4 Only House of People (or Assembly, in the case of
Solution: (c) the States).
Q.677 The total number of ministers, including the Q.679 Consider the following statements
Prime Minister, in the Central Council of 1. The President of India is the constitutional
Ministers is head of executive of the Union
(a) Decided by an order of the President 2. The Council of Ministers are collectively
(b) Set according to the will of the ruling responsible to both the houses of the
party Parliament
(c) Fixed by the Representation of People Which of the above statements is/are correct?
Act, 1950 (a)  1 Only (b)  2 Only
(d)  None of the above is correct. (c) Both (d) None
Solution: (d) Solution: (a)

Indian Polity Question Bank P.179

02-Indian Polity_Q413-927.indd 179 8/7/2018 7:40:37 PM


According to the Part V, Article 75 of the 1. Generally a Civil Servant/bureaucrat
Indian Constitution, the Council of Ministers holding an “Office of Profit”
are collectively responsible to only to the 2. Assigned charge of an entire government
Lok Sabha. department only in the absence of Cabinet
Q.680 Consider the following statements: Ministers
1. No minister can openly criticise any Which of the above is/are correct?
decision of the government, even if it is (a)  1 only (b)  2 only
about another Ministry or Department. (c)  Both 1 and 2 (d)  None
2. Every ministry has secretaries, who are Solution: (d)
civil servants. Justification and Learning: Parliamentary
3. The Cabinet as a team is assisted by the secretaries are ruling party members
Cabinet Secretariat. appointed to assist ministers. They often
Choose the correct answer using the codes holds the rank of Minister of State and has
below: the same entitlements and is assigned to a
(a)  All of the above (b)  1 and 3 only government department. They have access to
(c)  1 and 2 only (d)  2 and 3 only all official files and documents.
Solution: (a) • Manipur, Himachal Pradesh, Mizoram,
Since it is not practical for all ministers to Assam, Rajasthan, Punjab, Goa are some
meet regularly and discuss everything, the of the states where MLAs have been
decisions are taken in cabinet meetings. That appointed Parliament Secretaries by the
is why parliamentary democracy in most Government.
countries is often known as the cabinet form • Various petitions in the High Court have
of government. The cabinet works as a team. challenged the appointment of Parliament
The ministers may have different views and Secretary, including in Delhi most
opinions, but everyone has to own up to recently.
every decision of the Cabinet. It is called the • In June 2015, Calcutta HC quashed
principle of Collective Responsibility. appointment of 24 Parliamentary
Secretaries in West Bengal dubbing it
unconstitutional.
Composition of Council of Ministers Delhi High Court in 2016 quashed the
Q.681 The Council of Ministers does not include appointment of Parliamentary secretaries
which of the following? as they are deemed to be Ministers, and
1. Ministers without portfolio there is cap on the number of ministers
2. Cabinet Secretary in the government as per the Constitution
Choose the correct answer using the codes (The limit is 10% for Delhi, owing to its
below: special status).
(a)  Only 1 (b)  Only 2 Q.683 Consider the following.
(c)  Both (d)  None of the above 1. Government resigns if it fails to enact a
Solution: (b) public bill in Parliament.
Ministers without portfolio are also ministers 2. Prime Minister is a member of Lok Sabha.
officially. Hence, they are a part of the 3. The council of Ministers reflects the
council of ministers. The cabinet secy. is a socio-economic diversity of India.
bureaucrat and he cannot a part of the council Which of these conventions has/have
of ministers for he is a civil servant who is not been codified into rules/regulations/laws/
responsible to the Parliament. He, however, provisions?
attends the cabinet meetings. (a)  1 only (b)  2 and 3 only
Q.682 Many states in the Indian Union have (c)  1 and 3 only (d)  None of the above
instituted the post of Parliamentary Secretary. Solution: (d)
A Parliament Secretary is

P.180 For Civil Services Preliminary Examination

02-Indian Polity_Q413-927.indd 180 8/7/2018 7:40:37 PM


The government may resign only if it fails to the supreme decision-making body in India.
pass the budget in the Parliament. Failure in Only the Prime Minister and ministers of the
passing regular public bills is not considered rank of “Cabinet Minister” are members of the
a defeat of the government. Passing of a Cabinet.
no-confidence motion does. Prime Minister Q.686 Consider the following statements.
can be the member of any house. It is not a Assertion (A): Unlike the Council of
convention that he must be from the lower Ministers, the constitution does not mention
house, neither has it been codified. Statement the powers and functions of the cabinet.
3 is more of a political tool rather than a Reason (R): The cabinet does not find any
codified convention. mention in the constitution.
In the context of the above, which of these
Cabinet is correct?
(a) A is correct, and R is an appropriate
Q.684 The Central Cabinet includes which of the
explanation of A.
following?
(b) A is correct, but R is not an appropriate
1. The Prime Minister
explanation of A.
2. Minister of State with Independent charge
(c)  A is correct, but R is incorrect.
3. Cabinet Secretary
(d)  Both A and R are incorrect.
Choose the correct answer using the codes
Solution: (c)
below:
(a)  All of the above Q.687 Who among the following can be invited to
(b)  1 and 3 only the meetings of the Cabinet?
(c)  1 and 2 only 1. Cabinet Secretary
(d)  1 only 2. Union Ministers of State
Solution: (d) 3. Deputy Ministers
Cabinet Secretary only attends the meetings Choose the correct answer from the codes
of the Cabinet. He is not a part of it. below.
No one except the Prime Minister and (a)  1 and 2 only (b)  2 and 3 only
cabinet Ministers are included in the Central (c)  1 and 3 only (d)  All of the above
Cabinet. Some ministers Solution: (a)
The ministers of state can either be given
Q.685 Consider the following statements:
independent charge of ministries/departments
Assertion (A): Unlike the Council of
or can be attached to cabinet ministers. In
Ministers, the constitution does not mention
case of attachment, they may either be given
the powers and functions of the cabinet.
the charge of departments of the ministries
Reason (R): The cabinet is an extra-
headed by the cabinet ministers or allotted
constitutional body.
specific items of work related to the ministries
In the context of these above two
headed by cabinet ministers. In both the
statements above, which one of the following
cases, they work under the supervision and
is correct?
guidance as well as under the overall charge
(a) Both A and R are true and R is the correct
and responsibility of the cabinet ministers. In
explanation of A.
case of independent charge, they perform the
(b) Both A and R are true but R is not a
same functions and exercise the same powers
correct explanation of A.
in relation to their ministries/departments as
(c) A is true, but R is false.
cabinet ministers do.
(d) A is false, but R is true.
However, they are not members of the
Solution: (c)
cabinet and do not attend the cabinet meetings
The word cabinet was inserted in the
unless specially invited when something
constitution in Article 352 in 1978 by the
related to their ministries/departments are
44th CA Act. It only defines the cabinet, not
considered by the cabinet. Next in rank are
its powers and functions. Union Cabinet is

Indian Polity Question Bank P.181

02-Indian Polity_Q413-927.indd 181 8/7/2018 7:40:37 PM


the deputy ministers. They are not given number, nomenclature, and composition
independent charge of ministries/departments. varies from time to time.
They are attached to the cabinet ministers • Their membership vary from three to
or ministers of state and assist them in their eight. They usually include only Cabinet
administrative, political, and parliamentary Ministers. However, the non-cabinet
duties. They are not members of the cabinet Ministers are not debarred from their
and do not attend cabinet meetings. membership.
Q.688 Which of the following is/are NOT mentioned • They do not consist only of ministers, but
in the Constitution of India? can also contain ordinary citizens.
1. Powers and functions of the Union • They not only include the Ministers in
Cabinet charge of subjects covered by them but also
2. Powers and functions of the State Cabinets include other senior ministers.
3. Cabinet Committees • They are mostly headed by the Prime
Choose the correct answer from the codes Minister. Sometimes other Cabinet
below. Ministers, particularly the Home Minister
(a)  1 and 2 only (b)  2 and 3 only or the Finance Minister, also acts as their
(c)  1 and 3 only (d)  All of the above Chairman. But, in case the Prime Minister
Solution: (d) is a member of a committee, he invariably
The word ‘cabinet; was inserted in Article presides over it.
352 of the Constitution in 1978 by the 44th Q.690 Consider the following about Cabinet
Constitutional Amendment Act. Thus, it did Committees.
not find a place in the original text of the 1. They are extra-constitutional bodies.
Constitution. 2. They are only temporary in nature.
Now also, Article 352 only defines 3. Non-cabinet ministers cannot be its
the cabinet saying that it is ‘the council members.
consisting of the prime minister and other 4. Only the Prime Minister is authorized to
ministers of cabinet rank appointed under chair cabinet committees.
Article 75’ and does not describe its powers Select the correct answer using the codes
and functions. In other words, its role in our below.
politico-administrative system is based on the (a)  1 and 2 only (b)  3 and 4 only
conventions of parliamentary government as (c)  1 only (d)  2 and 4 only
developed in Britain. Solution: (c)
Justification: Statement 1: They are not
mentioned in the Constitution. However,
(20) CABINET COMMITTEES the Rules of Business provide for their
Q.689 Consider the following statements about establishment
Cabinet Committees. Statement 2: They are of two types—
1. They are setup by the President. standing and ad hoc. The former are of a
2. Non-cabinet ministers cannot become permanent nature while the latter are of a
members of these committees. temporary nature. So,the option 2 is wrong.
3. All cabinet committees are headed by the Statement 3 and 4 : As explained in
Home Minister. previous question.
Choose the correct answer from the codes Q.691 Consider the following about cabinet
below. committees.
(a)  1 and 2 only (b)  2 only 1. They are set up by the Prime Minister.
(c)  1 and 3 only (d)  None of the above 2. All committees are ad hoc in nature.
Solution: (d) 3. They cannot take a final decision on
• They are set up by the Prime Minister matters referred to them by cabinet.
according to the exigencies of the time and 4. Cabinet committees are also known as
requirements of the situation. Hence, their Empowered Group of Ministers (EGoM).

P.182 For Civil Services Preliminary Examination

02-Indian Polity_Q413-927.indd 182 8/7/2018 7:40:37 PM


Choose the correct answer using the codes Q.693 Consider the following about Cabinet
below. Committees in India.
(a)  1 and 3 only (b)  2, 3 and 4 only 1. They are not mentioned in the Constitution
(c)  1 only (d)  3 only and the Rules of Business of Council of
Solution: (c) Ministers.
They not only sort out issues and formulate 2. They cannot take final decisions on
proposals for the consideration of the matters referred to them by the Cabinet,
Cabinet, but also take decisions. However, which must be taken by the Council of
the Cabinet can review their decisions. Ministers as a whole.
Empowered Group of Ministers (EGoM) Which of the above is/are correct?
is a Group of Ministers (GoM) of the Union (a)  1 only (b)  2 only
Government who, after being appointed (c)  Both 1 and 2 (d)  None
by the Cabinet, a Cabinet Committee Solution: (d)
or the Prime Minister for investigating Justification: Statement 1: They are extra-
and reporting on such matters as may be constitutional in emergence. In other words,
specified, are also authorised(empowered) they are not mentioned in the Constitution.
by the appointing authority to take decisions However, the Rules of Business provide for
in such matters after investigation. their establishment.
As part of empowering the Ministries and Statement 2: explained in previous questions
Departments, the Prime Minister on 31 may Q.694 Cabinet Committee on Appointments is
2014 decided to abolish all the existing nine headed by
Empowered Group of Ministers (EGoMs) (a)  President of India
and 21 Groups of Ministers (GoMs). This is (b)  Prime Minister
expected to expedite the process of decision (c)  Union Minister of Home affairs
making and usher in greater accountability in (d)  Union Minister of Finance
the system. Solution: (b)
Q.692 Who among the following can review the Learning: The Appointments Committee of
decisions taken by the Cabinet Committees? the Cabinet (ACC) decides appointments to
1. Prime Minister several top posts under the Government of
2. Cabinet India.
3. Parliament • The committee is composed only of
4. Council of Ministers the Prime Minister of India (who is the
Choose the correct answer from the codes Chairman), and the Minister of Home
below. Affairs.
(a)  1 and 2 only (b)  2 and 4 only • The Establishment Officer’s Division
(c)  2 only (d)  1 and 3 only (EO Division) processes all proposals for
Solution: (a) senior appointments in the Government
Cabinet Committees not only sort out of India that require approval of the
issues and formulate proposals for the Appointments Committee of the Cabinet.
consideration of the Cabinet, but also take • The Establishment Officer (EO) is the
decisions. Ex: appointments. However, the ex-officio Member Secretary of the Civil
Cabinet can review their decisions. It means Services Board that is chaired by the
that the Prime Minister too can review their Cabinet Secretary.
decisions. Q.695 Which of these are the members of the
They are an organisational device to reduce Cabinet Committee on Economic Affairs
the enormous workload of the Cabinet. They (CCEA) is headed by
also facilitate in-depth examination of policy (a)  Minister of Home Affairs
issues and effective coordination. They are (b)  Minister of External Affairs
based on the principles of division of labour (c)  Minister of Corporate Affairs
and effective delegation. (d)  prime minister

Indian Polity Question Bank P.183

02-Indian Polity_Q413-927.indd 183 8/7/2018 7:40:37 PM


Select the correct answer using the codes by the Prime Minister. The CCEA also
below. considers cases of increase in the firmed
Solution: (d) up cost estimates/revised cost estimates
The Cabinet Committee on Economic for projects etc. in respect of the business
Affairs (CCEA) has a mandate to review allocated to the CCEA.
economic trends on a continuous basis, as Q.696 The Cabinet Committee on Security includes
also the problems and prospects, with a 1. Prime Minister
view to evolving a consistent and integrated 2. Minister of Home Affairs
economic policy framework for the 3. Minister of Finance
country. It also directs and coordinates all 4. Minister of Defence
policies and activities in the economic field Select the correct answer using the codes
including foreign investment that require below.
policy decisions at the highest level. (a)  1, 2 and 4 only (b)  2 and 3 only
Matters regarding fixation of prices of (c)  1 and 4 only (d)  1, 2, 3 and 4
agricultural products as well as reviewing Solution: (d)
progress of activities related to rural Justification: It also includes Minister of
development including those concerning Corporate Affairs; Minister of External
small and marginal farmers are in CCEA’s affairs and Minister of Information and
competence. Broadcasting.
Price controls of industrial raw materials The Cabinet Committee on Security
and products, industrial licensing policies (CCS) of the Central Government of India
including industrial licensing cases for decides on India’s defence expenditure,
establishment of Joint Sector Undertakings, matters of National Security, and makes
reviewing performance of Public Sector significant appointments.
Undertakings including their structural and Other committees of strategic nature are:
financial restructuring are also within the • Cabinet Committee on Economic Affairs
purview of CCEA, as are all matters relating • Cabinet Committee on Investment
to disinvestment including cases of strategic • Cabinet Committee on Political Affairs
sale, and pricing of Government shares in
Public Sector Undertakings (except to the
extent entrusted to an Empowered Group (21) CABINET SECRETARIAT
of Ministers).
Q.697 Consider the following statements about the
The CCEA also lays down priorities
Cabinet Secretariat.
for public sector investment and considers
1. It functions directly under the Ministry of
specific proposals for investment of not less
Personnel and Public Grievances.
than specific levels (Rs. 3 Billion at present)
2. It is responsible for the administration of
as revised from time to time. It is important to
the Government of India (Transaction of
note that the increase in the prices of essential
Business) Rules.
commodities or bulk goods under any form of
3. It ensures that the President, the Vice-
formal or informal control is decided by the
President and Ministers are kept informed
CCEA, even as the CCP monitors the price
of the major activities of all ministries and
behaviour of essential commodities, takes
departments.
decision on supply, imports and exports of
4. The administrative head of the Secretariat
essential commodities and prices for articles
is the ex-officio Chairman of the Civil
sold through the public distribution system.
Services Board.
CCEA facilitates finalization of factual
Select the correct answer using the codes
reports on the accomplishments of the
below.
Ministries, Agencies and Public Sector
(a)  1 and 4 only (b)  2, 3 and 4 only
Undertakings involved in implementation of
(c)  2 and 3 only (d)  1, 2, 3 and 4
prioritized schemes or projects for evaluation
Solution: (b)

P.184 For Civil Services Preliminary Examination

02-Indian Polity_Q413-927.indd 184 8/7/2018 7:40:37 PM


Justification: Statement 1: The Cabinet (22) PARLIAMENT
Secretariat functions directly under the Prime
Minister. Q.699 The word „Parliament‟ means
Statement 2 and 3: The business allocated (a)  Lok Sabha
to Cabinet Secretariat is (i) Secretarial (b)  Lok Sabha and Rajya Sabha
assistance to Cabinet and Cabinet Committees (c) President, Lok Sabha and Rajya Sabha
and (ii) Rules of Business. (d) President, Council of Ministers, Lok
• The Cabinet Secretariat is responsible Sabha and Rajya Sabha
for the administration of the Government Solution: (c)
of India (Transaction of Business) It is self-explanatory. Even though President
Rules, 1961 and the Government of does not attend the Parliamentary sessions,
India (Allocation of Business) Rules, his nod is required for a bill to become an
1961, facilitating smooth transaction of Act. Hence, he is considered an integral part
business in ministries/departments of the of the Parliament.
Government by ensuring adherence to Q.700 Consider the following statements.
these rules. Assertion (A): The First Lok Sabha met
• It assists in decision-making by ensuring on the eve of the first republic day.
inter-ministerial coordination, ironing Reason (R): India ceased to be a British
out differences amongst ministries/ dominion post enactment of constitution.
departments and evolving consensus In the context of the above, which of these
through the instrumentality of the is correct?
standing and ad hoc Committees of (a) A is correct, and R is an appropriate
Secretaries. explanation of A.
• Management of major crisis situation in (b) A is correct, but R is not an appropriate
the country and coordinating activities of explanation of A.
various ministries in such a situation is (c)  A is correct, but R is incorrect.
also one of the functions of the Cabinet (d)  A is incorrect, but R is correct.
Secretariat. Solution: (d)
Statement 4: The administrative head of the Justification: First Lok Sabha met in 1952
Secretariat is the Cabinet Secretary who Under GV Mavalankar. So, A is wrong.
is also the Chairman of the board of Civil The Indian Independence Act 1947 merely
Services. converted India into a British dominion.
Q.698 Consider the following statements about the It ceased to be dominion only after 26th
Cabinet Secretariat. January 1950.
1. It is under the direct charge of the Union Till 1952, the Constituent assembly itself
Minister of Personnel. functioned as the law making body.
2. It is the custodian of the papers of the It was only after the first general elections
Cabinet meetings. that Lok Sabha was constituted and the
3. It watches the implementation of the assembly was relived of its responsibility of
decisions taken by the Cabinet. acting as a dual body.
4. Management of major crisis situations in the
country is one of its functions. Organisation of Parliament
Select the correct answer using the codes
below. Composition of Two Houses
(a)  2, 3 and 4 only (b)  3 and 4 only
Q.701 With reference to the Parliamentary seats, the
(c)  2 and 3 only (d)  1, 2, 3 and 4
Constitution ensures that there is uniformity
Solution: (a)
of representation
1. between the states
2. between the different constituencies of the
state

Indian Polity Question Bank P.185

02-Indian Polity_Q413-927.indd 185 8/7/2018 7:40:37 PM


Which of the above correctly complete the (a) Fundamental Rights section of the
above statement? Constitution
(a)  1 only (b)  2 only (b) Representation of People Act, 1951
(c)  Both 1 and 2 (d)  None (c) Part V of the Constitution that deals with
Solution: (c) the Parliament
For the purpose of holding direct elections (d)  None of the above
to the Lok Sabha, each state is divided into Solution: (d)
territorial constituencies. In this respect, Learning: There are two provisions in the
the Constitution makes the following two Constitution that seek to achieve political
provisions. equality.
• Each state is allotted a number of seats • One, no person is to be declared
in the Lok Sabha in such a manner that ineligible for inclusion in electoral rolls
the ratio between that number and its on grounds of religion, race, caste or sex
population is the same for all states. This (Article 325).
provision does not apply to a state having • Two, elections to the Lok Sabha and the
a population of less than 6 millions. state assemblies to be on the basis of adult
• Each state is divided into territorial suffrage (Article 326).
constituencies in such a manner that the • Both these provisions are constitutionaL,
ratio between the population of each but cannot be found in options A, B or C.
constituency and the number of seats So, all of them are wrong.
allotted to it is the same throughout Therefore, the right to vote is basically a
the state. constitutional right, not a fundamental right.
Q.702 Consider the following statements: Q.704 The Constitution has empowered the
1. The parliament gives representation to all Parliament to prescribe the manner of
the Union Territories (UTs). choosing the representatives of the union
2. All parliamentary constituencies territories in the Lok Sabha. Members of Lok
have equal population as each state is allotted Sabha from the Union Territories are
seats based on its population. 
Which of these (a)  Chosen by direct election
is /are true? (b) Nominated by the Administrator/
(a)  Only 1 (b)  Only 2 Lieutenant Governor of the UT
(c) Both (d) None (c) Recommended by the Union Cabinet and
Solution: (a) chosen by the President
The Lok Sabha does not represent all the (d) Chosen by indirect elections by the local
UTs, but the Rajya Sabha does. So the bodies of the UTs
Parliament as a whole represents all UTs. Solution: (a)
Even while we follow a system where states Learning: Accordingly, the Parliament has
are allocated seats based on its population, all enacted the Union Territories (Direct Election
constituencies do not have same population. to the House of the People) Act, 1965, by
Also, some states are less populated and which the members of Lok Sabha from the
therefore they are represented more than their union territories are also chosen by direct
population makes them eligible to, example election. The representatives of each union
given, Nagaland. territory in the Rajya Sabha are indirectly
elected by members of an electroral college
Composition of Lok Sabha specially constituted for the purpose. This
election is also held in accordance with the
Q.703 Elections to the Lok Sabha and the state system of proportional representation by
assemblies will be held on the basis of adult means of the single transferable vote. Out of
suffrage is a provision that was originally the seven union territories, only two (Delhi
instituted in and Puducherry) have representation in

P.186 For Civil Services Preliminary Examination

02-Indian Polity_Q413-927.indd 186 8/7/2018 7:40:37 PM


Rajya Sabha. The populations of other five (b) Members of Legislative assemblies
union territories are too small to have any (c) Members of Local constitutional bodies
representative in the Rajya Sabha. (d) MPs, MLAs and ward councilors jointly
from a Union Territory
Composition of Rajya Sabha Solution: (a)
Refer previous explaination
Q.705 Consider the following statements.
Q.708 Consider the following statements about the
1. Rajya Sabha contains both nominated and
representation in Rajya Sabha.
indirectly elected members.
1. All the Union Territories (UTs) are
2. The members of Rajya Sabha are elected
represented in Rajya Sabha.
by both Legislative assembly and
2. Members in Rajya Sabha are nominated
Legislative council of the states.
by the president only on the advice of the
Which of these is/are true?
Council of Ministers.
(a)  1 only (b)  2 only
3. Nominated members in Rajya Sabha are
(c)  Both 1 and 2 (d)  None
more than those representing the UTs.
Solution: (a)
Choose the correct answer using the codes
They are elected only by the Legislative
below.
assembly.
(a)  1 and 2 only (b)  2 and 3 only
Q.706 The representatives of each union territory in (c)  1 and 3 only (d)  All of the above
the Rajya Sabha are Solution: (b)
(a) Nominated by the Union Territory (UT) The maximum strength of the Rajya Sabha
Government is fixed at 250, out of which, 238 are to be
(b) Indirectly elected by members of an the representatives of the states and union
electoral college specially constituted for territories (4 members elected indirectly) and
the purpose. 12 are nominated by the president.
(c)  Nominated by the President Out of the seven union territories, only two
(d)  Not represented in the house (Delhi and Puducherry) have representation
Solution: (b) in Rajya Sabha. The populations of other five
Learning: At present, the Rajya Sabha union territories are too small to have any
has 245 members. Of these, 229 members representative in the Rajya Sabha.
represent the states, 4 members represent
Q.709 Consider the following statements about
the union territories and 12 members are
Rajya Sabha:
nominated by the president.
1. All Union Territories (UTs) are not
• The representatives of each union territory
represented in the Rajya Sabha.
in the Rajya Sabha are indirectly elected
2. There is provision for the reservation of
by members of an electroral college
Scheduled Castes and Scheduled Tribes
specially constituted for the purpose.
both in the Lok Sabha and Rajya Sabha.
• This election is also held in accordance
3. The constitution fixes the term of a Rajya
with the system of proportional
Sabha member as six years.
representation by means of the single
Which of these is/are true?
transferable vote. Out of the seven
(a)  1 and 2 (b)  2 and 3
union territories, only two (Delhi and
(c)  1 and 3 (d)  Only 1
Puducherry) have representation in Rajya
Solution: (d)
Sabha. The populations of other five
Only Delhi and Puducherry are represented.
union territories are too small to have any
Reservation is there only in the Lok Sabha.
representative in the Rajya Sabha.
Not the constitution, but the Representation
Q.707 The representative of each Union territory in of People’s act fixes the term at 6 years for a
the Rajya Sabha are indirectly elected by RS MP.
(a) Members of an electoral college specially
constituted for this purpose

Indian Polity Question Bank P.187

02-Indian Polity_Q413-927.indd 187 8/7/2018 7:40:37 PM


Q.710 Consider the following statements: In other electoral systems, especially PR
1. Seats for the Schedules tribes are reserved systems, voters are often asked to choose a
both in Lok Sabha and Rajya Sabha. party and the representatives are elected on
2. Seats for Women are reserved at all levels the basis of party lists. As a result, there is
of local government–both Panchayat and no one representative whorepresents and is
Municipality. responsible for one locality. In constituency
Which of these is/are true? based system like the FPTP, the voters know
(a)  1 only (b)  2 only who their own representative is and can hold
(c)  Both 1 ad 2 (d)  None of the above him or her accountable.
Solution: (b)
No seats are reserved in Rajya Sabha as it Model Code of Conducts in Rajya Sabha
represents the interests of the state.
Not just the village Panchayat, but also Q.712 Consider the following about Rajya Sabha
the Block and Zila Panchayats has seats elections.
reserved for Women. It is the same for urban 1. Model code of conduct is not applicable to
government too. these elections.
2. Anti-defection law does not apply and
candidates can vote for cross-party
System of Elections in Lok Sabha candidates.
Q.711 Which of the following reasons may justify 3. Election Commission of India has no
India following the system of FPTP system powers to countermand Rajya Sabha
for Lok Sabha elections? elections.
1. Election system is easy to understand. Select the correct answer using the codes
2. Clear choice to voters in terms of choosing below.
candidates (a)  1 and 2 only (b)  1 and 3 only
3. Size and diversity of India. (c)  2 and 3 only (d)  1, 2 and 3
Choose the correct answer using the codes Solution: (a)
below: Justification: Statement 1: There is no
(a)  1 and 2 (b)  2 and 3 model code of conduct for the RS election
(c)  1 and 3 (d)  All of the above as the nature of the election is different.
Solution: (d) There are no public meetings, rallies, door-
Looking at the Rajya Sabha elections, it can to-door canvassing or media campaigns. The
be seen that it is a complicated system which constituency is confined to a small electoral
may work in a small country, but would be college consisting of MLAs. The voting is
difficult to work in a sub-continental country open so the party MLAs are subject to party
like India. discipline.
The reason for the popularity and success Statement 2: Experts have recommended
of the ‘first-past-the-post’ (FPTP) system that anti-defection provisions should apply in
is its simplicity. The entire election system case of RS elections.
is extremely simple to understand even for Statement 3: On a complaint brought
common voters who may have no specialised by three senior political leaders about
knowledge about politics and elections. rampant horse-trading among MLAs, the EC
There is also a clear choice presented to the countermanded the RS poll in Jharkhand in
voters at the time of elections. Voters have 2012. So, the option 3 is wrong.
to simply endorse a candidate or a party Q.713 Consider the following statements about
while voting. Depending on the nature of Rajya Sabha.
actual politics, voters may either give greater 1. The representatives of states in the Rajya
importance to the party or to the candidate Sabha are elected by the elected members
or balance the two. The FPTP system offers of state legislative assemblies.
voters a choice not simply between parties
but specific candidates.

P.188 For Civil Services Preliminary Examination

02-Indian Polity_Q413-927.indd 188 8/7/2018 7:40:37 PM


2. The election to Rajya Sabha is held other hand, parts of the country may be
in accordance with the system of given representation according to their
proportional representation by means of population.
the single transferable vote. • This second method means that regions
3. The seats are allotted to the states in the or parts having larger population would
Rajya Sabha on the basis of population. have more representatives in the second
Select the correct answer using the codes chamber than regions having less
below. population.
(a)  1 and 2 only (b)  2 and 3 only • In the U.S.A, every state has equal
(c)  1 and 3 only (d)  1, 2 and 3 only representation in the Senate. This ensures
Solution: (d) equality of all the states. But this also
Learning: The seats are allotted to the states means that a small state would have the
in the Rajya Sabha on the basis of population. same representation as the larger states.
Hence, the number of representatives varies The system of representation adopted for
from state to state. the Rajya Sabha is different from that in
For example, Uttar Pradesh has 31 the USA.
members while Tripura has 1 member only. • The number of members to be elected
However, in USA, all states are given from each State has been fixed by the
equal representation in the Senate irrespective fourth schedule of the Constitution.
of their population. Q.715 Elections to the Rajya Sabha from the State
USA has 50 states and the Senate has 100 legislative assemblies are done on the basis
members—two from each state. of proportional representation. This means
Q.714 The electoral system of Rajya Sabha follows (a) every district in the state is given due
which of the following principles? representation in Rajya Sabha from that
(a) Giving equal representation to all the state
parts of the country irrespective of their (b) all the states are given representation in
size or population Rajya Sabha
(b) Giving due representation to parts of (c) that all the states irrespective of their size
the country depending on their size or and population get due representation in
population Rajya Sabha
(c) Giving due representation to all (d)  None of the above
major political parties of the country Solution: (c)
irrespective of their size Option C is more preferable to option
(d) Giving due representation to all political B because the former is more complete
parties of the country depending on their and correct. Proportional representation
volunteer base essentially means representation that
Solution: (b) is proportionate to some factor e.g.
Learning: The Rajya Sabha represents the population here. The system of proportional
States of India. It is an indirectly elected representation aims at removing the defects of
body. Residents of the State elect members territorial representation. Under this system,
to State Legislative Assembly. The elected all sections of the people get representation in
members of State Legislative Assembly in proportion to their number. Even the smallest
turn elect the members of Rajya Sabha. section of the population gets its due share of
• We can imagine two different principles representation in the legislature.
of representation in the second chamber. Q.716 Which of the following is/are correct
One way is to give equal representation concerning the elections to Rajya Sabha?
to all the parts of the country irrespective 1. The nominee must be a domicile of the
of their size or population. We may call state from which he wishes to be elected.
this as symmetrical representation. On the 2. Voting is by secret ballot system.

Indian Polity Question Bank P.189

02-Indian Polity_Q413-927.indd 189 8/7/2018 7:40:37 PM


3. The elections are organized by the 2. The Election Commission of India revises
Election Commission of India. the election expenditure ceiling for
Choose the correct answer using the codes candidates for Parliamentary elections.
below. Which of these is/are true?
(a)  1 and 2 only (b)  2 and 3 only (a)  Only 1 (b)  Only 2
(c)  1 and 3 only (d)  3 only (c)  Both (d) None of the above
Solution: (d) Solution: (d)
In 2003, the following two changes were It is the central government which revises the
introduced with respect to elections to the election expenditure limits for Parliamentary
Rajya Sabha: elections. The requirement for being an
• Domicile or residency requirement of a elector of the state for being nominated to the
candidate contesting an election to the RS was waved in 2003.
Rajya Sabha was removed. Prior to this, a
candidate had to be an elector in the state Duration of Two Houses
from where he was to be elected. Now,
it would be sufficient if he is an elector Duration of Lok Sabha
in any parliamentary constituency in
the country. Q.719 Before the completion of a 5 year term, the
• Introducing open ballot system, instead Lok Sabha can be dissolved if
of secret ballot system, for elections to 1. No party or coalition can form the
the Rajya Sabha. This was done to curb government
cross-voting and to wipe out the role 2. The Prime Minister advices the President
of money power during Rajya Sabha to dissolve the Lok Sabha and hold fresh
elections. Under the new system, an elections
elector belonging to a political party has Which of the above is/are correct?
to show the ballot paper after marking (a)  1 only (b)  2 only
his vote to a nominated agent of that (c)  Both 1 and 2 (d)  None
political party Solution: (c)
Q.717 Consider the following statements about Justification: There are two ways in which
Elections to Rajya Sabha. the Lok Sabha can be dissolved.
1. A candidate has to be an elector in the 1. when the leader of the majority (the ruling
state from where he wishes to be elected party) dissolves the Lok Sabha. While in
in Rajya Sabha. this situation, the prerogative and timing
2. Secret Ballot system is used in Rajya is that of the leader of the majority, he has
Sabha elections. to dissolve the Lok Sabha if his 5-year
3. Both the Legislative assembly andthe term is up.
legislative council take part in the Rajya Former Indian Prime Minister Atal Bihari
Sabha elections. Choose the correct Vajpayee in 2004 recommended to President
answer from the codes given below. A P J Abdul Kalam that Parliament be
(a)  1 and 2 only dissolved.
(b)  2 and 3 only 2. when the incumbent government loses
(c)  1 and 3 only majority in the Lok Sabha: then, the
(d) None of the above prerogative passes to the President,
Solution: (d) who can ask another leader to prove his
It is the legislative assembly not legislative majority in the Lok Sabha or dissolve the
council. Lok Sabha, and call for elections.
Q.718 Consider the following statements: Q.720 Consider the following statements about the
1. A candidate has to be an elector in the term of Lok Sabha.
state from where he is being voted to 1. The President of India can reduce the term
Rajya Sabha. of Lok Sabha.

P.190 For Civil Services Preliminary Examination

02-Indian Polity_Q413-927.indd 190 8/7/2018 7:40:37 PM


2. The speaker of Lok Sabha by himself extended twice during the 1975 national
cannot shorten or extend the term of a emergency.
Lok Sabha.
3. Term of Lok Sabha can be extended Duration of Rajya Sabha
during emergency.
Choose the correct answer using the codes Q.722 Consider the following about Rajya Sabha.
below. 1. It is a permanent body and not subject to
(a)  1 and 2 only (b)  2 and 3 only dissolution.
(c)  1 and 3 only (d)  All of the above 2. The retiring members of Rajya Sabha are
Solution: (d) eligible for re-election and re-nomination
Unlike the Rajya Sabha, the Lok Sabha any number of times.
is not a continuing chamber. Its normal 3. The Constitution fixes the term of the
term is 5 years from the date of its first members of the Rajya Sabha.
meeting after the general elections, after 4. The President can curtail the term of a
which it automatically dissolves. However, Rajya Sabha member any time based on
the President is authorised to dissolve the the recommendation of the Union cabinet.
Lok Sabha at any time even before the Select the correct answer using the codes
completion of 5 years and this cannot be below.
challenged in a court of law. Further, the (a)  2, 3 and 4 only (b)  1 and 4 only
term of the Lok Sabha can be extended (c)  1 and 3 only (d)  1 and 2 only
during the period of national emergency Solution: (d)
be a law of Parliament for one year at a Justification: Statement 1: It is a continuing
time for any length of time. However, this chamber, that is, it is a permanent body and
extension cannot continue beyond a period not subject to dissolution. However, one-
of 6 months after the emergency has ceased third of its members retire every 2nd year.
to operate. Statement 2: The seats left are filled up by
fresh elections and presidential nominations
Q.721 The term of the Lok Sabha can be extended
at the beginning of every 3rd year. The
under which of these circumstances?
retiring members are eligible for re-election
1. When a national or financial emergency is
and re-nomination any number of times.
in operation
Statement 3: The Constitution has not
2. When a new government cannot be
fixed the term of office of members of the
formed even after the completion of the
Rajya Sabha and left it to the Parliament.
term of the previous Lok Sabha
Accordingly, the Parliament in the
3. When the Rajya Sabha approves so on
Representation of the People Act (1951)
a special recommendation made by the
provided that the term of office of a member
Prime Minister
of the Rajya Sabha shall be 6 years.
Select the correct answer using the codes
Statement 4: The act also empowered
below.
the president of India to curtail the term of
(a)  1 only (b)  2 and 3 only
members chosen in the first Rajya Sabha.
(c)  1 and 2 only (d)  None of the above
But, he cannot do it anytime based on any
Solution: (d)
recommendations. The term of MPs is fixed.
Justification: During National Emergency,
In the first batch, it was decided by lottery
the Duration of Lok Sabha can be extended
as to who should retire. Further, the act also
to a maximum of 1 year at a time by passing
authorised the President to make provisions
a law in the parliament.
to govern the order of retirement of the
However, the same is not applicable for
members of the Rajya Sabha.
financial emergency, so 1 is incorrect.
It can be extended by 6 months at a time. Q.723 The term of the Rajya Sabha
For e.g. the term of 5th Lok Sabha was (a) Can be extended by an extraordinary
Presidential order

Indian Polity Question Bank P.191

02-Indian Polity_Q413-927.indd 191 8/7/2018 7:40:37 PM


(b) Can be extended by 6 months at a time • To uphold the sovereignty and integrity of
by ordinary legislation India; and
(c) Can be extended by 1 year at a time • To faithfully discharge the duty upon
during the proclamation of emergency which he is about to enter.
(d)  None of the above Q.726 In the oath or affirmation by every Member
Solution: (d) of Parliament (MP), s/he swears to
Justification: Rajya Sabha is a permanent 1. uphold the sovereignty and integrity
continuing chamber and not subject to of India
dissolution. Hence, there is no term of RS, 2. do right to all manner of people in
it is the term of the MPs of the house. So, accordance with the Constitution and the
(d)  can be the only answer. law, without fear or favour, affection or
Q.724 With reference to the Rajya Sabha, consider ill will.
the following statements 3. preserve, protect and defend the
1. It is not subject to dissolution Constitution and the law.
2. One third of its members retire every Choose the correct answer using the codes
2nd year below.
3. All the elected members of the Rajya (a)  1 and 2 only (b)  2 and 3 only
Sabha are indirectly elected (c)  1 only (d)  All of the above
Which of the above statements is/are Solution: (c)
INCORRECT? Every member of either House of Parliament,
(a)  1 only (b)  2 only before taking his seat in the House, has to
(c) 3 only (d) None make and subscribe to an oath or affirmation
Solution: (d) before the President or some person
The Rajya Sabha meets in continuous appointed by him for this purpose. In his
sessions, and unlike the Lok Sabha, the oath or affirmation, a member of Parliament
lower house of Parliament, is not subject swears
to dissolution. The Rajya Sabha has equal • to bear true faith and allegiance to the
footing in all areas oflegislation with Lok Constitution of India;
Sabha, except in the area of supply, where the • to uphold the sovereignty and integrity of
Lok Sabha has overriding powers. India; and
Members sit for 6-year terms, with one • to faithfully discharge the duty upon
third of the members retiring every 2 years. which he is about to enter.
Option 2 is part of oath taken by a
Membership of Parliament Minister.
Option 3 is that taken by the President
Q.725 Every member of either House of Parliament, of India.
before taking his seat in the House, has to
Q.727 The constitution of India has provided
make and subscribe to an oath or affirmation
for certain qualifications in order to
before the
become members of the Parliament and
(a)  President of India
State legislatures. Which of the following
(b)  Presiding officer of the House
statements is/are correct?
(c)  Chief Justice of India
1. Minimum and maximum age requirements
(d) Leader of the concerned party or coalition
are provided in the constitution for a
Solution: (a)
candidate contesting for elections.
Learning: The oath is taken either before the
2. A person who has undergone imprisonment
President or some person appointed by him
for two or more years for some offence is
for this purpose.
disqualified from contesting elections.
In his oath or affirmation, a Member of
3. A candidate standing for election
Parliament swears:
should have at least completed his
• To bear true faith and allegiance to the
matriculation.
Constitution of India;

P.192 For Civil Services Preliminary Examination

02-Indian Polity_Q413-927.indd 192 8/7/2018 7:40:37 PM


4. There is a reasonable restriction on • The candidate should not be a direct
the income of the candidates standing beneficiary of the Government and should
for elections. not hold on office of profit under the
Select the correct answer using the codes Government.
below. Q.729 A Member of Parliament ceases to hold his
(a)  2 only (b)  1 and 2 only seat if
(c)  1, 2 and 3 only (d)  1, 2 and 4 only 1. S/he owns and runs a private commercial
Solution: (a) firm
What is true of the right to vote is also true 2. S/he holds any office of Profit under the
of right to contest election. All citizens have Union or State government
the right to stand for election and become the 3. S/he is removed from the political party
representative of the people. However, there on whose ticket he was elected
are different minimum age requirements for 4. S/he is expelled from the house.
contesting elections. For example, in order to Choose the correct answer using the codes
stand for Lok Sabha or Assembly election, a below.
candidate must be at least 25 years old. There (a)  2, 3 and 4 only (b)  1 and 3 only
are some other restrictions also. For instance, (c)  2 and 4 only (d)  All of the above
there is a legal provision that a person who Solution: (c)
has Cundergone imprisonment for two or Under the Constitution, a person shall be
more years for some offence is disqualified disqualified for being elected as a Member of
from contesting elections. But there are no Parliament.
restrictions of income, education or class or • if he holds any office of profit under the
gender on the right to contest elections. In Union or state government (except that of
this sense, our system of election is open to a minister or any other office exempted by
all citizens. Parliament).
Q.728 Consider the following statements. • if he is of unsound mind and stands so
1. A non-citizen of India cannot become a declared by a court.
Member of Parliament. • if he is an undischarged insolvent.
2. A person who does not have his/her name • if he voluntary (not expelled) gives up
on the electoral rolls cannot contest for the membership of the political party on
election to the Parliament. whose ticket he is elected to the House
Which of the above is/are correct? • if he is not a citizen of India or has
(a)  1 only (b)  2 only voluntarily acquired the citizenship
(c)  Both 1 and 2 (d)  None of a foreign state or is under any
Solution: (c) acknowledgement of allegiance to a
Justification: Statements 1 and 2: foreign state; and
Qualifications for Membership of Lok Sabha • if he is so disqualified under any law made
are the following: by Parliament.
• S/he should be a citizen of India; So, 1 is • He must not be a director or managing
correct. agent nor hold an office of profit in a
• S/he should not be less than 25 years of age; corporation in which the government has
• S/he should not be a proclaimed criminal at least 25% share.
(i.e. he/she should not be a convict, a • He must not have been dismissed from
confirmed debtor or otherwise disqualified government service for corruption or
by law); disloyalty to the State.
• S/he should have his/her name in the Q.730 Consider the following statements.
electoral rolls in any port of the country 1. Insulting the Indian National Flag entails
(he/she may not belong to the State from disqualification for contesting elections to
which he/she contests the election); so, Parliament and State Legislatures.
the option 2 is correct as well.

Indian Polity Question Bank P.193

02-Indian Polity_Q413-927.indd 193 8/7/2018 7:40:37 PM


2. A candidate cannot stand for election house without taking any leave or permission
from more than one Parliamentary of the speaker.
constituency. Statement 2: He will also be disqualified
3. All cases of disqualification of MPs are if he votes or abstains from voting in such
dealt by the speaker. House contrary to any direction issued by
Choose the correct answer using the codes his political party without obtaining prior
below. permission of such party and such act has not
(a)  1 and 2 only (b)  1 and 3 only been condoned by the party within 15 days
(c)  2 and 3 only (d)  1 only Statement 3: No-confidence motion can
Solution: (d) be passed for the entire council.
The disqualification of the MPs and MLAs Q.733 On the question whether a member is subject
are dealt by either the RPI Act or the anti- to any of the disqualifications mentioned
defection act. In case of the anti-defection under the Representation of People Act
act, it is the speaker. In case of the RPI Act, is (1951), whose decision is final?
the President who is advised by the Election (a) President
Commission. (b)  Supreme Court
A MP or MLA can stand from maximum (c)  Election Commission of India
two electoral constituencies. Narendra Modi (d)  Lok Sabha Speaker
contested from both Varansi and Vadodra. Solution: (a)
Q.731 If a member of Parliament voluntarily On the question whether a member is
acquires the citizenship of a foreign· country. subject to any of the disqualifications under
(a)  s/he will be penalized RPA 1951, the president’s decision is final.
(b) s/he will continue to be a member of However, he should obtain the opinion of the
Parliament election commission and act accordingly.
(c) s/he will be disqualified from membership The question of disqualification under the
(d) s/he will have a choice of renouncing Tenth Schedule is decided by the Chairman
either in the case of Rajya Sabha and Speaker in the
Solution: (c) case of Lok Sabha (and not by the president
The right to contest election is available only of India). In 1992, the Supreme Court ruled
to Indian citizens and Person after acquiring that the decision of the Chairman/Speaker in
the forgeign citizenship seizes to be the this regard is subject to judicial review
citizen of india. Q.734 Which of the following is true?
Q.732 A member of a House belonging to any (a) A person can be a member of both Houses
political party becomes disqualified for being of Parliament
a member of the House if (b) There is no bar to a person contesting to
1. He is absent from the first meeting of the as many seats and as many legislatures
any new session of the house. as he likes
2. He has voluntarily given up the (c) A member of Lok Sabha can hold an
membership of his political party. office of profit under the Government
3. A no-confidence motion is passed against (d) The population of a State is not the sole
the individual member. consideration for allotment of seats in the
Select the correct answer using the codes Rajya Sabha
below. Solution: (d)
(a)  1 and 2 only (b)  2 and 3 only Option (a): person cannot be a member of
(c)  2 only (d)  3 only both Houses of Parliament at the same time.
Solution: (c) Thus, the Representation of People Act, 1951
Justification and Learning: Statement 1: (R.P. Act) provides for the following:
There is no such provision, but there is a (a) If a person is elected to both the Houses
provision for disqualification if a member is of Parliament, he must intimate within 10
absent for more than 60 working days of the days in which House he desires to serve.

P.194 For Civil Services Preliminary Examination

02-Indian Polity_Q413-927.indd 194 8/7/2018 7:40:37 PM


In default of such intimation, his seat in Before 1921, the Governor-General of
the Rajya Sabha becomes vacant. India used to preside over the meetings of the
(b) If a sitting member of one House is also Central Legislative Council.
elected to the other House, his seat in the Statement 2: The GoI Act 1935 changed
first House becomes vacant. the nomenclatures of President and Deputy
(c) If a person is elected to two seats in a President of the Central Legislative
House, he should exercise his option Assembly to the Speaker and Deputy Speaker
for one. Otherwise, both seats become respectively.
vacant. However, the old nomenclature continued
Similarly, a person cannot be a member of till 1947 as the federal part of the 1935 Act
both the Parliament and the state legislature was not implemented.
at the same time. If a person is so elected, Q.736 The institutions of Speaker and Deputy
his seat in Parliament becomes vacant if he Speaker originated in India in 1921 under
does not resign his seat in the state legislature (a) A Charter issued by the British crown for
within consecutive 14 days. improving legislative business in British
Option (b): As per Section 33 (7) of R. P. India
Act, 1951, a person cannot contest from more (b) Provisions of the Government of India
than two constituencies for a Lok Sabha/ Act of 1919 (Montague–Chelmsford
Vidhan Sabha election. Reforms)
Option (c): According to Articles 102(1) (c) An order issued by the then Governor-
(a) and 191(1)(a)  of Constitution, legislators General of India to manage the Central
(MP or MLA) can be barred from holding Legislative Assembly better
office of profit under Central Gover or state (d) Amendments made to the Charter Act of
government as it can put them in position to 1891 that created the office of President
gain financial benefit and Vice-President of Legislative
assemblies
Presiding Officers of Parliament Solution: (b)
Learning: At that time, the Speaker and the
Q.735 Consider the following statements.
Deputy Speaker were called the President
1. Before the enactment of the Government
and Deputy President respectively and the
of India Act 1919, the Governor- General
same nomenclature continued till 1947.
of India presided over the meetings of the
Before 1921, the Governor- General of
Central Legislative Council.
India used to preside over the meetings of
2. The Government of India Act of 1935
the Central Legislative Council. In 1921,
instituted the office of the President
the Frederick Whyte and Sachidanand Sinha
and Deputy President of the Central
were appointed by the Governor-General
Legislative Assembly.
of India as the first Speaker and the first
Which of the above is/are correct?
Deputy Speaker (respectively) of the central
(a)  1 only (b)  2 only
legislative assembly.
(c)  Both 1 and 2 (d)  None
In 1925, Vithalbhai J. Patel became the
Solution: (a)
first Indian and the first elected Speaker of
Justification: Statement 1: The institutions
the central legislative assembly.
of Speaker and Deputy Speaker originated
The Government of India Act o 1935
in India in 1921 under the provisions of the
changed the nomenclatures of President and
Government of India Act of 1919 (Montague–
Deputy President of the Central Legislative
Chelmsford Reforms).
Assembly to the Speaker and Deputy Speaker
At that time, the Speaker and the Deputy
respectively.
Speaker were called the President and
Deputy President respectively and the same
nomenclature continued till 1947.

Indian Polity Question Bank P.195

02-Indian Polity_Q413-927.indd 195 8/7/2018 7:40:37 PM


Q.737 The Presiding officers of the Parliament are (c) Vice-President
chosen by a combination of: (d)  Leader of the House
(a) Indirect elections based on First Past the Solution: (a)
Post (FPTP) system only. Learning: In the Lok Sabha, the lower
(b) Indirect elections based on Proportional House of the Indian Parliament, both
Representation only. Presiding Officers—the Speaker and the
(c) Indirect elections based on First past the Deputy Speaker- are elected from among its
Post (FPTP) system and nomination. members by a simple majority of members
(d) Indirect elections based on both First Past present and voting in the House.
the Post (FPTP) system and Proportional The Speaker holds office from the date of
Representation and nomination her election till immediately before the first
Solution: (d) meeting of the Lok Sabha after the dissolution
The Presiding officers of the Parliament of the one to which she was elected. She is
include: Speaker and chairman of both the eligible for re-election. On the dissolution of
houses; deputy speaker and chairman of both the Lok Sabha, although the Speaker ceases
the houses; and the panel of chairpersons and to be a member of the House, she does not
vice-chairpersons of both Lok Sabha and vacate her office. The Speaker may, at any
Rajya Sabha. time, resign from office by writing under her
The speaker is indirectly elected based hand to the Deputy Speaker.
on FPTP. Q.740 The Speaker of Lok Sabha can be removed
The vice-president is indirectly elected on
based on PR system. 1. A resolution passed by majority of the
The chairpersons are nominated. membership of Lok Sabha
Hence, it is a combination of all. 2. A resolution passed by special majority in
Lok Sabha
(1) Speaker 3. An enquiry by the Supreme Court
4. An order of the President
Q.738 How is the Lok Sabha speaker chosen?
Choose the correct answer using the codes
(a) Nominated by a collegium of Prime
below.
Minister, Leader of Opposition and
(a)  1 only (b)  2 and 3 only
Minister for Parliamentary affairs
(c)  1 and 3 only (d)  1, 3 and 4 only
(b) Nominated by the party/coalition in
Solution: (a)
majority in Lok Sabha
Speaker has to vacate his office earlier in any
(c) Elected by the house from a body of
of the following three cases:
eminent citizens specially selected for
• if s/he ceases to be a member of the Lok
this purpose
Sabha;
(d) Elected by the house from amongst its
• if s/he resigns by writing to the Deputy
members
Speaker; and
Solution: (d)
• if s/he is removed by a resolution passed
Learning: The Speaker is elected by the Lok
by a majority of all the members of the Lok
Sabha from amongst its members (as soon as
Sabha. Such a resolution can be moved
may be, after its first sitting). Whenever the
only after giving 14 days advance notice.
office of the Speaker falls vacant, the Lok Sabha
When a resolution for the removal of the
elects another member to fill the vacancy. The
Speaker is under consideration of the House,
date of election of the Speaker is fixed by the
he cannot preside at the sitting of the House,
President. Usually, the Speaker remains in
though he may be present. However, he can
office during the life of the Lok Sabha.
speak and take part in the proceedings of the
Q.739 The Speaker of Lok Sabha resigns from House at such a time and vote in the first
office by writing, addressed to the instance, though not in the case of an equality
(a)  Deputy Speaker, Lok Sabha of votes.
(b) President

P.196 For Civil Services Preliminary Examination

02-Indian Polity_Q413-927.indd 196 8/7/2018 7:40:37 PM


It should be noted here that, whenever the (c) The President on an independent inquiry
Lok Sabha is dissolved, the Speaker does not conducted by a Committee of Lok Sabha
vacate his office and continues till the newly- (d)  None of the above
elected Lok Sabha meets. Solution: (d)
Q.741 Which of the following provisions ensure the Refer previous explanations.
independence and impartiality of the office of Q.743 To impeach the Lok Sabha speaker
the Lok Sabha Speaker? (a) Both houses need to pass a resolution to
1. He can be removed only by a resolution that effect by an absolute majority
passed by the Lok Sabha by a special (b) Both houses need to pass a resolution to
majority. that effect by an ordinary majority
2. His work and conduct cannot be discussed (c) Only Lok Sabha needs to pass a resolution
and criticised in the Lok Sabha except on to that effect by an absolute majority
a substantive motion. (d) Only Lok Sabha needs to pass a resolution
3. His salary and allowances are not subject to that effect by an ordinary majority
to the annual vote of Parliament. Solution: (c)
4. He cannot vote in the first instance. Refer previous explanations.
Choose the correct answer using the codes Q.744 Consider the following about the Office of
below. the Speaker.
(a)  1 and 2 only 1. She holds office during the pleasure of the
(b)  2, 3 and 4 only President.
(c)  None of the above 2. She need not be a member of the House at
(d)  All of the above the time of her election.
Solution: (b) 3. She is elected jointly by both the Lok
He is provided with a security of tenure. He Sabha and Rajya Sabha.
can be removed only by a resolution passed Select the correct answer using the codes
by the Lok Sabha by an absolute majority below.
(ie, a majority of the total members of the (a)  1 only (b)  2 and 3 only
House)  and not by an ordinary majority (ie, (c)  1 and 3 only (d)  None of the above
a majority of the members present and voting Solution: (d)
in the House). This motion of removal can Justification: Statement 1: She holds office
be considered and discussed only when it has as long as she enjoys the confidence of the
the support of at least 50 members. house. She can be removed only by the Lok
Moreover, his powers of regulating Sabha through impeachment.
procedure or conducting business or Statement 2: She must be a member. She
maintaining order in the House are not is elected from amongst them by a majority
subject to the jurisdiction of any Court. of votes.
He cannot vote in the first instance. He Statement 3: Rajya Sabha plays no role in
can only exercise a casting vote in the event her election. But, both LS and RS play a role
of a tie. This makes the position of Speaker in choosing the Presiding officer of RS, i.e.
impartial. the Vice-President.
Q.742 As the office of the Speaker is vested with Q.745 Consider the following statements.
great prestige, position and authority, 1. The Speaker is elected by the Lok Sabha
independence and impartiality becomes its from amongst its members.
sine qua non. Thus, she is provided with 2. The date of election of the Speaker is
a security of tenure. She can be removed fixed by the Leader of the House.
only by 3. Whenever the Lok Sabha is dissolved,
(a) An independent inquiry conducted by the the Speaker does not vacate his office and
Supreme Court continues till the newly elected Lok Sabha
(b) A resolution passed by the Lok Sabha by meets.
a special majority

Indian Polity Question Bank P.197

02-Indian Polity_Q413-927.indd 197 8/7/2018 7:40:38 PM


Select the correct answer using the codes Two members of Lok Sabha will be
below. chosen as Speaker and Deputy Speaker. And
(a)  1 and 2 only (b)  3 only therefore, the Speaker shall vacate the office
(c)  1 and 3 only (d)  1, 2 and 3 if he/ she cease to be a member of Lok Sabha.
Solution: (c) The speaker can only be removed by the
Justification: Statements 1 and 2: The Lok Sabha members by majority (not by the
Speaker is elected by the Lok Sabha from pleasure of the President).
amongst its members (as soon as may be, After the dissolution of Lok Sabha the
after its first sitting). Whenever the office of Speaker will not vacate the office immediately
the Speaker falls vacant, the Lok Sabha elects but will continue till the first meeting after
another member to fill the vacancy. the next general elections.
The date of election of the Speaker is fixed
by the President. So, the option 2 is wrong. Powers and Functions
Learning: When a resolution for the removal
of the Speaker is under consideration of the Q.748 The Speaker of the Lok Sabha derives his
House, he cannot preside at the sitting of the powers and duties from
House, though he may be present. 1. The Constitution of India
However, he can speak and take part in 2. Parliamentary Conventions
the proceedings of the House at such a time 3. Parliamentary laws
and vote in the first instance, though not in 4. Rules of Procedure of Lok Sabha
the case of an equality of votes. Choose the correct answer using the codes
It should be noted here that, whenever the below.
Lok Sabha is dissolved, the Speaker does not (a)  1, 2 and 4 only (b)  2 and 3 only
vacate his office and continues till the newly (c)  1 and 4 only (d)  All of the above
elected Lok Sabha meets. Solution: (a)
The Speaker is the head of the Lok Sabha,
Q.746 The provision that Speaker does not vacate and its representative. He is the guardian of
her office until immediately before the first powers and privileges of the members, the
meeting of the House after dissolution is House as a whole and its committees. He is
found in the the principal spokesman of the House, and
(a)  Parliamentary rules and procedure his decision in all Parliamentary matters is
(b)  Constitution of India final. He is thus much more than merely the
(c) Representation of People of India Act, presiding officer of the Lok Sabha. In these
1950 capacities, he is vested with vast, varied and
(d) Conventions of the Indian Parliament vital responsibilities and enjoys great honour,
Solution: (b) high dignity and supreme authority within
Q.747 During the period of incumbency of the the House.
Lok Sabha speaker, which of the following The Speaker of the Lok Sabha derives
conditions must remain fulfilled, failing which his powers and duties from three sources,
the Speaker has to resign from his/her post? that is, the Constitution of India, the Rules
1. He/she must enjoy the pleasure of the of Procedure and Conduct of Business of
President. Lok Sabha, and Parliamentary Conventions
2. He/she must be a member of Lok Sabha. (residuary powers that are unwritten or
3. Lok Sabha should not be dissolved. unspecified in the Rules).
Choose the correct answer using the codes Q.749 When can the Speaker exercise his right to
below: vote in the House?
(a)  1 and 2 (b)  2 and 3 (a)  Whenever he desires
(c)  1 and 3 (d)  only 2 (b)  Whenever the House desires
Solution: (d) (c) Only in the event of equality of votes
The following are the provisions related to (d)  Whenever his party directs
the office of the speaker:

P.198 For Civil Services Preliminary Examination

02-Indian Polity_Q413-927.indd 198 8/7/2018 7:40:38 PM


Solution: (c) He is the final interpreter of the provisions of
Speaker usually casts her vote in case of (a)  the Constitution of India,
equality of votes only. He doesn’t vote at first (b) the Rules of Procedure and Conduct of
instance. Business of Lok Sabha,
Q.750 The Speaker of the Lok Sabha derives his (c) the parliamentary precedents, within the
powers and duties from House.
(a)  Constitution of India He appoints the chairman of all the
(b) Rules of Procedure and Conduct of parliamentary committees of the Lok Sabha
Business of Lok Sabha and supervises their functioning. He himself
(c) Parliamentary Conventions which are is the chairman of the Business Advisory
unwritten rules Committee, the Rules Committee and the
(d)  All of the above General Purpose Committee.
Solution: (d) Q.752 The decision of the Lok Sabha speaker is
Learning: Option (a): For example, his power considered in which of the following cases?
to decide whether a bill is money bill or not. 1. Deciding whether a bill is money bill
Option (b): His powers to regulate the or not
business or the house and set the house in 2. Disqualifying MPs under anti-defection
order etc. law
Option (c): Rules like Mavalankar rule 3. Deciding on calling a joint meeting of
where the Speaker recognizes the leader of both the Houses
the largest opposition party as the Leader of Choose the correct answer using the codes
opposition. below.
Q.751 Consider the following about the role and (a)  1 and 2 only (b)  2 and 3 only
powers of the Speaker, Lok Sabha. (c)  1 and 3 only (d)  All of the above
1. He is the head and representative of the Solution: (a)
Lok Sabha. The Lok Sabha Speaker presides over a joint
2. His decision in all Parliamentary matters setting of the two Houses of Parliament.
is final. Such a sitting is summoned by the President
3. He is the final interpreter of the provisions to settle a deadlock between the two Houses
of the Constitution of India within on a bill.
the house. He can allow a ‘secret’ sitting of the
4. He appoints the chairman of all the House at the request of the Leader of the
parliamentary committees of the Lok House. When the House sits in secret, no
Sabha and supervises their functioning. stranger can be present in the chamber, lobby
Choose the correct answer using the codes or galleries except with the permission of
below. the Speaker.
(a)  2 only (b)  2, 3 and 4 only He decides whether a bill is a money bill
(c)  1 and 3 only (d)  All of the above or not and his decision on this question is
Solution: (d) final. When a money bill is transmitted to
He is the guardian of powers and privileges the Rajya Sabha for recommendation and
of the members, the House as a whole presented to the President for assent, the
and its committees. He is the principal Speaker endorses on the bill his certificate
spokesman of the House, and his decision that it is a money bill.
in all Parliamentary matters is final. He is He decides the questions of disqualification
thus much more than merely the presiding of a member of the Lok Sabha, arising on the
officer of the Lok Sabha. In these capacities, ground of defection under the provisions of
he is vested with vast, varied and vital the Tenth Schedule. In 1992, theSupreme
responsibilities and enjoys great honour, Court ruled that the decision of the Speaker
high dignity and supreme authority within in this regard is subject to judicial review.
the House.

Indian Polity Question Bank P.199

02-Indian Polity_Q413-927.indd 199 8/7/2018 7:40:38 PM


Q.753 Consider the following statements about the This motion of removal can be considered
Lok Sabha speaker. and discussed only when it has the support of
1. The speaker has to be elected from at least 50 members.
amongst the Lok Sabha members. Statement 2: Speaker regulates the
2. When Lok Sabha is dissolved, the speaker proceedings of the house. Her powers of
does not immediately cease to hold office. regulating procedure or conducting business
3. His decision in all Parliamentary matters or maintaining order in the House are not
is final. subject to the jurisdiction of any Court.
Choose the correct answer using the codes Q.756 Consider the following statements.
below. 1. He acts as the ex officio chairman of the
(a)  1 and 2 only (b)  2 and 3 only Indian Parliamentary Group of the Inter-
(c)  1 only (d)  All of the above Parliamentary Union.
Solution: (d) 2. He can allow a ‘secret’ sitting of the
Refer previous explanations House at the request of the Leader of the
Q.754 In the Lok Sabha, who decides the sitting House.
order of the members? The above refers to whom?
(a) Member can sit at a place of their (a) President of India as he is an integral part
preference barring the first rows. of Parliament
(b)  Secretary General, Lok Sabha (b) Vice-President of India since he chairs
(c)  Leader of the House the Rajya Sabha and has a high rank in
(d)  Speaker, Lok Sabha table of precedence
Solution: (d) (c) Prime Minister of India, since he is the
Learning: These rules pertain to Lok Sabha head of the Government
as “SUMMONS TO MEMBERS, SEATING, (d) Speaker, Lok Sabha since she is the
OATH OR AFFIRMATION AND ROLL OF presiding officer of the House
MEMBERS:” Solution: (d)
The Secretary-General shall issue summons Learning: Statement 1: He also acts as the ex-
to each member specifying the date and place officio chairman of the conference of presiding
for a session of the House. officers of legislative bodies in the country.
The members shall sit in such order as the • He appoints the chairman of all the
Speaker may determine. parliamentary committees of the Lok
Q.755 Consider the following statements about the Sabha and supervises their functioning.
Lok Sabha. He himself is the chairman of the
1. The work and conduct of the Speaker Business Advisory Committee, the Rules
cannot be discussed and criticised in Committee and the General Purpose
the Lok Sabha except on a substantive Committee.
motion. Statement 2: When the House sits in secret,
2. No discussion on a matter of general no stranger can be present in the chamber,
public importance can take place except lobby or galleries except with the permission
on a motion made with the consent of the of the Speaker.
Speaker. Q.757 Who is authorized to certify whether a bill is
Which of the above is/are correct? an ordinary bill or a money bill?
(a)  1 only (b)  2 only (a)  President of India
(c)  Both 1 and 2 (d)  None (b)  Speaker, Lok Sabha
Solution: (c) (c)  Chairman, Rajya Sabha
Justification: Statement 1: This is done with (d)  Leader of the House
a view of safeguard the independence and Solution: (b)
Impartiality of the office of the Speaker as Justification: If any question arises whether
it is vested with great prestige, position and a bill is a money bill or not, the decision of
authority. the Speaker of the Lok Sabha is final.

P.200 For Civil Services Preliminary Examination

02-Indian Polity_Q413-927.indd 200 8/7/2018 7:40:38 PM


His decision in this regard cannot be adjourn it for some time. So, the option 1
questioned in any court of law or in the either is wrong.
House of Parliament or even the president. She only notifies whether a bill is money
When a money bill is transmitted to bill or not.2 is wrong.
the Rajya Sabha for recommendation and The Prime Minister heads the appointments
presented to the president for assent, the committee of the Cabinet. There is no
Speaker endorses it as a money bill. appointments committee in the Lok Sabha.
Q.758 On days allotted for the transaction of So, 3 is wrong
Government business, such business shall Q.760 The Speaker of the Lok Sabha derives his
have precedence and the Secretary-General, powers and duties from the Constitution of
shall arrange that business in such order as India, the Rules of Procedure and Conduct
X may, after consultation with Y, determine. of Business of Lok Sabha, and Parliamentary
Who are X and Y in the aforementioned Conventions. Accordingly, she exercises
“Rules of Procedure and Conduct of Business which of the following powers?
in Lok Sabha”? 1. She is the final interpreter of the provisions
(a) President of India and Speaker of Lok of the Constitution of India for any matter
Sabha relating to the Parliament.
(b) Speaker of Lok Sabha and Leader of the 2. She has the final power in matters of
House decorum, order and business in the house.
(c) President and Secretary-General Lok 3. She decides whether a bill is a money bill
Sabha or not.
(d) Speaker and Chairman, Business 4. She appoints the chairman of all the
Advisory Committee of Lok Sabha parliamentary committees of the Lok
Solution: (b) Sabha and supervises their functioning.
Justification: Option A: The role of President Select the correct answer using the codes
is mainly to (a)  Address Motion of Thanks, below.
(b)  Prorogue a running session or commence (a)  2 and 3 only (b)  2, 3 and 4 only
a new session of Lok Sabha, and (c)  Pass or (c)  1, 2 and 3 only (d)  1 and 4 only
withhold bills passed by the house, and call Solution: (b)
a joint sitting in case of a deadlock between Justification: Statement 1: Two things
LS and RS. He is not concerned with the should be noted here:
business in house. (a) She is the final interpreter of the entire
Option D: The speaker herself is the constitution of India within the house
Chairman, Business Advisory Committee of relating to its proceedings.
Lok Sabha. (b) She is the final interpreter only within the
Q.759 Consider the following about the powers of house. Outside the house, it is the courts.
the Speaker, Lok Sabha. So, the option 1 is wrong.
1. She prorogues an ongoing session of the Statement 3: She decides whether a bill is
Lok Sabha. a money bill or not and his decision on this
2. She approves all the bills before it is sent question is final.
for Presidential assent. When a money bill is transmitted to
3. She heads the appointments committee of the Rajya Sabha for recommendation and
the Cabinet and in Lok Sabha. presented to the President for assent, the
Select the correct answer using the codes Speaker endorses on the bill his certificate
below. that it is a money bill.
(a)  2 only (b)  1 and 3 only Statement 4: She herself is the chairman
(c)  3 only (d)  None of the above of the Business Advisory Committee, the
Solution: (d) Rules Committee and the General Purpose
Justification: President prorogues the Committee
session, i.e. ends it. Speaker can only

Indian Polity Question Bank P.201

02-Indian Polity_Q413-927.indd 201 8/7/2018 7:40:38 PM


(2) Speaker pro tem 1. Usually, the seniormost member is
selected for this.
Q.761 Consider the following about the Speaker 2. The President himself administers oath to
pro tem. the Speaker pro tem.
1. She is elected by the Lok Sabha from 3. The Speaker pro tem has all the powers
amongst the members of the house. of the Speaker. He presides over the first
2. She is responsible for chairing the sitting of the newly-elected Lok Sabha.
meetings of Lok Sabha until the new 4. His main duty is to administer oath to the
elected Speaker becomes acquainted with new members.
the Parliamentary procedures. 5. He also enables the House to elect the
3. The Speaker pro tem has all the powers of new Speaker.
the Speaker. 6. When the new Speaker is elected by the
Select the correct answer using the codes House, the office of the Speaker pro tem
below. ceases to exist. Hence, this office is a
(a)  3 only (b)  1 and 2 only temporary office, existing for a few days
(c)  1 only (d)  None of the above
Q.763 Consider the following about Speaker
Solution: (a)
pro tem.
Justification: Statement 1: As provided by
1. The President can appoint anyone as the
the Constitution, the Speaker of the last Lok
Speaker pro tem of Lok Sabha.
2. T h e
Sabha vacates his office immediately before
Speaker pro tem has all the powers of the
the first meeting of the newly elected Lok
Speaker.
Sabha. Therefore, the President appoints
3. Office of the Speaker pro tem is temporary
a member of the Lok Sabha as the Speaker
in nature.
pro tem. Usually, the seniormost member is
Choose the correct answer using the codes
selected for this.
below.
Statement 2 and 3: The Speaker pro
(a)  1 and 2 only (b)  2 and 3 only
tem has all the powers of the Speaker. She
(c)  1 and 3 only (d)  All of the above
presides over the first sitting of the newly
Solution: (b)
elected Lok Sabha.
• Her main duty is to administer oath to the
new members. He also enables the House (3) Chairman of Rajya Sabha
to elect the new Speaker. Q.764 Consider the following statements.
• When the new Speaker is elected by the 1. The Chairman of the Rajya Sabha can
House, the office of the Speaker pro tem be removed from his office only if he
ceases to exist. Hence, this office is a is removed from the office of the Vice-
temporary office, existing for a few days. President.
Q.762 The President appoints a member of the 2. Unlike the Speaker, the Chairman of Rajya
Lok Sabha as the Speaker pro tem when Sabha is not a member of the House.
the Speaker of the last Lok Sabha vacates Which of the above is/are correct?
her office. What are the duties of Speaker (a)  1 only (b)  2 only
pro tem. (c)  Both 1 and 2 (d)  None
1. Administering oath of newly elected Solution: (c)
members Justification: Statements 1 and 2: The vice-
2. Enabling the house to select new Speaker president of India is the ex-officio Chairman
Which of the above is/are correct? of the Rajya Sabha.
(a)  1 only (b)  2 only If he is to be removed from the Chairman
(c)  Both 1 and 2 (d)  None of the house, he must be impeached as the
Solution: (c) Vice-President.
Learning: the President appoints a member This is unlike the institution of the speaker,
of the Lok Sabha as the Speaker pro tem. where the Speaker is a member of the house
and can be removed by the Lok Sabha.

P.202 For Civil Services Preliminary Examination

02-Indian Polity_Q413-927.indd 202 8/7/2018 7:40:38 PM


(4) Deputy–Chairman of Rajya Sabha There is also a ‘Leader of the House’
in the Rajya Sabha. He is a minister and a
Q.765 The Deputy Chairman of the Rajya Sabha member of the Rajya Sabha and is nominated
can be removed by a resolution: by the prime minister to function as such.
(a) passed by 2/3rd majority of its total The leader of the house in either House is
members present an important functionary and exercises direct
(b) passed by a majority of all members of influence on the conduct of business. He can
Rajya Sabha also nominate a deputy leader of the House.
(c) moved by Rajya Sabha but passed by
Q.767 Who among the following is generally the
Lok Sabha
leader of the Lok Sabha?
(d)  None of the above
(a)  Speaker, Lok Sabha
Solution: (b)
(b)  Leader of Opposition
Constitution says Deputy Chairman vacates
(c)  Prime Minister
his office in any of the following three cases:
(d)  Secretary General, Lok Sabha
1. if he ceases to be a member of the Rajya
Solution: (c)
Sabha;
Refer previous explanations.
2. if he resigns by writing to the Chairman;
and
3. if he is removed by a resolution passed by (2) Leader of Opposition
a majority of all the members of the Rajya Q.768 The ‘Mavalankar rule’ in Parliament is
Sabha. Such a resolution can be moved related to
only after giving 14 days’ advance notice. (a) Rule of Order in a House of Parliament
The Deputy Chairman performs the duties (b) Role of private members in the business
of the Chairman’s office when it is vacant of house
or when the Vice-President acts as President (c) Conduct of the Parliamentary Committees
or discharges the functions of the President. (d)  None of (a), (b) or (c) is correct.
He also acts as the Chairman when the latter Solution: (d)
is absent from the sitting of the House. In Learning: G. V. Mavalankar, the first Lok
both the cases, he has all the powers of the Sabha Speaker, had said the main opposition
Chairman. It should be emphasised here that party’s strength must equal the quorum —
the Deputy Chairman is not subordinate to which is 10 per cent of the total strength —
the Chair-man. He is directly responsible to required for functioning of the House. As per
the Rajya Sabha. the Mavalankar rule, any party needs to have
at least 10% of the total seats for its leader to
Leaders in Parliament claim the status of the leader of the opposition.
There was a controversy related to this rule in
the 16th Lok Sabha as the Main opposition
(1) Leader of Lok Sabha Congress failed to get more than 55 seats.
Q.766 The Leader of Lok Sabha is
(a)  Appointed by the Presiding Officer (3) Whip
(b)  Elected by the House
(c) Nominated by the President of India Q.769 Consider the following about the office of
(d)  None of the above ‘whip’.
Solution: (d) 1. The office has been established by Rules
Learning: Under the Rules of Lok Sabha, of Business in each House of Parliament.
the ‘Leader of the House’ means the prime 2. Only the ruling party is allowed to have a
minister, if he is a member of the Lok Sabha, whip in each house of Parliament.
or a minister who is a member of the Lok Which of the above is/are correct?
Sabha and is nominated by the prime minister (a)  1 only (b)  2 only
to function as the Leader of the House. (c)  Both 1 and 2 (d)  None
Solution: (d)

Indian Polity Question Bank P.203

02-Indian Polity_Q413-927.indd 203 8/7/2018 7:40:38 PM


Justification: Statement 1: The office of In the parliamentary form of Government,
‘whip’, on the other hand, is mentioned a party has its own internal organisation
neither in the Constitution of India nor in the inside Parliament and is served by a number
Rules of the House nor in a Parliamentary of officials known as the Whips, chosen
Statute. It is based on the conventions of the from members of the party itself. The main
parliamentary government. function of the Whips is to keep members of
• Though the offices of the leader of the their party within sound of the Division Bell
House and the leader of the Opposition whenever any important business is under
are not mentioned in the Constitution of consideration in the House. During sessions,
India, they are mentioned in the Rules the Whips of different parties send to their
of the House and Parliamentary Statute supporters periodic notices, also sometimes
respectively. called ‘Whips’, apprising them when
Statement 2: Every political party, whether important Divisions are expected, telling
ruling or Opposition has its own whip in the them the hour when a vote will probably take
Parliament. He is appointed by the political place, and requesting them to be in attendance
party to serve as an assistant floor leader. at that time. Realizing the vital role played
Learning: He is charged with the by the Chief Whips and leaders of parties
responsibility of ensuring the attendance and groups in Parliament as important party
of his party members in large numbers and functionaries, an Act namely the Leaders
securing their support in favour of or against and Chief Whips of Recognized Parties and
a particular issue. Groups in Parliament (Facilities) Act (Act 5
• He regulates and monitors their behaviour of 1999) was passed during the Twelfth Lok
in the Parliament. The members are Sabha. The Act inter alia provides additional
supposed to follow the directives given by Secretarial assistance and telephone facilities
the whip. Otherwise, disciplinary action to the Leaders and Chief Whips.
can be taken. Government Chief Whip: The Chief Whip
Q.770 Consider the following statements about of the Government Party in the Lok Sabha
‘Whips’ in Indian polity is the Minister of Parliamentary Affairs. In
1. The main function of the Whips is to the Rajya Sabha, the Minister of State for
keep members of their party within Parliamentary Affairs holds this position.
sound of the Division Bell whenever any The Chief Whip is directly responsible to the
important business is under consideration Leader of the House. It is part of his duties
in the House. to advise the Government on parliamentary
2. During sessions, the Whips of different business and to maintain a close liaison with
parties send to their supporters periodic the Ministers in regard to parliamentary
notices, also sometimes called ‘Whips’, business affecting their Departments. The
apprising them when important Divisions Chief Whip is assisted by one or two Ministers
are expected, telling them the hour when of State and at times by Deputy Ministers also.
a vote will probably take place, and
requesting them to be in attendance at Sessions of Parliament
that time.
Q.771 If you visit the Parliament of India, you
3. The Chief Whip of the Government Party
would NOT find it working in ordinary
in the Lok Sabha and Rajya Sabha is the
circumstances in which of the following
Minister of Parliamentary Affairs. Which
months?
of the statements given above is/are
(a) February (b) June
correct?
(c) September (d) December
(a)  1 and 3 only (b)  2 and 3 only
Solution: (b)
(c)  1 and 2 only (d)  All
Justification: There are usually three
Solution: (c)
sessions in a year, viz,

P.204 For Civil Services Preliminary Examination

02-Indian Polity_Q413-927.indd 204 8/7/2018 7:40:38 PM


• The Budget Session (February to May); 1. Any Union Minister
• The Monsoon Session (July to September); 2. Attorney general of India
and 3. Cabinet Secretary
• The Winter Session (November to Select the correct answer using the codes
December). below.
• June comes under Summer vacation. (a)  1 and 2 only (b)  2 and 3 only
The president from time to time summons (c)  1 only (d)  1, 2 and 3
each House of Parliament to meet. But, Solution: (a)
the maximum gap between two sessions of Justification: Every minister and the attorney
Parliament cannot be more than six months. general of India have the right to speak and
The Parliament must meet at least twice take part in the proceedings of either House,
a year. any joint sitting of both the Houses and any
Q.772 Which of the following is NOT one of the committee of Parliament of which he is a
sessions of Parliament? member, without being entitled to vote.
(a)  Budget Session There are two reasons underlying this
(b)  Economy Session constitutional provision:
(c)  Winter Session • A minister can participate in the
(d)  Monsoon Session proceedings of a House, of which he is
Solution: (b) not a member. In other words, a minister
Learning: The period during which the belonging to the Lok Sabha can participate
parliamentarians convene and conduct the in the proceedings of the Rajya Sabha and
business of each House is referred to as vice-versa.
session. Every year, the Parliament of India • A minister, who is not a member of either
conducts three sessions – Budget, Monsoon House, can participate in the proceedings
and Winter. of both the Houses.
Budget session, which is conducted Q.774 A session of Rajya Sabha can be terminated
from February to May, is considered by which of the following?
highly crucial for the matters it deals with. (a)  Only adjournment Sine dine
Monsoon session: After a brief hiatus of (b) Only adjournment or only prorogation
around two months, both the Houses of (c) Either by adjournment or by prorogation
the Parliament resume business in July or by dissolution of the house
and continue till September. Unlike the (d) Either prorogation or adjournment or
preceding session, the monsoon session is adjournment Sine dine
dedicated largely to discussions on matters Solution: (d)
of public interest. It is during the monsoon Justification: The Rajya Sabha is a permanent
session that the members of the Parliament, chamber, and not subject ot dissolution. So, C
including the ministers, can bring forward is wrong.
the legislative proposals in the form of a • A sitting of Parliament can be terminated
Bill Winter Session: It is the shortest session by adjournment or adjournment sine die
that spans over a period of one month or prorogation or dissolution (only in the
generally between mid-November and mid- case of the Lok Sabha).
December. It takes up the issues that could • An adjournment suspends the work in a
not be deliberated upon and makes up for sitting for a specified time, which may be
the lack of legislative business during the hours, days or weeks.
second session of the Parliament • The presiding officer (Speaker or
Q.773 In addition to the members of a House, who Chairman) declares the House adjourned
has the right to speak and take part in the sine die, when the business of a session is
proceedings of either House of the Parliament completed.
or any of its committees? • Within the next few days, the President
issues a notification for prorogation of the

Indian Polity Question Bank P.205

02-Indian Polity_Q413-927.indd 205 8/7/2018 7:40:38 PM


session. However, the President can also 2. Prorogation lapses all business pending
prorogue the House while in session. before the House and the same cannot be
• A session of Parliament consists of many resumed when the House meets again,
meetings. Each meeting of a day consists unlike adjournment.
of two sittings, that is, a morning sitting Which of the above is/are correct?
from 11 am to 1 pm and post-lunch sitting (a)  1 only (b)  2 only
from 2 pm to 6 pm. (c)  Both 1 and 2 (d)  None
Q.775 Consider the following about adjournment Solution: (d)
and prorogation of the house. Refer previous answer
1. Unlike Adjournment, Prorogation of the Q.777 What is the “Quorum” in a sitting of any of
house not only terminates not only the the houses the Parliament?
sitting but also the session of the House. (a) Mere presence of the Presiding office of
2. Adjournment and prorogation both are the House is considered as quorum.
done by the Speaker. (b) Presence of the Presiding officer, leader
3. Both do not affect the bills or any other of the house and leader of opposition
business pending before the House. jointly constituted the Quorum.
Choose the correct answer using the codes (c) It is one-tenth of the total number of
below. members in each House including the
(a)  1 and 2 only (b)  2 and 3 only presiding officer.
(c)  1 and 3 only (d)  All of the above (d)  None of the above
Solution: (c) Solution: (c)
Learning: Quorum is the minimum number
Adjournment Prorogation of members required to be present in the
1. It only terminates 1. It not only terminates a House before it can transact any business.
a sitting and not sitting but also a session of It is one-tenth of the total number of
a session of the the House. members in each House including the
House presiding officer. It means that there must be
2. It is done by 2. It is done by the president at least 55 members present in the Lok Sabha
presiding officer of India. and 25 members present in the Rajya Sabha,
of the House if any business is to be conducted.
If there is no quorum during a meeting
3. It does not affect 3. It also does not affect the of the House, it is the duty of the presiding
the bills or any bills or any other business officer either to adjourn the House or to
other business pending before the House. suspend the meeting until there is a quorum.
pending before However, all pending
the House and notices (other than those for Q.778 Which of the following authorities adjourns
the same can be introducing the bills) lapse the Lok Sabha House or suspends the meeting
resumed when on prorogation and fresh in absence of a quorum?
the House meets notices have to be given for (a)  President of India
again. the next session. In Britian, (b)  Speaker, Lok Sabha
prorogation brings to an (c)  Leader of the House
end all bills or any other (d)  Secretary, Lok Sabha
business pending before the Solution: (b)
House. Learning: The speaker has the following
powers and duties:
Q.776 Which of these is/are the key differences • He maintains order and decorum in the
between Prorogation and Adjournment of House for conducting its business and
the House? regulating its proceedings. This is his
1. Prorogation only terminates a sitting primary responsibility and he has final
and not a session of the House, unlike power in this regard.
Adjournment.

P.206 For Civil Services Preliminary Examination

02-Indian Polity_Q413-927.indd 206 8/7/2018 7:40:38 PM


• He is the final interpreter of the provisions Solution: (a)
of (a) the Constitution of India, (b) the Rules The lower house of the Parliament (Lok
of Procedure and Conduct of Business Sabha) can be dissolved by the President on
of Lok Sabha, and (c) the parliamentary recommendation of the Prime Minister. In
precedents, within the House. other words, the prime minister can advise
• He adjourns the House or suspends the the President to dissolve the Lok Sabha
meeting in absence of a quorum. The before the expiry of its term and hold fresh
quorum to constitute a meeting of the elections. This means that the executive
House is one-tenth of the total strength of enjoys the right to get the legislature
the House. dissolved in a parliamentary system.
Q.779 Consider the following statements. Similarily, the legislature can also pass
Assertion (A): There cannot be quorum in no confidence motion against the executive
Parliament if the Leader of Opposition (LoP) and dismiss it. With it, the Lok Sabha is
is absent from House proceedings or is not dissolved if there is no possibility of forming
selected by the opposition. an alternative government.
Reason (R): All censure motions or Resignation of any minister will not end
resolutions against the government on behalf Lok Sabha–only that of the Prime Minister
of the house can be raised only by the Leader (that too when no other person is chosen as
of Opposition (LoP). Prime Minister) will bring the Lok Sabha to
In the context of the above, which of these dissolution
is correct? Q.781 The lower house of the Parliament, Lok
(a) A is correct, and R is an appropriate Sabha, can be dissolved by
explanation of A. (a) The President on recommendation of the
(b) A is correct, but R is not an appropriate Prime Minister
explanation of A. (b) The Prime Minister on the advice of the
(c)  A is correct, but R is incorrect. Council of Ministers
(d)  Both A and R are incorrect. (c) The Supreme Court on an inquiry
Solution: (d) conducted by a Parliamentary Committee
Justification: Quorum in Parliament simply (d) The Speaker on the recommendations of
means presence of more than 10% of the a Joint Committee of both houses
actual strength of the House. It is not related Solution: (a)
to the absence of presence of any particular Learning: The prime minister can advise the
member. If LoP or even the Leader of the President to dissolve the Lok Sabha before
house (lets say Prime Minister) is absent, the the expiry of its term and hold fresh elections.
house still conducts business. This means that the executive enjoys the
Censure motions or resolutions against right to get the legislature dissolved in a
the government on behalf of the house can parliamentary system.
be raised by any opposition member. So, R is Q.782 Consider the following statements.
also wrong. 1. Lok Sabha is not automatically dissolved
Q.780 In which of the following cases, the Lok after the end of its term and continues
Sabha may be dissolved? until new house is not constituted after
1. Prime Ministers recommending elections.
dissolution of Lok Saha to the President 2. The Speaker of Lok Sabha continues in
2. No-confidence motion passed against the office even after the dissolution of the
incumbent government House and until ‘immediately before the
3. Resignation of majority of cabinet ministers first meeting of the House’.
Choose the correct answer using the codes Which of the above is/are correct?
below. (a)  1 only (b)  2 only
(a)  1 and 2 only (b)  2 and 3 only (c)  Both 1 and 2 (d)  None
(c)  1 and 3 only (d)  All of the above Solution: (b)

Indian Polity Question Bank P.207

02-Indian Polity_Q413-927.indd 207 8/7/2018 7:40:38 PM


Justification: Statement 1: Unless sooner 6. A bill passed by both Houses but returned
dissolved or there is an extension of the by the president forreconsideration of
term, there is an automatic dissolution of Houses does not lapse.
the Lok Sabha by efflux of time, at the Q.784 When the Lok Sabha is dissolved, which of
end of the period of five years, even if no these bills do NOT lapse?
formal order of dissolution is issued by the 1. A bill pending in the Rajya Sabha but not
President. passed by the Lok Sabha
Statement 2: But, the above does not 2. A bill passed by the Lok Sabha but
apply to Speaker, and she continues in office pending in the Rajya Sabha
until the new house is constituted and its first 3. A bill passed by both Houses but returned
meeting started by the president for reconsideration of
Q.783 When the Lok Sabha is dissolved, all business Houses
including bills, motions, resolutions, notices, Select the correct answer using the codes
petitions and so on pending before it or its below.
committees lapse. However, certain bills do (a)  1 and 2 only (b)  1 and 3 only
not lapse on the dissolution of the Lok Sabha. (c)  2 and 3 only (d)  1, 2 and 3
They are? Solution: (b)
1. A bill passed by the Lok Sabha but
pending in the Rajya Sabha Devices of Parliament
2. A bill pending in the Rajya Sabha but not
passed by the Lok Sabha Q.785 The proceeding in Lok Sabha always starts
3. A bill passed by both Houses but pending with ‘X’, move to ‘Y’ and then tends to ‘Z’.
assent of the president The X, Y and Z here are
Select the correct answer using the codes (a) Zero hour, Business hour and Agenda of
below. the house
(a)  1 and 2 only (b)  2 and 3 only (b) Zero hour, Question hour and Agenda of
(c)  1 and 3 only (d)  1, 2 and 3 the house
Solution: (b) (c) Question hour, Zero hour and Agenda of
Learning: Some pending bills and all the house
pending assurances that are to be examined (d) Business hour, Zero hour and Question
by the Committee on Government Assurances hour
do not lapse on the dissolution of the Lok Solution: (c)
Sabha. The position with respect to lapsing Question Hour: The first hour of every
of bills is as follows: sitting of Parliament is generally reserved
1. A bill pending in the Lok Sabha lapses for the asking and answering of questions.
(whether originating in the Lok Sabha or Zero Hour: The time immediately following
transmitted to it by the Rajya Sabha). the Question Hour has come to be known
2. A bill passed by the Lok Sabha but as “Zero Hour”. It starts at around 12 noon
pending in the Rajya Sabha lapses. (hence the name) and members can, with
3. A bill not passed by the two Houses due prior notice to the Speaker, raise issues
to disagreement and if the president has of importance during this time. Typically,
notified the holding of a joint sitting discussions on important Bills, the Budget,
before the dissolution of Lok Sabha, does and other issues of national importance take
not lapse. place from 2 pm onwards.
4. A bill pending in the Rajya Sabha but
not passed by the Lok Sabha does not (1) Question Hour
lapse.
Q.786 The Parliament, while in session, begins
5. A bill passed by both Houses but
with the
pending assent of the president does not
(a)  Zero hour
lapse.
(b)  Question hour

P.208 For Civil Services Preliminary Examination

02-Indian Polity_Q413-927.indd 208 8/7/2018 7:40:38 PM


(c)  Agenda of the day C. Short Notice 3. One for which
(d)  Half-an-hour discussion Question an oral
Solution: (b) answer is required to
Explanation and Learning: The Parliament be given by a minister
begins with a question hour. The question on the floor of the
hour is an important mechanism through House
which MPs can elicit information about the A B C
working of the government. This is a very (a) 1 2 3     (b) 2 1 3
important way through which the Parliament (c) 3 1 2     (d) 3 2 1
controls the executive. Solution: (d)
By asking questions the government is
alerted to its shortcomings, and also comes to (2) Zero Hour
know the opinion of the people through their
representatives in the Parliament, i.e. the Q.789 Consider the following statements about the
MPs. Asking questions of the government is proceedings of the Indian Parliament:
a crucial task for every MP. 1. The proceedings of the Parliament begin
with zero hour.
Q.787 A starred question
2. The question hour which follows zero
1. Compulsorily requires a written answer
hour, is characterized by MP asking
2. Supplementary questions cannot follow
questions from the government.
3. Must be asked by giving a notice of less
3. Opposition parties are not allowed to raise
than ten days
any matter in the zero hour.
Select the correct answer using the codes
Choose the correct answer using the codes
below.
below:
(a)  1 and 2 only (b)  2 and 3 only
(a)  1 and 2 (b)  2 and 3
(c)  1 only (d)  None of the above
(c)  1 and 3 (d)  None of the above
Solution: (d)
Solution: (d)
Justification: The questions are of three kinds,
It starts from question hour and zero hour
namely, starred, unstarred and short notice.
follows it. General matters can be raised by
A starred question (distinguished by an
any MP in the zero hour. So, the opposition is
asterisk) requires an oral answer and hence
allowed to raise matters.
supplementary questions can follow.
Question hour is a very important way of
An unstarred question, on the other
controlling the executive by keeping them
hand, requires a written answer and hence,
accountable for their deeds and misdeeds
supplementary questions cannot follow.
before the house.
A short notice question is one that is asked
by giving a notice of less than ten days. It is Q.790 Which of the following statements about the
answered orally. Zero Hour in Parliament is INCORRECT?
(a) The zero hour is not mentioned in the
Q.788 Match the following:
Rules of Procedure of the house.
Parliamentary Meaning
(b) Matters can be raised by members
Terms
without prior notice.
A.  Starred 1. One asked by a
(c) The zero hour starts before the question
Question member
hour after which the agenda for the day
on matters of public
is taken up.
importance of an
(d) It is an Indian innovation in the field of
urgent nature
parliamentary procedures and has been
B.  Unstarred 2. One for which
in existence since 1962.
Question the
Solution: (c)
Concerned minister
Justification: Unlike the question hour, the
has to lay on the table
zero hour is not mentioned in the Rules of
a written answer

Indian Polity Question Bank P.209

02-Indian Polity_Q413-927.indd 209 8/7/2018 7:40:38 PM


Procedure. Thus it is an informal device Which of these is/are true?
available to the members of the Parliament (a)  1 and 2 (b)  2 and 3
to raise matters without any prior notice. (c)  1 and 3 (d)  Only 1
The zero hour starts immediately after the Solution: (c)
question hour and lasts until the agenda for It starts after the Question hour, which is the
the day (ie, regular business of the House)  is first thing done in the house. Zero hour is an
taken up. In other words, the time gap Indian innovation and an informal device
between the question hour and the agenda is available to the MPs
known as zero hour.
Q.791 Consider the following about Zero Hour in (3) Motions
Parliament.
1. MPs are free to raise any matter which (a) Censure Motion
they believe to be important in the Zero
Hour. (b) Calling Attention Motion
2. Ministers are bound to reply on the house
floor on matters raised in Zero Hour. Q.793 Which of the following statements about the
Which of the above is/are correct? Calling Attention Motion is INCORRECT?
(a)  1 only (b)  2 only (a) It can be introduced only in Lok Sabha.
(c)  Both 1 and 2 (d)  None (b) The motion seeks an authoritative
Solution: (a) statement from the Minister concerned
Justification: Statement 1: Apart from on that matter.
deliberating on bills, control may also be (c) Like the zero hour, it is also an Indian
exercised during the general discussions innovation in the parliamentary
in the House. The Question Hour, which procedure.
is held every day during the sessions (d) It is mentioned in the Rules of Procedure.
of Parliament, where Ministers have to Solution: (a)
respond to searching questions raised by Justification: It can be introduced in both
the members; Zero Hour where members the houses. A member may, with the previous
are free to raise any matter that they think permission of the Speaker, call the attention
is important (though the ministers are not of a Minister to any matter of urgent public
bound to reply), half-an-hour discussion on importance and the Minister may make a
matters of public importance, adjournment brief statement or ask for time to make a
motion etc. are some instruments of statement at a later hour or date.
exercising control. It is the Censure motion and no-
Statement 2: Nobody knows which issue Confidence motion that can be moved only
a member would raise during this hour. As in Lok Sabha.
a result, questions so raised without prior
notice may be of no importance. Hence, the (c) Adjournment Motion
Ministers are not bound to reply and that too Q.794 In Parliament, the primary object of an
immediately on the house floor. adjournment motion is to
Q.792 Consider the following statements about the (a) suspend the proceedings of the house for
zero-hour in the Lok Sabha: that particular session
1. It is used to raise matters without any (b) draw the attention of the House to a
prior notice. recent matter of urgent public importance
2. It starts as soon as the proceedings of the (c) censure the government for its lack of
Lok Sabha for the day begin. action on sensitive issues
3. It is not mentioned in the Rules of (d) adjourn the introduction of a bill for the
procedure of the house. next session of the house
Solution: (b)

P.210 For Civil Services Preliminary Examination

02-Indian Polity_Q413-927.indd 210 8/7/2018 7:40:38 PM


The primary object of an adjournment (a) A is correct, and R is an appropriate
motion is to draw the attention of the House explanation of A.
to a recent matter of urgent public importance (b) A is correct, but R is not an appropriate
having serious consequences and in regard to explanation of A.
which a motion or a resolution with proper (c) A is correct, but R is incorrect.
notice will be too late. The matter proposed (d)  Both A and R are incorrect.
to be raised should be of such a character Solution: (a)
that something very grave which affects the
whole country and its security has happened (d) No Confidence Motion
and the House is required to pay its attention
immediately by interrupting the normal Q.797 Which of the following regarding the
business of the House. The adjournment procedure of motion of no-confidence in
motion is thus an extraordinary procedure Council of Ministers is INCORRECT?
which, if admitted, leads to setting aside the (a) It needs to state the reasons for its
normal business of the House for discussing adoption in the Lok Sabha.
a definite matter of urgent public importance. (b) The motion will not be admitted if less
than 50 MPs of Lok Sabha support it.
Q.795 The Adjournment motion is introduced in the (c) If the Speaker grants leave to the motion,
Parliament to draw attention of the House to a decision on it need not be taken
a definite matter of urgent public importance. immediately.
Which of the following statements about it is (d)  None of the above
INCORRECT? Solution: (a)
(a) It needs the support of 50 members to be Justification: A motion expressing want of
admitted. confidence in the Council of Ministers may
(b) It involves an element of censure against be made subject to the following restrictions,
the government. namely:
(c) Rajya Sabha is not permitted to make use • Leave to make the motion shall be asked
of this device. for by the member when called by the
(d) If it is passed by a special majority, the Speaker;
sitting of the house is adjourned after the • The member asking for leave shall, give
discussion. to the Secretary-General a written notice
Solution: (d) of the motion which he proposes to move.
Justification: As it interrupts the normal • The motion does not need to state the
business of the House, it is regarded as an reasons for its adoption in the Lok Sabha.
extraordinary device. It involves an element So, (a)  is incorrect.
of censure against the government and hence • The Speaker is of opinion that the motion
Rajya Sabha is not permitted to make use of is in order, he shall read the motion to the
this device. The discussion on an adjournment House and shall request those members
motion should last for not less than two hours who are in favour of leave being granted
and thirty minutes. So, (b) and (c) are correct. to rise in their places.
The house need not be adjourned. So, (d) • If not less than fifty members rise
is wrong. accordingly, the Speaker shall declare that
Q.796 Consider the following with reference to leave is granted and that the motion will
Adjournment Motion introduced in the be taken up on such day, not being more
Parliament. than ten days from the date on which the
Assertion (A): Rajya Sabha is not permitted leave is asked for as he may appoint.
to make use of this device. • If less than fifty members rise, the Speaker
Reason (R): It involves an element of shall inform themember that he has not
censure against the government. the leave of the House.
In the context of the above, which of these
is correct?

Indian Polity Question Bank P.211

02-Indian Polity_Q413-927.indd 211 8/7/2018 7:40:38 PM


Q.798 For a no-confidence motion to be admitted in Learning: Censure motion is moved for
Lok Sabha censuring the council of ministers for specific
1. Speaker must give consent to it. policies and actions. If it is passed in the Lok
2. Support of at least 50 members is Sabha, the council of ministers need not
necessary. resign from the office.
3. Leader of Opposition must lead the No-confidence motion is moved for
motion. ascertaining the confidence of Lok Sabha
4. Both houses of Parliament must be in in the council of ministers. If it is passed in
session. the Lok Sabha, the council of ministers must
5. Prior permission from the President must resign from office. The motion needs the
be obtained. support of 50 members to be admitted
Choose the correct answer using the codes
below. (e) Previlege Motion
(a)  1 and 2 only (b)  1, 2 and 5 only
(c)  2, 3 and 4 only (d)  2 only Q.800 Privilege Motion is brought in the Indian
Solution: (d) Parliament to fulfill which of the following
Article 75 of the Constitution says that the primary objectives?
council of ministers shall be collectively (a) To raise a matter regarding the breach of
responsible to the Lok Sabha. It means parliamentary privileges.
that the ministry stays in office so long as (b) To raise a demand regarding the privilege
it enjoys confidence of the majority of the of the ministers.
members of the Lok Sabha. In other words, (c) To raise a matter of urgent public
the Lok Sabha can remove the ministry from importance.
office by passing a no-confidence motion. (d) To raise a matter such as impeachment of
The motion needs the support of 50 members the President
to be admitted. Solution: (a)
It is concerned with the breach of
Q.799 Which of the following correctly point out parliamentary privileges by a minister by for
the difference(s) between Censure Motion example withholding facts or a case or giving
and No Confidence Motion? wrong or distorted information. Its purpose is
1. No-confidence motion should state the to censure the concerned minister
reasons for its adoption in the Lok Sabha,
Censure motion need not. Q.801 Consider the following about the “Privilege
2. No-confidence motion can only be moved motion” in Parliament.
against the entire council of ministers, 1. It can be admitted only in Lok Sabha.
Censure motion can be moved only 2. It is always raised against either the entire
against individual ministers. council of Ministers or the House.
Which of the above is/are correct? 3. All matters related to the motion must be
(a)  1 only (b)  2 only referred to the privilege committee of the
(c)  Both 1 and 2 (d)  None Lok Sabha.
Solution: (d) 4. It can be raised by a private member.
Justification: Statement 1: No-confidence Select the correct answer using the codes
motion need not state the reasons for its below.
adoption in the Lok Sabha, whereas Censure (a)  2 and 3 only (b)  4 only
motion should. (c)  1 and 4 only (d)  1 and 3 only
Statement 2: Censure motion can be Solution: (b)
moved against an individual minister or a Justification. Statement 1: It can be rised in
group of ministers or the entire council of both the houses
ministers. Noconfidence motion can only be Statement 2: Privilege Motion is moved
moved against the entire council of ministers. by a member when he feels that a minister
So, 2 is wrong. or any member has committed a breach of

P.212 For Civil Services Preliminary Examination

02-Indian Polity_Q413-927.indd 212 8/7/2018 7:40:38 PM


privilege of the House or one or more of its of Parliament to raise discussions and
members by withholding facts of a case or by debates to examine and criticise the
giving wrong or distorted facts. government and administration for its
Its purpose is to censure the concerned lapses and failures.
minister or any other member. Q.803 The “Motion of Thanks”, addressed by the
Statement 1: Each of the two houses, the President, is put to vote in the first session
Lok Sabha and the Rajya Sabha, has separate after each general election and the first
privilege committees, made up of their session of every fiscal year. Consider the
members. It can be introduced in both. following about it.
Statement 3: The presiding officers of the Assertion (A): The motion needs to be
two Houses, the Speaker and the Chairperson passed in Lok Sabha only.
respectively, can dismiss privilege notices, or Reason (R): The Council of Ministers is
refer them to the privilege committee, or get mainly responsible to the Lok Sabha.
a sense of the House before taking a decision In the context of the above, which of these
Statement 4: Privilege motion can be is correct?
moved by any lawmaker/MP against anyone (a) A is correct, and R is an appropriate
accused of breaching parliamentarians’ explanation of A.
privileges, their special rights and immunities. (b) A is correct, but R is not an appropriate
explanation of A.
(f) Motion of Thanks (c) A is incorrect, but R is correct.
Q.802 Consider the following with reference to the (d)  Both A and R are incorrect.
Motion of Thanks. Solution: (c)
1. It is addressed by the Leader of the House. Justification and Learning: The first session
2. The motion is put to vote only in Lok after each general election and the first
Sabha. session of every fiscal year is addressed by the
3. It is addressed at the beginning of every president. In this address, the president outlines
new session of the Parliament. the policies and programmes of the government
Select the correct answer using the codes in the preceding year and ensuing year.
below. This address of the president, which
(a)  1 and 2 only corresponds to the ‘speech from the Throne
(b)  3 only in Britain’, is discussed in both the Houses
(c)  1, 2 and 3 of Parliament on a motion called the ‘Motion
(d)  None of the above of Thanks’.
Solution: (d) At the end of the discussion, the motion is
Justification: The first session after each put to vote. This motion must be passed in the
general election and the first session of every House. Otherwise, it amounts to the defeat of
fiscal year is addressed by the president. So, the government.
1 is wrong. This inaugural speech of the president
• In this address, the president outlines is an occasion available to the members of
the policies and programmes of the Parliament to raise discussions and debates
government in the preceding year and to examine and criticise the government and
ensuing year. administration for its lapses and failures
• This address of the president is called
the ‘Motion of Thanks’. At the end of the (g) Closure Motion
discussion, the motion is put to vote. Q.804 With reference to the conduct of government
• This motion must be passed in the House. business in the Parliament of India, the term
Otherwise, it amounts to the defeat of the ‘closure’ refers to
government. (a) suspension of debate at the termination
• This inaugural speech of the president of a day’s sitting of the Parliament
is an occasion available to the members

Indian Polity Question Bank P.213

02-Indian Polity_Q413-927.indd 213 8/7/2018 7:40:38 PM


(b) a rule of legislative procedure under (c) when the time allotted for budget
which further debate on a motion can be discussions is exhausted
halted (d) when the demands are passed with a cut.
(c) the termination of a parliamentary Solution: (c)
session
(d) refusal on the part of the government to (4) Point of Order
have the opposition look at important
document. Q.807 A “Point of Order” raised in the Parliament
Solution: (b) is related to
Closure Motion It is a motion moved by a (a) Order of sitting of opposition and ruling
member to cut short the debate on a matter party members in the House
before the House. If the motion is approved by (b) Matters of urgent attention brought
the House, debate is stopped forthwith and the before the Government
matter is put to vote. There are four kinds of (c) Interpretation or enforcement of the
closure motions: Rules of Procedure and Conduct of
(a) Simple Closure: It is one when a member Business
moves that the ‘matter having been (d) Zero hour matters that are raised without
sufficiently discussed be now put to any notice
vote’. Solution: (c)
(b) Closure by Compartments: In this Learning: A Point of Order relates to the
case, the clauses of a bill or a lengthy interpretation or enforcement of the Rules
resolution are grouped into parts before of Procedure and Conduct of Business in the
the commencement of the debate. The House or convention or such Articles of the
debate covers the part as a whole and the Constitution as regulate the business of the
entire part is put to vote. House and raises a question which is within
(c) Kangaroo Closure: Under this type, only the cognizance of the Speaker.
important clauses are taken up for debate A Point of Order may be raised only in
and voting and the intervening clauses relation to the business before the House at
are skipped over and taken as passed. the moment, provided that the Speaker may
(d) Guillotine Closure: It is one when the permit a Member to raise a Point of Order
undiscussed clauses of a bill or a resolut- during the interval between the termination of
ion are also put to vote along with the one item of business and the commencement
discussed ones due to want of time (as of another if it relates to maintenance of order
the time allotted for the discussion is in, or arrangement of business before, the
over). House.
A Member may formulate a Point of
Q.805 According to the Parliamentary procedure
Order and the Speaker shall decide whether
the term Guillotine means that the Speaker
the point raised is a Point of Order and if so
(a) stops the business on hand and puts the
give the decision thereon, which is final.
bill to vote
(b) adjourns the House before expiry of the Q.808 A Point of Order is raised in the Parliament
time when
(c) disallows voting on a motion 1. A member of the house has made a factual
(d) reprimands a member for misbehavior error in his/her speech.
Solution: (a) 2. The privilege of a member of the
Parliament or the house is breached.
Q.806 The Speaker of the Lok Sabha has been
Which of the above is/are correct?
empowered to exercise the “Guillotine”
(a)  1 only (b)  2 only
(a)  after the demands are discussed
(c)  Both 1 and 2 (d)  None
(b) after the demands are discussed and
Solution: (d)
passed

P.214 For Civil Services Preliminary Examination

02-Indian Polity_Q413-927.indd 214 8/7/2018 7:40:38 PM


Justification: A member can raise a point of question hour, half-an hour discussion, short
order when the proceedings of the House do duration discussion or under adjournment
not follow the normal rules of procedure. motion, calling attention notice or under any
It is usually raised by an opposition rule of the House can be raised under the
member in order to control the government. special mention in the Rajya Sabha.
It is an extraordinary device as it suspends • Its equivalent procedural device in the
the proceedings before the House. No debate Lok Sabha is known as ‘Notice (Mention)
is allowed on a point of order. Under Rule 377’.
• When a matter is raised under Rule 377,
(5) Half-an-hour Discussion it is not necessary that the Minister
concerned with the subject matter of the
Q.809 Consider the following about Half-an-hour notice should he present in the House to
discussions in Lok Sabha. answer the point immediately.
1. No prior notice is required for raising • No reply from the Minister is anticipated
matters. in this rule. The Minister may make a
2. The speaker is bound to admit matters statement if necessary, later on.
raised by members that seek to revise the
Q.811 In the Lok Sabha, which of the following
policy of Government.
matters are dealt under Rule 377, that is also
Which of the above is/are correct?
often in news?
(a)  1 only (b)  2 only
(a) Urgent national issues under the Calling
(c)  Both 1 and 2 (d)  None
Attention Motion
Solution: (d)
(b) No-confidence motion or other disciplinary
Justification: Statement 1: A member
proceedings against the Ministers
wishing to raise a matter shall give notice in
(c) All money bills that are not ratified by the
writing to the Secretary-General three days
President
in advance of the day on which the matter is
(d)  None of the above
desired to be raised, and shall shortly specify
Solution: (d)
the point or points that he wishes to raise.
Provided that the notice shall be accompanied
by an explanatory note stating the reasons for (7) Resolutions
raising discussion on the matter in question. Q.812 The members of Parliament can move
Statement 2: The speaker is not obliged to resolutions to draw the attention of the House
admit the motion if it focuses on changing an or the government to matters of general
existing government policy, since it no half- public interest. Consider the following
hour discussion motions are passed by the about them.
house and thus they are admitted entirely on 1. A Government resolution can be moved
the request of the member. only by a minister and it can be taken
up every day of the week the house is
(6) Special Mention in session.
2. A Statutory resolution can be moved
Q.810 ‘Notice (Mention) Under Rule 377’ in Lok
either by a private member or a minister.
Sabha can be used to:
Which of the above is/are correct?
1. Raise non-public matters in question hour
(a)  1 only (b)  2 only
2. Ask questions during ‘calling attention’
(c)  Both 1 and 2 (d)  None
motion
Solution: (b)
Which of the above is/are correct?
Justification: Statement 1: It can be taken
(a)  1 only (b)  2 only
up any day from Monday to Thursday, other
(c)  Both 1 and 2 (d)  None
than Friday which is reserved for private
Solution: (d)
member’s resolution (raised by members
Justification: A matter which is not a point
other than a Minister). So, 1 is incorrect.
of order or which cannot be raised during

Indian Polity Question Bank P.215

02-Indian Polity_Q413-927.indd 215 8/7/2018 7:40:39 PM


A Private Member’s Resolution is A statutory resolution can be moved either
discussed only on alternate Fridays and in the by a minister or a private member. It is so
afternoon sitting. because it is always tabled in the pursuance
Statement 2: It is so called because it is of a provision in the constitution or an act of
always tabled in pursuance of a provision in Parliament.
the Constitution or an Act of Parliament.
Learning: Resolutions are different from (8) Others
motions in the following respects.
• All resolutions come in the category of Q.815 A member of the House in Parliament can be
substantive motions, that is to say, every asked by the speaker to stop speaking and let
resolution is a particular type of motion. another member speak. This is known as
• All motions need not necessarily be (a)  crossing the floor by the MP
substantive. Further, all motions are not (b) maintaining the decorum by the speaker
necessarily put to vote of the House, (c)  yielding the floor
whereas all the resolutions are required to (d)  speech adjournment
be voted upon. Solution: (c)
Q.813 Consider the following statements about “Crossing the floor” depicts that the passing
Parliamentary proceedings: between the member addressing the House
1. All resolutions come in the category of and the Chair which is considered breach
substantive motions. of Parliamentary etiquette. “Adjournment
2. After the “Half-an-hour discussion”, there of Debate” means that the adjournment on a
is a formal motion or voting before the motion adopted by the House, of the debate
house. on a Motion/Resolution/Bill on which the
3. All motions are put to the vote of the House is then engaged until a future day or
concerned house of the Parliament. sine die as specified in the motion.
Which of these is/are true? Q.816 Yielding the floor in Indian Parliament is
(a)  Only 1 (b)  Only 2 related to
(c)  Both (d)  None of the above (a) Passing between the member addressing
Solution: (a) the House and the Chair which is
Every resolution is a special type of considered breach of Parliamentary
motion. All motions need not necessarily be etiquette.
substantive. Further, not all motions are put to (b) A Member raising a discussion on topics
the vote of the house, whereas all resolutions different from the business schedule of
are put the house
Q.814 Consider the following statements. (c) Termination of a sitting of the House
1. A private member of the Parliament (other without any definite date being fixed for
than a minister) has a right to move a the next sitting.
resolution anytime s/he wants, subject to (d)  None of the above
the approval of the speaker. Solution: (d)
2. A statutory resolution can only be moved Learning: Option (a) is called “Crossing the
by a minister. floor”.
3. All resolutions come in the category of Option (b) is called adjournment motion.
substantive motions. Option (c) is called adjournment sine die.
Which of these is/are true? Yielding the floor means the Speaker can
(a)  1 and 2 (b)  2 and 3 ask a member of the House to stop speaking
(c)  1 and 3 (d)  Only 3 and let another member speak.
Solution: (d) Q.817 Consider the following statements.
A private member can move resolution only 1. Privilege Motion is concerned with the
on alternate Fridays. breach of parliamentary procedures by a
non-member of Parliament.

P.216 For Civil Services Preliminary Examination

02-Indian Polity_Q413-927.indd 216 8/7/2018 7:40:39 PM


2. Calling Attention Motion must be raised 1. The lay-out of the Chamber of their
as a pre-cursor of No-confidence motion Youth Parliament should resemble as far
to highlight the poor performance of as possible the lay-out of the Chamber of
government. Lok Sabha.
3. Censure motion can be moved against a 2. The first part of a sitting in the Youth
Minister only in the Lok Sabha. parliament should be devoted to oral
Select the correct answer using the codes questions.
below. 3. In the Youth Parliament, the Parliamentary
(a)  1 and 2 only (b)  3 only Business is arranged in the order in
(c)  2 and 3 only (d)  1 only which it is shown in the ‘Direction by the
Solution: (b) Speaker’.
Justification: Statement 1: It is concerned Select the correct answer using the codes
with the breach of parliamentary privileges below.
by a minister. (a)  1 and 2 only (b)  3 only
• It is moved by a member when he feels (c)  1 and 3 only (d)  1, 2 and 3
that a minister has committed a breach Solution: (d)
of privilege of the House or one or more Justification: Statement 1: The working of
of its members by withholding facts of Youth Parliament should broadly follow the
a case or by giving wrong or distorted pattern of the working of Lok Sabha. Same
facts. goes for the physical layout and other things.
• Its purpose is to censure the concerned Statement 2: It should be devoted to
minister. oral question commonly known as Starred
Statement 2: It is introduced in the Parliament Questions in the Indian Parliament.
by a member to call the attention of a minister It is the most popular hour for the
to a matter of urgent public importance, and members of Parliament and also for the
to seek an authoritative statement from him Public. From the point of view of a Minister
on that matter. it is the most searching test of his grasp on
• Like the zero hour, it is also an Indian the items or business allocated to him, his
innovation in the parliamentary procedure ability to lead as well as his ready with when
and has been in existence since 1954. answering supplementary.
However, unlike the zero hour, it is Statement 3: The order in which various
mentioned in the Rules of Procedure. items of business are normally taken up in the
Statement 3: It can be brought against the House is as follows:
ruling government or against any minister • Oath of affirmation
for the failure to act or seeking disapproval • Laying on the Table the President’s
of their policy. It can be moved only in Lok address to both Houses of Parliament
Sabha only by the opposition. • Obituary references and so on.
• A censure motion must specify the This resembles that of the Lok Sabha, which
charges against the government for which is decided by Lok Sabha Speaker and in the
it is moved. If a censure motion is passed Youth parliament by the concerned speaker.
in the Lok Sabha, the Council of ministers Q.819 The scheme of Youth Parliament was started
is bound to seek the confidence of the Lok with which of the following objectives?
Sabha as early as possible. 1. To acquaint first time young MPs with
the rules and procedures of the houses of
(9) Youth Parliament Parliament
2. To inculcate in the student community the
Q.818 Consider the following with reference to
basic values of democracy
the guidelines released by the Ministry of
3. To provide necessary training and capacity
Parliamentary Affairs on Youth Parliament.
building to young candidates contesting
national and state level elections

Indian Polity Question Bank P.217

02-Indian Polity_Q413-927.indd 217 8/7/2018 7:40:39 PM


Select the correct answer using the codes It has supported work in multiple fields
below. through varied mediums such a community
(a)  1 only (b)  2 and 3 only initiatives, the performing and visual arts,
(c)  2 only (d)  None of the above literary and research projects, interactive
Solution: (c) workshops, policy and government interaction
Justification: It was started on the and research and advocacy work amongst
recommendation of the Fourth All India Whips others
Conference. Its objectives are as follows.
• to acquaint the younger generations with Legislative Procedure
practices and procedures of Parliament;
• to imbibe the spirit of discipline and Q.821 Consider the following statements:
tolerance cultivating character in the 1. Members other than ministers cannot
minds of youth; and introduce bills.
• to inculcate in the student community the 2. Introduction of any new tax requires the
basic values of democracy and to enable approval of the Lok Sabha and Rajya
them to acquire a proper perspective on Sabha
the functioning of democratic institutions. 3. Budget is a mechanism for checks and
The ministry of parliamentary affairs provides balance system adopted in India.
necessary training and encouragement to the 4. A bill proposed by a minister is described
states in introducing the scheme. as Private Bill
Which of the above statements is/are correct?
Q.820 Consider the following statements about
(a)  2 Only (b)  3 Only
Youth Parliament.
(c)  2 and 3 (d)  1, 2, 3 and 4
1. It was established by an executive order
Solution: (b)
on the recommendation of the then Lok
No major bill is introduced in the Parliament
Sabha Speaker.
without the approval of the Cabinet. Members
2. It gives young elected MPs an opportunity
other than ministers can also introduce bills
to hone their debating skills as the debates
but these have no chance of being passed
are held between Parliamentary sessions
without the support of the government. If the
in Lok Sabha.
Government of India proposes to introduce
Which of the above is/are correct?
any new tax, it has to get the approval of
(a)  1 only (b)  2 only
the Lok Sabha. The Financial powers of the
(c)  Both 1 and 2 (d)  None
Parliament, involve grant of resources to the
Solution: (d)
government to implement its programmes.
Justification: Statement 1: It is a support
The government has to give an account to
organization for young people in India.
the Legislature about the money it has spent
It was established in July 2002 by social
and resources that it wishes to raise. The
entrepreneur Ishita Chaudhry with support
legislature also ensures that the government
from the India Habitat Centre, New Delhi.
does not misspend or overspend. This is
In 2007, The Youth Parliament was legally
done through the budget and annual financial
incorporated ‘The YP Foundation’–a
statements. A bill is a draft of the proposed
charitable trust.
law. There can be different types of bills.
Statement 2: The organization funds and
When a non- minister proposes a bill, it is
supports innovative work by young people,
called private member’s Bill. A bill proposed
by providing young people between the ages
by a minister is described as Government
of 13 and 28 years with a resource base to
Bill. Let us now see the different stages in the
conceptualize and execute their own projects
life of a bill.
and initiatives working with socio-cultural,
economic, legal and environmental issues Q.822 Consider the following statements.
they are passionate about. 1. A bill is required to pass through the Rajya
Sabha for it to become a law.

P.218 For Civil Services Preliminary Examination

02-Indian Polity_Q413-927.indd 218 8/7/2018 7:40:39 PM


2. A bill cannot become an Act without the Choose the correct answer using the codes
assent of the President. below:
Which of these is/are true? (a)  1 and 2 (b)  2 and 3
(a)  1 only (b)  2 only (c)  1 and 3 (d) All of the above
(c)  Both 1 and 2 (d)  None Solution: (c)
Solution: (c) The Parliament enacts legislations for the
Normally, the Rajya Sabha is as powerful as country. Despite being the chief law making
the Lok Sabha when passing the legislations. body, the Parliament often merely approves
If the Rajya Sabha rejects the bill passed by legislations. The actual task of drafting the
Lok Sabha then the President has to call for a bill is performed by the bureaucracy under
joint sitting of both the houses. the supervision of the minister concerned.
The substance and even the timing of the bill
(1) Public Bill are decided by the Cabinet. No major bill
is introduced in the Parliament without the
Q.823 Which of the following is correct with approval of the Cabinet. Members other than
reference to a public bill? ministers can also introduce bills but these
(a) It can be introduced by any Member of have no chance of being passed without the
Parliament after taking prior permission support of the government.
from the President However, in a democracy, legislature
(b) It must be first introduced in Lok Sabha. controls taxation and the way in which money
(c) It needs to be passed by special majority is used by the government. If the Government
in both houses to become a law. of India proposes to introduce any new tax, it
(d)  None of the above. has to get the approval of the Lok Sabha. The
Solution: (d) Financial powers of the Parliament, involve
Justification: A public bill can be introduced grant of resources to the government to
only by a Minister, since all such bills are implement its programmes. The government
considered as government bills. So, A is has to give an account to the Legislature about
wrong. the money it has spent and resources that it
It can be introduced in any house first. So, wishes to raise. The legislature also ensures
B is wrong. that the government does not misspend or
It is similar to an ordinary bill in that it overspend. This is done through the budget
needs to be passed by a simple majority. and annual financial statements.
On the other hand, a private bill is
Q.825 Which of the following statements about
introduced by members other than ministers
Public Bills is INCORRECT?
in the Parliament. There is a separate day
1. It is introduced in the Parliament by a
reserved for introducing private member’s
minister.
bill.
2. It can be introduced without any prior
Q.824 Consider the following statements as follows. notice in the house.
1. A public bill is introduced in the Which of the above is/are correct?
Parliament only after the approval of the (a)  1 only (b)  2 only
cabinet. (c)  Both 1 and 2 (d)  None
2. The permanent bureaucracy is no Solution: (a)
concerned with either framing, changing Justification: Statement 1: Public bills are
or anywhere in the legislative process as introduced by Ministers, whereas private bills
they are supposed to be politically neutral. are introduced by any Member of Parliament
3. The approval of Lok Sabha is mandatory other than a minister.
if the government wants to introduce a Statement 2: Introduction of public bill
new tax. in the House requires seven days’ notice.
Introduction of private bill in the House
requires one month’s notice.

Indian Polity Question Bank P.219

02-Indian Polity_Q413-927.indd 219 8/7/2018 7:40:39 PM


Learning: Public bill reflects of the policies 3. Appropriation of money out of the
of the government (ruling party). Its rejection Consolidated Fund of India
by the House amounts to the expression of Select the correct answer using the codes
want of parliamentary confidence in the below.
government and may lead to its resignation. (a)  1 and 2 only (b)  2 and 3 only
Q.826 Which of the following differentiates (c)  1 and 3 only (d)  1, 2 and 3
between a private bill and a public bill? Solution: (d)
(a) A private bill is not disclosed in public as it Learning: It includes the following matters:
contains sensitive matters, whereas a public The imposition, abolition, remission,
bill is published in the Gazette of India. alteration or regulation of any tax;
(b) A private bill is related to non-public goods The regulation of the borrowing of money
and commercial aspects of government by the Union government–
administration, whereas public bill relates • The custody of the Consolidated Fund of
to substantial public goods. India or the contingency fund of India, the
(c) A private bill can be proposed only to the payment of moneys into or the withdrawal
Council of Ministers, whereas a public of money from any such fund;
bill is a draft of the proposed law in • The appropriation of money out of the
Parliament. Consolidated Fund of India;
(d) A private bill is introduced by a non- • Declaration of any expenditure charged
Minister, whereas a public bill is on the Consolidated Fund of India
introduced by a Minister. or increasing the amount of any such
Solution: (d) expenditure;
Learning: A Member of Parliament, other • The receipt of money on account of the
than a Minister (i.e., not a member of the Consolidated Fund of India or the public
Government) is known as a Private Member. account of India or the custody or issue of
A Bill initiated by any such member is called such money, or the audit of the accounts
a Private Member’s Bill. of the Union or of a state; or
A bill proposed by a minister is described • Any matter incidental to any of the matters
as Government Bill. There are certain specified above. However, a bill is not to
restrictions on the introduction of a private be deemed to be a money bill by reason
member’s bill. only that it provides for:
In Rajya Sabha and Lok Sabha, as also • The imposition of fines or other pecuniary
in other State Legislatures, a particular day penalties, or
of every week is kept aside for transation of • The demand or payment of fees for
Private Members’ Legislative Business. In licenses or fees for services rendered; or
Rajya Sabha two and half hours i.e., from • The imposition, abolition, remission,
2.30 p.m. to 5.00 p.m. on every alternate alteration or regulation of any tax by any
Friday are generally allotted for transaction of local authority or body for local purposes.
the Private Members’ Legislative Business. If any question arises whether a bill is
a money bill or not, the decision of the
Speaker of the Lok Sabha is final.
(2) Money Bill
Q.828 A Bill is deemed to be a ‘Money Bill’ if it
Q.827 Article 110 of the Constitution deals with the contains ‘only’ provisions dealing with
definition of money bills. It states that a bill is 1. Imposition, abolition, remission,
deemed to be a money bill if it contains ‘only’ alteration or regulation of any tax
provisions dealing with all or any of which of 2. Appropriation of money from the
the following matters? Consolidated Fund of India
1. Abolition of a tax 3. Declaration of any expenditure charged
2. Regulation of the borrowing of money by on the Consolidated Fund of India
the Union government

P.220 For Civil Services Preliminary Examination

02-Indian Polity_Q413-927.indd 220 8/7/2018 7:40:39 PM


4. Regulation of the borrowing of money by Justification: After a money bill is passed by
the Union government the Lok Sabha, it is transmitted to the Rajya
Select the correct answer using the code Sabha for its consideration.
given below. • The Rajya Sabha has restricted powers
(a)  1 and 2 only (b)  2, 3 and 4 only with regard to a money bill. It cannot
(c)  1 and 4 only (d)  1, 2, 3 and 4 reject or amend a money bill. It can only
Solution: (d) make the recommendations. It must
Q.829 Consider the following statements about a return the bill to the Lok Sabha within
money bill. fourteen days, wither with or without
1. A money bill can only be introduced in the recommendations.
Lok Sabha. • The Lok Sabha can either accept or reject
2. Rajya Sabha cannot reject or amend a all or any of the recommendations of the
money bill. Rajya Sabha.
Which of the above is/are correct? • If the Lok Sabha accepts any
(a)  1 only (b)  2 only recommendation, the bill is then deemed
(c)  Both 1 and 2 (d)  None to have been passed by both the Houses in
Solution: (c) the modified form.
Justification: The Constitution lays down a • If the Lok Sabha does not accept any
special procedure for the passing of money recommendation, the bill is then deemed
bills in the Parliament. to have passed by both the Houses in the
1. A money bill can only be introduced form originally passed by the Lok Sabha
in the Lok Sabha and that too on the without any change.
recommendation of the president. Q.831 That “a Money Bill shall not be introduced in
2. Every such bill is considered to be a the Council of States” is a
government bill and can be introduced (a)  Parliamentary Convention
only by a minister. (b)  Parliamentary Legislation
3. After a money bill is passed by the Lok (c)  Constitutional provision
Sabha, it is transmitted to the Rajya Sabha (d)  Executive fiat
for its consideration. The Rajya Sabha has Solution: (c)
restricted powers with regard to a money
bill. It cannot reject or amend a money bill. Joint Sitting of Two Houses
It can only make the recommendations.
4. It must return the bill to the Lok Sabha Q.832 Which of the following type of bills can be
within 14 days, wither with or without passed in a joint sitting of the Parliament?
recommendations. The Lok Sabha can (a)  Constitutional amendment bill
either accept or reject all or any of the (b)  Money Bill
recommendations of the Rajya Sabha. (c)  Ordinary Bill
(d)  Any of the above
Q.830 What follows if a Money Bill passed by Lok
Solution: (c)
Sabha is recommended to be substantially
Learning: Any ordinary law needs to be
amended by the Rajya Sabha?
passed by both the Houses. But if there is a
(a) The bill stops there and does not become
difference between the two Houses, the final
an act.
decision is taken in a joint session in which
(b) The Lok Sabha may still proceed with
members of both the Houses sit together.
the Bill, accepting or rejecting the
Because of the larger number of members,
recommendations of the Rajya Sabha
the view of the Lok Sabha is likely to prevail
(c) The bill is sent to the President for a Joint
in such a meeting.
Sitting.
(d) The Lok Sabha may send the Bill to the Q.833 A joint sitting to resolve the Parliamentary
Rajya Sabha for reconsideration deadlock applies to which of the following
Solution: (b) types of bills apart from ordinary bills?

Indian Polity Question Bank P.221

02-Indian Polity_Q413-927.indd 221 8/7/2018 7:40:39 PM


1. Appropriation Bill as a part of Annual • It must be noted here that the provision
Budget of joint sitting is applicable to ordinary
2. Constitutional Amendment Bills bills or financial bills only and not
3. Financial Bills to money bills or Constitutional
Select the correct answer using the codes amendment bills.
below. • In the case of a money bill, the Lok
(a)  1 and 2 only (b)  2 and 3 only Sabha has overriding powers, while a
(c)  3 only (d)  1 only Constitutional amendment bill must be
Solution: (c) passed by each House separately.
Justification: Joint sitting cannot also be • The Speaker of Lok Sabha presides over
called in case of a money bill. a joint sitting of the two Houses and the
Statement 1: It is a money bill, and hence Deputy Speaker, in his absence.
joint sitting cannot be summoned. Q.835 In case of a Parliamentary deadlock, the
Statement 2: Each house has the same President summons a joint sitting of both
power with respect to a constitutional houses to resolve the issue. Who chairs the
amendment bill, and must pass it separately. joint sitting?
Statement 3: In case of a disagreement (a) President
between the two Houses over such a bill, the (b)  Chairman, Rajya Sabha
President can summon a joint sitting of the (c)  Speaker, Lok Sabha
two Houses to resolve the deadlock. (d)  Leader of the House of People
Q.834 Joint sitting is called by the President in Solution: (c)
which of the following situations? Q.836 If there is a difference between the two
1. If a bill passed on one house is rejected by Houses of Parliament on ordinary legislation,
the other House the matter
2. If more than six months have elapsed (a) must be referred to the President of India,
from the date of the receipt of the bill by whose opinion shall be final and binding
the other House (b) can be referred to the Supreme Court of
Which of the above is/are correct? India for its advisory jurisdiction
(a)  1 only (b)  2 only (c) will be referred to a joint sitting of the
(c)  Both 1 and 2 (d)  None Parliament convened by the President
Solution: (c) (d) will be settled through a Joint
Justification: It is extraordinary machinery Parliamentary Committee setup by both
provided by the Constitution to resolve a the houses
deadlock between the two Houses over the Solution: (c)
passage of a bill. Explanation and Learning: Any ordinary
A deadlock is deemed to have taken law needs to be passed by both the Houses.
place under any one of the following three But if there is a difference between the two
situations after a bill has been passed by one Houses, the final decision is taken in a joint
House and transmitted to the other House: session in which members of both the Houses
• if the bill is rejected by the other House; sit together. Because of the larger number
• if the Houses have finally disagreed as to of members, the view of the Lok Sabha is
the amendments to be made in the bill; or likely to prevail in such a meeting. However,
• if more than six months have elapsed Lok Sabha exercises more powers in money
from the date of the receipt of the bill by matters. Once the Lok Sabha passes the
the other House without the bill being budget of the government or any other money
passed by it. related law, the Rajya Sabha cannot reject it.
Learning: In the above three situations, The Rajya Sabha can only delay it by 14 days
the president can summon both the Houses or suggest changes in it. The Lok Sabha may
to meet in a joint sitting for the purpose of or may not accept these changes.
deliberating and voting on the bill.

P.222 For Civil Services Preliminary Examination

02-Indian Polity_Q413-927.indd 222 8/7/2018 7:40:39 PM


Budget in Parliament of the Parliament) immediately after the
presentation of the Union Budget, to give
Q.837 Which of these best characterizes the nature effect to the financial proposals of the
of the Annual Financial Statement or the Government of India for the immediately
Union Budget before it is presented to the following financial year.
Parliament for the first time in the year? It is through the Finance Act that
(a) It is a public document and the draft is amendments are made to the various
shared with major civil society bodies. Acts like Income Tax Act 1961, Customs
(b) It is a secret document until it is revealed Act 1962 etc.
first to the Parliament.
Q.839 If the annual Union Budget is not passed by
(c) It is disclosed to select corporate before
Lok Sabha,
unveiling to the Parliament.
(a) the speaker dissolves the Lok Sabha.
(d) It is circulated at least a month before to
(b) the Budget is sent to the Estimates
all individual MPs before being presented
Committee for reconsideration and
in the parliament on the budget day.
presented again to the house.
Solution: (b)
(c) the Prime Minister submits the
Learning: Budget is always kept as a secret,
resignation of his cabinet
otherwise those who have prior information
(d) any of the above depending on the
of the provisions can gain a lot from
direction of the Prime Minister.
information asymmetry.
Solution: (c)
For e.g. government may plan to build a
Justification: Since the government cannot
railway station in a remote area which would
function without funds in the financial year,
lead to shooting land prices; if some know
the Prime Minister will submit the resignation
this provision, they will invest beforehand
of the council of ministers.
and exploit the information asymmetry.
This is the stand taken by UPSC in its
When T. T. Krishnamachari was the
official keys, hence we would also stick with
Finance Minister, he actually had to resign
the stand.
because the budget was leaked before
The estimates committee does not have
presenting in the Parliament.
any role in the passing of Budget, so B is
Q.838 Which of these bills presented along with the anyways wrong; and since speaker can’t
Annual Financial Statement, provides details dissolve Lok Sabha. Thus the option (a) is
of the imposition, abolition, remission, also wrong.
alteration or regulation of taxes proposed in
Q.840 The budget goes through six stages in the
the Budget?
Parliament. Arrange the following in the
(a)  Appropriate Bill
correct order of proceeding in Parliament.
(b)  Medium Term Framework Bill
1. Presentation of budget.
(c)  Finance Bill
2. Scrutiny by departmental committees.
(d)  Budget Amendment Bill
3. General discussion.
Solution: (c)
4. Passing of finance bill.
Learning: While the Finance Bill generally
5. Passing of appropriation bill.
seeks approval of the Parliament for raising
6. Voting on demands for grants.
resources through taxes, cess etc., an
Select the correct order using the codes
Appropriation Bill seeks Parliament’s approval
below.
for the withdrawal from the Consolidated Fund
(a)  1 3 2 5 6 4 (b)  3 1 2 6 5 4
of India to meet the approved expenditures of
(c)  3 1 2 4 5 6 (d)  1 3 2 6 5 4
the Government. So, (a) is wrong.
Solution: (d)
The Medium Term Framework Bill does
Learning: Correct order is:
not exist as mentioned in option (b).
1. Presentation of budget.
Finance Bill is a secret bill introduced
2. General discussion.
every year in Lok Sabha (Lower chamber

Indian Polity Question Bank P.223

02-Indian Polity_Q413-927.indd 223 8/7/2018 7:40:39 PM


3. Scrutiny by departmental committees. (b) Excess grant awarded by the Parliament to
4. Voting on demands for grants. the executive that had been appropriated
5. Passing of appropriation bill. in the annual budget
6. Passing of finance bill. (c) Grant made before the ‘March Rush’ by
The general discussion on budget begins the Lok Sabha to the executive
a few days after its presentation. It takes (d) Grant sanctioned by Parliament to
place in both the Houses of Parliament and executive for meeting an unexpected
lasts usually for three to four days. After the demand upon the resources of India
general discussion on the budget is over, Solution: (a)
the Houses are adjourned for about three to Learning: The Appropriation Bill becomes
four weeks and 24 departmental standing the Appropriation Act after it is assented to
committees examine the budget. by the President.
After voting demand for grants, an This act authorises (or legalises) the
appropriation bill is introduced to provide payments from the Consolidated Fund of
for the appropriation, out of the Consolidated India. This means that the government cannot
Fund of India, all money required to meet: withdraw money from the Consolidated Fund
1. The grants voted by the Lok Sabha. of India till the enactment of the appropriation
2. The expenditure charged on the bill. This takes time and usually goes on till
Consolidated Fund of India the end of April.
Q.841 If the Rajya Sabha does not agree to the 1. But the government needs money to carry
provisions of the Annual Budget then on its normal activities after 31 March (the
(a) it returns the bill to the Lok Sabha end of the financial year).
with proposed amendments, which 2. To overcome this functional difficulty, the
then passes it or rejects the proposed Constitution has authorised the Lok Sabha
amendments, but the bill is not passed in to make any grant in advance in respect to
case of disagreement the estimated expenditure for a part of the
(b) it is sent to the President for financial year, pending the completion of
reconsideration and then transferred to the voting of the demands for grants and
Lok Sabha for consideration the enactment of the appropriation bill.
(c) it is sent to a joint select committee of 3. This provision is known as the ‘vote on
the Parliament for re-examination and re- account’. It is passed (or granted) after
introduction in Lok Sabha the general discussion on budget is over.
(d)  None of the above 4. It is generally granted for two months for
Solution: (d) an amount equivalent to one-sixth of the
Annual budget is a money bill. total estimation.
A Money Bill can be introduced only in Q.843 What is the difference between “vote-on-
the Lok Sabha and not in the Rajya Sabha. account” and “interim budget”?
Rajya Sabha cannot amend or reject a Money 1. The provision of a “vote-on-account” is
Bill. It should return the bill to the Lok Sabha used by a regular Government, while an
within 14 days, either with recommendations “interim budget” is a provision used by a
or without recommendations. The Lok Sabha caretaker Government.
can either accept or reject all or any of the 2. A “vote-on-account” only deals with
recommendations of the Rajya Sabha. In the expenditure in Government’s budget,
both the cases, the money bill is deemed to while an “interim budget” includes both
have been passed by the two Houses. expenditure and receipts.
Q.842 ‘Vote-on-account’ provision is related to Which of the statements given above is/are
(a) Any grant made in advance by the Lok correct?
Sabha to the executive before the passing (a)  1 only (b)  2 only
of appropriation bill (c)  Both 1 and 2 (d)  Neither 1 nor 2
Solution: (b)

P.224 For Civil Services Preliminary Examination

02-Indian Polity_Q413-927.indd 224 8/7/2018 7:40:39 PM


Interim budget is a temporary financial (c) Administer international treaties and
document that helps a business or public covenants that cannot be ordinarily stated
agency get through a period that’s shorter in advance in the budget
than a typical budget cycle, which is normally (d) Improve the efficiency of governance
one year. and mass marketing campaigns
An Interim Budget is not the same as a Solution: (a)
‘Vote on Account’. While a ‘Vote on Account’ Learning: Option B is vote on account
deals only with the expenditure side of the by which Government obtains the Vote of
government’s budget, an Interim Budget is Parliament for a sum sufficient to incur
a complete set of accounts, including both expenditure on various items for a part of the
expenditure and receipts. An Interim Budget year. Normally, the Vote on Account is taken
gives the complete financial statement, very for two months only.
similar to a full Budget. Vote on Credit is like a blank cheque
Interim Budgets also can be presented by Lok Sabha to government of India. It
by all governments whether incumbent or is granted when an unexpected demand
regular or caretaker has arisen upon resources of India whose
Q.844 With reference to cut motions in the magnitude or character of service cannot
Parliament, consider the following: be stated clearly. For example, in case of
1. A departmental standing committee that a rescue operation, the cost to be incurred
examines the Budget can suggest and cannot be described clearly.
examine cut motions. Q.846 The ‘Vote-of-Credit’ is given by the
2. Amendment of existing laws cannot be Parliament to the executive when
suggested by a cut motion. (a) Additional expenditure upon some new
3. Cut motion cannot be raised on service not contemplated in the budget
expenditure charged on the Consolidated for that year is sought
Fund of India. (b) Money has been spent on any service
Select the correct answer using the codes during a financial year in excess of the
below. amount granted for that service in the
(a)  2 and 3 only (b)  1 only budget for that year
(c)  1 and 3 only (d)  2 only (c) Meeting an unexpected demand upon
Solution: (a) the resources of India which cannot be
Justification: Statement 1: Such a standing stated with the details ordinarily given in
committee can only consider the demands a budget
for grants of the concerned ministries / (d) Executive seeks advance appropriation
departments before they are discussed and before the examination and enactment of
voted in the Lok Sabha. Its report should not the budget to meet current expenditures
suggest anything of the nature of cut motions. Solution: (c)
Statement 2: A cut-motion cannot raise Justification and Learning: The above
policy matters. It can only suggest reduction options refer to different kinds of grants that
of unnecessary expenditures. are given to executive apart from the normal
Statement 3: These expenditures are not annual budget.
to be voted upon by the Parliament, hence Additional Grant: It is granted when a need
exempt from cut motions. has arisen during the current financial year for
Q.845 A ‘vote-of-credit’ is granted by the additional expenditure upon some new service
Parliament to the executive to not contemplated in the budget for that year.
(a) Incur expenditures that cannot be clearly Excess Grant: It is granted when money has
stated or explained been spent on any service during a financial
(b) Sanction money for spending during the year in excess of the amount granted for that
first part of the financial year when the service in the budget for that year. It is voted
budget has not been approved by the Lok Sabha after the financial year.

Indian Polity Question Bank P.225

02-Indian Polity_Q413-927.indd 225 8/7/2018 7:40:39 PM


Vote-of-Credit: It is granted for meeting Fund of India and expenditure made from
an unexpected demand upon the resources the Consolidated Fund of India.
of India, when on account of the magnitude 2. The budget shall compulsorily distinguish
or the indefinite character of the service, the expenditure on revenue account from
demand cannot be stated with the details other expenditure to better reflect
ordinarily given in a budget. Hence, it is like allocation of public money.
a blank cheque given to the Executive by Which of the above is/are correct?
the Lok Sabha. (a)  1 only (b)  2 only
Exceptional Grant: It is granted for a (c)  Both 1 and 2 (d)  None
special purpose and forms no part of the Solution: (c)
current service of any financial year. Learning: These are some of the basic
Q.847 The Union Cabinet has approved the proposal principles followed in the budget as also
of Ministry of Finance to merge the Railway mentioned in the constitution.
budget with the General budget. How this Some other such provisions are:
decision will impact Indian Railways (IR)? • No demand for a grant shall be made
1. IR will cease to be a departmentally run except on the recommendation of the
commercial undertaking. President.
2. It will save Railways from the liability of • No money shall be withdrawn from the
payment of annual dividend to the Central Consolidated Fund of India except under
government. appropriation made by law.
3. The Ministry of Railways will stop • No money bill imposing tax shall be
receiving Gross Budgetary Support introduced in the Parliament except on
(GBS) from the government. the recommendation of the President, and
Select the correct answer using the codes such a bill shall not be introduced in the
below. Rajya Sabha.
(a)  1 and 3 only (b)  1 and 2 only • No tax shall be levied or collected except
(c)  2 only (d)  None of the above by authority of law.
Solution: (c)
Justification: The merger has been approved Funds
with the following administrative and
Q.849 All money received by or on behalf of the
financial arrangements of Railways
Government of India are credited to
• It will continue as a departmentally run
(a)  the Consolidated Fund of India
commercial undertaking to maintain its
(b)  the Public Account of India’
distinct entity as at present. So, 1 is wrong.
(c)  the Contingency Fund of India
• It will retain its functional autonomy and
(d)  Either (a) or (b)
delegation of financial powers etc. as per
Solution: (d)
the existing guidelines.
• Its existing financial arrangements will
continue wherein it will meet the revenue (1) Consolidated Fund of India
expenditure from revenue receipts. Hence Q.850 The expenditure ‘made’ from the
it will continue to receive the budgetary Consolidated Fund of India CANNOT be
support. 1. Discussed and voted by the Parliament
But Railways will be relieved from the 2. Considered as a part of the appropriation
liability of payment of annual dividend to the bill
Central government Which of the above is/are correct?
Q.848 Consider the following statements. (a)  1 only (b)  2 only
1. The estimates of expenditure embodied (c)  Both 1 and 2 (d)  None
in the budget shall show separately Solution: (d)
expenditure charged on the Consolidated Justification: The budget consists of two
types of expenditure—the expenditure

P.226 For Civil Services Preliminary Examination

02-Indian Polity_Q413-927.indd 226 8/7/2018 7:40:39 PM


‘charged’ upon the Consolidated Fund of Justification: Statements 1and 2: The budget
India and the expenditure ‘made’ from the consists of two types of expenditure—the
Consolidated Fund of India. expenditure ‘charged’ upon the Consolidated
The charged expenditure is non-votable Fund of India and the expenditure ‘made’
by the Parliament, that is, it can only be from the Consolidated Fund of India. The
discussed by the Parliament, while the other charged expenditure is non-votable by the
type has to be voted by the Parliament. Parliament, that is, it can only be discussed
The latter forms part of the appropriation by the Parliament, while the other type has to
bill, and voted by the Parliament, for example, be voted by the Parliament.
defence expenditure. Statement 3: The final bill containing all
Any withdrawal of money from the such expenditures must receive Presidential
consolidated fund of India by the government assent to become valid.
has to go through the same procedure as that Q.852 How is Judicial Independence secured by
of a charging the salaries and allowances of the
(a)  regular government budget Judges of the High Court to the Consolidated
(b) regular government budget only in case Fund of the State (CFS)?
of major expenditures that exceed the (a) Charged expenditure is not a part of
annual budget public money.
(c) ordinary withdrawal from the Public (b) Charged expenditure is non-votable by
Account of India the legislature.
(d) withdrawal from the Public Account of (c) Charged expenditure is operated by
India in emergency cases executive action.
Solution: (a) (d) Any charged expenditure can be allocated
The Constitution lays down three conditions only after obtaining Judicial consent.
without fulfilling which “No money out Solution: (b)
of the Consolidated Fund of India shall Justification and Learning: Since the
be appropriated” or withdrawn. These expenditure is non-votable, the legislature
conditions are as follows. cannot alter the judge’s salary at will
• The appropriation must be in accordance and interfere in judicial independence
with a law authorising appropriation of unnecessarily.
money from the Consolidated Fund; and The Article 202 deals with the state budget.
• The appropriation should be for a “purpose Article 202(3) says:
provided in this Constitution”; and The following expenditure shall be
• The appropriation must be made “in the expenditure charged on the Consolidated
manner provided in this Constitution” as Fund of each State—
the manner for appropriation of money (a) the emoluments and allowances of the
from the Consolidated Fund of India Governor and other expenditure relating
(which is that of a regular budget) to his office;
Q.851 The expenditure ‘charged’ upon the (b) the salaries and allowances of the
Consolidated Fund of India Speaker and the Deputy Speaker of the
1. Cannot be discussed by Parliament Legislative Assembly and, in the case
2. Is non-votable by the Rajya Sabha, but of State having a Legislative Council,
votable by Lok Sabha also of the Chairman and the Deputy
3. Does not require Presidential assent Chairman of the Legislative Council;
Select the correct answer using the codes (c) debt charges for which the State is
below. liable including interest, sinking fund
(a)  1 and 2 only (b)  1 only charges and redemption charges, and
(c)  2 and 3 only (d)  None of the above other expenditure relating to the raising
Solution: (d) of loans and the service and redemption
of debt;

Indian Polity Question Bank P.227

02-Indian Polity_Q413-927.indd 227 8/7/2018 7:40:39 PM


(d) expenditure in respect of the salaries and Select the correct answer using the codes
allowances of Judges of any High Court; below.
(e) any sums required to satisfy and (a)  3 only (b)  1 and 3 only
judgment, decree or award of any court (c)  2 and 3 only (d)  1 and 2 only
or arbitral tribunal; Solution: (b)
(f) any other expenditure declared by this Justification: Statement 2: The salaries
Constitution, or by the Legislature of the and allowances of the judges, the salaries,
State by law, to be so charged. allowances and pensions of the staff as well as
(Please note that here we are covering only the administrative expenses of a high court are
major expenditure items apart from HC charged on the consolidated fund of the state.
Judges.) Thus, they are non-votable by the state
However please note that the retired legislature (though they can be discussed by
Judges are entitled to a pension which is it). It should be noted here that the pension
drawn from the consolidated fund of India. of a high court judge is charged on the
Q.853 Which of the following is included in the list Consolidated Fund of India and not the state.
of charged expenditure? Learning: Other expenditures are:
1. Emoluments and allowances of the • Salaries and allowances of the Chairman
President and Vice-President and the Deputy Chairman of the Rajya
2. Salaries of Speaker and the Deputy Sabha and the Speaker and the Deputy
Speaker of the Lok Sabha Speaker of the Lok Sabha; Members and
3. Administrative expenses of the Supreme Chairman UPSC, CAG, judges of SC
Court • Administrative expenses of the Supreme
Select the correct answer using the codes Court, the office of the Comptroller and
below. Auditor General of India and the Union
(a)  1 and 2 only (b)  2 and 3 only Public Service Commission including the
(c)  1 and 3 only (d)  1, 2 and 3 salaries, allowances and pensions of the
Solution: (b) persons serving in these offices.
Justification: Salary and allowances of only • The debt charges for which the
President is included, not that of the Vice- Government of India is liable, including
President. So, 1 is incorrect. interest, sinking fund charges and
Other important items included are: redemption charges and other expenditure
1. The salaries, allowances and pensions relating to the raising of loans and the
payable to or in respect of Judges of the service and redemption of debt.
Supreme Court, • Any sum required to satisfy any
2. The salary, allowances and pension judgement, decree or award of any court
payable to or in respect of the Comptroller or arbitral tribunal.
and Auditor General of India;
3. Any sums required to satisfy any (2) Contingency Fund of India
judgment, decree or award of any court or
Q.855 The Contingency Fund of India
arbitral tribunal;
1. Has been established by an Act of the
Q.854 Which of following expenditure is/are Parliament
charged on the Consolidated Fund of India? 2. Is placed at the disposal of the President
1. Emoluments and allowances of the operated by executive action
President 3. Cannot be withdrawn except on prior
2. Salaries and allowances of the judges of authorization from the Parliament
the Supreme Court and High courts Select the correct answer using the codes
3. Administrative expenses of the office below.
of the Comptroller and Auditor General (a)  1 and 2 only (b)  2 only
of India (c)  3 only (d)  1 and 3 only
Solution: (a)

P.228 For Civil Services Preliminary Examination

02-Indian Polity_Q413-927.indd 228 8/7/2018 7:40:39 PM


Justification: The Constitution authorised the This includes provident fund deposits,
Parliament to establish a ‘Contingency Fund judicial deposits, savings bank deposits,
of India’, into which amounts determined departmental deposits, remittances and so on.
by law are paid from time to time. So, the This account is operated by executive action,
option 1 is correct. that is, the payments from this account can by
• Accordingly, the Parliament enacted the made without parliamentary appropriation.
contingency fund of India Act in 1950. Such payments are mostly in the nature of
So, 2 is correct. banking transactions.
• This fund is placed at the disposal of the Q.858 Which of the following statements with
president, and he can make advances out of reference to the Public Account of India is
it to meet unforeseen expenditure pending CORRECT?
its authorisation by the Parliament. So, 3 (a)  It is operated by executive action.
is incorrect. (b) All taxes collected by the Government of
• The fund is held by the finance secretary India are submitted to this account.
on behalf of the president. (c) It is used to fund contingencies and
• Like the public account of India, it is also disaster management.
operated by executive action.’ (d)  It is an extra-constitutional fund.
Q.856 Consider the following about Contingency Solution: (a)
Fund of India. Justification and Learning: All other public
1. The fund is held by the finance secretary money (other than those which are credited to
on behalf of the president. the Consolidated Fund of India)  received by
2. Parliamentary approval is not required or on behalf of the Government of India shall
in utilizing the fund as it falls outside the be credited to the Public Account of India.
ambit of Parliament. This includes provident fund deposits,
Which of the above is/are correct? judicial deposits, savings bank deposits,
(a)  1 only (b)  2 only departmental deposits, remittances and so on.
(c)  Both 1 and 2 (d)  None This account is operated by executive action,
Solution: (a) that is, the payments from this account can be
made without parliamentary appropriation.
(3) Public Account of India Such payments are mostly in the nature of
banking transactions.
Q.857 Consider the following statements with
Q.859 Consider the following statements.
reference to the Public Account of India.
1. The constitution establishes Consolidated
Assertion (A): This account is operated by
Fund of the Union as well as Consolidated
executive action
Fund of the States.
Reason (R): All capital receipts of the
2. There is no mention of Public Accounts
government are credited to this account.
of India and Contingency Fund in the
In the context of the above, which of these
constitution as they are established by
is correct?
legislation.
(a) A is correct, and R is an appropriate
Which of the above is/are correct?
explanation of A.
(a)  1 only (b)  2 only
(b) A is correct, but R is not an appropriate
(c)  Both 1 and 2 (d)  None
explanation of A.
Solution: (a)
(c)  A is correct, but R is incorrect.
Justification: Statement 1: Article 266
(d)  Both A and R are incorrect.
provides for a Consolidated Fund of India
Solution: (c)
in which the net proceeds of certain taxes
Justification: All other public money
and duties to States, all revenues received
(other than those which are credited to the
by the Government of India, all loans raised
Consolidated Fund of India) received by or
by that Government by the issue of treasury
on behalf of the Government of India shall be
bills etc. would come. It also provides for
credited to the Public Account of India.

Indian Polity Question Bank P.229

02-Indian Polity_Q413-927.indd 229 8/7/2018 7:40:39 PM


a Consolidated Fund of the States. So, 1 is lapses and cannot be presented in a joint
correct. sitting of the houses.
Statement 2: The same A266 provides Statement 4: Both houses can approve this
that “All other public moneys received by proclamation. Lok Sabha can also revoke an
or on behalf of the Government of India or emergency.
the Governmentn of a State shall be credited Statement 5: In case of SC, this is done by
to the public account of India or the public the President on the advice of a collegiums
account of the State, as the case may be.” So, consisting of the CJI and some senior-
clearly the option 2 is wrong. most judges of the High Court. Lok Sabha,
Article 267 says that “Parliament may however, takes part in the impeachment
by law establish a Contingency Fund to be of judiciary.
entitled “the Contingency Fund of India” Q.861 The Parliament exercises control over the
into which shall be paid from time to time Government using which of the following
such sums as may be determined by such devices?
law, and the said Fund shall be placed at the 1. Question Hour
disposal of the President to enable advances 2. Adjournment motion
to be made by him out of such Fund for the 3. Discussions
purposes of meeting unforeseen expenditure 4. Parliamentary committees
pending authorisation of such expenditure Choose the correct answer using the codes
by Parliament by law under article 115 or given below.
article 116. (a)  All of the above (b)  1, 2 and 3 only
(c)  1 and 2 only (d)  3 and 4 only
Multifunctional Role of Parliament Solution: (a)
The greatest advantage of the parliamentary
Q.860 Which of the following functions are
system is that it ensures harmonious
performed by the Lok Sabha?
relationship and cooperation between the
1. Approval of proposals for taxation
legislative and executive organs of the
2. Appointment of Chairman of Council of
government. The executive is a part of the
States
legislature and both are interdependent at
3. Approving a constitutional amendment
work. As a result, there is less scope for
bill
disputes and conflicts between the two organs.
4. Approving the Proclamation of emergency
By its very nature, the parliamentary
5. Appointment of members of higher
system establishes a responsible government.
judiciary
The ministers are responsible to the
Select the correct answer using the codes
Parliament for all their acts of omission and
below.
commission. The Parliament exercises control
(a)  1, 3 and 4 only (b)  1, 2 and 5 only
over the ministers through various devices
(c)  2, 3, 4 and 5 only (d)  1, 2, 3, 4 and 5
like question hour, discussions, adjournment
Solution: (a)
motion, no confidence motion, etc.
Justification: Statement 1: Such matters
come under money bills. Money bills can Q.862 Indian Constitution has adopted a system of
only be introduced in LS, not RS. Checks and Balance for smooth functioning
Statement 2: This is done by Rajya Sabha. of the three organs of the government.
If the question had asked the Vice-President Which of the following are the instruments
(VP), you should have chosen this statement of parliamentary control over the executive?
as correct. Lok Sabha serves as the collegiums 1. No Confidence Motion
for electing the VP. Moreover, appointment is 2. Both Zero Hour and Half-an- hour
not a correct term in this context; it should discussion
be election. 3. Comptroller and Auditor General
Statement 3: Both houses can do so. A bill 4. Amendment Provisions
for this purpose, if defeated by any house, 5. Public Accounts Committee

P.230 For Civil Services Preliminary Examination

02-Indian Polity_Q413-927.indd 230 8/7/2018 7:40:39 PM


6. Emergency provisions Solution: (d)
Select the correct code. In all democracies, an assembly of elected
(a)  1, 3, 4 and 5 Only representatives exercises supreme political
(b)  1, 2, 3 and 5 Only authority on behalf of the people. In
(c)  1, 2, 4, 5 and 6 Only India such a national assembly of elected
(d)  1, 2, 3,4, 5 and 6 representatives is called Parliament.
Solution: (b) At the state level this is called Legislature
Instruments of Parliamentary Control or Legislative Assembly.
The legislature in parliamentary system The name may vary in different countries,
ensures executive accountability at various but such an assembly exists in every
stages: policy making, implementation of law democracy. It exercises political authority
or policy and during and post implementation on behalf of the people in many ways as
stage. The legislature does this through the mentioned below.
uses of a variety of devices as described below. 1. Parliament is the final authority for
1. Deliberation and discussion: Zero Hour making laws in any country. This task of
where members are free to raise any law making or legislation is so crucial that
matter that they think is important (though these assemblies are called legislatures.
the ministers are not bound to reply), the Parliaments all over the world can make
discussion for half-an-hour on matters of new laws, change existing laws, or
public importance, adjournment motion etc. abolish existing laws and make new ones
are some instruments of exercising control. in their place.
2. Approval or Refusal of laws. 2. Parliaments all over the world exercise
3. Financial control: before granting money some control over those who run the
the Lok Sabha can discuss the reasons for government. In some countries like India
which the government requires money. this control is direct and full. Those who
It can enquire into cases of misuse of run the government can take decisions
funds on the basis of the report of the only so long as they enjoy support of
Comptroller and Auditor General and the Parliament.
Public Accounts committees. 3. Parliaments control all the money that
4. No confidence motion: The most powerful governments have. In most countries any
weapon that enables the Parliament to the public money can be spent only when
ensure executive accountability is the the Parliament sanctions it.
no-confidence motion. As long as the 4. Parliament is the highest forum of
government has the support of its party or discussion and debate on public issues
coalition of parties that have a majority in and national policy in any country.
the Lok Sabha, the power of the House to Parliament can seek information about
dismiss the government is fictional rather any matter.
than real. Q.864 Which of the following is/are the Constitutional
Q.863 Consider the following statements about the functions of the Indian Parliament?
power of the Indian Parliament. 1. Supervising the work of State Legislatures
1. Parliament can seek information about 2. Settling disputes between various
any matter from the government. government departments
2. Any public money can only be spent if the 3. Appointing the heads of all Constitutional
Parliament approves it. bodies
3. The Parliament keeps the government Select the correct answer using the codes
under control and accountable to itself. below.
Choose the correct answer using the codes (a)  1 and 2 only (b)  2 and 3 only

below: (c)  1 and 3 only (d)  None of the above
(a)  1 and 2 only (b)  2 and 3 only Solution: (d)
(c)  1 and 3 only (d)  All of the above

Indian Polity Question Bank P.231

02-Indian Polity_Q413-927.indd 231 8/7/2018 7:40:39 PM


Justification: Statement 1: No body Solution: (c)
supervises the work of state legislatures. At Justification: The CAG submits three
most, the Parliament can interfere in state audit reports to the President—audit report
legislative matters in special circumstances on appropriation accounts, audit report on
such as national emergency, Rajya Sabha finance accounts, and audit report on public
resolution etc. undertakings.
Statement 2: This is the responsibility of The President lays these reports before
the Cabinet and more importantly the Prime both the Houses of Parliament. After this, the
Minister. Public Accounts Committee examines them
Statement 3: Heads of constitutional and reports its findings to the Parliament.
bodies are appointed by the Presid. The appropriation accounts compare
Q.865 Arrange these financial accountability the actual expenditure with the expenditure
processes in chronological order (i.e. from sanctioned by the Parliament through the
start to end) in a financial year. Appropriation Act, while the finance accounts
1. Detailed Budget allocations by the show the annual receipts and disbursements
Ministry of Finance of the Union government.
2. The Comptroller and Auditor General of Q.866 Consider the following statements.
India (CAG) submits audit report to the 1. If the Government of India proposes to
President introduce any new tax, it must get the
3. Public Accounts Committee submits approval of the Lok Sabha.
report to Parliament 2. The government must give an account
Choose the correct order. about its receipts and expenditures to the
(a) 123 (b) 321 Legislature.
(c) 132 (d) 312 Which of the above is/are correct?
Solution: (a) (a)  1 only (b)  2 only
The CAG’s reports are submitted at the end of (c)  Both 1 and 2 (d)  None
every financial year. Then the PAC receives Solution: (c)
CAG’s reports, studies it; and sends its own Justification: The Financial powers of the
report to the Parliament. Parliament, involve grant of resources to the
Therefore, the first activity in a financial government to implement its programmes.
year is the examination of the budget by the The government has to give an account to
departmental standing committees after it has the Legislature about the money it has spent
been presented and budget allocated by the and resources that it wishes to raise. The
Ministry of Finance. legislature also ensures that the government
Consider the following with reference to does not misspend or overspend. This is
ensuring the financial accountability of the done through the budget and annual financial
legislature to the executive. statements.
1. The CAG submits audit reports to the For example, the government must get
President. a money bill passed in the Parliament to
2. The President lays CAG reports before appropriate money from the Consolidated
the Public Accounts Committee. Fund of India.
3. Public Accounts Committee examines Q.867 The Parliament exercises Budgetary control
CAG reports submitted to Parliament. over the executive, that is, control ‘before’
4. Public Accounts Committee submits the the appropriation of grants through the
audit report of CAG to the President after enactment of the budget by
due examination. (a)  Estimate Committee
Which of the above is/are the correct (b) Departmental Standing Committees
procedure(s) in practice? (c)  Public Accounts Committee
(a)  1 only (b)  2 and 4 only (d)  Committee on Public Undertaking
(c)  1 and 3 only (d)  1, 3 and 4 only Solution: (b)

P.232 For Civil Services Preliminary Examination

02-Indian Polity_Q413-927.indd 232 8/7/2018 7:40:39 PM


Learning: The Parliamentary control over Q.869 Which of these authorities/bodies authorizes
the Executive in financial matters operates in withdrawal of any funds from the
two stages: Consolidated Fund of India?
• Budgetary control, that is, control before (a)  President of India
the appropriation of grants through the (b)  Parliament of India
enactment of the budget; and (c)  Prime Minister of India
• Post-budgetary control, that is, control (d)  Union Finance Minister
after the appropriation of grants through Solution: (b)
the three financial committees–Public Learning: Not even a single penny can
Accounts Committee (PAC), Estimate be withdrawn without the approval of the
Committee and Committee on Public parliament.
Undertakings (CoPU). This includes not only the annual budget,
Option (b): The Departmental standing but also any additional expenditure required
committees consider the demands for by the executive during the financial year.
grants of the concerned ministries / Q.870 Which of these tend to erode the authority
departments before they are discussed and and influence of the Indian Parliament over
voted in the Lok Sabha. It also considers the executive?
the annual reports of the Ministries and 1. Increasing delegated legislation which is
departments. often technical in nature
Option (c): The function of the PAC is to 2. Low participation in the Parliament with
examine the annual audit reports of the CAG, respect attendance and discussions
which are laid before the Parliament by the 3. Frequent promulgation of ordinances by
president. This is a kind of post-mortem of the Judiciary
the executive expenditures. 4. Unilateral amendment of the Constitution
Q.868 Consider the following with reference to by the executive
Parliamentary control over the Budget. Select the correct answer using the codes
1. Departmental Standing committees have below.
the responsibility to draft and create the (a)  3 and 4 only (b)  1 and 2 only
annual budget. (c)  1, 2 and 4 only (d)  2, 3 and 4 only
2. Estimates Committee can impose cuts Solution: (b)
on the budget if the government cannot Justification: Statement 1: These are the
justify expenditure on a particular head. rules and bye laws made within laws by the
3. No money from the Consolidated Fund executive. Since all these are technical, MPs
of India can be withdrawn without the may lack the expertise to understand them
consent of the Parliament. and scrutinize the executive.
Select the correct answer using the codes Statement 3: Ordinances are promulgated
below. by the executive, not the Judiciary.
(a)  3 only (b)  1 and 2 only Statement 4: It is done by the Parliament
(c)  1 and 3 only (d)  2 only itself (not executive), however, frequent
Solution: (a) amendments lead to dilution in the authority
Justification: Statement 1: It is done by of Parliament.
Department of Economic Affairs. Q.871 Administration is held accountable to the
Statement 2: Estimates Committee only legislature by which of the following devices?
examines the economic efficiency and 1. Financial Committees of the Parliament
rationale behind government expenditures 2. Committee on delegated legislation
that too after the budget has been enacted. It 3. Committee on Business advisory
has no right to suggest cuts. 4. Question hour and zero hour in the
Statement 3: A bill needs to be passed by Parliament
the government for any such withdrawal.

Indian Polity Question Bank P.233

02-Indian Polity_Q413-927.indd 233 8/7/2018 7:40:39 PM


Choose the correct answer using the codes 2. A private member of Rajya Sabha can get
below. alterations passed to legislation introduced
(a)  1 and 3 only (b)  1, 2 and 4 only in the Lok Sabha.
(c)  2 and 4 only (d)  All of the above Which of the above is/are correct?
Solution: (b) (a)  1 only
Financial committees of Estimates, (b)  2 only
PSUs, and Public Accounts keep track of (c)  Both 1 and 2 only
expenditure, its usefulness and performance (d) None
of the administrative spending. Some laws Solution: (b)
need to be filled with administrative details Explanation: A private member is an MP
by the instrument of delegated legislation. who is not a minister. He enjoys similar rights
The committee on delegated legislation to that of a minister in Parliament, except that
scrutinizes such rules and regulations formed he cannot introduce money bills and some
by the executive or bureaucracy. Zero hour financial bills. So, the Statement 1 is wrong
and question hour are used to raise any matter as a private member can initiate legislation,
in Lok Sabha. The administration can be held even on a subject of national importance.
accountable. Statement 2 follows from the above. If a
Parliament exercises control over the private member can introduce legislation, he
administration because can also get alternations passed to them.
(a) the executive is collectively responsible Learning: The Rajya Sabha functions
to the legislature primarily as the representative of the states of
(b) it is the constitutional mandate of the India in the Parliament. The Rajya Sabha can
representatives to command and control also initiate legislation and a bill is required
administrative affairs to pass through the Rajya Sabha in order to
(c) executive branch of the government is become a law. It, therefore, has an important
subordinate to the legislative branch of role of reviewing and altering (if alterations
the government are needed) the laws initiated by the Lok
(d) administration is not directly controlled Sabha.
by the executive. Q.873 Consider the following statements:
Solution: (a) 1. The Rajya Sabha cannot initiate
None of the options is direct. You can work legislation, but a bill in order to become
out by elimination. a law is required to be passed through the
Option B is false because the Rajya Sabha.
representatives are not supposed to command 2. The members of the Rajya Sabha are
administration. nominated by the elected members of the
Option C is wrong because no branch of Legislative Assemblies of various states.
the government is subordinate to any other Select the correct code:
branch. (a)  1 Only
Option D is wrong because administration (b)  2 Only
is directly controlled by Ministers as head of (c)  Both 1 and 2
Ministries and departments. Administration (d)  Neither 1 nor 2
is a part of the executive branch. Hence, Solution: (d)
Option A. The Rajya Sabha can also initiate
legislation. The members of the Rajya
Position of Rajya Sabha Sabha are elected (not nominated) by
the elected members of the Legislative
Q.872 Consider the following statements.
Assemblies of various states.
1. A private member in Rajya Sabha cannot
initiate legislation on a subject of national Q.874 India has adopted bi-cameral legislature,
importance. wherein the Rajya Sabha forms the upper
House and Lok Sabha forms the lower

P.234 For Civil Services Preliminary Examination

02-Indian Polity_Q413-927.indd 234 8/7/2018 7:40:40 PM


House. Consider the following with regard to 3. Discussing and criticising the policies of
Rajya Sabha: the government
1. The number of members to be elected 4. Ascertaining the confidence of the
from each State has been fixed by the government on the house floor
second schedule of the Constitution. Select the correct answer using the codes
2. It is an indirectly elected body. below.
3. Members of the Rajya Sabha are elected (a)  2 and 3 only (b)  1, 3 and 4 only
for a term of six years and they are also (c)  3 only (d)  1 and 4 only
eligible for re-election. Solution: (c)
4. The Rajya Sabha is never fully dissolved; Justification: Statement 1: A Money Bill can
hence it is called a permanent body. be introduced only in the Lok Sabha and not
Which of the above statements is/are correct? in the Rajya Sabha.
(a)  1 and 2 Only (b)  2, 3 and 4 Only Rajya Sabha cannot amend or reject
(c)  2 and 3 Only (d)  1, 2, 3 and 4 Only a Money Bill. It should return the bill
Solution: (b) to the Lok Sabha within 14 days, either
Each of the two Houses of the Parliament has with recommendations or without
different bases of representation. The Rajya recommendations.
Sabha represents the States of India. It is an Statement 2: Rajya Sabha can only discuss
indirectly elected body. Residents of the State the budget but cannot vote on the demands
elect members to State Legislative Assembly. for grants (which is the exclusive privilege of
The elected members of State Legislative the Lok Sabha).
Assembly in turn elect the members of Rajya Statements 3 and 4: Moreover, the Rajya
Sabha. Sabha cannot remove the council of ministers
The number of members to be elected by passing a no-confidence motion.
from each State has been fixed by the fourth • This is because the Council of ministers
schedule of the Constitution. is collectively responsible only to the
Members of the Rajya Sabha are elected for Lok Sabha.
a term of six years. They can get re-elected. • But, the Rajya Sabha can discuss and
All members of the Rajya Sabha do not criticise the policies and activities of the
complete their terms at the same time. Every government.
two years, one third members of the Rajya Q.877 In legislative matters, Rajya Sabha enjoys an
Sabha complete their term and elections are equal Status with Lok Sabha in
held for those one third seats only. Thus, 1. Passing money bills
the Rajya Sabha is never fully dissolved. 2. Introduction and passage of Constitutional
Therefore, it is called the permanent House amendment bills
of the Parliament. 3. Discussing the annual budget
Q.875 One feature distinguishing the Rajya Sabha 4. Approval of ordinances issued by the
from the Vidhan Parlshad is : President
(a)  power of impeachment Select the correct answer using the codes
(b)  nomination of members below.
(c)  tenure of membership (a)  1 and 4 only (b)  2, 3 and 4 only
(d)  indirect election (c)  2 and 3 only (d)  1, 2, 3 and 4
Solution: (d) Solution: (b)
Justification: Some other matters where the
(1) Equal Power with Lok Sabha powers and status of the Rajya Sabha are
equal to that of the Lok Sabha:
Q.876 In which of these cases the Rajya Sabha has • Introduction and passage of financial
an equal Status with Lok Sabha? bills involving expenditure from the
1. Amending or rejecting a Money Bill Consolidated Fund of India.
2. Voting on the demands for grants • Election and impeachment of the president.

Indian Polity Question Bank P.235

02-Indian Polity_Q413-927.indd 235 8/7/2018 7:40:40 PM


• Election and removal of the Vice-President. Q.879 In which of the following cases are the
However, Rajya Sabha alone can initiate powers of the Lok Sabha and Rajya Sabha
the removal of the vice-president. co-equal?
• Making recommendation to the President 1. Election and impeachment of the President
for the removal of Chief Justice and judges 2. Approval of ordinances issued by the
of Supreme Court and high courts, chief President
election commissioner and comptroller 3. Removal of Judges of Supreme Court and
and auditor general. High Court
• Approval of proclamation of all three 4. Approval of proclamation of all three
types of emergencies by the President. types of emergencies Select the correct
Q.878 Lok Sabha and Rajya Sabha both have equal code.
powers with respect to: (a)  2 and 4 Only (b)  1 and 3 Only
1. Legislation of Ordinary bills. (c)  1, 2 and 3 Only (d)  1, 2, 3 and 4 Only.
2. Collective responsibility of the central Solution: (d)
executive. In all other spheres, including passing
3. Legislation related to financial matters. of non-money bills, constitutional
Choose the correct answer using the codes amendments, and impeaching the President
below: and removing the Vice Presidents, the
(a)  Only 1 (b)  1 and 3 only powers of Lok Sabha and Rajya Sabha are
(c)  1 and 2 only (d)  Only co-equal. Both of them participate in the
Solution: (b) election and removal of the President, Vice
The Council of Ministers is collectively President, Judges of Supreme Court and
responsible to Lok Sabha only. This means High Courts.
Rajya Sabha cannot pass a no-confidence Q.880 Consider a comparison of the powers of the
motion and remove the Government. Lok Sabha and Rajya Sabha.
It can, however, exercise control over the 1. Both houses can pass a constitutional
Government and this function becomes quite amendment bill, but the bill must be first
evident, particularly when the Government introduced in the Lok Sabha.
does not enjoy majority in Rajya Sabha. For 2. Both houses approve the proclamation of
e.g. in passing ordinary and some financial emergency, but an emergency cannot be
bills like in the recent case of the TRAI bill. revoked by any of the houses.
In case of an ordinary legislation, to Which of the above is/are correct?
resolve a deadlock between the two Houses, (a)  1 only (b)  2 only
the Constitution provides for the joint sitting (c)  Both 1 and 2 (d)  None
of both Houses. Issues in joint sitting are Solution: (d)
decided by a majority of the total number Justification: Statement 1: The bill can be
of members of both Houses present and introduced in any house. However it must
voting. The joint sitting is presided over by be passed by both houses. If even one house
the Speaker, Lok Sabha. However, in the case does not pass the bill, the bill ends there
of a Money Bill, there is no provision in the itself. There is no provision of joint sitting in
Constitution for a joint sitting of both Houses such a case.
as Lok Sabha clearly enjoys pre-eminence Statement 2: The proclamation of a war
over Rajya Sabha in financial matters. As emergency cannot be made by the president
regards a Constitution amendment Bill, it unless the Union cabinet gives him in
has been provided in the Constitution that written that such proclamation should be
such a Bill has to be passed by the specific made. If a proclamation is NOT revoked
majority, as prescribed under article 368 of subsequently, it should be laid before the
the Constitution, by both Houses (not by a parliament. The both houses of parliament
joint sitting). This means that if Rajya Sabha must approve such proclamation within
has reject two months. If the parliament does not

P.236 For Civil Services Preliminary Examination

02-Indian Polity_Q413-927.indd 236 8/7/2018 7:40:40 PM


approve the proclamation, it will become Q.883 Consider the following statements.
ineffective. 1. Lok Sabha exercises more powers in
It can be revoked either by the President matters of money bills than Rajya Sabha.
or by Lok Sabha as well. 2. Council of Ministers can be appointed
Q.881 In which of the following cases does the only by the Lok Sabha.
Rajya Sabha has equal status with that of Lok Which of the above is/are correct?
Sabha? (a)  1 only (b)  2 only
1. Passing of Financial bills involving (c)  Both 1 and 2 (d)  None
expenditure from the consolidated fund of Solution: (a)
India. Explanation and Learning: Once the Lok
2. Approval of ordinances by the President. Sabha passes the budget of the government or
3. Enlargement of jurisdiction of the any other money related law, the Rajya Sabha
Supreme Court. cannot reject it. The Rajya Sabha can only
Choose the correct answer using the codes delay it by 14 days or suggest changes in it.
below: The Lok Sabha may or may not accept these
(a)  1 and 2 (b)  2 and 3 changes. Thus, The statement 1 is correct.
(c)  1 and 3 (d)  All of the above Council of Ministers is not appointed by
Solution: (d) the Lok Sabha. It is done by the President.
Refer to the concerned section in Laxmikanth Ministers can come both from Lok Sabha and
– Indian Polity. Rajya Sabha. The statement 2 is incorrect.
Q.884 In which of the following matters Lok Sabha
(2) Unequal Powers with Lok Sabha exercises greater power than Rajya Sabha?
1. Passing money bills
Q.882 Consider the following statements. 2. Controlling the Government
Assertion (A): The Rajya Sabha has no power 3. Passing of constitutional amendment bills
to vote on the demand for grants. Choose the correct answer using the codes
Reason (R): Demand for grants is not below:
even discussed in the Rajya Sabha. (a)  1 and 2 only (b)  2 and 3 only
In the context of the above, which of these (c)  1 and 3 only (d)  All of the above
is correct? Solution: (a)
(a) A is correct, and R is an appropriate Our Constitution does give the Rajya Sabha
explanation of A. some special powers over the states. But
(b) A is correct, but R is not an appropriate on most matters, the Lok Sabha exercises
explanation of A. supreme power. Here are the ways;
(c) A is correct, but R is incorrect. 1 Any ordinary law needs to be passed
(d)  Both A and R are incorrect. by both the Houses. But if there is a
Solution: (c) difference between the two Houses, the
Justification: The Rajya Sabha has no final decision is taken in a joint session
power to vote on the demand for grants; it in which members of both the Houses sit
is the exclusive privilege of the Lok Sabha. together. Because of the larger number of
Moreover, the Rajya Sabha should return the members, the view of the Lok Sabha is
Money bill (or Finance bill or the Annual likely to prevail in such a meeting.
budget) to the Lok Sabha within fourteen 2 Lok Sabha exercises more powers in
days. The Lok Sabha can either accept or money matters. Once the Lok Sabha
reject the recommendations made by Rajya passes the budget of the government or
Sabha in this regard. any other money related law, the Rajya
This is because Rajya Sabha is not a Sabha cannot reject it. The Rajya Sabha
directly elected house (like the Lok Sabha) can only delay it by 14 days or suggest
and the government is not responsible to the changes in it. The Lok Sabha may or may
Rajya Sabha (like it is to the Lok Sabha). not accept these changes.

Indian Polity Question Bank P.237

02-Indian Polity_Q413-927.indd 237 8/7/2018 7:40:40 PM


3 Most importantly, the Lok Sabha controls Solution: (c)
the Council of Ministers. Only a person Background: Article 102 of the Constitution
who enjoys the support of the majority of provides that a person shall be disqualified
the members in the Lok Sabha is appointed from being chosen as a Member of Parliament
the Prime Minister. If the majority of the (MP) if he holds an office of profit under the
Lok Sabha members say they have ‘no government of India or the government of a
confidence’ in the Council of Ministers, state. However, Parliament can declare by
all ministers including the Prime Minister, law that the holding of certain offices will not
have to quit. The Rajya Sabha does not incur this disqualification.
have this power. The Parliament (Prevention of
Q.885 Consider the following statements with Disqualification) Act, 1959 lists certain
respect to the powers of the Rajya Sabha: offices of profit under the central and state
1. Any ordinary law needs to be passed by governments, which do not disqualify the
both the Lok Sabha and Rajya Sabha. holders from being an MP. The Chairperson
But if there is a difference between the of the National Commission for Scheduled
two Houses, the final decision is taken by Castes and Scheduled Tribes is exempted
Lok Sabha. from disqualification as per this list.
2. Once the Lok Sabha passes the budget Amendment: The Bill seeks to amend the
of the government or any other money Parliament (Prevention of Disqualification)
related law, the Rajya Sabha cannot Act, 1959. The amendment will exclude the
reject it. Chairperson of the National Commission for
3. The Rajya Sabha has power to pass motion the Scheduled Castes and the Chairperson of
of no-confidence against the Government. the National Commission for the Scheduled
Which of the statements is/are incorrect? Tribes from incurring any disqualification
(a)  2 Only (b)  1 and 3 from being an MP. The amendment to the
(c)  1, 2 and 3 (d)  None 1959 Act reflects the change brought about
Solution: (b) by the Constitution (89th Amendment)
Act, 2003. The constitutional amendment
bifurcated the National Commission for
Office of Profit–Article 102 the Scheduled Castes and Scheduled Tribes
Q.886 With reference to the Parliament (Prevention into two independent Commissions: the
of Disqualification) Amendment Bill, 2013, National Commission for the Scheduled
consider the following statements: Castes and the National Commission for the
1. Though Article 102 of the Constitution Scheduled Tribes. Hence, the amendment
provides that a person shall be disqualified to the 1959 Act was necessary to exclude
from being chosen as a Member of the Chairperson of these two Commissions
Parliament (MP) if he holds an office of from incurring any disqualification from
profit under the government of India or the being an MP
government of a state, the Parliament can Q.887 An ‘office-of-profit’ which disqualifies a
declare by law that the holding of certain person from being a member of the Union or
offices will not incur this disqualification. State Legislature does not include office held
2. The amendment will exclude the under:
Chairperson of the National Commission (a)  the Government of India
for the Scheduled Castes and the (b)  a State Government
Chairperson of the National Commission (c)  a local authority
for the Scheduled Tribes from incurring (d)  All of the above
any disqualification from being an MP. Solution: (c)
Which of the above statements is/are correct? The concept of office of profit has evolved
(a)  1 Only (b)  2 Only in England to preserve the independence of
(c)  Both 1 and 2 (d)  None the legislature by keeping the members away

P.238 For Civil Services Preliminary Examination

02-Indian Polity_Q413-927.indd 238 8/7/2018 7:40:40 PM


from any temptations from the executive that Parliamentary Privileges
can come in the way of independent discharge
of their duties. It also seeks to enforce the Q.888 Consider the following statements.
principle of separation of power between the 1. Parliamentary privileges are enshrined in
legislative, the judiciary and the executive – a the Constitution and enjoyed by individual
basic feature of the Constitution. MPs as well as each house of Parliament.
2. The presiding officer of the legislature has
Office-of-Profit under Indian Constitution
the final powers in deciding matters of
The term ‘office-of-profit’ has not been
breach of privilege.
defined in the Constitution. But, Articles
Which of the above is/are correct?
102(1) and 191(1)—which give effect to
(a)  1 only (b)  2 only
the concept of office of profit—prescribe
(c)  Both 1 and 2 (d)  None
restrictions at the central and state level on
Solution: (c)
lawmakers accepting government positions.
Justification: Statement 1: Privileges may be
Any violation attracts disqualification of MPs
classified into two groups:
or MLAs, as the case may be.
• Privileges that are enjoyed by the members
According to Article 102(1)(a), a
individually and
person shall be disqualified as a member
• Privileges that belong to each House (Lok
of Parliament for holding any office of
Sabha or Rajya Sabha) of Parliament
profit under the government of India or the
collectively.
government of any state, “other than an
Parliament has not yet codified its privileges.
office declared by Parliament by law not to
In fact, the privileges rest on conventions and
disqualify its holder”. Article 191(1)(a)  has
they may be ascertained by the practice and
a similar provision for the members of state
law that is in force In England
assemblies.
Statement 2: The Privileges and
However, Articles 102 and 191 clarify
immunities enjoyed by the members
that “a person shall not be deemed to hold an
individually are:
office of profit under the government of India
• Freedom of speech - The basic idea
or the government of any state by reason only
of extending this freedom being the
that he is a minister”.
necessity that every member would put
Further, the last part of the two provisions
forward without fear or favour his/her
protects a lawmaker holding a government
arguments for or against any matter before
position if the office has been made immune
the House.
to disqualification by law.
• Freedom from arrest - From this freedom
it is understood that no such member shall
Principles of Declaring Office-of-Profit be arrested in a civil case 40 days before
Four broad principles have evolved for and after the adjournment of the House
determining whether an office attracts the (Lok Sabha or Rajya Sabha) and also
constitutional disqualification. when the House is in session.
• First, whether the government exercises • Exemption from attendance as jurors and
control over appointment, removal and witnesses.
performance of the functions of the office. We will cover collective privileges in later tests.
• Second, whether the office has any Q.889 Which of the following is NOT a part of
remuneration attached to it. the Collective privileges of each house of
• Third, whether the body in which the Parliament?
office is held has government powers (a) The courts are prohibited to inquire
(releasing money, allotment of land, into the proceedings of a House or its
granting licences etc.). committees.
• Fourth, whether the office enables the (b) The house can make rules to regulate
holder to influence by way of patrona its own procedure and the conduct of

Indian Polity Question Bank P.239

02-Indian Polity_Q413-927.indd 239 8/7/2018 7:40:40 PM


its business and to adjudicate upon session. This privilege is available only in civil
such matters. cases and not in criminal cases or preventive
(c) The house can punish members as well detention cases. So, clearly (a)  is incorrect.
as outsiders for breach of its privileges They have freedom of speech in Parliament
or its contempt by even suspension and as under option (b). This freedom is subject
imprisonment. to the provisions of the Constitution and to
(d) It has the right to transfer to itself a case the rules and standing orders regulating the
pending in the Judiciary that is concerned procedure of Parliament.
with a member of that house. They are exempted from jury service as
Solution: (d) under option (c)
Learning: Other privileges are:
• It has the right to publish its reports, Sovereignty of Parliament
debates and proceedings and also the right
to prohibit others from publishing the same. Q.891 From which of the following reasons it may
• It can punish members as well as be concluded that the constitution enjoys a
outsiders for breach of its privileges or higher position than that of the parliament?
its contempt by reprimand, admonition (a) The constitution cannot be amended by
or imprisonment (also suspension or the parliament
expulsion, in case of members). (b) The constitution makers were national
• It has the right to receive immediate movement leaders; the members of the
information of the arrest, detention, parliament are not.
conviction, imprisonment and release of (c) The constitution specifies how parliament
a member. is to be formed and what are its powers.
• It can institute inquiries and order the (d) The constitution was framed before the
attendance of witnesses and send for parliament came into being
relevant papers and records. Solution: (c)
• No person (either a member or outsider) While D looks like it should be a reason; C
can be arrested, and no legal process is a better reason. Moreover, many Acts also
(civil or criminal) can be served within came before the constitution came into being
the premises of the House without the – but that does not mean those acts were
permission of the presiding officer superior to the Parliament.
Q.890 Which of the following is NOT an Individual Q.892 Which of the following factors limit the
Privilege accorded to a Member of sovereignty of the Indian Parliament
Parliament? 1. Federal system of government
(a) They cannot be arrested as a serving MP 2. Constitutionality of local bodies
and charges can be initiated only after 3. System of judicial review
they cede the office of MP. Choose the correct option from the codes
(b) No member is liable to any proceedings below:
in any court for anything said or any (a)  1 and 3 (b)  1 and 2
vote given by him in Parliament or its (c)  2 and 3 (d)  All of the above
committees. Solution: (a)
(c) They can refuse to give evidence and The constitutionality of local bodies
appear as a witness in a case pending in a has nothing to do with the powers of
court when Parliament is in session. the Parliament. In fact, the local bodies
(d) All the above privileges are enjoyed by themselves rely on the State government for
the MPs. power and authority.
Solution: (a) Q.893 The Parliament in India is not the supreme
Justification: They cannot be arrested during authority owing to which of the following:
the session of Parliament and 40 days before 1. Written Constitution
the beginning and 40 days after the end of a 2. Federal system

P.240 For Civil Services Preliminary Examination

02-Indian Polity_Q413-927.indd 240 8/7/2018 7:40:40 PM


3. Fundamental Rights people and therefore, it is expected to have
4. Separation of powers between the an upper hand over both Executive and
executive and the legislature. Judiciary. At the same time, there is the
Choose the correct answer using the text of the Constitution and it has given
following codes: powers to other organs of the government.
(a)  1, 2, 3 and 4 (b)  1, 2 and 3 Therefore, the supremacy of the Parliament
(c)  1 and 3 only (d)  2 and 4 only has to operate within this framework.
Solution: (b) Democracy is not only about votes and
A written Constitution and fundamental rights people’s representation. It is also about the
restrict the authority of the Parliament. It has to principle of rule of law.
ensure that its laws/constitutional amendments Democracy is also about developing
do not violate the constitution and the institutions and working through these
fundamental rights, and conform to the basic institutions. All the political institutions must
structure. be responsible to the people and maintain a
In a federal system, the state legislatures balance with each other.
also have a say in matters of making laws Contribution of the Judiciary
and constitutional amendments. Parliament During the controversy between the
cannot amend the constitution as it desires in Judiciary and the Parliament, the Parliament
certain cases. thought that it had the power and responsibility
There is no separation of powers in India. to make laws (and amendments) for
It is actually fusion of powers which makes furthering the interests of the poor, backward
the Parliament even more powerful. and the needy. The Judiciary insisted that all
Q.894 With reference to the supremacy of the this has to take place within the framework
Parliament, consider the following statements provided by the Constitution and pro-people
1. In a parliamentary democracy, the measures should not bypass legal procedures,
Parliament represents the people and because, once you bypass laws even with
therefore, it is expected to have an upper good intentions, that can give an excuse to the
supremacy of the Parliament has to power holders to use their power arbitrarily.
operate within this framework. And democracy is as much about checks on
3. The Judiciary, in its famous Golaknath arbitrary use of power as it is about the well-
ruling found a way out of the existing being of the people.
complications by turning to the spirit of The success of the working of the Indian
the Constitution rather than its letter in Constitution lies in resolving these tensions.
which it upheld that the balance of powers The Judiciary, in its famous Kesavananda
between various organs of the state was a ruling found a way out of the existing
basic structure of the constitution. complications by turning to the spirit of
Which of the above statements is/are correct? the Constitution rather than its letter. If
(a)  1 and 2 Only (b)  2 and 3 Only you read the Constitution, you will not find
(c)  1 and 3 Only (d)  3 Only any mention of the ‘basic structure’ of the
Solution: (a) Constitution. Nowhere does the Constitution
In the last fifty five years some very say that such and such are part of the basic
critical situations arose in the politics and structure. In this sense, the ‘basic structure’
constitutional development of the country. theory is the invention of the Judiciary. How
We have made a brief reference to some did it invent such a non- existent thing?
of these in this chapter already. In terms And how is it that all other institutions have
of constitutional-legal issues, the most accepted this during the past three decades?
serious question that came up again and There it lies the distinction between letter
again from 1950 was about the supremacy and spirit. The Court came to the conclusion
of the Parliament. In a parliamentary that in reading a text or document, we must
democracy, the Parliament represents the respect the intent behind that document. A

Indian Polity Question Bank P.241

02-Indian Polity_Q413-927.indd 241 8/7/2018 7:40:40 PM


mere text of the law is less important than Business. The work done by the Indian
the social circumstances and aspirations that Parliament is not only voluminous but also
have produced that law or document. The of a complex nature, hence a good deal of
Court was looking at the basic structure as its work is carried out in these Parliamentary
something without which the Constitution Committees.
cannot be imagined at all. This is an instance Both Houses of Parliament, Rajya Sabha
of trying to balance the letter and the spirit of and Lok Sabha, have similar Committee
the Constitution. structures with a few exceptions. Their
appointment, terms of office, functions
Committees of Parliament and procedures of conducting business are
broadly similar. These standing committees
General Provisions are elected or appointed every year, or
periodically by the Chairman of the Rajya
Q.895 With reference to various Parliamentary Sabha or the Speaker of the Lok Sabha, or as
committees in India, consider the following a result of consultation between them.
statements Q.896 Which of these is/are some of the general
1. Standing committee is permanent in nature characteristics of Parliamentary committees?
2. Standing committees are formed only by 1. All members should be elected by the
Lok Sabha house.
3. Ad hoc committees are temporary in 2. It presents its report to the Speaker or the
nature Chairman of the house.
4. The Committee on Petitions is an example 3. It must have the status of a consultative
of ad hoc committee committee with any Ministry at the
Which of the above statements is/are correct? Centre.
(a)  1 and 2 Only (b)  1,2 and 3 Only Select the correct answer using the codes
(c)  2 and 3 Only (d)  1 and 3 Only below.
Solution: (d) (a)  1 and 3 only (b)  2 only
Parliamentary Committees are of two (c)  1 and 2 only (d)  3 only
kinds: Ad hoc Committees and the Standing Solution: (b)
Committees. Ad hoc Committees are Justification: A parliamentary committee
appointed for a specific purpose and they cease means a committee that:
to exist when they finish the task assigned • Is appointed or elected by the House or
to them and submit a report. The principal nominated by the Speaker/Chairman. So,
Ad hoc Committees are the Select and Joint 1 is wrong.
Committees on Bills. Others like the Railway • Works under the direction of the Speaker /
Convention Committee, the Committees Chairman
on the Draft Five Year Plans and the Hindi • Presents its report to the House or to the
Equivalents Committee were appointed for Speaker / Chairman. So, the option 2 is
specific purposes. Apart from the Ad hoc correct.
Committees, each House of Parliament has • Has a secretariat provided by the Lok
Standing Committees like the Business Sabha / Rajya Sabha
Advisory Committee, the Committee on Statement 3: The consultative committees,
Petitions, the Committee of Privileges and the which also consist of members of Parliament,
Rules Committee, etc. are not parliamentary committees as they do
Standing committee is a committee not fulfill above four conditions.
consisting of Members of Parliament. It is
a permanent and regular committee which Q.897 A parliamentary committee means a
is constituted from time to time according committee which
to the provisions of an Act of Parliament 1. Is appointed or elected by the House or
or Rules of Procedure and Conduct of nominated by the Speaker / Chairman

P.242 For Civil Services Preliminary Examination

02-Indian Polity_Q413-927.indd 242 8/7/2018 7:40:40 PM


2. Presents its report to the House or to the The time at its disposal is limited. It
Speaker/Chairman cannot, therefore, give close consideration to
3. Has a secretariat provided by the Lok all the legislative and other matters that come
Sabha/Rajya Sabha up before it. A good deal of its business is,
Select the correct answer using the codes therefore, transacted by what are called the
below. Parliamentary Committees.
(a)  1 and 2 only (b)  1 and 3 only The committees that act as watchdogs are
(c)  2 and 3 only (d)  1, 2 and 3 the Committees on Subordinate Legislation
Solution: (d) (checks rules/regulation made under laws),
Justification: It also works under the the Committee on Government Assurances,
direction of the Speaker/Chairman of the the Committee on Estimates, the Committee
House. on Public Accounts and the Committee on
Any committee consisting of MPs will not Public Undertakings and Departmentally
necessarily be a Parliamentary Committee. Related Standing Committees (DRSCs).
For example, the consultative committees, The Committee on Estimates, the
which also consist of members of Parliament, Committee on Public Accounts, the
are not parliamentary Committees as they do Committee on Public Undertakings and
not fulfill above four conditions. DRSCs play an important role in exercising
Q.898 A Minister is NOT eligible for election to a check over governmental expenditure
which of these Parliamentary Committees? and Policy formulation. Committee on
1. Committee on Estimates Government Assurances (Lok Sabha)
2. Public Accounts Committee This Committee consists of 15 members
3. Committee on Public Sector Undertakings nominated by the Speaker. A Minister is not
(PSUs) nominated to this Committee.
Select the correct answer using the codes While replying to questions in the House
below. or during discussions on Bills, Resolutions,
(a)  1 and 2 only (b)  1 and 3 only Motions etc., Ministers at times give
(c)  2 and 3 only (d)  All of the above assurances or undertakings either to consider
Solution: (d) a matter or to take action or to furnish the
http://www.parliamentofindia.nic.in/ls/intro/ House further information later.
p21.htm The functions of this Committee are
All these committees deal with the to scrutinize the assurances, promises,
scrutiny of the government expenditure. undertakings etc. given by Ministers from
If a Minister is a part of such committees, time to time and to report to Lok Sabha on
it will create conflict of interest situation. the extent to which such assurances and
Hence, it is avoided. Moreover, the minister others have been implemented and to see
might influence the proceedings of the whether such implementation has taken
meetings. place within the minimum time necessary
for the purpose.
Q.899 Which of the following Committees acts
Q.900 With reference to Parliamentary Standing
as Parliament’s ‘Watch Dogs’ over the
Committees consider the following:
executive? 1. Standing Committees not only supervise
(a) Committee on Public assurances the work of various departments but also
(b)  Committee on Estimates their budget.
(c) Committees on Subordinate Legislation 2. Public Accounts Committee is an example
(d)  All of the above of Standing Committee.
Solution: (d) 3. Members of the Standing committees are
Justification: The work done by the selected from both Houses.
Parliament in modern times is not only varied 4. Standing Committees are permanent in
in nature, but considerable in volume. nature.

Indian Polity Question Bank P.243

02-Indian Polity_Q413-927.indd 243 8/7/2018 7:40:40 PM


5. Standing Committees are means of its members according to the principle of
parliamentary control over the executive. proportional representation by means of the
Which of the above statements is/are correct? single transferable vote. Thus, all parties get
(a) 1, 4 and 5 Only due representation in it.
(b) 1, 3 and 5 Only The chairman of the committee is
(c) 1, 2, 4 and 5 Only appointed by the Speaker from amongst its
(d) 1, 2, 3, 4 and 5 members.
Solution: (c) Until 1966–1967, the Chairman of the
Standing Committees not only supervise the Committee belonged to the ruling party.
work of various departments but also their However, since 1967, a convention has
budget, their expenditure and bills that come developed whereby the chairman of the
up in the house relating to the department. committee is selected invariably from the
It is a permanent and regular committee Opposition
which is constituted from time to time Q.902 Which of the following statements about
according to the provisions of an Act of the Public Accounts Committee (PAC) is
Parliament or Rules of Procedure and INCORRECT?
Conduct of Business. The work done by the (a) It was first setup under the provisions of
Indian Parliament is not only voluminous the Government of India Act of 1919.
but also of a complex nature, hence a good (b) Its members are drawn from both houses
deal of its work is carried out in these of the Parliament.
Parliamentary Committees. There are (c) A minister cannot be elected as a member
two types of Parliamentary Committee, of the committee.
the Standing Committee and the Ad hoc (d) Chairman of the Committee is appointed
Committee. Public Accounts Committee is an by the Leader of the House.
example of Standing Committee. Members Solution: (d)
of the Standing committees are selected Learning: This committee was setup first in
from both Houses, except for Estimates 1921 under the provisions of the Government
Committee where all the members are from of India Act of 1919 and has since been in
Lok Sabha Only. existence. So, A is correct.
At present, it consists of 22 members
(1) Public Accounts Committee (PAC) (15 from the Lok Sabha and 7 from the
Q.901 Which of the following statements Rajya Sabha). The members are elected by
about the Public Accounts Committee is the Parliament every year from amongst
INCORRECT? its members according to the principle of
(a) Its members are nominated by the proportional representation by means of the
Speaker giving due representation from single transferable vote. So, B is correct.
all parties. A minister cannot be elected as a member
(b) The term of office of the members is one of the committee. The chairman of the
year. committee is appointed by the Speaker from
(c) A minister cannot be chosen as a member amongst its members. So, D is incorrect.
of the committee. The function of the committee is to
(d) The committee examines the annual examine the annual audit reports of the
audit reports of the Comptroller and comptroller and auditor general of India
Auditor General of India (CAG). (CAG), which are laid before the Parliament
Solution: (a) by the president.
Justification: At present, it consists of 22 Q.903 Apart from selected MPs, Public Account
members (15 from the Lok Sabha and from Committee (PAC) members is/are
the Rajya Sabha). The members are elected 1. Finance Secretary, Union Government
by the Parliament every year from amongst 2. Eminent citizens from Industry and Trade

P.244 For Civil Services Preliminary Examination

02-Indian Polity_Q413-927.indd 244 8/7/2018 7:40:40 PM


3. Comptroller and Auditor General of India Which of these is/are true?
4. Governor, RBI (a)  All of the above (b)  1 and 3
Select the correct answer using the codes (c)  2, 3 and 4 (d)  1, 3 and 4
below. Solution: (b)
(a)  1 and 3 only (b)  2 and 4 only The Estimates Committee examines the
(c)  1, 2, 3 and 4 (d)  None of the above effectiveness and economy of the expenditure
Solution: (d) over the year and based on its own exercises.
Justification: The CAG assists the Members are drawn from both the houses in
committee; he is not its member. So, the the PAC.
option 3 is wrong.
RBI Governor may be called upon by (2) Estimates Committee
the PAC for justification, assistance etc. He
is also not its member. So, the option 4 is Q.906 Consider the following statements about a
wrong. certain Parliamentary Committee.
1. The Rajya Sabha has no representation in
Q.904 Who appoints the Chairman of the Public
this committee.
Accounts Committee (PAC)  of the
2. The committee examine the estimates
Parliament?
included in the budget and suggest
(a)  Prime Minister of India
‘economies’ in public expenditure.
(b)  Speaker of Lok Sabha
3. The committee works throughout the
(c)  Chairman of Rajya Sabha
financial year and reports to the House as
(d)  Secretary, Lok Sabha
its examination proceeds.
Solution: (b)
The above refer to?
Learning: The chairman of the committee is
(a) Committee on Public Undertakings
appointed by the Speaker from amongst its
(b) Departmental Standing Committees
members.
(c) Committee on Government Finances
• The members are elected by the
(d)  Estimates Committee
Parliament every year from amongst its
Solution: (d)
members according to the principle of
Learning: All the thirty members are from
proportional representation by means of
Lok Sabha only. These members are elected
the single transferable vote.
by the Lok Sabha every year from amongst
• Until 1966–67, the chairman of the
its members.
committee belonged to the ruling party.
1. A minister cannot be elected as a member
However, since 1967 a convention has
of the committee. The chairman of the
developed whereby the chairman of the
committee is appointed by the Speaker
committee is selected invariably from the
from amongst its members and he is
Opposition.
invariably from the ruling party.
• The function of the committee is to examine
2. The function of the committee is to
the annual audit reports of the CAG.
examine the estimates included in the
Q.905 Consider the following statements about the budget and suggest ‘economies’ in public
Public Accounts Committee: expenditure. Hence, it has been described
1. It consists of members from both the as a ‘continuous economy committee’.
houses of Parliament. 3. It is not incumbent on the committee to
2. Its members are nominated by the Speaker examine the entire estimates of any one
of Lok Sabha and the Chairman of Rajya year. The demands for grants are finally
Sabha. voted despite the fact that the committee
3. It examines the annual audit reports of the has made no report.
CAG submitted to the Parliament.
Q.907 Consider the following with reference to the
4. It examines the effectiveness and
Estimates Committee of the Parliament.
efficiency of the government expenditure
1. The Rajya Sabha has no representation in
based on the CAG reports.
this committee.

Indian Polity Question Bank P.245

02-Indian Polity_Q413-927.indd 245 8/7/2018 7:40:40 PM


2. A minister can be elected as a member of Solution: (b)
the committee. Justification: The options are set in a way
3. The chairman of the committee is that you can easily recognize the answer as
appointed by the Speaker. (b). Also, It is exclusively a committee of the
Select the correct answer using the codes Lower House.
below. Public Accounts Committee scrutinizes
(a)  1 and 3 only (b)  3 only the executive’s expenditure, so the
(c)  2 and 3 only (d)  1 and 2 only dominance of a ruling party member should
Solution: (a) not be expected in the committee. So, (a)  is
Justification: Statement 1: All the thirty incorrect.
members are from Lok Sabha only. The Business committee and Committee
Rajya Sabha has no representation in this on Delegated Legislation do not deal with
committee. These members are elected by economies. So, (c)  and (d)  are incorrect.
the Lok Sabha every year from amongst The function of the Estimates committee
its members, according to the principles of is to examine the estimates included in the
proportional representation by means of a budget and suggest ‘economies’ in public
single transferable vote. expenditure. Hence, it has been described as
Statement 2 and 3: All parties get due a ‘continuous economy committee’.
representation in it. The term of office is Q.909 Consider the following about Estimates
one year. A minister cannot be elected as a Committee.
member of the committee. The chairman of 1. It is exclusively a committee of the lower
the committee is appointed by the Speaker house.
from amongst its members and he is 2. Chairman of this committee is invariably
invariably from the ruling party. from the ruling party.
Learning: The origin of this committee can 3. The committee is guided by Comptroller
be traced to the standing financial committee and Auditor General of India.
set up in 1921. Choose the correct answer using the codes
The first Estimates Committee in the post- below.
independence era was constituted in 1950 on (a)  1 and 2 only (b)  2 and 3 only
the recommendation of John Mathai, the then (c)  2 only (d)  1 only
finance minister. Solution: (a)
The function of the committee is to Originally, it had 25 members but in 1956
examine the estimates included in the its membership was raised to 30. All the
budget and suggest ‘economies’ in public thirty members are from Lok Sabha only.
expenditure. Hence, it has been described as The Rajya Sabha has no representation
a ‘continuous economy committee’. in this committee. These members are
Q.908 Consider the following statements. elected by the Lok Sabha every year from
1. A minister cannot be elected as a member amongst its members, according to the
of the committee. principles of proportional representation
2. The chairman of the committee is by means of a single transferable vote.
appointed by the Speaker from amongst Thus, all parties get due representation in
its members and he is invariably from the it. The term of office is one year. A minister
ruling party. cannot be elected as a member of the
3. It has been described as a ‘continuous committee. The chairman of the committee
economy committee’. is appointed by the Speaker from amongst
The above refers to? its members and he is invariably from the
(a)  Public Accounts Committee ruling party.
(b)  Estimates Committee Q.910 Which of the following Parliamentary
(c)  Business Committee committees does NOT witness any
(d) Committee on Delegated Legislation participation from Rajya Sabha?

P.246 For Civil Services Preliminary Examination

02-Indian Polity_Q413-927.indd 246 8/7/2018 7:40:40 PM


(a) Public Accounts Committee alternative policies in order to bring about
(b) Committee on Empowerment of Women efficiency and economy in administration.
(c) Committee of Privileges From time to time the Committee selects
(d) Estimates Committee such of the estimates pertaining to a Ministry
Solution: (d) or a group of Ministries or the statutory
Committee of Privileges: The functions of and other Government bodies as may seem
this committee are semi-judicial in nature. It fit to the Committee. The Committee also
examines the cases of breach of privileges of examines matters of special interest which
the House and its members and recommends may arise orcome to light in the course of its
appropriate action. The Lok Sabha committee work or which are specifically referred to it
has 15 members, while the Rajya Sabha by the House or the Speaker.
committee has 10 members. Committee on Public Undertakings
Committee on Empowerment of Women: The Committee on Public Undertakings
This committee was constituted in 1997 and consists of 15 members elected by the Lok
consists of 30 members (20 from Lok Sabha Sabha and 7 members of Rajya Sabha are
and 10 from Rajya Sabha). associated with it. A Minister is not eligible
It considers the reports of the National for election to this Committee. The term of
Commission for Women and examines the the Committee is one year.
measures taken by the Union Government The functions of the Committee on Public
to secure status, dignity and equality for Undertakings are—(a)  to examine the
women in all fields. reports and accounts of Public Undertakings;
(b) to examine the reports, if any, of the
(3) Committee on Public Undertakings Comptroller and Auditor General on the
Public Undertakings; (c) to examine in
Q.911 Which of the following best describes the context of the autonomy and efficiency
the function of committee on public of the Public Undertakings whether the
undertakings? affairs of the Public Undertakings are being
(a) to examine the reports, if any, of the managed in accordance with sound business
Comptroller and Auditor General on the principles and prudent commercial practices;
Public Undertakings and (d) such other functions vested in the
(b) to suggest alternative policies in order to Committee on Public Accounts and the
bring about efficiency and economy in Committee on Estimates in relation to the
administration Public Undertakings as are not covered by
(c) to ascertain whether the money granted clauses (a), (b) and (c) above and as may
by Parliament has been spent by be allotted to the Committee by the Speaker
Government “within the scope of the from time to time. The Committee does
Demand” not, however, examine matters of major
(d) None of the above Government policy and matters of day-to-
Solution: (a) day administration of the Undertakings.
Committee on Estimates Committee on Public Accounts
This Committee consists of 30 members This Committee consists of 15 members
who are elected by the Lok Sabha every year elected by the Lok Sabha and 7 members
from amongst its members. A Minister is not of the Rajya Sabha are associated with it. A
eligible for election to this Committee. The Minister is not eligible for election to this
term of the Committee is one year. The main Committee. The term of the Committee is
function of the Committee on Estimates is one year.
to report what economies, improvements in The main duty of the Committee is to
organisation, efficiency, or administrative ascertain whether the money granted by
reform, consistent with the policy underlying Parliament has been spent by Government
the estimates may be effected and to suggest “within the scope of the Demand”. The

Indian Polity Question Bank P.247

02-Indian Polity_Q413-927.indd 247 8/7/2018 7:40:40 PM


Appropriation Accounts of the Government 4. The term of office of each standing
of India and the Audit Reports presented by committee is one year from the date of its
the Comptroller and Auditor General mainly constitution.
form the basis for the examination of the Select the correct answer using the codes
Committee. Cases involving losses, nugatory below.
expenditure and financial irregularities come (a)  1 only (b)  1, 3 and 4 only
in for severe criticism by the Committee. (c)  2 and 4 only (d)  1, 2 and 3 only
The Committee is not concerned with Solution: (b)
questions of policy. It is concerned only with Justification: Statement 1: They assist the
the execution of the policy laid down by Parliament in debating the budget more
Parliament and its results. effectively as they examine department’s
Business Advisory Committee (Lok Sabha) budget more closely than the Parliament
The Business Advisory Committee of Lok could do. The 24 standing committees cover
Sabha consists of 15 members including under their jurisdiction all the ministries /
the Speaker who is the ex-officio Chairman. departments of the Central Government.
The members are nominated by the Statement 2: Each standing committee
Speaker. Almost all sections of the House consists of 31 members (21 from Lok Sabha
are represented on the Committee as per and 10 from Rajya Sabha). The members of
the respective strength of parties in the the Lok Sabha are nominated by the Speaker
House. The function of the Committee is to from amongst its own members, just as the
recommend the time that should be allotted members of the Rajya Sabha are nominated
for the discussion of such Government by the Chairman from amongst its members.
legislative and other business as the Speaker, Statement 3: A minister is not eligible
in consultation with the Leader of the House, to be nominated as a member of any of the
may direct to be referred to the Committee. standing committees. In case a member,
The Committee, on its own initiative, may after his nomination to any of the standing
also recommend to the Government to bring committees, is appointed a minister, he then
forward particular subjects for discussion in ceases to be a member of the committee.
the House and recommend allocation of time Statement 4: The term of office of each
for such discussions. The decisions reached standing committee is one year from the date
by the Committee are always unanimousin of its constitution. Out of the 24 standing
character and representative of the collective committees, 8 work under the Rajya Sabha
view of the House. The Committee generally and 16 under the Lok Sabha.
meets at the beginning of each Session and
thereafter as and when necessary. (5) Other Committees
Q.913 Consider the following about bills referred to
(4) Departmental Standing a Joint Committee of the Parliament.
Committees 1. A Joint Committee cannot recommend
Q.912 Consider the following about Departmental amendment of the bill, and can only
standing committees. record its observations on the bill.
1. The main objective of the standing 2. A Money bill cannot be referred to a Joint
committees is to secure greater financial Committee of the Houses.
accountability of the Executive to the Which of the above is/are correct?
Legislature. (a)  1 only (b)  2 only
2. Such committees consist of members (c)  Both 1 and 2 (d)  None
from Lok Sabha alone. Solution: (b)
3. A minister is not eligible to be nominated Justification: Statement 1: If a Bill is
as a member of any of the standing referred to a Select or a Joint Committee, it
committees. considers the Bill clause-by-clause just as the
House does. Amendments can be moved to

P.248 For Civil Services Preliminary Examination

02-Indian Polity_Q413-927.indd 248 8/7/2018 7:40:40 PM


the various clauses by the members of the by the President after consultation with
Committee. the presiding officer of the house.
After the report of the Select or Joint Which of the above is/are correct?
Committee has been presented to the House, (a)  1 only (b)  2 only
the member-in-charge of the Bill usually (c)  Both 1 and 2 (d)  None
moves the motion for consideration of Solution: (c)
the Bill, as reported by the Select or Joint Justification: Statement 1: A98 says that
Committee, as the case may be. Each House of Parliament shall have a
Statement 2: A Money Bill or a Financial separate secretarial staff provided that
Bill containing any of the provisions nothing in this clause shall be construed as
calculated to make a Bill a Money Bill, preventing the creation of posts common to
however, cannot be referred to a Joint both Houses of Parliament.
Committee of the Houses. This is because RS Statement 2: The President may, after
has no powers of amending a money bill. consultation with the Speaker of the House
of the People or the Chairman of the Council
Secretariat of States, as the case may be, make rules
regulating the recruitment and the conditions
Q.914 Each House of Parliament has separate of service of persons appointed to the
secretarial staff of its own. The secretariat of secretarial staff of the House of the People or
each House is headed by a secretarygeneral. the Council of States and any rules so made
He is a permanent officer and is appointed by shall have effect subject to the provisions of
the any law made under the said clause.
(a)  Leader of the House
Q.916 Consider the following about the Secretary
(b)  Presiding Officer of the House
General, Lok Sabha.
(c)  President of India
1. He is appointed by the President of
(d) The Department of Personnel and
India in consultation with the Speaker of
Training
Lok Sabha.
Solution: (b)
2. The post of Secretary General is of the
Learning: The Secretariat of Rajya Sabha
rank of the Cabinet Secretary in the
was set up pursuant to the provisions
Government of India.
contained in Article 98 of the Constitution.
Which of the above is/are correct?
Parliament may by law regulate the
(a)  1 only (b)  2 only
recruitment and the conditions of service of
(c)  Both 1 and 2 (d)  None
persons appointed to the secretarial staff of
Solution: (b)
either House of Parliament.
Justification: As Secretary General, he is also
The President may, after consultation
the Administrative head of the Secretariat of
with the Speaker of the House of the People
the Lok Sabha.
or the Chairman of the Council of States, as
• The post of Secretary General is of the rank
the case may be, make rules regulating the
of the Cabinet Secretary in the Government
recruitment and the conditions of service of
of India, who is the senior most civil
persons appointed to the secretarial staff of
servant to the Indian Government.
the House.
• The incumbent to the post is appointed by
Q.915 With reference to the Secretariat of Rajya the Speaker of Lok Sabha in consultation
Sabha, consider the following statements. with the Prime Minister of India and the
1. The Secretariat was set up pursuant to Leader of the Opposition in the Lok Sabha.
a constitutional provision of separate • As per precedence, incumbents to the
secretarial staff for each House of post of Secretary General have either
Parliament. been senior officers in the Lok Sabha
2. Rules regulating the conditions of service Secretariat or senior civil servants in the
of appointees to the Secretariat are made Government of India.

Indian Polity Question Bank P.249

02-Indian Polity_Q413-927.indd 249 8/7/2018 7:40:40 PM


Parliamentary forums the states, if majority of the states pass a
resolution seeking for the same.
Q.917 Parliamentary forums provide a platform to Which of these is/are true?
the members to have interactions with the (a)  1 and 2 (b)  2 and 3
ministers and officials concerned. Which of (c)  1 and 3 (d)  Only 1
the following statements about Parliamentary Solution: (d)
Forums is correct? While Ministers vacate their offices,
(a) The Speaker of Lok Sabha is the President they do not lose their membership of the
of all the Forums. house.
(b) Members can be drawn from both Lok A resolution for making law from the
Sabha and Rajya Sabha. state list needs the approval of the council of
(c) Members of these forums are nominated states (Rajya Sabha)  to be applicable to all
by the President. the states. If a majority of the states pass a
(d) None of the above resolution to this effect, it will apply only to
Solution: (b) those states.
Justification: Option (a): The Speaker
Q.919 The Indian Parliamentary Group is
of Lok Sabha is the President of all the
(a) An autonomous body, membership of
Forums except the Parliamentary Forum on
which is open to all current or former
Population and Public Health wherein the
members of the Indian Parliament.
Chairman of Rajya Sabha is the President
(b) A standing committee of the Parliament
and the Speaker is the Co-President.
presided by the Speaker
Option (b): Each Forum consists of
(c) A youth forum eliciting their participation
not more than 31 members (excluding the
in active national politics
President and ex-officio VicePresidents) out
(d) An attached office under the Ministry
of whom not more than 21 are from the Lok
of Parliamentary Affairs that trains
Sabha and not more than 10 are from the
and assists the newly elected MPs in
Rajya Sabha.
discharge of their responsibilities
Option (c): Members (other than the
Solution: (a)
President and Vice-Presidents) of these
Learning: The Indian Parliamentary Group
forums are nominated by the Speaker/
is an autonomous body formed in the year
Chairman from amongst the leaders of
1949 in pursuance of a Motion adopted by
various political parties/groups or their
the Constituent Assembly (Legislative)
nominees, who have special knowledge/keen
in 1948.
interest in the subject.
Membership of the Indian Parliamentary
The duration of the office of members
Group is open to all Members of Parliament
of the forum is co-terminus with their
and ex-Members of Parliament. A Member of
membership in the respective Houses. A
Parliament can become a life Member of the
member may also resign from the forum by
Group on payment of life subscription of Rs.
writing to the Speaker/Chairman.
500 by sending an application together with
the life subscription to the Secretary-General
Miscellaneous of Lok Sabha (who is ex-officio Secretary-
Q.918 Consider the following statements: General of the Group).
1. When Lok Sabha is dissolved, ministers The aims and objects of the Indian
in the Rajya Sabha also have to vacate Parliamentary Group are as follows.
their office in the council of Ministers. 1. to promote personal contacts between
2. Rajya Sabha cannot reject a money bill, Members of Parliament;
but it has the power to vote for public 2. to study questions of public importance
expenditure. that are likely to come up before Parliament
3. Lok Sabha can enact a law from the and arrange Seminars and discussions
subjects in the state list applicable to all

P.250 For Civil Services Preliminary Examination

02-Indian Polity_Q413-927.indd 250 8/7/2018 7:40:41 PM


and orientation courses and bring out Q.921 Consider the following about the Economic
publications for the dissemination of Survey.
information to the Members of the Indian 1. It is published by the Ministry of Finance.
Parliamentary Group; 2. It is presented along with the budget to the
3. to arrange lectures on political, defence, Lok Sabha.
economic, social and educational 3. It is not presented to Rajya Sabha.
problems by Members of Parliament and Choose the correct answer using the codes
distinguished persons; and below.
4. to arrange visits to foreign countries with (a)  1 and 2 only (b)  3 only
a view to develop contacts with Members (c)  2 and 3 only (d)  1 only
of other Parliaments Solution: (d)
Q.920 Consider the following statements with The Finance Ministry of India presents the
reference to the Rules of Procedure and Economic Survey in the parliament every
Conduct of Business in Lok Sabha. year, just before the Union Budget. It is the
1. The Annual Budget shall be presented to ministry’s view on the annual economic
the House on such day as the Presiding development of the country. Economic
Officer of the house may direct. Survey reviews the developments in the
2. The Budget shall be presented to the Indian economy over the previous 12
House in such form as the Estimates months, summarizes the performance on
Committee decides. major development programs, and highlights
3. There shall be no discussion of the Budget the policy initiatives of the government and
on the day on which it is presented to the the prospects of the economy in the short to
House. medium term. This document is presented to
Select the correct answer using the codes both houses of Parliament during the Budget
below. Session (not along with the budget).
(a)  1 and 2 only (b)  3 only Q.922 Consider the following statements:
(c)  2 and 3 only (d)  1 and 3 only 1. A public bill can only be introduced by a
Solution: (b) minister.
Justification: Statement 1: The Annual 2. The Rajya Sabha can delay an ordinary
Financial Statement of the Government of bill passed by the Lok Sabha for indefinite
India in respect of each financial year (‘the period of time till the bill lapses.
Budget’) shall be presented to the House on 3. Rajya Sabha cannot delay passing a
such day as the President may direct. money bill for more than two weeks.
Statement 2: The Budget shall be Which of these is/are true?
presented to the House in such form as the (a)  1 and 2 (b)  2 and 3
Finance Minister may, after considering (c)  1 and 3 (d)  Only 1
the suggestions, if any, of the Estimates Solution: (d)
Committee, settle. A public bill reflects the policies of the ruling
Statement 3: On a day to be appointed by party and hence can be introduced only by a
the Speaker subsequent to the day on which minister. A private bill on the other hand deals
the Budget is presented and for such time as with general matters, and is thus introduced
the Speaker may allot for this purpose, the by a private MP.
House shall be at liberty to discuss the Budget Rajya Sabha cannot delay the passing of a
as a whole or any question of principle money bill as central budget or any spending
involved therein, but no motion shall be should be decided by house of people and not
moved nor shall the Budget be submitted to house of the states.
the vote of the House. The Finance Minister In case the delay is more than six months
shall have a general right of reply at the end for an ordinary bill, a joint sitting is called by
of the discussion. the President.

Indian Polity Question Bank P.251

02-Indian Polity_Q413-927.indd 251 8/7/2018 7:40:41 PM


Q.923 Under which of the following conditions Q.924 Match List I with List II and select the correct
security deposits of a candidate contesting answer from the codes given below the list.
for a Lok Sabha seat is returned to him/her? List I List II
I.  The nomination made by the candidate if A. Supreme 1. Law based on
found to be invalid. Legislation discretion of
II. The candidate has withdrawn his/her authority
nomination even through it is found valid. B. Executive 2. Law enacted
III. The candidate lost the polls but secured other than by authority
1/6th of the total number of valid votes Legislation legislature
polled in that election. C. Delegated 3. Law enacted
Select the correct answer using the code Legislation by legislation
given below [CDS 2009] D. Conditional 4. Ordinance
(a)  I and II (b)  I, II and III Legislation issued by the
(c)  II and III (d)  I only president/
Solution: (a) governor
Section 158 of the Representation of the 5. Law made by
People Act, 1951 lays down the method judges while
of deposal of the deposits made by the deciding the case
candidates. According to it every candidate Code:
whose nomination paper was found valid ABCD
must have made the requisite deposit of (a)  3 4 2 1 (b)  2 1 5 4
Rs.10,000 of in the case of an election from (c)  3 1 2 4 (d)  2 4 5 1
a Parliamentary Constituency or, as the case Solution: (a)
may be, Rs.5,000 in the case of an election Self-explanatory
from an Assembly Constituency. (The Q.925 Under the Constitution “Doctrine of Eclipse’”
amount of deposit is half in the case of a applies to only
candidate belonging to a scheduled Caste or (a) to the pre-constitutional laws.
Scheduled Tribe). (b) to the post-constitutional laws but only
The deposit made by a candidate shall in respect of non-citizens.
be returned if the following conditions are (c) to the post-constitutional laws in respect
satisfied. of citizens only.
(i) the candidate is not shown in the list (d) to all laws, pre-constitutional or post
of contesting candidates, that is to say, Constitutional of citizens only.
either his nomination was rejected or Solution: (a)
after his nomination was accepted, he Doctrine of Eclipse says that any law
withdrew his candidature; or inconsistent with Fundamental Rights
(ii) he dies before the commencement of the is not invalid. It is not dead totally but
poll; or overshadowed by the fundamental right.
(iii) he is elected; or The inconsistency (conflict) can be removed
(iv) he is not elected but gets more than 1/6th by constitutional amendment to the relevant
of the total number of valid votes polled fundamental right so that eclipse vanishes
by all the candidates at the election. and the entire law becomes valid
In furtherance, (1) If the candidate has polled Supreme court has held that article 13(1)
exactly 1/6th of the total number of valid had the effect of nullifying or rendering the
votes polled by all the candidates, the deposit existing law which had become inconsistent
will not be refunded. with fundamental right as it then stood,
(2) if the candidate was elected, the ineffectual, nugatory and devoid of any legal
deposit will be refunded even if he did not force or binding effect, only with respect to
poll more than 1/6th of the total valid votes the exercise of the fundamental right on and
polled by all the candidates. after the date of the commencement of the

P.252 For Civil Services Preliminary Examination

02-Indian Polity_Q413-927.indd 252 8/7/2018 7:40:41 PM


Constitution. Hence the doctrine applies to legislative power may be overt or it may be
the pre constitutional laws. covert. When it is overt, we say the law is
Q.926 The term ‘Colourable Legislation’ implies obviously bad for non- compliance with the
(a) a legislation which is openly and directly requirements of the Constitution, that is to
out of the purview of the legislature say, the law is ultra vires. When, however,
(b) a Statute passed by the legislature the non-compliance is covert, we say that
purporting to act within the limits of its it is a ‘fraud on the Constitution’, the fraud
powers, but in substance and in reality it complained of being that the Legislature
has transgressed these powers pretends to act within its power while in fact
(c) a legislation aimed at reducing the it is not so doing.
power of judiciary Limitations on the Application of
(d) any law that is fit to be declared as ultra Doctrine of Colourable Legislation
vires by the judiciary Rule of lapse 1. The doctrine has no application where the
Solution: (b) powers of a Legislature are not fettered by
Doctrine also traces its origin to a Latin any Constitutional limitation.
Maxim: 2. The doctrine is also not applicable to
“Quando aliquid prohibetur ex directo, Subordinate Legislation.
prohibetur et per obliquum” 3. The doctrine of colourable legislation
This maxim implies that “when anything does not involve any question of bona
is prohibited directly, it is also prohibited fides or mala fides on the part of the
indirectly”. In common parlance, it is meant legislature.
to be understood as “Whatever legislature 4. There is always a Presumption of
can’t do directly, it can’t do indirectly”. Constitutionality in favour of the Statute.
In our Constitution, this doctrine is usually Q.927 What does the “Rule of lapse’ mean?
applied to Article 246 which has demarcated (a) All pending bills in Parliament lapse
the Legislative Competence of the Parliament with its prorogation
and the State Legislative Assemblies by (b) All appropriations voted by the
outlining the different subjects under List I legislature expire at the end of the
for the Union, List II for the States and List financial year
III for both, as mentioned in the Seventh (c) The demand for grants of a ministry
Schedule. lapses with criticism of its policy by the
This doctrine comes into play when a opposition
Legislature does not possess the power (d) The appropriation bill lapses if it is not
to make law upon a particular subject returned by the Rajya Sabha within
but nonetheless indirectly makes one. 14 days.
By applying this principle the fate of the Solution: (b)
Impugned Legislation is decided. The budget is based on the principle of
Supreme court has upheld the principle in annuality, that is, the Parliament grants
K. C. Gajapati Narayana Deo v. The State of money to the government for one financial
Orissa, AIR 1953 Ori 185. The idea conveyed year. If the granted money is not spent by
by the expression is that although apparently the end of the financial year, then the balance
a legislature in passing a statute purported expires and returns to the Consolidated Fund
to act within the limits of its powers, yet in of India. This practice is known as the ‘rule
substance and in reality it transgressed these of lapse’. It facilitates effective financial
powers, the transgression being veiled by control by the Parliament as no reserve
what appears, on proper examination, to be a funds can be built without its authorisation.
mere presence or disguise.” However, the observance of this rule leads to
This Doctrine is also called as “Fraud on heavy rush of expenditure towards the close
the Constitution”. The failure to comply with of the financial year. This is popularly called
a Constitutional condition for the exercise of as ‘March Rush’.

Indian Polity Question Bank P.253

02-Indian Polity_Q413-927.indd 253 8/7/2018 7:40:41 PM


Q.928 Which one of the following is a common STATE GOVERNMENT –
objective of the ‘Rule of Lapse’, Sunset
Legislation and Zero Based Budgeting? PART VI – ARTICLES 152 to 237
(a)  Economy in expenditure
(b)  Legislative control (23) GOVERNOR – PART VI
(c) Review and reauthorisation of
expenditure Q.929 In a judgment, Supreme Court has declared
(d) Ensuring achievement of physical that Governor is not the “conscience keeper”
targets. of the legislative assembly. What does this
Solution: (c) imply?
Refer previous explanation. 1. It is inappropriate for the Governor
to manage proceedings of legislative
assembly to keep the right order.
2. It is wrong for the Governor to attend the
legislative proceedings of the assembly.
Which of the above is/are correct?
(a)  1 only (b)  2 only
(c)  Both 1 and 2 (d)  None
Solution: (a)
Justification: In past Supreme Court had
restored the Congress government in Arunachal
Pradesh and declared all decisions of
Governor as “unconstitutional”.
• Governor’s decision had led to imposition
of President’s rule in the state and later
formation of a new government.
• This judgment is historic since it is the
first time the Supreme Court has restored
a government after it was dismissed and a
new government was sworn in.
• SC directed that a Governor cannot have
the freedom to determine when and in
which situation he can take a decision at his
own discretion without the aid and advice
of the Chief Minister and his Council of
Ministers.
• The court said that a Governor is not
an elected representative, but only an
executive nominee whose powers flow
from the aid and advice of the Cabinet.
• As per the judgment, Governor can act
without the aid and advice only when
a government has lost its majority in a
floor test.
• Judgment also ruled that the Governor is not
the conscience-keeper of the “Legislative
Assembly” and that he had to stay away
from the business of the Assembly.
• The court said that the Constitution does
not assign any role to a Governor to
interfere in the activities of the Assembly.

P.254 For Civil Services Preliminary Examination

03-Indian Polity_Q928-1069.indd 254 8/7/2018 7:41:51 PM


• Hence it would be outside the domain of The governor is the chief executive head of the
his powers to fix a date for an Assembly state. But, like the president, he is also nominal
session or to decide how the Assembly executive head (titular or constitutional head).
functions. The governor also acts as an agent of the
Q.930 Consider the following statements with central government. Therefore, the office of
reference to the state administration. governor has a dual role.
Assertion (A): There is no office of vice-
governor in the state like that of Vice-President Appointment
at the Centre.
Q.932 How is the Governor of a state appointed?
Reason (R): The office of governor of a
(a) Indirectly elected by the State Legislative
state is not an employment under the Central
Assembly
government.
(b)  Nominated by the President
In the context of the above, which of these
(c) Elected by a collegium consisting of the
is correct?
heads of all local bodies of the concerned
(a) A is correct, and R is an appropriate
State
explanation of A.
(d) Nominated by the State Government
(b) A is correct, but R is not an appropriate
subject to the approval of the Union
explanation of A.
Government
(c) A is incorrect, but R is correct.
Solution: (b)
(d)  A is correct, but R is incorrect.
Justification and Learning: The governor
Solution: (b)
is neither directly elected by the people nor
Justification: The two statements are
indirectly elected by a specially constituted
unrelated even though correct.
electoral college as is the case with the president.
A is correct as only the office of Governor
He is appointed by the president by
has been constitutionally provided.
warrant under his hand and seal. In a way,
R is correct in the following way.
he is a nominee of the Central government.
The governor is neither directly elected
But, as held by the Supreme Court in 1979,
by the people nor indirectly elected by a
the office of governor of a state is not an
specially constituted electoral college as is
employment under the Central government.
the case with the president.
It is an independent constitutional office and
He is appointed by the president by
is not under the control of or subordinate to
warrant under his hand and seal. In a way,
the Central government.
he is a nominee of the Central government.
But, as held by the Supreme Court in 1979, Q.933 Consider the following statements about the
the office of governor of a state is not an office of the governor as mentioned in the
employment under the Central government. constitution of India:
It is an independent constitutional office 1. His office is constitutionally under the
and is not under the control of or subordinate control and subordinate to the Central
to the Central government. government.
2. The Governor of a particular state should
Q.931 What do you understand by the dual role of
not belong to that state.
the Governor?
3. The President of India must consult the
(a)  Constitutional and real Executive
Chief Minister of the concerned state
(b) Head of a State and head of government
before making the appointment of the
under certain circumstances
Governor of that state.
(c) Belonging to Central as well as State
Which of these is/are true?
Executive
(a)  1 and 2 (b)  2 and 3
(d) Constitutional ruler and an agent of the
(c)  1 and 3 (d)  None of the above
Centre
Solution: (d)
Solution: (d)

Indian Polity Question Bank P.255

03-Indian Polity_Q928-1069.indd 255 8/7/2018 7:41:51 PM


The governor holds an independent office Which of the above is/are correct?
under the constitution. Options 2 and 3 (a)  1 only (b)  2 only
are conventions and not mentioned in the (c)  Both 1 and 2 (d)  None
constitution. It is solely on the discretion of Solution: (b)
the President on how he wants to appoint Justification: Statement 1: The resignation
the Governor (given some qualifications letter is submitted to thenPresident, and not
mentioned in the constitution of India) the Chief Minister. So, the option 1 is wrong.
Q.934 Which of the following is not an essential Statement 2: A governor holds office for
qualification for appointment as a a term of five years from the date on which
Governor? he enters upon his office. However, this term
(a)  He should be a citizen of India of five years is subject to the pleasure of the
(b) He should be a domicile of the State to President.
which he is being appointed The Supreme Court held that the pleasure
(c) He must have completed the age of of the President is not justifiable. The
35 years governor has no security of tenure and no
(d) He must not be a member of either House fixed term of office. He may be removed by
of Parliament the President at any time without mentioning
Solution: (b) any grounds for his removal.
The Constitution lays down only two Q.936 Consider the following statements about the
qualifications for the appointment of a person removal of the Governor:
as a governor. These are: 1. The constitution does not lay any grounds
1. He should be a citizen of India. for his removal by the President.
2. He should have completed the age of 2. The removal of the governor is not subject
35 years. to judicial review.
Additionally, two conventions have also Which of these is/are true?
developed in this regard over the years. (a)  Only 1 (b)  Only 2
• First, he should be an outsider, that is, he (c)  Both (d)  None of the above
should not belong to the state where he is Solution: (c)
appointed, so that he is free from the local The Governor literally holds office during the
politics. pleasure of the President. His removal is not
• Second, while appointing the governor, needed to be justified by the President and is
the president is required to consult the hence not subject to judicial scrutiny
chief minister of the state concerned. Q.937 Consider the following statements.
So that the smooth functioning of the 1. The Constitution does not lay down any
constitutional machinery in the state is grounds upon which a governor may be
ensured. removed by the President
However, both the conventions have been 2. The President can make such provision as
violated in some of the cases. he thinks fit for the discharge of the functions
of the governor in any contingency not
Terms of Office and Removal provided for in the Constitution.
Which of the above is/are correct?
Q.935 Consider the following about the terms of the (a)  1 only (b)  2 only
Office of the Governor. (c)  Both 1 and 2 (d)  None
1. The Governor can resign at any time by Solution: (c)
addressing a resignation letter to the Chief Justification: Statement 1: This is the reason
Minister of the State. why the office of the Governor is not immune
2. The Constitution does not lay down the to regime change at the Centre. For example,
grounds upon which a governor may be very recently more than a dozen Governors
removed by the President. were asked to resign or removed as the NDA
came to power.

P.256 For Civil Services Preliminary Examination

03-Indian Polity_Q928-1069.indd 256 8/7/2018 7:41:51 PM


Similar things have happened in the 1990s 2. In case a Governor is appointed the
when the Congress government assumed administrator of a Union Territory (UT),
power. he alone has the right to appoint the Chief
Statement 2: He can make such rules. For Minister of the UT.
example, the death of a sitting governor, the Which of the above is/are correct?
chief justice of the concerned state high court (a)  1 only (b)  2 only
may be appointed temporarily to discharge (c)  Both 1 and 2 (d)  None
the functions of the governor of that state. Solution: (d)
The President may also transfer a Justification: Statement 1: The Governor
Governor appointed to one state to another can be made to bear the responsibilities
state for the rest of the term. of more than two or more states or Union
Further, a Governor whose term has territories at the same time. There is no legal
expired may be reappointed in the same state or constitutional limitation on this.
or any other state. Statement 2: It is the President that
Learning: A governor can hold office appoints the Cheif Minister, whether the
beyond his term of five years until his Governor serves as the administrator, or the
successor assumes charge. administrator has been appointed separately
The underlying idea is that there must be by the Central government.
a governor in the state and there cannot be an Q.940 Governor is appointed by the Central
interregnum. government to oversee the functioning of
Q.938 Consider the following about the Office of state governments. Which of the following is
the Governor. NOT such a function?
1. The tenure of his Office is not mentioned (a) Ensuring that stateGovernment works
in the Constitution. within the Constitutional provisions
2. The same person cannot act as Governor (b) Ensuring that laws made by State
for two or more states. legislature are within the constitutional
Which of the above is/are correct? framework
(a)  1 only (b)  2 only (c) Ensuring that State governments do not
(c)  Both 1 and 2 (d)  None disregard constitutional directions given
Solution: (d) by the Central government
Justification: Statement 1: Article 156: (d)  None of the above
Governor is appointed by President and Solution: (d)
hold office during the pleasure of President. Explanation: If a state government does not
It should be also noted that this article also work within the constitutional framework or
mentions tenure of Governor. In this regard if the state government disregards centre’s
it says that Governor shall hold office for directions, the Governor can recommend
the term of 5 years from date he enters to the Centre the imposition of President’s
upon his office. rule in the State. So, options (a)  and (c)  are
Statement 2: Article 153: It says that functions of the governor.
there shall be Governor for each state. But Governor also reserves state bills which
7th Constitutional Amendment Act, 1956 (a) seem ultra vires the constitution;
facilitated the appointment of the same (b) seem to be breach of Centre’s laws;
person as a governor for two or more states. (c)  seems to endanger the position of High
Court etc. He also has the authority to veto a
Condition of Office state legislation. Therefore, option (c)  is also
a function of the Governor.
Q.939 Consider the following statements. Thus, the only option (d) can be the
1. The same person cannot be appointed as answer.
Governor for two or more states at the
same time.

Indian Polity Question Bank P.257

03-Indian Polity_Q928-1069.indd 257 8/7/2018 7:41:51 PM


Q.941 The Constitution lays down which of the person is appointed as the governor of two or
following conditions for the governor’s more states, the emoluments and allowances
office or selection to the Governor’s office? payable to him are shared by the states in such
1. The Governor should not belong to the proportion as determined by the president.
state where is appointed. His emoluments and allowances cannot
2. The Governor’s appointment must be be diminished during his term of office.
made after consulting the chief Minister Q.943 Mark the most correct response:
of the state. (a) No court has power to compel the
3. A serving Governor cannot be a member Governor to exercise or not to exercise
of either House of Parliament or a House any power or to perform or not to perform
of the state legislature. any duty
Select the correct answer using the codes (b) The Governor cannot be prosecuted in
below. a civil and criminal court for any act
(a)  1 and 2 only (b)  3 only of omission or commission during the
(c)  1 and 3 only (d)  2 and 3 only period he holds office
Solution: (b) (c)  Both above statements are correct
Justification: Statements 1 and 2: These (d)  statement (a) is correct while (b)  is not
are conventions rather than constitutional Solution: (d)
provisions. Writ of Mandamus cannot be issued against
The Constitution lays down only two governor and president. Hence the option (a)
qualifications for the appointment of a person is right.
as a governor. Like the President, the governor is
These are: also entitled to a number of privileges and
• He should be a citizen of India. immunities to preserve the sanctity of the
• He should have completed the age of post.
35 years. • He enjoys personal immunity from legal
Statement 3: Additionally, two conventions liability for his official acts.
have also developed in this regard over the • During his term of office, he is immune from
years. any criminal proceedings, even in respect of
First, he should be an outsider, that is, he his personal acts.
should not belong to the state where he is • He cannot be arrested or imprisoned.
appointed, so that he is free from the local • However, after giving two months’ notice,
politics. civil proceedings can be instituted against
Second, while appointing the governor, him during his term of office in respect of
the president is required to consult the chief his personal acts.
minister of the state concerned, so that the
smooth functioning of the constitutional
machinery in the state is ensured.
Powers and Functions
However, both the conventions have been
violated in some of the cases (1) Executive Power
Q.942 The salary and allowances of the Governor Q.944 Who acts as the Chancellor of State
are charged to: Universities?
(a)  Consolidated Fund of the State (a) Governor
(b)  Consolidated Fund of India (b)  Chief Minister
(c)  Contingency Fund of India (c)  Chief Justice of High Court
(d)  ‘a’ and ‘b’ in equal proportion (d) President
Solution: (a) Solution: (a)
He is entitled to such emoluments, He acts as the chancellor of universities in the
allowances and privileges as may be state. He also appoints the vice-chancellors
determined by Parliament. When the same of universities in the state.

P.258 For Civil Services Preliminary Examination

03-Indian Polity_Q928-1069.indd 258 8/7/2018 7:41:51 PM


Q.945 The Governor requires advice of the CM in As they are appointed on the advice of
appointing which of the following important CM, they are to be dismissed on the advice
officials? of the CM.
1. State election Commissioner Q.947 Which of the following is/are the executive
2. Advocate General powers and functions of the Governor?
3. Chief Justice of High Court 1. All executive actions of the government
4. Chairman, State Public Service Commission of a state are formally taken in his name.
Choose the correct answer using the codes 2. He can make rules for more convenient
below. transaction of the business of a state
(a)  1, 2 and 3 only (b)  1, 2 and 4 only government and for the allocation among
(c)  3 and 4 only (d)  1 and 2 only the ministers of the said business.
Solution: (b) 3. He appoints the chief minister and other
The Chief Minister enjoys the following powers ministers.
in relation to the governor: 4. He has the power to both appoint and
• He is the principal channel of remove the state election commissioner.
communication between the governor 5. He acts as the chancellor of universities in
and the council of ministers. It is the duty the state.
of the Chief Minister: 6. He can impose Governor’s rule in the State
• to communicate to the Governor of the based on a report of State Secretariat.
state all decisions of the council of Select the correct answer using the codes
ministers relating to the administration of below.
the affairs of the state and proposals for (a)  1, 2, 3, 4 and 5 only
legislation; (b)  1, 2, 3 and 4 and 6 only
• to furnish such information relating to (c)  4, 5 and 6 only
the administration of the affairs of the (d)  1, 2, 3 and 5 only
state and proposals for legislation as the Solution: (d)
governor may call for; and Justification: Statement 1: He can make
• if the governor so requires, to submit rules specifying the manner in which the
for the consideration of the council of Orders and other instruments made and
ministers any matter on which a decision executed in his name shall be authenticated.
has been taken by a minister but which Statement 2 and 3: He not only allocated
has not been considered by the council. business, but also appoints the chief minister
• He advises the governor with regard to the and other ministers.
appointment of important officials like Statement 4: He appoints the state election
advocate general, chairman and members commissioner and determines his conditions
of the state public service commission, of service and tenure of office. However, the
state election commissioner, and so on. state election commissioner can be removed
Q.946 As per the Constitution, the state ministers only in like manner and on the like grounds
hold office during the pleasure of the as a judge of a high court, and not by the
Governor. The words “during the pleasure of Governor.
the Governor” actually implies pleasure of the Statement 5: He also appoints the vice-
(a) President chancellors of universities in the state.
(b)  Chief Justice of India Statement 6: In the state of Jammu and
(c)  Chief Minister Kashmir, failure of constitutional machinery
(d)  Legislative Assembly results in Governor’s rule, imposed by invoking
Solution: (c) Section 92 of Constitution of Jammu and
Justification: The Chief Minister shall be Kashmir. But,
appointed by the Governor and the other (a) the proclamation can be issued by the
Ministers shall be appointed by the Governor state’s Governor after obtaining the consent
on the advice of the Chief Minister. of the President of India and

Indian Polity Question Bank P.259

03-Indian Polity_Q928-1069.indd 259 8/7/2018 7:41:51 PM


(b) Governor’s rule is not applicable to Chief justice of a high court is appointed
Indian states except J&K. So, this by the President.
statement would be incorrect. The governor has the authority to remove
Q.948 Which of these constitutional functionaries is the CM but only when he has lost the
NOT appointed by the Governor? confidence of the legislative assembly of the
(a) State election commissioner as he is state.
appointed by the President Q.950 Consider the following statements about
(b) Advocate general of a state who is the executive power of the governor as
appointed by the State Minister of Law mentioned below.
and Justice with the concurrence of the 1. He can act even without the aid and advice
Chief Minister of the council of ministers.
(c) Chairman and members of the state 2. According to the 42nd Constitutional
public service commission as they are Amendment, the ministerial advice has
appointed and removed by the President been made binding on a lot of matters on
(d)  None of the above the governor also along with the President.
Solution: (d) 3. He has special responsibilities for the
Justification: Option A: He appoints the administration of some of the backward/
advocate general of a state and determines special areas in the state.
his remuneration. The advocate general holds Which of these is/are true?
office during the pleasure of the governor. (a)  1 and 2 (b)  2 and 3
Option B: He appoints the state election (c)  1 and 3 (d)  All of the above
commissioner and determines his conditions Solution: (c)
of service and tenure of office. However, the 42nd amendment made ministerial advice
state election commissioner can be removed binding only on the President of India.
only in like manner and on the like grounds Special areas and the ones under fifth
as a judge of a high court. and sixth schedule are under the special
Option C: He appoints the chairman responsibility of the Governor. At times
and members of the state public service he needs Presidential assent in such
commission. However, they can be removed administration.
only by the president and not by a governor.
Q.949 As per the constitution, the Governor has the (2) Legislative Powers
authority to make appointments, but cannot
Q.951 A governor is an integral part of the state
remove which of the following from their
legislature. In that capacity, he has which of the
posts?
following legislative powers and functions?
1. Chief Justice of High Court
1. He can summon or prorogue the state
2. Chief Minister
legislature and dissolve the state
3. Advocate general of the state
legislative assembly.
4. State Election Commissioner
2. He can address the state legislature at the
Choose the correct option from the codes
commencement of the first session after
below:
each general election and the first session
(a)  2, 3 and 4 (b)  2 and 3 only
of each year.
(c)  1, 3 and 4 (d)  Only 4
3. He can send messages to the houses of
Solution: (d)
the state legislature, with respect to a bill
While the governor can appoint the state
pending in the legislature or otherwise.
election commissioner, he can only be
4. He can appoint any member of the State
removed in the same manner as that of a
legislative assembly to preside over its
high court judge. That is the Governor cannot
proceedings when the offices of both
remove him.
the Speaker and the Deputy Speaker fall
Advocate general holds office during the
vacant.
pleasure of the Governor.

P.260 For Civil Services Preliminary Examination

03-Indian Polity_Q928-1069.indd 260 8/7/2018 7:41:51 PM


Select the correct answer using the codes Q.953 When an ordinary bill is sent to the governor
below. after it is passed by state legislature, he cannot
(a)  1 and 2 only (b)  2, 3 and 4 only (a) Reserve the bill for the consideration of
(c)  1 and 3 only (d)  1, 2, 3 and 4 the president
Solution: (d) (b)  Withhold his assent to the bill
Justification: Other Legislative powers of (c) Return the bill for reconsideration of the
the Governor include: state legislature
• He can appoint any member of the state (d) Amend the bill and send it the Council
legislature council to preside over its of Ministers for re-introduction in the
proceedings when the offices of both assembly
Chairman and Deputy Chairman fall Solution: (d)
vacant. Learning: He cannot amend the bill on his
• He nominates one-sixth of the members of own, so D is wrong.
the state legislative council from amongst If the bill resent to the legislature is passed
persons having special knowledge or again by it with or without amendments, the
practical experience in literature, science, governor has to give his assent to the bill.
art, cooperative movement and social Where the bill passed by the state legislature
service. endangers the position of the state high court
• He can nominate one member to the state or ultra vires the constitution, the Governor
legislature assembly from the Anglo- must reserve the bill for the President.
Indian Community.
• He decides on the question of Reserving State Bill for President of
disqualification of members of the state
legislature in consultation with the
India’s Consideration
Election Commission. Q.954 In which of the following cases can the
Q.952 When a bill is sent to the Governor after it is Governor reserve a State bill for the approval
passed by state legislature, he can of the President?
1. Withhold his assent to the bill 1. During a financial emergency
2. Return the bill, including a money bill, for 2. If the bill restricts inter-state free trade
reconsideration of the state legislature and commerce
3. Reserve the bill for the consideration of 3. If the Central Cabinet specifies in a written
the President order so.
Select the correct answer using the codes Choose the correct answer using the codes
below. below.
(a)  1 and 2 only (b)  2 and 3 only (a)  1 and 2 only (b)  2 and 3 only
(c)  1 and 3 only (d)  1 only (c)  1 and 3 only (d)  All of the above
Solution: (c) Solution: (a)
Justification: Statement 2: A money bill The Constitution empowers the Centre to
cannot be sent back for the reconsideration exercise control over the state’s legislative
of the legislature. This is because it is matters in the following ways:
introduced in the legislature only after the (i)  The governor can reserve certain types
prior recommendation of the Governor. of bills passed by the state legislature for
Statement 3: The Governor reserves the the consideration of the President. The
bill for the consideration of the president. president enjoys absolute veto over them.
In one case such reservation is obligatory, (ii) Bills on certain matters enumerated in
that is, where the bill passed by the state the State List can be introduced in the
legislature endangers the position of the state state legislature only with the previous
high court. In addition, the governor can sanction of the president. (For example,
also reserve the bill if it is ultra-vires, that is, the bills imposing restrictions on the
against the provisions of the constitution. freedom of trade and commerce).

Indian Polity Question Bank P.261

03-Indian Polity_Q928-1069.indd 261 8/7/2018 7:41:51 PM


(iii) The President can direct the states to Select the correct answer using the codes
reserve money bills and other financial below.
bills passed by the state legislature for (a)  1 and 4 only (b)  1, 2 and 3 only
his consideration during a financial (c)  2, 3 and 4 only (d)  1, 2, 3 and 4
emergency. Solution: (d)
Q.955 In which of the following cases is it mandatory Q.958 When the governor reserves a bill for the
for the Governor to refer the bill passed by consideration of the President
the state assembly to the President? (a) The bill cannot be returned by
1. If the bill endangers the position of the the President or Governor for the
state high court. reconsideration of the legislature
2. If it is opposed to the Directive Principles (b) Assent of the Governor is no longer
of State Policy. required if the President approves the bill
3. If the bill falls under the concurrent list. (c) The President must give assent to the bill
Which of these is/are true? (d) The bill is sent to the Parliament for
(a)  1 and 2 (b)  2 and 3 amendment and approval
(c)  1 and 3 (d)  All of the above Solution: (b)
Solution: (a) Justification: Option A: If the bill is retuned
If a bill falls in the concurrent list, and if the by the President for the reconsideration of the
Governor feels that the bill violates some House or Houses and is passed again, the bill
of the provisions of the Central law on the must be presented again for the presidential
same subject, then it can be referred to the assent only.
President. Option B: If the President gives his assent to
Q.956 State Bills relating to which of the following the bill, it becomes an act. This means that the
categories can be reserved for Presidential assent of the Governor is no longer required.
assent by the Governor? Option C: There is no such requirement;
1. Bills that impinge on the fundamental only if it is a money bill, it can’t be sent for
rights of citizens reconsideration of the legislature.
2. Bills that prima facie appear ultra vires
the constitution Ordinance Making Power
3. Bills that contravene Central laws Q.959 Consider the following statements about
Choose the correct answer using the codes ordinances in states and Union Territories
below. (UTs):
(a)  1 and 2 only (b)  2 and 3 only 1. The Lieutenant Governor of a UT has
(c)  1 and 3 only (d)  All of the above the power to issue ordinance only in the
Solution: (d) recess of the assembly and only after
Any Bill passed by an Assembly on issues receiving the prior assent of the President.
contravening Central laws needs Presidential 2. The Governor cannot issue an ordinance
assent. without the assent of the President in
Q.957 The governor can reserve a bill passed by the certain cases.
State Legislature for the consideration of the Which of these is/are true?
President in which of the following cases? (a)  only 1 (b)  Only 2
1. If it is against the provisions of the (c) Both (d) None
Constitution. Solution: (c)
2. If it is opposed to the Directive Principles Both the statements are true.
of State Policy. The cases when the governor requires
3. If it is against the larger interest of the President’s assent are:
country or of grave national importance. (a) If the bill would have required the
4. If it endangers the position of the State permission of the President if it were to
High Court be enacted in the assembly.

P.262 For Civil Services Preliminary Examination

03-Indian Polity_Q928-1069.indd 262 8/7/2018 7:41:51 PM


(b) If such a bill would have to be reserved Learning: Statement 1: He lays the reports
for the consideration of the President. of the State Finance Commission, the
(c) In an act of state legislature would State Public Service Commission and the
have been invalid without receiving Comptroller and Auditor-General relating
President’s assent. to the accounts of the state, before the state
Q.960 The Governor cannot make an ordinance legislature.
without the instructions from the President in Statement 2: This is similar to the powers
which of the following cases? of the President at the Centre.
1. If the subject of ordinance falls in the Statement 3: Further, Money bills can be
concurrent list introduced in the state legislature only with
2. If the same act of the state legislature his prior recommendation.
would have been invalid without receiving No demand for a grant can be made except
the President’s assent on his recommendation.
Which of the above is/are correct? Q.962 Consider the following statements about the
(a)  1 only (b)  2 only Powers of the Governor:
(c)  Both 1 and 2 (d)  None 1. He holds the Contingency fund of the
Solution: (b) state.
Justification: Statement 1: Only if a Central 2. He establishes the State Finance
law exists on the matter in the concurrent commission as per part VI of the
list, the Governor may require the prior constitution.
instruction of the President. 3. Unlike the President in the Lok Sabha, he
Statement 2: Moreover, he had to take cannot nominate members of the Anglo-
President’s instructions if a bill containing Indian community to the legislative
the same provisions would have required assembly of the state.
the previous sanction of the President for its Choose the correct answer using the codes
introduction into the state legislature. below:
(a)  1 and 2 (b)  2 and 3
(3) Financial Powers (c)  Only 1 (d)  1 and 3
Solution: (c)
Q.961 Which of the following statements with He nominates only one member of the
regard to the financial role of the Governor in Anglo-Indian community.
a state is correct? The state finance commissions were
1. He lays down the report of Comptroller and added by the 73rd amendment and are hence
Auditor-General relating to the accounts of mentioned in the Part IX of the constitution.
the state, before the state legislature. The governor establishes it.
2. He can make advances out of the
Contingency Fund of the state to meet any
unforeseen expenditure.
(4) Judicial Powers
3. He sees that the Annual Financial Q.963 The judicial powers and functions of the
Statement of the state is laid before the Governor include which of the following?
state legislature. 1. He is consulted by the president while
4. He constitutes a state finance commission appointing the judges of the concerned
after every five years to review the state high court.
financial position of the panchayats and 2. He can commute the sentence of any
the municipalities. person convicted of any offence against
Select the correct answer using the codes any law operating in India.
below. Which of the above is/are correct?
(a)  3 and 4 only (b)  1 and 3 only (a)  1 only (b)  2 only
(c)  1, 2 and 4 only (d)  1, 2, 3 and 4 (c)  Both 1 and 2 (d)  None
Solution: (d) Solution: (a)

Indian Polity Question Bank P.263

03-Indian Polity_Q928-1069.indd 263 8/7/2018 7:41:51 PM


Justification and Learning: Statement (a)  Governor (b)  Chief Minister
1: He also makes appointments, postings (c)  State High Court (d)  President
and promotions of the district judges in Solution: (a)
consultation with the state high court. If any question arises whether a matter falls
Moreover, he also appoints persons to within the governor’s discretion or not,
the judicial service of the state (other than the decision of the governor is final and the
district judges) in consultation with the validity of anything done by him cannot be
state high court and the State Public Service called in question on the ground that he ought
Commission. or ought not to have acted in his discretion.
Statement 2: He can grant pardons, Further, the nature of advice tendered by
remissions etc of punishment or suspend, ministers to the governor cannot be enquired
remit and commute the sentence of any person by any court. This provision emphasises the
convicted of any offence against any law intimate and the confidential relationship
relating to a matter to which the executive between the governor and the ministers.
power of the state extends. This isn’t applicable In 1971, the Supreme Court ruled that
to Central laws. So, 2 is wrong. a council of ministers must always exist
to advise the governor, even after the
Constitutional Position dissolution of the state legislative assembly
or resignation of a council of ministers.
Q.964 Consider the following about state
Q.966 The Council of Ministers aids and advises
administration.
the Governor in the exercise of his functions,
1. Governor of the State is appointed
except in so far as he is required to act in his
by the Central Government but the
discretion. If any question arises whether a
State Assembly may disapprove her
matter falls within the Governor’s discretion
appointment in effect repealing it.
or not, whose decision shall be final and why?
2. Governor of the state appoints the chief
(a) President of India as he is the head of
minister and other ministers only after
the Indian state and responsible for the
obtaining the advice of the Central
administration of the States
Council of Ministers.
(b) Governor of the State since the
Which of the above is/are correct?
Constitution confers him this authority
(a)  1 only (b)  2 only
(c) State Legislature as it is the supreme
(c)  Both 1 and 2 (d)  None
law-making institution within the State
Solution: (d)
(d) Chief Minister as he heads the Council
Justification: Statement 1: Governor is
of Ministers
appointed by the President. State has no say
Solution: (b)
in either the appointment or removal of the
Justification: If any question arises whether
Governor. So, 1 is wrong.
a matter falls within the Governor’s discretion
Statement 2: Governor doesn’t consult
or not, decision of the Governor shall be
the Union government in making state
final, and the validity of anything done by the
appointments, even though the Governor is an
Governor shall not be called in question on
agent of the Centre.
the ground that he ought or ought not to have
She is bound to choose the leader of the
acted in his discretion.
largest party/coalition as the CM and others
Also, the constitution says that the advice
recommended by the CM as ministers.
tendered by Ministers to the Governor
Q.965 Article 163 provides for a council of ministers shall not be inquired into in any court.
with the chief minister at the head to aid and This provision emphasises the intimate and
advise the governor in the exercise of his the confidential relationship between the
functions except the discretionary ones. If governor and the ministers.
any question arises whether a matter falls
within the governor’s discretion or not, the
final decision lies with the

P.264 For Civil Services Preliminary Examination

03-Indian Polity_Q928-1069.indd 264 8/7/2018 7:41:51 PM


Q.967 In which of the following cases, the Governor (b) Recommendation for the imposition of
can act on his own discretion without the the President’s Rule in the state.
advice of Council of Ministers? (c) While exercising his functions as the
1. Recommend President’s rule in the state. administrator of an adjoining union
2. Reserve certain bills for the consideration territory (in case of additional charge).
of the President. (d) Determining the amount payable by
3. Administering a Union territory which he the Government of Assam, Meghalaya,
has been authorized to in addition to that Tripura and Mizoram to an autonomous
particular state. Tribal District Council as royalty
Choose the correct answer using the codes accruing from licenses for mineral
below: exploration.
(a)  1 and 2 (b)  2 and 3 (e) Seeking information from the chief
(c)  1 and 3 (d)  All of the above minister with regard to the administrative
Solution: (d) and legislative matters of the state.
The constitution also makes clear that In addition to the above constitutional
wherever a situation arises as to whether a discretion (i.e., the express discretion
matter falls within the Governor’s discretion mentioned in the Constitution), the governor,
or not, the decision of the Governor will be like the president, also has situational
final and the validity of anything done by him discretion. Please see the Q source for details.
cannot be questioned on the ground that he Q.969 Consider the following statements.
ought not have acted in his discretion. Assertion (A): The Governor exercises more
Q.968 The Governor has constitutional or situational discretion that the President does in their
discretion in which of the following cases? respective spheres of influences.
1. Seeking information from the chief Reason (R): The constitutional provisions
minister with regard to the administrative explicitly limit the overall discretion of the
and legislative matters of the state President, which is not so clearly defined for
2. Approving any ordinary bill that falls the Governor.
under the concurrent list In the context of the above, which of these
3. Recommendation for the imposition of is correct?
the President’s Rule in the state (a) A is correct, and R is an appropriate
4. Exercising his functions as the explanation of A.
administrator of an adjoining union (b) A is correct, but R is not an appropriate
territory in case of additional charge explanation of A.
Select the correct answer using the codes (c)  A is incorrect, but R is correct.
below. (d)  Both A and R are incorrect.
(a)  1, 2 and 3 only (b)  1 and 4 only Solution: (a)
(c)  1, 3 and 4 only (d)  2, 3 and 4 only Justification: Article 74(1) clearly prescribes
Solution: (c) that the President is bound by the advice of
Justification: The Constitution makes it clear the council of Ministers. However, in the
that if any question arises whether a matter case of Governor, the respective provision
falls within the governor’s discretion or not, does not make the advice of the council of
the decision of the governor is final and the ministers in state binding on him.
validity of anything done by him cannot be Article 163 is clear in this regard:
called in question on the ground that he ought 1. There shall be a Council of Ministers
or ought not to have acted in his discretion. with the Chief Minister at the head to aid
The governor has constitutional discretion and advise the Governor in the exercise
in the following cases: of his function, except in so far, as he is
(a) Reservation of a bill for the consideration by or under this Constitution required to
of the President. exercise his functions or any of them in
his discretion.

Indian Polity Question Bank P.265

03-Indian Polity_Q928-1069.indd 265 8/7/2018 7:41:51 PM


2. If any question arises whether any Q.971 Consider the following statements in relation
matter is or is not a matter as respects to the office of the Chief Minister of a state:
which the Governor is by or under 1. His term of office is not fixed and he can
this Constitution required to act in his be dismissed anytime by the governor.
discretion, the decision of the Governor 2. The ministers in the states can also be
in his discretion shall be final, and dismissed by the governor anytime at his
the validity of anything done by the will.
Governor shall not be called in question Which of these is/are true?
on the ground that he ought or ought not (a)  Only 1 (b)  Only 2
to have acted in his discretion. (c)  Both (d)  None of the above
Solution: (d)
Until the council of ministers in the state has
(24) CHIEF MINISTER the majority support in the state legislature,
Appointment no member of it can be dismissed by the
Q.970 Consider the following statements. governor except on the advice of the CM.
1. The Chief Minister (CM) may be a member He himself can only be dismissed only after
of any of the two Houses of a state losing majority.
legislature. Q.972 Consider the following statements.
2. The CM may be dismissed by the Assertion (A): The Constitution requires
Governor even if he enjoys majority that a person must prove his majority in the
support in state legislature legislative assembly before he is appointed as
Which of the above is/are true? the Chief Minister.
(a)  1 only (b)  2 only Reason (R): Only a person who is a member
(c)  Both 1 and 2 (d)  None of the legislative assembly can be appointed
Solution: (a) as Chief Minister.
A person who is not a member of the state In the context of the above, which of these
legislature can be appointed as Chief is correct?
Minister for six months, within which time, (a) A is correct, and R is an appropriate
he should be elected to the state legislature, explanation of A.
failing which he ceases to be the Chief (b) A is correct, but R is not an appropriate
Minister. According to the Constitution, explanation of A.
the Chief Minister may be a member of any (c)  A is correct, but R is incorrect.
of the two Houses of a state legislature. (d)  Both A and R are incorrect.
Usually Chief Ministers have been Solution: (d)
selected from the Lower House (legislative Justification: The governor may first appoint
assembly), but, on a number of occasions, him as the Chief Minister and then ask him to
a member of the Upper House (legislative prove his majority in the legislative assembly
council) has also been appointed as Chief within a reasonable period. This is what has
Minister. been done in a number of cases. So, A is
The term of the Chief Minister is not fixed incorrect.
and he holds office during the pleasure of the A person who is not a member of the
governor. However, this does not mean that state legislature can be appointed as Chief
the governor can dismiss him at any time. Minister for six months, within which time,
He cannot be dismissed by the governor as he should be elected to the state legislature,
long as he enjoys the majority support in failing which he ceases to be the Chief
the legislative assembly. But, if he loses the Minister. So, R is wrong too.
confidence of the assembly, he must resign or According to the Constitution, the Chief
the governor can dismiss him. Minister may be a member of any of the two
Houses of a state legislature.

P.266 For Civil Services Preliminary Examination

03-Indian Polity_Q928-1069.indd 266 8/7/2018 7:41:51 PM


Q.973 A person who is not a member of the state Like at the Centre, in the states too, the council
legislature can be appointed as Chief Minister of ministers consists of three categories of
and serve a five-year term only if ministers, namely, cabinet ministers, ministers
(a) The prior consent of the Governor of the of state, and deputy ministers. The difference
State is taken between them lies in their respective ranks,
(b) The President agrees to the same on the emoluments, and political importance. At
advice of the Union Cabinet the top of all these ministers stands the chief
(c) S/he had served in administrative minister—supreme governing authority in
capacities for durations as specified by the state.
the State laws The Constitution does not specify the
(d) S/he is elected to the State legislature size of the state council of ministers or the
within a time of six months ranking of ministers. They are determined by
Solution: (d) the chief minister according to the exigencies
Justification: S/he can be indirectly elected of the time and requirements of the situation.
to the legislative council too and remain the Q.976 Resignation or dissolution of which of the
Chief Minister. If s/he fails to get elected to following may bring a collapse of the Council
the State legislature, he will cease to be the of Ministers at the state level?
Chief Minister of the State. For example, 1. Governor
Bansilal and S. B. Chavan were appointed as 2. Chief Minister
Chief Ministers of Haryana and Maharashtra 3. State Legislative assembly
respectively, even though they were not 4. State Legislative Council
members of the state legislature. Subsequently, Choose the correct answer using the codes
they were elected to the state legislature. below.
Similar is true for the Prime Minister too. (a)  2, 3 and 4 only (b)  2 and 3 only
He must become a MP within six months. (c)  1, 2 and 4 only (d)  3 only
Q.974 The office of Deputy Chief Minister has been Solution: (b)
created by Since the Chief Minister is the head of the
(a)  The Constitution of India council of ministers, his resignation or
(b)  Parliamentary Legislation death automatically dissolves the council of
(c)  State-specific legislations ministers. The resignation or death of any
(d)  None of the above other minister, on the other hand, merely
Solution: (d) creates a vacancy, which the Chief Minister
Justification: At times, the council of may or may not like to fill.
ministers may also include a deputy chief Even though the ministers may be
minister. For example, Andhra Pradesh had appointed from any of the houses, only by the
the office of deputy chief minister till 1956. dissolution of the assembly does the council
This post was created in West Bengal in 1967. of minister dissolves. The Legislative council
The deputy chief ministers are appointed is a permanent body is never dissolved.
mostly for local political reasons. Q.977 Consider the following statements.
Assertion (A): Chief Minister of the State
Powers and Functions can disqualify a sitting MLA after obtaining
the advice of the State Election Commission.
Q.975 The size of the state council of ministers
Reason (R): Chief Minister holds the
and the ranking of ministers is specified and
de facto authority in the State Government.
determined by the
In the context of the above, which of these
(a)  Constitution of India
is correct?
(b)  Chief Minister
(a) A is correct, and R is an appropriate
(c) Governor
explanation of A.
(d) Laws enacted by the State Legislative
Assembly
Solution: (b)

Indian Polity Question Bank P.267

03-Indian Polity_Q928-1069.indd 267 8/7/2018 7:41:52 PM


(b) A is correct, but R is not an appropriate Q.979 In relation to the Council of Ministers (CoM)
explanation of A. of Centre and State, which of the following
(c)  A is incorrect, but R is correct. powers are enjoyed by both the President and
(d)  A is correct, but R is incorrect. the Governor respectively?
Solution: (c) 1. Sending a decision of the CoM for
Justification: An MLA can be disqualified re-consideration
on many grounds. 2. Dismissing the CoM
• On grounds of defection (to be covered 3. Getting the CoM to furnish desired
later in tests), the Speaker decides the information to Governor/President
disqualification. Choose the correct answer using the codes
• If he holds any office of profit under the below.
Government of India or a state or an office (a)  1 and 2 only (b)  2 and 3 only
declared by a law of the state, he can be (c)  1 and 3 only (d)  All of the above
disqualified. Solution: (d)
• If any competent court declares any Article 164:
member to be of unsound mind, then the • The Chief Minister shall be appointed by
MLA is disqualified. the governor and other ministers shall be
• Under Article 192 of the Constitution, “if appointed by the governor on the advice
any question arises as to whether or not of the Chief Minister;
the member of a house of the legislature • The ministers shall hold office during the
of a state has become subject to any of the pleasure of the governor; and
disqualification criteria, the question shall • The council of ministers shall be
be cited to the Governor of the State for collectively responsible to the legislative
decision who will act as per the opinion of assembly of the state.
the Election Commission (EC).” Article 167: It shall be the duty of the Chief
• His decision shall be final and may not be Minister:
reviewed by any court of law. So, clearly 1. to communicate to the governor of the
A is wrong. state all decisions of the council of
ministers relating to the administration of
the affairs of the state and proposals for
(25) STATE COUNCIL OF MINISTERS legislation;
Q.978 Consider the following statements about the 2. to furnish such information relating to
council of ministers in the states. the administration of the affairs of the
1. It can advise the governor even after the state and proposals for legislation as the
resignation of the council of ministers. governor may call for; and
2. In case of a dispute of jurisdiction between 3. if the governor so requires, to submit
the council of ministers and the governor, for the consideration of the council of
the decision of the former is final and ministers any matter on which a decision
binding. has been taken by a minister but which
Which of these is/are true? has not been considered by the council.
(a)  Only 1 (b)  Only 2
(c)  Both (d)  None of the above (26) STATE LEGISLATURE
Solution: (a)
The governor in the states has much more Q.980 Consider the following statements:
discretion as compared to the President at the 1. State governments in India draw their
Centre. He is not bound by the aid and advice authority from the Central government.
of the council of ministers. 2. State governments in India are subordinate
The council can advise him even after to the Central government.
resignation to maintain political continuity.

P.268 For Civil Services Preliminary Examination

03-Indian Polity_Q928-1069.indd 268 8/7/2018 7:41:52 PM


Which of these is/are true? (a)  1 and 3 (b)  3 and 5
(a)  1 only (b)  2 only (c)  5 and 6 (d)  only 6
(c) Both (d) None Solution: (d)
Solution: (d) As of now only seven states have bi-cameral
State governments are not mere agents legislature. Earlier Tamil Nadu, Punjab and
of Central government. They derive their west Bengal had Legislative Councils which
authority from the Indian constitution. They were abolished in 1969 along with Punjab.
are not subordinate to the Centre. Instead, The Legislative Council of Tamil Nadu was
the Centre is more powerful than the state abolished in 1986.
governments to maintain the unity and
integrity of the Indian union. Legislative Assemblies
Q.981 States that have a bicameral legislature in
Q.983 Consider the following with respect to the
India are?
composition of legislative assemblies in states.
1. Jharkhand
1. The constitution does not prescribe any
2. Karnataka
minimum strength of the assembly.
3. Gujarat
2. The constitution provides that all members
4. Uttar Pradesh
of legislative assembly for all states must
Select the correct answer using the codes
be either elected directly or nominated by
below.
the Governor.
(a)  1 and 4 only (b)  2 and 3 only
Which of the above is/are correct?
(c)  2 and 4 only (d)  1, 2, 3 and 4
(a)  1 only (b)  2 only
Solution: (c)
(c)  Both 1 and 2 (d)  None
Justification: Seven Indian States, Andhra
Solution: (d)
Pradesh, Telangana, Bihar, Jammu-Kashmir,
Justification: Statement 1: The legislative
Karnataka, Maharashtra and Uttar Pradesh,
assembly consists of representatives directly
have bicameral Legislatures, these are called
elected by the people on the basis of universal
legislative councils (Vidhan Parishad).
adult franchise.
A bicameral legislature makes it possible
Its maximum strength is fixed at 500 and
to have every decision reconsidered.
minimum strength at 60. It means that its
Learning: The Constitution provides for
strength varies from 60 to 500 depending on
the abolition of the second chamber in a state
the population size of the state. So, 1 is wrong.
where it exists as well as for the creation of
However, in case of Arunachal Pradesh,
such a chamber in a state where there is none
Sikkim and Goa, the minimum number
at present.
is fixed at 30 and in case of Mizoram and
If a State Legislature passes a resolution
Nagaland, it is 40 and 46 respectively.
by an absolute majority, together with not
Statement 2: Further, some members
less than two-thirds of the members actually
of the legislative assemblies in Sikkim and
present and voting in favour of the creation
Nagaland are also elected indirectly.
of the second chamber and if Parliament
However, in other cases either members
gives concurrence to such a resolution, the
are directly elected or nominated (i.e.
concerned State can have two Houses in the
Anglo-Indians)
Legislature.
Q.984 Consider the following statements.
Q.982 Which of these States previously had
Assertion (A): The minimum and maximum
Legislative Councils?
strength of the Legislative assembly of each
1. Andhra Pradesh
state is fixed by the State law.
2. Gujarat
Reason (R): The legislative assembly
3. Kerala
mainly consists of representatives directly
4. Manipur
elected by the people on the basis of universal
5. West Bengal
adult franchise.
6. Tamil Nadu

Indian Polity Question Bank P.269

03-Indian Polity_Q928-1069.indd 269 8/7/2018 7:41:52 PM


In the context of the above, which of these of elders), while the legislative assembly
is correct? (Vidhan Sabha) is the lower house (first
(a) A is correct, and R is an appropriate chamber or popular house). So, here the R
explanation of A. is correct.
(b) A is correct, but R is not an appropriate
explanation of A. Legislative Councils
(c)  A is correct, but R is incorrect.
(d)  A is incorrect, but R is correct. Q.987 The Legislative Council of a State:
Solution: (d) I. is not subject to dissolution.
II. can be abolished by the State Legislative
Q.985 Membership of the legislative Assembly can
Assembly.
vary between 60 and 500, but the exception
III. can be abolished by the President on the
to this rule is/are found in:
Governor’s recommendation.
I. Puducherry
(a)  I only (b)  II only
II.  Mizoram
(c)  I and II (d)  III only
III. Goa
Solution: (c)
(a)  I and II (b)  II and III
Like the Rajya Sabha, the legislative
(c)  II only (d)  I, II and III
council is a continuing chamber, that is, it
Solution: (d)
is a permanent body and is not subject to
Though constitution prescribes minimum
dissolution. But, one-third of its members
and maximum strength of the legislative
retire on the expiration of every second
assembly there are some expectations.
year. So, a member continues as such for six
Sikkim – 32 – Article 371F: 36th
years. The vacant seats are filled up by fresh
Constitutional Amendment
elections and nominations (by governor) at
Mizoram – 40 – Article 371G 53rd
the beginning of every third year. The retiring
Constitutional Amendment
members are also eligible for re-election and
Goa – 40 – Article 371I – 56th
re-nomination any number of times.
Constitutional Amendment
The idea of having a second chamber in
Puducherry – 30 – Article 239A
the states was criticised in the Constituent
Q.986 Consider the following statements. Assembly on the ground that it was not
Assertion (A): All the members of the representative of the people, that it delayed
Legislative Assembly of any state must be legislative process and that it was an expensive
elected directly. institution. Consequently the provision
Reason (R): Legislative assembly is the was made for the abolition or creation of a
popular house. legislative council to enable a state to have a
In the context of the above, which of these second chamber or not according to its own
is correct? willingness and financial strength.
(a) A is correct, and R is an appropriate
explanation of A. Constitution of the house
(b) A is correct, but R is not an appropriate
explanation of A. Q.988 Which of the following is the authority
(c)  A is incorrect, but R is correct. to create or abolish the state legislative
(d)  Both A and R are incorrect. councils?
Solution: (c) (a)  President of India
Justification: Some members of the (b)  Union Council of Ministers
legislative assemblies in Sikkim and (c) Parliament
Nagaland are elected indirectly. In most (d)  Governor of the Concerned State
states, however, they are elected directly. Solution: (c)
So, A is incorrect. Learning: The question in hand shows the
The legislative council (Vidhan Parishad) unitary tendencies of the Indian federation-
is the upper house (second chamber or house cum-Union. The Parliament can create or

P.270 For Civil Services Preliminary Examination

03-Indian Polity_Q928-1069.indd 270 8/7/2018 7:41:52 PM


abolish the state legislative councils on the can be abolished by the Parliament on
recommendation of the concerned state the recommendation of the assembly.
legislative assemblies. It is notable that these The position of the council vis-à-vis the
councils are created by the Parliament too. It assembly is much weaker than the position
can also increase or decrease the area, alter of the Rajya Sabha vis-à-vis the Lok Sabha.
the boundaries and change the names of The council is subordinate to the assembly
states of the Indian Union. in all respects.
Q.989 The Parliament can abolish a legislative Q.991 Consider the following statements.
council only 1. The Constitution fixes the maximum
(a) if the legislative assembly of the and the minimum size of the Legislative
concerned state passes an ordinary bill to Council.
that effect 2. The size of the council depends on the size
(b) if the Parliament approves the resolution of the assembly of the concerned state.
by a simple majority, which had been Which of the above is/are correct?
passed by special majority in the state (a)  1 only (b)  2 only
assemblies (c)  Both 1 and 2 (d)  None
(c) if the President approves so after Solution: (c)
receiving a written report from the Justification: The maximum strength of
Governor of the state the council is fixed at one-third of the total
(d) if a constitutional amendment act to that strength of the assembly and the minimum
effect is passed by the Parliament with strength is fixed at 40.
special majority It means that the size of the council depends
Solution: (b) on the size of the assembly of the concerned
Justification: The Constitution provides state. This is done to ensure the predominance
for the abolition or creation of legislative of the directly elected House (assembly) in
councils in states. the legislative affairs of the state. Though
Accordingly, the Parliament can abolish the Constitution has fixed the maximum and
a legislative council (where it already exists) the minimum limits, the actual strength of a
or create it (where it does not exist), if the Council is fixed by Parliament.
legislative assembly of the concerned state Q.992 The Constitution provides for the abolition
passes a resolution to that effect. or creation of legislative councils in states.
Such a specific resolution must be passed Accordingly, the Parliament can abolish or
by the state assembly by a special majority, create a legislative council in a state when
that is, a majority of the total membership of 1. The state assembly passes the specific
the assembly and a majority of not less than resolution with special majority
two-thirds of the members of the assembly 2. It is passed in the Parliament by ordinary
present and voting. majority.
Q.990 Abolition of the legislative council in a state 3. A prior recommendation has been made
requires by the President.
(a) the consent of the President and Governor Choose the correct answer using the codes
concerned below.
(b) the consent of the Parliament and the (a)  1 and 2 only (b)  2 and 3 only
state legislature concerned (c)  1 and 3 only (d)  All of the above
(c) the consent of the State government and Solution: (a)
State legislature Q.993 Consider the following statements about the
(d) the consent of Parliament, Governor and legislative council in a state.
State legislature concerned 1. Its strength is fixed by the Parliament.
Solution: (b) 2. The size of the council depends on size of
The very existence of the council depends the assembly of the concerned state.
on the will of the assembly. The council

Indian Polity Question Bank P.271

03-Indian Polity_Q928-1069.indd 271 8/7/2018 7:41:52 PM


3. None of its members are directly elected. Q.995 In the election to the Legislative Council of
Choose the correct answer using the codes a State, which of the following participate?
below. 1. Municipalities
(a)  1 and 2 only (b)  2 and 3 only 2. Members of Legislative Assembly of the
(c)  1 and 3 only (d)  All of the above State
Solution: (d) 3. Important District Functionaries
Unlike the members of the legislative Choose the correct answer using the codes
assembly, the members of the legislative below.
council are indirectly elected. (a)  1 and 2 only
Q.994 Five out of every six members of a legislative (b)  2 and 3 only
council are indirectly elected and the rest (c)  1 and 3 only
are nominated by the governor. Out of all (d)  All of the above
those participate in the indirect election of Solution: (a)
the members, highest number of members is
elected by? Powers and Functions
(a) Graduates of three years standing and
Q.996 Consider the following statements about
residing within the state
State Legislative Council:
(b) Teachers of three years standing in
1. Approval of ordinances issued by the
the state, not lower in standard than
Governor.
secondary school
2. Consideration of reports of the
(c) Members of local bodies in the state
constitutional bodies like State finance
(d) All elect equal number of candidates
commission, CAG etc.
Solution: (c)
3. Enlargement of the jurisdiction of State
Justification: Of the total number of
public service Commission
members of a legislative council:
In which of these, the council has an equal
1. 1/3 are elected by the members of local
status with that of legislative assembly?
bodies in the state like municipalities,
(a)  1 and 2 (b)  2 and 3
district boards, etc.,
(c)  1 and 3 (d)  All of the above
2. 1/12 are elected by graduates of three years
Solution: (d)
standing and residing within the state,
Other two cases are:
3. 1/12 are elected by teachers of three years
(a) Selection of chief Ministers and other
standing in the state, not lower in standard
ministers can be made from council too.
than secondary school,
(b) Introduction and passage of ordinary
4. 1/3 are elected by the members of the
bills. However, in case of disagreement,
legislative assembly of the state from
the will of assembly prevails.
amongst persons who are not members of
the assembly, and Q.997 In which of the following matters, the powers
5. The remainder are nominated by the and status of the State Legislative council are
governor from amongst persons who broadly equal to that of the State Legislative
have a special knowledge or practical assembly?
experience of literature, science, art, 1. Ratification of a constitutional amendment
cooperative movement and social bill
service. 2. Consideration of the report of State
Thus, 5/6 of the total numbers of members Finance Commission
of a legislative council are indirectly elected 3. Approval of ordinances issued by the
and 1/6 are nominated by the governor. The governor
members are elected in accordance with the 4. Election of the representatives of the state
system of proportional representation by in the Rajya Sabha
means of a single transferable vote. 5. Discussing the State Annual Budget

P.272 For Civil Services Preliminary Examination

03-Indian Polity_Q928-1069.indd 272 8/7/2018 7:41:52 PM


Select the correct answer using the codes • A person to be elected to the legislative
below. council must be an elector for an assembly
(a)  1, 2, 4 and 5 only constituency in the concerned state
(b)  2, 3 and 4 only and to be qualified for the governor’s
(c)  2, 3 and 5 only nomination, he must be a resident in the
(d)  3 and 5 only concerned state. So, the option 1 is right
Solution: (c) and the option 2 is wrong.
Justification: Statement 1: The council has • A person to be elected to the legislative
no effective say in the ratification of a assembly must be an elector for an assembly
constitutional amendment bill. Only the constituency in the concerned state.
assembly ratifies it. • He must be a member of a scheduled caste
Statement 2: Both have equal powers or scheduled tribe if he wants to contest
in the consideration of the reports of a seat reserved for them. However, a
the constitutional bodies like State member of scheduled castes or scheduled
Finance Commission, state public service tribes can also contest a seat not reserved
commission and Comptroller and Auditor for them.
General of India. Q.999 Consider the following statements.
Statement 3: Similar to passing of a law, 1. A person to be elected to the legislative
ordinances also need to be approved by both council need not be an elector for assembly
houses. constituency in the concerned state.
Statement 4: Only the assembly 2. To be qualified for the governor’s
participates in the election of the Rajya Sabha nomination, the person must be a resident
MPs based on proportional representation in the concerned state.
system (single transferrable votes). 3. A person to be elected to the legislative
Statement 5: The council can discuss the assembly must be an elector for an
budget but cannot vote on the demands for assembly constituency in the concerned
grants (which is the exclusive privilege of the state.
assembly). Select the correct answer using the codes
below.
Qualification and Disqualification (a)  1 and 2 only (b)  1 and 3 only
(c)  3 only (d)  1, 2 and 3
of Members Solution: (c)
Q.998 The Indian Parliament has laid down which Justification: Statement 1: He must be an
of the following qualifications for members elector for assembly constituency in the
of the legislative council? concerned state. So, 1 would be wrong.
1. A person to be elected to the legislative Moreover, he must be a member of a
council must be an elector for an assembly scheduled caste or scheduled tribe if he
constituency in the concerned state. wants to contest a seat reserved for them.
2. The Governor can nominate even those However, a member of scheduled castes or
persons to the legislative council who scheduled tribes can also contest a seat not
are ordinarily not the residents of the reserved for them.
concerned state. Statement 2: He nominates one-sixth
Which of the above is/are correct? of the members of the state legislative
(a)  1 only (b)  2 only council from amongst persons having
(c)  Both 1 and 2 (d)  None special knowledge or practical experience
Solution: (a) in literature, science, art, Cooperative
Justification: These conditions have been Movement and Social service.
laid down in the Representation of People He can nominate one member to the state
Act (1951): legislature assembly from the Anglo-Indian
Community.

Indian Polity Question Bank P.273

03-Indian Polity_Q928-1069.indd 273 8/7/2018 7:41:52 PM


Presiding Officers of the House As in the case of the Speaker, the salaries
and allowances of the Chairman are also fixed
Q.1000 The Deputy Speaker of the State Legislative by the state legislature. They are charged on
Assembly is the Consolidated Fund of the State and thus
(a) Nominated by the Speaker of the House are not subject to the annual vote of the state
(b) Nominated by the Governor of the legislature.
concerned State He resigns by writing to the Deputy
(c) Elected by the house from amongst its Chairman. He can be removed by a resolution
members passed by a majority of all the then members
(d) Always the Leader of the largest party of the council.
from the opposition
Q.1002 Consider the following with reference to the
Solution: (c)
Chairman of the Legislative Council of a
Learning: Like the Speaker, the Deputy
State.
Speaker is also elected by the assembly
1. He is nominated by the Governor on
itself from amongst its members. He is
the recommendation of the Council of
elected after the election of the Speaker has
Ministers.
taken place.
2. His salary and allowances are not subject
The Deputy Speaker performs the duties
to the annual vote of the state legislature
of the Speaker’s office when it is vacant.
as they are charged on the Consolidated
He also acts as the Speaker when the latter
Fund of the State.
is absent from the sitting of assembly. In
Which of the above is/are correct?
both the cases, he has all the powers of the
(a)  1 only (b)  2 only
Speaker.
(c)  Both 1 and 2 (d)  None
The Speaker nominates from amongst
Solution: (b)
the members a panel of chairmen. Any one
Refer the previous explanation.
of them can preside over the assembly in
the absence of the Speaker or the Deputy Q.1003 The Chairman of the State Legislative
Speaker. Council
1. Has to be a member of the Council for
being eligible
Chairman of Legislative Council 2. Elected by the Council
Q.1001 How is the Chairman of Legislative Council 3. Appointment is approved by the Governor
selected? Choose the correct answer using the codes
(a) Nominated by the Governor from below.
amongst the members of the Council (a)  1 and 2 only (b)  2 and 3 only
(b) Selected by the Speaker of the Legislative (c)  1 and 3 only (d)  All of the above
Assembly Solution: (a)
(c) Elected by the council itself from Refer the previous explanation
amongst its members.
(d) The Senior-most member of the Council Legislative Process
automatically becomes its Chairman.
Solution: (c) Q.1004 State legislatures can make laws to enforce
Learning: As a presiding officer, the 1. Fundamental Rights
powers and functions of the Chairman in the 2. Directive Principles of State Policy
council are similar to those of the Speaker 3. Fundamental Duties
in the assembly. However, the Speaker has Select the correct answer using the codes
one special power which is not enjoyed by below.
the Chairman. The Speaker decides whether (a)  1 and 2 only (b)  2 and 3 only
a bill is a Money Bill or not and his decision (c)  1 and 3 only (d)  1, 2 and 3
on this question is final. Solution: (b)

P.274 For Civil Services Preliminary Examination

03-Indian Polity_Q928-1069.indd 274 8/7/2018 7:41:52 PM


Justification: Statement 1: Most of them are (b) Parliament’s authority to make Laws on
directly enforceable (self-executed) while a State subjects if Rajya Sabha passes a
few of them can be enforced on the basis of a resolution as required by the Constitution
law made for giving effect to them. (c) The Governor’s discretionary power to
Such a law can be made only by the dissolve the legislature
Parliament and not by state legislatures so (d) The Governor’s power to reserve certain
that uniformity throughout the country is bills for the consideration of the President
maintained (Article 35). So, 1 is incorrect. Solution: (c)
Statement 2: For e.g. Tamil Nadu
government came up with many schemes to Ordinary Bill
universalize food distribution, health care,
education and others which fulfil provisions Q.1007 When a bill passed by the legislative assembly
of DPSP. So, the option 2 is correct. transmitted to the legislative council is
Statement 3: For example, environmental rejected by it, which of the following is
conservation and forests fall in concurrent possible?
lists, the preservation of which also falls (a) The assembly will pass the bill again and
under fundamental duties. State legislatures the bill will be deemed to be passed by
can make laws on these matters, so, the the legislature.
option 3 is correct. (b) The assembly will pass the bill again,
transmit it to the council and only after
Q.1005 Which of the following limits the legislative
the assent of the council will the bill be
sphere and authority of the State Legislatures?
deemed to be passed by the legislature.
1. A written constitution
(c) The assembly will pass the bill again,
2. Division of powers in the Seventh Schedule
transmit it to the council and even without
3. Judicial review of State enactments by the
the assent of the council will the bill be
High courts
deemed to be passed by the legislature.
4. Veto powers of the Governor and President
(d) The Governor will have to call a joint
over state legislations
sitting of both the houses
Choose the correct answer using the codes
Solution: (c)
below.
When a bill is passed by the legislative
(a)  2, 3 and 4 only (b)  2 and 3 only
assembly and transmitted to the legislative
(c)  1 and 4 only (d)  All of the above
council, the latter has four alternatives
Solution: (d)
before it:
Since legislative division is made in 7th
• it may pass the bill as sent by the assembly
schedule, in the Union, State and Concurrent
(i.e., without amendments);
list, the state legislatures are bound to their
• it may pass the bill with amendments
allocated subjects. Both the Supreme and
and return it to the assembly for
High courts can review the acts of the state
reconsideration;
legislatures and declare them null and void
• it may reject the bill altogether; and
if they are ultra vires the constitution or any
• it may not take any action and thus keep
fundamental rights. The Governor has every
the bill pending.
right to reserve state bill for President in
• If the council passes the bill without
certain cases. The president may or may not
amendments or the assembly accepts the
sign the bills in which case they lapse. The
amendments suggested by the council, the
governor can also exercise his veto powers
bill is deemed to have been passed by both
over state enactments. He can also delay a bill.
the Houses and the same is sent to the the
Q.1006 The limitations on the authority of the State governor for his assent.
Legislature do not include: • On the other hand, if the assembly rejects
(a) Parliament’s authority to make laws the amendments suggested by the council
on subjects in the State List during an or the council rejects the bill altogether or
Emergency

Indian Polity Question Bank P.275

03-Indian Polity_Q928-1069.indd 275 8/7/2018 7:41:52 PM


the council does not take any action for Select the correct answer using the codes
three months, then the assembly may pass below.
the bill again and transmit the same to (a)  1 and 2 only (b)  2 and 3 only
the council. If the council rejects the bill (c)  1 and 3 only (d)  All of the above
again or passes the bill with amendments Solution: (a)
not acceptable to the assembly or does Justification: The Governor cannot accept a
not pass the bill within one month, then bill which has not been passed by the assembly.
the bill is deemed to have been passed Doing so would be unconstitutional. So, 3 is
by both the Houses in the form in which incorrect.
it was passed by the assembly for the Joint sitting provision is not there, because
second time. the assembly has overriding powers with
• Therefore, the ultimate power of passing respect to passing any bill in the state. So, the
an ordinary bill is vested in the assembly. Council is much weaker than the assembly.
At the most, the council can detain or delay Thus, 1 and 2 are correct.
the bill for a period of four months—three Q.1010 If the Legislative council of a State rejects the
months in the first instance and one month bill passed once by the Legislative Assembly.
in the second instance. (a) The bill lapses and does not become
Q.1008 If the Legislative assembly (LA) of a state an act
passes a bill, which is then rejected by (b) A joint sitting of both houses is called by
the legislative council (LC), which of the the Governor
following may follow? (c) The bill is passed again by the assembly
1. The bill will stand repealed. and transmitted to the Council
2. The bill will be referred to a joint sitting (d) The bill is reserved for Presidential
of both the houses. review by the Governor
3. The bill will be referred to the Governor Solution: (c)
for a final view. Q.1011 When the legislative assembly disagrees to a
4. The bill can be again passed by the LA bill originated in the legislative council and
and re-sent to the LC. passed by the later, a deadlock is reached,
5. The LA can directly reject the view of the following which
LC, and go ahead with enacting the bill. (a) The Governor is bound to reserve the
Choose the correct answer using the codes bill for the consideration of the President
below. whose decision shall be final in this
(a)  4 or 5 only (b)  1 or 2 or 3 only regard
(c)  2 or 5 only (d)  4 only (b) A joint sitting of both houses is called by
Solution: (d) the Governor to resolve the issue
Refer previous explanations. (c) The legislative assembly sends the
Q.1009 Consider the following statements about the bill to the legislative council for
legislative process in the states. its reconsideration with proposed
1. The status of legislative councils in amendments
passing ordinary bills is weaker than (d) The bill becomes dead and cannot be
Rajya Sabha. further considered by any of the houses
2. There is no provision of joint sitting in Solution: (d)
case of a disagreement between the two Justification and Learning: A deadlock
houses in the states. between the two Houses takes place when
3. If the Governor accepts the changes made the legislative council, after receiving a bill
by the legislative council which were not passed by the legislative assembly, rejects
accepted by the assembly, then the bill is the bill or proposes amendments that are not
deemed to be passed. acceptable to the legislative assembly or does
not pass the bill within three months.

P.276 For Civil Services Preliminary Examination

03-Indian Polity_Q928-1069.indd 276 8/7/2018 7:41:52 PM


• The mechanism of passing the bill for on the recommendation of the governor.
the second time to resolve a deadlock Every such bill is considered to be a
applies to a bill originating in the government bill and can be introduced
legislative assembly only. When a bill, only by a minister.
which has originated in the legislative • After a Money Bill is passed by the
council and sent to the legislative legislative assembly, it is transmitted
assembly, is rejected by the latter, the to the legislative council for its
bill ends and becomes dead. So, the consideration. The legislative council has
option (d) is correct. restricted powers with regard to a Money
• The Constitution does not provide for Bill. It cannot reject or amend a Money
the mechanism of joint sitting of two Bill. It can only make recommendations
Houses of the state legislature to resolve a and must return the bill to the legislative
deadlock between them over the passage assembly within 14 days.
of a bill. • The legislative assembly can either
• The legislative assembly can override the accept or reject all or any of the
legislative council by passing the bill for recommendations of the legislative
the second time and not vice versa. council. If the legislative assembly
• When a bill is passed by the assembly accepts any recommendation, the bill is
for the second time and transmitted to then deemed to have been passed by both
the legislative council, if the legislative the Houses in the modified form. If the
council rejects the bill again, or proposes legislative assembly does not accept
amendments that are not acceptable to any recommendation, the bill is then
the legislative assembly, or does not pass deemed to have been passed by both the
the bill within one month, then the bill Houses in the form originally passed
is deemed to have been passed by both by the legislative assembly without any
the Houses in the form in which it was change.
passed by the legislative assembly for the • Finally, when a Money Bill is presented
second time. to the governor, he may, either, give his
assent, withhold his assent or reserve
Money Bills the bill for presidential assent but cannot
return the bill for reconsideration of the
Q.1012 Consider the following about the State state legislature. Normally, the governor
Legislature. gives his assent to a money bill as it is
1. A Money Bill cannot be introduced in the introduced in the state legislature with his
legislative council. prior permission.
2. Money bills can be introduced only on the • When a money bill is reserved for
recommendation of the Governor. consideration of the President, the
3. The Governor cannot reserve a money bill president may either give his assent to the
for the consideration of the President. bill or withhold his assent to the bill but
Choose the correct answer using the codes cannot return the bill for reconsideration
below. of the state legislature.
(a)  1 and 2 only (b)  2 and 3 only
Q.1013 Consider the following statements about
(c)  1 and 3 only (d)  All of the above
money bills with regard to State legislature.
Solution: (a)
1. A money bill can be introduced in the
The Constitution lays down a special
legislative assembly only and that too on
procedure for the passing of Money Bills in
the recommendation of the governor.
the state legislature.
2. Since every such bill is considered to be a
• A Money Bill cannot be introduced in the
government bill, it can be introduced only
legislative council. It can be introduced in
by a minister.
the legislative assembly only and that too

Indian Polity Question Bank P.277

03-Indian Polity_Q928-1069.indd 277 8/7/2018 7:41:52 PM


Which of the above is/are correct? Miscellaneous
(a)  1 only (b)  2 only
(c)  Both 1 and 2 (d)  None Q.1015 Consider the following statements.
Solution: (c) 1. A minister, who is not a member of either
Refer previous explanation. House, can participate in the proceedings
of both the Houses.
Privileges 2. The advocate general of the state has the
right to speak, take part in the proceedings
Q.1014 The privileges that belong to each House of of both Houses and vote in case of a tie.
the state legislature collectively are? Which of the above is/are correct?
1. It can make rules to regulate its own (a)  1 only (b)  2 only
procedure and the conduct of its business (c)  Both 1 and 2 (d)  None
and to adjudicate upon such matters. Solution: (a)
2. The courts are prohibited to inquire Justification: Statement 1: In addition to the
into the proceedings of a House or its members of a House, every minister and the
Committees. advocate general of the state have the right
3. No person, either a member or an outsider, to speak and take part in the proceedings
can be arrested within the precincts of of either House or any of its committees of
the House without the permission of the which he is named a member, without being
Leader of the House. entitled to vote. So, the option 2 would be
Select the correct answer using the codes incorrect.
below. There are two reasons underlying this
(a)  1 and 2 only (b)  2 and 3 only constitutional provision:
(c)  3 only (d)  1, 2 and 3 • A minister can participate in the
Solution: (a) proceedings of a House, of which he is not
Justification: Other privileges (apart from a member.
the ones mentioned in the Q) are: • A minister, who is not a member of either
• It has the right to publish its reports, House, can participate in the proceedings
debates and proceedings and also the of both the Houses.
right to prohibit others from publishing Q.1016 Uttar Pradesh has the largest Legislative
the same. Assembly among all the States and Union
• It can exclude strangers from its Territories with more than 400 seats. The
proceedings and hold secret sittings to second largest legislative assembly is in
discuss some important matters. (a) Rajasthan
• It can make rules to regulate its own (b)  Andhra Pradesh
procedure and the conduct of its business (c)  Madhya Pradesh
and to adjudicate upon such matters. (d)  West Bengal
• It can punish members as well as Solution: (d)
outsiders for breach of its privileges or
its contempt by reprimand, admonition Q.1017 There is a provision for having nominated
or imprisonment (also suspension or members in which of the following houses
expulsion, in case of members). apart from Lok Sabha?
• It has the right to receive immediate 1. Rajya Sabha
in-formation of the arrest, detention, 2. Vidhan Sabha
conviction, imprisonment and release of a 3. Vidhan Parishad
member. Select the correct answer using the codes
• It can institute inquiries and order the below.
attendance of witnesses and send for (a)  1 and 2 only (b)  2 and 3 only
relevant papers and records. (c)  1 and 3 only (d)  1, 2 and 3
Solution: (d)

P.278 For Civil Services Preliminary Examination

03-Indian Polity_Q928-1069.indd 278 8/7/2018 7:41:52 PM


Justification: President can nominate 12 SPECIAL STATUS FOR
members to the Rajya Sabha.
Governor may appoint 1 member to STATES – PART XXI –
represent minorities, e.g. the Anglo- Indian ARTICLES 370 AND 371
community, if he finds that minority
inadequately represented in the Vidhan
Sabha.
(28) SPECIAL STATUS OF JAMMU
In a Vidhan Parishad, one-sixth of AND KASHMIR – PART XXI –
members are nominated by the Governor ARTICLE 370
from persons having knowledge or practical
experience in fields such as literature, Q.1019 The Delhi Agreement of 1952 is remarkable
science, arts, the co-operative movement and in Modern Indian History because
(a) It was crucial to the integration of the
state of Jammu and Kashmir in India.
(27) CHIEF SECRETARY OF THE STATE (b) It proposed constitutional recognition to
Q.1018 Consider the following statements. the panchayati raj system in India.
1. Chief Secretary of the State acts as (c) It settled the Line of control between
secretary to the state cabinet. India and Pakistan to reduce border
2. Cabinet Secretary is the ex-officio skirmishes.
Principal Secretary to the Prime Minister. (d) It gave the Union territory of Delhi greater
3. All top officials of Prime Minister’s Office power to function as an autonomous unit.
(PMO) must be present at Union cabinet Solution: (a)
meetings. Learning: This agreement endorsed the
Select the correct answer using the codes main decisions of the Constituent Assembly
below. of the State of J&K.
(a)  1 only (b)  1 and 3 only It was reached between the representatives
(c)  2 only (d)  1, 2 and 3 of Kashmir Government and the Indian
Solution: (a) Government. Later it came to be known as
Justification: Statement 1: The Chief the “Delhi Agreement, 1952”.
Secretary is the administrative head of This led to the conferment of special
state administration and in many ways status to Kashmir, including a separate flag,
counterpart of Cabinet Secy. He is also the jurisdiction of the Parliament and Supreme
Chief Coordinator of State secretariat and court over the state etc.
other administrative departments. He attends Q.1020 Consider the following statements about
cabinet meetings. Article 370 of the Indian Constitution
Statement 2 Principal Secretary to the 1. Article 370 gives greater autonomy to
Prime Minister & Cabinet Secretary are two Jammu and Kashmir compared to other
different positions. States of India.
2. The State has its own Constitution.
3. All provisions of the Indian Constitution
are applicable to the State.
4. Laws passed by the Parliament apply to
J&K only if the State agree.
Which of the statements given above is/are
correct?
(a)  1 and 4 Only (b)  1, 2 and 4 Only
(c)  1, 3 and 4 Only (d)  All
Solution: (b)
Internally, there is a dispute about the
status of Kashmir within the Indian union.

Indian Polity Question Bank P.279

03-Indian Polity_Q928-1069.indd 279 8/7/2018 7:41:52 PM


Article 370 gives greater autonomy to Jammu Justification: Statement 4: The power of
and Kashmir compared to other States of Parliament to make laws for the state is
India. The State has its own Constitution. All limited to:
provisions of the Indian Constitution are not I. Those matters in the Union List and the
applicable to the State. Laws passed by the Concurrent List which correspond to
Parliament apply to J&K only if the State matters specified in the state’s Instrument
agrees. of Accession. These matters are to be
This special status has provoked two declared by the president in consultation
opposite reactions. with the state government. The Instrument
There is a section of people outside of of Accession contained matters classified
J&K that believes that the special status of under four heads, namely, external affairs,
the State conferred by Article 370 does not defense, communications and ancillary
allow full integration of the State with India. matters.
This section feels that Article 370 should II. Such other matters in the Union List and
therefore be revoked and J&K should be like the Concurrent List which are specified by
any other State in India. the president with the concurrence of the
Another section, mostly Kashmiris believe state government. This means that laws
that the autonomy conferred by Article 370 can be made on these matters only with
is not enough. A section of Kashmiris have the consent of the State of J&K.
expressed at least three major grievances. Q.1022 Consider the following statements about the
First, the promise that accession would be state of Jammu and Kashmir (J&K):
referred to the people of the State after the 1. It has a separate constitution.
situation created by tribal invasion was 2. No other state in India enjoys the kind of
normalised, has not been fulfilled. This has special status that has been accorded to it
generated the demand for a ‘Plebiscite’. by the Indian constitution.
Secondly, there is a feeling that the special 3. It is a temporary state of the Union of
federal status guaranteed by Article 370, India.
has been eroded in practice. This has led Which of these is/are true?
to the demand for restoration of autonomy (a)  1 and 2 (b)  2 and 3
or ‘Greater State Autonomy’. Thirdly, it (c)  1 and 3 (d)  All of the above
is felt that democracy which is practiced Solution: (a)
in the rest of India has not been similarly There is no such thing as a temporary or a
institutionalised in the State of Jammu and permanent state. J&K is an integral part of
Kashmir. India. However, it enjoys special status under
Q.1021 Consider the following about the state of Article 370 of the Indian constitution.
J&K. Q.1023 Consider the following statements about
1. It is the only state in India which has its Article 370 of the constitution of India which
own separate state Constitution. grants special status to J&K.
2. All the provisions of the Constitution of 1. Article 370 can be repealed unilaterally
India do not apply to the state. by the Parliament of India.
3. The Constitution clearly states that the 2. As provided, the other provisions of the
special provisions with respect to J&K are constitution can be applied with such
only temporary and not permanent. modifications and exceptions as specified
4. Parliament has no power to make laws for by the President of India.
the state. Which of these is/are true?
Select the correct answer using the codes (a)  Only 1 (b)  Only 2
below. (c)  Both (d)  None of the above
(a)  1 and 2 only (b)  3 and 4 only Solution: (d)
(c)  1, 2 and 3 only (d)  1, 2, 3 and 4 As provided, the other provisions of the
Solution: (c) constitution can be applied with such

P.280 For Civil Services Preliminary Examination

03-Indian Polity_Q928-1069.indd 280 8/7/2018 7:41:52 PM


modifications and exceptions as specified by 2. If the President specifies some subjects
the President of India only after taking the from the Union list for J&K, then it
consent of J&K state. curtails Parliamentary jurisdiction.
Any modification in the Article 370 will Which of these is/are true?
have to be approved by the J&K government. (a)  Only 1 (b)  Only 2
Q.1024 Article 370 gives a special status to the (c)  Both (d)  None of the above
polity of Jammu and Kashmir (J&K) in the Solution: (c)
Indian state. A bill for the amendment of the Q.1026 Consider the following statements with
Constitution of the J&K respect to special provisions provided to
(a) Must be introduced in the Parliament Jammu and Kashmir:
and agreed to by both the Parliament and 1. The concurrence of the State is required
State Legislature of J&K. for making any law in matters mentioned
(b) Can be introduced in the state assembly in both the Union and Concurrent lists.
and must be passed by it for such 2. Emergency due to internal disturbances
amendment to be effective. can be declared without the concurrence
(c) Must be referred to the Governor of J&K of the State.
who should refer it to the President and 3. The Directive Principles of State policy
pass the bill after taking the Council of applies to the State.
Ministers into confidence. 4. Amendments to the Indian Constitution,
(d) Cannot be introduced in either the under Article 368, can only apply in
Parliament or state assembly without concurrence with the government of
an amendment in Article 370 of the Jammu and Kashmir.
constitution. Which of the above statements is/are
Solution: (b) incorrect?
Learning: It can be amended by a bill (a)  2 and 3 Only (b)  1, 2 and 4 Only
passed in each house of the state legislature (c)  1, 3 and 4 Only (d)  1, 2, 3 and 4
by a majority of two-thirds of the total Solution: (a)
membership of that house. According to Article 370, the concurrence
• Such a bill must be introduced in the of the State is required for making any
assembly only. laws in matters mentioned in the Union and
• However, no bill of constitutional Concurrent lists. This is different from the
amendment can be moved in either House position of other States
if it seeks to change the relationship of the The remaining differences between the
state with the Union of India. other States and the State of J&K are that no
• It vests the executive powers of the state emergency due to internal disturbances can
in the governor appointed by the president be declared in J&K without the concurrence
for a term of five years. of the State. The union government cannot
• But, the Governor does not enjoy impose a financial emergency in the State
legislative powers apart from those andthe Directive Principles do not apply
enjoyed by other governors of Indian in J&K. Finally, amendments to the Indian
states. This means OPTION C is wrong. Constitution (under Article 368) can only
• The Constitution of J&K was adopted apply in concurrence with the government
in 1957. of J&K.
Q.1025 Consider the following statements about the Q.1027 Consider the following statements about the
parliamentary jurisdiction with reference to polity of Jammu and Kashmir (J&K).
the state of Jammu and Kashmir (J&K). 1. The state does not guarantee any
1. Parliament cannot make laws in the fundamental rights to its residents due
concurrent list for J&K without the to the enforcement of martial law in the
consent of the state. state.

Indian Polity Question Bank P.281

03-Indian Polity_Q928-1069.indd 281 8/7/2018 7:41:52 PM


2. An amendment made to the Constitution J&K. Finally, amendments to the Indian
of India does not apply to the state unless Constitution (under Article 368) can only
it is extended by a presidential order. apply in concurrence with the government of
3. The President has no power to suspend J&K.
the Constitution of the state on the ground Q.1029 Consider the following about the special
of failure to comply with the directions status of Jammu and Kashmir in Indian polity.
given by him. 1. The boundaries of the state cannot be
Select the correct answer using the codes changed by the Parliament without the
below. consent of its state legislature.
(a)  1 and 2 only (b)  1 only 2. The state legislature has the power to make
(c)  2 and 3 only (d)  1, 2 and 3 laws on many matters in the residuary list.
Solution: (c) 3. A National Emergency declared on the
Justification: The very definition of ‘state’ ground of internal disturbance will not
under PART VI of Indian constitution does have effect in the state except with the
not include the State of J&K. concurrence of the state government.
Hence, Part VI of the Constitution of Select the correct answer using the codes
India (dealing with state governments) is not below.
applicable to this state. (a)  1 only (b)  2 and 3 only
Q.1028 Consider the following about the (c)  1 and 3 only (d)  1, 2 and 3
administration of Jammu and Kashmir Solution: (d)
(J&K). Justification: Statement 1: Jammu and
1. No emergency due to internal disturbances Kashmir is a constituent state of the
can be declared in J&K without the Indian Union and has its place in Part I
concurrence of the State. and Schedule I of the Constitution of India
2. The union government cannot impose a (dealing with the Union and its Territory).
financial emergency in the State. But its name, area or boundary cannot be
3. Amendments to the Indian Constitution changed by the Union without the consent
can only apply in concurrence with the of its legislature.
government of J&K. Statement 2: Parliament can make laws in
4. The Government of India is responsible relation to the state on most of the subjects
for the external security of J&K. enumerated in the Union List and on a
Select the correct answer using the codes good number of subjects enumerated in the
below. Concurrent List. But, the residuary power
(a)  1, 2 and 3 only belongs to the state legislature except in
(b)  2, 3 and 4 only few matters such as prevention of activities
(c)  1 and 4 only involving terrorist acts.
(d)  1, 2, 3 and 4 Statement 3: Moreover, the President has
Solution: (d) no power to declare a financial emergency
Justification: According to Article 370, in relation to the state. The President has no
the concurrence of the State is required for power to suspend the Constitution of the state
making any laws in matters mentioned in the on the ground of failure to comply with the
Union and Concurrent lists. directions given by him.
The remaining differences between the Q.1030 The term of the legislative assembly is six
other States and the State of J&K are that no years in which of these states of India?
emergency due to internal disturbances can (a) Nagaland
be declared in J&K without the concurrence (b)  Jammu and Kashmir
of the State. (c) Assam
The union government cannot impose (d) The term limit is five years in all the
a financial emergency in the State and states of India.
the Directive Principles do not apply in Solution: (b)

P.282 For Civil Services Preliminary Examination

03-Indian Polity_Q928-1069.indd 282 8/7/2018 7:41:52 PM


Learning: The term for the legislature is six President’s Rule is imposed under Article
years under its own state Constitution. 356 of the Constitution.
Also, the minimum strength fixed at 40 • But in case of J&K, as per Section 92 of
by the Constitution of India is not applicable state Constitution, the Governor’s Rule
to Jammu and Kashmir. Its council has 36 can be imposed in the state for a period
members under the provisions of its own of six months only after the consent of
state Constitution. the President of India in case of failure
Q.1031 The Fundamental Right to Property is still of constitutional machinery. During the
guaranteed in which of the following states Governors rule, State Assembly is either
of India? suspended or dissolved.
(a)  Jammu and Kashmir • If the Constitutional machinery is not
(b) Assam restored before the expiry of this six
(c) Nagaland month period, the provision of Article 356
(d)  Telangana of the Constitution of India are extended
Solution: (a) to J&K and the President’s rule is imposed
Learning: Part III (dealing with in the State.
Fundamental Rights) is applicable to the • It should be noted that, Governors rule has
state of J&K with some exceptions and been imposed in the state for the 6 time
conditions. The Fundamental Right to since 1977.
Property is still guaranteed in the state. Q.1033 Governor’s rule is a unique feature of the
Also, certain special rights are granted to J&K polity. It is imposed when
the permanent residents of the state with (a) The Central Government imposes
regard to public employment, acquisition President’s rule in the state
of immovable property, settlement and (b) There is National emergency or internal
government scholarships. Part IV (dealing disturbance
with Directive Principles of State Policy) (c) The state administration is not carried
and Part IVA (dealing with Fundamental in accordance with the provisions of the
Duties) are not applicable to the state J&K constitution
Q.1032 Governor’s rule has been imposed in the (d) The President is satisfied that the state
state of Jammu and Kashmir (J&K) recently. government is incapable of administering
Consider the following statements about the state and hands over the governance
Governor’s rule. to the Governor of the state
1. It is applicable as per the Indian Solution: (c)
constitution. The constitution of J&K provides for
2. It can be imposed only after the consent of Governor’s Rule. Hence, the governor, with
the President. the concurrence of the President of India,
3. This is the first time Governor’s rule has can assume to himself all the powers of
been imposed in J&K. the state government, except those of the
Choose the correct answer using the codes high court. He can dissolve the assembly
below: and dismiss the council of ministers. The
(a)  1 and 2 only (b)  2 only Governor’s Rule can be imposed when the
(c)  1 and 3 only (d)  All of the above state administration cannot be carried on in
Solution: (b) accordance with the provisions of the J&K
The election for Jammu Kashmir State Constitution. It was imposed for the first
assembly was held in the month of December time in 1977. Notably, in 1964, Article 356
in 2014 which has given the hung verdict as of the Indian Constitution (dealing with the
no party has majority to form government in imposition of President’s Rule in a state)
the State. was extended to the state of J&K.
• In case of failure of constitutional Q.1034 Consider the following statements about
machinery in any other state of India, the Jammu and Kashmir (J&K).

Indian Polity Question Bank P.283

03-Indian Polity_Q928-1069.indd 283 8/7/2018 7:41:52 PM


1. An amendment made to the Constitution of the State. The union government cannot
of India does not apply to the state unless impose a financial emergency in the State and
it is extended by a presidential order. the Directive Principles do not apply in J&K.
2. It is the only state in the Indian Union Finally, amendments to the Indian Constitution
which has its own separate state (under Article 368) can only apply in
Constitution. concurrence with the government of J&K.
3. The President has no power to suspend
the Constitution of the state on the ground
of failure to comply with the directions
(29) SPECIAL PROVISION FOR SOME
given by him. STATES
Choose the correct answer using the codes
below. Article 371–Part xxi
(a)  1 and 2 only (b)  2 and 3 only
Q.1036 Under Article 371, Part XXI, of the
(c)  1 and 3 only (d)  All of the above
Constitution certain constitutional authorities
Solution: (d)
have been given greater responsibilities with
Under Article 1 of the Indian Constitution,
respect to some special regions of India.
the State of Jammu and Kashmir (J&K) is
These authorities/bodies are?
a constituent state of Indian Union and its
1. President of India
territory forms a part of the territory of India.
2. Governor of the concerned state
On the other hand, Article 370 in Part XXI of
3. Parliament of India
the Constitution grants a special status to it.
Select the correct answer using the codes
Accordingly, all the provisions of the
below.
Constitution of India do not apply to it. It is
(a)  1 only (b)  2 only
also the only state in the Indian Union which
(c)  2 and 3 only (d)  1, 2 and 3
has its own separate state Constitution—the
Solution: (d)
Constitution of Jammu and Kashmir.
Justification: Statement 1: For e.g. for the
Q.1035 Consider the following statements with erstwhile undivided state of AP, the President
reference to the status of J&K in the Indian was empowered to provide for equitable
federal structure. opportunities and facilities for the people
1. The Central legislation in the Union list belonging to different parts of the state in the
and Concurrent list applies to J&K only matter of public employment and education
after the consent of the state government. and different provisions can be made for
2. The Directive Principles of State Policy various parts of the state.
do not apply to the state. Statement 2: Similarly, the Governor
3. Amendments to the Indian Constitution should submit an annual report to the
(under Art. 368) can only apply in President regarding the administration of the
concurrence with the government of J&K. Hill Areas of Manipur.
Choose the correct answer using the codes Moreover, the Governor shall have
below. special responsibility for peace and for an
(a)  1 and 2 only (b)  2 and 3 only equitable arrangement for ensuring the social
(c)  1 and 3 only (d)  All of the above and economic advancement of the different
Solution: (d) sections of the Sikkim population.
According to Article 370, the concurrence Statement 3: Article 371E empowers the
of the State is required for making any Parliament to provide for the establishment
laws in matters mentioned in the Union and of a Central University in the undivided
Concurrent lists. state of Andhra Pradesh.
The remaining differences between the
Q.1037 Special provisions (under Part XXI) of the
other States and the State of J&K are that no
Constitution are applicable to the States of
emergency due to internal disturbances can
1. Maharashtra
be declared in J&K without the concurrence
2. Gujarat

P.284 For Civil Services Preliminary Examination

03-Indian Polity_Q928-1069.indd 284 8/7/2018 7:41:52 PM


3. Jharkhand Originally, the constitution did not make
4. Chhattisgarh any special provisions for these states.
5. Andhra Pradesh They have been incorporated by the various
Select the correct answer using the codes subsequent amendments made in the context
below. of reorganisation of the states or conferment
(a)  1 and 2 only (b)  3 and 4 only of statehood on the Union Territories
(c)  1, 2 and 5 only (d)  1, 2, 3, 4 and 5 Q.1039 Consider the following statements in relation
Solution: (c) to the Article 371-A of the constitution
Justification: The provisions of the making special provisions for Nagaland.
Constitution pertaining to the states are 1. The Nagaland state assembly can declare
applicable to all the states (except Jammu that a law made by the Parliament is not
and Kashmir) in the same manner. applicable to it on religious and social
However, the special provisions (under grounds.
Part XXI) applicable to the States of 2. The Nagaland state assembly can override
Maharashtra, Gujarat, Nagaland, Assam, the laws of the Parliament concerning
Manipur, Andhra Pradesh, Sikkim, Mizoram, transfer of land and its resources.
Arunachal Pradesh, karnataka and Goa Which of the above is/are correct?
override the general provisions relating to the (a)  1 only (b)  2 only
states as a class. (c)  Both 1 and 2 (d)  None
Statement 2 and 3: But, the provisions Solution: (c)
for Jharkhand and Chhattisgarh are found Justification: As per Article 371(1),
only in the 5th Schedule (tribal areas). notwithstanding anything in this Constitution,
Further, the Sixth Schedules contain separate no Act of Parliament in respect of
provisions with respect to the administration 1. religious or social practices of the Nagas,
of scheduled areas and tribal areas within the 2. Naga customary law and procedure,
NE states. 3. administration of civil and criminal
Q.1038 Articles 371 to 371J in Part XXI of the justice involving decisions according to
Constitution contain special provisions for Naga customary law,
eleven states. Which of the following states 4. ownership and transfer of land and its
are listed there? resources, shall apply to the State of
1. Gujarat Nagaland unless the Legislative Assembly
2. Bihar of Nagaland by a resolution so decides.
3. Sikkim The Governor of Nagaland shall have special
4. Goa responsibility for law and order in the state
5. Himachal Pradesh so long as internal disturbances caused by
Select the correct answer using the codes the hostile Nagas continue. In the discharge
below. of this responsibility, the Governor, after
(a)  2 and 3 only consulting the Council of Ministers, exercises
(b)  1, 3 and 4 only his individual judgement and his decision
(c)  1, 4 and 5 only is final. This special responsibility of the
(d)  1, 2, 3, 4 and 5 only Governor shall cease when the President so
Solution: (b) directs.
The intention behind them is to meet the Q.1040 The State of Nagaland is a special state as per
aspirations of the people of backward regions the Indian constitution. Nagaland has relative
of the states or to protect the cultural and political autonomy in deciding which of the
economic interests of the tribal people of the following matters?
states or to deal with the disturbed law and 1. Administration of Civil and Criminal
order condition in some parts of the states or Justice
to protect the interests of the local people of 2. Ownership of land and its transfer
the states.

Indian Polity Question Bank P.285

03-Indian Polity_Q928-1069.indd 285 8/7/2018 7:41:52 PM


3. Customary law and its practice UNION TERRITORIES AND
Choose the correct answer using the codes
below. SPECIAL AREAS
(a)  1 and 2 only
(b)  2 and 3 only UNION TERRITORIES— PART VIII –
(c)  1 and 3 only ARTICLES 239 TO 242
(d)  All of the above
Solution: (d) Q.1042 What are designated as Union Territories
(UTs) presently were
Special Category States (SCSs) 1. Constituted as ‘scheduled districts’ during
British rule
Q.1041 Consider the following statements about the
2. Declared as ‘special category’ provinces
Special Category States (SCSs) in India:
as per the Government of India Act, 1935.
1. A state must share an international
Which of the above is/are correct?
boundary to be treated as a SCS.
(a)  1 only (b)  2 only
2. All the SCSs in India till date show a
(c)  Both 1 and 2 (d)  None
budget surplus despite being lower in the
Solution: (a)
ladder of development.
Justification: Statement 1: During the
3. These states receive special financial
British Rule, certain areas were constituted
packages from the central government
as ‘scheduled districts’ in 1874. Later, they
and higher support in the Central Sector
came to be known as ‘chief commissioners’
Schemes (CSS) as compared to the General
provinces’.
Category States.
Statement 2: After independence, they
Choose the correct answer using the codes
were placed in the category of Part ‘C’
below:
and Part ‘D’ states. In 1956, they were
(a)  1 and 2 (b)  2 and 3
constituted as the ‘Union Territories’ by the
(c)  1 and 3 (d)  Only 3
7th Constitutional Amendment Act (1956)
Solution: (d)
and the States Reorganisation Act (1956).
Here we should know that the international
Learning:
boundary criteria are important, but not
The union territories have been created for
essential for being a Special Category State.
a variety of reasons. These are mentioned
below.
1. Political and administrative consideration—
• Delhi and
• Chandigarh.
2. Cultural distinctiveness—
• Puducherry,
• Dadra and Nagar Haveli, and
• Daman and Diu.
3. Strategic importance—
• Andaman and Nicobar Islands and
• Lakshadweep.
4. Special treatment and care of the backward
and tribal people—
• Mizoram,
• Manipur,
Gradually, some of these union territories
have been elevated to statehood. Thus,
Himachal Pradesh, Manipur, Tripura,
Mizoram, Arunachal Pradesh and Goa,

P.286 For Civil Services Preliminary Examination

03-Indian Polity_Q928-1069.indd 286 8/7/2018 7:41:53 PM


which are states at present but were formerly 2. An administrator of a union territory is
union territories. an agent of the President and not head of
On the other hand, the territories that were state like a Governor.
acquired from the Portuguese (Goa, Daman 3. The governor is to act independently of his
and Diu, and Dadra and Nagar Haveli) and council of ministers if he is administering
the French (Puducherry) were constituted as a UT too.
the union territories. Choose the correct answer using the codes
Q.1043 At the Central level, the subject of ‘urban below.
local government’ in Union Territories (UT) (a)  1 and 2 only (b)  2 and 3 only
is dealt by (c)  1 and 3 only (d)  All of the above
(a)  Ministry of Urban Development Solution: (b)
(b)  Ministry of Defence Articles 239 to 241 in Part VIII of the
(c)  Ministry of Home Affairs Constitution deal with the union territories.
(d)  Office of the President of India Even though all the union territories belong
Solution: (c) to one category, there is no uniformity in
Justification: Ministry of Urban Development their administrative system.
deals with the same subject in States of India. Every union territory is administered by
Ministry of Defence deals with cantonments the President acting through an administrator
boards only in the context of urban local appointed by him. An administrator of a
government. union territory is an agent of the President
Learning: Under the Government of India and not head of state like a governor.
(Allocation of Business) Rules 1961, Ministry The President can specify the designation
of Home Affairs is the nodal ministry for all of an administrator; it may be Lieutenant
matters of Union Territories relating to Governor or Chief Commissioner or
• Legislation, Administrator. At present, it is Lieutenant
• Finance and budget, Governor in the case of Delhi, Puducherry
• Services and appointment of Lt. Governors and Andaman and Nicobar Islands and
and Administrators. Administrator in the case of Chandigarh,
All the five UTs without a legislature Dadra and Nagar Haveli, Daman and Diu
(Andaman and Nicobar Islands, Chandigarh, and Lakshadweep.
Daman and Diu, Dadra and Nagar Haveli, The President can also appoint the
and Lakshadweep) have the forum of Home governor of a state as the administrator of
Minister’s Advisory Committee (HMAC), an adjoining union territory. In that capacity,
whose members are, the governor is to act independently of his
• The Administrator council of ministers.
• Member of Parliament from the respective Q.1045 What is the constitutional qualification
Union Territory, mentioned for appointment as a Lieutenant
• Members from the local elected bodies, e.g., Governor of a Union Territory (UT)?
• District Panchayats and (a) The person must have served either the
• Municipal Council / Committees. Central or State government for 10 years
Meetings of the HMAC are chaired by The or more.
Union Home Minister, or, in his absence, by (b) The person should neither be related to
the Minister of State in the Ministry of Home political parties nor had a political career.
Affairs. The Committee discusses the general (c) The person must be a resident of the
issues relating to social and economic Union Territory or adjoining states.
development of the Union territories (d)  None of the above
Q.1044 Consider the following statements. Solution: (d)
1. The constitution provides for a uniform Learning: The President has recently
administrative system in all Union appointed BJP leader and former IPS officer
Territories (UTs). Kiran Bedi as the Lieutenant Governor (LG)

Indian Polity Question Bank P.287

03-Indian Polity_Q928-1069.indd 287 8/7/2018 7:41:53 PM


of Puducherry as per Article 239 of the appointed by him. An administrator of a
Constitution. So, the option (b) is wrong. union territory is an agent of the President
The constitution does not mention explicit and not head of state like a governor. He
qualifications for the post of Lt. Governor. does not have absolute veto power. Only in
He acts as an agent of the President and few cases like in concurrent list legislations,
administers the UT on behalf of him. he needs to refer the bill to the President of
Q.1046 Consider the following statements. India. At most, he can delay a bill.
1. An administrator of a union territory is Q.1048 Consider the following statements about the
an agent of the President and not head of administrator of the UTs appointed by the
state like a governor. President:
2. The President can appoint the governor of 1. He need not necessarily adhere to the
a state as the administrator of an adjoining advice tendered by the council of ministers
union territory. in UTs where an assembly exists.
Which of the above is/are correct? 2. He appoints the Chief Minister of that UT.
(a)  1 only (b)  2 only 3. He can promulgate ordinances in the
(c)  Both 1 and 2 (d)  None recess of the assembly.
Solution: (c) Which of these is/are correct?
Justification: Statement 1: In case of a (a)  1 and 2 (b)  2 and 3
state of India, the Governor acts as the (c)  1 and 3 (d)  All of the above
head of the state despite being an unelected Solution: (c)
representative. In a UT, although the Lt. The administrator can take a different view
Governor is appointed in the same way as a in case his opinion conflicts with that of the
Governor is appointed in a state, he neither council of ministers but only after referring it
has as much discretion as the Governor of to the President.
a state has, nor is he the head of the state. The President appoints the CM.
Statement 2: In that capacity, the governor Q.1049 The Chief Minister of a Union Territory is
is to act independently of his council of appointed by
ministers, i.e. he does not act on the aid and (a) The Administrator of the UT on
advice of the CoM. However, his position the recommendation of the Central
with respect to the UT is similar as that of a government
Lt. Governor or Administrator. (b) The President of India on the
Q.1047 Consider the following statements regarding recommendation of the Administrator of
the administrator of UTs in India. the UT
1. An administrator of a union territory is (c) The President of India based on the
an agent of the President and not head of recommendation of the Cabinet
state like a governor. (d) The Administrator of the UT based on his
2. The administrator of a UT has absolute personal discretion
veto power over the laws enacted by the Solution: (b)
legislative assembly of that UT. The following best sums up the relationship
3. In a UT having an elected government, between the President, Administrator and
administrator is appointed after the UT government by giving the example
consultation with the CM of the UT. of Delhi. The chief minister is appointed
Choose the correct answer using the codes by the President (not by the lt. governor).
below. The other ministers are appointed by
(a)  1 and 2 only (b)  2 and 3 only the president on the advice of the chief
(c)  1 only (d)  1 and 3 only minister. The ministers hold office during
Solution: (c) the pleasure of the president. The council of
Every union territory is administered by the ministers is collectively responsible to the
President acting through an administrator assembly. The council of ministers headed

P.288 For Civil Services Preliminary Examination

03-Indian Polity_Q928-1069.indd 288 8/7/2018 7:41:53 PM


by the chief minister aid and advice the (b) A is correct, but R is not an appropriate
lt. governor in the exercise of his functions explanation of A.
except in so far as he is required to act in (c)  A is correct, but R is incorrect.
his discretion. In the case of difference of (d)  Both A and R are incorrect.
opinion between the lt. governor and his Solution: (c)
ministers, the lt. governor is to refer the Justification: The Parliament can make
matter to the president for decision and act laws on any subject of the three lists
accordingly. (including the State List) for the union
Q.1050 The council of ministers headed by the territories. This power of Parliament also
chief minister aid and advise the Lieutenant extends to Puducherry and Delhi, which
governor in the exercise of his functions have their local legislatures.
except in so far as he is required to act in • This means that the legislative power
his discretion. In the case of difference of of Parliament for the union territories
opinion between the Lieutenant governor and on subjects of the State List remain
his ministers unaffected even after establishing a local
(a) The advice of the Council of Ministers legislature for them. So, A is correct.
prevails if the same advice is tendered • But, the legislative assembly of Puducherry
again after reconsideration. can also make laws on any subject of the
(b) The Chief Minister is to refer the matter State List and the Concurrent List. So, R
to the Ministry of Home Affairs and act is incorrect.
accordingly. • Similarly, the legislative assembly of
(c) The Lieutenant governor is to refer the Delhi can make laws on any subject of the
matter to the president for decision and State List (except public order, police and
act accordingly. land) and the Concurrent List.
(d) The view of the Lieutenant governor Q.1052 Consider the following about legislation in
prevails over that of the Council of the Union Territories (UTs).
Ministers. 1. The UT legislature cannot make laws on
Solution: (c) matter relating to the Concurrent List.
Learning: It is notable that the same 2. The Parliament cannot make laws in the
provision is not given for the Governor of a State List pertaining to those UTs having
state because of three reasons: their own legislatures.
1. He is the Head of the state unlike the Which of the above is/are correct?
Lt. Governor. (a)  1 only (b)  2 only
2. A state is not directly under the control of (c)  Both 1 and 2 (d)  None
the Union Government unlike UTs. Solution: (d)
3. The constitution is clear over the powers Justification: Statement 1: The Parliament
of the Governor where he is supposed to can make laws on any subject of the three
exercise discretion. lists (including the State List) for the union
Q.1051 Consider the following statements. territories.
Assertion (A): The establishment of a This power of Parliament also extends
legislative assembly in the union territories to Puducherry and Delhi, which have their
does not invalidate the supreme control of the own local legislatures.This means that, the
president and Parliament over them. legislative power of Parliament for the union
Reason (R): Legislative assemblies in UTs territories on subjects of the State List remain
cannot enact any law without Presidential unaffected even after establishing a local
assent or his prior recommendation. legislature for them.
In the context of the above, which of these Statement 2: But, the legislative assembly
is correct? of Puducherry can also make laws on any
(a) A is correct, and R is an appropriate subject of the State List and the Concurrent
explanation of A. List. Similarly, the legislative assembly of

Indian Polity Question Bank P.289

03-Indian Polity_Q928-1069.indd 289 8/7/2018 7:41:53 PM


Delhi can make laws on any subject of the The people of UTs vote only for their own
State List (except public order, police and legislative assembly, if any. Moreover, they
land)  and the Concurrent List. get to vote for general elections.
Q.1053 Which of the following statements is correct There is no nomination by the Central
about the legislative control of Parliament Government in the Legislative Assembly
over the Union Territories (UTs)? of a UT (Delhi and Pudducherry). The
(a) The Parliament cannot make laws in State Administrator of the UT is appointed by
List for those UTs that have a legislative the President on the advice of the central
assembly. cabinet.
(b) The Parliament can make laws in State
List for even those UTs that have a National Capital Territory—
legislative assembly but only after prior 69th Constitutional Amendment
recommendation of the President.
(c) The Parliament can make laws on any Q.1055 The Delhi Legislative assembly can make
subject of the three lists (including the laws on all the matters of the State List and
State List) for the union territories. the Concurrent List except the three matters
(d) None of the above. of the State List, that is
Solution: (c) 1. Public Order
The Parliament can make laws on any 2. Police
subject of the three lists (including the 3. Land
State List) for the union territories. This Choose the correct answer using the codes
power of Parliament also extends to below.
Puducherry and Delhi, which have their (a)  1 and 2 only (b)  2 and 3 only
own local legislatures. This means that, (c)  1 and 3 only (d)  All of the above
the legislative power of Parliament for Solution: (d)
the union territories on subjects of the The 69th Constitutional Amendment Act
State List remain unaffected even after of 19915 provided a special status to the
establishing a local legislature for them. Union Territory of Delhi, and re-designated
But, the legislative assembly of Puducherry it the National Capital Territory of Delhi
can also make laws on any subject of and designated the administrator of Delhi
the State List and the Concurrent List. as the lieutenant (lt.) governor. It created
Similarly, the legislative assembly of Delhi a legislative assembly and a council of
can make laws on any subject of the State ministers of Delhi. Previously, Delhi had
List (except public order, police and land) a metropolitan council and an executive
and the Concurrent List. council. The strength of the assembly is
fixed at 70 members, directly elected by the
Q.1054 Consider the following statements:
people. The elections are conducted by the
1. The people residing in the nearest Union
election commission of India. The assembly
territory (UT) to a state also vote in that
can make laws on all the matters of the State
particular State Assembly elections.
List and the Concurrent List except the three
2. One-third of the members of the
matters of the State List, that is, public order,
Legislative assembly in UTs, where it
police and land. But, the laws of Parliament
exists, are nominated by the Central
prevail over those made by the Assembly.
Government.
Which of these is/are true? Q.1056 Consider the following about the National
(a)  1 only Capital Territory (NCT) of Delhi.
(b)  2 only 1. All the members of the Legislative
(c)  Both 1 ad 2 assembly are chosen by direct election.
(d)  None of the above 2. Laws of assembly on municipal matters
Solution: (d) prevail over those made by the Parliament.

P.290 For Civil Services Preliminary Examination

03-Indian Polity_Q928-1069.indd 290 8/7/2018 7:41:53 PM


3. Chief Minister of the NCT of Delhi is and order etc.) considering the strategic
appointed by the Lieutenant Governor. importance of capital New Delhi.
4. The size of council of Ministers is decided All acts passed by the legislative assembly
by the Lieutenant Governor through an require Lt Governor’s approval.
order issued by the Union Ministry of Where there is a difference of opinion
Home Affairs. between assembly and the Lt Governor, the
Select the correct answer using the codes matter is referred to the president.
below. Q.1058 Consider the following statements about the
(a)  1 only (b)  1, 3 and 4 only legislature of Delhi.
(c)  1 and 2 only (d)  2 and 3 only 1. The Lieutenant Governor shall from
Solution: (a) time-to-time summon the Legislative
Justification: Statement 1: The strength of Assembly to meet at such time and place
the assembly is fixed at 70 members, directly as he thinks fit.
elected by the people. The elections are 2. Only the President is authorized to
conducted by the election commission of prorogue or dissolve the Delhi Assembly.
India. Which of the above is/are correct?
Statement 2: The assembly can make laws (a)  1 only (b)  2 only
on all the matters of the State List and the (c)  Both 1 and 2 (d)  None
Concurrent List except the three matters of Solution: (a)
the State List, that is, public order, police and Justification: Statement 1: A Bill or
land. But, the laws of Parliament prevail over amendment shall not be introduced into, or
those made by the Assembly. moved in the Legislative Assembly except
Statement 3: The chief minister is on the recommendation of the Lieutenant,
appointed by the President (not by the lt. Governor, if such Bill or amendment makes
governor). The other ministers are appointed provision for any of the following matters,
by the president on the advice of the chief Statement 2: When a Bill has been
minister. passed by the Legislative Assembly, it shall
Statement 4: The strength of the council be presented to the Lieutenant Governor
of ministers is fixed at ten per cent of the total and the Lieutenant Governor shall declare
strength of the assembly, that is, seven—one either that he assents to the Bill or that
chief minister and six other ministers. he withholds assent therefrom or that he
Q.1057 The Lieutenant Governor of Delhi is reserves the Bill for the consideration of
appointed by the President.
(a) The President on the advice of the The Lieutenant Governor may, from time
Ministry of Home Affairs to time prorogue the Assembly or dissolve
(b) The Legislative Assembly of Delhi on the Assembly.
the advice of the UT Council of Ministers Q.1059 The Lieutenant Governor is the Administrator
(c) The Chief Minister of Delhi after of the Delhi UT. In the case of difference of
consulting a collegium consisting of the opinion between the lt. governor and his
Chief Justice of the Delhi High court and ministers, what option can he chose?
other senior judges (a) Refer the matter to the president for
(d) The Delhi Development Authority decision and act accordingly.
(DDA) after obtaining consent from the (b)  Refer the matter to the Legislative
President of India Assembly for decision and act
Solution: (a) accordingly.
Learning: Lt. Governor is the ‘administrator’ (c) The decision of the Council of Ministers
of Delhi appointed by the President at the endorsed by the CM prevails.
recommendation of Union Home Ministry. (d) The decision of the Lieutenant Governor,
Specifically for New Delhi, Lt even though opposed by CM, prevails.
Governor has additional powers (land, law Solution: (a)

Indian Polity Question Bank P.291

03-Indian Polity_Q928-1069.indd 291 8/7/2018 7:41:53 PM


The chief minister of Delhi is appointed by Select the correct answer using the codes
the President (not by the lt. governor). below.
The other ministers are appointed by the (a)  1 only (b)  1 and 2 only
president on the advice of the chiefminister. (c)  2 and 3 only (d)  2 only
The ministers hold office during the pleasure Solution: (d)
of the president. The council of ministers is Justification: Statement 1: He is appointed
collectively responsible to the assembly. by the President.
The council of ministers headed by the chief Statement 2: An ordinance has the same
minister aid and advice the lt. governor in the force as an act of the assembly. Every such
exercise of his functions except in so far as ordinance must be approved by the assembly
he is required to act in his discretion. In the within six weeks from its reassembly. He can
case of difference of opinion between the lt. also withdraw an ordinance at any time.
governor and his ministers, the lt. governor But, he cannot promulgate an ordinance
is to refer the matter to the president for when the assembly is dissolved or
decision and act accordingly. suspended. Further, no such ordinance can be
Q.1060 A ordinance can only be issued by the promulgated or withdrawn without the prior
Lieutenant Governor (LG) in Delhi UT when permission of the President.
(a) The Chief Minister of Delhi has Statement 3: The council of ministers
endorsed it. headed by the chief minister aid and advice
(b) The CM along with the Council of the lt. governor in the exercise of his functions
Ministers has endorsed it. except in so far as he is required to act in his
(c) The President has given prior discretion.
recommendation to issue it. In the case of difference of opinion
(d) The LG is satisfied that the situation between the lt. governor and his ministers,
demands it and no approval is required at the lt. governor is to refer the matter to the
the time of issuing the ordinance. president for decision and act accordingly.
Solution: (c)
The Lt. Governor is empowered to
promulgate ordinances during recess of the
(31) SCHEDULE & TRIBAL AREAS
assembly. An ordinance has the same force as
an act of the assembly. Every such ordinance Fifth Schedule – Scheduled Areas
must be approved by the assembly within Q.1062 Consider the following statements about
six weeks from its reassembly. He can also Scheduled Areas.
withdraw an ordinance at any time. But, he 1. Scheduled areas are declared under laws
cannot promulgate an ordinance when the passed by the Parliament.
assembly is dissolved or suspended. Further, 2. The concerned state cannot exercise
no such ordinance can be promulgated or executive power within the Scheduled
withdrawn without the prior permission of area.
the President. 3. The administration of these areas is
Q.1061 Consider the following with reference to the entirely under the control of the President.
National Capital Territory of (NCT) Delhi. Choose the correct answer using the codes
1. The chief minister is appointed by the below.
Lieutenant Governor. (a)  1 and 2 only (b)  2 only
2. The Lt. Governor is empowered to (c)  1 and 3 only (d)  None of the above
promulgate ordinances during recess of Solution: (d)
the assembly. The various features of administration
3. In the case of difference of opinion contained in the Fifth Schedule are as follows:
between the lt. governor and his • Declaration of Scheduled Areas: The
ministers, the view of Lt. Governor president is empowered to declare an area
prevails invariably. to be a scheduled area. He can also increase

P.292 For Civil Services Preliminary Examination

03-Indian Polity_Q928-1069.indd 292 8/7/2018 7:41:53 PM


or decrease its area, alter its boundary Justification: Statement 5: ‘The scheduled
lines, rescind such designation or make areas are treated differently from the other
fresh orders for such re-designation on an areas in the country because they are
area in consultation with the governor of inhabited by ‘aboriginals’ who are socially
the state concerned. and economically rather backward, and
• Executive Power of State and Centre: The special efforts need to be made to improve
executive power of a state extends to the their condition.
scheduled areas therein. But the governor Therefore, the whole of the normal
has a special responsibility regarding administrative machinery operating in a state
such areas. He has to submit a report to is not extended to the scheduled areas and the
the president regarding the administration Central government has somewhat greater
of such areas, annually or whenever so responsibility for these areas.
required by the president. The executive Statements 1 and 2: Explained in previous
power of the Centre extends to giving question
directions to the states regarding the Statement 3: He can declare that such laws
administration of such areas. do not apply to a scheduled area or apply
• Tribes Advisory Council: Each state with specified modifications and exceptions.
having scheduled areas has to establish He can also make regulations for the peace
a tribes advisory council to advise and good government of a scheduled area
on welfare and advancement of the after consulting the tribal advisory council.
scheduled tribes. It is to consist of 20 Statement 4: The Constitution requires the
members, three-fourths of whom are to president to appoint a commission to report
be the representatives of the scheduled on the administration of the scheduled areas
tribes in the state legislative assembly. A and the welfare of the scheduled tribes in the
similar council can also be established states. He can appoint such a commission
in a state having scheduled tribes but not at any time but compulsorily after ten years
scheduled areas therein, if the president of the commencement of the Constitution.
so directs. Hence, a commission was appointed in 1960
Q.1063 Consider the following about the areas under and later in 2002.
the Fifth Schedule of the Constitution. Q.1064 Consider the following statements about the
1. The president is empowered to declare an schedule area
area to be a scheduled area. 1. The Governor of the state is empowered
2. The executive or legislative power of to declare scheduled areas in the particular
a state extends to the scheduled areas state.
therein. 2. The Governor can direct that a particular
3. The governor is empowered to direct Act of the Parliament may not apply to the
that any particular act of Parliament or scheduled area.
the state legislature does not apply to a Which of these is/are true?
scheduled area. (a)  Only 1 (b)  Only 2
4. The Constitution requires the president (c)  Both (d)  None of the above
to appoint a commission to report on the Solution: (b)
administration of the scheduled areas. The President declares scheduled areas under
5. The Union Government has no 5th schedule, not the Governor.
responsibility for the administration of
these areas. Sixth Schedule
Select the correct answer using the codes
below. Q.1065 The Sixth Schedule contains the provisions
(a)  3 and 4 only (b)  1, 2, 3 and 4 only for the administration of tribal areas in the
(c)  1, 2 and 5 only (d)  1, 2, 3, 4 and 5 States of
Solution: (b) (a) 
Arunachal Pradesh, Sikkim, Nagaland
and Manipur

Indian Polity Question Bank P.293

03-Indian Polity_Q928-1069.indd 293 8/7/2018 7:41:53 PM


(b) Assam, Meghalaya, Tripura and Mizoram. during the pleasure of the governor. Each
(c) Nagaland, Meghalaya, Mizoram and autonomous region also has a separate
Manipur regional council.
(d) Assam, Arunachal Pradesh, Tripura and Q.1067 Consider the following statements.
Nagaland Assertion (A): The tribal areas in the four
Solution: (b) states of Assam, Meghalaya, Tripura and
Justification: It is very easy to eliminate the Mizoram fall outside the executive authority
above options. of the state concerned.
Nagaland is mentioned in all options Reason (R): They have been constituted
other than B. Nagaland is not under the 6th as autonomous districts under the Sixth
schedule, rather special provisions have been Schedule of the Constitution of India.
earmarked in the constitution for the state. In the context of the above, which of these
So, B is the answer. is correct?
The Governor has a large share (a) A is correct, and R is an appropriate
of responsibilities with respect to the explanation of A.
administration of scheduled areas. (b) A is correct, but R is not an appropriate
Q.1066 Consider the following statements about explanation of A.
tribal areas in Sixth Schedule. (c)  A is incorrect, but R is correct.
1. They are completely autonomous of the (d)  A is correct, but R is incorrect.
State Government. Solution: (c)
2. The autonomous districts are governed by Learning: The governor is empowered to
an elected District Council. organise and re-organise the autonomous
3. The district and regional councils are districts. Thus, he can increase or decrease
empowered to assess and collect land their areas or change their names or define
revenue and to impose certain specified their boundaries and so on.
taxes. If there are different tribes in an
Choose the correct answer using the codes autonomous district, the governor can
below. divide the district into several autonomous
(a)  1 and 2 only (b)  2 and 3 only regions.
(c)  1 and 3 only (d)  All of the above • The district and regional councils
Solution: (b) administer the areas under their
The tribal areas in the four states of Assam, jurisdiction. They can make laws on
Meghalaya, Tripura and Mizoram have been certain specified matters like land, forests,
constituted as autonomous districts. But, they canal water, shifting cultivation, village
do not fall outside the executive authority of administration and inheritance of property,
the state concerned. marriage and divorce, social customs and
The governor is empowered to organise so on. But all such laws require the assent
and re-organise the autonomous districts. of the governor.
Thus, he can increase or decrease their • The acts of Parliament or the state
areas or change their names or define their legislature do not apply to autonomous
boundaries and so on. districts and autonomous regions or
Each autonomous district has a district apply with specified modifications and
council consisting of 30 members, of whom exceptions.
four are nominated by the governor and the Sixth Schedule administers the areas under
remaining 26 are elected on the basis of adult their jurisdiction. Consider the following
franchise. statements about it.
The elected members hold office for a term 1. They can constitute village councils or
of five years (unless the council is dissolved courts for trial of suits and cases between
earlier) and nominated members hold office the tribes.

P.294 For Civil Services Preliminary Examination

03-Indian Polity_Q928-1069.indd 294 8/7/2018 7:41:53 PM


2. Laws made by them do not require the Select the correct answer using the codes
assent of either the Legislature or the below.
Governor. (a)  1 and 2 only (b)  2 only
Which of the above is/are true? (c)  3 only (d)  1, 2 and 3 only
(a)  1 only (b)  2 only Solution: (c)
(c)  Both 1 and 2 (d)  None Justification: In Assam: The North Cachar
Solution: (a) Hills District; The Karbi Anglong District;
The district and regional councils The Bodoland Territorial Areas District.
administer the areas under their In Meghalaya: Khasi Hills District; Jaintia
jurisdiction. They can make laws on Hills District; The Garo Hills District.
certain specified matters like land, forests, In Tripura: Tripura Tribal Areas District
canal water, shifting cultivation, village In Mizoram: The Chakma District; The
administration, inheritance of property, Mara District; The Lai District.
marriage and divorce, social customs and Learning: Some of these regions
so on. But all such laws require the assent are undergoing autonomy and separatist
of the Governor. The district and regional movements.
councils within their territorial jurisdictions
can constitute village councils or courts
for trial of suits and cases between the
tribes. They hear appeals from them. The
jurisdiction of high court over these suits
and cases is specified by the governor. The
councils can adjudicate on minor civil and
criminal disputes.
Q.1068 Consider the following about the areas under
the Sixth schedule of the Constitution.
1. The governor is empowered to organise
and re-organise the autonomous districts
under these areas.
2. The district and regional councils can
make laws on certain specified matters.
3. The acts of Parliament or the state
legislature necessarily do not apply to
autonomous districts.
4. The village councils can do trial of suits
and cases between the tribes.
Choose the correct answer using the codes
below.
(a)  1 and 2 only (b)  2, 3 and 4 only
(c)  1 and 3 only (d)  All of the above
Solution: (d)
Q.1069 Consider the following autonomous districts
and the state they are located in.
1. The North Cachar Hills District:
Meghalaya
2. The Mara District: Tripura
3. The Bodoland Territorial Areas District:
Assam

Indian Polity Question Bank P.295

03-Indian Polity_Q928-1069.indd 295 8/7/2018 7:41:53 PM


JUDICIARY Decisions made by the Supreme Court
are binding on all other courts within the
territory of India. Orders passed by it are
(32) JUDICIARY IN INDIA enforceable throughout the length and
Q.1070 The Constitution of India provides for a breadth of the country. The Supreme Court
single integrated Judiciary system. What itself is not bound by its decision and can at
does this imply? any time review it. Besides, if there is a case
1. Decisions made by higher court may be of contempt of the Supreme Court, then the
binding on the lower courts Supreme Court itself decides such a case.
2. Appellate system exists in India The Supreme Courts and High Courts can
3. It means all administration of Courts in hear appeals from lower courts and exercises
the country is based on the same principle superintendence and control over courts
of Justice. below it. Both enjoy appellate jurisdiction.
Which of the statements given below is/are Only Supreme Court has Advisory
correct? jurisdiction.
(a)  1 and 3 (b)  3 Only Q.1072 Consider the following statements about the
(c)  2 and 3 (d)  1 and 2 Judiciary in India.
Solution: (d) 1. The judiciary is not financially dependent
Integrated Judiciary means that the decisions on either the executive or legislature.
made by higher courts ‘are’ binding on lower 2. Parliament cannot discuss the conduct of
courts. By appellate system it means a person the judges except when the proceeding to
can appeal to a higher court if they believe remove a judge is being carried out.
that the judgement passed by the lower court 3. The judiciary has the power to penalise
is not just. those who are found guilty of contempt
Q.1071 The Constitution of India provides for a of court in case of unfair criticism of its
single Integrated Judicial System. In this decisions.
context, which of the following statements Choose the correct answer using the codes
is/are correct? below:
1. District Court decides cases involving (a)  1 and 2 (b)  2 and 3
serious criminal offences. (c)  1 and 3 (d)  All of the above
2. Subordinate Court considers cases of both Solution: (d)
civil and criminal nature. The judges have a fixed tenure. They hold
3. Both High Court and Supreme Court have office till reaching the age of retirement.
appellate jurisdiction Only in exceptional cases, judges may be
4. The Supreme Court is bound by its own removed. But otherwise, they have security
decision and cannot review it. of tenure. Security of tenure ensures that
5. Both High Court and Supreme Court have judges could function without fear or favour.
advisory jurisdiction The Constitution prescribes a very difficult
Select the correct code. procedure for removal of judges. The
(a)  1, 2 and 3 Only (b)  1, 3 and 5 Only Constitution makers believed that a difficult
(c)  2, 3, and 4 Only (d)  None procedure of removal would provide security
Solution: (a) of office to the members of judiciary.
District Court deals with cases arising in the Statement 1: The judiciary is not
District. It considers appeals on decisions financially dependent on either the executive
given by lower Courts and decides cases or legislature. The Constitution provides that
involving serious criminal offences. The the salaries and allowances of the judges
Subordinate Courts consider cases of Civil are not subjected to the approval of the
and Criminal nature. legislature.

P.296 For Civil Services Preliminary Examination

04-Indian Polity_Q1070-1506.indd 296 8/7/2018 7:42:37 PM


Statement 2: The actions and decisions Which of the above statements is/are correct
of the judges are immune from personal (a)  1 and 2 Only (b)  2 and 3 Only
criticisms. Parliament cannot discuss the (c)  1 Only (d)  3 Only
conduct of the judges except when the Solution: (a)
proceeding to remove a judge is being carried Statement 1: The Judicial system provides
out. This gives the judiciary independence to a mechanism for resolving disputes between
adjudicate without fear of being criticised. the citizens, between the citizen and the
Statement 3: The judiciary has the power government, between two states and between
to penalise those who are found guilty of the centre and state government.
contempt of court. This authority of the court Statement 2: Every citizen of India can
is seen as an effective protection to the judges approach the Supreme Court(Article 32) or
from unfair criticism. the High Court (Article 226)if they believe
Q.1073 Consider the following statements: that their Fundamental rights are violated.
1. The Judiciary in India is an organ of the Statement 3: Courts can strike down only
Government. those laws which violate constitution.
2. There is a hierarchy of courts in India with Q.1075 The judiciary in India can resolve disputes
the Supreme Court at the top to which the legally between
other courts are subordinate. 1. Citizens
3. All the courts in India have the power to 2. Citizens and Government
uphold the law and enforce fundamental 3. State governments
rights. 4. Central government and State
Choose the correct answer using the codes governments
below: Choose the correct answer using the codes
(a)  1 and 2 (b)  2 and 3 below:
(c)  1 and 3 (d)  Only 2 (a)  1 and 2 only (b)  1, 3 and 4 only
Solution: (d) (c)  All of the above (d)  2, 3 and 4 only
Statement 1: Judiciary is independent of Solution: (c)
the Government based on the doctrine of Q.1076 Which of the following powers of the
separation of powers. Judiciary help protecting and sustaining the
Statement 2: India has integrated judicial federal character of the Indian polity?
system i.e., hierarchy of courts with supreme 1. Power of judicial review
court at the apex. 2. Power to issue writs
Statement 3: While all courts uphold the 3. Power to adjudicate disputes between the
law, only the Supreme Court and High Courts Central and state governments.
are authorized to enforce fundamental rights. 4. Power to adjudicate disputes between the
Q.1074 Consider the following statements with state governments.
respect to the role of Judiciary in India: Choose the correct answer using the
1. The Judicial system provides a mechanism following codes:
for resolving disputes not only between (a)  All of the above (b)  1, 2 and 3
the citizen and the government but also (c)  1 and 3 (d)  1, 3 and 4
between two states and between the centre Solution: (d)
and state government. A federal government does not only mean
2. Every citizen of India can approach the that the adjudication of disputes between
Supreme Court or the High Court if they states is not a part of the federal dynamics.
believe that their Fundamental rights are States are autonomous in their areas of
violated. functioning and an independent court
3. Judiciary has the power to strike down ensures that conflicts between them are
any law resolved amicably.

Indian Polity Question Bank P.297

04-Indian Polity_Q1070-1506.indd 297 8/7/2018 7:42:37 PM


Power to issue writs is to protect the This can be best seen in the case of inter-
political democracy in India by protecting state water disputes where states have taken
fundamental rights, not federalism. disputes to the Supreme court despite an
Q.1077 Which of these statements is/are correct with award from a water tribunal.
reference to ‘Contempt Of Court’? Statement 2: Independence is crucial to
1. The constitution empowers the Judiciary unbiased dispute resolution.
to issue contempt of court notices. If the Supreme court was regulated by
2. Such a notice cannot be issued against any the Union government, we could not expect
of the judges of higher judiciary. it to take sides with the states on crucial
Which of the above is/are correct? issues such as inter-state water disputes as
(a)  1 only (b)  2 only mentioned above.
(c)  Both 1 and 2 (d)  None Q.1079 Which of the following statements regarding
Solution: (a) judiciary in India are correct?
Justification: Statement 1: Article 129 and 1. Supreme Court of India is free from the
Article 142 (2) of the Constitution enables control and influence of legislature and
the Supreme Court to issue notice and punish executive
anyone including Judges of the High Court 2. Subordinate courts are at the head of the
for the contempt of court. judicial hierarchy of the state.
Statement 2: For the first time in the 3. The Chief Justice and other judges of the
history of Independent India, the Supreme High Court are appointed by the Governor
Court issued contempt of court notice against in consultation with the Chief Justice
sitting Calcutta HC judge, C. S. Karnan, of India
for impeding justice administration and 4. A High Court can withdraw a case from a
bringing discredit to the judicial institution subordinate court and can deal with the case
by writing scurrilous letters about sitting and itself if it is satisfied that the case involves
retired judges. a substantial point of constitutional law
Learning: As per the Contempt of Courts Choose the correct answer from the codes
Act, 1971, following qualifies as contempt. given below:
• Civil contempt as the wilful disobedience (a)  1 and 2 (b)  1 and 3
to any judgement, decree, direction, order (c)  1 and 4 (d)  2, 3 and 4
or writ of the court Solution: (c)
• Criminal Contempt as the publication (by Statement 2: The High Court is at the head of
spoken words, written material, signs, the judicial hierarchy in that state.
visual representation or otherwise) that Statement 3: The President appoints
interfere with justice processes to lower the judges of a high court. Moreover, their
the authority of court removal can only be done by the Indian
Q.1078 Independent of Judiciary is helpful in a Parliament and not the state legislatures
Federal System because Q.1080 How the Project Progress Monitoring System
1. It helps prevent usurpation of authority by (Prime Ministers) is helpful to the Judiciary?
higher levels of government. 1. It assists judicial administration in
2. It allows impartial dispute resolution reducing the pendency of cases.
between different tiers of the government. 2. It ensures greater transparency of
Which of the above is/are correct? information for the litigants.
(a)  1 only (b)  2 only 3. It provides access to legal and judicial
(c)  Both 1 and 2 (d)  None databases to the judges.
Solution: (c) Select the correct answer using the codes
Justification: Statement 1: Many of the below.
Supreme Court judgments have helped the (a)  1 and 2 only (b)  1 only
states balance the authority of the Centre. (c)  1, 2 and 3 (d)  2 and 3 only

P.298 For Civil Services Preliminary Examination

04-Indian Polity_Q1070-1506.indd 298 8/7/2018 7:42:37 PM


Solution: (c) • No one should be made a judge in his own
Justification: It is an E-courts mission mode case, or the rule against bias
project. • Rule that no one should be condemned
• The project implements the Information unheard or the rule of fair hearing
and Communication Technology (ICT) Statement 1: The hearings in all courts are
in Indian judiciary. The project’s scope is based on these two fundamental principles.
to develop, deliver, install and implement The judge is an impartial observer and
automated decision making and decision allows the prosecution and defence to present
support system in the courts of India. It their sides before arriving at a judgment.
also provides regional languages support. Statement 2: The expression “Natural
• It also makes it possible to provide ICT Justice” hasn’t been explicitly used in the
coverage of judicial process from filing to constitution. However, the idea passes
execution level & also of all administrative through the body of Indian constitution.
activities. • Apart from preamble, Article 14 ensures
• It essentially creates information gateways equality before law and equal protection
between courts & public agencies & of law to the citizen of India striking at the
departments. So, 2 and 3 are correct. root of arbitrariness.
• The project is expected to lead to complete • Article 21 guarantees right to life and
demystification of the adjudicatory liberty which is the fundamental provision
process thereby ensuring transparency, to protect liberty and ensure life with
accountability & cost-effectiveness. So, dignity.
1 is correct. • Article 22 guarantees natural justice and
Q.1081 Consider the following statements. provision of fair hearing to the arrested
1. The Subordinate courts in India deal with person.
both civil and criminal cases. Q.1083 Consider the following instances.
2. The High court cannot issue writs for 1. A tenant who is being forced to move out
restoring fundamental rights. files a case in court against the landlord.
3. Lok Adalats do not have the jurisdiction 2. A group of girls are persistently harassed
to deal with criminal cases. by a group of boys while walking to
Choose the correct answer using the codes school.
below. 3. A woman is harassed to bring more dowry
(a)  1 and 2 only (b)  2 and 3 only 4. A woman files for a divorce, due to
(c)  1 and 3 only (d)  All of the above harassment by her in-laws
Solution: (a) Which of the above is/are classified under
Civil Law?
(33) CRIMINAL JUSTICE SYSTEM (a)  2 Only (b)  1, 2 and 4
(c)  1 and 4 (d)  2 and 4
IN INDIA Solution: (c)
Q.1082 Consider the following about “Principles of A Civil law deals with any harm/injury to
Natural Justice”. rights of individuals. For example, disputes
1. It is employed by courts in India in their relating to sale of land, purchase of goods,
proceedings. rent matters, divorce cases. A petition has to
2. It finds explicit mention in Part III of the filed before the relevant court by the affected
constitution as a constitutional right. party only and the court gives the specific
Which of the above is/are correct? relief asked for.
(a)  1 only (b)  2 only Whereas a Criminal Law deals with
(c)  Both 1 and 2 (d)  None conduct or acts that law defines as offences.
Solution: (a) For example, theft, harassing a woman to
Justification: Two core points in the concept bring more dowry, murder are those conducts
of principles of natural justice are as regarded.

Indian Polity Question Bank P.299

04-Indian Polity_Q1070-1506.indd 299 8/7/2018 7:42:37 PM


Q.1084 Consider the following statements about the Arrange in the correct order in which they
criminal justice system: happen.
1. The right to be defended by a lawyer is a (a) 132 (b) 123
fundamental right. (c) 321 (d) 312
2. The constitution places a duty on the state Solution: (d)
to provide a lawyer to any citizen who Q.1088 In the criminal justice system, consider
is unable to engage on due to poverty or the following statements about the role of
other disability. the Police.
Which of these is/are true? 1. It investigates complaints about the
(a)  1 only (b)  2 only commissioning of a crime with the
(c)  Both 1 and 2 (d)  None statement of witnesses and evidences.
Solution: (c) 2. It files a charge sheet in the court.
Article 22 of the constitution says that 3. It can punish the criminal based on the
the right to be defended is a fundamental charges it has framed against him or her.
right. Choose the correct answer using the codes
Article 39A (DPSP) places the duty on the below:
state as mentioned in statements 2. (a)  1 and 2 (b)  2 and 3
Q.1085 Consider the following Statements: (c)  1 and 3 (d)  All of the above
1. According to Article 21 of the constitution, Solution: (a)
every person has a Fundamental Right to The police merely forms an opinion based on
be defended by a lawyer its investigation. It is reflected in the charge
2. Article 39A of the constitution places a sheet. It cannot decide whether a person is
duty upon the State to provide a lawyer to guilty or not, and then punish him. This is
any citizen who is unable to engage one left to the courts.
due to poverty or other disability Q.1089 Consider the following statements about a
3. The Criminal Justice system includes First Information Report (FIR).
police, public prosecutor, defence lawyer, 1. It is compulsory to register a FIR when
judge only. a victim gives information about a
Select the correct code: cognisable offence.
(a)  1 and 2 (b)  1 and 3 2. FIR cannot be registered without oral
(c)  1 Only (d)  2 and 3 testimony of the complainant
Solution: (d) 3. The complainant has a constitutional right
It’s Article 22 that says defence by the lawyer to get a free copy of the FIR from the
is a fundamental right. police.
Q.1086 Which of the following are the parts of the Select the correct answer using the codes
Criminal Justice system in India? below.
1. Police 2.  Public Prosecutor (a)  1 and 2 only (b)  1 only
3. Judge 4.  Defence lawyer (c)  2 and 3 only (d)  None of the above
Choose the correct answer using the codes Solution: (b)
below: Explanation: Only when the complaint
(a)  Only 3 (b)  2, 3 and 4 pertains to a non-cognizable offence,
(c)  All of the above (d)  1 and 2 only registering a FIR is not mandatory. So,
Solution: (c) statement 1 is correct.
Q.1087 Consider the following stages in the working FIR can be given orally or in written. So,
of the Criminal Justice system. statement 2 is incorrect.
1. Trial The complainant has only a legal right to
2. Sentencing get a free copy of the FIR from the police. So,
3. Prosecution statement 3 is also incorrect.

P.300 For Civil Services Preliminary Examination

04-Indian Polity_Q1070-1506.indd 300 8/7/2018 7:42:37 PM


Learning: Crime can be classified as • The police can file a First Information Report
cognizable or Non-Cognizable. (FIR) in cases of cognizable offences.
In a non-cognizable offence, the Police • Normally, serious offences are defined as
cannot arrest a person without orders of the cognizable; these usually carry a sentence
court, i.e. without a Court warrant and can of 3 years or more.
investigate into the case only on the express • In India, crimes like rape, murder, theft
directions of the court to that effect. etc. are considered cognizable and crimes
Therefore, the Police Station officer like public nuisance, simple hurt, mischief
records the complaint as a non- cognizable etc. are considered non-cognizable.
offense, commonly referred as NCs and Learning: Generally, cognizable offence
advises the complaint or victim to approach means a police officer has the authority to
the court for further directions. make an arrest without a warrant. The police
In a cognizable offense, the police can is also allowed to start an investigation with
take cognizance of the offense on its own i.e. or without the permission of the court.
it need not wait for the court orders as the law • By contrast, in the case of a non-
envisages that in such offences permission of cognizable offence, a police officer does
the court to the police to investigate the crime not have the authority to make an arrest
is implicit. Such cases are mainly, murder, without a warrant and an investigation
attempt to murder, rape and in heinous cannot be initiated without a court order.
offences. Q.1092 “Special Category of Punishment” mentioned
Q.1090 Consider the following statements about First in a recent ruling of the Supreme Court
Information Report (FIR). applies to
1. Only with the filing of an FIR can the (a)  Cases where certiorari was issued
police start investigating a complaint. (b) Arbitration cases where any one of the
2. It is registered whenever a person parties declined to accept arbitration
gives information about a cognizable award
offence. (c) All cases handled by fast track courts
3. The police can arrest a person for a (d) Punishments more severe than life
cognizable offence without the permission imprisonment but less severe than capital
of the court. punishment
Choose the correct answer using the codes Solution: (d)
below: Learning: This special category of
(a)  1 and 2 (b)  2 and 3 Punishment for the first time was mentioned
(c)  1 and 3 (d)  All of the above in the judgement delivered on 22 July 2008 by
Solution: (d) the Supreme Court in Swamy Shraddananda
Q.1091 As a police inspector, you can file a First v. State of Karnataka.
Information Report (FIR) in which of these • The Supreme Court has moved away
cases? from death penalty (capital punishment)
(a) Minor public nuisance created by a freak after it commuted the death sentence of
(b) Waging of war by a group of people a youth for raping and killing a seven-
against the government of India year-old girl with the Special Category
(c) Forgery of some local government of Punishment.
documents • Actual life imprisonment is only
(d)  A student cheating in an exam 14 years of imprisonment. The SC in
Solution: (b) its judicial innovation awarded the
Justification: Options A, C and D come convict 25 years of jail term considering
under the category of minor offences which it appropriate punishment between
are non-cognizable in nature. The option B is death sentence and life imprisonment in
a serious cognizable offence. heinous crimes.

Indian Polity Question Bank P.301

04-Indian Polity_Q1070-1506.indd 301 8/7/2018 7:42:37 PM


Q.1093 Consider the following important legal (34) SUPREME COURT
provisions of Indian Penal Code (IPC) often
seen in news. Match them correctly with their Q.1095 The Supreme Court of India is
mandate. 1. A federal court
1. Section 499A of IPC A.Seditious writing 2. The guarantor of the fundamental rights
or speech of the citizens
2. Section 124A of IPC B. Defamation 3. The guardian of the Constitution
3. Section 295 of IPC C. Outraging 4. The enforcer of Central as well as state laws
Communal sentiments Choose the correct answer using the codes
Select the correct match from the codes below.
below. (a)  1 and 2 only (b)  2 and 3 only
(a)  1-A, 2-B, 3-C (b)  1-B, 2-A, 3-C (c)  1 and 4 only (d)  1, 2 and 3 only
(c)  1-C, 2-C, 3-B (d)  1-B, 2-C, 3-A Solution: (d)
Solution: (b) The Supreme Court stands at the top of the
Learning: Section 295 of Indian Penal integrated judicial system in the country.
Code incriminates any act that outrages the Below it, there are high courts at the state
religious feelings or sentiments of others. level. Under a high court, there is a hierarchy
Section 124A criminalizes seditious acts, of subordinate courts, that is, district courts
speech or writings. and other lower courts. This single system
Section 499A criminalizes defamation. of courts enforces both the central laws as
Section 66A (now repealed) criminalized well as the state laws, unlike in USA, where
a speech on electronic medium that has the the federal laws are enforced by the federal
potential to hurt community sentiments. judiciary and the state laws are enforced by
These have been often seen in news due to the state judiciary.
many cases like in the cases of Miss Ramya; The Supreme Court is a federal court,
JNUSU President; Aseem Trivedi and so on the highest court of appeal, the guarantor of
in recent times. the fundamental rights of the citizens and
Q.1094 In a court proceeding, DNA tests are allowed the guardian of the Constitution. Hence, the
to be used as evidence in which of the Constitution has made various provisions to
following cases? ensure its independence—security of tenure
1. Minor Civil maintenance cases of the judges, fixed service conditions for the
2. Paternity and maternity disputes judges, all the expenses of the Supreme Court
3. Certain criminal cases charged on the Consolidated Fund of India,
Choose the correct answer using the codes prohibition on discussion on the conduct of
below. judges in the legislatures, ban on practice after
(a)  1 and 2 only (b)  1 and 3 only retirement, power to punish for its contempt
(c)  2 and 3 only (d)  All of the above vested in the Supreme Court, separation of
Solution: (d) the judiciary from the executive, and so on.
The proo of DNA is a powerful tool for Courts are not law enforcers; they are the
identification purposes and is admissible upholders of the law. Laws are enforced by
evidence. DNA or Deoxyribonucleic Acid is executive agencies.
like a blue print of biological guidelines. DNA
analysis is commonly known as DNA Finger Appointment
printing, it gives a unique and specific profile
similar to a thumb impression and hence it is Q.1096 Consider the following statements about the
named as DNA Finger Printing. DNA analysis collegium system,
plays an important role in the case of murder, 1. It is specifically mentioned in Article 124
rape, paternity and maternity disputes and also of the Indian Constitution.
in cases of baby exchange. This is a technique 2. It is a system under which appointments
of “Justice through advanced science”. and transfers of judges are decided by a

P.302 For Civil Services Preliminary Examination

04-Indian Polity_Q1070-1506.indd 302 8/7/2018 7:42:38 PM


forum of the Chief Justice of India and the 1. to bear true faith and allegiance to the
four senior-most judges of the Supreme Constitution of India;
Court. 2. to uphold the sovereignty and integrity
Which of the above statements is/are incorrect? of India;
(a)  1 Only (b)  2 Only 3. to duly and faithfully and to the best of
(c) Both (d) none his ability, knowledge and judgement
Solution: (b) perform the duties of the Office without
It is a system under which appointments and fear or favour, affection or ill-will; and
transfers of judges are decided by a forum of 4. to uphold the Constitution and the laws.
the Chief Justice of India (CJI) and the four
senior-most judges of the Supreme Court. It Qualification
has no place in the Indian Constitution. Article
124 deals with the appointment of Supreme Q.1098 Consider the following in connection with the
Court judges. It says the appointment should appointment of the judges of Supreme Court:
be made by the President after consultation 1. No judge can be appointed to the Supreme
with such judges of the High Courts and the Court without the assent of the Chief
Supreme Court as the President may deem Justice of India.
necessary. The CJI is to be consulted in all 2. Only the senior most judge of the
appointments, except his or her own. In Re Supreme Court can be appointed as the
Three Judges Case (28 October 1998), the Chief Justice of India.
Court opined that the Consultation process 3. The judge for being eligible for appointment
to be adopted by the Chief justice of India to the court should have necessarily served
requires ‘consultation of plurality judges’. in the High Courts of India.
That is the sole opinion of the chief justice Which of these is/are true?
of India does not constitute the consultation (a)  1 and 2 (b)  2 and 3
process. He should consult a collegium of four (c)  1 and 3 (d)  Only 2
senior-most judges of the Supreme Court And Solution: (d)
even if two judges give an adverse opinion, The assent of the majority of the judges in
he should not send the recommendation the 4 judge collegiums (including CJI) is
to the government. The court held that the necessary for appointment of the judges of
recommendation made by the chief justice of the court. Sole opinion of the CJI does not
India without complying with the norms and stand much ground.
requirements of the consultation process are A learned jurist or a legal luminary can
not binding on the government. also be appointed as a judge. This is in
view of adding academic expertise to the
Q.1097 The Judges of the Supreme Court take an
judgements of the SC.
oath or affirmation before entering upon
office, conducted by:
(a)  Chief Justice of India Removal of Judges
(b)  President or Vice-President Q.1099 A judge of the Supreme Court can be
(c) President or some person appointed, removed from his Office by an order of the
by him president after an address by Parliament has
(d)  None of the above been presented to him in the same session for
Solution: (c) such removal supported by
A person appointed as a judge of the Supreme (a)  Special majority in Lok Sabha only
Court, before entering upon his Office, has to (b) Absolute majority in both Houses of
make and subscribe an oath or affirmation Parliament
before the President, or some person (c) Simple majority in each house of
appointed by him for this purpose. Parliament approved by the Presiding
In his oath, a judge of the Supreme officer of the House
Court swears:

Indian Polity Question Bank P.303

04-Indian Polity_Q1070-1506.indd 303 8/7/2018 7:42:38 PM


(d) Special majority in both Houses of Learning: If the committee finds the judge
Parliament to be guilty of misbehaviour or suffering
Solution: (d) from incapacity, the House can take up the
Learning: The President can issue the consideration of the motion.
removal order only after an address by • After the motion is passed by each House
Parliament has been presented to him in the of Parliament by special majority, an
same session for such removal. address is presented to the president for
• The address must be supported by a special removal of the judge.
majority of each House of Parliament • Finally, the president passes an order
(i.e., a majority of the total membership of removing the judge.
that House and a majority of not less than Q.1101 Consider the following statements about the
two-thirds of the members of that House removal of judges of Supreme Court and
present and voting). High Courts.
• The grounds of removal are two—proved 1. Motion to impeach a judge is admitted
misbehaviour or incapacity. by the respective chairman/speaker of the
• The Judges Enquiry Act (1968) regulates houses of Parliament.
the procedure relating to the removal of a 2. Judges cannot be removed without
judge of the Supreme Court by the process an inquiry by a bench of the Supreme
of impeachment. Court.
Q.1100 The Judges Enquiry Act (1968) regulates 3. Absolute majority is required in the
the procedure relating to the removal of a Parliament to remove judges.
judge of the Supreme Court by the process Choose the correct answer using the codes
of impeachment. Consider the following with below:
reference to the procedure. (a)  1 and 2 (b)  2 and 3
1. The prior permission of President is (c)  1 and 3 (d)  All of the above
necessary for introducing any motion to Solution: (a)
impeach a judge. The removal of judges of the Supreme
2. The charges against the judge must be Court and the High Courts is also extremely
investigated by a Parliamentary panel difficult. A judge of the Supreme Court
appointed by the presiding officer of or High Court can be removed only on the
the house. ground of proven misbehaviour or incapacity.
Which of the above is/are correct? A motion containing the charges against the
(a)  1 only (b)  2 only judge must be approved by special majority
(c)  Both 1 and 2 (d)  None in both Houses of the Parliament.
Solution: (b) It is clear from this procedure that removal
Justification: Statement 1: No prior consent of a judge is a very difficult procedure and
of President is needed. However, a removal unless there is a general consensus among
motion signed by 100 members (in the case of Members of the Parliament, a judge cannot
Lok Sabha) or 50 members (in the case of Rajya be removed. It should also be noted that while
Sabha) is to be given to the Speaker/Chairman. in making appointments, the executive plays
Statement 2: The Speaker/Chairman may a crucial role; the legislature has the powers
admit the motion or refuse to admit it. If it of removal. This has ensured both balance of
is admitted, then the Speaker/Chairman is power and independence of the judiciary. So
to constitute a three-member committee to far, only one case of removal of a judge of
investigate into the charges. the Supreme Court came up for consideration
The committee should consist of before the Parliament. In that case, though
• the chief justice or a judge of the Supreme the motion got two-thirds majority, it did not
Court have the support of the majority of the total
• a chief justice of a high court strength of the House and therefore, the judge
• a distinguished jurist. was not removed.

P.304 For Civil Services Preliminary Examination

04-Indian Polity_Q1070-1506.indd 304 8/7/2018 7:42:38 PM


Independence of Supreme Court hold their office during the pleasure of the
President, though they are appointed by him.
Q.1102 The Constitution of India establishes an Statement 2: While it cannot be changed
independent judiciary to serve which of during normal circumstances, it can be
these purposes? changed during a financial emergency. So,
1. To protect the supremacy of the the option 2 is wrong.
Constitution The salaries, allowances, privileges, leave
2. To settle any disputes between Centre and pension of the judges of the Supreme
and states Court are determined from time to time by
3. For the judiciary to be able to exercise the the Parliament.
power of Judicial review Statement 3: They can be discussed, but
Choose the correct answer using the codes they are non-votable. So, the option 3 is wrong.
below. They are expenses Charged on Consolidated
(a)  1 and 2 only (b)  2 and 3 only Fund of India.
(c)  1 and 3 only (d)  All of the above Learning: Other such provisions are:
Solution: (d) • Conduct of Judges cannot be discussed in
An independent judiciary is needed: one, to Parliament or in a State Legislature except
protect the supremacy of the Constitution when an impeachment motion is under
by exercising the power of judicial review; consideration of the Parliament.
and two, to settle the disputes between the • Ban on Practice after Retirement
Centre and the states or between the states. • Power to Punish for its Contempt
The Constitution contains various measures
Q.1104 The Indian Constitution has ensured the
like security of tenure to judges, fixed service
independence of the judiciary through
conditions and so on to make the judiciary
a number of measures. Which of the
independent of the government. Also,
following are these measures provided in
since the judiciary is also administratively
the Constitution?
controlled by the higher judiciary,
1. The Executive is not involved in the
independence is strengthened.
process of appointment of judges.
Q.1103 The Constitution has made which of the 2. The judges have a fixed tenure.
following provisions to safeguard and ensure 3. The expenses of the judiciary are charged
the independent and impartial functioning of on the Consolidated Fund of India.
the Supreme Court? 4. The Supreme court and the High Court
1. The judges of the Supreme Court are can punish any person for its contempt
provided with the Security of Tenure. 5. A judge of Supreme court can only be
2. Their service conditions cannot be removed by a special majority of the
changed to their disadvantage after their Parliament.
appointment even during a financial Select the correct code.
emergency. (a)  2, 3, and 5 Only
3. The salaries, allowances and pensions of (b)  1, 2, 3 and 5 Only
judges cannot be discussed and are non- (c)  2, 3, 4 and 5 Only
votable by the Parliament. (d)  1, 2, 3, 4 and 5
Select the correct answer using the codes Solution: (c)
below. The legislature is not involved in the process of
(a)  1 and 2 only (b)  2 and 3 only appointment of judges. Thus, it was believed
(c)  1 only (d)  1, 2 and 3 that party politics would not play a role in the
Solution: (c) process of appointments. But the judges of the
Justification: Statement 1: They can be Supreme Court and High Court are appointed
removed from office by the President only by the President in consultation with the
in the and on the grounds mentioned in the members of the Judiciary. It should also be
Constitution. This means that they do not noted that while in making appointments, the

Indian Polity Question Bank P.305

04-Indian Polity_Q1070-1506.indd 305 8/7/2018 7:42:38 PM


executive plays a crucial role; the legislature may impede their functioning. It ensures
has the powers of removal. Hence the first independence of judiciary which is crucial
statement is wrong. In order to be appointed for rule of law.
as a judge, a person must have experience as Q.1106 Which of the following ensure judicial
a lawyer and/or must be well versed in law. independence in India?
Political opinions of the person or his/her 1. Security of tenure for judges
political loyalty should not be the criteria for 2. Ban on practice after retirement of judges
appointments to judiciary. 3. Prohibition on discussion on the conduct
The judges have a fixed tenure. They hold of judges in the legislatures
office till reaching the age of retirement. 4. Power of the courts to punish for contempt
Only in exceptional cases, judges may be Which of the following codes is correct?
removed. But otherwise, they have security (a)  1, 2 and 3 (b)  2, 3 and 4
of tenure. Security of tenure ensures that (c)  1, 3 and 4 (d)  All of the above
judges could function without fear or favour. Solution: (d)
The Constitution prescribes a very difficult
Q.1107 Which of the following would be affected
procedure (special majority is required)  for
if India does not have an Independent
removal of judges. The removal of judges of
Judiciary?
the Supreme Court and the High Courts is also
1. Rule of Law and Supremacy of Law
extremely difficult. A judge of the Supreme
2. Enforcement of Fundamental Rights
Court or High Court can be removed only
3. Legitimacy of Indian democracy
on the ground of proven misbehaviour or
Choose the correct answer using the codes
incapacity. A motion containing the charges
below:
against the judge must be approved by special
(a)  1 and 2 (b)  2 and 3
majority in both Houses of the Parliament.
(c)  1 and 3 (d)  All of the above
The Constitution makers believed that a
Solution: (d)
difficult procedure of removal would provide
In any society, disputes are bound to arise
security of office to the members of judiciary.
between individuals, between groups
Q.1105 Article 121 of the Constitution says that “no and between individuals or groups and
discussion shall take place in Parliament government.
with respect to the conduct of any judge All such disputes must be settled by an
of the Supreme Court or of a high court in independent body in accordance with the
the discharge of his duties…”. What is the principle of rule of law. This idea of rule of
principle behind this provision? law implies that all individuals—rich and
(a)  Separation of powers poor, men or women, forward or backward
(b)  Popular Sovereignty castes—are subjected to the same law. The
(c)  Supremacy of Constitution principal role of the judiciary is to protect
(d) Principle of Collective Responsibility rule of law and ensure supremacy of law. It
Solution: (a) safeguards rights of the individual, settles
Justification: Popular Sovereignty implies disputes in accordance with the law and
rule for and by the people; it does not imply ensures that democracy does not give way
separation of powers necessarily. So, the to individual or group dictatorship. In order
option (b) cannot be the answer. to be able to do all this, it is necessary
Supremacy of Constitution is crucial for a that the judiciary is independent of any
democracy, but does not guarantee separation political pressures.
of powers, for example, there is fusion
Q.1108 Which of the following is/are a part of an
of powers to an extent in India between
Independent Judicial system?
legislature and executive.
1. Judiciary is not accountable to the
Article 121 is based on separation of
legislature.
powers, where judges should not be subject
2. Judicial appointments are not affected
to minute scrutiny by the legislature which
solely by political considerations.

P.306 For Civil Services Preliminary Examination

04-Indian Polity_Q1070-1506.indd 306 8/7/2018 7:42:38 PM


3. The Executive is bound to follow and (c)  Supreme Court alone
implement the decisions of the Judiciary (d)  Supreme Court in consultation with the
even if it disagrees with it. Bar Council of India
Choose the correct answer using the codes Solution: (b)
below:
(a)  1 and 2 (b)  2 and 3 Jurisdiction and Power of
(c)  1 and 3 (d)  All of the above
Solution: (b)
Supreme Court
Simply stated independence of judiciary Q.1111 Match the following.
means that
• the other organs of the government like Jurisdiction of
the executive and legislature must not the Supreme
restrain the functioning of the judiciary in Court Cases Covered
such a way that it is unable to do justice. A. Original 1. Advice on any
• the other organs of the government should Jurisdiction question of law as
not interfere with the decision of the may be referred to
judiciary. the Supreme Court
• judges must be able to perform their for consideration
functions without fear or favour. by the President
Independence of the judiciary does not imply B. Appellate 2. Case involving
arbitrariness or absence of accountability. Jurisdiction interpretation of
Judiciary is a part of the democratic political the Constitution
structure of the country. It is therefore
C. Advisory 3. Appointment
accountable to the Constitution, to the
Jurisdiction of officers and
democratic traditions and to the people of
servants of the
the country.
Supreme Court
Q.1109 When can the salaries of the judges of the
Supreme Court be reduced during their term 4. Dispute between
of office? the Government of
(a) If the Parliament passes a bill to this India and a State
effect ABC
(b)  During a Financial Emergency (a)  1 2 3 (b)  4 3 2
(c)  As and when the President desires (c)  4 2 1 (d)  2 1 3
(d) Never Solution: (c)
Solution: (b)
Article 360 of the Constitution of India (1) Original Jurisdiction
says that if any Proclamation of financial Q.1112 A dispute between two or more states will fall
emergency is in operation, it is competent for under Supreme Court’s
the President of India to issue directions for (a)  Exclusive Original Jurisdiction
the reduction of salaries and allowances of all (b)  Appellate Jurisdiction
or any class of persons serving in connection (c)  Writ Jurisdiction
with the affairs of the Union but excluding (d)  Advisory Jurisdiction
the Judges of the Supreme Court and the Solution: (a)
High Courts. Learning: Original jurisdiction means
Q.1110 The rules for regulating the practice and cases that can be directly considered by the
procedure of Supreme Court under Article Supreme Court without going to the lower
145 of the Constitution are made by the: courts before that.
(a)  President of India As a federal court, the Supreme Court
(b)  Supreme Court with the approval of the decides the disputes between different units
President of India

Indian Polity Question Bank P.307

04-Indian Polity_Q1070-1506.indd 307 8/7/2018 7:42:38 PM


of the Indian Federation. More elaborately, Q.1114 Which of the following would come under
any dispute between: the Original Jurisdiction of the Supreme
• The Centre and one or more states; or Court of India?
• The Centre and any state or states on one 1. Advising the President on matters of
side and one or more states on the other; or public importance and law.
• Between two or more states. 2. Granting special leave to an appeal from
In the above federal disputes, the Supreme any judgement or matter passed by any
Court has exclusive original jurisdiction. court in the territory of India.
Exclusive means, no other court can decide 3. Settling disputes between Union and
such disputes and original means, the power States and amongst States.
to hear such disputes in the first instance, not Choose the correct answer using the codes
by way of appeal. below:
Q.1113 The Supreme Court of India has original (a)  1 and 2 (b)  2 and 3
jurisdiction in any dispute arising: (c)  3 only (d)  1 and 3
1. Between the Government of India and one Solution: (c)
or more States. Q.1115 Which of the following institutions resolves
2. Between the Government of India and the disputes between Centre and States
any State or States on one side and one or arising out of official decisions?
more States on the other. (a)  Union Finance Commission (UFC)
3. Between two or more States. (b)  Supreme Court (SC)
Choose the correct answer using the codes (c)  Central Administrative Tribunal (CAT)
below: (d)  Chairman, Rajya Sabha
(a)  Only 1 (b)  Only 2 & 3 Solution: (b)
(c)  Only 2 (d)  All Explanation: Finance Commission is not
Solution: (d) a dispute resolving body. It recommends
In addition, Article 32 of the Constitution allocation of tax resources between Centre
gives an extensive original jurisdiction to the and states. So, the option (a) is wrong.
Supreme Court in regard to enforcement of CAT resolves disputes between the
Fundamental Rights. It is empowered to issue officials and government with regard to
directions, orders or writs, including writs personnel matters like promotion, recruitment
in the nature of habeas corpus, mandamus, etc. So, option (c) is wrong.
prohibition, quo warranto and certiorari to Chairman, RS, only presides over the
enforce them. The Supreme Court has been meetings of the Rajya Sabha. Rajya Sabha
conferred with power to direct transfer of any may discuss disputes over decisions of Centre
civil or criminal case from one State High and States. It cannot pass a final and binding
Court to another State High Court or from judgment alone. So, option (d) is wrong.
a Court subordinate to another State High Supreme Court and the High Courts
Court. The Supreme Court, if satisfied that in India settle disputes arising out of
cases involving the same or substantially the governmental decisions. For e.g. reservation
same questions of law are pending before it of creamy layer Other Backward Classes
and one or more High Courts or before two or (OBCs) in the case of Indira Sawhney v. UOI,
more High Courts and that such questions are AIR 1993 SC 477.
substantial questions of general importance, Q.1116 Which of the following do not come under
may withdraw a case or cases pending before the original jurisdiction of the Supreme
the High Court or High Courts and dispose Court?
of all such cases itself. Under the Arbitration 1. Inter-state water disputes
and Conciliation Act, 1996, International 2. Matters related to the finance commission
Commercial Arbitration can also be initiated 3. A suit by a private citizen against the
in the Supreme Court. Centre or a state.

P.308 For Civil Services Preliminary Examination

04-Indian Polity_Q1070-1506.indd 308 8/7/2018 7:42:38 PM


Choose the correct answer using the codes 1. SC holds this power exclusively.
below: 2. Citizens may approach the SC directly
(a)  1 and 2 (b)  Only 3 without necessarily going through appeals
(c)  2 and 3 (d)  All of the above from lower courts.
Solution: (d) Which of the above is/are correct?
Other disputes of commercial nature between (a)  1 only (b)  2 only
the centre and the states; questions of (c)  Both 1 and 2 (d)  None
political nature; disputes arising out of a pre- Solution: (b)
constitutional agreement or treaty etc. also Justification: Supreme Court has been
do not come under original jurisdiction of the constituted as the defender and guarantor of
Supreme Court. the fundamental rights of the citizens. It has
Q.1117 How the original jurisdiction of the Supreme been vested with the ‘original’ and ‘wide’
Court with regard to federal disputes is different powers for that purpose.
from its original jurisdiction with regard to • It is original, because an aggrieved citizen
disputes relating to fundamental rights? can directly go to the Supreme Court, not
1. The aggrieved party can move either necessarily by way of appeal.
Supreme Court or High Court in case of • It is wide, because its power is not
fundamental right violations, but only restricted to issuing of orders or directions
Supreme Court in case of federal disputes. but also writs of all kinds.
2. The Supreme Court can issue writs for Statement 1: In case of the enforcement of
the settlement of federal disputes, but not Fundamental Rights, the jurisdiction of the
those related to fundamental rights. Supreme Court is original but not exclusive.
Which of the above is/are correct? It is concurrent with the jurisdiction of the
(a)  1 only (b)  2 only high court under Article 226.
(c)  Both 1 and 2 (d)  None It vests original powers in the high court
Solution: (a) to issue directions, orders and writs of all
Justification: Statement 1: The federal dispute kinds for the enforcement of the Fundamental
jurisdiction is exclusive to the Supreme Court. Rights. It means when the Fundamental
It cannot be raised in the High Courts. Rights of a citizen are violated, the aggrieved
But, when the Fundamental Rights of a party has the option of moving either the high
citizen are violated, the aggrieved party has court or the Supreme Court directly.
the option of moving either the high court or Q.1119 How is the writ jurisdiction of the Supreme
the Supreme Court directly. Court (SC) different from that of a High
In this case, it is concurrent with high Court (HC)?
court’s jurisdiction. 1. SC can issue writs for any purpose,
Statement 2: There is also a difference whereas High Courts can only issue them
between the writ jurisdiction of the Supreme for the enforcement of fundamental rights.
Court and that of the high court. 2. SC can issue writ against the government,
The Supreme Court can issue writs only for whereas High Courts cannot.
the enforcement of the Fundamental Rights Which of the above is/are correct?
and not for other purposes. The high court, on (a)  1 only (b)  2 only
the other hand, can issue writs not only for (c)  Both 1 and 2 (d)  None
the enforcement of the fundamental rights Solution: (d)
but also for other purposes. Justification: Statement 1: The Supreme
Court can issue writs only for the enforcement
(2) Writ Jurisdiction of fundamental rights whereas a high court
can issue writs not only for the enforcement
Q.1118 Supreme Court has been vested with
of Fundamental Rights but also for any
‘original’ and ‘wide’ powers for defending
other purpose.
the fundamental rights of citizens. What do
you understand by original powers of SC?

Indian Polity Question Bank P.309

04-Indian Polity_Q1070-1506.indd 309 8/7/2018 7:42:38 PM


Statement 2: The Supreme Court can • A remedy under Article 32 is in itself a
issue writs against a person or government Fundamental Right and hence, the Supreme
throughout the territory of India whereas a Court may not refuse to exercise its writ
high court can issue writs against a person jurisdiction. On the other hand, a remedy
residing or against a government or authority under Article 226 is discretionary and
located within its territorial jurisdiction only hence, a high court may refuse to exercise
or outside its territorial jurisdiction only if its writ jurisdiction. Article 32 does not
the cause of action arises within its territorial merely confer power on the Supreme Court
jurisdiction. as Article 226 does on a high court to issue
Q.1120 The writ jurisdiction of the Supreme Court is writs for the enforcement of fundamental
narrower and less discretionary than that of a rights or other rights as part of its general
high court in what respects? jurisdiction. The Supreme Court is thus
1. High courts can enforce legal rights too, constituted as a defender and guarantor of
Supreme Court cannot. the fundamental rights.
2. High court can issues it both against
persons and state, SC can issue it only (3) Appellate Jurisdiction
against the later. Q.1121 The Supreme Court enjoys appellate
3. The Supreme Court cannot refuse to issues jurisdiction in
writs when it comes to Fundamental 1. Civil matters
Rights, High courts can. 2. Criminal matters
Choose the correct answer using the codes 3. Constitutional matters
given below. Select the correct answer using the codes
(a)  1 and 2 only (b)  2 and 3 only below.
(c)  1 and 3 only (d)  All of the above (a)  1 and 2 only (b)  2 and 3 only
Solution: (c) (c)  1 and 3 only (d)  1, 2 and 3
The writ jurisdiction of the Supreme Court Solution: (d)
differs from that of a high court in three Learning: The Supreme Court is primarily
respects: a court of appeal and hears appeals against
• The Supreme Court can issue writs only the judgements of the lower courts. It enjoys
for the enforcement of fundamental rights a wide appellate jurisdiction which can be
whereas a high court can issue writs not classified under four heads:
only for the enforcement of Fundamental 1. Appeals in constitutional matters.
Rights but also for any other purpose. 2. Appeals in civil matters.
The expression ‘for any other purpose‘ 3. Appeals in criminal matters.
refers to the enforcement of an ordinary 4. Appeals by special leave.
legal right. Thus, the writ jurisdiction In the constitutional cases, an appeal can
of the Supreme Court, in this respect, is be made to the Supreme Court against the
narrower than that of high court. judgement of a high court if the high court
• The Supreme Court can issue writs against certifies that the case involves a substantial
a person or government throughout the question of law that requires the interpretation
territory of India whereas a high court of the Constitution.
can issue writs against a person residing
Q.1122 In which of the following cases appeals
or against a government or authority
cannot be lodged with the Supreme Court ?
located within its territorial jurisdiction
(a)  Constitutional matters involving a
only or outside its territorial jurisdiction
substantial point of law
only if the cause of action arises within its
(b) A sentence in a criminal case given by
territorial jurisdiction. Thus, the territorial
the lower court and confirmed by the
jurisdiction of the Supreme Court for the
High Court
purpose of issuing writs is wider than that
of a high court.

P.310 For Civil Services Preliminary Examination

04-Indian Polity_Q1070-1506.indd 310 8/7/2018 7:42:38 PM


(c)  Criminal case in which High Court has of importance before taking action on it. This
given a death sentence J a criminal may prevent unnecessary litigations later.
found nonguilty by a lower court Secondly, in the light of the advice of the
(d)  Criminal case in which High Court after Supreme Court, the government can
withdrawing a case from the lower Q.1125 Article 143 of the Constitution authorises the
court, has given a death sentence president to seek the opinion of the Supreme
Solution: (b) Court on any question of law or fact of public
importance which is likely to arise. Consider
(4) Advisory Jurisdiction the following about it.
Q.1123 Under Article 143 of the Constitution, 1. The Supreme Court is bound to given its
the Supreme Court can exercise advisory advice to the President.
jurisdiction to advise 2. The advice of the SC is binding on the
1. President 2. Governors President and thus the Government.
3. Prime Minister 4.  Chief Ministers Which of the above is/are correct?
Select the correct answer using the codes (a)  1 only (b)  2 only
below: (c)  Both 1 and 2 (d)  None
(a)  1, 2 and 3 only (b)  2, 3 and 4 only Solution: (d)
(c)  1 and 2 only (d)  1 only Justification: Two categories are covered by
Solution: (d) Article 143:
Refer previous explanations. 1. On any question of law or fact of public
importance which has arisen or which is
Q.1124 Consider the following about the advisory
likely to arise.
jurisdiction of the Supreme Court (SC) under
2. On any dispute arising out of any pre-
Article 143 of the constitution.
constitution treaty, agreement, covenant,
1. Being a constitutional organ, the SC is
engagement or other similar instruments.
bound to give advice when it is referred.
In the first case, the Supreme Court may
2. The SC being the highest court of Justice,
tender or may refuse to tender its opinion
legal advice tendered by the court is
to the president. But, in the second case, the
legally binding on the President.
Supreme Court ‘must’ tender its opinion to
Which of the above is/are true?
the president. In both the cases, the opinion
(a)  1 only (b)  2 only
expressed by the Supreme Court is only
(c)  Both 1 and 2 (d)  None
advisory and not a judicial pronouncement.
Solution: (d)
It makes suitable changes in its action or Q.1126 Which of the following statements regarding
legislations. the advisory jurisdiction of the Supreme
In addition to original and appellate Court are correct?
jurisdiction, the Supreme Court of India 1. It is binding on the Supreme Court to give
possesses advisory jurisdiction also. This advice to the President every time it is
means that the President of India can refer referred to.
any matter that is of public importance or that 2. The opinion tendered by the Supreme
which involves interpretation of Constitution Court is binding on the President.
to Supreme Court for advice. However, the Choose the correct answer using the codes
Supreme Court is not bound to give advice on below.
such matters and the President is not bound to (a)  1 only (b)  2 only
accept such an advice. (c)  Both 1 and 2 (d)  None
What then is the utility of the advisory Solution: (d)
powers of the Supreme Court? The utility Refer previous explanations.
is two-fold. In the first place, it allows the Q.1127 Which of the following statements regarding
government to seek legal opinion on a matter the advisory jurisdiction of the Supreme
Court are correct?

Indian Polity Question Bank P.311

04-Indian Polity_Q1070-1506.indd 311 8/7/2018 7:42:38 PM


1. It is binding on the Supreme Court to give Which of the above is/are correct?
its opinion on any matter referred to it by (a)  1 only (b)  2 only
the President. (c)  Both 1 and 2 (d)  None
2. The full bench of the Supreme Court hears Solution: (b)
any reference made to it under its power Justification: Statement   1: The official
of advisory jurisdiction. language of the court is English, and the court
3. The opinion given by the Supreme Court can reject submissions made in languages
on a reference under advisory jurisdiction other than English. So, the statement 1 is
is not binding on the government. wrong.
Choose the correct answer from the codes Statement 2: As a Court of Record, the
given below: Supreme Court has two powers:
(a)  1 and 2 (b)  1 and 3 • The judgements, proceedings and acts
(c)  2 and 3 (d)  Only 3 of the Supreme Court are recorded for
Solution: (d) perpetual memory and testimony. These
Statement 2: The Constitutional cases or records are admitted to be of evidentiary
references made by the President under value and cannot be questioned when
Article 143 are decided by a Bench consisting produced before any court. They are
of at least five judges. All other cases are recognised as legal precedents and legal
usually decided by a bench consisting of not references.
less than three judges • Further, It has power to punish for
Q.1128 Which of the following is implied by contempt of court. In 1991, the Supreme
Advisory jurisdiction of the Supreme Court? Court has ruled that it has power to punish
1. The President of India can refer any matter for contempt not only of itself but also
that is of public importance or that which of high courts, subordinate courts and
involves interpretation of Constitution to tribunals functioning in the entire country.
Supreme Court for advice. Q.1130 Supreme court as court of record would mean
2. The advice of Supreme Court is binding (a) Its judgments are recognised as legal
on the President. precedents and legal references.
3. This jurisdiction of the Supreme Court (b) Its records all the judgments made by all
may prevent unnecessary litigations. subordinate courts in India.
4. The President is not bound to accept such (c) Proceedings in Supreme Court are
an advice. always recorded by way of writing or
Select the correct code. digital media.
(a)  2 and 3 Only (b)  1 and 4 Only (d) All courts in India are subordinate to the
(c)  1, 3 and 4 Only (d)  1, 2, 3 and 4 Supreme Court.
Solution: (c) Solution: (a)
Refer previous explanations. Refer previous explanations.

(5) Court of Record (6) Power of Judicial Review


Q.1129 As a “Court of Record”, the Supreme Court Q.1131 The constitutional validity of a legislative
has which of the following powers? enactment or an executive order can be
1. The Court is bound to accept evidences challenged in the Supreme Court on which of
and proceedings in all official languages the following grounds?
mentioned in the Eighth schedule of the 1. If it infringes the Fundamental Rights
constitution. 2. If it is repugnant to the constitutional
2. The judgments of Supreme Court are provisions
admitted as evidentiary value and legal 3. If it is outside the competence of the
precedents. authority which has framed it

P.312 For Civil Services Preliminary Examination

04-Indian Polity_Q1070-1506.indd 312 8/7/2018 7:42:38 PM


Select the correct answer using the codes personal liberty of a person, but also see if
below. the law made is fair, just and not arbitrary.
(a)  1 and 2 only (b)  2 and 3 only If SC finds that any law as not fair, it will
(c)  1 and 3 only (d)  1, 2 and 3 declare it as null and void. This doctrine
Solution: (d) provides for more fair treatment of individual
Learning: Judicial review is the power rights. Under due process, it is the legal
of the Supreme Court to examine the requirement that the state must respect all
constitutionality of legislative enactments of the legal rights that are owed to a person
and executive orders of both the Central and and laws that states enact must confirm to the
state governments. On examination, if they laws of the land like – fairness, fundamental
are found to be violative of the Constitution rights, liberty etc. It also gives the judiciary
or fundamental rights (ultra vires), they can to access the fundamental fairness, justice,
be declared as illegal, unconstitutional and and liberty of any legislation
invalid (null and void)  by the Supreme Court. However in Maneka Gandhi v. UOI, 1978
Consequently, they cannot be enforced by AIR 597, Indian supreme court adopted
the Government. Judicial review is needed due process of law expanding the scope of
for the following reasons: judicial review.
1. To uphold the principle of the supremacy Q.1133 Which of the following power vests with the
of the Constitution. Supreme Court in India?
2. To maintain federal equilibrium (balance 1. Interpreting the constitution
between Centre andstates). 2. It can declare invalid any law of the
3. To protect the fundamental rights of the legislature or the actions of the executive,
citizens whether at the Union level or at the state
Q.1132 Judicial review in India is based on the level, if it finds such a law or action is
‘procedure established by law’ contained in against the Constitution.
the Indian Constitution (Article 21). What is 3. It makes appointments of the judges of
meant by this principle? High court and district courts.
(a) Courts can review a Parliamentary Choose the correct answer using the codes
enactment if it is not reasonable. below:
(b) Courts can review a Parliamentary (a)  All of the above (b)  1 and 3 only
enactment if it goes against the goals and (c)  1 and 2 only (d)  2 and 3 only
objectives enshrined in the constitution. Solution: (c)
(c) Courts can review a Parliamentary Appointments to the High Courts are made
enactment only if it was not enacted by the President. Governor does the same for
following the right procedure. district courts.
(d)  None of the above Q.1134 Consider the following statements about
Solution: (c) judicial review.
Procedure established by law means that 1. The term ‘judicial review’ is mentioned
a law that is duly enacted by legislature nowhere in the Constitution.
or the concerned body is valid if it has 2. Supreme Court can use the review powers
followed the correct procedure. Following if a law is inconsistent with the distribution
this doctrine means that, a person can be of powers laid down by the Constitution.
deprived of his life or personal liberty 3. The review power extends to the laws
according to the procedure established by passed by State legislatures also.
law. So, if Parliament pass a law, then the Choose the correct answer using the codes
life or personal liberty of a person can be below:
taken off according to the provisions and (a)  1 and 2 (b)  2 and 3
procedures of the that law. (c)  1 and 3 (d)  All of the above
Due process of law doctrine not only Solution: (d)
checks if there is a law to deprive the life and

Indian Polity Question Bank P.313

04-Indian Polity_Q1070-1506.indd 313 8/7/2018 7:42:38 PM


Perhaps the most important power of the Select the correct code.
Supreme Court is the power of judicial (a)  4 and 5 Only (b)  1, 3 and 5 Only
review. Judicial Review means the power (c)  1, 2, 3 and 5 Only (d)  1, 2, 3, 4 and 5
of the Supreme Court (or High Courts) to Solution: (c)
examine the constitutionality of any law if The structure of the judiciary in India is
the Court arrives at the conclusion that the pyramidal with the Supreme Court at the
law is inconsistent with the provisions of top, High Courts below them and district
the Constitution, such a law is declared as and subordinate courts at the lowest level.
unconstitutional and inapplicable. The term The lower courts function under the direct
judicial review is nowhere mentioned in the superintendence of the higher courts. Supreme
Constitution. However, the fact that India Court of India:
has a written constitution and the Supreme Its decisions ‘are’ binding on all courts and
Court can strike down a law that goes against can transfer Judges of High Courts. Can move
fundamental rights, implicitly gives the cases from any court to itself
Supreme Court the power of judicial review.
In the case of federal relations too, the American versus Indian Judiciary
Supreme Court can use the review powers Consider the following statement about the
if a law is inconsistent with the distribution comparison of powers between the American
of powers laid down by the Constitution. and Indian judiciary.
Suppose, the central government makes 1. The doctrine of judicial supremacy in the
a law, which according to some States, Indian constitution is associated with that
concerns a subject from the State list- then of the American Supreme court (SC).
the States can go to the Supreme Court 2. The scope of judicial powers of the
and if the court agrees with them, it would Supreme Court in the U.S.A. is broader
declare that the law is unconstitutional. In than what exists in India.
this sense, the review power of the Supreme 3. The powers of judicial review of laws
Court includes power to review legislations enacted by legislature are greater with the
on the ground that they violate fundamental U.S.A Supreme Court as compared to the
rights or on the ground that they violate the Indian SC.
federal distribution of powers. The review Which of the following statements is/are
power extends to the laws passed by State true? Choose the correct option using the
legislatures also. codes below:
Together, the writ powers and the review (a)  1 and 2 (b)  2 and 3
power of the Court make judiciary very (c)  1 and 3 (d)  All of the above
powerful. In particular, the review power Solution: (c)
means that the judiciary can interpret the Comparing Indian and American Supreme
Constitution and the laws passed by the Courts
legislature.
Indian Supreme American Supreme
(7) Other Powers Court Court
Which of the following statements is/are 1. Its original 1. Its original
correct with reference to the powers of the jurisdiction jurisdiction covers
Supreme Court of India? is confined to not only federal
1. It can transfer any judge of a High Court. federal cases cases but also
2. It can deal with any case within the cases relating
jurisdiction of the State. to naval forces,
3. It can move cases from any court to itself. maritime activities,
4. Its decisions may be binding on all courts. ambassadors, etc.
5. It can transfer cases from one High Court
to another High Court

P.314 For Civil Services Preliminary Examination

04-Indian Polity_Q1070-1506.indd 314 8/7/2018 7:42:38 PM


2. Its appellate 2. Its appellate Choose the correct answer using the codes
jurisdiction covers jurisdiction below.
constitutional, is confined to (a)  1 and 2 only (b)  1 and 3 only
civil and criminal constitutional (c)  2 and 3 only (d)  1 only
cases. cases only. Solution: (d)
Refer previous explanations.
3. It has a very wide 3. It has no such
discretion to grant plenary power. Some of Cases
special leave to
appeal in any Q.1136 The “rarest of the rare” principle in handing
matter against the out the death penalty was given by the
judgement of any Supreme Court in which case?
court or tribunal (a)  Shatrughan Chauhan v. UOI, 2014
(except military). (b)  Bachan Singh v. State of Punjab, 1980
4. It has advisory 4. It has no advisory (2) SCC 684.
jurisdiction. jurisdiction. (c)  Mithu v. State of Punjab, 1983 AIR 473
(d)  Ladani v. UOI
5. Its scope of judicial 5. Its scope of judicial Solution: (b)
review is limited. review is very wide. The Supreme Court in Mithu v. State of
6. It defends rights 6. It defends rights Punjab, 1983 AIR 473 struck down Section
of the citizen of the citizen 303 of the Indian Penal Code, which provided
according to according to the for a mandatory death sentence for offenders
the ‘procedure ‘due process of serving a life sentence.
established by law’. In Bachan Singh v. State of Punjab,
law’. 1980 (2) SCC 684, it gave the rarest of rate
7. Its jurisdiction 7. Its jurisdiction and doctrine according to which death penalty
and powers can powers are limited should be given only in the extreme cases
be enlarged by to that conferred by and where life sentence should not at all be
Parliament. the Constitution. appropriate. Last year, while handing down
its Shatrughan Chauhan v. UOI, 2014, the
8. It has power 8. It has no such
Supreme Court spelt out clear guidelines
of judicial power due
on the legal rights of prisoners on death
superintendence to double (or
row.
and control over separated) judicial
state high courts system. Q.1137 Which of the following cases in the Supreme
due to integrated Court of India is related to resolving the
judicial system. conflict between Fundamental Rights and
Directive Principles?
Q.1135 Which of the following correctly understate 1. L. C. Golaknath v. State of Punjab, 1967
the difference(s) between the Indian and AIR 1643
American (USA) Supreme Court? 2. State of Madras v. Champakam
1. While the Indian Supreme Court’s Dorairajan, AIR 1951 SC 226
jurisdiction extends to constitutional, 3. Minerva Mills Ltd. v. UOI, AIR 1980 SC
civil and criminal cases; the American 1789
Supreme Court jurisdiction is confined to 4. Bhikan Singh case
constitutional cases only. Select the correct answer using the codes
2. The American Supreme Court does not below.
defend the fundamental rights of citizens, (a)  1 and 3 only (b)  2, 3 and 4 only
unlike the Indian Supreme Court. (c)  1 and 4 only (d)  1, 2 and 3 only
3. The American Supreme Court does not Solution: (d)
have any powers of judicial review, unlike
with the Indian Supreme Court.

Indian Polity Question Bank P.315

04-Indian Polity_Q1070-1506.indd 315 8/7/2018 7:42:38 PM


Learning: In the case of State of Madras v. (a) D. C. Wadhwa v. State of Bihar, 1987
Champakam Dorairajan, AIR 1951 SC 226,, AIR 579
the Supreme Court ruled that in case of any (b)  S. R. Bommai v UOI, 1994 AIR 1918
conflict between the Fundamental Rights and (c)  Rameshwar Prasad v. UOI, 2006
the Directive Principles, the former would (d) M. C. Mehta v. UOI, 1987 SCR (1) 819
prevail. Solution: (b)
In L. C. Golaknath v. State of Punjab, Learning: The Supreme Court declared in
1967 AIR 1643, the Supreme Court ruled that D. C. Wadhwa v. State of Bihar, 1987 AIR 579,
the Parliament cannot take away or abridge that it is unconstitutional to re-promulgate
any of the Fundamental Rights, which are ordinances (in certain conditions).
‘sacrosanct’ in nature. The case of M. C. Mehta v. UOI, 1987 SCR
In the Minerva Mills Ltd. v. UOI, AIR (1) 819 originated in the aftermath of
1980 SC 1789, the Supreme Court also held oleum gas leak from Shriram Food and
that ‘the Indian Constitution is founded Fertilisers Ltd. complex at Delhi.
on the bedrock of the balance between The case of Rameshwar Prasad v. UOI,
the Fundamental Rights and the Directive 2006 was related to dissolution of Bihar
Principles Assembly
Q.1138 The famous case of Minerva Mills Ltd. v. Q.1140 The case of Maneka Gandhi v. UOI, 1978
UOI, AIR 1980 SC 1789 was related to AIR 597 is famously known for
1. Evolution of the basic structure doctrine (a) Wider interpretation of the Article 21
of the Constitution of India (b) Judicious rebalancing of fundamental
2. Status of Directive Principles of State rights and directive principles
Policy over the Fundamental Rights (c) Evolution of the ‘rarest of rare’ doctrine
Which of the above is/are correct? (d) Clarifying the provisions related to
(a)  1 only (b)  2 only President’s rule
(c)  Both 1 and 2 (d)  None Solution: (a)
Solution: (c) Learning: In the case of Maneka Gandhi v
Justification: Statement 1: As per the UOI, 1978 SCR (2) 621, the Supreme Court
judgment, the Parliament cannot, under overruled its judgement in the Gopalan case by
Article 368, expand its amending power so taking a wider interpretation of the Article 21.
as to acquire for itself the right to repeal or Therefore, it ruled that the right to life and
abrogate the Constitution or to destroy its personal liberty of a person can be deprived
basic and essential features. The donee of a by a law provided the procedure prescribed
limited power cannot be the exercise of that by that law is reasonable, fair and just.
power convert the limited power into an • In other words, it has introduced the
unlimited one. American expression ‘due process of
Statement 2: The 42nd Amendment had law’. In effect, the protection under
amended Article 31C of the Constitution to Article 21 should be available not only
accord precedence to the Directive Principles against arbitrary executive action but also
of State Policy articulated in Part IV of the against arbitrary legislative action.
Constitution over the Fundamental Rights of • Further, the court held that the ‘right to life’
individuals articulated in Part III. as embodied in Article 21 is not merely
The court held that part of the 42nd confined to animal existence or survival
Amendment to be unconstitutional and thus but it includes within its ambit the right
restored the primacy of fundamental rights to live with human dignity and all those
over the DPSP except in some situations, aspects of life which go to make a man’s
which we will cover in later tests. life meaningful, complete and worth living.
Q.1139 The presidential proclamation imposing • It also ruled that the expression ‘Personal
President’s Rule is subject to judicial review Liberty’ in Article 21 is of the widest
was declared by the Supreme Court in amplitude and it covers a variety of rights

P.316 For Civil Services Preliminary Examination

04-Indian Polity_Q1070-1506.indd 316 8/7/2018 7:42:38 PM


that go to constitute the personal liberties President but it can question the material
of a man. behind the satisfaction of the President
Q.1141 The case Subhash Kumar v. State. of Bihar, • If there is improper use of Article 356 then
(1991) 1 SCC 598 is famous because in this the court will provide remedy.
Supreme Court declared that • Under Article 356(3) it is the limitation
(a) Land grabbing cases can’t be adjudicated on the powers of the President. Hence, the
by District executive authorities. president shall not take any irreversible
(b) Right to Life is a Fundamental Right action until the proclamation is approved
under Article 21 of the Constitution by the Parliament i.e. he shall not dissolve
(c) Fundamental rights and Directive the assembly.
principles have equal status. • Article 356 is justified only when there is
(d) Defamation is a crime and will invite penal a breakdown of constitutional machinery
provisions if not within the reasonable and not administrative machinery
limits of the freedom of speech. Q.1143 The landmark judgment in ‘Coelho case’
Solution: (b) dealt with
Learning: The Supreme Court held that the (a) Inclusion of Ninth Schedule under the
Right to Life is a Fundamental Right under ambit of Judicial Review
Article 21 of the Constitution and it includes (b) Limitations of the power of pardon to
the right to the enjoyment of pollution-free President and Governors
water and air for full enjoyment of life. (c) Power of Parliament to amend the Preamble
The government is responsible for setting (d) Conflict between certain minority religious
up laws and procedures that can check practices and their fundamental rights
pollution, clean rivers and introduce heavy Solution: (a)
fines for those who pollute. Learning: The first amendment to the Indian
Many judgments added upon this historic Constitution added the Ninth Schedule to it.
judgment and gave rise to the expanded It was introduced to address judicial review
(in interpretation) Article 21 we know today. decisions especially about abridging certain
Q.1142 The case S. R. Bommai v UOI, 1994 AIR 1918 fundamental rights.
is well known for significantly impacting The state wanted to pursue nationalisation,
(a)  Centre-State relations take away lands from the zamindars,
(b)  Powers of Indian Parliament re-distribute them, and make special
(c)  Status of All-India Services provisions for the socially and economically
(d)  Jurisdiction of tribunals in India backward. This would have violated certain
Solution: (a) Fundamental Rights (FRs), thus the need
Learning: It was a landmark judgment of for 9th schedule to shield executive and
the Supreme Court of India, where the Court legislative actions from Judicial review.
discussed at length provisions of Article 356 The case of I. R. Coelho v. State of Tamil
of the Constitution of India and related issues. Nadu, 11 January 2007, raised the following
This case had huge impact on Centre- questions.
State Relations. • Is it permissible to make the Ninth
The SC laid down certain guidelines so as Schedule immunized from the Judicial
to prevent the misuse of Article 356 of the Review of the Supreme Court?
constitution. • Whether the Basic Structure test would
• The majority enjoyed by the Council of include Judicial Review of Ninth Schedule
Ministers shall be tested on the floor of laws on the touchstone of Fundamental
the House. rights?
• Centre should give a warning to the state • The court ruled that it is absolutely not
and a time period of one week to reply. permissible to make the Ninth Schedule
• The court cannot question the advice immunized from the Judicial Review of
tendered by the Council of Ministers to the the Constitution.

Indian Polity Question Bank P.317

04-Indian Polity_Q1070-1506.indd 317 8/7/2018 7:42:38 PM


The court said that the Doctrine of Explanation: Option (c) refers to the case of
Basic Structure is the very essence of the Sakshi v. UOI (2004); it is thus incorrect.
Constitution of India and therefore, there Option (d) refers to Tulasamma v. Reddi,
cannot be any act, rules or regulations which 1977, and thus it is also wrong.
can overrule the Basic Structure doctrine. Supreme Court has not issued any specific
Q.1144 The D.K. Basu Guidelines given by Supreme guidelines for prevention of domestic. A
Court are related to Parliamentary enactment of 2005 deals with
(a) Attachment of the assets of bankrupt the same. So, the option (b) is also wrong.
companies Learning: The Supreme Court in the
(b) Rehabilitation of children who are Vishakha Judgment in 1977 has held that each
victims of trafficking and sexual abuse incident of sexual harassment results in the
(c) Procedure of arrest, detention and violation of the fundamental rights of Gender
interrogation of any person Equality and the Right to Life and Liberty.
(d) Regulation of foreign NGOs working in Sexual harassment is a clear violation of
India woman’s right to gender equality as guaranteed
Solution: (c) under Articles 14 and 15, her right to live with
Learning: The Supreme Court of India dignity under Article 21 and her right to work
has laid down specific requirements and with dignity in a safe environment under
procedures that the police and other agencies Article 19 (1) (g) of the Constitution of India.
have to follow for the arrest, detention and Supreme Court thus makes it obligatory
interrogation of any person. These are known for every employer and other responsible
as the D.K. Basu Guidelines and they include persons to follow the guidelines laid down by
the following. the Court and to evolve a specific policy to
• The police officials who carry out the combat sexual harassment in the workplace.
arrest or interrogation should wear clear, The Supreme Court’s Vishakha Judgment in
accurate and visible identification and 1997 makes it mandatory for each Committee
name tags with their designations; to have a woman chairperson.
• A memo of arrest should be prepared at
the time of arrest and should include the Miscellaneous
time and date of arrest. It should also be Q.1146 Besides its permanent seat at Delhi, the
attested by at least one witness who could Supreme Court can also meet at
include a family member of the person (a)  any other metropolitan city
arrested. The arrest memo should be (b)  any other major city
counter-signed by the person arrested. (c) any other place as decided by the Chief
• The person arrested, detained or being Justice of India in consultation with the
interrogated has a right to inform a President
relative, friend or well-wisher. (d)  any other Union Territory
• When a friend or relative lives outside the Solution: (c)
district, the time, place of arrest and venue The Constitution declares Delhi as the seat of
of custody must be notified by police the Supreme Court. But, it also authorises the
within 8 to 12 hours after arrest. chief justice of India to appoint other place or
Q.1145 ‘Vishakha Guidelines’ refers to places as seat of the Supreme Court.
(a) Prevention of sexual harassment at He can take decision in this regard
workplace only with the approval of the President.
(b)  Prevention of domestic violence This provision is only optional and not
(c) Conducting trial of child sexual abuse or compulsory. This means that no court can
rape give any direction either
(d) Settlement of alimony and inheritance of To the President or To the Chief Justice
property between divorced couple To appoint any other place as a seat of the
Solution: (a) Supreme Court.

P.318 For Civil Services Preliminary Examination

04-Indian Polity_Q1070-1506.indd 318 8/7/2018 7:42:39 PM


Q.1147 Consider the following statements. In the recent judgements of the Supreme
1. The Parliament cannot enlarge the Court, Articles 32, 136, 141 and 142 have
jurisdiction of the Supreme Court of India come out to be a part of the basic structure.
as its jurisdiction is fixed to that conferred Article 136 refers to the provision of special
by the unamended Constitution. leave to appeals by the SC.
2. The Parliament can increase the number Article 141 refers to the laws declared by
of judges in the Supreme Court by an the SC to be binding on the courts in India.
amendment of the constitution with a Article 142 refers to the enforcement of
simple majority in the Parliament. decrees and orders of the SC etc
Which of the above is/are correct? Consider the following statements with
(a)  1 only (b)  2 only respect to the jurisdiction of the Supreme
(c)  Both 1 and 2 (d)  None Court:
Solution: (b) 1. The government wanted to know if it
Justification: Statement 1: Number of puisne can pass a law about the citizenship status
judges in the Supreme Court and Conferment of residents of Pakistan occupied areas
of more jurisdiction on the Supreme Court of Jammu and Kashmir, comes under
are matters that can be dealt by the Parliament original jurisdiction of the Supreme Court
with a simple majority. 2. In order to resolve the dispute about river
Statement 2: If the Parliament wishes to Cauvery the government of Tamil Nadu
curtail the SC’s or HC’s authority, it will have wants to approach the court, comes under
to amend the constitution by a special majority appellate jurisdiction of the Supreme
with the consent of majority of the states. Court
Q.1148 Consider the following statements: 3. Court rejected the appeal by people
1. Any enlargement in the powers of the against the eviction from the dam site,
Supreme Court would require a special comes under writ jurisdiction of the
agreement between the Centre and states Supreme Court.
i.e. the nod of the Parliament as well as Which of the above statements is/are correct?
that of a majority of the state legislatures. (a)  2 Only (b)  1 and 2 Only
2. The Supreme Court is the final interpreter (c)  1, 2 and 3 Only (d)  None
of the constitution and the Parliament Solution: (a)
cannot reverse these interpretation by Statement 1: It comes under advisory
a constitutional amendment or a legal jurisdiction
enactment. Statement 2: Supreme court admitted the
Which of these is/are true? appeal of Karnataka against Tribunal award.
(a)  only 1 (b)  Only 2 Q.1150 Which of the following come under the
(c) Both (d) None jurisdiction of both the Supreme Court and
Solution: (b) High courts?
Refer previous explanations. 1. Disputes between the Centre and the
Q.1149 Which, among others, are powers of the States
Supreme Court which fall under the Doctrine 2. Disputes between the States
of „Basic Structure‟? 3. Issuing writs to enforce fundamental
1. Ancillary powers of the Court. rights
2. The provision of Special Leave petition. 4. Judicial review of administrative acts
3. Power to declare laws binding on all Choose the correct answer using the codes
courts in India. below.
Choose the correct answer using the codes (a)  1 and 2 only (b)  1 and 3 only
below (c)  3 only (d)  3 and 4 only
(a)  2 and 3 (b)  1 and 3 Solution: (d)
(c)  Only 2 (d)  None of the above The Article 226 of the Constitution empowers
Solution: (a) a high court to issue writs including habeas

Indian Polity Question Bank P.319

04-Indian Polity_Q1070-1506.indd 319 8/7/2018 7:42:39 PM


corpus, mandamus, certiorari, prohibition (d) The 1773 Regulating Act created the
and quo warranto for the enforcement of Supreme Court as well as the High Courts.
the fundamental rights of the citizens and Solution: (c)
for any other purpose. Judicial review is Learning: The institution of high court
the power of a high court to examine the originated in India in 1862 when the high
constitutionality of legislative enactments courts were set up at Calcutta, Bombay and
and executive orders of both the Central and Madras.
state governments. On examination, if they • In 1866, a fourth High Court was
are found to be violative of the Constitution established at Allahabad. In the course of
(ultra vires), they can be declared as illegal, time, each province in British India came
unconstitutional and invalid (null and to have its own high court.
void)  by the high court. Consequently, they • After 1950, a high court existing in a
cannot be enforced by the government. province became the high court for the
corresponding state.
(35) HIGH COURT • The Constitution of India provides for a
high court for each state, but the Seventh
Q.1151 Shortage of judges in High courts has been a Amendment Act of 1956 authorised the
major constraint in timely delivery of justice Parliament to establish a common high
in India. To redress the situation, who among court for two or more states or for two or
the following authorities has the right to more states and a union territory.
determine the strength of a high court from Q.1153 Consider the following statements about
time to time? High Courts in India.
(a)  President of India Assertion (A): The State governments
(b)  Chief Justice of India have established a High Court in every State
(c)  Union Minister of Law and Justice and Union territory (UT) of India.
(d)  Chairman, Bar Council of India Reason (R): The Constitution of India
Solution: (a) empowers only the State governments to
Learning: Every high court (whether create High Courts.
exclusive or common) consists of a chief In the context of the above, which of these
justice and such other judges as the president is correct?
may from time to time deem necessary to (a) A is correct, and R is an appropriate
appoint. explanation of A.
Thus, the Constitution does not specify (b) A is correct, but R is not an appropriate
the strength of a high court and leaves it to the explanation of A.
discretion of the president who determines it (c)  A is correct, but R is incorrect.
based on its workload. (d)  Both A and R are incorrect.
This is provided for in the Constitution Solution: (d)
of India, which deals with the organisation, Constitution provides for high court in each
independence, jurisdiction, powers, procedures state. The high court are established by the
and so on of the high courts. Parliament. The territorial jurisdiction of a
Q.1152 The institution of high court originated in high court is co-terminus with the territory
India when of a state. Further The Parliament can extend
(a) Supreme Court took charge in Independent the jurisdiction of a high court to any union
India and established High courts in many territory or exclude the jurisdiction of a high
states court from any union territory
(b) The Morley-Minto Act provided for the Q.1154 Concerning the High Court, which of the
establishment of High courts following is done by the President?
(c) During the British rule when the high 1. Determining the Strength of High Court
courts were set up at Calcutta, Bombay 2. Appointing the Judges
and Madras 3. Removal of Judges

P.320 For Civil Services Preliminary Examination

04-Indian Polity_Q1070-1506.indd 320 8/7/2018 7:42:39 PM


Choose the correct answer using the codes Appointment & Transfer
below.
(a)  1 and 2 only (b)  2 and 3 only Q.1156 The judges of the High court are appointed
(c)  1 and 3 only (d)  All of the above by the President of India. Who does he
Solution: (d) consult among the following while making
Every high court (whether exclusive or the appointment?
common) consists of a chief justice and such 1. Chief Justice of India
other judges as the president may from time 2. Advocate general of the state concerned
to time deem necessary to appoint. Thus, the 3. Governor of the state concerned
Constitution does not specify the strength of 4. Supreme Court Judges
a high court and leaves it to the discretion Choose the correct answer using the codes
of the president. Accordingly, the President below:
determines the strength of a high court from (a)  1, 2 and 3 (b)  2, 3 and 4
time to time depending upon its workload. (c)  1, 3 and 4 (d)  1, 2 and 4
The judges of a high court are appointed by Solution: (c)
the President. The chief justice is appointed The judges of a high court are appointed by
by the President after consultation with the the President.
chief justice of India and the governor of the The chief justice is appointed by the
state concerned. President after consultation with the chief
A judge of a high court can be removed justice of India and the governor of the state
from his office by an order of the President. concerned.
The President can issue the removal order • For appointment of other judges, the chief
only after an address by the Parliament has justice of the concerned high court is also
been presented to him in the same session for consulted.
such removal. • In case of a common high court for two or
more states, the governors of all the states
Q.1155 In the Indian Judiciary, which of the following
concerned are consulted by the president.
factors may work as constraints over the High
In the Third Judges case (1998), the Supreme
Court judges in giving their rulings?
Court opined that in case of the appointment
1. Precedents set by Supreme Court
of high court judges, the chief justice of India
2. Constitution of India
should consult a collegium of two senior-
3. Instructions of the Executive to the courts
most judges of the Supreme Court. Thus, the
Choose the correct answer using the codes
sole opinion of the chief justice of India alone
below.
does not constitute the ‘consultation’ process.
(a)  1 and 2 only (b)  2 and 3 only
(c)  1 and 3 only (d)  All of the above Q.1157 A High Court consists of a Chief Justice and:
Solution: (a) (a)  at least five other judges
A high court cannot go against previous (b) such other judges as specified in the
rulings of the SC which set a precedent. Constitution
But the SC and HCs can go against their (c) such other judges as determined by
own respective precedents. For e.g. the Parliament
SC changed its stance on constitutional (d) such other judges as determined by the
amendment in Kesavananda Bharati case President
from what it was in the Golaknath case. Solution: (d)
But the same could not be done by the HC Every high court (whether exclusive or
against an SC ruling. common) consists of A chief justice and
The executive cannot issue any instruction Such other judges as the president may
to the courts in India. from time to time deem necessary to appoint.
Thus, the Constitution does not specify the
strength of a high court and leaves it to the
discretion of the president.

Indian Polity Question Bank P.321

04-Indian Polity_Q1070-1506.indd 321 8/7/2018 7:42:39 PM


Q.1158 Consider the following statements. (c) Governor
1. Transfer of judges of the high courts is the (d) President
sole prerogative of the Chief Justice of Solution: (c)
India (CJI). A person appointed as a judge of a high court,
2. The state government can take disciplinary before entering upon his office, has to make
action against erring judges of the High and subscribe an oath or affirmation before
Court in that respective state. the governor of the state or some person
3. The CM of the state is consulted before appointed by him for this purpose.
appointing the judges of the concerned In his oath, a judge of a high court swears:
High Court. 1. to bear true faith and allegiance to the
Choose the correct answer using the codes Constitution of India;
below. 2. to uphold the sovereignty and integrity of
(a)  1 and 2 only (b)  1 only India;
(c)  3 only (d)  None of the above 3. to duly and faithfully and to the best of
Solution: (d) his ability, knowledge and judgement
The Chief Justice of India must consult a perform the duties of the office without
collegium of judges. After this the President fear or favour, affection or ill-will; and
of India gives orders for the transfer of 4. to uphold the Constitution and the laws.
judges, even while the several High Courts
come under the administrative control of the Tenure
SC. The state governments have no control
Q.1161 Consider the following statements.
over the HC judges. Only the Governor is
1. The tenure of a high court judge has not
consulted before appointing the judges and
been fixed by the constitution.
CJI of the HCs.
2. The concerned state assembly can
Q.1159 Consider the following statements: impeach the judges of High court with a
Assertion (A): Transfer of judges of the special majority.
high courts is the sole prerogative of the Which of these is/are true?
Chief Justice of India (CJI). (a) Only 1 (b)  Only 2
Reason (R): The high courts come under (c) Both (d)  None of the above
the administrative control of the Supreme Solution: (a)
court headed by the CJI. The Constitution has not fixed the tenure of a
In the context of the above two statements, judge of a high court. However, it makes the
which one of these is true? following four provisions in this regard.
(a) Both A and R is true and R is the correct 1. He holds office until he attains the age
explanation of A. of 62 years. Any questions regarding his
(b) Both A and R is true but R is not a correct age is to be decided by the President after
explanation of A. consultation with the chief justice of India
(c) A is true, but R is false. and the decision of the president is final.
(d) R is true, but A is false. (Note: in case of SC judges it is decided
Solution: (d) by such authority and in such manner as
The Chief Justice of India must consult a provided by Parliament)
collegium of judges. After this the President 2. He can resign his office by writing to the
of India gives orders for the transfer of judges, president.
even while the high courts come under the 3. He can be removed from his office by
administrative control of the SC. the President on the recommendation
Q.1160 The oath to a High Court Judge is of the Parliament. Judges enquiry act
administered by the: (1968) regulates the procedure relating
(a)  Chief Justice of India to the removal of a judge of the Supreme
(b)  Chief Justice of that High Court Court/High Court by the process of
impeachment.

P.322 For Civil Services Preliminary Examination

04-Indian Polity_Q1070-1506.indd 322 8/7/2018 7:42:39 PM


4. He vacates his office when He is appointed an address is presented to the president for
as a judge of the Supreme Court or When removal of the judge.
he is transferred to another high court. Finally, the president passes an order
Q.1162 A Judge of a High Court wanting to resign removing the judge.
addresses his letter of resignation to: Q.1164 Consider the following statements.
(a)  the President Assertion (A): Motion for the impeachment
(b)  the Chief Justice of his High Court of a High Court Judge cannot be introduced
(c)  the Chief Justice of India in the Rajya Sabha.
(d)  the Governor of the State Reason (R): A High Court Judge is
Solution: (a) appointed by the President.
He holds office until he attains the age of In the context of the above, which of these
62 years. Further he can resign his office by is correct?
writing to the president. (a) A is correct, and R is an appropriate
explanation of A.
Removal (b) A is correct, but R is not an appropriate
explanation of A.
Q.1163 The Judges Enquiry Act (1968) regulates the
(c)  A is correct, but R is incorrect.
procedure relating to the removal of a judge of
(d)  A is incorrect, but R is correct.
a high court by the process of impeachment.
Solution: (d)
Consider the following with reference to it.
Justification: Impeachment motions can be
1. The procedure for the impeachment of a
moved in either house.
judge of a high court is the same as that
For e.g. in 2011 for the first time in the
for a judge of the Supreme Court.
history of Rajya Sabha and only the second
2. The Speaker or Chairman of the house
case in the history of Indian Parliament, a
may refuse to admit a motion for the
Motion for the removal of a Judge of High
impeachment of a high court judge.
Court was formally moved, discussed and
3. No judge of a high court has been
finally voted on 18 August 2011.
impeached so far.
Learning: It was Justice Soumitra Sen.
4. Only the Chief Justice of India can pass
He resigned ahead of the impeachment
the final order for the removal of a high
motion against him in the Lok Sabha.
court judge post-impeachment.
Later, Lok Sabha decided to drop the
Select the correct answer using the codes
impeachment proceedings against him
below.
because he had resigned.
(a)  1, 2 and 3 only (b)  3 and 4 only
(c)  1 and 2 only (d)  1 and 3 only
Solution: (a)
Independence of High Court
Justification: Statements 1 and 2: A removal Q.1165 Which of the following confirms the
motion signed by 100 members (in the case independence of the High Courts?
of Lok Sabha) or 50 members (in the case of 1. Expenses charged on the Consolidated
Rajya Sabha) is to be given to the Speaker/ fund of India.
Chairman. 2. Power to punish for contempt.
The Speaker/Chairman may admit the 3. Power to control subordinate courts.
motion or refuse to admit it. Which of these is/are true?
If it is admitted, then the Speaker/ (a) 1 and 2 (b) 2 and 3
Chairman is to constitute a three-member (c) 1 and 3 (d) Only 2
committee to investigate into the charges. Solution: (d)
The procedure is same for a SC judge. So, Expenses are charged on the consolidated
the statement 1 and 2 both are correct. fund of the concerned state and not that of
Statement 4: After the motion is passed by India. Pension is however charged on the
each House of Parliament by special majority, latter.

Indian Polity Question Bank P.323

04-Indian Polity_Q1070-1506.indd 323 8/7/2018 7:42:39 PM


The power to control subordinate courts Expenses are charged on Consolidated fund
is not a sign of independence. Independence of the State (not India).
here means the ability to give impartial The judges of a high court are provided
judgements without fear and favour. with the security of tenure. They can be
Q.1166 Consider the following statements. removed from office by the president only
1. A High Court judge is appointed by the in the manner and on the grounds mentioned
Governor of the State concerned who also in the Constitution. This means that they
administers oath to the judge. do not hold their office during the pleasure
2. A person who has acted as a High Court of the president, though they are appointed
Judge cannot plead before any authority by him. This is obvious from the fact that
in India after retirement. no judge of a high court has been removed
Which of the above is/are correct? (or impeached) so far. The salaries and
(a)  1 only (b)  2 only allowances of the judges, the salaries,
(c)  Both 1 and 2 (d)  None allowances and pensions of the staff as well
Solution: (d) as the administrative expenses of a high court
Justification: Statement 1: President appoints are charged on the consolidated fund of the
a High Court judge in consultation with the state. Thus, they are non-votable by the state
Governor and Chief Justice of the High Court. legislature (though they can be discussed by
Every person appointed to be a Judge it). It should be noted here that the pension
of a High Court shall, before he enters of a high court judge is charged on the
upon his office, make and subscribe before Consolidated Fund of India and not the state.
the Governor of the State, or some person The Constitution prohibits any discussion
appointed in that behalf by him an oath or in Parliament or in a state legislature with
affirmation according to the form set out for respect to the conduct of the judges of a high
the purpose in the Third Schedule. court in the discharge of their duties, except
Statement 2: As per the constitution, “No when an impeachment motion is under
person who, after commencement of this consideration of the Parliament.
Constitution, has held office as a permanent Q.1168 The Constitution places the High Courts
judge of High Court shall plead or act in under the control of the Union in certain
any court or before any authority except the matters in order to keep them outside
Supreme Court and the other High Courts.” the range of regional politics. The Union
So, the statement 2 is incorrect as he can exercises its control in the matters of:
plead before other courts except the one I. transfer of Judges from one High Court
where he held charge. to another.
Q.1167 The Constitution has made which of the II. being able, to establish a common High
following provisions to safeguard and ensure Court for two or more States.
the independent and impartial functioning of III. determining disputes as to age of High
a high court? Court Judges.
1. Conduct of Judges cannot be discussed in (a)  I only (b)  II and III
state legislature (c)  I and II (d)  I, II and III
2. Their expenses are charged on Solution: (d)
Consolidated Fund of India President is empowered to transfer the judges
3. Security of Tenure of high court and is the authority to decide the
Choose the correct answer using the codes disputes relating to ages of high court judges.
below. President exercises this power in consultation
(a)  1 and 2 only (b)  2 and 3 only with central council of ministers. In this
(c)  1 and 3 only (d)  All of the above way central government can exercise some
Solution: (c) influence on him.
The Constitution of India provides for a
high court for each state, But the Seventh

P.324 For Civil Services Preliminary Examination

04-Indian Polity_Q1070-1506.indd 324 8/7/2018 7:42:39 PM


Amendment Act of 1956 authorised the The territorial jurisdiction of a high court is
Parliament to Establish a common high court co-terminus with the territory of a state.
For two or more states or For two or more Constitution empowers the Parliament
states and a union territory. and the state legislature to change the
jurisdiction and powers of a high court
Jurisdiction and Power of High Court except jurisdiction and powers of a high
court in so far as they are specified in the
Q.1169 A high court does NOT enjoy which of the
Constitution.
following jurisdiction and powers?
1. Advisory Jurisdiction
2. Writ jurisdiction
(1) Original Jurisdiction
3. Supervisory Jurisdiction Q.1172 Disputes relating to the election of members
4. Original Jurisdiction of Parliament and state legislatures fall under
Choose the correct answer using the codes which of the respective jurisdictions of the
below. High Courts?
(a)  1 only (b)  2 and 4 only (a)  Original jurisdiction
(c)  1 and 3 only (d)  1 and 4 only (b)  Exclusive jurisdiction
Solution: (a) (c)  Writ jurisdiction
Q.1170 Apart from Constitutional provisions, the (d)  Appellate jurisdiction
present jurisdiction and powers of a high Solution: (a)
court are governed by Learning: Original jurisdiction means the
1. Acts of Parliament power of a high court to hear disputes in the
2. Acts of State Legislature first instance, not by way of appeal. It extends
3. Civil Procedure Code to the following:
Select the correct answer using the codes • Matters of Admiralty, will, marriage,
below. divorce, company laws and contempt of
(a)  1 only (b)  1 and 2 only court.
(c)  2 and 3 only (d)  1, 2 and 3 only • Disputes relating to the election of
Solution: (d) members of Parliament and state
Justification: The Constitution of India has legislatures.
not made any clear and detailed description of • Regarding revenue matter or an act
the powers and functions of the High Court as ordered or done in revenue collection.
it has done in the case of the Supreme Court. • Enforcement of fundamental rights of
The Constitutions says that the Jurisdiction citizens.
of the High Court shall be the same as • Cases ordered to be transferred from
immediately before the commencement of a subordinate court involving the
the Constitution. The gap is filled by laws interpretation of the Constitution to its
made by Central as well as state legislatures, own file.
of which Civil and Criminal Procedure Code • The four high courts (i.e., Calcutta,
is also a part. Bombay, Madras and Delhi High Courts)
have original civil jurisdiction in cases of
Q.1171 Jurisdiction of a High Court is decided on the
higher value.
basis of:
(a)  territorial area of state Q.1173 Which of the following would come under
(b)  funds at the disposal the original jurisdiction of the High court?
(c)  intention of the government 1. Disputes relating to the elections of MPs
(d)  number of judges available and MLAs.
Solution: (a) 2. Enforcement of fundamental rights of
citizens.
3. Urgent constitutional matters.

Indian Polity Question Bank P.325

04-Indian Polity_Q1070-1506.indd 325 8/7/2018 7:42:39 PM


Which of these is/are true? Justification: Statement 1: The Supreme
(a) 1 and 2 (b) 2 and 3 Court can issue writs only for the enforcement
(c)  1 and 3 (d)  All of the above of fundamental rights and not for any other
Solution: (a) purpose, that is, it does not extend to a case
Refer previous explanations. where the breach of an ordinary legal right
Q.1174 High court has original jurisdiction with is alleged.
respect to which of these matters? Statement 2: The writ jurisdiction of the
1. Dispute between two adjoining states high court (under Article 226) is not exclusive
2. Selective cases involving Constitutional but concurrent with the writ jurisdiction
Interpretation of the Supreme Court (under Article 32).
3. Disputes relating to the election of MPs It means, when the fundamental rights of a
and MLAs citizen are violated, the aggrieved party has
4. Enforcement of fundamental rights of the option of moving either the high court or
citizens the Supreme Court directly.
Choose the correct answer using the codes This is valid for both centre/state laws/
below. rules/regulations. So, the statement 2 is
(a)  1 and 2 only (b)  2, 3 and 4 only incorrect.
(c)  3 and 4 only (d)  All of the above Learning: In the Chandra Kumar case
Solution: (b) (1997), the Supreme Court ruled that the
Refer previous explanations. writ jurisdiction of both the high court and
the Supreme Court constitute a part of the
(2) Writ Jurisdiction basic structure of the Constitution. Hence, it
cannot be ousted or excluded even by way of
Q.1175 Which of the following can be challenge in an amendment to the Constitution.
a high court?
1. Executive orders (3) Supervisory Jurisdiction
2. Ordinances issued by the Governor of the
concerned state Q.1177 Which of the following is not a power of the
3. Central legislations High Court?
Which of these is/are true? (a) Supervision over all courts under its
(a) 1 and 2 (b) 2 and 3 jurisdiction
(c)  1 and 3 (d)  All of the above (b)  Jurisdiction over revenue matters
Solution: (d) (c)  Supervision over tribunals constituted by
The High courts have this power under law relating to the armed forces
Article 226 of the constitution. (d)  Issue writs for enforcement of
fundamental rights or for any other
Q.1176 It is said that the writ jurisdiction of the high
purpose
court is wider than that of the Supreme Court.
Solution: (c)
This is because
A high court has the power of superintendence
1. The Supreme Court can issue writs only
over all courts and tribunals functioning in its
for the enforcement of fundamental rights
territorial jurisdiction except military courts
and not legal rights which the high court
or tribunals.
can do
2. The writ jurisdiction of the high court
is exclusive with respect to the matters
(4) Control over Subordinate Courts
under state specific laws. Q.1178 Which of the following shows that the High
Which of the above is/are correct? court has the power to control subordinate
(a)  1 only (b)  2 only courts?
(c)  Both 1 and 2 (d)  None 1. It is consulted by the Governor in the
Solution: (a) personnel matters of the subordinate
courts.

P.326 For Civil Services Preliminary Examination

04-Indian Polity_Q1070-1506.indd 326 8/7/2018 7:42:39 PM


2. Its law is binding on the subordinate Choose the correct answer using the codes
courts. below.
3. It can withdraw any pending case from the (a)  1 and 2 (b)  2 and 3 only
subordinate courts. (c)  1 and 3 only (d)  All of the above
Which of these is/are true? Solution: (d)
(a) 1 and 2 (b) 2 and 3 Judicial review is the power of a high court
(c)  1 and 3 (d)  All of the above to examine the constitutionality of legislative
Solution: (a) enactments and executive orders of both
Not all cases can be withdrawn, but only the Central and state governments. On
those where there is a substantial question examination, if they are found to be violative
of law involved; or the interpretation of of the Constitution (ultra vires), they can
the constitution is needed; or the court is be declared as illegal, unconstitutional and
exceeding its jurisdiction; or the court has invalid (null and void) by the high court.
given a mala fide judgement. Consequently, they cannot be enforced by
the government.
(5) Court of Record Though the phrase ‘judicial review‘ has
no where been used in the Constitution, the
Q.1179 The High courts are also called as courts of
provisions of Articles 13 and 226 explicitly
record. It means that
confer the power of judicial review on a
1. The judgements, proceedings and acts of
high court. The constitutional validity of a
the high courts are recorded for testimony.
legislative enactment or an executive order
2. The Judgements of High Courts are
can be challenged in a high court on the
recognized as legal precedents.
following three grounds:
3. The acts of the High Courts cannot
• it infringes the fundamental rights (Part III),
be questioned if produced before any
• it is outside the competence of the
subordinate court in India.
authority which has framed it, and
Choose the correct answer using the codes
• it is repugnant to the constitutional
below.
provisions.
(a)  1 and 2 only (b)  2 and 3 only
(c)  1 and 3 only (d)  All of the above
Solution: (d)
Miscellaneous
As per Article 129 and 215 of the Constitution, Q.1181 When the Chief Justice of a High Court acts
the SC and High Courts will be courts of in an administrative capacity, he is subject to
record. (a) The writ jurisdiction of any of the other
It means that their judgement is basically judges of the High Court
a legal precedent before other courts. The (b) Special control exercised by the Chief
subordinate courts can cite and use these Justice of India
precedents in their judgements. (c) Discretionary powers of the Governor of
Moreover, as a court of record, a HC also the state
has power to punish for contempt. (d) Special powers provided to the Chief
Minister in this regard
(6) Judicial Review Solution: (b)
On the administrative side, the Chief Justice
Q.1180 Judicial review is one the powers of a high
of the High Court carries out the following
court. Under Judicial review constitutionality
functions:
of which of the following can be examined
1. Maintenance of the roster;
by the HC?
2. Appointment of court officials;
1. Legislative enactments of Parliament
3. General and miscellaneous matters
2. Executive orders of State governments
relating to the supervision and functioning
3. Executive orders of Central government
of the High Court and subordinate courts.

Indian Polity Question Bank P.327

04-Indian Polity_Q1070-1506.indd 327 8/7/2018 7:42:39 PM


It is logical that he would not be in the control Both can review central as well as state
of other high court judges and the executive. laws. Even here the view of the SC will
The control would lie with the CJI. prevail over HCs.
Q.1182 Consider the following statements. Q.1184 Which of the following roles/functions are
1. The Advocate General of a State in India performed by the High Court in India?
is appointed by the President of India on 1. Determining the Constitutional validity of
the recommendation of the Governor of any legislation or action of the executive
the concerned State. in the country
2. High Courts have not been granted any 2. Giving judgments and directives to
power under the constitution and derive protect public interest and human rights
all their authority from Civil and Criminal throughout India
Procedure Codes. Which of the above is/are correct?
Which of the above is/are correct? (a)  1 only (b)  2 only
(a)  1 only (b)  2 only (c)  Both 1 and 2 (d)  None
(c)  Both 1 and 2 (d)  None Solution: (c)
Solution: (d) Explanation & Learning: The Supreme
Justification: Statement 1: The Office of the Court and the High Courts have the power
Advocate General is a constitutional office to interpret the Constitution of the country.
created under Article 165 of the Constitution They can declare invalid any law of the
of India. The Governor of the State appoints legislature or the actions of the executive,
a person who is qualified to be appointed as a whether at the Union level or at the state
judge of the High Court as Advocate General level, if they find such a law or action is
of the State. against the Constitution. This is known as the
Statement 2: The High court derive judicial review. So, statement 1 is correct.
their writ jurisdiction, judicial review The powers and the independence of the
powers, original jurisdiction etc. from the Indian judiciary allow it to act as the guardian
constitution of the Fundamental Rights. In recent years
Q.1183 Which of the following is/are the same for the Courts have given several judgments
both the Supreme Court and High Courts? and directives to protect public interest and
1. Appointment procedure of Judges human rights. Anyone can approach the
2. Extent of writ issuing authority courts if public interest is hurt by the actions
3. Power to interpret the constitution of government. This is called public interest
4. Power of Judicial review of Central laws litigation. The courts intervene to prevent the
Choose the correct answer using the codes misuse of the government’s power to make
below. decisions. They check malpractices on the
(a)  2 and 3 only (b)  1, 3 and 4 only part of public officials. So, statement 2 is also
(c)  1, 2 and 4 only (d)  3 and 4 only correct.
Solution: (d) Q.1185 Match the following.
Unlike the SC, in the appointment to HC Union Territory Jurisdiction (High Court)
judges, the Governor of the state is also A. Puducherry 1. Kerala
consulted. B. Andaman and 2. Mumbai
High Courts can issue writs for legal rights    Nicobar Islands
as well as fundamental rights. The SC can C. Lakshadweep 3. Chennai
issue writs only for enforcing fundamental D. Daman and Diu 4. Kolkata
rights. 5 Guwahati
Both can interpret the constitution, but ABCD
the version of the SC will prevail over (a)  3 4 1 2 (b)  1 3 4 2
the HCs. (c)  1 5 3 4 (d)  1 5 3 2
Solution: (a)

P.328 For Civil Services Preliminary Examination

04-Indian Polity_Q1070-1506.indd 328 8/7/2018 7:42:39 PM


(36) SUBORDINATE COURT – 1. District Collector
2. State Governor
PART VI – ARTICLES 233 TO 237 3. State Law Minister
Q.1186 The Constitution makes provisions to Choose the correct answer using the codes
1. Regulate the organization of subordinate below.
courts (a)  1 only (b)  2 and 3 only
2. Ensure the independence of subordinate (c)  3 only (d)  None of the above
courts from the executive Solution: (d)
Which of the above is/are correct? The District courts are subordinate courts
(a)  1 only (b)  2 only that work under the authority of the High
(c)  Both 1 and 2 (d)  None court and Supreme Court. Immediate control
Solution: (c) is exerted by the HC, and final control by
Justification: The subordinate courts are a the SC. The High Courts are responsible for
part of state Judiciary functioning under the appointment, postings, transfers and general
State High Court in district and lower levels. administration of the district courts. The
Articles 233 to 237 in Part VI of the district courts are free from interference by
Constitution make provisions to regulate the state or central executive.
the organization of subordinate courts Q.1189 Consider the following statements:
and to ensure their independence from the 1. The highest criminal court of the district is
executive, such as appointment of District the Court of District and Sessions Judge.
Judges, control over subordinate courts etc. 2. The District Judges are appointed by the
Q.1187 Consider the following statements. Governor in consultation with the High
1. The Constitution of India does not Courts
recognize subordinate courts. 3. A person to be eligible for appointment
2. The Independence of Subordinate courts as a District Judge should be an advocate
from the executive is secured via State or a pleader of seven years’ standing or
laws. more, or an officer in judicial service of
3. Only the High Court, not the District the Union or the State.
court, can exercise superintendence over 4. When the Session judge awards death
the lower courts. sentence, it must be confirmed by the
Choose the correct answer using the codes High Court before it is carried out.
below. Which of the statements given above are
(a)  1 and 2 only (b)  2 and 3 only correct? [IAS 2004]
(c)  1 only (d)  None of the above (a)  1 and 2 (b)  2, 3 and 4
Solution: (d) (c)  3 and 4 (d)  1, 2, 3 and 4
The state judiciary consists of a high court and Solution: (d)
a hierarchy of subordinate courts, also known
as lower courts. The subordinate courts are Appointment
so called because of their subordination to Q.1190 The appointment, posting and promotion of
the state high court. They function below district judges in a state are made by the
and under the high court at district and lower (a) Governor in consultation with the High
levels. Court
Articles 233 to 237 in Part VI of the (b) Chief Justice of the concerned High Court
Constitution make the following provisions (c) Governor in consultation with the High
to regulate the organization of subordinate Court and the State Public Service
courts and to ensure their independence from Commission
the executive. (d) Chief Justice of the concerned High
Q.1188 The District and sessions Judge in a state Court in consultation with a collegium of
works directly under the control of which of senior-most judges
the following authorities? Solution: (a)

Indian Polity Question Bank P.329

04-Indian Polity_Q1070-1506.indd 329 8/7/2018 7:42:39 PM


The appointment, posting and promotion Learning: Appeals against his orders and
of district judges in a state are made by the judgements lie to the High Court. He is the
governor of the state in consultation with the highest judicial authority in the district.
high court. Statement 1: The district judge is also the
A person to be appointed as district judge Sessions Judge. When he deals with civil
should have the following qualifications: cases, he is known as the district judge and
• He should not already be in the service of when he hears the criminal cases, he is called
the Central or the state government. as the Sessions judge.
• He should have been an advocate or a Statement 2: The district judge exercises
pleader for seven years. both judicial and administrative powers over
• He should be recommended by the high subordinate courts.
court for appointment. Statement 3: However, a capital punishment
Appointment of persons (other than district passed by him is subject to confirmation by the
judges) to the judicial service of a state High Court, whether there is an appeal or not
are made by the governor of the state after Q.1193 The highest criminal court of the district is
consultation with the State Public Service the Court of District and Sessions Judge.
Commission and the high court. Consider the following about it.
Q.1191 Who among the following is/are involved in 1. There is no mention of District Courts in
the appointment of district judges? the Constitution.
1. Governor of the concerned state. 2. The District Judges are appointed by the
2. The High court concerned. Governor in consultation with the High
3. The Supreme Court. Court.
Choose the correct answer using the codes 3. A person to be eligible for appointment
below: as a District Judge must be an officer in
(a) 1 and 2 (b) 2 and 3 judicial service of the Union or the State.
(c)  1 and 3 (d)  All of the above 4. A death sentence awarded by the Sessions
Solution: (a) court needs the approval of the High Court
The Governor must consult the high court as before it is executed.
a whole before making an appointment. The Select the correct answer using the codes
opinion of the Supreme Court is not required below.
in the appointment of district judges. (a)  1 and 3 only (b)  2 and 4 only
Q.1192 Consider the following about a District (c)  1, 2 and 3 only (d)  1, 2 and 4 only
Judge. Solution: (b)
1. He possesses original and appellate Justification: Statement 1: As per the
jurisdiction in both civil as well as constitution (Article 223), appointments of
criminal matters. persons to be, and the posting and promotion
2. He has supervisory powers over all the of, district judges in any State shall be made
subordinate courts in the district. by the Governor of the State in consultation
3. The Sessions Judge has the power to with the High Court exercising jurisdiction in
impose any sentence including life relation to such State.
imprisonment and capital punishment. So, the statements 1 and 3 are incorrect
Select the correct answer using the codes and the statement 2 is correct.
below. Statement 4: When the Sessions Judge
(a)  1 and 2 only (b)  2 and 3 only awards death sentence, it must be confirmed
(c)  1 and 3 only (d)  1, 2 and 3 by the High Court before it is carried out
Solution: (d)

P.330 For Civil Services Preliminary Examination

04-Indian Polity_Q1070-1506.indd 330 8/7/2018 7:42:39 PM


(37) OTHER DISPUTE RESOLUTION (a) They are informal courts setup by the
panchayat community.
MECHANISMS (b) Cases at pre-litigation stage or pending
National Legal Services Authority in a court of law can be settled by the
Lok Adalat.
Q.1194 Consider the following about National Legal (c) The award made by the Lok Adalat is
Services Authority (NALSA). deemed to be the decree of a civil court.
1. It is a statutory body. (d) All of the above
2. It provides for free legal aid to the poor Solution: (a)
and weaker sections of the society. Justification and Learning: It is a forum
3. It appoints judges of Gram Nyayalayas in where the disputes/cases pending in the court
consultation with the District Judge. of law or at pre-litigation stage are settled/
4. It also organises Lok Adalats for amicable compromised amicably. So, (b) is correct.
settlement of disputes. The Lok Adalats have been given statutory
Select the correct answer using the codes status under the Legal Services Authorities
below. Act, 1987. So, (a) is incorrect.
(a)  1, 2 and 4 only (b)  2 and 4 only Under the said Act, the award made by
(c)  1 and 3 only (d)  1, 2, 3 and 4 the Lok Adalats is deemed to be the decree
Solution: (a) of a civil court and is final and binding on
Justification: Statement 1: It has been all parties and no appeal lies before any court
constituted under the Legal Services against its award.
Authorities Act, 1987 Nature of cases to be referred to lok adalat
Statement 2: Its aim is to ensure that 1. Any case pending before any court
opportunities for securing justice are not 2. Any dispute which has not been brought
denied to any citizen by reasons of economic before any court and is likely to be filed
or other disabilities. NALSA also identifies before the court.
specific categories of marginalised and
Q.1196 Lok Adalats are a useful to instrument to
excluded groups and formulates various
reduce the burden on Judiciary in India.
schemes for implementation of legal service
Which of the following types of cases can be
programmes.
handled by Lok Adalat?
It provides services of free legal aid in
1. Consumer grievance against food
civil and criminal matters for the poor and
standards in rural processed products
marginalised people who cannot afford the
2. Cases involving property disputes
services of a lawyer in any court or tribunal.
3. Cases in involving Pension and other
Statement 3: The Gram Nyayalayas are
transfer payment rights
presided over by a Nyayadhikari, who has the
4. Cases involving road accidents
same power; enjoys same salary and benefits
Choose the correct answer using the codes
of a Judicial Magistrate of First Class. Such
below.
Nyayadhikari are to be appointed by the
(a) 1 and 2 only (b) 2 and 3 only
State Government in consultation with the
(c) 1 and 4 only (d) All of the above
respective High Court.
Solution: (d)
Statement 4: It also organises Lok Adalats
Matters may be civil or criminal in nature,
for amicable settlement of disputes. It works
but any matter relating to an offence not
in close coordination with various State Legal
compoundable under any law cannot be
Services Authorities, District Legal Services
decided by the Lok Adalat even if the parties
Authorities and other agencies.
involved therein agree to settle the same.
One important condition is that both
Lok Adalats parties in dispute should agree for settlement
Q.1195 Which of the following statements is through Lok Adalat and abide by its decision
INCORRECT about Lok Adalats?

Indian Polity Question Bank P.331

04-Indian Polity_Q1070-1506.indd 331 8/7/2018 7:42:39 PM


Gram Nyayalaya the respective High Court. The Court can
function as a mobile court at any place
Q.1197 Consider the following statements with within the jurisdiction of such Gram
reference to the Gram Nyayalayas: Nyayalaya, after giving wide publicity to
1. The Gram Nyayalayas are presided that regards.
over by a Nyayadhikari, who will have 4. The Gram Nyayalayas have both civil
the same power, enjoy same salary and and criminal jurisdiction over the
benefits of a Judicial Magistrate of First offences. The pecuniary jurisdiction of
Class. the Nyayalayas are fixed by the respective
2. A Gram Nyayalaya have jurisdiction over High Courts.
an area specified by a notification by the 5. Both the Central and the State Government
State Government in consultation with the can add or remove items in the Schedule.
respective High Court. While the Central Government can
3. The Gram Nyayalayas have only civil amend the list in Schedules I and II, by
jurisdiction over the offences. notifying them and thereafter laying it in
Which of the above statements are true? the Parliament, the State Government can
(a)  Only 1 & 2 (b)  Only 2 & 3. amend the items in Part III of Schedules
(c)  Only 2. (d)  All. I or II, in the areas of law which the
Solution: (a) state is competent to enact law after due
Explanation: consultation with the respective High
Gram Nyayalayas: Court and notifying it. Such notification
Gram Nyayalayas or village courts are has to be laid in the State Legislature.
established under the Gram Nyayalayas 6. Gram Nyayalayas can follow special
Act, 2008 for speedy and easy access to procedures in civil matters, in a manner it
justice system in the rural areas of India. The deem just and reasonable in the interest of
Act came into force from 2 October 2009. justice.
However, the Act has not been enforced 7. Gram Nyayalayas allow for conciliation
properly, with only 194 functional Gram of the dispute and settlement of the same
Nyayalayas in the country against a target of in the first instance.
5000 such courts. The major reason behind 8. Gram Nyayalayas has been given power
the non-enforcement includes financial to accept certain evidences which would
constraints, reluctance of lawyers, police and otherwise not be acceptable under Indian
other government officials. Evidence Act.
Important features of Gram Nyayalayas: 9. Appeals in criminal matter can be made
1. Gram Nyayalayas are established to the Sessions Court in the respective
generally at headquarter of every jurisdiction and in civil matters to the
Panchayat at intermediate level or a group District Court within a period of one
of contiguous panchayat in a district where month from the date of judgment.
there is no panchayat at intermediate
Q.1198 Consider the following about Gram
level.
Nyayalayas as established under Gram
2. The Gram Nyayalayas are presided over
Nyayalayas Act, 2008.
by a Nyayadhikari, who will have the same
1. They can act as mobile courts.
power, enjoy same salary and benefits of
2. They can exercise the powers of both
a Judicial Magistrate of First Class. Such
Criminal and Civil Courts.
Nyayadhikari are to be appointed by the
3. The presiding officer of the Gram
State Government in consultation with the
Nyayalayas is appointed by the concerned
respective High Court.
Panchayat.
3. A Gram Nyayalaya have jurisdiction over
4. No appeals lie to any court on a decree of
an area specified by a notification by the
Gram Nyayalaya.
State Government in consultation with

P.332 For Civil Services Preliminary Examination

04-Indian Polity_Q1070-1506.indd 332 8/7/2018 7:42:39 PM


5. They have been given power to accept it was in existence even under the previous
certain evidences which would otherwise Arbitration Act, 1940. The Arbitration and
not be acceptable under Indian Evidence Conciliation Act, 1996 was enacted and then
Act. the ordinance in 2015. So, 1 is wrong.
Select the correct answer using the codes Statement 2: Decision reached through
below. arbitration can be binding and need not
(a)  2 and 3 only be approved by the courts. However, the
(b)  1, 2 and 5 only decisions can be challenged in some cases in
(c)  3, 4 and 5 only the courts.
(d)  1, 2, 3 and 4 only Q.1200 Consider the following about mediation – an
Solution: (b) alternative dispute resolution mechanism.
Justification: Statements 1 and 2: Gram 1. Consent of all parties is required to start
Nyayalaya is a mobile court and exercises mediation.
the powers of both Criminal and Civil 2. Decision arrived after mediation can be
Courts; i.e., the seat of the Gram Nyayalaya enforced in a court of law.
will be located at the headquarters of the 3. Only a court of law can appoint the
intermediate Panchayat, but they will go to mediator.
villages, work there and dispose of the cases. Choose the correct answer using the codes
It can try criminal cases, civil suits, below.
claims or disputes which are specified in the (a)  1 and 2 only (b)  2 and 3 only
First Schedule and the Second Schedule to (c)  1 and 3 only (d)  All of the above
the Gram Nyayalaya Act and the scope of Solution: (a)
these cases can be amended by the Central The term “mediation” broadly refers to
as well as the State Governments, as per their any instance in which a third party helps
respective legislative competence; others reach agreement. More specifically,
Statements 3 & 4: refer last question mediation has a structure, timetable and
Statement 5: The Gram Nyayalaya will not dynamics that “ordinary” negotiation lacks.
be bound by the rules of evidence provided in The process is private and confidential,
the Indian Evidence Act, 1872 but shall be possibly enforced by law. Participation is
guided by the principles of natural justice and typically voluntary. The mediator acts as a
subject to any rule made by the High Court; neutral third party and facilitates rather than
directs the process. These are some of the
Alternative Dispute Resolution (ADR) rules laid down by Delhi HC; Appointment
Q.1199 Consider the following statements about of mediator/conciliator
Alternative dispute resolution (ADR) in (a) Parties to a suit or other proceeding may
India. agree on the name of the sole mediator/
1. Alternative Dispute Resolution (ADR) is a conciliator for mediating between them.
new concept in India and its practice started (b) Where, there are two or more sets of
only after passing of the Arbitration and parties and are unable to agree on a sole
Conciliation (Amendment) Ordinance, mediator/conciliator, the Court may ask
2015. each party to nominate the mediator/
2. Decisions reached through ADR cannot conciliator or may nominate/appoint the
be binding on the parties unless approved mediator/conciliator, as it deems fit.
by the courts. Q.1201 Consider the following statements about
Which of the above is/are correct? Alternative dispute resolution (ADR) in India.
(a)  1 only (b)  2 only 1. Alternative Dispute Resolution (ADR) is a
(c)  Both 1 and 2 (d)  None new concept in India and its practice started
Solution: (d) only after passing of the Arbitration and
Justification: Statement 1: Alternative Conciliation (Amendment) Ordinance,
dispute resolution in India is not new and 2015.

Indian Polity Question Bank P.333

04-Indian Polity_Q1070-1506.indd 333 8/7/2018 7:42:39 PM


2. Decisions reached through ADR cannot Nyaya Panchayat
be binding on the parties unless approved
by the courts. Q.1203 Nyaya Panchayats have been setup in some
Which of the above is/are correct? of the states in India. Their jurisdiction
(a)  1 only (b)  2 only extends to
(c)  Both 1 and 2 (d)  None (a)  Petty civil and criminal cases both
Solution: (d) (b)  Petty civil cases only
Justification: Statement 1: Alternative (c)  All local civil cases
dispute resolution in India is not new and (d)  Petty criminal cases only
it was in existence even under the previous Solution: (a)
Arbitration Act, 1940. The Arbitration and Nyaya Panchayats have been constituted in
Conciliation Act, 1996 was enacted and then some states. They possess the authority to
the ordinance in 2015. So, 1 is wrong. hear some petty, civil and criminal cases.
Statement 2: Decision reached through They can impose fines but cannot award
arbitration can be binding and need not a sentence. These village courts have
be approved by the courts. However, the often been successful in bringing about an
decisions can be challenged in some cases in agreement amongst contending parties. They
the courts. have been particularly effective in punishing
men who harass women for dowry and
Panchayat Adalats perpetrate violence against them.
Q.1202 Consider the following about Panchayati
adalats.
(38) JUDICIAL REVIEW AND ACTIVISM
1. They can be established by state
governments.
Judicial Review
2. They can be authorized to try offences Q.1204 The power of judicial review ensures the
listed under Indian Penal Code. following.
3. They can be endowed with judicial (a)  The supremacy of the Supreme Court
functions based on broad principles of (b)  The Supreme Court can review its own
natural justice. judgements
Select the correct answer using the codes (c)  The constitutionality of laws
below. (d)  The justice by the subordinate courts
(a)  1 and 2 only (b)  3 only Solution: (c)
(c)  1 and 3 only (d)  1, 2 and 3 Self-explanatory
Solution: (d) Q.1205 Which of these constitutional provisions give
Justification: Statement 1: the state the Judiciary the power of Judicial Review?
government establishes such adalats. 1. Article 13 which says laws ultra vires the
Statement 2 and 3: A Nyaya Panchayat constitution shall be void
is a system of dispute resolution at village 2. Article 32 which gives Supreme Court the
level in India. The nyaya panchayats can power to issue writs
be endowed with functions based on broad 3. Article 226 which gives High Courts the
principles of natural justice and can tend to power to issue writs
remain procedurally as simple as possible. Select the correct answer using the codes
They can be given civil and minor criminal below.
jurisdiction. (a)  1 and 2 only (b)  2 and 3 only
But they should never follow civil and (c)  1 only (d)  1, 2 and 3
criminal procedure code in letter and spirit. Solution: (d)
For e.g. the offences triable by the Delhi Justification: Statement 1: Article 13 declares
Panchayati Adalat includes some petty that all laws that are inconsistent with or in
offences under the Indian Penal Code, Cattle derogation of any of the fundamental rights
Trespass Act and Delhi gambling Act.

P.334 For Civil Services Preliminary Examination

04-Indian Polity_Q1070-1506.indd 334 8/7/2018 7:42:39 PM


shall be void. In other words, it expressively Statement 2: It is the power of judicial
provides for the doctrine of judicial review. review.
Statement 2 and 3: This power has been Under the powers of judicial review, the
conferred on the Supreme Court (Article 32) judiciary can strike down laws, administrative
and the High Courts (Article 226) that can acts and even constitutional amendments
declare a law unconstitutional and invalid if they violate the constitution or the basic
on the ground of contravention of any of the premises of the constitution.
Fundamental Rights. Q.1208 Consider the following statements:
Q.1206 Consider the following statements about 1. A citizen can move the Supreme Court if
Judicial review. s/he finds a law unfair or unconstitutional.
1. By way of a constitutional amendment, 2. The Supreme Court and High courts can
the Parliament can turn down a judicial scrap a law completely if it does not
ruling in so far as it conforms to the basic Conform to the constitutional framework.
structure of the constitution. Which of these is/are true?
2. The judiciary cannot only review (a)  1 only (b)  2 only
administrative acts but also the decisions (c)  Both 1 and 2 (d)  None
taken by some of the regulatory authorities Solution: (c)
in India. Judicial review is adopted in the Constitution
Which of these is/are true? of India from judicial review in the United
(a)  Only 1 (b)  Only 2 States. In the Indian constitution, Judicial
(c)  Both (d)  None of the above review is dealt with under Article 13. Judicial
Solution: (c) Review refers that the Constitution is the
The judiciary in India is immensely powerful. supreme power of the nation and all laws are
Barring a few cases, it has the right to review under its supremacy. Article 13 states that:
most acts/rules and regulations. Decisions 1. All pre-constitutional laws, if in part or
taken by regulatory authorities like the completely in conflict with the Constitution,
Telecom Regulatory Authority of India shall have all conflicting provisions
(TRAI) are although subjected to the scrutiny deemed ineffective until an amendment
of Appellate Tribunals, they can also be to the Constitution ends the conflict. In
challenged in the High courts and Supreme such the provision of that law will again
Court. come into force, if it is compatible with the
Q.1207 Consider the following statements. constitution as amended. This is called the
1. Judiciary is the final interpreter of the Doctrine of Eclipse.
Constitution. 2. In a similar manner, laws made after
2. Judiciary has the power to strike down laws adoption of the Constitution by the
passed by the Parliament if they violate Constituent Assembly must be compatible
the basic structure of the Constitution. with the constitution, otherwise the laws
Which of the above is/are correct? and amendments will be deemed to be
(a)  1 only (b)  2 only void ab initio.
(c)  Both 1 and 2 (d)  None 3. In such situations, the Supreme Court
Solution: (c) or High Court interprets the laws to
Justification: Statement 1: Supreme Court is decide if they are in conformity with the
the guardian, custodian and final interpreter Constitution. If such an interpretation is
of the constitution. This means that if a not possible because of inconsistency,
dispute arises regarding the meaning of and where a separation is possible,
specific provisions of the constitution, the the provision that is inconsistent with
court gives the final version of the meaning constitution is considered to be void. In
of those specific provisions, for example, addition to article 13, articles 32, 226
the Right to Life under Article 21 of the and 227 provide a constitutional basis to
constitution. judicial review in India.

Indian Polity Question Bank P.335

04-Indian Polity_Q1070-1506.indd 335 8/7/2018 7:42:39 PM


Judicial Activism powers and jurisdiction of the others. Judicial
activism may be creating strains on this
Q.1209 With reference to judicial activism, consider democratic principle.
the following statements
1. It has democratized the judicial system
by giving not just to individuals but also
(39) PUBLIC INTEREST LITIGATION (PIL)
groups access to the courts Q.1210 The concept of Public Interest Litigation
2. It has forced executive accountability originated in: [IAS 1997]
3. Judicial activism may be creating (a)  the United Kingdom
strains on this (b) Australia
4. It has overburdened the courts. (c)  the United States
5. It has also made an attempt to make the (d) Canada
electoral system much more free and fair Solution: (c)
Which of the above statements is incorrect?
Q.1211 Consider the following statements about the
(a) 4 Only (b) 5 Only
Public Interest Litigations (PILs).
(c) None (d) 2 Only
1. It need not be filed by the aggrieved party
Solution: (c)
only.
Judicial activism has had manifold impact
2. It may be introduced suo motu by the court.
on the political system. It has democratized
3. The provision of PILs is mentioned in the
the judicial system by giving not just to
constitution of India.
individuals but also groups access to the
Choose the correct answer using the codes
courts. It has forced executive accountability.
below:
It has also made an attempt to make the
(a)  1 and 2 (b)  2 and 3
electoral system much more free and fair.
(c)  1 and 3 (d)  All of the above
The court asked candidates contesting
Solution: (a)
elections to file affidavits indicating their
It was not mentioned in the constitution of
assets and income along with educational
India neither any law enacted by Parliament.
qualifications so that the people could elect
It is a result of Judicial activism
their representatives based on accurate
Public Interest Litigation is directly filed
knowledge.
by an individual or group of people in the
There is however a negative side to
supreme court. It was felt that their interests
the large number of the Public Interest
are undermined by the government.In such a
Litigations (PILs) and the idea of a pro-active
situation, the court directly accepts the public
judiciary. In the first place it has overburdened
good. It is a new legal horizon in which court
the courts. Secondly, judicial activism has
of law can initiate and enforce action to serve
blurred the line of distinction between the
and Secure significant Public Interest
executive and legislature on the one hand and
the judiciary on the other. The court has been Q.1212 Consider the following about Public Interest
involved in resolving questions which belong litigation (PIL).
to the executive. Thus, for instance, reducing 1. It is a legal instrument.
air or sound pollution or investigating cases 2. It can be entertained by both administrative
of corruption or bringing about electoral and judicial bodies.
reform is not exactly the duty of the 3. Representatives of victims can also file a
Judiciary. These are matters to be handled by PIL.
the administration under the supervision of 4. It can be filed only in social and
the legislatures. Therefore, some people feel environmental cases.
that judicial activism has made the balance Choose the correct answer using the codes
among the three organs of government very below.
delicate. Democratic government is based (a)  1, 2 and 4 only (b)  2 and 3 only
on each organ of government respecting the (c)  3 only (d)  3 and 4 only
Solution: (c)

P.336 For Civil Services Preliminary Examination

04-Indian Polity_Q1070-1506.indd 336 8/7/2018 7:42:39 PM


Public-Interest Litigation is litigation for the Select the correct answer using the codes
protection of the public interest. In Indian below.
law, Article 32 of the Indian constitution (a)  1 and 2 only (b)  2 and 3 only
contains a tool which directly joins the public (c)  2 only (d)  1 and 3 only
with judiciary. A PIL may be introduced in Solution: (c)
a court of law by the court itself (suo motu), Justification: Statement 1: It was introduced
rather than the aggrieved party or another by Justice P.N. Bhagwati of the Supreme
third party. For the exercise of the court’s Court.
jurisdiction, it is not necessary for the Statement 2: A PIL may be introduced
victim of the violation of his or her rights in a court of law by the court itself (suo
to personally approach the court. In a PIL, motu), rather than the aggrieved party or
the right to file suit is given to a member another third party. For the exercise of the
of the public by the courts through judicial court’s jurisdiction, it is not necessary for the
activism. The member of the public may be victim of the violation of his or her rights to
a Non-Governmental Organizations (NGOs), personally approach the court.
an institution or an individual. The Supreme Statement 3: refer previous question
Court of India, rejecting the criticism of Q.1215 Consider the following statements about
judicial activism, has stated that the judiciary Public Interest Litigations (PILs).
has stepped in to give direction because 1. Only the affected individual/group/
due to executive inaction, the laws enacted institution can file a PIL in the court.
by Parliament and the state legislatures for 2. The PILs are mentioned in Article 144 of
the poor since independence have not been the constitution of India to ensure social
properly implemented. Subodh Markandeya justice to the marginalized.
well known Senior Advocate of Supreme 3. Judiciary can consider a case on its own
court of India and Judicial activist believes based on a newspaper report or postal
that public interest litigation is the principal complaint received by the court.
legal remedy For a common man and it is Choose the correct answer using the codes
main weapon of judicial activist. below:
Q.1213 Consider the following statements about (a)  1 and 2 (b)  2 and 3
Public Interest Litigations (PILs) in India. (c)  1 and 3 (d)  3 only
1. The court can suo motu admit a PIL. Solution: (d)
2. The requirement of locus standi is waived The chief instrument through which judicial
off in PIL. activism has flourished in India is Public
3. The power of courts to issue PIL is Interest Litigation (PIL) or Social Action
mentioned in Articles 32 and 226 under Litigation (SAL).
special leave petitions. In normal course of law, an individual can
Select the correct answer using the codes approach the courts only if he/she has been
below. personally aggrieved. That is to say, a person
(a)  1 and 2 only (b)  2 and 3 only whose rights have been violated, or who is
(c)  2 only (d)  1 and 3 only involved in a dispute, could move the court
Solution: (a) of law. This concept underwent a change
Please refer previous questions. around 1979. In 1979, the Court set the trend
Q.1214 Consider the following statements about when it decided to hear a case where the case
Public Interest Litigations (PILs) in India. was filed not by the aggrieved persons but by
1. The PIL has been introduced by the others on their behalf. As this case involved
implementation of Directive Principles of a consideration of an issue of public interest,
State Policy (DPSP) by the Parliament. it and such other cases came to be known as
2. A PIL may be introduced in a court of law public interest litigations. Around the same
by the court itself (suo motu). time, the Supreme Court also took up the
3. Only an individual can file a PIL. case about rights of prisoners.

Indian Polity Question Bank P.337

04-Indian Polity_Q1070-1506.indd 337 8/7/2018 7:42:40 PM


This opened the gates for large number LOCAL GOVERNMENT
of cases where public spirited citizens and
voluntary organisations sought judicial (40) PANCHAYATI RAJ – PART IX –
intervention for protection of existing rights,
betterment of life conditions of the poor, ARTICLES 243 TO 243-O
protection of the environment, and many Q.1216 How does the state government exercise
other issues in the interest of the public. The control over the local self-governing bodies?
PIL has become the most important vehicle 1. By enacting laws, rules and regulations
of judicial activism. Judiciary, which is 2. By providing for provisions for regular
an institution that traditionally confined to elections
responding to cases brought before it, began 3. By releasing finances for development
considering many cases merely on the basis projects
of newspaper reports and postal complaints 4. By appointing the chairman and members
received by the court. Therefore, the term of Panchayats and Ward councils
judicial activism became the more popular Choose the correct answer using the codes
description of the role of the judiciary. below.
(a) 1 and 2 only (b) 2 and 3 only
(c) 1, 3 and 4 only (d) 1 and 3 only
Solution: (d)
All the members of a municipality shall be
elected directly by the people of the municipal
area. For this purpose, each municipal area
shall be divided into territorial constituencies
to be known as wards. The State government
does not directly appoint the members.
The state legislature may only provide
for the manner of reservation of offices
of chairpersons in the municipalities for
SCs, STs and women. The state legislature
may (a) authorise a municipality to levy,
collect and appropriate taxes, duties, tolls
and fees; (b) assign to a municipality taxes,
duties, tolls and fees levied and collected by
state government; (c) provide for making
grants-in-aid to the municipalities from the
consolidated fund of the state and (d) provide
for constitution of funds for crediting all
moneys of the municipalities.
Q.1217 The institutions of urban local government
originated and developed in modern India
during the period of British rule. Which of
the following substantiate the argument?
1. Local self-government was declared a
provincial subject under the Government
of India Act of 1935.
2. Lord Ripon’s Resolution of 1882 has been
hailed as the ‘Magna Carta’ of local self-
government.

P.338 For Civil Services Preliminary Examination

04-Indian Polity_Q1070-1506.indd 338 8/7/2018 7:42:40 PM


3. First Panchayats and Municipal (c) A is incorrect, but R is correct.
Corporations in India were setup under (d) A and R both incorrect.
the British. Solution: (a)
Choose the correct answer using the codes The federal system thus has dual objectives:
below. to safeguard and promote unity of the
(a)  1 and 2 only (b)  2 and 3 only country, while at the same time accommodate
(c)  1 and 3 only (d)  All of the above regional diversity.
Solution: (a) Therefore, two aspects are crucial for
The major events in this context are as the institutions and practice of federalism.
follows: Governments at different levels should agree
• In 1687-88, the first municipal corporation to some rules of power-sharing. They should
in India was set up at Madras. also trust that each would abide by its part
• In 1726, the municipal corporations were of the agreement. An ideal federal system
set up in Bombay and Calcutta. has both aspects: mutual trust and agreement
• Lord Mayos Resolution of 1870 on to live together. Panchayati Raj fits this
financial decentralisation visualised the description.
development of local self-government Q.1219 When the constitution was finalised
institutions. panchayats did not find a mention in it
• Lord Ripon’s Resolution of 1882 has been in compulsory provisions. Which of the
hailed as the ‘Magna Carta’ of local self- following can be cited as the most appropriate
governments. He is called as the father of reason among the following?
local self-governments in India. (a) Local elites and upper castes were so
• The Royal Commission on decentralisation well entrenched in society that local self-
was appointed in 1907 and it submitted its government only meant a continuing
report in 1909. Its chairman was C.E.H exploitation of the downtrodden masses
Hobhouse. of Indian society
• Under the dyarchical scheme introduced (b) There was a fierce opposition in the
in Provinces by the Government of Constituent assembly that panchayats
India Act of 1919, local self-government as self-governing bodies will collapse
became a transferred subject under the immediately on establishment
charge of a responsible Indian minister. (c) It was difficult to monitor the activities
• In 1924, the Cantonments Act was passed of the local bureaucracy from the Centre
by the Central legislature. (d) In a federal model of polity, the
• Under the provincial autonomy scheme constitution makers did not want to add
introduced by the Government of India a third tire because of the complexities it
Act of 1935, local self-government was will create.
declared a provincial subject. Solution: (a)
Q.1218 Consider the following statements: When the constitution was being drafted
Assertion (A): The Panchayati Raj in India is panchayats did not find a mention in it. At this
based on the principle of federalism. juncture, a number of members expressed
Reason (R): It has been established by the their sorrow, anger and disappointment
constitution and exercises administrative and over this issue. At the same time, drawing
taxation powers independent of Central and on his own rural experience Dr. Ambedkar
State government. argued that local elites and upper castes
In the context of the above statements, were so well entrenched in society that local
which of these is/are true? self-government only meant a continuing
(a) A and R both are true, and R is the exploitation of the downtrodden masses of
correct explanation for A. Indian society. The upper castes would no
(b) A and R both are true, and R is the NOT doubt silence this segment of the population
the correct explanation for A.

Indian Polity Question Bank P.339

04-Indian Polity_Q1070-1506.indd 339 8/7/2018 7:42:40 PM


further. The concept of local government 3. S. Ramachandran Committee
was dear to Gandhiji too. He envisaged each 4. Balwant Rai Mehta Committee
village as a self-sufficient unit conducting Select the correct answer using the codes
its own affairs and saw gram-swarajya to below.
be an ideal model to be continued after (a)  2 and 4 only (b)  1 and 2 only
independence. (c)  1 and 4 only (d)  3 only
Q.1220 Community Development Programme Solution: (d)
(1952) and the National Extension Service Learning: Details of these committees have
(1953) focussed on already been covered in Laxmikanth’s Indian
1. Rural reconstruction Polity, Fifth Edition and are not important
2. Technological upgradation of educational from examination point of view.
institutions • Balwant Roy Mehta committee (1957)
3. Capital goods led growth process to examine the working of Community
Select the correct answer using the codes development programme & National
below. extension programme It recommended
(a)  1 only (b)  2 and 3 only three-tier panchayati raj system
(c)  1 and 3 only (d)  1, 2 and 3 • Ashok Mehta committee (1977) to revive
Solution: (a) and strengthen the declining panchayati
Justification: Statement 1: Post raj system – It recommended two-tier
Independence, the first major development panchayati raj system with District
programme launched in India was panchayat as the executive body.
Community Development Programme, 1952. • L. M. Singhvi committee (1986)
Core philosophy was overall development of recommended that the Panchayati raj
rural areas and people’s participation. institutions should be constitutionally
• A year later, in 1953, the National recognised, protected and preserved.
Extension Service (NES) programme was • Various other Committees on Panchayati
launched with the idea of having wider Raj:
coverage at less cost and more people’s • V. T. Krishnammachari: 1960
participation. • Takhatmal Jain Study Group: 1966
• Each NES block had about 100 villages • G. V. K Rao committee:1985
and about 65 thousand population officer • Sarkaria Commission: 1986
(BDO) and had a number of extension Q.1222 Mandal Panchayats were recommended by:
officers (EDs). (a)  Balwant Rai Mehta Committee
• National Extension Service (NES) was (b)  Narasimhan Committee
thought of as the agency and CD as the (c)  Ashok Mehta Committee
method to bring about socio-economic (d)  Vengal Rao Committee
transformation of the rural people. Solution: (c)
Statement 2: Technology was a strong focus Ashok Mehta committee was constituted in
of later schemes, but the CDP and NES were 1977 to revive and strengthen the declining
aimed at rural revitalization with a pro-active Panchayati Raj system. It recommended two
administration. So, 2 is wrong. tier panchat system
Statement 3: The capital goods led • Mandal panchayat for group of villages
strategy was resorted to only after the 2nd • Zilla Parishad at the district level
FYP after the year 1956. So, 3 can be easily
eliminated. 73rd Constitutional Amendment
Q.1221 Which of the following committees is/
Q.1223 Consider the following with reference to the
are NOT associated with Panchayati Raj
73rd Constitutional Amendment Act of 1992.
Institutions in India?
1. The act gave a constitutional status to the
1. Ashok Mehta Committee
Panchayati Raj institutions.
2. L. M. Singhvi Committee

P.340 For Civil Services Preliminary Examination

04-Indian Polity_Q1070-1506.indd 340 8/7/2018 7:42:40 PM


2. This act has added a new directive them under the purview of the justiciable
principle of state policy under Part part of the Constitution. In other words, the
IV exhorting the state governments to state governments are under constitutional
promote self-government. obligation to adopt the new Panchayati Raj
3. The act institutionalises planning at the system in accordance with the provisions of
local level. the act. Consequently, neither the formation
Select the correct answer using the codes of panchayats nor the holding of elections
below. at regular intervals depend on the will of the
(a)  1 and 2 only (b)  1 and 3 only state government any more.
(c)  2 and 3 only (d)  1, 2 and 3 only The provisions of the act can be grouped
Solution: (b) into two categories—compulsory and
Justification: Statement 1: This means the voluntary. The compulsory (mandatory or
state governments are under constitutional obligatory) provisions of the act have to
obligation to adopt the new Panchayati Raj be included in the state laws creating the
system in accordance with the provisions of new Panchayati Raj system. The voluntary
the act. provisions, on the other hand, may be
Consequently, neither the formation of included at the discretion of the states. Thus
Panchayats nor the holding of elections at the voluntary provisions of the act ensures
regular intervals depends on the will of the the right of the states to take local factors like
state government any more. geographical, politico–administrative and
Statement 2: The act has given a practical others, into consideration while adopting the
shape to Article 40 (which already existed and new Panchayati Raj system.
statement 2 is thus wrong)  of the Constitution The act is a significant landmark in the
which says that, “The State shall take steps to evolution of grass-root democratic institutions
organise village panchayats and endow them in the country. It transfers the representative
with such powers and authority as may be democracy into participatory democracy. It is
necessary to enable them to function as units a revolutionary concept to build democracy at
of self-government.” This article forms a part the grass-root level in the country.
of the Directive Principles of State Policy. Q.1225 What was/were the essential change(s)
Statement 3: The creation of District brought by the 73rd constitutional amendment
Planning Committee to consolidate plans Act in the Panchayati Raj system in India?
prepared by Panchayati Raj Institutions 1. Panchayats were created by Act as they
(PRIs) and Urban Local Bodies (ULBs) is a did not exist in Independent India.
clear validation of this statement. 2. Panchayati Raj System received
Q.1224 Which of the following were the implications constitutional status.
of the 73rd Constitutional Amendment? 3. It fixed the size of Panchayats and granted
1. Constituting Panchayats became executive power to the panchs.
mandatory for states. Select the correct answer using the codes
2. Panchayats were brought under the below.
justiciable part of the Constitution. (a)  1 and 2 only (b)  2 only
3. The Structure of Panchayats across (c)  1 and 3 only (d)  1, 2 and 3
the country was made uniform and Solution: (b)
harmonised. Justification: Statement 1: Rajasthan was
Choose the correct answer using the codes the first state to establish Panchayati Raj in
below. 1959. Other states also followed suit, so 1 is
(a)  1 and 2 only (b)  2 and 3 only wrong.
(c)  1 and 3 only (d)  All of the above Statement 2: Though most of the states
Solution: (d) created Panchayati Raj institutions by mid
The act gives a constitutional status to the 1960s, there were differences from one state
Panchayati Raj institutions. It has brought to another with regard to the number of tiers,

Indian Polity Question Bank P.341

04-Indian Polity_Q1070-1506.indd 341 8/7/2018 7:42:40 PM


relative position of samiti and parishad, their (a)  Public Utility Services
tenure, composition, functions, finances and (b)  Law and Order
so on. (c)  Public Health
The 73rd Act harmonized all the systems (d) Sanitation
into a common structure and granted PRIs Solution: (b)
constitutional status. Justification: Law and order is a state subject
Statement 3: Panchayat is elected from Twenty-nine subjects (of 11th Schedule of
area wards, and its size is not fixed by the the Constitution), which were earlier in the
constitution. State list, were transferred to the Panchayati
Q.1226 Consider the following statements about the Raj Institutions (optionally) by the 73rd
73rd amendment to the constitution: amendment, such as animal husbandry, local
1. Before the amendment the Panchayats in markets etc.
the states existed at the will of the state The functions of Urban local bodies have
government. been listed in the Twelfth Schedule of the
2. It establishes a three-tier system of Constitution, such as maintenance of public
Panchayati Raj all over the country parks, street lighting, drainage and sewerage
uniformly. etc.
3. It prescribes for direct elections to fill all Q.1229 Which of the following functions can be
the posts in these rural local bodies. devolved to a Gram Panchayat by the State
Which of these is/are true? Legislature?
a. 1 and 2 (b) 2 and 3 1. Framing rules and regulations on social
c. 1 and 3 (d) Only 1 conduct in villages
Solution: (d) 2. The construction and maintenance of
Three tier Panchayat is established only in water sources in the village
states with population more than 10 lakhs. 3. Levying and collecting taxes on public
The posts of chairman at intermediate and utilities
higher levels are filled by indirect elections. Select the correct answer using the codes
Q.1227 As per the 73rd Constitutional amendment, below.
the power to expand the responsibilities of (a)  1 and 2 only (b)  2 only
the panchayati raj bodies rests with (c)  2 and 3 only (d)  1, 2 and 3
(a) State legislatures (b) Governor Solution: (c)
(c)  President of India (d)  Parliament Justification: Statement 1: Laws concerning
Solution: (a) villages can be made only by the State
The provisions of the act can be grouped into Legislature, and rules/regulations by the
two categories—compulsory and voluntary. State government. So, 1 is incorrect.
The compulsory (mandatory or obligatory) The work of a Gram Panchayat includes
provisions of the act have to be included in the construction and maintenance of water
the state laws creating the new Panchayati Raj sources, roads, drainage, school buildings
system. The voluntary provisions, on the other and other
common property resources.
hand, may be included at the discretion of the It can also levy and collect local taxes
states. Thus the voluntary provisions of the act and execute government schemes related to
ensure the right of the states to take local factors generating employment in the village.
like geographical, politico–administrative and Q.1230 As per the Constitution, Which of the
others, into consideration while adopting the following functional items can be kept within
new Panchayati Raj system. the purview of the Panchayats?
1. Public Distribution System
Eleventh Schedule 2. Welfare of Weaker Sections
3. Primary Health Centres
Q.1228 Which is of the following subjects is NOT
4. Adult and non-formal education
the concern of a local government in India?

P.342 For Civil Services Preliminary Examination

04-Indian Polity_Q1070-1506.indd 342 8/7/2018 7:42:40 PM


Choose the correct answer using the codes Other Important items in the list are:
below. 1. Agriculture, including agricultural
(a)  1 and 2 only (b)  2, 3 and 4 only extension
(c)  1, 3 and 4 only (d)  All of the above 2. Small-scale industries, including food
Solution: (d) processing industries
Some of the Subjects under the 11th schedule 3. Roads, culverts, bridges, ferries, waterways
that can be devolved to the Panchayats are: and other means of communication
1. Poverty alleviation programme 4. Rural electrification, including
2. Education, including primary and distribution of electricity
secondary schools Q.1232 Which of the following bodies were
3. Technical training and vocational constituted in pursuance of the 73rd and 74th
education constitutional amendment?
4. Adult and non-formal education 1. State Finance Commission
5. Libraries 2. State Election Commission
6. Cultural activities 3. District Planning Committees
7. Markets and fairs 4. District Rural Development Agency
8. Health and sanitation including hospitals, Choose the correct answer using the codes
primary health centres and dispensaries below.
9. Family welfare (a)  1 and 2 only (b)  2, 3 and 4 only
10. Women and child development (c)  1, 3 and 4 only (d)  1, 2 and 3 only
11. Social welfare, including welfare of the Solution: (d)
handicapped and mentally retarded • The governor of a state shall, after every
12. Welfare of the weaker sections, and in five years, constitute a finance commission
particular, of the scheduled castes and to review the financial position of the
the scheduled tribes Panchayats.
13. Public distribution system • The superintendence, direction and
14. Maintenance of community assets control of the Preparation of electoral
Q.1231 Eleventh Schedule contains a list of functional rolls and the conduct of all elections to
items that can be placed within the purview of the Panchayats shall be vested in the state
panchayats by the State Legislatureas provided election commission. It consists of a state
for under the 73rd Constitutional Amendment election commissioner to be appointed by
Act. the governor. His conditions of service and
Which of these are parts of the list? tenure of office shall also be determined
1. Minor irrigation and watershed by the governor.
development • The District Rural Development Agency
2. Implementation of land reforms (DRDA) has been abandoned now. They
3. Technical training and vocational were not constituted in furtherance of
education the provisions of these constitutional
4. Public distribution system amendments.
Select the correct answer using the codes
below. Gram Sabha
(a)  1, 2 and 3 only (b)  1 and 2 only
Q.1233 As per the Constitution, every Panchayat
(c)  3 and 4 only (d)  1, 2, 3 and 4
should have a Gram Sabha. A Gram Sabha is
Solution: (d)
a body consisting of
Learning: The State Legislature can devolve
(a) The whole population of the villages
powers and responsibilities upon Panchayats
under the Panchayat other than Children
to prepare plans for economic development
of less than five years
and social justice; and to perform some or
(b) The adult population of the villages
all of the 29 functions listed in the Eleventh
under the Panchayat
Schedule of the Constitution.

Indian Polity Question Bank P.343

04-Indian Polity_Q1070-1506.indd 343 8/7/2018 7:42:40 PM


(c) The registered voters of the villages Q.1235 Consider the following statements about
under the Panchayat Panchayati raj:
(d) All the Village committees constituted 1. All the residents of the village are
by the Panchayat members of Gram Sabha.
Solution: (c) 2. Panchayat is elected by the Gram Sabha.
The 73rd amendment act provides for a Gram 3. The Gram Panchayat is financially
Sabha as the foundation of the Panchayati accountable to the Gram Sabha.
Raj system. It is a body consisting of persons Which of the above statements is/are true?
registered in the electoral rolls of a village (a)  1 and 2 (b)  2 and 3
comprised within the area of Panchayat at the (c)  1 and 3 (d)  Only 2
village level. Thus, it is a village assembly Solution: (b)
consisting of all the registered voters in the area Any one who has the right to vote and lives
of a Panchayat. It may exercise such powers in the premises if a Panchayat is a member
and perform such functions at the village level of the Gram Sabha. Panchayat is elected by
as the legislature of a state determines. it. The Sarpanch is usually elected by the
Three-Tier System: The act provides Panchayat–it depends from state to state.
for a three-tier system of Panchayati Raj in The Gram Sabha keeps a track of the work
every state, that is, Panchayats at the village, of Gram Panchayat and prevents misuse of
intermediate, and district levels. Thus, the money. Some of the works by Gram Panchayat
act brings about uniformity in the structure has to be approved by the Gram Sabha.
of Panchayati Raj throughout the country. Q.1236 Consider the following statements.
However, a state having a population not 1. The Gram Sabha keeps a track of the work
exceeding 20 lakh may not constitute of Gram Panchayat and prevents misuse
Panchayats at the intermediate level. of money.
Q.1234 Gram Sabha is organized so that 2. The Gram Sabha can be assigned
(a) All eligible voters can register themselves responsibility for the identification of
periodically persons as beneficiaries under the poverty
(b) Panchayat is elected alleviation programmes.
(c) people directly participate and seek Which of the above is/are correct?
answers from their elected representatives (a)  1 only (b)  2 only
(d) Local bureaucracy can dictate (c)  Both 1 and 2 (d)  None
development targets for the Panchayat Solution: (c)
Solution: (c) Justification: Statement 1: Every Panchayat
Explanation: Elections take place when at the village level are required to obtain from
State Election Commission organizes it. So the Gram Sabha a certification of utilisation
both options (a) and (b) are wrong. of funds by that Panchayat for such plans,
Local bureaucracy cannot dictate terms programmes and projects implemented.
to a body of voters who elect the Panchayat. Statement 2: They also approve of the
The duty of the bureaucracy is to help the plans, programmes and projects for social
Gram Sabha in the discharge of its duties. So, and economic development before such
option (d) is also wrong. plans, programmes and projects are taken up
Learning: The Gram Sabha is a meeting for implementation by the Panchayat at the
of all adults who live in the area covered by village level.
a Panchayat. This could be only one village Learning: The term Gram Sabha has been
or a few villages. In some states, as in the defined in the Constitution of India under
example above, a village meeting is held for Article 243(b) and some of its powers and
each village. responsibilities have been mentioned in acts
Anyone who is 18 years old or more and the Provisions of the Panchayats (Extension
who has the right to vote is a member of the to Scheduled Areas) Act (PESA), 1996 and
Gram Sabha. Forest Rights Act (FRA), 2006.

P.344 For Civil Services Preliminary Examination

04-Indian Polity_Q1070-1506.indd 344 8/7/2018 7:42:40 PM


Some other powers of Gram Sabha the manner of selection of the Panchayat
include: chairperson is decided by the state assembly.
• the power to enforce prohibition or Q.1239 Consider the following statements about
to regulate or restrict the sale and local self-government in India.
consumption of any intoxicant; 1. It is constitutionally mandatory to hold
• the ownership of minor forest produce; regular elections to local government
• the power to prevent alienation of land in bodies.
the Scheduled Areas 2. An independent institution, State Election
Commission, has been created in each
Elections–Duration State to conduct Panchayat and municipal
Q.1237 All the members of Panchayats at the village, elections.
intermediate and district levels are generally Which of the above is/are correct?
(a)  Elected directly by the people (a)  1 only (b)  2 only
(b) Indirectly elected by the ward councillors (c)  Both 1 and 2 (d)  None
(c) Partly elected by the people and partly Solution: (c)
nominated by the State government Learning: Seats are reserved in the elected
(d) Nominated by the State government bodies and the executive heads of these
based on the recommendation of the Zila institutions for the Scheduled Castes
Parishad Scheduled Tribes and Other Backward
Solution: (a) Classes (OBC provision can be made by
Learning: The constitution provides that State legislatures as it is not compulsory). At
these members of Panchayats at the village, least one-third of all positions are reserved
intermediate and district levels shall be for women.
elected directly by the people. The State governments are required to
Further, the chairperson of Panchayats at share some powers and revenue with local
the intermediate and district levels shall be government bodies. The nature of sharing
elected indirectly—by and from amongst the varies from State to State.
elected members thereof. Q.1240 The superintendence, direction and control
However, the chairperson of a Panchayat of the preparation of electoral rolls and the
at the village level shall be elected in such conduct of all elections to the Panchayats
manner as the state legislature determines. is vested in the State Election Commission.
Q.1238 Consider the following statements about the Consider the following statements about it.
election of chairpersons in rural local bodies: 1. The state election commissioner is
1. Chairperson of a Panchayat at the village appointed by the President.
level is invariably chosen by the ward 2. His conditions of service and tenure
representatives (panchs). of office are determined by the State
2. The Chairpersons at the intermediate Legislature.
and district level need not necessarily be 3. He can be removed from the office only in
from amongst the members of these local the manner and grounds prescribed for the
bodies. removal of a judge of the state high court.
3. One-third of seats of chairpersons at all Select the correct answer using the codes
levels in these bodies are reserved for below.
women. (a)  1 and 2 only (b)  3 only
Which of these is/are true? (c)  2 and 3 only (d)  1, 2 and 3
(a) 1 and 2 (b) 2 and 3 Solution: (b)
(c) 1 and 3 (d) Only 3 Learning: Statements 1 and 2: He is
Solution: (d) appointed by the Governor, and his terms of
Chairperson at the intermediate and district service are also determined by the Governor.
level must necessarily be a member. Besides, So, both 1 and 2 would be wrong.

Indian Polity Question Bank P.345

04-Indian Polity_Q1070-1506.indd 345 8/7/2018 7:42:40 PM


Statement 3: This means, while he Q.1243 Every Panchayat, unless sooner dissolved
is appointed by the Governor, he can be under any law for the time being in force,
removed only by the President, and not the shall continue for five years from the date
Governor. It is not a subordinate body to appointed for its
the Election Commission of India. It has a (a) Taking of oath of office by the elected
separate constitutional existence. members
Q.1241 Consider the following statements about the (b)  First audit by Gram Sabha
elections to the Panchayats: (c)  Declaration of the election results
1. A person who is not qualified to contest (d)  Its first meeting
a state assembly election cannot contest Solution: (d)
local body elections too. Learning: Panchayat is an institution of self-
2. The elections to Panchayats are conducted government constituted under article 243B,
by the State Election Commission which for the rural areas.
is subordinate to the Central election Duration is five years from the date of
Commission. its first meeting. Moreover, an election to
Which of these is/are true? constitute a Panchayat shall be completed
(a)  Only 1 (b)  Only 2 before the expiry of its duration or before six
(c)  Both (d)  None of the above months from the date of its dissolution.
Solution: (d) A Panchayat constituted upon the
A person of the age 21 is not qualified to fight dissolution of a Panchayat before the
a state assembly election, but can fight a local expiration of its duration shall continue only
body election. for the remainder of the period for which the
The SEC is not subordinate to the dissolved Panchayat would have continued.
CEC in the same way as the states are not
subordinates to the centre. Powers and Function
Q.1242 You are a student in a local university and Q.1244 Which of the following powers of can
have filed a nomination paper for contesting be devolved to the Panchayats as per the
Panchayat elections. The State Election Constitution of India?
Commission would deem your nomination 1. Levying taxes
valid only if 2. Preparing plans for economic development
1. You have obtained an approval letter from 3. Managing local water supply and ponds
your university Choose the correct answer using the codes
2. You have a voter ID card below:
3. You have already held some leadership (a)  1 and 2 only (b)  2 and 3 only
position in the college/university. (c)  1 and 3 only (d)  All of the above
Choose the correct answer using the codes Solution: (d)
below. Panchayats should be given powers and
(a)  1 and 2 only (b)  1 and 3 only authority to function as institutions of self-
(c)  2 and 3 only (d)  2 only government. It, thus, requires all state
Solution: (d) governments to revitalise local representative
The qualifications needed to fight local institutions.
elections is that The following powers and responsibility
• You should be at least 21 years of age were delegated to the Panchayats:
• You should be a citizen of India. • to prepare plans and schemes for economic
• You should not have been disqualified development
under the Representation of People  Act • to promote schemes that will enhance
(RPI), 1951. social justice
• All other qualifications remaining the • to levy, collect and appropriate taxes,
same as that of a MP or a MLA. No college duties, tolls and fees
verification, approval etc. is needed.

P.346 For Civil Services Preliminary Examination

04-Indian Polity_Q1070-1506.indd 346 8/7/2018 7:42:40 PM


• help in the devolution of governmental Which of these is/are true?
responsibilities, especially that of finances (a) 1 and 2 (b) 2 and 3
to local authorities (c)  1 and 3 (d)  All of the above
Social welfare responsibilities of the Solution: (a)
Panchayats include the maintenance of Panchayat can levy the taxes if authorized by
burning and burial grounds, recording the state legislature. They plan for the socio
statistics of births and deaths, establishment economic development.
of child welfare and maternity centres, But higher level panchayats can’t dissolve
control of cattle etc. lower level panchayats. Only state legislature
Q.1245 The state legislature may can do so.
1. Authorise a panchayat to levy a tax Q.1247 Development plans prepared by Panchayat
2. Provide for making grants-in-aid to the Samitis are forwarded up in the hierarchy to
panchayats from the consolidated fund of 1. Zila Parishad
the state 2. District Planning Committee
3. Provide for constitution of funds for 3. Gram Panchayat
crediting all moneys of the panchayats 4. State Planning departments
Select the correct answer using the codes Select the correct answer using the codes
below. below.
(a)  1 and 2 only (b)  3 only (a)  1, 2 and 4 only (b)  2 and 3 only
(c)  2 and 3 only (d)  1, 2 and 3 (c)  1 only (d)  1 and 3 only
Solution: (d) Solution: (a)
Justification: Following can be provided by Justification: Hierachy is as follows: Gram
the state legislature- Sabha (Gram Panchayat)< Panchayat Samitis
• Authorise a panchayat to levy, collect and < Zila Parishad. The plans forwarded from
appropriate taxes, duties, tolls and fees; here go to the District Planning Committee
• Assign to a panchayat taxes, duties, tolls (DPC) and then to the state planning boards,
and fees levied and collected by the state and then to the Centre in case some assistance
government; is needed.
• Provide for making grants-in-aid to the DPC, as a constitutional body, consolidates
panchayats from the consolidated fund of the plans prepared by the PRIs and urban
the state; and local bodies and sends it to the higher levels.
• Provide for constitution of funds for
crediting all moneys of the panchayats Other Provisions
If you observe, this is similar to the pattern
Q.1248 The Panchayati Raj Institutions (PRIs)
of financial relations between the Union
as established by the 73rd Constitutional
and States. This is why the coming of of
Amendment do NOT exist in
Panchayati Raj Institutions (PRIs) is said
1. All North-eastern states
to add a new wheel of local federalism in
2. Any of the Union Territories
India.
Which of the above is/are correct?
Q.1246 Consider the following statements about the (a)  1 only
powers and functions of the Panchayati raj (b)  2 only
bodies? (c)  Both 1 and 2
1. They can levy and collect taxes if provided (d) None of the above options is correct
by the state legislature. Solution: (d)
2. They can plan for economic development Justification: The Panchayat Raj system
and social justice. exists in all the states except Nagaland,
3. A higher level Panchayati raj body can Meghalaya and Mizoram and in all the Union
dissolve a lower level body if provided by territories except Delhi.
a state law.

Indian Polity Question Bank P.347

04-Indian Polity_Q1070-1506.indd 347 8/7/2018 7:42:40 PM


In Sixth Schedule areas, autonomous Justification: Statement 1: The Act bars the
tribal institutions exist for governance, and in interference by courts in the electoral matters
fifth schedule areas where 73rd amendment of panchayats.
does not apply, PESA 1996 was enacted to • It declares that the validity of any
extend the PRI institution. law relating to the delimitation of
Q.1249 The 73rd Constitutional Amendment Act constituencies or the allotment of seats to
does NOT apply to the states of such constituencies cannot be questioned
1. Jammu & Kashmir in any court.
2. Nagaland • It further lays down that no election to
3. Mizoram any panchayat is to be questioned except
4. Assam by an election petition presented to such
5. Arunachal Pradesh authority and in such manner as provided
Select the correct answer using the codes by the state legislature.
below. Q.1251 Consider the following statements.
(a)  1, 2 and 3 only (b)  2, 3 and 5 only 1. The constitution declares that the validity
(c)  1, 3, 4 and 5 only (d)  1, 2, 4 and 5 only of any law relating to the delimitation of
Solution: (a) constituencies of local bodies cannot be
Justification and Learning: The act does questioned in any court.
not apply to the states of Jammu and Kashmir, 2. The constitution provides that the awards
Nagaland, Meghalaya and Mizoram and of the State Finance Commission with
certain other areas. respect to the local bodies cannot be
These areas include, modified by the State government other
1. the scheduled areas and the tribal areas than in extraordinary situations.
in the states; Which of the above is/are correct?
2. the hill area of Manipur for which a (a)  1 only (b)  2 only
district council exists; (c)  Both 1 and 2 (d)  None
3. Darjeeling district of West Bengal for Solution: (a)
which Darjeeling Gorkha Hill Council Justification: Statement 1: This is because
exists. the 73rd amendment act bars the interference
However, the Parliament may extend the by courts in the electoral matters of
provisions of this Part to the scheduled areas panchayats.
subject to such exceptions and modifications It further lays down that no election to any
as it may specify. panchayat is to be questioned except by an
Under this provision, the Parliament has election petition presented to such authority
enacted the Provisions of the Panchayats and in such manner as provided by the state
(Extension to the Scheduled Areas) Act, legislature.
(PESA), 1996. Statement 2: The 73rd amendment act only
Q.1250 The 73rd Constitutional Amendment Act establishes SFCs and gives them authority to
1. Declares that the validity of any recommend devolution of resources to local
law relating to the delimitation of bodies. However, their award, similar to
constituencies cannot be questioned in UFCs, is not binding on the state government
any court. as they are advisory bodies.
2. Authorizes the State Legislature to make
provisions with respect to all matters Compulsory & Voluntary Provision
relating to elections to the panchayats. Q.1252 Which of the following is NOT a compulsory
Which of the above is/are correct? provision under the 73rd constitutional
(a)  1 only (b)  2 only amendment Act?
(c)  Both 1 and 2 (d)  None (a) 21 years to be the minimum age for
Solution: (c) contesting elections to panchayats.

P.348 For Civil Services Preliminary Examination

04-Indian Polity_Q1070-1506.indd 348 8/7/2018 7:42:40 PM


(b) Organisation of Gram Sabha in a village Choose the correct answer using the codes
or group of villages. below.
(c) Establishment of a State Election (a)  1 and 2 only (b)  2 and 3 only
Commission for conducting elections to (c)  1 and 3 only (d)  All of the above
the panchayats. Solution: (a)
(d) Establishment of District Planning Devolution of funds and the authorizing
Committee for consolidation of plans panchayats to levy taxes, fees are the
prepared by the local bodies prerogative of the state legislature.
Solution: (d) Q.1255 As per the compulsory provisions of the 73rd
Justification: Option (d) is a compulsory amendment Act to the Constitution of India
provision under the 74th constitutional 1. All the members of panchayats at the
amendment Act. village, intermediate and district levels
Other important compulsory provisions are: shall be elected directly by the people.
• Constitution of a State Finance 2. Not less than one-third of the total number
Commission after every five years to of seats will be reserved for women.
review the financial position of the 3. All expenditure within a Panchayat will
panchayats. be socially audited by the Gram Sabha,
• Reservation of one-third seats (both led by the Panchayat Secretary.
members and chairpersons) for women in 4. All chairpersons in the Panchayati Raj
panchayats at all the three levels. Institutions (PRIs) will be nominated and
• Fixing tenure of five years for panchayats appointed by the Governor of the State.
at all levels and holding fresh elections Select the correct answer using the codes
within six months in the event of below.
supersession of any panchayat. (a)  1 and 2 only (b)  1, 3 and 4 only
Q.1253 Which of the following provision are (c)  2 and 3 only (d)  1, 2 and 3 only
compulsory provisions under the 73d Solution: (a)
amendment to the constitution? Learning: Statement 1 and 4: Only
1. Constitution of state finance commission chairpersons of panchayats at the
every five years. intermediate and district levels shall be
2. Providing reservations of seats for elected indirectly—by and from amongst the
backward classes in Panchayats all levels. elected members thereof.
3. Fixing tenure of five years for Panchayats However, the chairperson of a panchayat
for all levels. at the village level shall be elected in such
Choose the correct answer using the codes manner as the state legislature determines.
below: Statement 2: Also, not less than one-third
(a) 1 and 2 (b) 2 and 3 of the total number of offices of chairpersons
(c) 1 and 3 (d) All of the above in the panchayats at each level shall be
Solution: (d) reserved for women.
The higher level bodies have no power to Statement 3: Expenditure is socially
dissolve the lower level bodies. Only the audited when mandated by the law or
state government can do it that too on some government scheme, such as Mahatma Gandhi
conditions National Rural Employment Guarantee Act
Q.1254 Which of these constitutional provisions (MGNREGA). However, Gram Sabha is
under 73rd amendment is/are compulsory for authorized to look into the financial records
every Indian state to follow? maintained by the Gram Panchayat.
1. Organisation of Gram Sabha in a village Q.1256 Consider the following provisions under the
or group of villages 73rd amendment.
2. Direct elections to all seats in panchayats at 1. Direct elections to all seats in panchayats
the village, intermediate and district levels at the village, intermediate and district
3. Granting financial powers to the panchayats levels.

Indian Polity Question Bank P.349

04-Indian Polity_Q1070-1506.indd 349 8/7/2018 7:42:40 PM


2. Indirect elections to the post of chairperson (a)  1 and 2 (b)  2 and 3
of panchayats at the intermediate and (c)  1 and 3 (d)  None of the above
district levels. Solution: (a)
3. Providing reservation of seats for The constitution via the 73rd amendment
backward classes in panchayats at any provides the skeleton for rural government.
level The rest is decided by the states by laws
Under the 73rd amendment, these above enacted in the legislature.
provisions are The District Panchayat is the at the top of
(a)  Compulsory for most states the hierarchy in rural government. It prepares
(b)  Voluntary for most states plan for the whole district with the help of
(c) Some are compulsory, some voluntary Panchayat samitis and municipalities. In this
for states way it regulates the sanctioning of money to
(d) Dependent on whether the state is a the gram Panchayats.
special category state or not. Panchayat secretary is a bureaucrat and
Solution: (c) selected by the Civil Services examination
These are voluntary provisions: of the concerned state. The state government
• Giving representation to members of appoints the Panchayat secretary.
the Parliament (both the Houses) and Q.1258 Which of the following goes against the
the state legislature (both the Houses) in ideal of Panchayati Raj Institutions (PRIs) as
the panchayats at different levels falling enshrined in the constitution?
within their constituencies. (a) Delegation of financial powers to local
• Providing reservation of seats (both bodies
members and chairpersons) for backward (b) Centralization of powers in the hands of
classes in panchayats at any level. bureaucracy
• Granting powers and authority to the (c) Decentralization of planning process to
panchayats to enable them to function as Gram Sabhas
institutions of self-government (in brief, (d) Increased political accountability
making them autonomous bodies). through local social audits
• Devolution of powers and responsibilities Solution: (b)
upon panchayats to prepare plans for Justification: Option B: Core values behind
economic development and social justice; establishment of PRIs are decentralization,
and to perform some or all of the 29 accountability and increased delegation of
functions listed in the Eleventh Schedule function for empowerment of people at grass
of the Constitution. roots level.
• Granting financial powers to the Centralization of powers in hands of civil
pachayats, that is, authorizing them to servants goes against this very notion of
levy, collect and appropriate taxes, duties, empowerment, and thus wrong.
tolls and fees.

Miscellaneous (41) MUNICIPALITIES – PART IXA –


ARTICLES 243-P TO 243ZG
Q.1257 Consider the following statements about
rural administration: Q.1259 Consider the following statements.
1. Each state has its own set of laws regarding 1. The first municipal corporation in India
Panchayats. was set up after Lord Ripon’s Resolution
2. The District or Zila Parishad plays a role of 1882 on self-governance.
in the sanctioning of money to Gram 2. Under the provincial autonomy scheme
Panchayats. introduced by the Government of India
3. Panchayat secretary, who assists the Act of 1935, local self-government was
Panchayat, is chosen by the Sarpanch. declared a provincial subject.
Which of the above statements is/are true?

P.350 For Civil Services Preliminary Examination

04-Indian Polity_Q1070-1506.indd 350 8/7/2018 7:42:40 PM


Which of the above is/are correct? became a transferred subject under the charge
(a)  1 only (b)  2 only of a responsible Indian minister. So, 2 is also
(c)  Both 1 and 2 (d)  None wrong.
Solution: (b) One of the reasons for its late constitutional
Justification: Statement 1: In 1687-88, the recognition was the lack of political
first municipal corporation in India was set consensus in India, especially between the
up at Madras. Centre and States.
• In 1726, the municipal corporations were Q.1261 At the Central level, the subject of ‘urban
set up in Bombay and Calcutta. local government’ is dealt with by which of
• Lord Mayo’s Resolution of 1870 on the following ministries?
financial decentralisation visualised the 1. Ministry of Home Affairs
development of local self-government 2. Ministry of Urban Development
institutions. 3. Ministry of Defence
• Lord Ripon’s Resolution of 1882 has been Choose the correct answer using the codes
hailed as the ‘Magna Carta’ of local self- below.
government. He is called as the father of (a)  1 and 2 only (b)  2 and 3 only
local-self government in India. (c)  1 and 3 only (d)  All of the above
Statement 2: Under the dyarchical scheme Solution: (d)
introduced in Provinces by the Government The system of urban government was
of India Act of 1919, local self-government constitutionalised through the 74th
became a transferred subject under the charge Constitutional Amendment Act of 1992.
of a responsible Indian minister. At the Central level, the subject of ‘urban
Then later it wade a provincial subject local government‘ is dealt with by the
under Government of India (GoI) act, 1935. following three ministries: Ministry of Urban
Q.1260 The institutions of urban local government Development, created as a separate ministry
originated and developed in modern India in 1985; Ministry of Defence in the case of
during the period of British rule. But, in cantonment boards; and Ministry of Home
Independent India it received constitutional Affairs in the case of Union Territories.
recognition quite late in 1993. Which of Q.1262 A State Legislature may provide for the
the following can be the possible historical representation of which of the following
reason(s) for it? persons in a municipality?
1. The subject of Local self-government was 1. Persons having special knowledge or
never devolved and decentralized at the experience in municipal administration
provincial level. 2. Social activists
2. The subject of Local self-government was 3. Professors of Deemed Universities
never handed over to a Indian Minister situated within the municipal area
who would be responsible to a legislative 4. The MLAs or MPs registered as electors
assembly. within the municipal area
Which of the above is/are correct? Select the correct answer using the codes
(a)  1 only (b)  2 only below.
(c)  Both 1 and 2 (d)  None (a)  1 and 4 only (b)  2, 3 and 4 only
Solution: (d) (c)  1 only (d)  1, 2 and 3 only
Justification: Statement 1: Under the Solution: (a)
provincial autonomy scheme introduced by Learning: This is the complete list:
the Government of India Act of 1935, local • Persons having special knowledge or
selfgovernment was declared a provincial experience in municipal administration
subject. So, 1 is clearly wrong. without the right to vote in the meetings
Statement 2: Under the dyarchical scheme of municipality.
introduced in Provinces by the Government • The members of the Lok Sabha and the
of India Act of 1919, local self-government state legislative assembly representing

Indian Polity Question Bank P.351

04-Indian Polity_Q1070-1506.indd 351 8/7/2018 7:42:40 PM


constituencies that comprise wholly or not the ECI which conducts elections at the
partly the municipal area. Union and state levels.
• The members of the Rajya Sabha and DPCs are bodies constituted by indirect
the state legislative council registered as elections by bodies like municipalities and
electors within the municipal area. panchayats. It can never conduct or supervise
• The chairpersons of committees (other their polling.
than wards committees) Q.1265 Consider the following statements about
Q.1263 Every state is required to constitute a District District Planning Committees. Which of
Planning Committee (DPC) to consolidate these is/are correct?
the plans prepared by panchayats and 1. The whole committee consists of only
municipalities in the district. As per the elected members of the local bodies of all
Constitution of India, all the members of the levels.
DPC should be 2. It consolidates the plans prepared by both
(a) Elected by the elected members of the the rural and urban local bodies.
panchayats and municipalities 3. The Chairperson of the committee
(b) Indirectly elected by the Zila Parishad invariably is a directly elected member of
(c) Nominated by the Governor of the State the local bodies.
in consultation with the Chairman of the Choose the correct answer using the codes
Zila Parishad below:
(d) None of the above (a)  1 and 2 (b)  Only 2
Solution: (d) (c)  1 and 3 (d)  2 and 3
Learning: The act lays down that only four- Solution: (c)
fifths of the members of a district planning Reservations for other backward communities
committee should be elected by the elected (OBCs) fall under the voluntary provisions.
members of the district panchayat and Q.1266 Which of the following functions are
municipalities in the district from amongst obligatory in all municipalities?
themselves. 1. Registration of births and deaths.
The representation of these members in 2. Supply of potable water.
the committee should be in proportion to the 3. Planting and maintenance of road side trees.
ratio between the rural and urban populations 4. Provision of transport facilities within the
in the district. municipality.
The State Legislature makes provision for (a)  I and II (b)  II and III
rest of the composition of DPC. (c)  III and IV (d)  I, II, III and IV
The chairperson of such committee shall Solution: (d)
forward the development plan to the state 12th schedule prescribes the list of functions
government. that is to be performed by the municipalities.
Q.1264 The superintendence, direction and control
of the preparation of electoral rolls and the Types of Urban Government
conduct of all elections to the municipalities
Q.1267 In a metropolitan city, who is the authority to
is vested in the
usually decide where a public park should be
(a)  Election Commission of India
established?
(b) District Planning Committee (DPC)
(a)  CEO, Zila Parishad
(c)  State Election Commission
(b)  District Collector
(d)  State Department of Urban Affairs
(c)  Local MLAs Committee
Solution: (c)
(d)  Ward Councillors
Learning: The state legislature makes
Solution: (d)
provision with respect to all matters relating
Explanation: CEO, Zila Parishad is usually
to elections to the municipalities. But, the
an IAS officer who is incharge of executing
conduct of elections is vested with the SEC,
decision taken by the Ward Councillors.

P.352 For Civil Services Preliminary Examination

04-Indian Polity_Q1070-1506.indd 352 8/7/2018 7:42:40 PM


He does not take such public decisions (d) a panchayat that is temporarily being
himself. He can advise the councillors governed by a Municipality or Municipal
though. So, option (a) is incorrect. corporation
District Collector in a metropolitan area Solution: (b)
does not enjoy as many powers as he does 74th Amendment Act of 1992 provides for
in a small town or a village. Municipal the constitution of the following three types
Corporation enjoys greater authority in of municipalities in every state.
allocating public resources in a metropolitan (a) A Nagar Panchayat for a transitional area,
city. Therefore, option (b) is incorrect. that is, an area in transition from a rural
There is no official local MLAs committee area to an urban area.
as such. Even if the MLAs were to form (b) A municipal council for a smaller urban
such committee, they can only recommend area.
establishing a public park. The final decision (c) A municipal corporation for a larger
would rest with the Ward Councillors. So, urban area.
option (c) is also incorrect. Q.1269 Consider the following statements with
Learning: The city is divided into different respect to township:
wards and ward councillors get elected. Assertion (A): The laws of the state apply
The complicated decisions that affect the with some modifications and restrictions on
entire city are taken by groups of councillors these bodies.
who form committees to decide and debate Reason (R): A township form of urban
issues. For example, if bus stands need to be government has no elected members.
improved, or a crowded market-place needs In the context of the statements mentioned
to have its garbage cleared more regularly, or above, which one of these is true?
there is a drain that cuts through the city that (a) Both A and R is true and R is the correct
needs cleaning etc. It is these committees for explanation of A.
water, garbage collection, street lighting etc. (b) Both A and R is true but R is not a correct
that decide on the work to be done. explanation of A.
When the problems are within a ward then (c)  A is true, but R is false.
the people who live in the ward can contact (d)  R is true but A is false
their councillors. For example, if there are Solution: (d)
dangerous electrical wires hanging down The laws of the state do apply to township
then the local Councillor can help them get in in every way possible. These townships
touch with the electricity authority. however have their own set of rules and
While the Councillor’s Committees regulations conforming to the laws of the
and the councillors decide on issues, the state.
Commissioner and the administrative staff
Q.1270 Consider the following statements about the
implement these. The Commissioner and
composition and functioning of Municipal
the administrative staff are appointed.
Corporations (MCs).
Councillors are elected.
1. The composition of the MCs is governed
Q.1268 As per the 74th Constitutional Amendment solely by State laws.
Act, a ‘Nagar Panchayat’ is a term used for 2. The Municipal Commissioner heads the
(a) a village panchayat that has been recently council in the MCs.
given the status of a municipality and 3. All decisions in the MCs are subject to the
the administrative structure of which approval of the Municipal Commissioner.
resembles that of a Municipality Choose the correct answer using the codes
(b) an area in transition from a rural area to below.
an urban area (a)  1 only (b)  2 and 3 only
(c) a conglomeration of all panchayats that (c)  1 and 3 only (d)  None of the above
comes under a common Metropolitan Solution: (d)
Planning Committee

Indian Polity Question Bank P.353

04-Indian Polity_Q1070-1506.indd 353 8/7/2018 7:42:40 PM


Municipal corporations are created for It is set up under the provisions of the
the administration of big cities like Delhi, Cantonments Act of 2006—a legislation
Mumbai, Kolkata, Hyderabad, Bangalore and enacted by the Central government.
others. They are established in the states by Statement 2: It works under the
the acts of the concerned state legislatures, administrative control of the defence ministry
and in the union territories by the acts of of the Central government.
the Parliament of India. There may be one Thus, unlike some other types of
common act for all the municipal corporations urban local bodies, which are created and
in a state or a separate act for each municipal administered by the state government, a
corporation. A municipal corporation has three cantonment board is created as well as
authorities, namely, the council, the standing administered by the Central government.
committees and the commissioner. Statement 3: The executive officer
The Council is the deliberative and implements all the resolutions and decisions
legislative wing of the corporation. It consists of the board and its committees. He belongs to
of the Councillors directly elected by the the central cadre established for the purpose.
people, as well as a few nominated persons
having knowledge or experience of municipal (42) RURAL & DISTRICT
administration. In brief, the composition of
the Council including the reservation of seats
ADMINISTRATION IN INDIA
for SCs, STs and women is governed by the Q.1272 Which of the following functions may be
74th Constitutional Amendment Act. performed by a Panchayat Secretary?
The Council is headed by a Mayor. He is 1. Keeping a record of all official socio-
assisted by a Deputy Mayor. He is elected in economic statistics of the village
majority of the states for a one-year renewable 2. Keeping a record of the proceedings of
term. He is basically an ornamental figure Gram Sabha and Panchayat
and a formal head of the corporation. His 3. Issuing residentship certificates to the
main function is to preside over the meetings villagers
of the Council. 4. Managing the office of the Panchayat
Q.1271 Consider the following about cantonment 5. Preparation of budget and Annual
boards which are part of municipal Administration Report of the Panchayat
administration in India. 6. To execute the resolution passed by the
1. A cantonment board is established for Panchayat body
municipal administration of defence Select the correct answer using the codes
personnel residing in civilian areas. below.
2. A cantonment board is created by the (a)  1, 2, 3 and 4 only
State Government but administered by (b)  2, 4, 5 and 6 only
the Central government on grounds of (c)  1, 3, 5 and 6 only
national security. (d)  All of the above
3. The executive officer of the cantonment Solution: (b)
board is appointed by the President of Explaination and Learning: For e.g. in
India. Andaman & Nicobar, the following functions
Select the correct answer using the codes have been assigned to Panchayat Secretary
below. (he may be given additional or fewer
(a)  1 and 2 only (b)  2 and 3 only responsibilities in other states/UTs as PRIs is
(c)  3 only (d)  None of the above a state subject):
Solution: (c) The duties/functions of the Panchayat
Justification and Learning: Statement 1: A Secretary:
cantonment board is established for municipal (a) He will be in charge of the office of the
administration for civilian population in the Panchayat.
cantonment area. (b) He will work under the direct supervision

P.354 For Civil Services Preliminary Examination

04-Indian Polity_Q1070-1506.indd 354 8/7/2018 7:42:41 PM


of the Pradhan of the Gram Panchayat He provides all this information to the
and shall be responsible to the Panchayat government thus keeping it up to date. He
in all matters. is supervised by senior bureaucrats in the
(c) He will be responsible for all office work revenue department. He is also under the
like Preparation of budget and Annual control of District collector.
Administration Report; keeping all Q.1275 Measuring land and keeping land records of a
records of the Panchayat in safe custody village is the main work of
and producing of all records to the (a)  Gram Sevak
authorities as and when required. (b)  Block Development Officer
(d) Allotment of duties to the staff posted in (c) Patwari
the Gram Panchayat. (d)  Panchayat Samiti
(e) Submission of application for grant-in- Solution: (c)
aid. Explanation: The gram sevak is appointed
(f) Maintenance of grant-in-aid register and by the government and is responsible
preparation of utilization certificates. for the administrative functioning of the
(g) Inspection of worksites for assessing the panchayat in the village. She assists the
works in progress and to submit reports development officers and panchayat in
to the authority. proper administration. He does not maintain
(h)  Attend to complaints relating to or measure or land records. So, option (a) is
developmental works in progress and to incorrect.
submit reports thereof. The Block Development Officer (BDO)
(i) Visit the sites where works are undertaken is at a relatively senior hierarchy in a
by individuals who are granted loan by District. He supervises and implements
the Panchayat and report to the competent major development programmes. While he
authority if the loans are misused and to has access to all land records, he does not
cause recovery of loss thereof. measure and keep them in his custody. So,
Q.1273 Consider the following about the Panchayat the option (b) is also incorrect.
Secretary. Panchayat Samiti is a block level
1. She is elected by the Gram Panchayat organization and comes in the second tier
from amongst its members. structure of PRIs. It oversees issues with
2. She is responsible for calling the meeting Gram Panchayats. It does not measure, keep
of the Gram Sabha and Gram Panchayat or maintain land records. So, option (d) is also
and keeping a record of the proceedings. wrong.
Which of the above is/are correct? Learning: The Patwari is known by
(a)  1 only (b)  2 only different names in different states – in some
(c)  Both 1 and 2 (d)  None villages such officers are called Lekhpal, in
Solution: (b) others Kanungo or Karamchari or Village
Q.1274 In rural administration, which of the Officer etc.
following is/are usually the functions of Each Patwari is responsible for a group of
lekhpal/patwari/kanungo? villages. The Patwari maintains and updates
1. Keeps record of the land in the village the records of the village. Measuring land
2. Keeps record of the crops grown in a and keeping land records is his main work.
particular area The Patwari is also responsible for
3. Collects land revenue from farmers. organising the collection of land revenue from
Which of the above statements is/are true? the farmers and providing information to the
(a)  1 and 2 (b)  2 and 3 government about the crops grown in this area.
(c)  1 and 3 (d)  All of the above Q.1276 In rural administration, which of the
Solution: (d) following functions usually are performed by
the Tehsildar?

Indian Polity Question Bank P.355

04-Indian Polity_Q1070-1506.indd 355 8/7/2018 7:42:41 PM


1. He supervises patwari/kanungo/lekhpal’s (43) COOPERATIVES – PART IX-B –
work.
2. He hears land disputes.
ARTICLES 243ZH TO 243ZT
3. He is the authority issuing land record and Q.1278 The 97th Amendment Act of 2011 added
caste certificates. a new Directive Principle relating to
Which of the above statements is/are true? co-operative societies in the Constitution.
(a)  1 and 2 (b)  2 and 3 The amendment is concerned with which
(c)  1 and 3 (d)  All of the above of the following aspects of cooperative
Solution: (a) societies?
The District collector or the BDO issues such 1. Autonomous functioning
important certificates. The only duty of the 2. Democratic control
Tehsildar is to make sure that these certificates 3. Professional management
are disbursed to the needy properly. Choose the correct answer using the codes
Q.1277 Which of the following statements is/are below.
correct? (a)  1 and 2 only (b)  2 and 3 only
1. District Collector usually supervises the (c)  1 and 3 only (d)  All of the above
revenue department and land acquisition Solution: (d)
in the district. The Constitution (Ninety Seventh
2. District Commissioner of Police usually Amendment) Act 2011 relating to the
grants licences for arms and explosives in co-operatives is aimed to encourage
the district. economic activities of cooperatives which
Which of the above is/are correct? in turn help progress of rural India. It is
(a)  1 only (b)  2 only expected to not only ensure autonomous and
(c)  Both 1 and 2 (d)  None democratic functioning of cooperatives, but
Solution: (c) also the accountability of the management to
Justification: Statement 1: District the members and other stakeholders.
Collectors are entrusted with a wide range of • It makes Right to form cooperatives as a
duties in the jurisdiction of the district. While fundamental right.
the actual extent of the responsibilities varies • Reservation of one seat for SC/ST and
in each State, they generally involve two seats for women on the board of every
As Collector: land assessment; land co-operative society.
acquisition; collection of land revenue; • Cooperatives could set up agency which
collection of income tax dues, excise would oversee election.
duties, irrigation dues etc.; distribution of • Uniformity in the tenure of Cooperative
agricultural loans; Chairman of the District Board of Directors.
Bankers Coordination Committee etc. • Provisions for incorporation, regulation
As District Magistrate: maintenance of and winding up of co-operative societies
law and order; supervision of the police and based on the principles of democratic
jails etc. process and specifying the maximum
Statement 2: The application for the grant number of directors as twenty-one.
of Arm License is submitted in the office of • Providing for a fixed term of five years
Deputy Commissioner of Police (DCP) by from the date of election in respect of
the applicant or through his representative on the elected members of the board and its
the prescribed Performa. office bearers;
Where the licensing authority deems it • Providing for a maximum time limit of six
necessary for the security of the public, peace months during which a board of directors
or for public safety it may refuse to grant of co-operative society could be kept
such license. under suspension;
• Providing for independent professional
audit;

P.356 For Civil Services Preliminary Examination

04-Indian Polity_Q1070-1506.indd 356 8/7/2018 7:42:41 PM


• Providing for right of information to the Q.1280 The term “Cooperatives” or “Cooperative
members of the co-operative societies; Societies” can be found in which part of the
• Empowering the State Governments to Constitution?
obtain periodic reports of activities and (a)  Part III – Fundamental Rights
accounts of co-operative societies; which (b) Part IV – Directive Principles of State
have individuals as members from such Policy
categories; (c)  Part IVA – Fundamental Duties
• Providing for offences relating to co- (d)  Both (a) and (b)
operative societies and penalties in respect Solution: (d)
of such offences. Learning: The 97th constitutional
Q.1279 The 97th Constitutional Amendment Act amendment act gave a constitutional status
of 2011 gave a constitutional status and and protection to cooperative societies
protection to co-operative societies. In this and made the following changes in the
context, which of the following changes were constitution:
made in the constitution? • It made the right to form co-operative
1. Right to form co-operative societies societies a fundamental right (Article 19).
became a fundamental right • It included a new Directive Principle of
2. It included a new Directive Principle of State Policy on promotion of co-operative
State Policy on promotion of co-operative societies (Article 43-B).
societies • It added a new Part IX-B in the
3. It established a Central constitutional Constitution which is entitled “The Co-
authority for overseeing matters related to operative Societies”.
cooperatives. Q.1281 The right to form co-operative societies is a
Choose the correct answer using the codes (a) Directive Principle of State Policy
below. (b) Fundamental Right under Article 19 of
(a)  1 and 2 only (b)  2 and 3 only the Constitution
(c)  All of the above (d)  3 only (c)  Legal Right
Solution: (a) (d) A fundamental duty under Part IVA of the
The Constitution (Ninety Seventh Amendment) constitution
Act 2011 relating to the co-operatives is aimed to Solution: (b)
encourage economic activities of cooperatives Learning: The 97th constitutional
which in turn help progress of rural India. It is amendment act gave a constitutional status
expected to not only ensure autonomous and and protection to cooperative societies
democratic functioning of cooperatives, but and the right to form cooperative society a
also the accountability of the management to fundamental right.
the members and other stakeholders.
As per the amendment the changes done
to constitution are:-
• In Part III of the constitution, after words
“or unions” the words “Cooperative
Societies” was added.
• In Part IV a new Article 43B was inserted,
which says: The state shall endeavour
to promote voluntary formation,
autonomous functioning, democratic
control and professional management of
the co-operative societies”.
• After Part IXA of the constitution, a Part
IXB was inserted to accommodate state
versus centre roles.

Indian Polity Question Bank P.357

04-Indian Polity_Q1070-1506.indd 357 8/7/2018 7:42:41 PM


CONSTITUTIONAL BODIES Statement 1: Article 324 of the Constitution
has made the following provisions with regard
(44) ELECTION COMMISSION –PART to the composition of election commission:
1. The Election Commission shall consist of
XV—ARTICLE 324 • The chief election commissioner and
Q.1282 Consider the following statements about the • Such number of other election
Election Commission of India (ECI). commissioners, if any, as the president
1. Being an all-India body it serves both the may from time to time fix.
Central and State Governments. 2. The appointment of the chief election
2. It is not at all concerned with elections to commissioner and other election
any level of Panchayats and Municipalities. commissioners shall be made by the
3. The State election Commissions work president.
under the overall supervision of the ECI. 3. When any other election commissioner
Choose the correct answer using the codes is so appointed, the chief election
below. commissioner shall act as the chairman of
(a)  1 and 2 only (b)  2 and 3 only the election commission
(c)  1 and 3 only (d)  All of the above Therefore it’s composition is not mentioned
Solution: (a) in constitution. Election Commission of
The Constitution of India directly to ensure India can either be a single member or a
free and fair elections in the country. multi-member body. Till 1989, the Election
Article 324 of the Constitution provides Commission was single member. Just before
that the power of superintendence, direction the 1989 general elections, two Election
and control of elections to parliament, state Commissioners were appointed, making
legislatures, the office of president of India and the body multi-member. Soon after the
the office of vice-president of India shall be elections, the Commission reverted to its
vested in the election commission. Thus, the single member status. In 1993, two Election
Election Commission is an all-India body in Commissioners were once again appointed
the sense that it is common to both the Central and the Commission became multi-member
government and the state governments. It must and has remained multi-member since then.
be noted here that the election commission is Initially there were many apprehensions
not concerned with the elections to panchayats about a multi-member Commission. There
and municipalities in the states. For this, the was a sharp difference of opinion between
Constitution of India provides for a separate the then Chief Election Commissioner and
State Election Commission. the other Commissioners about who had how
much power. The matter had to be settled by
Composition the Supreme Court. Now there is a general
consensus that a multi-member Election
Q.1283 Consider the following statements about the Commission is more appropriate as power is
Election Commission of India. shared and there is greater accountability.
1. Its composition is mentioned in the Statement 2: The Chief Election
constitution. Commissioner (CEC) presides over the
2. It conducts local body elections also. Election Commission, but does not have
3. The Chief Election Commissioner does more powers than the other Election
not have more powers than the other Commissioners. The CEC and the two
Election Commissioners. Election Commissioners have equal powers
Choose the correct answer using the codes to take all decisions relating to elections as a
below: collective body
(a)  1 and 2 (b)  2 and 3 Statement 3: Article 324 of the Constitution
(c)  1 and 3 (d)  3 only provides that the power of superintendence,
Solution: (d) Direction and control of elections to

P.358 For Civil Services Preliminary Examination

04-Indian Polity_Q1070-1506.indd 358 8/7/2018 7:42:41 PM



Parliament, Justification: Statement 1: They are appointed

State legislatures, by the President.

The office of president of India and Statement 2: The condition of service is

The office of vice-president of India shall also determined by the President of India.
be vested in the election commission. The service conditions of the chief election
So election commission is not concerned commissioner cannot be varied to his
with the election bodies. It’s the responsibility disadvantage after his appointment.
of state election commission appointed by Statement 3: The CEC and the two other
the governor. election commissioners have equal powers
Q.1284 Consider the following statements. and receive equal salary, allowances and other
Assertion (A): The Election Commission perquisites. In case of difference of opinion
of India (ECI) can cancel local body elections amongst the Chief Election Commissioner
in case of rigging and both capturing. and/or two other election commissioners,
Reason (R): The ECI is responsible for the the matter is decided by the Commission by
conduct of local body elections. majority.
In the context of the above, which of these Statement 4: The chief election
is correct? commissioner is provided with the security
(a) A is correct, and R is an appropriate of tenure. He cannot be removed from his
explanation of A. office except in same manner and on the
(b) A is correct, but R is not an appropriate same grounds as a judge of the Supreme
explanation of A. Court. Others can be removed only when s/
(c) A is correct, but R is incorrect. he recommends.
(d) Both A and R are incorrect. Q.1286 In case of difference of opinion amongst the
Solution: (d) Chief Election Commissioner and/or two
Explained in previous questions. other election commissioners, how is the
Q.1285 Consider the following statements with matter decided?
reference to the Election Commission of (a) It is decided by majority in the
India. Commission.
1. The appointment of the chief election (b) The Chief Election Commissioner’s
commissioner and other election decision prevails over other
commissioners is made by the Union commissioners.
Minister of Parliamentary Affairs. (c) The matter is referred to the Parliamentary
2. The conditions of service and tenure of Standing Committee on Subordinate
office of the election commissioners and Legislation – the decision of which is
the regional commissioners are determined final and binding on ECI.
by the Chief Election Commissioner. (d) If two or more Election Commissioners
3. In case of difference of opinion amongst disagree on a matter, then their
the Chief Election Commissioner disagreement will prevail.
(CEC)  and other election commissioners, Solution: (a)
the view of the CEC prevails. The Election Commission has been
4. Any election commissioner or a regional functioning as a multi-member body
commissioner cannot be removed from consisting of three election commissioners. It
office except on the recommendation of can be more too.
the CEC. The chief election commissioner and the
Select the correct answer using the codes two other election commissioners have equal
below. powers and receive equal salary, allowances
(a)  1 and 4 only (b)  2 and 3 only and other perquisites, which are similar to
(c)  4 only (d)  3 and 4 only those of a judge of the Supreme Court. In
Solution: (c) case of difference of opinion amongst the
Chief Election Commissioner and/or two

Indian Polity Question Bank P.359

04-Indian Polity_Q1070-1506.indd 359 8/7/2018 7:42:41 PM


other election commissioners, the matter 3. The Chief Election Commissioner cannot
is decided by the Commission by majority. be removed by the Central cabinet.
They hold office for a term of six years Choose the correct answer using the codes
or until they attain the age of 65 years, below:
whichever is earlier. They can resign at any (a) 1 and 2 (b) 2 and 3
time or can also be removed before the expiry (c) 1 and 3 (d)  All of the above
of their term. Solution: (d)
Q.1287 In case of difference of opinion amongst the The Election Commission of India is an
Chief Election Commissioner and/or two autonomous, constitutionally established
other election commissioners, the matter is federal authority responsible for
decided by administering all the electoral processes in the
(a) President of India Republic of India. Under the supervision of
(b) Majority within the Commission the commission, free and fair elections have
(c) Chief Election Commissioner solely been held in India at regular intervals as per
(d) The matter reaches deadlock and no the principles enshrined in the Constitution.
further decision can be taken. The Election Commission has the power of
Solution: (b) superintendence, direction and control of
Explained in previous question. all elections to the Parliament of India and
the state legislatures and of elections to the
Q.1288 In order to ensure free and fair elections the
office of the President of India and the Vice-
constitution of Indian has provide for an
President of India.
independent Election Commission. Which of
The power of superintendence, direction
the following statements is/are correct about
and control of all elections to the Local
Election Commission?
Government/Municipal Corporation by the
1. The Election Commission of India can
State Election Commission.
either be a single member or a multi-
The Chief Election Commissioner and the
member body.
two Election Commissioners draw salaries
2. The Chief Election Commissioner
and allowances at par with those of the
(CEC)  presides over the Election
Judges of the Supreme Court of India as per
Commission and has more powers than
the Chief Election Commissioner and other
the other Election Commissioners.
Election Commissioners (Conditions of
3. The Chief and the other Election
Service) Rules, 1992.
Commissioners are appointed by the
President on the recommendation of the
Prime Minister.
Independence
4. The removal of the Chief Election Q.1290 Article 324 of the Constitution has made
commissioner requires absolute majority which of the following provisions to
Select the correct answer using the codes safeguard and ensure the independent
below: and impartial functioning of the Election
(a) 1 Only (b) 1 and 2 Only Commission?
(c)  1, 2 and 3 Only (d)  1, 2 and 4 Only 1. The election commissioners are provided
Solution: (a) with the security of tenure because
Explained in previous questions their term is fixed as provided in the
Q.1289 Consider the following statements about the constitution.
Election Commission of India. 2. The Constitution has prescribed the
1. It is a constitutional body. qualifications for appointment to the
2. Even though it is independent of office of the Election Commission.
the Government, the Chief Election Which of the above is/are correct?
Commissioner is appointed by the President (a)  1 only (b)  2 only
on the advice of the Central cabinet. (c)  Both 1 and 2 (d)  None
Solution: (d)

P.360 For Civil Services Preliminary Examination

04-Indian Polity_Q1070-1506.indd 360 8/7/2018 7:42:41 PM


Justification and Learning: The chief election removed from his office except in same
commissioner is provided with the security of manner and on the same grounds as that of
tenure. He cannot be removed from his office (a) A Supreme Court Judge
except in same manner and on the same grounds (b) UPSC Chairman
as a judge of the Supreme Court. (c) Comptroller and Auditor General (CAG)
In other words Chief Election of India
Commissioner can be removed from his (d) Either (a) or (c)
office by Parliament with special majority in Solution: (d)
Lok Sabha and Rajya Sabha on the grounds He cannot be removed from his office except
of proven misbehaviour or incapacity. Other in same manner and on the same grounds
Election Commissioners can be removed by as a judge of the Supreme Court. In other
the President on the recommendation of the words, he can be removed by the president
Chief Election Commissioner. on the basis of a resolution passed to that
Though the constitution has sought to effect by both the Houses of Parliament
safeguard and ensure the independence and with special majority, either on the ground
impartiality of the Election Commission, of proved misbehaviour or incapacity. Thus,
some flaws with respect to the members of he does not hold his office till the pleasure
the EC can be noted, viz., of the president, though he is appointed by
1. The Constitution has not prescribed him. The service conditions of the chief
the qualifications (legal, educational, election commissioner cannot be varied to
administrative or judicial) of the members his disadvantage after his appointment.
of the Election Commission. Q.1293 Consider the following about the Election
2. The Constitution has not specified the Commission of India (ECI).
term of the members of the Election 1. The Chief Election Commissioner (CEC)
Commission (the term of the Chief EC has is appointed by the President of India.
been prescribed though). 2. It enjoys the same kind of independence
3. The Constitution has not debarred the that the judiciary enjoys.
retiring election commissioners from any 3. Once appointed, the Chief Election
further appointment by the government. Commissioner is fully answerable to
Q.1291 Consider the following statements. the government for its powers and
Assertion (A): The Chief Election responsibilities.
Commissioner and other election Select the correct answer using the codes
commissioners enjoy security of tenure. below.
Reason (R): Their terms are specified and (a)  1 only (b)  2 and 3 only
fixed by the Constitution of India. (c)  1 and 3 only (d)  1 and 2 only
In the context of the statements above, Solution: (d)
which of these is true? Explanation: The ECI is an independent
(a) A and R both are true, and R is the correct body. It is not answerable to the government.
explanation for A. Answerability on day to day matters and
(b) A and R both are true, and R is the NOT even major policy decisions can tend to
the correct explanation for A. erode autonomy. Its autonomy is central to
(c) A is correct, R is incorrect. the Indian democracy. Hence, statement 3 is
(d) A is incorrect, R is correct incorrect.
Solution: (c) The election commissioners usually
Only CEC is provided with security of enjoy security of tenure, salary charged on
tenure. Other election commissioners can be Consolidated Fund of India, independence
removed by the president on recommendation from government interference, difficult
of CEC. process of impeachment etc. Hence, they
Q.1292 The chief election commissioner is provided enjoy similar independence as judges do.
with the security of tenure. He cannot be

Indian Polity Question Bank P.361

04-Indian Polity_Q1070-1506.indd 361 8/7/2018 7:42:41 PM


Q.1294 Though the constitution has sought to Choose the correct answer using the codes
safeguard and ensure the independence and below:
impartiality of the Election Commission of (a) 1 and 2 (b) 2 and 3
India (ECI), which of the following has not (c) None of the above (d) Only 2
been specified explicitly by the Constitution? Solution: (d)
1. Qualification of the Election The President appoints him after consulting
Commissioners the cabinet only. There has been a demand to
2. Term of the members of the ECI change this system.
3. Manner of Appointment The other commissioners cannot be
Choose the correct answer using the codes removed without the recommendation of the
below. CEC.
(a)  1 and 2 only (b)  2 and 3 only
(c)  1 and 3 only (d)  All of the above Powers and Functions
Solution: (a)
Q.1297 Consider the following statements about the
Some flaws can be noted in the Constitution
powers of the Election Commission of India.
with regard to the ECI, viz. flaws in
1. It can cancel the elections in entire country
nomination process, non-application of mind
on the grounds that a fair and free election
in scrutiny, wrongful rejection of nomination
is not possible in the country.
paper, error in ballot paper printing including
2. It can stop the transfer of officials related
photo mismatch, non-deletion of mock poll
to election duty.
results etc.
3. It implements the model code of conduct
Q.1295 The constitution has sought to safeguard and for the political parties.
ensure the independence and impartiality of Choose the correct answer using the codes
the Election Commission of India. However, below:
which of the following has NOT been (a)  1 and 2 (b)  2 and 3
prescribed/specified by the Constitution? (c)  1 and 3 (d)  All of the above
1. Qualifications of the members of the Solution: (d)
Election Commission The Election Commission of India has a wide
2. Term of the members of the Election range of functions.
Commission 1. It supervises the preparation of up-to-
3. Functions of the Election Commission date voters‘ list. It makes every effort to
Select the correct answer using the codes ensure that the voters‘ list is free of errors
below. like nonexistence of names of registered
(a)  1 and 2 only (b)  2 only voters or existence of names of those non-
(c)  1 and 3 only (d)  1, 2 and 3 eligible or non-existent.
Solution: (a) 2. It also determines the timing of elections
Explained in previous question and prepares the election schedule. The
Q.1296 Consider the following statements about the election schedule includes the notification
Election Commission of India. Which of of elections, date from which nominations
these is/are correct? can be filed, last date for filing nominations,
1. The Commissioners are appointed by the last date of scrutiny, last date of withdrawal,
President of India on the recommendation date of polling and date of counting and
of a committee which includes the Prime declaration of results.
Minister, leaders of opposition, and the 3. During this entire process, the Election
Chief Justice of India. Commission has the power to take
2. All the election commissioners have equal decisions to ensure a free and fair poll.
powers. It can postpone or cancel the election
3. The process of removing all commissioners in the entire country or a specific State
from office is the same. or constituency on the grounds that the
atmosphere is vitiated and therefore, a

P.362 For Civil Services Preliminary Examination

04-Indian Polity_Q1070-1506.indd 362 8/7/2018 7:42:41 PM


free and fair election may not be possible. party is not used by any other political entity in
The Commission also implements a polls across India.
model code of conduct for parties and • Other registered but unrecognised political
candidates. It can order a re-poll in a parties have to choose from a pool of “free
specific constituency. It can also order symbols” announced by the commission
a recount of votes when it feels that the from time to time.
counting process has not been fully fair • Besides, these parties get land or
and just. buildings from the government to set
4. The Election Commission accords up their party offices. They can have
recognition to political parties and allots up to 40 ‘star campaigners’ during
symbols to each of them. electioneering. Others can have up to 20
5. The Election Commission has very ‘star campaigners’.
limited staff of its own. It conducts Q.1299 When on election duty, government officers
the elections with the help of the work under the control of
administrative machinery. However, (a) The concerned District Administration
once the election process has begun, (b)  State Planning Board
the commission has control over the (c)  Chief Secretary of the State
administration as far as election related (d)  Election Commission
work is concerned. During the election Solution: (d)
process, the administrative officers of Explanation: They work under the control
the State and central governments are of the Election Commission of India (or State
assigned election related duty and in this Election Commission as the case may be).
respect, the Election Commission has full Hence, option (d)
control over them. The EC can transfer Learning: It is very common for the
the officers, or stop their transfers; it can Election Commission to reprimand the
take action against them for failing to act government and administration for their
in a non-partisan manner. lapses. This is because of the independence
Q.1298 Consider the following statements about the vested in the commission and the election
Election Commission of India (ECI). officials. When election officials come to
1. The ECI is the only authority that can the opinion that polling was not fair in some
grant National Party or State Party status booths or even an entire constituency, they
to a political party in India. order a repoll. The ruling parties generally
2. Election Symbols (Reservation and comply with the orders of the ECI.
Allotment) Order is issued by the ECI and Q.1300 Which of the following powers can be wielded
can be modified by it. by the Election Commission of India?
Which of the above is/are correct? 1. The Commission can suppress the results
(a)  1 only (b)  2 only of opinion polls if it deems such an action
(c)  Both 1 and 2 (d)  None fit for the cause of democracy.
Solution: (c) 2. The Commission can advise for
In August 2016, the ECI had brought changes disqualification of members after the
in Paragraph 6C of the Election Symbols elections if it thinks they have violated
(Reservation and Allotment) Order, 1968 to certain guidelines.
review the status of political parties every 3. The Commission can suspend candidates
10 years, instead of present norm of five years. who fail to submit their election expense
• For instance, the status of a party (either accounts timely.
national or state party) that did not Select the correct answer using the codes
perform up to the required criteria in 2014 below.
election will be reviewed in 2024. (a)  1 and 2 only (b)  2 and 3 only
Learning: Recognition as a national or a State (c)  1 and 3 only (d)  1, 2 and 3
party ensures that the election symbol of that Solution: (d)

Indian Polity Question Bank P.363

04-Indian Polity_Q1070-1506.indd 363 8/7/2018 7:42:41 PM


Learning: In case, a candidate is found guilty The powers related to elections are
of corrupt practices during the elections, the • To act as a court for settling disputes
Supreme Court and High Courts consult the related to granting of recognition to
Commission. political parties and allotment of election
It can also cancel the polls in event of symbols to them.
rigging, booth capture etc. • To appoint officers for inquiring into
Also, the main functions of the ECI are disputes relating to electoral arrangements.
as follows: To determine the code of conduct to be
• The Election Commission of India is observed by the parties and the candidates
considered the custodian of free and fair at the time of elections.
elections. • To prepare a roster for publicity of the
• It issues the Model Code of Conduct policies of the political parties on radio
in every election for political parties and TV in times of elections.
and candidates so that the dignity of • To advise the president on matters relating
democracy is maintained. to the disqualifications of the members of
• It regulates political parties and registers Parliament.
them for being eligible to contest elections. • To advise the governor on matters relating
• It publishes the allowed limits of to the disqualifications of the members of
campaign expenditure per candidate to all state legislature.
the political parties, and also monitors the • To request the president or the governor
same. for requisitioning the staff necessary for
• The political parties must submit their conducting elections.
annual reports to the ECI for getting tax • To supervise the machinery of elections
benefit on contributions. throughout the country to ensure free and
• It ensures that all the political parties fair elections.
regularly submit their audited financial • To advise the president whether elections
reports. can be held in a state under president’s rule
Q.1301 Which of the following functions are NOT in order to extend the period of emergency
performed by the Election Commission of after one year.
India? Q.1302 Which of the following is/are among the
1. Determining the territorial areas of the functions of the Election Commission of
electoral constituencies India ?
2. Advising the president on matters relating 1. Conduct of election for the posts of the
to the disqualifications of the members of Speaker and the Deputy Speaker, Lok
Parliament. Sabha and the Deputy Chairman, Rajya
3. To cancel polls in the event of Violence at Sabha.
election booths 2. Conduct of election to the Corporations
Choose the correct answer using the codes and Municipalities.
below. 3. Deciding on all doubts and disputes
(a)  1 and 2 only arising out of the elections.
(b)  2 and 3 only Select the correct answer from the codes
(c)  1 only given below.
(d)  All are performed by the ECI. (a)  I and 2 (b)  I and 3
Solution: (d) (c)  2 and 3 (d)  None of these
To determine the territorial areas of the Solution: (d)
electoral constituencies throughout the The SC or the HC decides on election
country on the basis of the Delimitation disputes.
Commission Act of Parliament. Elections to local bodies are conducted by
the State Election Commission.

P.364 For Civil Services Preliminary Examination

04-Indian Polity_Q1070-1506.indd 364 8/7/2018 7:42:41 PM


Q.1303 The Election Commission of India does NOT Q.1306 Consider the following statements about the
conduct elections to the powers of the Election Commission of India:
(a) Office of the President and Vice- 1. It can cancel polls in the event of rigging
President of India or violence or other irregularities in the
(b)  Parliament of India polls.
(c)  State Legislative Assembly 2. It can punish political parties not
(d)  Panchayats and Municipalities following the model code of conduct by
Solution: (d) withdrawing their party symbol.
Q.1304 The powers and functions exercised by the 3. It advises the governor on matters relating
Election Commission of India include? to the disqualifications of the Member of
1. To notify the dates and schedules of Parliaments.
elections and to scrutinise nomination Which of these is/are correct? Choose the
papers correct answer using the codes below:
2. To act as a court for settling disputes (a)  1 and 2 (b)  2 and 3
related to granting of recognition to (c)  1 and 3 (d)  All of the above
political parties and allotment of election Solution: (d)
symbols to them Q.1307 Consider the following statements about the
3. To cancel polls in the event of rigging, power of the ECI.
booth capturing or other irregularities 1. ECI takes decisions on every aspect of
4. To grant political parties the status of conduct and control of elections from
national or state parties on the basis of the announcement of elections to the
their poll performance declaration of results.
Select the correct answer using the codes 2. During the election period, the ECI can
below. order the government to follow some
(a)  1 and 4 only (b)  2 and 3 only guidelines, to prevent use and misuse
(c)  1, 2 and 4 only (d)  1, 2, 3 and 4 of governmental power to enhance its
Solution: (d) chances to win elections, or to transfer
Q.1305 Consider the following statements some government officials.
with reference to functions of Election 3. When on election duty, government
Commission: officers work under the control of the EC
1. It determines the timing of elections and and not the government.
prepares the election schedule. 4. It can order re-polling in case of rigging of
2. It can postpone or cancel the election in elections.
the entire country or a specific State or Choose the correct statements from the codes
constituency on the grounds that a free below:
and fair election may not be possible. (a)  All of the above (b)  1 and 4 only
3. During the election process, the Election (c)  2 and 3 only (d)  1, 3 and 4 only
Commission can transfer or even stop the Solution: (a)
transfers of the administrative officers of
the State and central governments. Miscellaneous
4. The Election Commission implements Q.1308 Law Commission of India in its 255th report
model code of conduct for both parties on electoral reforms recently submitted
and candidates to the Government of Indian has pitched
Which of the statements is/are correct? for stronger Election Commission of India
(a) 1, 2 and 4 Only (ECI). Its recommendations include
(b) 1, 3 and 4 Only 1. Equal Constitutional protection to all
(c) 1, 2 and 3 Only members of ECI in removability from
(d) 1, 2, 3 and 4 Only Office
Solution: (d)

Indian Polity Question Bank P.365

04-Indian Polity_Q1070-1506.indd 365 8/7/2018 7:42:41 PM


2. Appointment of the Election Commissioners Learning: The Election Commission of
by a three member Selection panel instead India nominates or designates an Officer of
of the present system of appointment by the Government of the State/Union Territory
President as the Chief Electoral Officer in consultation
3. The elevation of an EC should be on the with that State Government/Union Territory
basis of seniority. Administration.
Choose the correct answer using the codes As per the Representation of the People
below. Act 1950 and the Representation of the
(a)  1 and 2 only (b)  2 and 3 only People Act, 1951, the Chief Electoral Officer
(c)  1 and 3 only (d)  All of the above’ of a State/ Union Territory is authorised
Solution: (d) to supervise the election work in the State/
Recommendations of 255th Report of Law Union Territory subject to the overall
Commission with respect to ECI superintendence, direction and control of the
1. Appointments of Chief Election Election Commission.
Commissioners (CEC) and all Election The Election Commission of India also
Commissioners (ECs) - It should be made nominates or designates an Officer of the
by the President in consultation with a three- State Government as the District Election
member collegium or selection committee. Officer in consultation with the State
2. The collegium or committee shall consist Government.
of the Prime Minister, the Leader of the
Opposition of the Lok Sabha (or the (45) U nion Public Service
leader of the largest opposition party in
the Lok Sabha) and the Chief Justice of
Commission (UPSC)
India (CJI). Q.1310 Which of the following authorities appoints
3. Seniority based elevation- The elevation of the members of the Union Public Service
an EC should be on the basis of seniority. Commission (UPSC)?
If senior EC is not appointed due to unfit (a) President of India on the advice of the
reasons, the three member collegium or a Council of Ministers
committee give reasons in writing. (b) Union Minister of Personnel and Training
4. Equal constitutional protection to all on the advice of Law Ministry
members of the ECI- Equal constitutional (c) Chairman, UPSC on the recommendations
protection must be given to all members of a committee of former members
of the ECI in matters of removability from (d) None of the above
office. Solution: (a)
5. In this regard, Article 324(5) of the Learning: The Union Public Service
Constitution should be amended. Commission consists of one chairman and
Q.1309 The Chief Electoral Officer (CEO) supervises 7-10 full-time members apart from one
the election work in a State. He is appointed secretary who usually belongs to the IAS.
by the The UPSC chairman does not sit in any
(a) Election Commission of India in board (interview) rather is responsible for
consultation with the State Government overall management and coordination of
(b) Governor of the State concerned the commission. Each UPSC member heads
in consultation with the Election a board which consists of guest members
Commission of India having unique subject knowledge or have a
(c) Chief Secretary of the State concerned distinguished career.
in consultation with the Chief Election Q.1311 Consider the following about the Union
Commissioner Public Service Commission (UPSC).
(d) State Election Commission after 1. The Constitution has not specified the
consulting Election Commission of India strength of the Commission and has left it
Solution: (a) to the discretion of the President.

P.366 For Civil Services Preliminary Examination

04-Indian Polity_Q1070-1506.indd 366 8/7/2018 7:42:41 PM


2. No educational qualifications are Choose the correct answer using the codes
prescribed for the Commission’s below.
membership. (a)  1 and 2 only (b)  2 and 3 only
3. The members of UPSC can be removed (c)  1 and 3 only (d)  All of the above
only on the grounds and in the manner of Solution: (d)
removal of Supreme Court judges. Refer previous explanation.
Select the correct answer using the codes Q.1313 Consider the following about the Office of
below. the UPSC Chairman.
(a)  1 and 2 only (b)  2 and 3 only 1. She is appointed by the Department of
(c)  1 and 3 only (d)  1, 2 and 3 Personnel and Training (DoPT) on the
Solution: (a) recommendation of UPSC members.
Justification: Statement 1: The UPSC 2. The Chairman must serve a fixed term
consists of a chairman and other members post-appointment even if retirement is
appointed by the president of India. The due.
Constitution, without specifying the strength 3. She can be removed by a resolution
of the Commission has left the matter to the passed by a majority of UPSC members
discretion of the president, who determines and agreed to by the Union Cabinet.
its composition. Usually, the Commission 4. A UPSC member can be re-appointed as
consists of nine to eleven members including the Chairman.
the chairman. Select the correct answer using the codes
Statement 2: No qualifications are below.
prescribed for the Commission’s membership (a)  1 and 2 only (b)  4 only
except that one-half of the members of the (c)  2, 3 and 4 only (d)  None of the above
Commission should be such persons who Solution: (b)
have held office for at least ten years either Justification: Statement 1: She is appointed
under the Government of India or under by the President on the recommendation of
the government of a state. The Constitution the Union Council of Ministers.
also authorises the president to determine Statement 2: The chairman and members
the conditions of service of the chairman of the Commission hold office for a term if
and other members of the Commission. The six years or until they attain the age of 65
chairman and members of the Commission years, whichever is earlier.
hold office for a term of six years or until Statement 2: He can be removed by the
they attain the age of 65 years, whichever is President based on a Supreme Court inquiry.
earlier. Statement 4: UPSC member can be
Statement 3: The President can remove appointed as UPSC Chairman.
the Chairman and members of UPSC on
Q.1314 Which of these persons have served or are
grounds of insolvency etc. On the ground
serving as Chairman of Union Public Service
of misbehaviour, he can remove them only
Commission (UPSC)?
based on an inquiry by the SC.
1. Akhlaq Ur Rahman Kidwai
Q.1312 Consider the following statements about the 2. Purna Chandra Hota
Union Public Service Commission (UPSC). 3. Deepak Sarkar
1. The Chairman and other members are 4. David R. Syiemlieh
appointed by the President. Select the correct answer using the codes
2. The constitution does not mention below.
the qualification for the post of UPSC (a)  1, 2 and 4 only (b)  2 and 3 only
Chairman. (c)  1 and 4 only (d)  1, 2, 3 and 4
3. One-half of the members of the Solution: (a)
Commission should have worked under Justification: You can always expect such
either the Government of India or any questions from UPSC on UPSC.
state government.

Indian Polity Question Bank P.367

04-Indian Polity_Q1070-1506.indd 367 8/7/2018 7:42:41 PM


Q.1315 The President places UPSC’s annual A UPSC member can be removed by only a
report before the Parliament along with a SC enquiry. But a chief election commissioner
memorandum explaining the cases where the can be removed only in the same manner as
advice of the Commission was not accepted that of a supreme court judge.
and the reasons for such nonacceptance. Q.1318 Consider the following statements about the
All such cases of non-acceptance must be role of UPSC.
approved by the Assertion (A): It has to be consulted
(a)  Concerned Union Ministry in all promotion and disciplinary matters
(b)  Cabinet Secretary concerned with higher Public Services.
(c) Appointments Committee of the Union Reason (R): It is the Central personnel
cabinet agency in India.
(d) Union Department of Personnel and In the context of the statements above,
Training which of these is true?
Solution: (c) (a) A and R both are true, and R is the correct
Justification: An individual ministry or explanation for A.
department has no power to reject the advice (b) A and R both are true, and R is the NOT
of the UPSC. the correct explanation for A.
So, options (a) and (d) cannot be correct. (c) A is correct, R is incorrect.
All such cases of non-acceptance must be (d) A and R both are incorrect.
approved by the Appointments Committee of Solution: (d)
the Union cabinet, despite UPSC being only an The Constitution visualises the UPSC to be
advisory body under the Constitution of India. the ‘watch-dog of merit system’ in India. It
Q.1316 If the UPSC tenders an advice to the is concerned with the recruitment to the all-
Government of India, which of these can India services and Central services— group
follow? A and group B and advises the government,
(a) Only the Department of Personnel and when consulted, on promotion and
Training (DoPT) can reject the advice of disciplinary matters. It is not concerned with
UPSC on reasonable grounds. the classification of services, pay and service
(b) Only the Appointments Committee of conditions, cadre management, training,
the Cabinet can reject UPSC’s advice. and so on. These matters are handled by the
(c) Any concerned Ministry can reject Department of Personnel and Training—one
UPSC’s advice. of the three departments of the Ministry of
(d) For UPSC’s advice to be rejected, Personnel, Public Grievances and Pensions.
approval from the Departmental Standing Therefore, UPSC is only a central recruiting
Committees of the Parliament is needed. agency while the Department of Personnel
Solution: (b) and Training is the central personnel agency
Q.1317 Consider the following statements about the in India. The role of UPSC is not only limited,
Union Public Service Commission (UPSC): but also recommendations made by it are only
1. Its decisions are not binding on the of advisory nature and hence, not binding
government. on the government. It is up to the Union
2. A UPSC member can be removed in the government to accept or reject that advise.
same manner as that of the central election The only safeguard is the answerability
commissioners. of the government to the Parliament for
3. An individual ministry has no power to departing from the recommendation of the
reject the advice of the UPSC. Commission. Further, the government can
Which of these is/are correct? Choose the also make rules which regulate the scope of
correct answer using the codes below: the advisory functions of UPSC .
(a)  1 and 2 (b)  2 and 3 Q.1319 Consider the following statements about the
(c)  1 and 3 (d)  All of the above powers, function and role of the Union Public
Solution: (c) Service Commission (UPSC):

P.368 For Civil Services Preliminary Examination

04-Indian Polity_Q1070-1506.indd 368 8/7/2018 7:42:41 PM


1. UPSC not only selects the civil servants or a member. Under the provisions of the
but also plays an important role in their Constitution, the advice tendered by the
training. Supreme Court in this regard is binding on
2. There is an overlap in the jurisdiction the president.
of UPSC and Central Vigilance However, during the course of enquiry by
Commissioner (CVC), when it comes to the Supreme Court, the governor can suspend
disciplinary action against a civil servant. the concerned chairman or member, pending
3. It is not necessary for the President to the final removal order of the president on
consult the UPSC always while making receipt of the report of the Supreme Court.
appointment to central services or all- Q.1321 Consider the following statements about the
Indian services. State Public Service Commission (SPSC):
Which of these is/are correct? Choose the 1. A member of a SPSC is appointed by the
correct answer using the codes below: Governor of that state.
(a) 1 and 2 (b) 2 and 3 2. A member of a SPSC cannot be removed
(c)  1 and 3 (d)  All of the above by the Governor of that particular state.
Solution: (b) 3. A member of a SPSC can be removed
The UPSC is the central recruiting agency. on the same grounds and with the same
It only recommends the candidates. The procedure as that of the UPSC.
cadre allocation, determining their service Which of these is/are correct? Choose the
condition is done by the DoPT. correct answer using the codes below:
(a) 1 and 2 (b) 2 and 3
(46) STATE PUBLIC SERVICE (c) 1 and 3 (d)  All of the above
COMMISSIONS Solution: (d)
Explained as above in previous question.
Q.1320 Consider the following about the State Public
Service Commissions. (47) TRIBUNALS – PART XIVA –
1. The Chairman is appointed by the
President but can be removed both by the
ARTICLES 323A AND 323B
President and Governor. Q.1322 Which of the following statements is
2. The advice tendered by the Supreme INCORRECT concerning tribunals in India?
Court, as regards the removal of the (a) Only the Central government is authorized
Chairman, is binding on the concerned by the Constitution to establish tribunals.
authority. (b) Tribunals can be established for settling
Which of the above is/are correct? disputes other than administrative
(a)  1 only (b)  2 only disputes.
(c)  Both 1 and 2 (d)  None (c) Appeal from the tribunals lie to both the
Solution: (b) High Courts and Supreme Court.
Justification: Statement 1: He and the (d) Central Administrative Tribunal in India
members of SPSC can be removed only by exercises original jurisdiction in relation
the President and not the Governor. This acts to service matters of Central government
as a mechanism to ensure the independence employees.
of the SPSC. Solution: (a)
Statement 2: The president can also Article 323 A empowers the Parliament
remove the chairman or any other member to provide for the establishment of
of SPSC for misbehaviour. In this case, administrative tribunals for the adjudication
the president has to refer the matter to the of disputes relating to recruitment and
Supreme Court for an enquiry. conditions of service of persons appointed to
If the Supreme Court, after the enquiry, public services of the Centre, the states, local
upholds the cause of removal and advises bodies, public corporations and other public
so, the president can remove the chairman authorities.

Indian Polity Question Bank P.369

04-Indian Polity_Q1070-1506.indd 369 8/7/2018 7:42:41 PM


Under Article 323 B, the Parliament Q.1325 Article 323 A empowers the Parliament
and the state legislatures are authorised to to provide for the establishment of
provide for the establishment of tribunals for administrative tribunals for the adjudication
the adjudication of disputes relating to the of disputes relating to recruitment and
following matters: conditions of service of persons appointed to
1. Taxation; public services of the
2. Foreign exchange, import and export 1. Other Public Authorities
3. Industrial and labour; 2. State
4. Land reforms; 3. Local Bodies
5. Ceiling on urban property; 4. Public Corporations
6. Elections to Parliament and state Choose the correct answer using the codes
legislatures; below.
7. Food stuffs; (a)  All of the above (b)  2 only
8. Rent and tenancy rights. (c)  2, 3 and 4 only (d)  3 and 4 only
Q.1323 Consider the following about the provision Solution: (a)
of tribunals as under Article 323 of the The Centre, the states, local bodies, public
Constitution of India. corporations and other public authorities
Assertion (A): Only the Parliament is can have the services of a dedicated tribunal
empowered to create a tribunal. according to article 323A
Reason (R): There cannot be any Centre- In pursuance of Article 323 A, the
state hierarchy of tribunals as exists in the Parliament has passed the Administrative
case of Supreme Court and High Courts. Tribunals Act in 1985. The act authorises the
In the context of the above, which of these Central government to establish one Central
is correct? administrative
(a) A is correct, and R is an appropriate tribunal and the state administrative
explanation of A. tribunals. This act opened a new chapter in the
(b) A is correct, but R is not an appropriate sphere of providing speedy and inexpensive
explanation of A. justice to the aggrieved public servants.
(c) A is incorrect, but R is correct. Q.1326 With reference to the Central Administrative
(d) Both A and R are incorrect. Tribunal (CAT), consider the following:
Solution: (d) 1. It was established in pursuance of the
Justification Under Article 323A, only one provisions of the Constitution.
tribunal for the Centre and one for each state 2. It adjudicates disputes with respect to
or two or more states may be established. recruitment and conditions of service of
There is no question of hierarchy of tribunals persons appointed to public services.
under Article 323A, whereas under Article 3. A person who has been a former Secretary
323B a hierarchy of tribunals may be created. in the Government of India only can head
Q.1324 As per the Constitution, which of these the Central Administrative Tribunal.
authorities is competent enough to create a 4. It is bound by the procedure prescribed in
Tribunal? the code of civil procedure.
(a) President of India only, after obtaining Select the correct answer using the codes
the due advice of the Council of Ministers below.
(b) Parliament only (a)  1 and 2 only (b)  2 and 3 only
(c) Only Parliament and State Legislatures (c)  1, 2 and 4 only (d)  3 and 4 only
(d) Only Supreme Court and High Courts, in Solution: (a)
special circumstances Justification: Statement 1: Article 323-A
Solution: (c) provides for establishment of tribunals.
Explained in previous question. Statement 2: The CAT exercises original
jurisdiction in relation to recruitment and all

P.370 For Civil Services Preliminary Examination

04-Indian Polity_Q1070-1506.indd 370 8/7/2018 7:42:41 PM


service matters of public servants covered 1. The Chairman and members are appointed
by it. Its jurisdiction extends to the all-India by a collegiums consisting of senior
services, the Central civil services, civil posts Supreme court judges headed by the Chief
under the Centre and civilian employees of Justice of India.
defence services. 2. The members are drawn from both judicial
Statement 3: Someone who has been a and administrative stream.
sitting or serving high court judge can chair 3. CAT orders and decrees are final, and no
the CAT. provision of appeal lies whatsoever.
Statement 4: The CAT is not bound by the Choose the correct answer using the codes
procedure laid down in the Civil Procedure below.
Code of 1908. It is guided by the principles (a)  1 and 2 only (b)  2 and 3 only
of natural justice. These principles keep the (c)  2 only (d)  1 and 3 only
CAT flexible in approach. Solution: (c)
Q.1327 In the functioning of the Central The CAT is a multi-member body consisting
Administrative Tribunal (CAT), it is guided of a chairman and members. Earlier, the CAT
by and follows the consisted of a Chairman, Vice-Chairmen
(a)  Civil Procedure Code of 1908 and members. With the amendment in
(b)  Principles of natural justice Administrative Tribunals Act, 1985 in 2006,
(c)  Criminal Procedure Code the members have been given the status of
(d)  Indian Penal Code judges of High Courts.
Solution: (b) They are drawn from both judicial and
The CAT is not bound by the procedure laid administrative streams and are appointed by
down in the Civil Procedure Code of 1908. It the president. They hold office for a term of
is guided by the principles of natural justice. five years or until they attain the age of 65
These principles keep the CAT flexible in years, in case of chairman and 62 years in
approach. Only a nominal fee of `50 is to case of members, whichever is earlier.
be paid by the applicant. The applicant may Originally, appeals against the orders of
appear either in person or through a lawyer. the CAT could be made only in the Supreme
The Principles of Natural Justice have Court and not in the high courts. However, in
come out from the need of man to protect the Chandra Kumar case (1997), the Supreme
himself from the excesses of organized power Court declared this restriction on the jurisdict-
man has always appealed to someone beyond ion of the high courts as unconstitutional,
his own creation. Such someone is the God holding that judicial review is a part of the
and His laws, divine law or natural law, to basic structure of the Constitution.
which all temporal laws and actions must Q.1329 Consider the following with reference to the
confirm. Natural Law is of the ‘higher law Central Administrative Tribunal (CAT).
of nature’ or ‘natural law’ Two core points in 1. Chairman and members are appointed by
the concept of principles of natural justice the Chief Justice of India after obtaining
1. Nemo in propria causa judex, esse debet: the recommendation of a Supreme Court
No one should be made a judge in his own collegium.
case, or the rule against bias. 2. No appeal lies to any tribunal or court in
2. Audi alteram partem: Hear the other party, India to the awards given by the CAT.
or the rule of fair hearing, or the rule that 3. Members of CAT have been given the
no one should be condemned unheard. status of Supreme Court Judges.
These two are the basic pillars of the Principles Select the correct answer using the codes
of Natural Justice. No system of law can below.
survive without these two basic pillars. (a)  1 only (b)  2 and 3 only
Q.1328 Consider the following about the composition, (c)  1, 2 and 3 (d)  None of the above
powers and appointments related to the Solution: (d)
Central Administrative Tribunal (CAT). Refer previous answer

Indian Polity Question Bank P.371

04-Indian Polity_Q1070-1506.indd 371 8/7/2018 7:42:42 PM


Q.1330 Which of the following statements about the While it looks logical that the High courts
Central Administrative Tribunal (CAT) is should not intervene, as per a Supreme Court
INCORRECT? ruling in the Chndra Kumar case, they can.
(a) Chairman and members of the CAT enjoy The Court declares review of CAT decisions
the status of judges of High Courts as per by the HC as a part of the judicial review. It
statutory provisions. cannot be taken away.
(b) Its members must be qualified to be a judge Q.1333 Under Article 323 B, the Parliament and the
of a High court or have served as one. state legislatures are authorised to provide
(c) Members are appointed by the President. for the establishment of tribunals for the
(d) Appeals against the orders of the CAT adjudication of disputes relating to which of
can be made in both the Supreme Court the following matters?
and the High courts. 1. Food stuffs
Solution: (b) 2. Rent and tenancy rights
See as explained in previous answer. 3. Industrial labour
Q.1331 Consider the following statements about 4. Elections to State legislatures
Administrative tribunals in India: Choose the correct answer using the codes
1. Only a sitting or a retired judge can be the below.
chairman of the Central Administrative (a)  All of the above (b)  2 only
tribunal (CAT). (c)  2, 3 and 4 only (d)  3 and 4 only
2. Administrative tribunals are active in the Solution: (a)
Centre as well as the states. Refer first question in the chapter.
3. Appeals against the judgement of the CAT
can be made only before the Supreme (48) FINANCE COMMISSION – PART
Court, and not the High courts.
Choose the correct option from the codes
XII—ARTICLE 280
below: Q.1334 Article 280 of the Constitution of India
(a)  1 and 3 (b)  1 and 2 provides for a Finance Commission.
(c)  2 and 3 (d)  All of the above Which kind of a body is Finance
Solution: (b) Commission?
A person shall not be qualified for (a) Quasi-judicial (b) Executive
appointment as the Chairman unless he is, or (c) Administrative (d) Quasi-legislative
has been, a judge of a High Court. Appeal can Solution: (a)
be made first only in the High courts and then It is a quasi-judicial body as its work
the SC can be moved. involves fixing the fair share of states in the
Q.1332 Consider the following statements: overall grant from Centre. Since it uses both
Assertion (A): The decisions of the legal principles and administrative dictums in
Central Administrative Tribunal (CAT) making decisions, it is called a quasi-judicial
cannot be challenged in the High Courts. body.
Reason (R): The CAT deals with the It is constituted by the president of India
grievances of Central government employees every fifth year or at such earlier time as he
and All-India services mainly. considers necessary.
In the context of the statements above, The Finance Commission consists of
which of these is true? a chairman and four other members to be
(a) Both A and R are true and R is the correct appointed by the president. And since it
explanation for A. is a Quasi-judicial body, the Chairman is a
(b) Both A and R are true and R is not the person having experience in public affairs.
correct explanation for A. Other members are judges, administrators
(c)  A is incorrect but R is correct. or experts. So both administrative side and
(d)  Both A and R are incorrect. judicial sides are balanced in the Commission.
Solution: (d)

P.372 For Civil Services Preliminary Examination

04-Indian Polity_Q1070-1506.indd 372 8/7/2018 7:42:42 PM


Q.1335 Consider the following statements about the Statement 3: It is constituted by the president
Finance Commission: of India Every fifth year or at such earlier
1. It is a constitutional quasi-judicial body. time as he considers necessary. State don’t
2. The constitution does not prescribe any have any role in the process.
qualifications for the members of the Q.1336 Consider the following about Finance
commission. Commission.
3. It is constituted jointly by the Centre and 1. It is a body set up under Article 275(1) of
the states as the nature of the commission’s the Constitution.
work is essentially federal. 2. Members of National Statistical
Which of these is/are correct? Choose the Commission and NITI Aayog are ex-
correct answer using the codes below: officio members of the commission.
(a) 1 and 2 (b) 2 and 3 Select the correct answer using the codes
(c) 1 and 3 (d) Only 1 below.
Solution: (d) (a)  1 and 2 (b)  1 only
Statement 1: Finance commission is (c)  2 only (d)  None of the above
constituted under article 280 of Indian Solution: (d)
Constitution. Hence it’s constitutional body. Justification: Statement 1: Article 275(1)
The Finance Commission shall have all concerns grants-in-aid. While the FC deals
powers of civil court under Code of Civil with this article, it is a body set up under
Procedure (1908) in matters of summoning Article 280 of the Constitution.
& enforcing attendance and requisitioning Its primary job is to recommend measures
any public record from any court of office. and methods on how revenues need to be
Also Finance Commission shall be deemed distributed between the Centre and states.
to be a civil court for purposes of sections Statement 2: There are no ex officio
480 and 482 of the CrPC provided under members of the commission as such, because
Finance Commission Act. Hence it’s quasi- it is constituted afresh every five years.
judicial body. Learning: Besides suggesting the
Statement 2: It is constituted only by mechanism to share tax revenues, the
the President of India. The constitution the Commission also lays down the principles
Constitution authorises the Parliament to for giving out grant-in-aid to states and other
determine the qualifications of members local bodies.
of the commission and the manner in The commission has to take on itself
which they should be selected. the job of addressing the imbalances that
Accordingly, the Parliament has specified often arise between the taxation powers and
the qualifications of the chairman and expenditure responsibilities of the centre and
members of the states, respectively.
the commission.
Q.1337 The Finance Commission consists of a
• The chairman should be a person having
chairman and four other members to be
experience in public affairs and
appointed by the president. Who among the
• The four other members should be
following is/are eligible to become a member
selected from amongst the following:
of the Finance Commission?
1. A judge of high court or one qualified
1. A judge of high court
to be appointed as one.
2. A person who has special knowledge of
2. A person who has specialised
economics
knowledge of finance and accounts of
3. An eminent citizen who has worked as a
the government.
social activist
3. A person who has wide experience in
4. An academician who specializes in
Financial matters and Administration.
federalism and related issues
4. A person who has special knowledge
of economics.

Indian Polity Question Bank P.373

04-Indian Polity_Q1070-1506.indd 373 8/7/2018 7:42:42 PM


Select the correct answer using the codes the states of the respective shares of such
below. proceeds.
(a)  1 and 2 only (b)  2 and 4 only 2. The principles that should govern the
(c)  1 only (d)  2, 3 and 4 only grants-in-aid to the states by the Centre
Solution: (a) (i.e., out of the consolidated fund of
As explained in previous questions India).
Hence it can be seen as institutional
Functions mechanism for transferring resources from
the Centre to states.
Q.1338 Which of these bodies recommends the
Statement 3:1971 is taken as the population
distribution of the net proceeds of taxes to
base for all the policy purpose. It’s true even
be shared between the Centre and the states,
in case of 14th finance commission.
and the allocation between the states, the
Statement 4: Finance commission recom-
respective shares of such proceeds?
mendations only concerns centre not states.
(a)  Inter-State Council
(b)  NITI Aayog Q.1340 The Finance Commission is required to make
(c)  National Development Council recommendations to the president of India on
(d)  Finance Commission which of the following matters?
Solution: (d) 1. The distribution of the net proceeds of
taxes to be shared between the Centre and
Q.1339 With reference to Finance commissions in
the states
India, consider the following statements
2. Allocation of proceeds between the states
1. Because of their constitutional nature,
3. The principles that should govern the
their recommendations are binding of the
grants-in-aid to the states by the Centre
government
from the Public Accounts of India
2. Finance commissions are seen as
Select the correct answer using the codes
institutional mechanism for transferring
below.
resources from the Centre to states.
(a)  1 and 2 only (b)  2 and 3 only
3. The 14th Finance Commission has been
(c)  1 and 3 only (d)  1, 2 and 3
asked to take the base of population figures
Solution: (a)
as of 1971 in all cases where population is
Justification: In Statement 3, it should be
a factor for determination of devolution of
Consolidated Fund of India, and not Public
taxes and duties and grants-in-aid
accounts of India (PCI). PCI is operated by
4. The recommendations of Finance
executive action.
Commission applies only to the Union
Other recommendation giving areas of the
government
commission are:
Which of the above statements is/are correct?
• The measures needed to augment the
(a)  1,2 and 4 Only (b)  1 and 2 Only
consolidated fund of a stateto supplement
(c)  2,3 and 4 Only (d)  All
the resources of the Panchayats and the
Solution: (c)
municipalities in the state on the basis of
Statement 1: finance commission is a advisory
the recommendations made by the state
body. It’s recommendations are not binding on
finance commission
the union government. It is up to the Union
• Any other matter referred to it by the
government to implement its recommendations
president in the interests of sound finance.
on granting money to the states.
The commission submits its report to the
Statement 2: The Finance Commission
president. He lays it before both the Houses
is required to make recommendations to the
of Parliament along with an explanatory
president of India onthe following matters:
memorandum as to the action taken on its
1. The distribution of the net proceeds of
recommendations.
taxes to be shared between the Centre
and the states, and the allocation between

P.374 For Civil Services Preliminary Examination

04-Indian Polity_Q1070-1506.indd 374 8/7/2018 7:42:42 PM


Q.1341 The Finance Commission is required to make Those to be implemented by an order of
recommendations to the president of India in the President: The recommendations relating
which of the following matters? to distribution of Union Taxes and Duties and
1. Augmenting the resources of the Public Grants-in-aid fall in this category.
Sector Undertakings (PSUs) Those to be implemented by executive
2. Augmenting the resources of the states to orders: The recommendations in respect of
support local bodies sharing of Profit Petroleum, Debt Relief,
3. Principles that should govern the grants- Mode of Central Assistance, etc. are
in-aid to the states by the Centre implemented by executive orders.
Choose the correct answer using the codes
below. (49) STATE FINANCE COMMISSIONS
(a)  1 and 2 only (b)  2 and 3 only
(c)  1 and 3 only (d)  All of the above
(SFCS) – PART IX & IXA—
Solution: (b) ARTICLES 243I & 243Y
As explained in previous questions
Q.1344 Consider the following about State Finance
Q.1342 Consider the following statements. Commissions (SFCs).
Assertion (A): The recommendations 1. They are constitutional bodies.
made by the Finance Commission binding on 2. Their mandate is to recommend devolution
the government. of tax funds from the Consolidated Fund
Reason (R): Finance Commission is a of Union Government to local bodies.
constitutional body. 3. Their recommendations must be approved
In the context of the statements above, by the Union Finance Commission, in
which of these is true? order to be effective.
(a) A and R both are true, and R is the correct 4. They are established by the President
explanation for A. from time to time.
(b) A and R both are true, and R is the NOT Select the correct answer using the codes below.
the correct explanation for A. (a)  1, 2 and 4 only (b)  2 and 3 only
(c) A is incorrect, R is correct. (c)  1 only (d)  1, 3 and 4 only
(d) A and R both are incorrect. Solution: (c)
Solution: (c) Justification: Statements 1 and 2:  As per
It is nowhere laid down in the Constitution 73rd and 74th CA, under Part XI and XII and
that the recommendations of the commission Article 243(I) and 243 (Y) of the Constitution,
shall be binding upon the Government of a State Finance Commission (SFC), is to be
India or that it would give rise to a legal right appointed after every 5 years, to;
in favour of the beneficiary states to receive • Recommend devolution of tax funds
the money recommended to be offered to from the Consolidated Fund of State
them by the Commission. Government (not Union government,
Q.1343 How are the recommendations of Finance which makes statement 2 incorrect) to
Commissionimplemented? Local Bodies
(a)  An order of the President • Suggest measures for augmenting their
(b)  A legislation of the Parliament Own Resources by determining which
(c) A joint resolution of the Centre and States Taxes, Duties, Tolls and Fees which may
(d) They need not be implemented and take be assigned to or appropriated by PRIs
effect immediately on the publication of and ULBs
the recommendations by the commission. • The Grants-in-Aid to Local Bodies from
Solution: (a) the consolidated fund of the state
Learning: The recommendations of the • The measures needed to improve the
Finance Commission are implemented as financial position of the Local Bodies
under:- • Estimation of Revenue Gap of Local
Bodies, separately for PRIs and ULBs,

Indian Polity Question Bank P.375

04-Indian Polity_Q1070-1506.indd 375 8/7/2018 7:42:42 PM


Statement 3: The Union FC does not play (50) NATIONAL COMMISSION FOR
any role with respect to either constitution
or working of the SFCs. It may only help
SCHEDULED CASTES (SCs) – PART
guide the overall functioning of the SFCs XVI—ARTICLE 338
by specific recommendations (that are not
Q.1346 Consider the following statements about the
binding on the SFCs).
National Commission for SCs.
Statement 4: The Governor constitutes
1. The Chairman of the Commission is
them. President constitutes Union Finance
appointed by a Selection panel which
Commission (UFC).
includes he Prime Minister, Lok Sabha
Q.1345 Consider the following about the State Speaker and Leader of Opposition, Lok
Finance Commissions (SFCs). Sabha.
Assertion (A): It is constituted every five 2. The commission presents an annual report
years by the Union Finance Commission to the president which is presented to
(UFC) and later dissolved by it after the Parliament.
completion of the financial devolution 3. The orders of the commission are binding
exercise. on the Government.
Reason (R): There is a need to ensure Choose the correct answer using the codes
synergy and harmony between UFC and below.
SFCs recommendations and financial outlays. (a)  1 only (b)  2 only
In the context of the above, which of these (c)  2 and 3 only (d)  1 and 3 only
is correct? Solution: (b)
(a) A is correct, and R is an appropriate The separate National Commission for SCs
explanation of A. came into existence in 2004. It consists of
(b) A is correct, but R is not an appropriate a chairperson, a vice-chairperson and three
explanation of A. other members. They are appointed by the
(c) A is incorrect, but R is correct. President by warrant under his hand and seal.
(d) A is correct, but R is incorrect. Their conditions of service and tenure of
Solution: (c) office are also determined by the President.
Justification: Statement A: The governor The commission presents an annual report
of a state, after every five years, constitutes to the president. It can also submit a report as
a finance commission to review the financial and when it thinks necessary. The President
position of the Panchayats. It is not the UFC places all such reports before the Parliament,
that constitutes it. along with a memorandum explaining the
Statement R: However, there is definitely action taken on the recommendations made
a need to ensure harmony and balance by the Commission. The memorandum
between the recommendations of all finance should also contain the reasons for the non
commissions as: acceptance of any of such recommendations.
1. Both recommend devolution of funds to
PRIs (for example 14th UFC). Powers
2. Both take into account the fiscal situation
of the state before recommending any Q.1347 Consider the following statements about the
such outlays. powers of the National Commission for SCs:
3. UFC is mandated to suggest the measures 1. It has the powers of a civil court while
needed to augment the consolidated fund trying specific cases.
of a state to supplement the resources of 2. It is also required to discharge similar
the panchayats in the states (on the basis functions in respect of the Anglo-Indian
of the recommendations made by the community (a minority) as it does with
finance commission of the state). respect to the SCs.
3. The Central government and the state
governments are required to consult the

P.376 For Civil Services Preliminary Examination

04-Indian Polity_Q1070-1506.indd 376 8/7/2018 7:42:42 PM


commission on all major policy matters SCs and to report to him (Article 338). Similarly,
affecting the SCs. the President should also set up a National
Which of these is/are correct? Choose the Commission for the STs to investigate all
correct answer using the codes below: matters relating to the Constitutional safeguards
(a) 1 and 2 (b) 2 and 3 for the STs and to report to him (Article 338-A).
(c) 1 and 3 (d)  All of the above The President should place all such reports
Solution: (d) before the Parliament, along with the action taken
Powers of the commission memorandum. Previously, the Constitution
The Commission is vested with the provided for a combined National Commission
power to regulate its own procedure. The for SCs and STs. The 89th Amendment Act of
Commission, while investigating any matter 2003 bifurcated the combined commission into
or inquiring into any complaint, has all the two separate bodies.
powers of a civil court trying a suit and in The National Commission for SCs is
particular in respect of the following matters: also required to discharge similar functions
(It follows the process of court) with regard to the OBCs and the Anglo-
(a) Summoning and enforcing the Indian Community as it does with respect
attendance of any person from any part to the SCs. In other words, the commission
of India and examining him on oath; has to investigate all matters relating to the
(b) Requiring the discovery and production Constitutional and other legal safeguards for
of any document; the OBCs and the Anglo-Indian community
(c) Receiving evidence on affidavits; and report to the President upon their working.
(d) Requisitioning any public record from
any court or office; Functions
(e) Issuing summons for the examination of
Q.1349 Consider the following statements about
witnesses and documents; and
the nature and functions of the National
(f) Any other matter which the President
Commission for SCs:
may determine.
1. It is statutory body.
The Central government and the state
2. It enquires into specific complaints with
governments are required to consult the
respect to deprivation of rights of the SCs.
Commission on all major policy matters
3. In case of non-acceptance of its
affecting the SCs.
recommendations, the government must
Statement 2: The Commission is also
explain the reasons to the concerned
required to discharge similar functions with
house of Parliament.
regard to the other backward classes (OBCs)
Which of these is/are correct? Choose the
and the Anglo-Indian Community as it does
correct answer using the codes below:
with respect to the SCs.
(a) 1 and 2 (b) 2 and 3
Q.1348 As it does with respect to the SCs, the National (c) 1 and 3 (d)  All of the above
Commission for SCs is also required to Solution: (b)
discharge similar functions with regard to the It is a constitutional body. It works for
1. OBCs ensuring the right of the SCs.
2. STs Functions of the commission
3. Anglo-Indian Community The functions of the Commission are:
Choose the correct answer from the codes (a) To investigate and monitor all matters
given below. relating to the Constitutional and other
(a)  1 and 2 only (b)  2 and 3 only legal safeguards for the SCs and to
(c)  1 and 3 only (d)  All of the above evaluate their working;
Solution: (c) (b)  To inquire into specific complaints with
The President should set up a National respect to the deprivation of rights and
Commission for the SCs to investigate all matters safeguards of the SCs;
relating to the constitutional safeguards for the

Indian Polity Question Bank P.377

04-Indian Polity_Q1070-1506.indd 377 8/7/2018 7:42:42 PM


(c)  To participate and advise on the planning (51) NATIONAL COMMISSION FOR
process of socio-economic development
of the SCs and to evaluate the progress
SCHEDULED TRIBES (STs) – PART
of their development under the Union or XVI—ARTICLE 338A
a state;
Q.1351 Consider the following statements about the
(d) To present to the President, annually
National Commission for STs.
and at such other times as it may deem
1. It comes under the Ministry of Social
fit, reports upon the working of those
Justice and empowerment.
safeguards;
2. The Chairman is appointed by the
(e) To make recommendations as to the
President after a recommendation is made
measures that should be taken by
from the Ministry of Social Justice and
the Union or a state for the effective
empowerment.
implementation of those safeguards and
3. The Conditions of Service and tenure
other measures for the protection, welfare
of the members of the Commission is
and socio-economic development of the
determined by the Constitution.
SCs; and
Choose the correct answer using the codes
(f) To discharge such other functions in
below.
relation to the protection, welfare and
(a)  1 and 2 only (b)  2 and 3 only
development and advancement of the
(c)  3 only (d)  None of the above
SCs as the president may specify
Solution: (d)
Q.1350 Which of the following is/are the functions The National Commission for SCs and STs
of the National Commission for Scheduled came into being consequent upon passing
Castes (SCs)? of the 65th Constitutional Amendment Act
1. Inquiring into specific complaints with of 1990. The Commission was established
respect to the deprivation of rights and under Article 338 of the Constitution with
safeguards of the SC the objective of monitoring all the safeguards
2. Governing the Autonomous tribal councils provided for the SCs and STs under the
in Fifth Schedule districts Constitution or other laws.
3. Monitoring and directing the High Courts Geographically and culturally, the STs are
to grant relief in cases pertaining to SCs different from the SCs and their problems are
Select the correct answer using the codes also different from those of SCs. In 1999, a
below. new Ministry of Tribal Affairs was created
(a)  1 and 2 only (b)  1 only to provide a sharp focus to the welfare and
(c)  2 and 3 only (d)  1 and 3 only development of the STs. It was felt necessary
Solution: (b) that the Ministry of Tribal Affairs should
Justification: Statement 1: The Commission, co-ordinate all activities relating to the STs
while investigating any matter or inquiring as it would not be administratively feasible
into any complaint, has all the powers of a for the Ministry of Social Justice and
civil court trying a suit. Empowerment to perform this role
Statement 2: It is an advisory body to the The separate National Commission for STs
Government. It does not govern any tribal or came into existence in 2004. It consists
autonomous councils. of a chairperson, a vice-chairperson and
Statement 3: It cannot direct the courts. It three other members. They are appointed
can only recommend punitive measures in by the President by warrant under his hand
cases pertaining to the SCs after conducting and seal. Their conditions of service and
due inquiry. The Central government and the tenure of office are also determined by the
state governments are required to consult President.
the Commission on all major policy matters
affecting the SCs.

P.378 For Civil Services Preliminary Examination

04-Indian Polity_Q1070-1506.indd 378 8/7/2018 7:42:42 PM


Functions & Powers Q.1353 Consider the following statements about the
power of the National Commission for STs.
Q.1352 Which of the following functions have 1. The Central government and the state
been specified for the National Commission governments are required to consult the
for STs with relation to the protection and Commission on all major policy matters
welfare of STs? affecting the STs.
1. Measures to be taken to promote the 2. Annual report submitted by the
traditional practice of shifting cultivation commission is placed before the
2. Measures to be taken over conferring Parliament by the President.
ownership rights in respect of minor 3. The commission has powers of
forest produce to STs living in forest areas investigation into cases concerning the
3. Measures to be taken to safeguard rights rights and safeguards of STs.
of the tribal communities over mineral Choose the correct answer using the codes
resources below.
Select the correct answer using the codes (a)  1 and 2 only (b)  2 and 3 only
below. (c)  3 only (d)  All of the above
(a)  1 only (b)  2 and 3 only Solution: (d)
(c)  1 and 3 only (d)  None of the above The Commission presents an annual report to
Solution: (b) the President. It can also submit a report as
Justification: In 2005, the President and when it thinks necessary. The President
specified the following other functions of places all such reports before the Parliament,
the Commission in relation to the protection, along with a memorandum explaining the
welfare and development and advancement action taken on the recommendations made
of the STs. by the Commission. The memorandum
Statement 1: As per the notification, the should also contain the reasons for the non-
commission should strive to reduce and acceptance of any of such recommendations.
eliminate shifting cultivation, not encourage it, The President also forwards any report
that lead to their continuous disempowerment of the Commission pertaining to a state
and degradation of land and the environment. government to the state governor. The
Learning: Others measures (apart from governor places it before the state legislature,
statements 2 and 3 are): along with a memorandum explaining the
• Development of tribal communities and to action taken on the recommendations of the
work for more viable livelihood strategies Commission. The memorandum should also
• Improve the efficacy of relief and contain the reasons for the nonacceptance of
rehabilitation measures for tribal groups any of such recommendations.
displaced by development projects
• Prevent alienation of tribal people from
land and to effectively rehabilitate such
(52) SPECIAL OFFICER FOR LINGUISTIC
people in whose case alienation has MINORITIES – PART XVII –
already taken place ARTICLE 350B
• Elicit maximum cooperation and
involvement of tribal communities for Q.1354 Which of the following statements with
protecting forests and undertaking social reference to the Special Officer for Linguistic
afforestation Minorities is INCORRECT?
• Ensure full implementation of the (a) The officer is appointed by the President
Provisions of Panchayats (Extension to of India.
the Scheduled Areas) Act, 1996 (b) It is a constitutional post.
The Commission presents an annual report to (c) Its reports are placed before the
the President. It can also submit a report as Parliament by the President.
and when it thinks necessary.

Indian Polity Question Bank P.379

04-Indian Polity_Q1070-1506.indd 379 8/7/2018 7:42:42 PM


(d) It is an autonomous body and does not The Commissioner is assisted at headquarters
operate under any Ministry of Central by Deputy Commissioner and an Assistant
government. Commissioner. He maintains liaison with
Solution: (d) the State Governments and Union Territories
Learning: The Seventh Constitutional through nodal officers appointed by them.
Amendment Act of 1956 inserted a new At the Central level, the Commissioner falls
Article 350-B in Part XVII of the Constitution under the Ministry of Minority Affairs.
providing for the office.
The Constitution does not specify (53) CAG – PART V – ARTICLES 148
the qualifications, tenure, salaries and
allowances, service conditions and procedure
TO 151
for removal of the Special Officer for Q.1356 Apart from Constitutional provisions, which
Linguistic Minorities. of the following has/have a bearing on the
At the Central level, the Commissioner functioning of CAG in India?
falls under the Ministry of Minority Affairs. (a)  Parliamentary laws
Hence, he submits the annual reports or (b)  Supreme Court Judgments
other reports to the President through the (c)  Instruction of Government of India
Union Minority Affairs Minister. The (d)  All of the above
President should place all such reports before Solution: (d)
each House of Parliament and send to the Learning: Option (a): The Comptroller
governments of the states concerned. and Auditor General’s (Duties, Powers and
Q.1355 Consider the following statements about the Conditions of Service) Act, 1971 was enacted
Special Officer for Linguistic Minorities. by the Parliament.
1. Originally the Constitution did not make Option (b): For example, in Arvind Gupta
any provision for it. v. UOI, 2012, the Supreme Court dealt with the
2. The Constitution does not mention powers of CAG to Conduct Performance Audit.
qualifications for the Office. Option (c): For e.g. Clarification was
3. At the Central level, the Commissioner sought by CAG from the government once
falls under the Ministry of Minority Affairs. whether Performance Audit falls within the
Choose the correct answer using the codes scope of audit by CAG under the Comptroller
below. and Auditor General’s (Duties, Powers
(a)  1 and 2 only (b)  2 and 3 only and Conditions of Service)Act, 1971. The
(c)  1 and 3 only (d)  All of the above Department of Economic Affairs instructed
Solution: (d) that it does
The Seventh Constitutional Amendment Act
of 1956 inserted a new Article 350-B in Part Appointment
XVII of the Constitution providing for the
Q.1357 Consider the following statements about the
office. The article 350B says as follows.
Controller general of Accounts (CGA) in
1. There should be a Special Officer
India.
for Linguistic Minorities. He is to be
1. He is appointed by the President of India.
appointed by the President of India.
2. He is the principal accounts adviser to the
2. It would be the duty of the Special
Union government.
Officer to investigate all matters relating
3. Union finance account prepared by him is
to the safeguards provided for linguistic
presented to the Parliament.
minorities under the Constitution. He
Choose the correct answer using the codes
would report to the President upon those
below.
matters at such intervals as the President
(a)  1 and 2 only (b)  All of the above
may direct.
(c)  2 and 3 only (d)  1 and 3 only
Solution: (b)

P.380 For Civil Services Preliminary Examination

04-Indian Polity_Q1070-1506.indd 380 8/7/2018 7:42:42 PM


Statement 1: Article 148 says there shall be Article 148: Comptroller and Auditor-
a Comptroller and Auditor-General of India General of India (2) Every person appointed
who shall be appointed by the President by to be the Comptroller and Auditor-General of
warrant under his hand and seal and shall only India shall, before he enters upon his office,
be removed from office in like manner and on make and subscribe before the President or
like grounds as a Judge of the Supreme Court. some person appointed in that behalf by him,
The CGA is the principal Accounts an oath or affirmation according to the form
Adviser to the Union Government. He is set out for the purpose in the Third Schedule.
responsible for maintaining and establishing (3) The salary and other conditions of service
a technically sound management accounting of the Comptroller and Auditor-General shall
system. Every month he prepares a critical be such as may be determined by Parliament
analysis of revenues, expenditures, by law and, until they are so determined,
borrowings and the deficit for Union shall be as specified in the Second Schedule:
Finance Minister. He also prepares annual Provided that neither the salary of a
Union Finance Accounts and Appropriation Comptroller and Auditor-General nor his
Accounts (Civil) for presentation to the rights in respect of leave of absence, pension
Parliament. or age of retirement shall be varied to his
Q.1358 The Comptroller and Auditor General of disadvantage after his appointment.
India is appointed by the President of India (4) The Comptroller and Auditor-General
based on the advice by shall not be eligible for further office either
1. Council of Ministers under the Government of India or under the
2. Committee consisting of the Prime Government of any State after he has ceased
Minister, Lok Sabha speaker and Leader to hold his office.
of Opposition (5) Subject to the provisions of this
3. Former CAG of India Constitution and of any law made by
Choose the correct answer using the codes parliament, the conditions of service of
below. persons serving in the Indian Audit and
(a)  1 and 2 only (b)  2 only Accounts Department and the administrative
(c)  1 and 3 only (d)  1 only powers of the Comptroller and Auditor-
Solution: (d) General shall be such as may be prescribed by
The CAG is appointed by the president rules made by the President after consultation
of India by a warrant under his hand and with the Comptroller and Auditor-General.
seal. The recommendation is passed by the (6) The administrative expenses of the office
Council of Ministers. of the Comptroller and Auditor-General
including all salaries, allowances and
Q.1359 With reference to the Comptroller and
pensions payable to or in respect of persons
Auditor General of India, consider the
serving in that office, shall be charged upon
following statements
the Consolidated Fund of India.
1. The reports of the Comptroller and
149. Duties and Powers of the CAG
Auditor-General of India relating to the
150. Form of accounts of the Union and of
accounts of the States shall be submitted
the States:- The accounts of the Union and
to the president, who shall cause them
of the States shall be kept in such form as the
to be laid before concerned Legislative
President may, on the advice of the Comptroller
Assemblies
and Auditor-General of India, prescribe.
2. He shall only be removed from office
151. Audit Reports:- (1) The reports of the
in like manner and on like grounds as a
Comptroller and Auditor-General of India
Judge of the Supreme Court
relating to the accounts of the Union shall
Which of the above statements is/are correct?
be submitted to the president, who shall
(a)  1 Only (b)  2 Only
cause them to be laid before each House of
(c) Both (d) None
Parliament.
Solution: (b)

Indian Polity Question Bank P.381

04-Indian Polity_Q1070-1506.indd 381 8/7/2018 7:42:42 PM


(2) The reports of the Comptroller and to that effect by both the Houses of Parliament
Auditor-General of India relating to the with special majority, either on the ground of
accounts of a State shall be submitted to the proved misbehaviour or incapacity.
Governor of the State, who shall cause them Q.1362 Consider the following statements about the
to be laid before the Legislature of the State. removal of the CAG:
1. He can only be removed after a Supreme
Independence Court inquiry in the pertinent case.
Q.1360 The Comptroller and Auditor General of 2. Both the houses need to approve the
India can resign any time from his office removal by absolute majority.
by addressing the resignation letter to the 3. After removal or retirement he is not
president. But, he can also be removed from eligible for any further employment in the
the office in the same manner as that of Government of India.
(a)  A member of the UPSC Which of these is/are true?
(b)  A Supreme Court Judge (a) 1 and 2 (b) 2 and 3
(c)  Cabinet Secretary (c)  1 and 3 (d) All of the above
(d) Chairman, Public Accounts Committee Solution: (c)
Solution: (b) Both the houses need to approve it by a
Learning: The CAG is appointed by the special majority.
president of India by a warrant under his hand He can be removed in the exact same
and seal. He can be removed by the president manner and on the same grounds as that of
on the basis of a resolution passed to that a SC judge.
effect by both the Houses of Parliament with Q.1363 Consider the following statements about
special majority, either on the ground of the office of the Comptroller and Auditor
proved misbehaviour or incapacity. Member General of India (CAG):
of UPSC can be removed by the President Assertion (A): His constitutional status in
based on a SC inquiry. It does not require the Table of precedence is same as that of a
Parliament’s approval. Supreme Court Judge.
Q.1361 To remove the CAG from his office, which of Reason (R): He can be removed in the
the following would be absolutely required? same manner as that of a Supreme Court
1. Resolution passed with special majority Judge, and his allowances, service conditions
by Lok Sabha and other benefits are at par with the latter.
2. Resolution passed with special majority In the context of the statements above,
by Rajya Sabha which of these is true?
3. Enquiry by the Supreme Court (a) Both A and R are true and R is the correct
4. Order of the President explanation for A.
Choose the correct answer using the codes (b) Both A and R are true and R is not the
below. correct explanation for A.
(a)  1 and 4 only (b)  2, 3 and 4 only (c)  A is incorrect but R is correct.
(c)  1 and 2 only (d)  All of the above (d)  A is correct but R is incorrect.
Solution: (d) Solution: (c)
The CAG holds office for a period of six It is important to note that the table
years or up to the age of 65 years, whichever of Precedence is not mentioned in the
is earlier. He can resign any time from his constitution of India. It was issued in a
office by addressing the resignation letter to notification in the year 1959 by the Central
the president. He can also be removed by the Government. This table of precedence has
president on same grounds and in the same no official standing. It is only used in sitting
manner as a judge of the Supreme Court. arrangements and honorary protocols during
In other words, he can be removed by the official ceremonies, functions etc.
president on the basis of a resolution passed

P.382 For Civil Services Preliminary Examination

04-Indian Polity_Q1070-1506.indd 382 8/7/2018 7:42:42 PM


Therefore, it would not be incorrect to Justification: No minister can represent the
compare his or any other ‘constitutional CAG in Parliament in any of the Houses.
authorities’ status with that of other high Further, no minister can be called upon to
authorities take any responsibility for any actions done
Q.1364 Consider the following statements about the by CAG.
Comptroller and Auditor General of India This is because CAG is independent
(CAG). of the government. The finance minister
Assertion (A): No minister can represent representing CAG would bring conflict of
the CAG in Parliament or be called upon to interest and defeat the very purpose of having
take any responsibility for any actions done an independent financial watchdog.
by him.
Reason (R): The CAG is an autonomous Functions
body and not appointed by the President or Q.1366 The Comptroller and Auditor General of
Council of Ministers. India (CAG)  is not concerned with the audit
In the context of the above, which of these of receipts and expenditure of
is correct? (a)  Central Government
(a) A is correct, and R is an appropriate (b)  State Government
explanation of A. (c)  Government Companies
(b) A is correct, but R is not an appropriate (d)  Local Bodies
explanation of A. Solution: (d)
(c) A is correct, but R is incorrect. *Justification*: Option D: Local Bodies
(d) Both A and R are incorrect. Audit reports are prepared by the concerned
Solution: (c) state Accountant Generals (AGs). On being
Justification: The CAG is appointed by the approved by the CAG, the Local Bodies
president of India by a warrant under his Audit reports are categorized into two types
hand and seal. So, R is wrong. as ‘Tabled in the Legislature’ and ‘Issued to
Moreover, the CAG, before taking over State Government’. So, D is incorrect.
his office, makes and subscribes before the Option C: Apart from Central and State
president an oath or affirmation. government, the CAG is also the external
Ministers don’t represent CAG because auditor of Government-owned corporations
CAG audits the expenditures sanctioned by and conducts supplementary audit of
the Ministers. If they represent CAG, this government companies, i.e., any non-
will lead to conflict of interest. Further, CAG banking/ non-insurance company in which
is a part of the Union Executive. Thus, A is Union Government has an equity share of
correct and reasonable. at least 51 per cent or subsidiary companies
Q.1365 Consider the following statements. of existing government companies. So, the
Assertion (A): The Minister of Finance option C is correct.
represents the CAG in the Parliament. Q.1367 As per constitutional provisions and
Reason (R): CAG was established to CAG’s (Duties, Powers and Conditions of
ensure the financial accountability of the Service) act, 1971, the Comptroller and
executive to the legislature. Auditor General of India is authorized to
In the context of the above, which of these audit which of the following accounts apart
is correct? from the Consolidated Fund of India?
(a) A is correct, and R is an appropriate 1. Contingency Fund of India
explanation of A. 2. Public Account of India
(b) A is correct, but R is not an appropriate 3. Contingency fund of each state
explanation of A. 4. Public account of each state
(c) A is incorrect, but R is correct. 5. Consolidated fund of each state
(d) Both A and R are incorrect.
Solution: (c)

Indian Polity Question Bank P.383

04-Indian Polity_Q1070-1506.indd 383 8/7/2018 7:42:42 PM


Select the correct answer using the codes Justification: Statement 2: He is called by
below. the PAC to help it examine the government
(a)  1, 2 and 5 only (b)  1 and 4 only expenditure in the last financial year. CAG is
(c)  5 only (d)  1, 2, 3, 4 and 5 not a member of the PAC.
Solution: (d) Q.1369 Consider the following statements about the
Learning: The duties and functions of the powers and functions of the CAG:
CAG as laid down by the Parliament and the 1. The Parliament can reduce the powers of
Constitution are: the CAG with only a simple majority in
1. He audits the accounts related to all each of the houses.
expenditure from the Consolidated Fund 2. He audits the expenditure both from
of India, consolidated fund of each state the Consolidated Fund of India and
and consolidated fund of each union Contingency fund of India.
territory having a Legislative Assembly. 3. He can also audit the accounts of a Zila
2. He audits all expenditure from the Parishad if provided by the President of
Contingency Fund of India and the India.
Public Account of India as well as the Which of these is/are true?
contingency fund of each state and the (a) 1 and 2 (b) 2 and 3
public account of each state. (c) 1 and 3 (d) All of the above
3. He audits all trading, manufacturing, Solution: (d)
profit and loss accounts, balance sheets The Constitution empowers the Parliament
and other subsidiary accounts kept by any under Article 149 to prescribe the powers
department of the Central Government and duties of the CAG. Accordingly the
and state governments. CAG’s (Duties, powers and Conditions of
4. He audits the receipts and expenditure of service) act, 1971 was enacted. Thus, the
the Centre and each state to satisfy himself Parliament can reduce the powers of the
that the rules and procedures in that behalf CAG just by amending the provisions of the
are designed to secure an effective check bill by a simple majority.
on the assessment, collection and proper
allocation of revenue Role
5. He also audits government companies.
Q.1370 The Comptroller General of India is
Q.1368 Consider the following about the duties
responsible to
and responsibilities of the Comptroller and
(a)  The Parliament
Auditor General of India (CAG).
(b)  The Constitution of India
1. He audits the accounts related to all
(c)  The President of India
expenditure from the Consolidated Fund
(d) Public Accounts Committee of Lok
of India and Consolidated fund of each
Sabha
state.
Solution: (a)
2. He is an ex officio member of the Public
The role of CAG is to uphold the Constitution
Accounts Committee guiding its conduct
of India and the laws of Parliament in
of financial affairs.
the field of financial administration. The
3. He is an advisory member of the
accountability of the executive (i.e., council
Department of Economic Affairs
of ministers) to the Parliament in the sphere
suggesting economy cuts in the design of
of financial administration is secured through
annual budget.
audit reports of the CAG. The CAG is an
Select the correct answer using the codes
agent of the Parliament and conducts audit
below.
of expenditure on behalf of the Parliament.
(a)  1 only (b)  3 only
Therefore, he is responsible only to the
(c)  2 and 3 only (d)  1 and 2 only
Parliament
Solution: (a)

P.384 For Civil Services Preliminary Examination

04-Indian Polity_Q1070-1506.indd 384 8/7/2018 7:42:42 PM


Q.1371 What do you understand by ‘proprietary In addition to this legal and regulatory
audit’ of government expenditure by CAG? audit, the CAG can also conduct the
1. That the money that has been disbursed propriety audit, that is, he can look into
was legally available for the applied the ‘wisdom, faithfulness and economy’ of
service or purpose government expenditure and comment on
2. That the money was efficiently deployed the wastefulness and extravagance of such
in desirable government schemes and expenditure.
wastage of funds was avoided However, unlike the legal and regulatory
3. That the public money was not invested to audit, which is obligatory on the part of the
buy speculative and risky assets abroad CAG, the propriety audit is discretionary.
4. That the expenditure conforms to the Q.1373 The ultimate responsibility of taking due
authority that governs it action on the comments of the Comptroller
Select the correct answer using the codes and Auditor General of India (CAG) vests
below. with the
(a)  2 and 3 only (b)  1 and 4 only (a)  President of India (b)  Supreme Court
(c)  1 and 2 only (d)  2 and 4 only (c) Parliament (d) Prime Minister
Solution: (b) Solution: (c)
Justification: Statement 2: This is a part of Justification: CAG reports the activities and
Performance audit which is discretionary for finances of the government to the PAC of the
CAG. Parliament. The CAG is like a watchdog of
In scrutinising the appropriation accounts the Parliament. Since it reports executive’s
and the audit report of CAG on it, the PAC activities, the executive cannot initiate due
satisfies proprietary audit that: action on its comments. It must be the other
• the money that has been disbursed was wings of the State. So, (a) and (d) are incorrect.
legally available for the applied service or The Supreme Court doesn’t generally
purpose; interfere unless the case is brought to it by a
• the expenditure conforms to the authority party. So, (b) is incorrect.
that governs it; and It is Parliament that censures the executive
• every re-appropriation has been made in in case of any irregularities. However, it must
accordance with the related rules be noted that the executive enjoys a majority
Q.1372 Apart from the legal and regulatory audit, in the house, so any disciplinary action may
the CAG can also conduct the propriety audit not be taken by the Parliament
which means Q.1374 Consider a situation where the incumbent
(a) It can audit the accounts of the recipients executive has been found guilty of financial
of government spending. irregularities based on a CAG report as
(b) It can look into the usefulness and examined by the Public Accounts Committee.
appropriateness of government The ultimate responsibility to take appropriate
expenditure. action on this report lies on?
(c) It can audit the accounts of secret (a)  Prime Minister of India
services. (b)  President of India
(d) It can audit the source from where (c) Parliament
government raises money. (d)  Supreme Court
Solution: (b) Solution: (c)
Learning: The CAG has ‘to ascertain Every department and official is given
whether money shown in the accounts as information about the amount of funds they
having been disbursed was legally available have and they have to perform within it - if
for and applicable to the service or the it goes overboard then they are accountable.
purpose to which they have been applied These funds are passed by the Parliament/
or charged and whether the expenditure Legislature through voting. The auditing and
conforms to the authority that governs it’. reporting work is ex post facto as it is not

Indian Polity Question Bank P.385

04-Indian Polity_Q1070-1506.indd 385 8/7/2018 7:42:42 PM


easy to get the finance back once the Finance 3. He must be a member of either House of
Ministry sanctions funds to the departments. Parliament.
So, ultimately Parliament has to hold the 4. He can be removed by impeachment by
executive financially accountable. Parliament.
Which of these statements are correct ?
(54) ATTORNEY GENERAL (a)  1 and 2 (b)  1 and 3
(c)  2, 3 and 4 (d)  3 and 4
(AG) – PART V – ARTICLE 76 Solution: (a)
Q.1375 Consider the following statements about the Refer previous question explanation.
office of the Attorney General of India. Q.1377 Consider the following about the office of the
1. It has been created by the Constitution of Attorney General (AG).
India. 1. Appointed by the president, he must be a
2. The Attorney General (AG)  is appointed person who is qualified to be appointed a
by the Prime Minister. judge of the Supreme Court.
3. He must have served as a judge of the 2. The term of office of the AG is fixed by
Supreme Court in the past. the Constitution giving him security of
4. He appears on behalf of the Government tenure.
of India (GoI) in the cases in Supreme 3. His office is under the Ministry of Law
Court in which the GoI is concerned. and Justice and the AG reports the Union
Select the correct answer using the codes Law Minister.
below. Select the correct answer using the codes
(a)  3 and 4 only (b)  1 and 3 only below.
(c)  1 and 4 only (d)  1, 2, 3 and 4 (a)  2 only (b)  1 and 3 only
Solution: (c) (c)  2 and 3 only (d)  1 only
Justification: Statement 1: Article 76 Solution: (d)
provides for the office of the Attorney Justification: Statement 1: In other words,
General of India. he must be a citizen of India and he must
Statement 2: He is appointed by the have been a judge of some high court for five
President. years or an advocate of some high court for
Statement 3: He must be a person who ten years or an eminent jurist, in the opinion
is qualified to be appointed a judge of the of the president.
Supreme Court. He need not have served for Statement 2: The term of office of the AG
it necessarily. Qualifications to be appointed is not fixed by the Constitution. Further, the
for a SC judge are: Constitution does not contain the procedure
• He must be a citizen of India and grounds for his removal. He holds
• he must have been a judge of some high office during the pleasure of the president.
court for five years or an advocate of some This means that he may be removed by the
high court for ten years OR an eminent president at any time.
jurist, in the opinion of the president. Statement 3: During the prime
Statement 4: He is the highest law officer of ministership of Jawaharlal Nehru, a proposal
the country and represents the government was put forward by the Central government
where he is required to. He also represents that the office of the Attorney General be
the Government of India in any reference merged with the office of the law minister. It
made by the president to the Supreme Court did not materialise.
under Article 143 of the Constitution Learning: In the performance of his
Q.1376 Consider the following statements about the official duties, the Attorney General has the
Attorney-General of India: right of audience in all courts in the territory
1. He is appointed by the President of India. of India.
2. He must have the same qualifications as are Further, he has the right to speak and to
required for a Judge of the Supreme Court. take part in the proceedings of both the Houses

P.386 For Civil Services Preliminary Examination

04-Indian Polity_Q1070-1506.indd 386 8/7/2018 7:42:42 PM


of Parliament or their joint sitting and any Q.1380 Consider the following statements about the
committee of the Parliament of which he may Attorney-General (AG)  of India:
be named a member, but without a right to vote. 1. He must have been either a judge of
He enjoys all the privileges and immunities that Supreme or High courts, or an advocate in
are available to a member of Parliament. the High courts.
Q.1378 Consider the following about the Office of 2. He is guaranteed security of tenure by
Attorney General of India. the constitution of India considering the
1. The Attorney General (AG)  is appointed important duties that he performs.
by the president after consultation with 3. The grounds of removal of the AG are not
the Chief Justice of India. mentioned in the constitution.
2. He must be a person who is qualified to be Which of these is/are true?
appointed a judge of the Supreme Court. (a) Only 3 (b) 2 and 3
3. The Constitution does not contain the (c) 1 and 3 (d) 1 and 2
procedure and grounds for his removal. Solution: (a)
4. He may be removed by the president at He conventionally resigns with the coming
any time. of the new government, other than the one
Choose the correct answer using the codes which appointed him.
below. Q.1381 Consider the following statements.
(a)  1 and 4 only (b)  2, 3 and 4 only 1. The Attorney General (AG) is not a
(c)  1 and 2 only (d)  All of the above member of the Central cabinet.
Solution: (b) 2. Just like the AG, other offices subordinate
Explained in previous answers. to AG like Solicitor General and others
Q.1379 Consider the following statements about the are provided for by the Constitution.
Attorney General. 3. The AG looks after all the legal matters of
1. He is a constitutional authority. the Government.
2. He comes under the purview of the Right Choose the correct answer using the codes
to Information (RTI) Act. below.
3. He is appointed by the Chief Justice of (a)  1 and 2 only (b)  2 and 3 only
India. (c)  1 and 3 only (d)  All of the above
Choose the correct answer using the codes Solution: (c)
below. Statement 1: The AG is not a member of
(a)  1 and 2 only (b)  2 and 3 only the Central cabinet. There is a separate law
(c)  1 and 3 only (d)  All of the above minister in the Central cabinet to look after
Solution: (a) legal matters at the government level.
Statement 1: article 76 Statement 2: In addition to the AG, there
Statement 2: Delhi High Court in its ruling are other law officers of the Government of
has held that Attorney General of India comes India. They are the solicitor general of India
under the ambit of Right to Information (RTI) and additional solicitor general of India. They
Act, 2005. Delhi High Court held that assist the AG in the fulfilment of his official
• Attorney General is not merely a lawyer responsibilities. It should be noted here that
for the government, but is a constitutional only the office of the AG is created by the
authority. This is because functions Constitution. In other words, Article 76 does
performed by the AG under Article 76 not mention about the solicitor general and
(2) of the Constitution of India are in the additional solicitor general.
nature of public functions. Statement 3: AG is the highest law officer
• So the office of Attorney General is a of the country. He represents government of
public authority and comes under the ambit India in the courts. As the chief law officer
of section 2(h) of the RTI Act. of the Government of India, the duties of the
Statement 3: He is appointed by President of AG include the following:
India

Indian Polity Question Bank P.387

04-Indian Polity_Q1070-1506.indd 387 8/7/2018 7:42:42 PM


1. To give advice to the Government of Q.1383 Which of these statements regarding the office
India upon such legal matters, which are of Attorney General of India is INCORRECT?
referred to him by the President. (a) He has the right to take part in the
2. To perform such other duties of a legal proceedings of both houses of Parliament.
character that are assigned to him by the (b) Advocate General of India is subordinate
president. to Attorney General of India.
3. To discharge the functions conferred on (c) Attorney General of India forms part of
him by the Constitution or any other law. the Union Executive.
The president has assigned the following (d) His office does not come under the Union
duties to the AG: Law ministry.
1. To appear on behalf of the Government of Solution: (b)
India in all cases in the Supreme Court in The Union Executive consists of the
which the Government of India is concerned. President, Vice-President, Prime Minister,
2. To represent the Government of India in Council of Ministers and Attorney General of
any reference made by the president to the India. The AG is not a member of the Central
Supreme cabinet. There is a separate law minister in
Court under Article 143 of the the Central cabinet to look after legal matters
Constitution. at the government level.
3. To appear (when required by the In the performance of his official duties,
Government of India)  in any high court the Attorney General has the right of audience
in any case in which the Government of in all courts in the territory of India. Further,
India is concerned. he has the right to speak and to take part in the
proceedings of both the Houses of Parliament
Rights & Limitations of Auditor or their joint sitting and any committee of
General of India the Parliament of which he may be named a
member, but without a right to vote.
Q.1382 Who among the following can attend the
Q.1384 Which of the following rights have been
meetings of Rajya Sabha while being not a
granted to the Attorney General of India?
member of the House?
1. Right to take part in the proceedings in
1. Any MP who is a member of Lok Sabha
any of the houses of Parliament
2. Attorney General of India
2. Right to be a member of a Parliamentary
3. Comptroller and Auditor General of India
Committee
Select the correct answer using the codes
3. Right to all the privileges and immunities
below.
granted to the MPs
(a)  1 only (b)  1 and 3 only
Choose the correct answer using the codes
(c)  2 only (d)  2 and 3 only
below.
Solution: (c)
(a)  1 and 2 only (b)  2 and 3 only
Justification: Statement 1: Only Ministers are
(c)  1 and 3 only (d)  All of the above
entitled to attend the meetings of both houses.
Solution: (d)
Statement 2:  Attorney general has right
of audience in all courts within the territory Q.1385 Consider the following statements about
of India. He has also the right to speak and the roles, powers and responsibilities of the
take part in proceedings of both the houses of Attorney General of India.
parliament including joint sittings. However, 1. He does not fall in the category of
he cannot vote in parliament. government servants.
• Further, attorney general can also be made 2. He enjoys all the privileges and immunities
a member of any parliamentary committee that are available to a Member of Parliament.
but in the committee also, he has no power 3. He is not debarred from private legal
to vote. practice.
• Attorney General has all the powers and 4. He has the right to speak and to take part
privileges that of a member of parliament. in the proceedings of both the Houses of

P.388 For Civil Services Preliminary Examination

04-Indian Polity_Q1070-1506.indd 388 8/7/2018 7:42:43 PM


Parliament and any committees thereof Which of these is/are true?
where he is a member. (a) Only 1 (b) Only 2
Choose the correct answer using the codes (c) Both (d) None of the above
below. Solution: (d)
(a)  1 and 4 only (b)  2, 3 and 4 only Only the office of the AG is created by the
(c)  1 and 2 only (d)  All of the above constitution. Looking after the legal matters
Solution: (d) is the responsibility of the law ministry.
Following limitations are placed on the
Attorney General in order to avoid any (55) ADVOCATE GENERAL IN THE
complication and conflict of duty:
1. He should not advise or hold a brief
STATE – PART VI – ARTICLE 165
against the Government of India. Q.1388 The Advocate General in the State
2. He should not advise or hold a brief in 1. is appointed by the President in
cases in which he is called upon to advise consultation with the Chief Justice of
or appear for the Government of India. High Court
3. He should not defend accused persons 2. is the Chief Law officer of the Government
in criminal prosecutions without the 3. may not be a citizen of India.
permission of the Government of India. Choose the correct answer using the codes
4. He should not accept appointment as a below.
director in any company or corporation (a)  1 and 2 only (b)  2 only
without the permission of the Government (c)  1 and 3 only (d)  All of the above
of India. Solution: (b)
However, the Attorney General is not a The advocate general is appointed by the
full-time counsel for the Government. He governor. He must be a person who is
does not fall in the category of government qualified to be appointed a judge of a high
servants. Further, he is not debarred from court. In other words, he must be a citizen of
private legal practice. India and must have held a judicial office for
Q.1386 Consider the following statements about ten years or been an advocate of a high court
the rights and limitations of the Attorney- for ten years.
General of India: As the chief law officer of the government
1. He has the right to speak and take part in in the state, the duties of the advocate general
the proceedings of both the houses of the include the following:
Parliament. 1. To give advice to the government of the
2. He enjoys all the privileges and immunities state upon such legal matters which are
that are available to a Member of Parliament. referred to him by the governor.
3. He cannot advise or hold a brief against 2. To perform such other duties of a legal
the Government of India. character that are assigned to him by the
Which of these is/are true? governor.
(a)  1 and 2 (b) 2 and 3 3. To discharge the functions conferred on
(c)  1 and 3 (d)  All of the above him by the Constitution or any other law.
Solution: (d) 4. In the performance of his official duties,
the advocate general is entitled to appear
Solicitor General of India before any court of law within the state.
Q.1387 Consider the following statements about the Q.1389 Consider the following statements about the
office of the Solicitor General of India: Advocate General in the states of India:
1. The office is created by the constitution of Assertion (A): While he is appointed by
India. the Governor of the state, he can be removed
2. He looks after the legal matters of the only by the President of India.
Central government and assists the Central
Cabinet in the discharge of its duties.

Indian Polity Question Bank P.389

04-Indian Polity_Q1070-1506.indd 389 8/7/2018 7:42:43 PM


Reason (R): He has the same constitutional NON-CONSTITUTIONAL BODIES
status as that of the Chairman of the State
Public Service Commission. (56) NITI AYOG
In the context of the statements above,
which of these is true? Q.1390 Who among the following is/are the
(a) Both A and R are true and R is the correct member(s) of NITI Ayog?
explanation for A. 1. Chief Ministers of all States
(b) Both A and R are true and R is not the 2. Chief Minister of UTs (with legislature)
correct explanation for A. 3. Nominated Union Cabinet Ministers
(c)  A is incorrect but R is correct. 4. Union Secretary of Defence, Finance,
(d)  None of the above Planning and Home
Solution: (d) 5. State Finance Ministers
He is both appointed and removed by the Choose the correct answer using the codes
Governor of the state. below.
There is nothing like higher or lower (a)  1, 2 and 5 only (b)  1, 3, 4 and 5 only
constitutional status. Even the Table of (c)  3 only (d)  1, 2 and 3 only
Precedence is not about constitutional status Solution: (c)
or importance of that particular office The NITI Aayog comprises the following:
1. Prime Minister of India as the Chairperson
2. Governing Council comprising the Chief
Ministers of all the States and union
territories with legislature and lieutenant
governors of other Union Territories
3. Regional Councils
4. Experts, specialists and practitioners with
relevant domain knowledge as special
invitees nominated by the Prime Minister
5. Full-time organizational framework
(in addition to Prime Minister as the
Chairperson) comprising
1. Vice-Chairperson: Arvind Panagariya
2. Members: Two (2) Full-time
3. Part-time members: Maximum of two
from leading universities research
organizations and other relevant
institutions in an ex-officio capacity. Part-
time members will be on a rotational basis
4. Ex Officio members: Maximum of
four members of the Union Council of
Ministers to be nominated by the Prime
Minister
5. Chief Executive Officer: To be appointed
by the Prime Minister for a fixed tenure, in
the rank of Secretary to the Government of
India.
6. Secretariat as deemed necessary
Q.1391 Which of the following statements is/are
correct about NITI aayog? 
1. A serving IES officer can become a part
time member of Niti Aayog.

P.390 For Civil Services Preliminary Examination

04-Indian Polity_Q1070-1506.indd 390 8/7/2018 7:42:43 PM


2. A union minister can become a full time 3) To focus on technology upgradation and
members of Niti Aayog capacity building for implementation of
(a) Only 1 (b) Only 2 programmes and initiatives
(c) Both (d) None (a) 1, 3 (b) 1, 2
Solution: (d) (c) 2, 3 (d) All of the above
• Maximum of two from leading Solution: (d)
universities research organizations and Apart from above NITI Aayog’s functions
other relevant institutions can become are:
part -time members of Niti Aayog on • To develop mechanisms to formulate
rotational basis. credible plans at the village level and
• Maximum of four members of the Union aggregate these progressively at higher
Council of Ministers to be nominated by levels of government
the Prime Minister can become Ex-officio • To pay special attention to the sections
members of Niti Aayog. of our society that may be at risk of not
Q.1392 Which of the following are functions of NITI benefitting adequately from economic
Aayog? progress
1) Fostering cooperative federalism • To ensure, on areas that are specifically
2) Platform for resolution of intersectoral referred to it, that the interests of national
and inter-departmental issues security are incorporated in economic
3) Instrument to bring outside ideas into strategy and policy
policy-making • Feedback for constant innovative
(a) 1, 3 (b) 1, 2 improvements
(c) 2, 3 (d) All of the above • To mainstream external ideas into
Solution: (d)  Government policies, through
Apart from above NITI Aayog’s functions collaboration with national and
are: international experts;
• To maintain a state-of-the-art Resource • To be the Government’s link to the outside
Centre, be a repository of research on world, outside experts and practitioners;
good governance and best practices in
sustainable and equitable development as (57) NHRC – NATIONAL HUMAN
well as help their dissemination to stake- RIGHTS COMMISSION
holders
• To offer a platform for resolution of inter- Q.1394 Consider the following about National
sectoral and inter­departmental issues in Human Rights Commission.
order to accelerate the implementation of 1. The Commission is independent of the
the development agenda. government.
• To create a knowledge, innovation and 2. It is a constitutional authority.
entrepreneurial support system through a 3. Its recommendations are binding on the
collaborative community of national and courts but not government.
international experts, practitioners and 4. There is no fee to approach the NHRC.
other partners. Select the correct answer using the codes
• To be the instrument through which ideas below.
from outside are incorporated into policy- (a)  1 only (b)  1 and 4 only
making (c)  1, 3 and 4 only (d)  2 and 3 only
Q.1393 Which of the following are functions of NITI Solution: (b)
Aayog? Explanation: The NHRC cannot by itself
1) To formulate credible plans at the punish the guilty. That is the responsibility
village level of courts. The NHRC is there to make
2) Partnerships with National and independent and credible inquiry into any
International Think Tanks case of violation of human rights. Thus, its

Indian Polity Question Bank P.391

04-Indian Polity_Q1070-1506.indd 391 8/7/2018 7:42:43 PM


recommendations are not binding on neither Further, a sitting judge of the Supreme Court
the courts not the government. Statement 3 or a sitting chief justice of a high court can
is incorrect. be appointed only after consultation with the
The commission has been established by chief justice of India.
law as autonomous of the government, and The committee does not include the CJI.
not by the constitution, so statement 2 is So, D is the answer.
wrong, and 1 is correct. The chairman and members hold office
Any citizen of India can write a letter to for a term of five years or until they attain the
the NHRC to complain against the violation age of 70 years, whichever is earlier. After
of human rights. There is no fee or any their tenure, the chairman and members are
formal procedure to approach the NHRC. So, not eligible for further employment under the
the statement 4 is correct. Central or a state government
Q.1395 Consider the following statements about the Q.1397 The chairman and members of National
National Human Rights Commission (NHRC): Human Rights Commission (NHRC)  are
1. It is a non-constitutional and a non- appointed by the president on the
statutory body. recommendations of a six-member committee
2. NHRC cannot be dictated by the Union consisting of the
government while looking into cases of 1. Chief Justice of India as its head
human rights violations. 2. Speaker of the Lok Sabha
Which of these is/are true? 3. Leaders of the Opposition in both the
(a) Only 1 (b) Only 2 Houses of Parliament
(c) Both (d)  None of the above 4. Two renowned human rights activists
Solution: (b) Select the correct answer using the codes
NHRC works independently of the below.
Government. IT was established under (a)  2 and 3 only (b)  1, 3 and 4 only
a legislation enacted by the Parliament, (c)  1 and 4 only (d)  1, 2, 3 and 4
namely, the Protection of Human Rights Act, Solution: (a)
1993. The Act was amended in 2006 Please refer previous question

Appointment Composition
Q.1396 The chairman and members of National Human Q.1398 The NHRC is a multi-member body consisting
Rights Commission (NHRC)  are appointed of a chairman and four members. Apart from
by the president on the recommendations of a the full-time members, the commission also
six-member committee. The committee does has four ex-officio members which are
NOT include 1. Union Minister for Minority Affairs
(a) Deputy Chairman of the Rajya Sabha 2. Chairman of the National Commission for
(b)  Union Home Minister Minorities
(c) Leaders of the Opposition in both the 3. Chairman of the National Commission for
Houses of Parliament Women
(d)  Chief Justice of India 4. Chairperson, United Nations Human
Solution: (d) Rights Commission (UNHRC) South
Learning: The six-member committee Asia Division
consists of the Select the correct answer using the codes
• prime minister as its head below.
• Speaker of the Lok Sabha (a)  2 and 3 only (b)  2, 3 and 4 only
• Deputy Chairman of the Rajya Sabha (c)  1 and 4 only (d)  1, 2, 3 and 4
• leaders of the Opposition in both the Solution: (a)
Houses of Parliament Learning: The chairman should be a retired
• Central home minister chief justice of India, and members should

P.392 For Civil Services Preliminary Examination

04-Indian Polity_Q1070-1506.indd 392 8/7/2018 7:42:43 PM


be serving or retired judges of the Supreme 4. It is mandatory to appoint a woman as a
Court, a serving or retired chief justice of a member of the Commission.
high court and two persons having knowledge Which of the above statements are correct?
or practical experience with respect to human (a)  1, 2, 3 and 4 (b)  2 and 4
rights. (c)  1, 2 and 3 (d)  1 and 3
In addition to these full-time members, Solution: (c)
the commission also has four ex officio It is necessary to appoint women members in
members—the chairmen of the National National Commission for women and Child
Commission for Minorities, the National Welfare Committees etc. there is no such
Commission for SCs, the National provision in NHRC
Commission for STs and the National
Commission for Women. Removal
Q.1399 Consider the following statements about the Q.1402 The Chairman of NHRC can be removed on
composition of the NHRC: the same grounds and procedure as that of the
1. The Chairman of the NHRC must be a (a) Chairman, UPSC
retired Chief Justice of India. (b) CAG
2. The members can be serving judges in the (c) Member, Election Commission of India
Supreme Court. (d) Judge, High Court
3. The National Commission of women Solution: (a)
(NCW) is also represented in the NHRC. The president can remove the chairman
Which of these is/are correct? or any member from the office under the
(a) 1 and 2 (b) 2 and 3 following circumstances:
(c) 1 and 3 (d)  All of the above • If he is adjudged an insolvent; or
Solution: (d) • If he engages, during his term of office, in
Explained in previous question. any paid employment outside the
Q.1400 The Chairman and members of the National • duties of his office; or
Human Rights Commission (NHRC) should • If he is unfit to continue in office by reason
be of infirmity of mind or body; or
(a) Retired chief justice of India; and Serving • If he is of unsound mind and stand so
judges of the Supreme Court declared by a competent court; or
(b) Retired chief justice of India; and Serving • If he is convicted and sentenced to
or retired judges of the Supreme Court imprisonment for an offence.
(c) Serving chief justice of India; and In addition to these, the president can also
Serving or retired judges of the Supreme remove the chairman or any member on the
Court or Chief Justice of High Court ground of proved misbehaviour or incapacity.
(d) Retired chief justice of India; and However, in these cases, the president has to
Serving or retired judges of the Supreme refer the matter to the Supreme Court for
Court or Chief Justice of High Court; and an inquiry. If the Supreme Court, after the
persons having knowledge or practical inquiry, upholds the cause of removal and
experience with respect to human rights advises so, then the president can remove the
Solution: (d) chairman or a member.
Q.1401 Consider the following statements regarding Powers & Functions
the National Human Rights Commission of
India: Q.1403 The National Human Rights Commission is
1. Its Chairman must be a retired Chief authorized to
Justice of India. 1. Enquire suo motu in cases of human rights
2. It has formations in each state as State violation
Human Rights Commission. 2. Intervene in a proceeding involving
3. Its powers are only recommendatory in allegation of human rights pending before
nature. a court of India

Indian Polity Question Bank P.393

04-Indian Polity_Q1070-1506.indd 393 8/7/2018 7:42:43 PM


3. Take action against those guilty of human Select the correct answer using the codes
rights violation below.
4. Tender constitutional interpretations (a)  1 and 2 only (b)  3 and 4 only
of human rights provisions in the (c)  1, 3 and 4 only (d)  1, 2, 3 and 4
Constitution when requested by President Solution: (a)
Choose the correct answer using the codes Justification: Statement 1 & 2: explained in
below. previous question
(a)  1 and 2 only (b)  2 and 4 only Statement 3: It visits jails and detention
(c)  1 and 3 only (d)  2 and 3 only places to study the living conditions of
Solution: (a) inmates and make recommendation thereon.
The functions of the Commission are, inter It does not administer them.
alia: Statement 4: The commission is the
• To inquire into any violation of human watchdog of human rights in the country,
rights or negligence in the prevention of of rights guaranteed by the Constitution or
such violation by a public servant, either embodied in the international covenants
suo motu or on a petition presented to it or and enforceable by courts in India. It does
on an order of a court. not enter into such agreements with foreign
• To intervene in any proceeding involving nations.
allegation of violation of human rights Q.1405 Consider the following statements about the
pending before a court. powers of the NHRC:
• To visit jails and detention places to study Assertion (A): The NHRC can prosecute
the living conditions of inmates and make the persons found guilty for grave violation
recommendation thereon. of human rights.
• To review the constitutional and other Reason (R): It has all the powers of a
legal safeguards for the protection of regular court in India and its proceedings
human rights and recommend measures have a judicial character.
for their effective implementation. In the context of the statements above,
• To review the factors including acts of which of these is true?
terrorism that inhibit the enjoyment of (a) Both A and R are true and R is the correct
human rights and recommend remedial explanation for A.
measures. (b) Both A and R are true and R is not the
• To study treaties and other international correct explanation for A.
instruments on human rights and make (c)  A is incorrect but R is correct.
recommendations for their effective (d)  Both A and R are incorrect
implementation. Solution: (d)
Q.1404 The powers, functions and objectives of the It does not have powers of a regular court
National Human Rights Commission include but only that of a civil court. The basic
1. To look into allegations of excesses point to note is that it can only enquire and
of human rights independently of the recommend prosecution, not actually punish
government the offenders.
2. To intervene in any proceeding involving Q.1406 Consider that the NHRC has just now
allegation of violation of human rights completed inquired into a case of women
pending before a court in India trafficking in India due to gross neglect by a
3. To administer and supervise prisons and public servant. Which of the following actions
associated officials can be taken by it?
4. To enter into human right related 1. Imposing a punishment on the guilty
international agreements with foreign public servant
governments and such bodies on behalf of 2. recommending to the concerned
the government government or authority the initiation of
proceedings for prosecution

P.394 For Civil Services Preliminary Examination

04-Indian Polity_Q1070-1506.indd 394 8/7/2018 7:42:43 PM


3. approaching the Supreme Court or the Statement 4: The commission does not
high court concerned for the necessary have the power of prosecution. It can merely
directions, orders or writs make recommendations to the government or
Choose the correct answer using the codes recommend to the courts to initiate proceedings
below. based on the inquiry that it conducts.
(a)  1 and 2 only (b)  2 and 3 only Q.1408 The NHRC is vested with which of the
(c)  1 and 3 only (d)  All of the above following powers under the Protection of
Solution: (b) Human Rights Act, 1993?
The commission is not empowered to inquire 1. Suo motu inquiry
into any matter after the expiry of one year 2. Power to regulate its own procedure
from the date on which the act constituting 3. Powers of Civil Court
violation of human rights is alleged to have 4. Powers to utilise the services of any officer
been committed. In other words, it can or investigation agency of the Central
look into a matter within one year of its government or any state government
occurrence. The commission may take any Choose the correct answer using the codes
of the following steps during or upon the below.
completion of an inquiry: (a)  1 and 3 only (b)  2, 3 and 4 only
1. it may recommend to the concerned (c)  1 and 2 only (d)  All of the above
government or authority to make payment Solution: (d)
of compensation or damages to the victim; The commission’s headquarters is at Delhi
2. it may recommend to the concerned and it can also establish offices at other
government or authority the initiation of places in India. It is vested with the power
proceedings for prosecution or any other to regulate its own procedure. It has all the
action against the guilty public servant; powers of a civil court and its proceedings
3. it may recommend to the concerned have a judicial character. It may call for
government or authority for the grant of information or report from the Central and
immediate interim relief to the victim; state governments or any other authority
4. it may approach the Supreme Court or the subordinate thereto.
high court concerned for the necessary The commission has its own nucleus
directions, orders or writs. of investigating staff for investigation into
Q.1407 Consider the following about the National complaints of human rights violations.
Human Rights Commission (NHRC). Besides, it is empowered to utilise the services
1. It can be headed only by the serving Chief of any officer or investigation agency of the
Justice of India. Central government or any state government
2. It can start an inquiry at its own initiative for the purpose. It has also established
on a human rights violation issue. effective cooperation with the NGOs with
3. It undertakes and promoting research in first-hand information about human rights
the field of human rights. violations. NHRC can inquire into any
4. The commission does not have the power violation of human rights or negligence in
of prosecution. the prevention of such violation by a public
Select the correct answer using the codes servant, either suo motu or on a petition
below. presented to it or on an order of a court.
(a)  1 and 4 only Q.1409 The Human Rights (Amendment) Act, 2006
(b)  2 and 3 only brought which of these important changes
(c)  2, 3 and 4 only in the functioning and/or composition of the
(d)  1, 2, 3 and 4 only National Human Rights Commission?
Solution: (c) 1. It empowered the NHRC to transfer
Justification: Statement 1, 2 & 3: explained complaints received by it to the concerned
in previous questions the State Human Rights Commission
(SHRC).

Indian Polity Question Bank P.395

04-Indian Polity_Q1070-1506.indd 395 8/7/2018 7:42:43 PM


2. It declared that Chairperson of the Protection of Human Rights Act. The NHRC
National Commission for the Scheduled has limited jurisdiction in human rights
Castes shall be deemed to be a member of violations by the armed forces.
the NHRC.
Which of the above is/are correct? (58) SHRC – STATE HUMAN RIGHTS
(a)  1 only (b)  2 only
(c)  Both 1 and 2 (d)  None
COMMISSION
Solution: (c) Q.1411 Which of the following provisions ensure
Justification: It brought amendments in the the independence of the State Human Rights
Protection of Human Rights Act, 1993, relate Commission (SHRC) from the government?
to the following important issues (only few 1. After their tenure, the Chairman and
listed here): the members are not eligible for further
• Changing the eligibility condition for employment under both the Central and
appointment of member of SHRCs State Governments.
• Strengthening the investigative 2. The Chairman of the SHRC can be
machinery available with Human Rights removed in the same manner and grounds
Commissions as that of the High Court judges.
• Empowering the Commissions to Which of these is/are true?
recommend award of compensation, etc. (a) Only 1 (b) Only 2
even during the course of enquiry (c) Both (d)  None of the above
• Empowering the NHRC to undertake Solution: (a)
visits to jails even without intimation to For removing the members of the SHRC or
the state governments the NHRC, only an enquiry by the Supreme
• Enabling the NHRC to transfer complaints Court is required. The removal does not
received by it to the concerned SHRC require a special majority in the Parliament
• Providing that the Chairperson of the as is the case for high court judges.
National Commission for the Scheduled Giving the lure of employment in any of
Castes and the Chairperson of the the governments can hurt the independence
National Commission for the Scheduled of the commission. Hence, it protects their
Tribes shall be deemed to be members of independence.
the NHRC
• Enabling the Central Government to
notify future international covenants and
(59) CIC – CENTRAL INFORMATION
conventions to which the Act would be COMMISSION
applicable
Q.1412 Consider the following statements about the
Central Information Commission (CIC):
Human Rights Court 1. It is a body independent of the Union
Q.1410 Consider the following statements: government as far as its powers and
1. Lawfully, there should be a Human Rights functioning is concerned.
Court in every district of India. 2. It is a statutory body as it is formed under
2. The NHRC has limited jurisdiction in the provisions of the Right to Information
the cases of human rights violation even (RTI) Act , 2005.
by the Central Reserved Police Force 3. This body also deals with the complaints
(CRPF). pertaining to information sought under the
Which of these is/are true? RTI
(a)  Only 1 (b) Only 2 Act forwarded from the Union Territories.
(c)  Both (d)  None of the above Which of these is/are correct?
Solution: (c) (a) 1 and 2 (b) 2 and 3
CRPF is a para-military force. It is included (c) 1 and 3 (d) All of the above
in the definition of “armed forces” under the Solution: (d)

P.396 For Civil Services Preliminary Examination

04-Indian Polity_Q1070-1506.indd 396 8/7/2018 7:42:43 PM


Statement 2: RTI act, 2005 provides for the (b) body that comes under Ministry of
establishment of CIC at centre and state Information & Broadcasting, which inter
information commissioner at the state. Hence alia looks into the complaints forwarded to
it is statutory body. it by the Ministry and decide the appeals
Statement 1: CIC is independent body as (c) independent body that refers to
it is quasi-judicial body and its members can complaints only submitted to it by the
be removed only on the provisions provided Department of Public Grievances
in the act (d) quasi-judicial body that is supervised by
Statement 3: CIC is authorized under the one of the special benches of the Supreme
act to look into the complaints pertaining to Court, and works independently of the
information sought under the act like delay in Government of India
providing the information, charging exorbitant Solution: (a)
charges, providing false information etc. It is an independent body which inter alia
Q.1413 Consider the following statements. looks into the complaints made to it and
Assertion (A): The Central Information decide the appeals. It entertains complaints
Commission is not a constitutional body. and appeals pertaining to offices, financial
Reason (R): It has been established under institutions, public sector undertakings, etc.,
the provisions of the Right to Information under the Central Government and the Union
Act (2005). Territories.
In the context of the statements above, The Department of Administrative
which of these is true? Reforms and Public Grievances is the nodal
(a) A and R both are true, and R is the correct agency of the Government of India for
explanation for A. administrative reforms as well as redressal
(b) A and R both are true, and R is the NOT of public grievances relating to the states
the correct explanation for A. in general and those pertaining to Central
(c) A is correct, R is incorrect. Government agencies in particular.
(d) A and R both are incorrect.
Solution: (a) Appointment
The Central Information Commission (CIC) Q.1415 The members of the Central Information
set up under the Right to Information Act Commission are appointed by the President
is the authorised body, established in 2005, on the recommendation of a committee
under the Government of India to act upon consisting of
complaints from those individuals who have 1. Prime Minister
not been able to submit information requests 2. Chief Justice of India
to a Central Public Information Officer or 3. Chief Information Commissioner
State Public Information Officer due to Select the correct answer using the codes
either the officer not having been appointed, below.
or because the respective Central Assistant (a)  1 and 2 only (b)  2 and 3 only
Public Information Officer or State Assistant (c)  1 only (d)  1, 2 and 3
Public Information Officer refused to receive Solution: (c)
the application for information under the RTI Learning: They are appointed by the
Act. It was constituted through an Official President on the recommendation of a
Gazette Notification under the provisions of committee consisting of
the Right to Information Act (2005). Hence, 1. the Prime Minister as Chairperson
it is not a constitutional body. 2. the Leader of Opposition in the Lok Sabha
Q.1414 The Central Information Commission is a/an or single largest party
(a) high-powered independent body which 3. Union Cabinet Minister nominated by the
inter alia looks into the complaints made Prime Minister.
to it and decide the appeals

Indian Polity Question Bank P.397

04-Indian Polity_Q1070-1506.indd 397 8/7/2018 7:42:43 PM


They should be persons of eminence in public Information Commissioner from the office
life with wide knowledge and experience in under the following circumstances:
law, science and technology, social service, 1. if he is adjudged an insolvent; or
management, journalism, mass media or 2. if he has been convicted of an offence
administration and governance. which (in the opinion of the President)
Q.1416 Who among the following do NOT form part involves a moral turpitude; or
of the selection committee that recommends 3. if he engages during his term of office in
appointment of the Central Information any paid employment outside the duties of
Commissioner (CIC) and other members? his office etc.
(a)  Chief Justice of India In addition to these, the President can also
(b)  Speaker, Lok Sabha remove the Chief Information Commissioner
(c) Leader of Opposition in the Lok Sabha or any Information Commissioner on the
(d)  Both (b) and (c) ground of proved misbehaviour or incapacity.
Solution: (d) However, in these cases, the President has
Explained in previous answer to refer the matter to the Supreme Court for
an enquiry. If the Supreme Court, after the
Q.1417 Who among the following is eligible to become
enquiry, upholds the cause of removal and
the Chief Information Commissioner?
advises so, then the President can remove
(a)  Member of Parliament
him.
(b) Member of Board of Directors of a
Private bank
(c) A lawyer employed by a private company
Powers and Functions
(d)  None of the above is eligible. Q.1419 While inquiring, the Central Information
Solution: (d) Commission (CIC) may enjoy the powers
Justification: The CIC/IC shall not be a of a civil court. Which of these statement(s)
Member of Parliament or Member of the supports the above assertion?
Legislature of any State or Union Territory. 1. All public records asked for must be given
He shall not hold any other office of profit or to the Commission during inquiry for
connected with any political party or carrying examination.
on any business or pursuing any profession. 2. The RTI Act designates the CIC as a
Central tribunal in the discharge of its
Removal functions.
Which of the above is/are correct?
Q.1418 A news channel proves with conclusive
(a)  1 only (b)  2 only
evidence that a member of the Central
(c)  Both 1 and 2 (d)  None
Information Commission has indulged in
Solution: (a)
corrupt practices. The appeal for his removal
Justification: The CIC functions as a quasi-
reaches the Central government. How can the
judicial authority. But, the RTI Act does
member be removed?
not make it either a tribunal or a judicial
(a) The Prime Minister may end his contract
authority. So, the statement 2 is wrong.
on an enquiry conducted by the Cabinet
It has the powers of a civil court in several
Secretariat.
matters. It can also order suo motu inquiry.
(b) The Parliament passes an impeachment
During the inquiry of a complaint, the
motion by a simple majority.
Commission may examine any record which
(c) The Chief Information Commissioner
is under the control of the public authority
may remove his from his office.
and no such record may be withheld from it
(d) None of the above
on any grounds.
Solution: (d)
In other words, all public records must be
Learning: The President can remove the
given to the Commission during inquiry for
Chief Information Commissioner or any
examination.

P.398 For Civil Services Preliminary Examination

04-Indian Polity_Q1070-1506.indd 398 8/7/2018 7:42:43 PM


Q.1420 Consider the following statements. the President or eligible to be a
Assertion (A): The Commission can order Supreme Court Judge.
suo motu inquiry into matters related to RTI Choose the correct answer from the codes
Act if there are reasonable grounds. below.
Reason (R): The RTI Act 2005 declares (a)  1 and 2 only
the CIC to be a “Judicial authority”. (b)  2 and 3 only.
In the context of the above, which of these (c)  3 only
is correct? (d)  None of the above.
(a) A is correct, and R is an appropriate Solution: (d)
explanation of A. The Commission consists of a Chief
(b) A is correct, but R is not an appropriate Information Commissioner and not more
explanation of A. than ten Information Commissioners.
(c) A is correct, but R is incorrect. They are appointed by the President on the
(d) Both A and R are incorrect. recommendation of a selection committee.
Solution: (c) They should be persons of eminence
As explained in previous question in public life with wide knowledge and
Composition – qualification – appointment- experience in law, science and technology,
removal social service, management, journalism, mass
Q.1421 Consider the following statements about media or administration and governance.
the Composition of the Central Information They should not be a Member of Parliament
Commission (CIC); or Member of the Legislature of any State or
1. Union Home Minister is invariably Union Territory. They should not hold any
a member of the committee which other office of profit or connected with any
recommends the appointment of the CIC political party or carrying on any business or
to the President of India. pursuing any profession.
2. The RTI Act mentions the qualifications Q.1423 The Chief Information Commissioner
of the CIC. is appointed by a selection committee
3. The CIC must necessarily have been a consisting of
public servant served for a period of at least 1. Union Home Minister
10 years under the Government of India. 2. Union Law Ministry
Which of these is/are correct? 3. Chief Justice of India
(a) 1 and 2 (b) 2 and 3 4. Lok Sabha Speaker
(c) 1 and 3 (d) Only 2 5. Leader of Opposition, Rajya Sabha
Solution: (d) Choose the correct answer using the codes
The CIC should be a person of eminence below.
in public life with wide knowledge and (a)  1, 4 and 5 only (b)  All of the above
experience in law, science and technology (c)  None of the above (d)  2, 3 and 4 only
etc. He need not necessarily be a public Solution: (c)
servant. They are appointed by the President on the
Q.1422 Consider the following statements recommendation of a committee consisting
about the Central Information of the Prime Minister as Chairperson, the
Commission (CIC). Leader of Opposition in the Lok Sabha
1. The CIC consists of a Chairperson and a Union Cabinet Minister nominated
and two other members. by the Prime Minister. They should be
2. MPs and MLAs are eligible to persons of eminence in public life with wide
become members of the CIC. knowledge and experience in law, science
3. The Chairperson should be an and technology, social service, management,
eminent justice in the opinion of journalism, mass media or administration
and governance.

Indian Polity Question Bank P.399

04-Indian Polity_Q1070-1506.indd 399 8/7/2018 7:42:43 PM


Powers and Functions Choose the correct answer using the codes
below.
Q.1424 The Central Information Commission of (a)  1 and 2 only (b)  2 and 3 only
India performs which of the following kind (c)  1 and 3 only (d)  All of the above
of tasks? Solution: (d)
1. Quasi-judicial The Commission can order inquiry into
2. Delegated legislation any matter if there are reasonable grounds
3. Administrative (suo motu power). While inquiring, the
Choose the correct answer using the codes Commission has the powers of a civil court
below. in respect of the following matters:
(a)  1 and 2 only (b)  2 and 3 only • summoning and enforcing attendance of
(c)  1 and 3 only (d)  3 only persons and compelling them to give oral
Solution: (c) or written evidence on oath and to produce
It is a quasi-judicial body since it decides and documents or things; requiring the
disposes complaints relating to the RTI Act. discovery and inspection of documents;
It does not do delegated legislation because • receiving evidence on affidavit;
that would mean it is framing rules and • requisitioning any public record from any
regulations. It only enforces the existing rules court or office;
and regulations pertaining to the RTI act. • issuing summons for examination of
It is also an administrative body as it witnesses or documents; and
provides information services to citizens of • any other matter which may be prescribed.
India. During the inquiry of a complaint, the
Q.1425 Consider the following statements about the Commission may examine any record which
powers and functions of the CIC: is under the control of the public authority
1. The CIC can order suo motu inquiry into a and no such record may be withheld from
relevant matter on reasonable grounds. it on any grounds. In other words, all public
2. The CIC can admit a complaint where records must be given to the Commission
wrong or misleading information to the during inquiry for examination
appellant has been provided.
3. It is the duty of the CIC to admit a (60) SIC – STATE INFORMATION
complaint where the appellant has not COMMISSION
been able to submit an information
request because of non-appointment of Q.1427 Consider the following statements.
the information officer. Assertion (A): The State Information
Which of these is/are correct? Commissions are extra-statutory bodies.
(a) 1 and 2 (b) 2 and 3 Reason (R): States constitute Information
(c) 1 and 3 (d)  All of the above Commission by official gazette notifications.
Solution: (d) In the context of the above, which of these
Refer to the powers and functions of the CIC is correct?
in Laxmikanth’s previous edition. The RTI (a) A is correct, and R is an appropriate
Act mentions the powers and functions of the explanation of A.
CIC. (b) A is correct, but R is not an appropriate
Q.1426 Consider the following about the power explanation of A.
of inquiry of the Central Information (c) A is incorrect, but R is correct.
Commission. The CIC (d)  Both A and R are incorrect.
1. can exercise suo motu inquiry powers Solution: (c)
2. has powers similar to a civil court during Justification: The Right to Information
inquiry Act of 2005 provides for the creation of not
3. may examine any record which is under only the Central Information Commission
the control of the public authority but also a State Information Commission

P.400 For Civil Services Preliminary Examination

04-Indian Polity_Q1070-1506.indd 400 8/7/2018 7:42:43 PM


at the state level. Accordingly, all the states In addition to these, the Governor can
have constituted the State Information also remove the State Chief Information
Commissions through Official Gazette Commissioner or any State Information
Notifications. Commissioner on the ground of proved
Learning: The State Information misbehaviour or incapacity. However, in
Commission is a high-powered independent these cases, the Governor has to refer the
body which inter alia looks into the matter to the Supreme Court for an enquiry. If
complaints made to it and decide the appeals. the Supreme Court, after the enquiry, upholds
It entertains complaints and appeals the cause of removal and advises so, then the
pertaining to offices, financial institutions, Governor can remove him.
public sector undertakings, etc. under the
concerned state government. (61) CVC – CENTRAL VIGILANCE
Q.1428 The Chief State Information Commissioner COMMISSION
can be removed by
(a) the State Governor after an enquiry by Q.1429 Which of the following is/are limitations on
the Supreme Court upholds the cause of the authority and power of Central Vigilance
removal Commission (CVC)?
(b) the State Governor after an enquiry by 1. CVC is only an advisory body.
the High Court upholds the cause of 2. CBI is under the administrative control of
removal CVC, but it cannot direct CBI to initiate
(c) the President after an enquiry by the any inquiry on its own.
Supreme Court upholds the cause of 3. CVC does not have powers to register
removal criminal case.
(d) the President after an enquiry by a 4. CVC is appointed by the Prime Minister
Parliamentary Committee upholds the without consulting any collegium as in
cause of removal case of other autonomous bodies.
Solution: (a) Select the correct answer using the codes
The State Chief Information Commissioner below.
and a State Information Commissioner hold (a)  1 and 3 only (b)  2 and 3 only
office for a term of 5 years or until they attain (c)  1, 2 and 3 only (d)  1, 2 and 4 only
the age of 65 years, whichever is earlier. They Solution: (a)
are not eligible for reappointment. Justification: Statements 1 and 2: Central
The Governor can remove the State Chief Government Departments are free to either
Information Commissioner or any State accept or reject CVC’s advice in corruption
Information Commissioner from the cases. It is only an advisory body. So, 1 is
office under the following circumstances: correct.
1. If he is adjudged an insolvent; or Statement 2: The CVC has supervisory
2. if he has been convicted of an offence powers over CBI. However, the CVC does
which (in the opinion of the Governor) not have the power to call for any file from
involves a moral turpitude; or CBI or to direct CBI to investigate any case
3. if he engages during his term of office in in a particular manner.
any paid employment outside the duties of CBI is under administrative control
his office; or of Department of Personnel and Training
4. if he is (in the opinion of the Governor) (DoPT). This means that, the powers to
unfit to continue in office due to infirmity appoint, transfer, suspend CBI officers lie
of mind or body or with DoPT. So, the statement 2 is incorrect.
5. if he has acquired such financial or other
interest as is likely to affect prejudicially
his official functions.

Indian Polity Question Bank P.401

04-Indian Polity_Q1070-1506.indd 401 8/7/2018 7:42:43 PM


Appointment Choose the correct answer using the codes
below.
Q.1430 The Central Vigilance Commissioner and the (a)  1 and 2 only (b)  2 and 3 only
Vigilance Commissioners are appointed by the (c)  1 and 3 only (d)  1 only
President after obtaining the recommendation Solution: (d)
of a Committee consisting of Explained in previous question.
(a) Prime Minister of India, Minister of
Home Affairs, Speaker and Chairman of Removal
Lok Sabha and Rajya Sabha and Senior
judges of Supreme Court Q.1432 Consider the following procedures.
(b) Prime Minister of India, Speaker of Lok 1. President referring the matter to the Lok
Sabha and Senior judges of Supreme Sabha.
Court 2. Inquiry by the Supreme Court.
(c) Minister of Home Affairs, Leader of 3. Parliament passes the motion by a simple
Opposition Lok Sabha and Chief Justice majority in both the houses.
of India Which of the following procedures are
(d) Prime Minister of India, Minister of followed in the removal of CVC. Choose
Home Affairs and Leader of Opposition the order that is chronological from the ones
Lok Sabha given below.
Solution: (d) (a) 123 (b) 213
Learning: The Commission consists (c)  2 only (d)  13 only
of a Central Vigilance Commissioner Solution: (c)
(Chairperson) and not more than two The president can also remove the Central
Vigilance Commissioners (members). Vigilance Commissioner or any vigilance
The Central Vigilance Commissioner commissioner on the ground of proved
and the Vigilance Commissioners misbehaviour or incapacity. However, in
shall be appointed by the President on these cases, the president has to refer the
recommendation of a Committee consisting matter to the Supreme Court for an enquiry.
of If the Supreme Court, after the enquiry,
1. the Prime Minister (Chairperson) upholds the cause of removal and advises
2. the Minister of Home Affairs (Member) so, then the president can remove him. He is
3. Leader of the Opposition in the House of deemed to be guilty of misbehaviour, if he
the People (Member). (a) is concerned or interested in any contract
The term of office of the Central Vigilance or agreement made by the Central
Commissioner and the Vigilance government, or
Commissioners would be four years from the (b) participates in any way in the profit of
date on which they enter their office or till such contract or agreement or in any
they attain the age of 65 years, whichever is benefit or emolument arising there from
earlier. otherwise than as a member and in
The Commission, while conducting the common with the other members of an
inquiry, shall have all the powers of a Civil incorporated company
Court with respect to certain aspects.
Q.1431 Consider the following statements about the
Powers & Functions
appointment of CVC. Q.1433 The Government of India has authorized
1. He is appointed by the President. which of the following as the “Designated
2. The recommendation of the nominee Agency” to receive written complaints for
of the post comes from the Council of disclosure on any allegation of corruption or
Ministers. misuse of office?
3. Appointment of CVC cannot be (a)  National Investigation Agency
challenged in court. (b)  Central Bureau of Investigation

P.402 For Civil Services Preliminary Examination

04-Indian Polity_Q1070-1506.indd 402 8/7/2018 7:42:43 PM


(c)  Central Vigilance Commission Q.1435 Which of the following powers/
(d)  Anti-Corruption Bureau responsibilities are exercised by CVC?
Solution: (c) 1. Part of the Selection Committee in the
Learning: In 2004, the Government of India appointment of the Director, CBI
authorised the CVC as the “Designated 2. Instituting inquiries against Union and
Agency” for the above. It can also recommend State Ministers
appropriate action. 3. Superintendence of Police in matters of
The CVC is conceived to be the apex corruption
vigilance institution, free of control from any Choose the correct answer using the codes
executive authority, monitoring all vigilance below.
activity under the Central Government (a)  1 and 2 only
and advising various authorities in Central (b)  2 and 3 only
Government organisations in planning, (c)  None of the above
executing, reviewing and reforming their (d)  1 only
vigilance work. Solution: (c)
Q.1434 Consider the following statements about the The amended Delhi Special Police
Central Vigilance Commission (CVC). Establishment Act empowers a committee to
1. It has been authorized as the designated appoint the director of CBI. The committee
agency to receive written complaints for consists the following people:
disclosure on allegations of corruption in • Prime Minister – chairperson
Central government. • Leader of Opposition – member
2. It has been empowered to take appropriate • Chief Justice of India or a Supreme Court
disciplinary action including suspension Judge recommended by the Chief Justice
against erring public servants in cases of – member
misuse of office. When making recommendations, the committee
Which of the above is/are correct? considers the views of the outgoing director.
(a)  1 only (b)  2 only Above Selection committee was constituted
(c)  Both 1 and 2 (d)  None under The Lokpal and Lokayuktas Act,
Solution: (a) 2013. Before this central vigilance
Justification: Statements 1 and 2: In 2004, the commissioner, under CVC act, had this
Government of India authorised the CVC as power.
the “Designated Agency” for the above. But, Police comes under the State governments.
it can only recommend appropriate action, and CVC is concerned only with the Central
not take it on behalf of the government. government and its authorities
Learning: The CVC is conceived to be Q.1436 Consider the following statements about the
the apex vigilance institution, free of control Central Vigilance Commission (CVC):
from any executive authority, monitoring 1. The chairperson of the CVC is appointed
all vigilance activity under the Central by the President on the recommendation
Government and advising various authorities of the Union Cabinet.
in Central Government organisations in 2. The jurisdiction of CVC extends to the
planning, executing, reviewing and reforming civil servants of the Centre as well as the
their vigilance work. states.
• CVC has the responsibility to exercise Which of these is/are true?
superintendence over the functioning (a) Only 1 (b) Only 2
of Delhi Special Police Establishment (c) Both (d)  None of the above
(which is a part of Central Bureau of Solution: (d)
Investigation) in so far as it relates to the Its jurisdiction extends to only some selected
investigation of offences alleged to have Central government employees and the all-
been committed under the Prevention of India services.
Corruption Act, 1988.

Indian Polity Question Bank P.403

04-Indian Polity_Q1070-1506.indd 403 8/7/2018 7:42:43 PM


(62) CBI – CENTRAL BUREAU OF India (GoI) have a bearing on the Central
Bureau of Investigation (CBI)?
INVESTIGATION 1. Ministry of Law and Justice
Q.1437 The Central Bureau of Investigation (CBI) is 2. Ministry of Home Affairs
under the administrative control of 3. CVC
(a) Ministry of Home Affairs 4. UPSC
(b) Department of Personnel and Training 5. Department of Personnel and Training
(DoPT) Choose the correct answer using the codes
(c)  Department of Economic Affairs below.
(d) Central Vigilance Commission (CVC) (a)  1, 2 and 3 only
Solution: (b) (b)  All of the above
Justification: Option D: CVC has (c)  None of the above
supervisory powers over CBI. However, (d)  1, 3 and 5 only
CVC does not have the power to call for any Solution: (b)
file from CBI or to direct CBI to investigate The CBI is headed by a director, an IPS
any case in a particular manner. officer with a rank of Director General of
Option B: It is under administrative Police or Commissioner of Police (State).
control of DoPT. This means that, the powers The director is selected based on the Lokpal
to appoint, transfer, suspend CBI officers lie and Lokayuktas Act, 2013, and has a two-
with DoPT. year term.
The CBI is subject to five ministries of the
Q.1438 Consider the following statements.
Government of India -
Assertion (A): The Central Bureau of
• Ministry of Home Affairs: Cadre clearance
Investigation (CBI) is a statutory body.
• DoPT: Administration, budget and
Reason (R): CBI derives its powers from
induction of non IPS officers
the Delhi Special Police Establishment Act
• Union Public Service Commission:
(DSPE), 1946.
Officers above the rank of Deputy SP
In the context of the above, which of these
• Law and Justice Ministry: Public
is correct?
prosecutors
(a) A is correct, and R is an appropriate
• Central Vigilance Commission: Anti-
explanation of A.
corruption cases
(b) A is correct, but R is not an appropriate
explanation of A.
(c) A is incorrect, but R is correct.
Appointment
(d) Both A and R are incorrect. Q.1440 The Central Bureau of Investigation
Solution: (c) (CBI) Director is selected by a collegium
Justification: To be a statutory body, it must comprising
have been established by the DSPE Act. It 1. Prime Minister
was instead established through an executive 2. Chief Justice of India
resolution. 3. Speaker, Lok Sabha
It was set up in 1963 by a resolution of the 4. Union Minister of Home Affairs
Ministry of Home Affairs. 5. Leader of Opposition, Lok Sabha
Later, it was transferred to the Ministry of Select the correct answer using the codes
Personnel and now it enjoys the status of an below.
attached office. (a)  1, 2, 3 and 4 only
The CBI is the main investigating agency (b)  1, 2 and 5 only
of the Central Government. It plays an (c)  2, 3, 4 and 5 only
important role in the prevention of corruption. (d)  1, 3, 4 and 5 only
Q.1439 Which of the following ministries/ Solution: (b)
departments/organs of the Government of Justification: It is headed by Director, an IPS
officer of DGP rank. He is selected as per the

P.404 For Civil Services Preliminary Examination

04-Indian Polity_Q1070-1506.indd 404 8/7/2018 7:42:43 PM


provisions of Central Vigilance Commission • Combating corruption in public life,
(CVC) Act, 2003. curbing economic and violent crimes
The CBI Director is selected by a through meticulous investigation and
collegium comprising prosecution
• Prime Minister: chairperson • Evolving effective systems and
• Leader of Opposition: member procedures for successful investigation
• Chief Justice of India or a Supreme Court and prosecution of cases in various law
Judge recommended by the Chief Justice courts
– member • Helping fight cyber and high technology
When making recommendations, the crime
committee considers the views of the • Creating a healthy work environment
outgoing director. that encourages team-building, free
Above Selection committee was communication and mutual trust
constituted under The Lokpal and Lokayuktas • Supporting state police organisations and
Act, 2013. law enforcement agencies in national and
• CBI is the premier investigative police international cooperation, particularly
agency of India. It derives power to relating to enquiries and investigation of
investigate from Delhi Special Police cases
Establishment Act, 1946. • Playing a lead role in the war against
• The appointment of CBI Director has national and transnational organized
been quite contentious after SC’s “caged crime
parrot” comments on CBI. • Upholding human rights, protecting the
The committee that selects the CBI Director environment, arts, antiques and heritage
does NOT include of our civilisation
(a) Prime Minister of India • Developing a scientific temper, humanism
(b) Leader of the single largest opposition and the spirit of inquiry and reform
party • Striving for excellence and professionalism
(c) Chief justice of India or his nominee in all spheres of functioning so that
(d) Speaker, Lok Sabha the organisation rises to high levels of
Solution: (d) endeavour and achievement.
Some of the important CBI divisions are:
Functions • Anti-Corruption Division
• Economic Offences Division
Q.1441 The Central Bureau of Investigation (CBI)
• Special Crimes Division
can be handed over cases pertaining to
• Policy and International Police
1. National and transnational organised
Cooperation Division
crime
2. Protection of the environment Q.1442 The CBI is the main investigating agency of
3. Conservation and protection of arts, the Central Government playing an important
antiques and heritage of India role in maintaining integrity in administration
4. Offences of Economic and Cyber domains handling a variety of cases. Which of the
Select the correct answer using the codes following types of cases can NOT be referred
below. to the CBI?
(a)  1 and 4 only (b)  2 and 3 only (a)  Cyber Crime
(c)  1, 2 and 3 only (d)  1, 2, 3 and 4 (b)  Transnational organised crime
Solution: (d) (c)  Economic Offences
Learning: Based on its motto, mission (d)  All of the above can be referred.
and the need to develop professionalism, Solution: (d)
transparency, adaptability to change and use Explained in previous question
of science and technology in its working, the
CBI focuses on

Indian Polity Question Bank P.405

04-Indian Polity_Q1070-1506.indd 405 8/7/2018 7:42:44 PM


(63) LOKPAL & LOKAYUKTA • He cannot be reappointed as chairperson /
member of Lokpal.
Q.1443 Assertion (A): Lokpal and Lokayuktas are • Cannot take any diplomatic assignment
constitutional bodies that have been given the • Cannot be appointed as administrator to a
status of Supreme Court. Union Territory
Reason (R): All members of Lokpal and • Any constitutional / statutory post in
Lokayuktas must be Judicial members, who which appointment is made by President
are appointed by the Chief Justice of India. • Any other office under the government of
In the context of the above, which of these India
is correct? • He cannot contest any of the elections
(a) A is correct, and R is an appropriate such as President / Vice President / MLA
explanation of A. / MLC/ Local bodies for 5 years after
(b) A is correct, but R is not an appropriate relinquishing the post.
explanation of A.
(c) A is correct, but R is incorrect. Appointment
(d) Both A and R are incorrect.
Q.1445 The selection of chairperson and members
Solution: (d)
of Lokpal shall be through a selection
Justification: Lokpal will consist of a
committee consisting of
chairperson and a maximum of eight
1. Prime Minister
members, of which 50 per cent shall be
2. Speaker of Lok Sabha
judicial members. So, R is wrong.
3. Chairman, Rajya Sabha
The selection of chairperson and members
4. Chief Justice of India or a sitting Supreme
of Lokpal shall be through a selection
Court judge nominated by CJI
committee consisting of Prime Minister,
5. An Eminent jurist to be nominated
Speaker of Lok Sabha, Leader of Opposition
by the President of India on the basis
in the Lok Sabha, Chief Justice of India or
of recommendations of the first four
a sitting Supreme Court judge nominated
members of the selection committee
by CJI, eminent jurist to be nominated
Select the correct answer using the codes
by the President of India on the basis of
below.
recommendations of the first four members
(a)  3 and 5 only
of the selection committee.
(b)  1, 2, 3 and 4 only
(c)  1, 2 and 4 only
Lokpal (d)  1, 2, 4 and 5
Q.1444 Which of the following provisions of the Solution: (d)
Lokpal and Lokayuktas Act 2013 ensure the Learning: Lokpal will consist of a chairperson
independence of the Lokpal, after he ceases and a maximum of eight members, of which
to hold his post? 50 per cent shall be judicial members.
1. He cannot be reappointed. The selection of chairperson and members
2. He can never contest the elections of of Lokpal shall be through a selection
President and Vice-President. committee consisting of
3. He cannot be appointed as an administrator 1. Prime Minister,
of a Union Territory (UT). 2. Speaker of Lok Sabha
Choose the correct answer using the codes 3. Leader of Opposition in the Lok Sabha,
below: 4. Chief Justice of India or a sitting Supreme
(a) 1 and 2 (b) 2 and 3 Court judge nominated by CJI
(c) 1 and 3 (d)  All of the above 5. Eminent jurist to be nominated by
Solution: (c) the President of India on the basis
Once a Lokpal chairperson / member has of recommendations of the first four
ceased to be so, he cannot take up the members of the selection committee.
following jobs:

P.406 For Civil Services Preliminary Examination

04-Indian Polity_Q1070-1506.indd 406 8/7/2018 7:42:44 PM


Eligibility 2. Former administrators in the government
of India can be appointed as members of
Q.1446 Who amongst the following is/are not eligible the Lokpal.
to be a member of the Lokpal? Which of these is/are true?
1. A person who is affiliated to a political party. (a) Only 1 (b) Only 2
2. Carries on some business/profession. (c) Both (d)  None of the above
3. MPs and MLAs Solution: (b)
Choose the correct answer using the codes The person who is to be appointed as the
below: chairperson of the Lokpal should be either of
(a) 1 and 2 (b) 2 and 3 the following:
(c) 1 and 3 (d)  All of the above • Either the former Chief Justice of India
Solution: (d) • Or the former Judge of Supreme Court
The following persons cannot become • Or an eminent person with impeccable
chairperson of Lokpal: integrity and outstanding ability, having
• MPs and MLAs special knowledge and expertise of
• Persons convicted of any offense minimum 25 years in the matters
involving moral turpitude relating to anti-corruption policy, public
• Less than 45 years of age administration, vigilance, finance
• Members of Panchayats or Municipality including insurance and banking, law and
• A person who was removed or dismissed management.
from the public service
• A person who holds any office of trust/ Removal
profit; if so, he would need to resign from
Lokpal Q.1449 With reference to the Lokpal and Lokayuktas
• A person who is affiliated to a political party act, 2013, consider the following statements
• Carries on some business / profession; if 1. The Chairperson and members of the
so, he would need to quit some business Lok Pal may be removed on grounds of
misbehaviour by an order of the President
Composition on the basis of the inquiry made by the
Selection committee.
Q.1447 Consider the following about the composition 2. Lokpal will have power of superintendence
of the Lokpal at the Central Level. and direction over any investigation
1. The 50 per cent of the members shall be agency including CBI for cases referred
judicial members. to them by Lokpal.
2. Seats are reserved for minority and 3. 50 per cent of members of Lokpal shall be
women in Lokpal members. from SC/ST/OBCs, minorities and women.
3. One member will be an eminent jurist Which of the above statements is/are correct?
nominated by the President. (a)  1 and 2 only (b)  1 and 3 only
Choose the correct answer using the codes (c)  2 and 3 only (d)  1, 2 and 3
below. Solution: (c)
(a)  1 and 2 only (b)  2 and 3 only Removal of Lokpal
(c)  All of the above (d)  3 only The Chairperson and members of the Lokpal
Solution: (c) may be removed on grounds of misbehaviour
Explained in previous question by an order of the President. The President
Q.1448 Consider the following statements about the may make a reference to the Supreme Court
composition of the Lokpal in India as per the to enquire into the Lokpal’s conduct (a) on
recently passed Lokpal and Lokayuktas Act, his own, (b) on the basis of a petition signed
2013: by at least 100 MPs or (c) if he is satisfied
1. The chairperson of the Lokpal must have with a petition by a citizen. The President
been either the Chief Justice of India or a may issue an order of removal on the basis of
former judge of Supreme Court. the inquiry made by the Supreme Court

Indian Polity Question Bank P.407

04-Indian Polity_Q1070-1506.indd 407 8/7/2018 7:42:44 PM


Jurisdiction Such inquiry against the Prime Minister will
be done in camera. If the Lokpal concludes
Q.1450 Consider the following statements about the that the allegation is false and the inquiry
jurisdiction of the Lokpal: should be dismissed, the records of the
1. It can take up cases of inquiry against inquiry shall not be published or made
the Prime Minister barring under a few available to anyone.
conditions.
Q.1451 Who/Which among the following comes
2. Any person who is involved in giving or
under the jurisdiction of the Lokpal?
taking bribe will come under its purview.
1. Prime Minister
3. All Ministers of the Union would come
2. All categories of public servants.
under its purview.
3. Entities receiving donation from foreign
Which of these is/are correct?
sources
(a) 1 and 2 (b) 2 and 3
Choose the correct answer using the codes
(c) 1 and 3 (d) All of the above
below.
Solution: (d)
(a)  1 and 2 only (b)  2 and 3 only
The following come under the jurisdiction of
(c)  All of the above (d)  3 only
Lokpal:
Solution: (c)
• Prime Minister of India, under certain
Following in the jurisdiction of Lokpal;
conditions as stipulated in the adjacent
Prime Minister has been brought under
box.
the purview of the Lokpal. The Lokpal’s
• All ministers of the Union
jurisdiction will cover all categories of public
• Members of Parliament except for matters
servants. All entities receiving donations
related to article 105 of constitution. (that
from foreign source in the context of the
is anything said or a vote given by him in
Foreign Contribution Regulation Act (FCRA)
Parliament)
in excess of Rs. 10 lakh per year are brought
• Group ‘A’ or Group ‘B’ officers
under the jurisdiction of Lokpal.
Group ‘C’ or Group ‘D’ officials
• Provides adequate protection for honest
• Any person who is or has been in-charge
and upright public servants.
(director / manager/ secretary) of anybody
• Lokpal will have power of superintendence
/ society set up by central act or any other
and direction over any investigation
body financed / controlled by central
agency including CBI for cases referred
government.
to them by Lokpal.
• Any other person involved in act of
abetting, bribe giving or bribe taking
According to the Lokpal and Lokayukta Act
Lokayukta
2013, the Prime Minister comes within the Q.1452 Consider the following about the institution
jurisdiction of Lokpal but Lokpal will not of Lokayukta.
inquire the Prime Minister if the allegation of 1. A state is not legally obliged to create the
corruption is related to international relations, institution of Lokayukta.
external and internal security, public order, 2. Powers of Lokayukta vary from state to
atomic energy and space. Further, allegation state.
against Prime Minister can be taken up for 3. Only a serving or retired Supreme Court
inquiry only when the two conditions as judge can be appointed as a Lokayukta.
follows are satisfied: 4. A Lokayukta can also investigate cases
• Full bench of the Lokpal consisting of its pertaining to Central civil servants.
Chairperson and all Members considers Choose the correct answer using the codes
the initiation of inquiry below.
• At least two-thirds of its members (a)  1 and 4 only (b)  2 and 3 only
approves of such inquiry (c)  2 only (d)  1, 2 and 3 only
Solution: (c)

P.408 For Civil Services Preliminary Examination

04-Indian Polity_Q1070-1506.indd 408 8/7/2018 7:42:44 PM


http://www.thehindu.com/news/national/ Miscellaneaus
c o r r u p t i o n - c h a rg e s - d e n t - k a r n a t a k a -
lokayuktasimage/article7371954.ece Q.1454 With reference to the role of CBI, CVC and
A mandate for setting up of the institution Lok Pal, consider the following statements:
of Lokayukta through enactment of a law by 1. Both the CVC and Lokpal will have
the State Legislature within a period of 365 jurisdiction over Group ‘A’ officers.
days from the date of commencement of the 2. Transfer of officers of CBI investigating
Act. The Lokayukta is an anti-corruption cases referred by Lokpal should be done
authority orombudsman (An ombudsman is with the approval of Lokpal
an official, appointed by the government or Which of the above statements is/are correct?
by parliament to represent the interests of the (a)  1 Only (b)  2 Only
public). He works along with the Income Tax (c)  1 and 2 (d)  None
Department and the Anti-Corruption Bureau. Solution: (c)
The Lokayukta (sometimes referred to the The Lokpal shall have jurisdiction over
institution itself) investigates allegations public servants categorised as Group
of corruption and mal-administration ‘A’ officers. Presently, these officers fall
against public servants and is tasked with under the purview of the Central Vigilance
speedy redressal of public grievances. The Commission (CVC). Therefore, both the
Lokayukta is usually a former High Court CVC and Lokpal will have jurisdiction over
Chief Justice or former Supreme Court judge Group ‘A’ officers.
and has a fixed tenure. Lokayukta’s power Transfer of officers of CBI investigating
varies from State to State. In some States, cases referred by Lokpal with the approval
the Lokayukta enquires into allegations of Lokpal.
against public functionaries including Chief
Minister, Ministers and MLAs. While
some has the power to investigate into civil
servants/bureaucrats, judiciary and police.
Q.1453 Consider the following statements about the
Lokpal and Lokayuktas Act, 2013:
1. As per the Act, all states in India must
create the institution of Lokayukta.
2. The Act also prescribes the powers,
jurisdiction and method of appointment of
the Lokayukta.
Which of these is/are correct?
(a) Only 1 (b) Only 2
(c) Both (d) None of the above
Solution: (a)
There is only one section on Lokayukta in the
act which says that within one year, the states
shall enact the Lokayukta act. However,
there is nothing regarding their composition,
powers etc. In fact, states are free to define
how their own Lokayuktas would be
appointed, how they would work and under
what circumstances they would serve.

Indian Polity Question Bank P.409

04-Indian Polity_Q1070-1506.indd 409 8/7/2018 7:42:44 PM


OTHER CONSTITUTIONAL created states on linguistic basis rejected the
DIMENSIONS theory ‘one language-one state’.

Official Languages of Union


(64) OFFICIAL LANGUAGE –
PART XVII – ARTICLES 343 to 351 Q.1456 The use of English language for all official
purposes of the Union and also for the
Q.1455 Consider the following statements: transaction of business in Parliament
1. Indian Constitution has given the status of is a/an
national language to Hindi. (a) Convention
2. Seventh Schedule contains the languages (b)  Statutory requirement
recognized by Indian Constitution. (c) Rule framed by the Central Government
3. Languages in India are grouped as (d)  Informal agreement
scheduled and non-scheduled languages. Solution: (b)
4. In India, states have been created on Learning: Articles 343-351 of the
the linguistic basis with the theory ‘one constitution deal with the official language
language- one state’. of the Union. It empowers the Parliament
Which of the statements is/are correct? to provide for the continued use of English
(a) 3 Only (b) 2 and 3 language even after 1965. Accordingly,
(c) 1, 2 and 4 (d) 1, 2 and 3 Parliament enacted the Official Languages
Solution (a) Act 1963.
Indian Constitution did not give the status The act provides for the continued use
of national language to any one language. of English (even after 1965), in addition to
Hindi was identified as the official language. Hindi, for all official purposes of the Union
But Hindi is the mother tongue of only and also for the transaction of business in
about 40 per cent of Indians. Therefore, Parliament.
there were many safeguards to protect other Q.1457 Consider the following statements:
languages. 1. The Constitution prescribes the continued
Besides Hindi, there are 21 other languages use of English as the official language
recognized as Scheduled Languages by of the Union even after 15 years of its
the Constitution (Eighth Schedule contains constitution.
a list of 22 languages recognized by the 2. The Constitution does not specify the
constitution). Of the 22 languages that official language of different states.
are included in the Eighth Schedule of the Which of these is/are correct?
Indian constitution and are therefore called (a) Only 1 (b) Only 2
‘Scheduled Languages’. Others are called (c) Both (d)  None of the above
‘non-scheduled Languages’. Solution: (b)
According to the Constitution, the use of The continued use of English was prescribed,
English for official purposes was to stop in not by the constitution, but by the Official
1965. But the Central Government continues Languages Act, 1963.
to use of English along with Hindi for official The legislatures of the states decide their
purposes. Promotion of Hindi continues official language. The constitution is silent on
to be the official policy of the Government deciding their official languages.
of India. Promotion does not mean that the Q.1458 Consider the following about the use of
Central Government can impose Hindi language(s) in Parliament.
on States where people speak a different Assertion (A): A member of the house cannot
language. address the House in his mother tongue.
The States reorganization Commission Reason (R): The Constitution has
under the chairmanship of Fazl Ali, which declared Hindi and English to be the

P.410 For Civil Services Preliminary Examination

04-Indian Polity_Q1070-1506.indd 410 8/7/2018 7:42:44 PM


languages for transacting business in the like Meghalaya, Arunachal Pradesh and
Parliament. Nagaland have adopted English. Notably,
In the context of the above, which of these the choice of the state is not limited to the
is correct? languages enumerated in the Eighth Schedule
(a) A is correct, and R is an appropriate of the Constitution.
explanation of A. Q.1460 Consider the following statements:
(b) A is correct, but R is not an appropriate Assertion (A): The states have to choose their
explanation of A. official language only from the list given in
(c) A is correct, but R is incorrect. the Eighth Schedule of the constitution.
(d)  A is incorrect, but R is correct. Reason (R): The Eighth Schedule
Solution: (d) mentions all the constitutionally recognized
Learning: Hindi and English are to be the languages.
languages for transacting business in the In the context of the statements above,
Parliament as per the constitution. However, which of these is true?
the presiding officer can permit a member to (a) Both A and R are true and R is the correct
address the House in his mother tongue. In explanation for A.
both the Houses, arrangements are made for (b) Both A and R are true and R is not the
simultaneous translation. correct explanation for A.
(c)  A is incorrect but R is correct.
Official Languages of State (d)  A is correct but R is incorrect.
Q.1459 In deciding the official language of a State, Solution: (c)
the choice of the state is The choice of the states in choosing their
(a) Limited to the languages mentioned in official language is not limited to the 8th
the Eighth Schedule of the Constitution schedule.
(b) Limited to the languages officially Q.1461 Consider the following statements.
recognized by the State Government 1. The Constitution does not specify the
(c) Either an aboriginal language or a official language of different states.
classical language or English/Hindi 2. The choice of the state is not limited
(d) Not limited; any language can be choose. to the languages enumerated in the
Solution: (d) Constitution.
The legislature of a state may adopt any one 3. States may have more than one official
or more of the languages in use in the state language, but in such a case Hindi must
or Hindi as the official language of that state. be one of them.
Under this provision, most of the states Select the correct answer using the codes
have adopted the major regional language as below.
their official language. For example, Andhra (a)  1 only (b)  1 and 2 only
Pradesh has adopted Telugu, Kerala— (c)  2 and 3 only (d)  1 and 3 only
Malayalam, Assam—Assamese, West Solution: (b)
Bengal—Bengali, Odisha—Odia. The nine As explained in previous questions
northern states of Himachal Pradesh, Uttar Q.1462 Consider the following statements.
Pradesh, Uttarakhand, Madhya Pradesh, Assertion (A): Official language of every
Chhattisgarh, Bihar, Jharkhand, Haryana and state is confined to the languages listed in the
Rajasthan have adopted Hindi. Eighth Schedule of the Constitution.
Gujarat has adopted Hindi in addition to Reason (R): The Constitution specifies the
Gujarati. official language of different states.
Similarly, Goa has adopted Marathi in In the context of the above, which of these
addition to Konkani Jammu and Kashmir is correct?
has adopted Urdu (and not Kashmiri). On (a) A is correct, and R is an appropriate
the other hand, certain north-eastern States explanation of A.

Indian Polity Question Bank P.411

04-Indian Polity_Q1070-1506.indd 411 8/7/2018 7:42:44 PM


(b) A is correct, but R is not an appropriate 347 Special provision relating
explanation of A. to language spoken by a
(c) A is correct, but R is incorrect. section of the population
(d) Both A and R are incorrect. of state
Solution: (d)
Explained in previous answers Language of the Supreme court,
High Courts, etc,
Special Directives 348 Language to be used in the
Supreme Court and in the
Q.1463 Which of the following is the criterion for
High Courts and for Acts,
a language to be officially recognized in a
Bills, etc.
State of India?
(a) If the Governor is satisfied that a certain 349 Special procedure for
language should be recognized; and enactment of certain laws
declares so officially relating to language.
(b) If the State Legislature passes a resolution Special Directives
to that effect
350 Language to be used in
(c) When the President is satisfied that a
representation for redress
substantial population of the state speaks
of grievances
that particular language; and declares so
officially 350A Facilities for instruction in
(d) None of the above mother-tongue at primary
Solution: (c) stage
When the President (on a demand being 350B Special Officer for
made) is satisfied that a substantial linguistic minorities
proportion of the population of a state desire 351 Directive for development
the use of any language spoken by them to be of the Hindi Language
recognised by that state, then he may direct
that such language shall also be officially Q.1464 When does the President direct that a
recognised in that state. This provision particular language be officially recognized
aims at protecting the linguistic interests of in a state?
minorities in the states. (a) When the State Council of Ministers pass
a resolution to this effect
Article Nos Subject matter (b) When the Governor personally advises
Language of the Union him to
(c) When the local bodies of the area pass a
343 Official Language of the
joint resolution to this effect
Union
(d) When the President is satisfied that a
344 Commission and substantial proportion of the population
committee of Parliament desire their language to be recognised
on official language Solution: (d)
Regional Languages Q.1465 Which of these constitutional provision(s)
345 Official Language or aim at protecting the linguistic interests of
language minorities in the states?
of a state 1. President can direct the government to
provide compulsory financial grants
346 Official language for
and reservation in public jobs to such
communication between
minorities.
one state and another or
2. President can direct the official recognition
between a state and Union.
of a minority language in the state.

P.412 For Civil Services Preliminary Examination

04-Indian Polity_Q1070-1506.indd 412 8/7/2018 7:42:44 PM


Which of the above is/are correct? may be. This means that a representation
(a)  1 only (b)  2 only cannot be rejected on the ground that it is not
(c)  Both 1 and 2 (d)  None in the official language
Solution: (b) Q.1467 Availing facilities for instruction in mother-
Justification: Statement 1: There is no such tongue at primary stage is the
provision for reservation in public jobs, (a)  Fundamental Right of a Child
however it shall be the endeavour of every (b)  Duty of the State
State and of every local authority within (c)  Legal right of a Child
the State to provide adequate facilities (d)  Constitutional right of a Child
for instruction in the mother-tongue at Solution: (b)
the primary stage of education to children Justification: Article 350A: It shall be the
belonging to linguistic minority groups. endeavour of every State and of every local
The President may issue such directions authority within the State to provide adequate
to any State as he considers necessary or facilities for instruction in the mother-tongue
proper for securing the provision of such at the primary stage of education to children
facilities. belonging to linguistic minority groups; and
Statement 2: explained in previous the President may issue such directions to
answers. any State as he considers necessary or proper
Q.1466 The Constitution of India contains explicit for securing the provision of such facilities.
and specific provisions for which of following It is a directive principle outside of part IV of
language related aspects? the constitution. It is a duty of the state, and
1. Directive for development of the Hindi not a right of the citizen.
language Q.1468 “It shall be the endeavour of every state
2. Facilities for instruction in mother-tongue and every local authority within the state to
at primary stage for Children provide adequate facilities for instruction in
3. Language to be used in representation for the mother-tongue at the primary stage of
redress of grievances education to children belonging to linguistic
Choose the correct answer using the codes minority groups.” This is a
below. 1. Provision made under an Act of Parliament
(a)  1 and 2 only (b)  2 and 3 only 2. Justiciable in courts if not implemented
(c)  All of the above (d)  1 and 2 only by the Government of India
Solution: (c) 3. Forms part of Fundamental rights under
Article 351: The Constitution imposes a Part III of the Constitution
duty upon the Centre to promote the spread Select the correct answer using the codes
and development of the Hindi language so below.
that it may become the lingua franca of the (a)  1 only (b)  2 and 3 only
composite culture of India. (c)  1 and 3 only (d)  None of the above
Further, the Centre is directed to secure Solution: (d)
the enrichment of Hindi by assimilating Justification: It forms a part of Directive
the forms, style and expressions used in principles.
hindustani and in other languages specified Apart from the Directives included in
in the Eighth Schedule and by drawing Part IV, there are some other Directives
its vocabulary, primarily on sanskrit and contained in other Parts of the Constitution.
secondarily on other languages. Instruction in mother tongue is a part of
Article 350: Every aggrieved person has the Article 350-A in Part XVII.
right to submit a representation for the redress Hence, all the above statements 1, 2 and
of any grievance to any officer or authority of 3 are wrong, as directive principles are non-
the Union or a state in any of the languages justiciable.
used in the Union or in the state, as the case

Indian Polity Question Bank P.413

04-Indian Polity_Q1070-1506.indd 413 8/7/2018 7:42:44 PM


Q.1469 You are aggrieved by an order of the development of the Hindi language so that
Municipality. You submit a grievance it may become the lingua franca of the
redressal request to the District nodal composite culture of India. Further, the
officer who handles the matter. But, your Centre is directed to secure the enrichment
representation is rejected. The officer rejected of Hindi by assimilating the forms, style
your application because and expressions used in hindustani and
1. Any such application must be submitted in other languages specified in the Eighth
in the official language of the State. Schedule and by drawing its vocabulary,
2. A citizen cannot submit any grievance primarily on sanskrit and secondarily on
redressal request without first moving the other languages. Also, the members of these
court. languages are to be given representation in
Which of the above is/are correct? the Official Language commission. The
(a)  1 only (b)  2 only Commission consisted of a chairman and
(c)  Both 1 and 2 (d)  None other members representing the different
Solution: (d) languages specified in the Eighth Schedule of
Justification: Statement 1: Every aggrieved the Constitution.
person has the right to submit a representation Q.1471 Consider the following statements.
for the redress of any grievance to any officer Assertion (A): The Constitution imposes a
or authority of the Union or a state in any duty upon the Centre to promote the spread
of the languages used in the Union or in and development of the Hindi language.
the state, as the case may be. This means Reason (R): Hindi is the mother tongue of
that a representation cannot be rejected majority of the Indian population.
on the ground that it is not in the official In the context of the above, which of these
language. is correct?
Statement 2: Moving the court is not (a) A is correct, and R is an appropriate
mandatory in case of every executive order, explanation of A.
especially in cases where relief can be (b) A is correct, but R is not an appropriate
granted by the administrative authorities. explanation of A.
Court should be moved when the order is not (c) A is correct, but R is incorrect.
bonafide, illegal, outside the competence of (d) A is incorrect, but R is correct.
the authority or against the interests of public Solution: (c)
at large. Justification: More than 59% Indians list
Q.1470 What is the objective(s) behind the a language other than Hindi as their mother
specification of regional languages in the tongue. So, Hindi is the mother tongue of
Eighth schedule? only nearly 41% of the population.
1. These languages are to be used for • As per the 2001 census, around 42.2 crore
enriching Hindi Language people listed Hindi as their mother
2. Members of these languages are tongue, of which 25.79 crore spoke Hindi
represented in the Official Languages in its pure form, and another 16.41 crore
Commission. in 49 mother tongues similar to Hindi.
3. Restricting the choice of state • As for other scheduled languages, Bengali
government’s official language was the most spoken mother-tongue
Choose the correct answer using the codes after Hindi (with 8.11% of the country’s
below. population citing it as their mother
(a)  1 and 2 only (b)  2 and 3 only tongue), followed by Telugu (7.19%),
(c)  1 and 3 only (d)  All of the above Marathi (6.99%), Tamil (5.91%) and
Solution: (a) Urdu (5.01%).
The Constitution imposes a duty upon • Only a dozen states - including Uttar
the Centre to promote the spread and Pradesh, Madhya Pradesh, Bihar,

P.414 For Civil Services Preliminary Examination

04-Indian Polity_Q1070-1506.indd 414 8/7/2018 7:42:44 PM


Chhattisgarh, Rajasthan, Uttarakhand, (c) A Statutory requirement enforced by a
Himachal Pradesh, Haryana, Delhi Parliamentary statue
and Jharkhand–had a majority of (d) Imitated from the Parliamentary
their population listing Hindi as their rulebooks and business procedures
mother tongue. Solution: (c)
Learning: Articles 343-351 of the
Languages of Judiciary constitution deal with the official language
of the Union. It empowers the Parliament
Q.1472 Which of the following prescribes that all
to provide for the continued use of English
proceedings in the Supreme Court of India
language even after 1965.
shall be in English language?
Accordingly, Parliament enacted the
(a) In-house rules made by the Supreme
Official Languages Act 1963.
Court
The act provides for the continued use
(b) A Legislation made by the Parliament
of English (even after 1965), in addition to
(c) A constitutional provision
Hindi, for all official purposes of the Union
(d)  An executive order
and also for the transaction of business
Solution: (c)
in Parliament.
Justification: Article 348 of the Constitution
of India says that until Parliament by law Q.1474 Consider the following statements:
otherwise provides 1. The Parliament has not made any provision
(a) all proceedings in the Supreme Court and for the use of Hindi in the Supreme
in every High Court, Court.
(b) the authoritative texts of all Bills to be 2. If a substantial part of the population
introduced or amendments thereto to be of a state speak a certain language, it is
moved in either House of Parliament the Governor who may direct that such
or in the House or either House of the language be also officially recognized in
Legislature of a State, the state.
(c) of all Acts passed by Parliament or the 3. The Constitution imposes a duty on
Legislature of a State the Centre to promote the spread and
(d) of all Ordinances promulgated by the development of the Hindi Language.
President or the Governor of a State Which of these is/are correct?
(e) of all orders, rules, regulations and bye (a) Only 1 (b) Only 2
laws issued under this Constitution or (c) 2 and 3 (d) 1 and 3
under any law made by Parliament or Solution: (d)
the Legislature of a State, shall be in the The proceedings of the Supreme Court are
English language. in English. It is not allowed constitutionally
Learning: Article 145 of the Constitution to use any other language for hearing
of India prescribes that subject to the appeals.
provisions of any law made by Parliament
the Supreme Court may from time to time, Eighth Schedule
with the approval of the President, make Q.1475 Eighth Schedule recognizes which of the
rules for regulating generally the practice and following languages?
procedure of the Court. 1. English 2. Sanskrit
Q.1473 In the transaction of government business 3. Kashmiri 4. Hindi
and proceedings of Supreme Court, the use Select the correct answer using the codes
of English for official purposes is below.
(a) Merely a Convention from the Colonial (a)  3 and 4 only (b)  1 and 2 only
Era and not a mandatory requirement (c)  2, 3 and 4 only (d)  1, 2, 3 and 4
(b) Part of the executive fiat issued from Solution: (c)
time to time by the Central Government

Indian Polity Question Bank P.415

04-Indian Polity_Q1070-1506.indd 415 8/7/2018 7:42:44 PM


Justification: Originally, it had 14 languages 1854 following Lord Macaulay’s Select
but presently there are 22 languages. Committee Report.
They are: Assamese, Bengali, Bodo, 2. A Central Public Service Commission
Dogri (Dongri), Gujarati, Hindi, Kannada, was set up in 1926 as provided by the
Kashmiri, Konkani, Maithil (Maithili), Government of India Act of 1919.
Malayalam, Manipuri, Marathi, Nepali, Which of the above is/are correct?
Oriya, Punjabi, Sanskrit, Santhali, Sindhi, (a)  1 only (b)  2 only
Tamil, Telugu and Urdu. (c)  Both 1 and 2 (d)  None
Statement 1: English is not recognized Solution: (c)
under 8th schedule, even though it is Justification: Statement 1: Civil Servants
the official language of India. So, 1 is for the East India Company used to be
incorrect. nominated by the Directors of the Company
Sindhi was added by the 21st Amendment and thereafter trained at Haileybury
Act of 1967; Konkani, Manipuri and Nepali College in London and then sent to India.
were added by the 71st Amendment Act This changed after Lord Macaulay’s
of 1992; and Bodo, Dongri, Maithili and Report.
Santhali were added by the 92nd Amendment Statement 2: It was entrusted with the task
Act of 2003. of recruiting civil servants.
Q.1476 Consider the following statements. The Government of India Act of 1935
Assertion (A): The constitution doesn’t provided for the establishment of not only a
recognize any tribal language. Federal Public Service Commission but also
Reason (R): The constitution recognizes only a Provincial Public Service Commission and
classical languages. Joint Public Service Commission for two or
In the context of the above, which of these more provinces.
is correct? Q.1478 The bureaucracy in India is expected to be
(a) A is correct, and R is an appropriate politically neutral. Political neutrality implies
explanation of A. that the bureaucracy will not?
(b) A is correct, but R is not an appropriate 1. Take a purely political position on policy
explanation of A. matters.
(c)  A is correct, but R is incorrect. 2. Contest for political offices.
(d)  Both A and R are incorrect. 3. Implement policies and schemes that have
Solution: (d) a political objective
Justification: The eighth schedule of the Select the correct answer using the codes
Constitution recognizes about 22 languages. below.
Two tribal languages, Bodo and Santhali (a)  1 and 2 only (b)  2 only
are recognised by the Constitution. Sanskrit, (c)  2 and 3 only (d)  1, 2 and 3
Tamil, Telugu, Kannada, Odiya and Solution: (a)
Malayalam are the classical languages as Justification: Statement 1: Persons working
per the status accorded by the Government in the State services (Civil servants)
of India. are required to act in the course of their
The constitution, thus, recognizes both duties in a politically neutral manner. This
classical and non-classical languages. Both A includes the requirement to act impartially
and R are incorrect. and to implement the Government’s
policies.
(65) PUBLIC SERVICES – PART XIV – The requirement that State servants
must be apolitical when carrying out their
ARTICLES 308 TO 323 duties, functions and powers. This means,
Q.1477 Consider the following statements. essentially, that State servants must keep their
1. The concept of a merit based modern jobs out of their politics and their politics out
Civil Service in India was introduced in of their jobs.

P.416 For Civil Services Preliminary Examination

04-Indian Polity_Q1070-1506.indd 416 8/7/2018 7:42:44 PM


Statement 2: If they contest for such Rajya Sabha is empowered to make laws
offices, they will no longer be able to creating one or more All India Services,
remain neutral in day to day administration which would be common to the Union
and will tend to extract political gains from and State, if it is deemed to serve the
their actions. national interest.
Statement 3: Most policies have a political Q.1480 Consider the following statements about the
objective, for example, policy to distribute Civil Services created by the Constitution
natural resources based on auctions, rather of India.
than first come first serve basis, clearly has a Assertion (A): A State government can
political objective. It is to curb the exploitation remove an officer of the All-India Services
of political power by the high and mighty. if he has caused a grave loss to the State
But, civil servants must implement these exchequer.
policies, for example, by way of executing the Reason (R): The all-India services are also
auction sale. controlled by the state governments, apart
from the Central government, based on the
Classification of Services principle of federalism.
In the context of the above, which of these
All India services is correct?
(a) A is correct, and R is an appropriate
Q.1479 With reference to All India Services (AIS),
explanation of A.
consider the following statements:
(b) A is correct, but R is not an appropriate
1. Only the central government can take
explanation of A.
disciplinary action against them
(c)  A is incorrect, but R is correct.
2. There are two all India Civil Services of
(d)  Both A and R are incorrect.
India, namely the Indian Administrative
Solution: (c)
Service (IAS) and the Indian Police
Please refer previous answer.
Service (IPS)
3. Power of creation of AIS is vested in the Q.1481 Consider the following statements about All
lower house of the Central Legislature India services:
Which of the above statements is/are 1. The ultimate control of the All India
correct? services officers lie with the state
(a)  1 Only (b)  2 and 3 Only governments.
(c)  1 and 3 Only (d)  1, 2 and 3 2. The Lok Sabha can unilaterally create a
Solution: (a) new All India service without the approval
There are three all India Civil Services of of the Rajya Sabha.
India, namely the Indian Administrative Which of these is/are correct?
Service (IAS), the Indian Forest Service (a)  Only 1 (b)  Only 2
(IFS) and the Indian Police Service (IPS). (c)  Both (d)  None of the above
AIS officer is governed by the service Solution: (d)
conditions laid down by the central The Rajya Sabha must pass a resolution to
government? the effect of creating a new All India service.
An IAS or IPS officer is assigned to a For, it lies in the federal domain.
particular State, where he or she works under Q.1482 Consider the following statements:
the supervision of the State government. Assertion (A): The Parliament cannot create
However, the IAS or IPS officers are the All-India Judicial service by only a simple
appointed by the central government, they majority.
can go back into the service of the central Reason (R): Creation of the All-India judicial
government and most importantly, only the service would be a constitutional amendment
central government can take disciplinary that would require a special majority and the
action against them. consent of the states.

Indian Polity Question Bank P.417

04-Indian Polity_Q1070-1506.indd 417 8/7/2018 7:42:44 PM


In the context of the statements above, Which of the above statements is/are
which of these is true? incorrect?
(a) Both A and R are true and R is the correct (a)  1 Only (b)  2 Only
explanation for A. (c) Both (d) None
(b) Both A and R are true and R is not the Solution: (d)
correct explanation for A. A Bench of Justices K.S. Radhakrishnan
(c)  A is incorrect but R is correct. and Pinaki Chandra Ghose referred to the
(d)  Both A and R are incorrect. recommendations of the Hota Committee
Solution: (d) (2004) and the Santhanam Committee report
The 42nd amendment to the constitution (1964), which highlighted “the necessity
already provides for an all-India judicial of recording instructions and directions by
service. Any amendment in the constitution public servants.”
to provide for this post would not be deemed The Bench said: “We notice that much
as a constitutional amendment. No such law of the deterioration of the standards of
has been made for so far. probity and accountability with the civil
Q.1483 An All-India judicial service can be created servants is due to the political influence of
by the Parliament if the Rajya Sabha passes persons purporting to represent those who
a resolution declaring that it is necessary or are in authority. The Santhanam Committee
expedient in the national interest to do so on Prevention of Corruption, 1962 has
by a/an recommended that there should be a system
(a) Two-thirds of the members present and of keeping some sort of records in such
voting situations. Rule 3(3) (iii) of the All India
(b) Simple majority of members present and Service Rules specifically requires that all
voting orders from superior officers shall ordinarily
(c) Special majority approved by the Speaker be in writing.”
of Lok Sabha
(d) Motion moved by 50 members of Rajya Central services
Sabha approved by the Chairman, Rajya Q.1485 Consider the following statements about
Sabha Central Services and All-India Services.
Solution: (a) 1. Any disciplinary action (imposition of
Justification: Such a resolution in the Rajya penalties) against these officers can only
Sabha should be supported by two-thirds of be taken by the Central government.
the members present and voting. 2. The personnel of Central services (even
This power of recommendation is given though are posted in states) work under
to the Rajya Sabha to protect the interests of the exclusive jurisdiction of the Central
states in the Indian federal system. government.
Q.1484 In the light of the recent judgment by the Which of the above is/are true?
Supreme Court refraining Civil servants from (a)  1 only (b)  2 only
acting on oral instructions of political bosses, (c)  Both 1 and 2 (d)  None
except in certain exceptional circumstances, Solution: (c)
consider the following statements, The all-India services are controlled jointly
1. The Santhanam Committee on Prevention by the Central and state governments.
of Corruption, 1962 has recommended The ultimate control lies with the Central
that there should be a system of keeping government while the immediate control
some sort of records in such situations. is vested in the state governments. Any
2. The All India Service Rules specifically disciplinary action (imposition of penalties)
requires that all orders from superior against these officers can only be taken by the
officers shall ordinarily be in writing. Central government.

P.418 For Civil Services Preliminary Examination

04-Indian Polity_Q1070-1506.indd 418 8/7/2018 7:42:44 PM


The personnel of Central services work Any disciplinary action (imposition of
under the exclusive jurisdiction of the penalties or removal or reduction in rank)
Central government. They hold specialised against these officers can only be taken by
(functional and technical) positions in various the Central government.
departments of the Central government. Q.1487 Consider the following about All-India
The personnel of state services work services.
under the exclusive jurisdiction of the state 1. Service conditions are determined by the
government. They hold different positions President.
(general, functional and technical) in the 2. Services can be abolished only by a
departments of the state government. constitutional amendment.
However, they occupy lower positions (in 3. UPSC is responsible for appointments to
the administrative hierarchy of the state) than the posts in All- India services.
those held by the members of the all-India 4. Members of All-India services cannot be
services (IAS, IPS and IFS). removed from services by Prime Minister
alone.
Constitutional Provisions Choose the correct answer using the codes
Q.1486 Consider the following statements about the below.
Civil Services created by the Constitution of (a)  1, 2 and 4 only (b)  3 only
India. (c)  2, 3 and 4 only (d)  2 and 4 only
Assertion (A): An officer of All-India Solution: (d)
services cannot be dismissed by any authority UPSC only certifies eligible candidates.
in India other than the President of India. Appointment is done by DoPT (Government
Reason (R): A civil servant cannot be of India). Service Conditions of AIS are
dismissed or removed by an authority determined by the Parliament. Members
subordinate to that by which he was of AIS cannot be removed by an authority
appointed. subordinate to the one that appointed it.
In the context of the above, which of these President appoints members of AIS. So, they
is correct? cannot be removed by the Prime Minister.
(a) A is correct, and R is an appropriate Q.1488 India has imposed several restrictions
explanation of A. on the political activities of serving civil
(b) A is correct, but R is not an appropriate servants. Which of these are among those
explanation of A. restrictions?
(c) A is incorrect, but R is correct. 1. She cannot become a member of a
(d)  A is correct, but R is incorrect. political party.
Solution: (a) 2. She cannot voice her opinion on any
Justification: As per the constitution, a civil matter on a social media platform.
servant cannot be dismissed or removed by an 3. She does not enjoy a right to vote.
authority subordinate to that by which he was 4. She is prohibited from participating in a
appointed. Since, the Central government strike.
makes their appointment on the warrant Select the correct answer using the codes
of the President, they can’t be removed by below.
anyone other than the President. (a)  1 and 2 only (b)  2 and 4 only
However, the all-India services are (c)  1 and 4 only (d)  2 and 3 only
controlled jointly by the Central and state Solution: (c)
governments. Justification: Political neutrality of civil
The ultimate control lies with the Central servants has been regarded as one of cardinal
government while the immediate control is conditions for the success of a democratic
vested in the state governments. government.

Indian Polity Question Bank P.419

04-Indian Polity_Q1070-1506.indd 419 8/7/2018 7:42:44 PM


Statement 1: If they are engaged in (b) A bill to this effect is passed in Parliament
political activities, it violates the above by special majority
principle. They cannot contest Lok Sabha or (c) The prior approval of the President has
State assembly elections. been taken
Statement 2: Civil servants are not (d) Rajya Sabha passes a resolution to
allowed to criticise govt on social media or this effect declaring it to be in national
press. These do not apply for general interest
Statement 3: They do enjoy a right to vote Solution: (d)
as is the right of every voter in India. Learning:
Statement 4: Striking by civil servants Article 312: The Parliament can create new
constitutes a disciplinary offence. all-India services (including an all-India
Q.1489 There have been many concerns judicial service), if the Rajya Sabha passes
regarding Articles 310 and 311 and even a resolution declaring that it is necessary
recommendations for their deletion from the or expedient in the national interest to do
Constitution. These constitutional articles so. Such a resolution in the Rajya Sabha
deal with should be supported by two-thirds of the
(a)  Civil Services members present and voting. This power of
(b) Ordinance making power of executive recommendation is given to the Rajya Sabha
(c)  Legislative Council in States to protect the interests of states in the Indian
(d) Special privileges extended to certain federal system.
states Q.1491 Which among the following can authorize the
Solution: (a) Parliament to create new All-India Services
Articles: Article 310 essentially conveys that common to both the Centre and states?
civil servants of the Union hold office during (a)  President of India
the pleasure of the President, and those at the (b)  Speaker, Lok Sabha
state at the pleasure of the Governor. (c)  Rajya Sabha
Article 311 conveys that they cannot be (d)  Inter-State Council
removed by an authority subordinate to the Solution: (c)
one that appointed them, and that due inquiry Learning: Due to its federal character, the
must be conducted where they are allowed Rajya Sabha has been given two exclusive
to hear charges against them and defend or special powers that are not enjoyed by the
themselves appropriately. Lok Sabha:
Justification: It is believed by many, • It can authorise the Parliament to make a
including the second Administrative Reforms law on a subject enumerated in the State
Commission (ARC) that these articles provide List (Article 249).
undue protection to dishonest and corrupt • It can authorise the Parliament to create
civil servants and shield them from action. new All-India Services common to both
This is because the protection is excessive, the Centre and states (Article 312).
and hearings go through many bureaucratic An analysis of the above points makes it
layers, which ultimately corrodes the fabric clear that the position of the Rajya Sabha
of administration and its efficiency. in our constitutional system is not as weak
There are many arguments against this as as that of the House of Lords in the British
well, but delving into there is not a subject of constitutional system nor as strong as that
concern here. of the Senate in the American constitutional
Q.1490 The Parliament can create new all-India system.
services only when Except in financial matters and control
(a) Majority of State legislatures pass a over the council of ministers, the powers and
resolution to this effect upholding the status of the Rajya Sabha in all other spheres
principle of federalism

P.420 For Civil Services Preliminary Examination

04-Indian Polity_Q1070-1506.indd 420 8/7/2018 7:42:44 PM


are broadly equal and coordinate with that of Q.1494 Consider the following statements.
the Lok Sabha. Assertion (A): Under the Constitution,
the civil servants are conferred personal
(66) RIGHTS AND LIABILITIES OF THE immunity from legal liability for official
contracts signed in official capacity.
GOVERNMENT Reason (R): Government is liable for that
Q.1492 Consider the following with regard to official contract
the ownership and jurisdiction of marine In the context of the above, which of these
resources. Which of the following statements is correct?
is correct? (a) A is correct, and R is an appropriate
(a) Major minerals and land under the ocean explanation of A.
waters within the territorial waters of (b) A is correct, but R is not an appropriate
India are owned by State government explanation of A.
whereas those beyond territorial waters (c)  A is correct, but R is incorrect.
by Central government (d)  Both A and R are incorrect.
(b) All major minerals and land under the Solution: (a)
ocean waters whether within or outside Justification: Statement A means that the
the territorial waters of India, until the civil servant who made a contract in his
end of Exclusive Economic Zone, are official capacity is not personally liable
owned solely by the Central Government in respect of that contract but it is the
(c) All major minerals under the ocean government (Central or state) that is liable
waters in the Exclusive Economic Zone for the contract. But, if the contract is made
are owned solely by the state Government without complying the conditions specified
(d) None of the above is correct. in the Constitution, then the civil servant who
Solution: (b) made the contract is personally liable.
Learning: Lands, minerals and other things Further, the civil servants also enjoy
of value under the waters of the ocean within immunity from legality for their tortious acts
the territorial waters of India, the continental in respect of the sovereign functions of the
shelf of India and the exclusive economic government. In other cases, the liability of
zone of India vests in the Union. the civil servants for torts or illegal acts is the
Hence, a state near the ocean cannot same as of any ordinary citizen.
claim jurisdiction over these things. India’s The Constitution does not grant any
territorial waters extend to a distance of 12 immunity to the ministers for their official
nautical miles from the appropriate base line. acts. But, since they are not required to
Similarly, India’s exclusive economic countersign (as in Britain) the official acts of
zone extends upto 200 nautical miles. the president and the governors, they are not
liable in the courts for those acts.
Q.1493 The minerals under the waters of the ocean
within the territorial waters of India and its
exclusive economic zone near a coastal state (67) AUTHORITATIVE TEXT OF THE
is owned by CONSTITUTION IN HINDI –
(a)  The concerned State Government PART XXII
(b)  The Central Government
(c)  Local government Q.1495 Consider the following statements.
(d) Owned jointly by Central, State and Assertion (A): The unamended Constitution
Local governments of India did not make any provision with
Solution: (b) respect to an authoritative text of the
Explained in previous answer Constitution in the Hindi language.

Indian Polity Question Bank P.421

04-Indian Polity_Q1070-1506.indd 421 8/7/2018 7:42:44 PM


Reason (R): The Constitution of India derived Solution: (d)
its core administrative arrangements from the Justification: These special provisions are
Government of India Act 1935. contained in Part XVI of the Constitution
In the context of the above, which of these from Articles 330 to 342. They are also
is correct? related to Special Representation in
(a) A is correct, and R is an appropriate Legislatures; Reservation in Services and
explanation of A. Posts; Educational Grants and Appointment
(b) A is correct, but R is not an appropriate of National Commissions for Scheduled
explanation of A. Tribes (NCST).
(c) A is correct, but R is incorrect. Statement 2: Such grants are provided for
(d)  Both A and R are incorrect. example, under Article 275 for the purpose of
Solution: (b) promoting the welfare of Scheduled Tribes in
Justification: Statement A is correct. that State or raising the level of administration
• Later, a provision in this regard was made of the Scheduled Areas therein to that of the
by the 58th Constitutional Amendment administration of the rest of the areas of that
Act of 1987. This amendment inserted a State.
new Article 394-A in the last part of the
Constitution
• The reasons for adding the above
Scheduled Castes and Scheduled
provisions in the Constitution by the 58th Tribes
Constitutional Amendment Act of 1987
are as follows: Q.1497 The list of castes or tribes that are to be
• The Constitution of India adopted by the identified as SCs or STs are specified in
Constituent Assembly on 26 November (a) A Presidential order with respect to each
1949, was in the English language. State and Union Territory (UT)
• A Hindi translation of the Constitution, (b) The Fifth and Sixth Schedule of the
signed by the members of the Constituent Constitution of India
Assembly, was also published in 1950 (c) The resolutions passed by the respective
under the authority of the President of the state legislatures
Constituent Assembly. (d) The order issued by the State Governor
• However, there had been a general demand in the extra-ordinary Gazette of India
for the publication of an authoritative text Solution: (a)
of the Constitution in Hindi incorporating The constitution does not specify the castes
in it all the subsequent amendments. or tribes which are to be called the SCs orthe
STs. It leaves to the President the power to
specify as to what castes or tribes in each state
(68) SPECIAL PROVISIONS RELATING and union territory are to be treated as the SCs
TO CERTAIN CLASSES – and STs. Thus, the lists of the SCs or STs vary
PART XVI – ARTICLE 330 TO 342 from state to state and union territory to union
territory. In case of the states, the President
Q.1496 Special provisions for marginalized classes issues the notification after consulting the
in the constitution are related to governor of the state concerned.
1. Reservation in Legislatures and local
Q.1498 Any inclusion or exclusion of any caste or
bodies
tribe from the list of Scheduled Castes and
2. Educational Grants
Scheduled Tribes can be done only by the
3. Appointment of Commissions to
(a) President
investigate their socio-economic condition
(b) Parliament
Select the correct answer using the codes
(c)  Governors of Respective States
below.
(d)  State Legislative Assemblies
(a)  2 and 3 only (b)  1 only
Solution: (b)
(c)  1 and 2 only (d)  1, 2 and 3

P.422 For Civil Services Preliminary Examination

04-Indian Polity_Q1070-1506.indd 422 8/7/2018 7:42:44 PM


Learning: Constitution does not specify the 2. Providing for quotas to socially,
castes or tribes which are to be called the SCs economically and educationally backward
or the STs. classes is mentioned in the DPSP outside
• It leaves to the President the power to of Part IV of the constitution.
specify as to what castes or tribes in each Which of these is/are true?
state and union territory are to be treated (a)  Only 1 (b)  Only 2
as the SCs and STs. Thus, the lists of the (c) Both (d) None
SCs or STs vary from state to state and Solution: (a)
union territory to union territory. The quotas are to be provided to only SCs and
• In case of the states, the President issues STs which are socially backward. There is
the notification after consulting the no provision for quotas to economically and
governor of the state concerned. But, any educationally backward classes. In fact, in a
inclusion or exclusion of any caste or famous Mandal case the SC cut down a 10%
tribe from Presidential notification can be quota for economically backward classes.
done only by the Parliament and not by a Q.1501 The Constitution obliges the President to
subsequent Presidential notification. appoint a commission to investigate and
• Similarly, the Constitution has neither report on the conditions of
specified the BCs nor used a single (a)  Women and children only
uniform expression to characterise the (b)  Scheduled castes only
BCs. (c) Socially and educationally backward
Q.1499 The constitution of India empowers which classes
of these authorities to modify the list of (d)  Scheduled tribes only
notified scheduled castes as first notified by Solution: (c)
the President of India? Special provisions relating to certain Classes
(a) Parliament can be found even outside the FRs and
(b)  National Commission for SCs DPSP.
(b) Chairman, National Human Rights Article 340 provides for - Appointment of
Commission (NHRC) a Commission to investigate the conditions
(d)  Cabinet Secretary of backward classes:
Solution: (a) (1) The President may by order appoint a
Learning: The Parliament of India can Commission consisting of such persons as
modify the list of notified scheduled he thinks fit to investigate the conditions
castes, which upon receiving the assent of socially and educationally backward
of the President of India can be notified as classes within the territory of India and
Scheduled Castes. the difficulties under which they labour
• Under the provision of Article 341, and to make recommendations as to the
list of SCs in relation to a states/UT is steps that should be taken by the Union
to be issued by a notified Order of the or any State to remove such difficulties
President after consulting concerned state and to improve their condition and as to
Government. the grants that should be made for the
• Any subsequent inclusion in or exclusion purpose by the Union or any State and
from the list of Scheduled Castes can the conditions subject to which such
be effected only through an Act of grants should be made, and the order
Parliament. appointing such Commission shall
define the procedure to be followed by
Other Backward Communities (OBC) the Commission
(2) A Commission so appointed shall
Q.1500 Consider the following statements:
investigate the matters referred to them
1. The constitution of India does not define
and present to the President a report
backward classes.

Indian Polity Question Bank P.423

04-Indian Polity_Q1070-1506.indd 423 8/7/2018 7:42:45 PM


setting out the facts as found by them Minorities
and making such recommendations as
they think proper Q.1503 The Constitution of India recognizes:
(3) The President shall cause a copy of (a)  Only religious minorities
the report so presented together with (b)  Only linguistic minorities
a memorandum explaining the action (c)  Religious and linguistic minorities
taken thereon to be laid before each (d) Religious, linguistic and ethnic minorities
House of Parliament Solution: (c)
There is no such term either mentioned or
Q.1502 Consider the following about the National
defined in the constitution as ethnic minorities
Commission for Backward Classes.
even though the constitution does not permit
1. It was established based on a Supreme
discrimination based on race.
Court judgment directing the government
to evaluate under and over inclusion of Q.1504 In a multi-cultural democracy, which of the
castes in OBC list. following may lead to the alienation of the
2. It has been empowered to suo moto look minority?
into the grievances of persons of Other 1. Eliminating local autonomy
Backward Classes. 2. Adopting one ‘national’ language
Which of the above is/are correct? 3. Lack of cultural safeguards and rights
(a)  1 only (b)  2 only Choose the correct answer using the codes
(c)  Both 1 and 2 (d)  None below:
Solution: (a) (a)  1 and 2 only (b)  1 and 3 only
Justification: Statement 1: The Supreme (c)  2 and 3 only (d)  All of the above.
Court of India in its Indra Sawhney Solution: (d)
judgment directed the Govt. of India to Assimilationist and integrationist strategies
constitute a permanent body for entertaining, try to establish singular national identities
examining and recommending upon requests through various interventions like:
for inclusion and complaints of over- • Centralising all power to forums where
inclusion and under-inclusion in the list the dominant group constitutes a majority,
of OBCs. and eliminating the autonomy of local or
Statement 2: NCBC receives a number minority groups;
of representations from OBCs. However, • Imposing a unified legal and judicial system
NCBC has not yet been empowered to look based on the dominant group’s traditions
into the grievances of persons of Other and abolishing alternative systems used by
Backward Classes. other groups;
• Under Article 338 of the Constitution, • Adopting the dominant group’s language
National Commission for Scheduled as the only official ‘national’ language
Castes is the competent authority to and making its use mandatory in all public
look into all the grievances, rights institutions;
and safeguards relating to Backward • Promotion of the dominant group’s
Classes. language and culture through nationa
Learning: The NCBC Act provides that the linstitutions including state-controlled
Commission shall consist of five Members, media and educational institutions;
comprising of a Chairperson who is or has • Adoption of state symbols celebrating
been a judge of the Supreme Court or of a the dominant group’s history, heroes
High Court; a social scientist; two persons, and culture, reflected in such things as
who have special knowledge in matters choice of national holidays or naming of
relating to backward classes; and a Member- streets etc.;
Secretary, who is or has been an officer of • Seizure of lands, forests and fisheries from
the Central Government in the rank of a minority groups and indigenous people
Secretary to the Government of India. and declaring them ‘national resources’

P.424 For Civil Services Preliminary Examination

04-Indian Polity_Q1070-1506.indd 424 8/7/2018 7:42:45 PM


Q.1505 Consider the following statements. of parents habitually resident therein and
Assertion (A): The National Commission for not established there for temporary purposes
Minorities is not aconstitutional body. only. Indian State means any territory which
Reason (R): The constitution does not the Government of the Dominion of India
define “minorities”. recognized as such a State. Ruler means
In the context of the above, which of these the Prince, Chief or other person who, at
is correct? any time before the commencement of the
(a) A is correct, and R is an appropriate Constitution (Twenty-sixth Amendment) Act,
explanation of A. 1971, was recognised by the President as the
(b) A is correct, but R is not an appropriate Ruler of an Indian State or any person who,
explanation of A. at anytime before such commencement, was
(c)  A is correct, but R is incorrect. recognised by the President as the successor
(d)  Both A and R are incorrect. of such Ruler.
Solution: (b)
Justification: The constitution does not
define the term “devolution of taxes” does
not mean that Finance Commission cannot
be a constitution body. Similarly, whether
a body is constitutional or not is decided
by whether the body is established through
constitutional provisions or not, irrespective
of whether ancillary terms related to it are
defined or note defined in the constitution.
The Union Government set up the National
Commission for Minorities (NCM) under
the National Commission for Minorities
Act, 1992. Six religious communities, viz;
Muslims, Christians, Sikhs, Buddhists,
Zoroastrians (Parsis) and Jains have been
notified as minority communities by the
Union Government.

Anglo-Indians
Q.1506 Article 366 contains the definitions of various
terms used in different provisions of the
constitution. Which of the following terms
have NOT been defined in the Constitution?
1. Indian State
2. Ruler
3. Anglo-Indian
Choose the correct answer from the codes
given below.
(a)  1 only (b)  2 only
(c)  3 only (d)  All have been defined.
Solution: (d)
Anglo-Indian means a person whose father
or any of whose other male progenitors in
the male line is or was of European descent
but who is domiciled within the territory of
India and is or was born within such territory

Indian Polity Question Bank P.425

04-Indian Polity_Q1070-1506.indd 425 8/7/2018 7:42:45 PM


POLITICAL DYNAMICS Q.1510 Right to vote in Lok Sabha and State
assembly elections is a:
(a)  Constitutional right
(69) RIGHT TO VOTE (b)  Statutory right
(c)  Fundamental right
Q.1507 The core of political democracy lies in the
(d)  None of the above
basic axiom of electoral democracy which is
Solution: (a)
based on
Fundamental rights are meant to protect
(a)  Right to education
human rights against executive tyranny and
(b)  Freedom of speech
establish political democracy. However,
(c)  Right to equality
Article 326 in Part XV of the constitution
(d)  Universal adult franchise
depicts the adult suffrage. It is not mentioned
Solution: (d)
under the fundamental rights section
Political democracy would essentially mean
neither under any law enacted by the
political power of the state is equally shared
parliament of India. Therefore it is only a
by the citizens. Citizens have the real power
constitutional right.
to legislate, which they perform by delegating
this function to their elected representatives. Q.1511 If a person’s “Right to Vote” is infringed,
It is implied by the universal adult franchise. what remedy does she he have?
(a) Move the Supreme Court citing violation
Q.1508 The constitution of India provides for
of fundamental rights
Universal Adult Suffrage. What was the
(b) File a non-judicial petition with the
objective behind which the Universal Adult
Election Commission of India
Suffrage was conceptualised by the members
(c) File a case against the Chief Electoral
of the Constituent Assembly?
Officer of the State
1. To encourage democratic mindset
(d) Move the High court for infringement of
2. To break the clutches of tradition, caste,
a constitutional right
class.
Solution: (d)
3. To remove gender hierarchy
Justification: Right to Vote is a constitutional
4 . To increase peoples participation
right, not a fundamental right. So, SC cannot
Which of the above statement is/ are correct?
be approached for issuing a writ petition.
(a)  1 Only (b)  1 and 4
High court enforces legal rights, and should
(c)  1, 2 and 4 (d)  1, 2, 3 and 4
be approached in case of its infringement.
Solution: (d)
ECI only conducts elections and maintains
The members of the Constituent Assembly
the database of voters. It does not have the
felt that the freedom struggle had prepared
authority to enforce the Right to Vote. Same
the masses for universal adult suffrage and
is true for the Chief Electoral Officer.
this would help encourage a democratic
mindset and break clutches of traditional Q.1512 The voting age for Indians was lowered to
caste, class and gender hierarchies. And since 18 under the Universal Adult Franchise based
the people of India will have a direct role in on the
electing their representatives, the people’s (a) Parliamentary enactment immediately
participation would also increase. post-independent
(b) Unamended Indian Constitution adopted
Q.1509 Which authority in India maintains the
in 1950
Voter’s list for a particular state?
(c) Legislation backed by the Government
(a)  Chief Electoral Officer of the State
in 1991
(b)  District Magistrates of the concerned
(d)  None of the above
areas
Solution: (d)
(c)  Chief Secretary
Justification and Learning: The Indian
(d)  Zila Parishad
Constitution adopts universal adult franchise
Solution: (a)

P.426 For Civil Services Preliminary Examination

05-Indian Polity_Q1507-2004.indd 426 8/7/2018 7:49:57 PM


as a basis of elections to the Lok Sabha and Which of these is/are true?
the state legislative assemblies. (a)  Only 1 (b)  Only 2
Every citizen who is not less than (c)  Both (d)  None of the above
18 years of age has a right to vote without Solution: (c)
any discrimination of caste, race, religion, Q.1515 Consider the following statements:
sex, literacy, wealth, and so on. The voting 1. Citizens of India who do not have a Voter
age was reduced to 18 years from 21 years in ID card cannot vote in elections.
1989 by the 61st Constitutional Amendment 2. Voter rolls are created and maintained by
Act of 1988. the Election Commission of India under
The introduction of universal adult the supervision of the Department of Law
franchise by the Constitution-makers was a and Justice, Government of India.
bold experiment and highly remarkable in 3. People who are mentally unsound may be
view of the vast size of the country, its huge debarred from voting in the election by
population, high poverty, social inequality the Election Commission of India.
and overwhelming illiteracy. Which of these is/are true?
Q.1513 The Election Commission of India allows (a)  1 and 2 (b)  Only 3
which of these categories of persons to do (c)  1 and 3 (d)  All of the above
‘proxy voting’ and/or vote through e-postal Solution: (b)
ballots? If you are issued with a voter ID card, you
1. Members of armed forces have to compulsorily carry it on the day of
2. Government of India employee posted voting as an identity proof. If you do not
abroad possess a voter ID card, you can carry any
3. Non-Resident Indian Voters one of the approved identity proof documents
Select the correct answer using the codes on Election Day. It can be ration card,
below. passport etc.
(a)  1 and 2 only (b)  2 only The ECI is an independent body. It does
(c)  3 only (d)  1, 2 and 3 not function under the supervision of the
Solution: (d) Government of India.
Justification: ECI defines a category of votes Q.1516 Who among the following have the right to
as service voters. These include members of vote in the elections to both Lok Sabha and
the armed forces, GoI employees serving Rajya Sabha?
abroad or Armed Police Force of a State, and (a) Elected members of the Lower House of
serving outside that State. the Parliament
Statements 1 and 2: Service voters are (b) Elected members of the Upper House of
allowed to vote through proxy persons. the Parliament
Statement 3: NRIs have recently been (c) Elected members of the Upper House of
given the facility to vote through e-postal the State Legislature
ballots and proxy voting both. (d) Elected members of the Lower House of
This new rule removes an “unreasonable the State Legislature
restriction” posed by Section 20(A) of the Solution: (d)
Representation of the People (Amendment) The MLAs also vote as ordinary citizens in
Act of 2010, requiring overseas electors to be general elections. The MPs also do. But they
physically present in their constituencies to cannot vote in the elections to Rajya Sabha.
cast their votes.
Q.1517 Consider the following statements:
Q.1514 Consider the following statements: 1. All those eligible to vote for local body
1. A certain class of persons, as notified by elections are also eligible to stand for the
the Election Commission, can give their local body elections.
votes by postal ballot in India. 2. Illiterate citizens of India, as defined
2. There is also a provision of a proxy vote legally, cannot contest Lok Sabha and
for members of the armed forces. Rajya Sabha elections.

Indian Polity Question Bank P.427

05-Indian Polity_Q1507-2004.indd 427 8/7/2018 7:49:57 PM


Which of these is/are true? (b) all the citizens who possess the right to
(a)  1 only (b)  2 only vote and elect their representatives in an
(c)  Both 1 and 2 (d)  None of the above election
Solution: (d) (c) those people who have the right to be
Minimum age for voting – 18, to fight local elected
body elections – 21. (d) those people who have the right to live
There is no educational qualification laid in a country
down for becoming an MP or MLA. S/he Solution: (b)
must be able to understand people’s problems One can be a voter if he/she has attained
and concerns. No formal education or degree age of 18 years and a citizen of India unless
is necessary for this. People themselves disqualified by the constitution or the law.
decide whether a person is fit to become their Q.1520 Consider the following statements.
MP, MLA etc. or not. 1. There is only one general electoral roll for
Q.1518 On what grounds can a person be disqualified every territorial constituency for election
as a voter? to the Parliament and the state legislatures.
1. Unsoundness of mind 2. The state legislatures, apart from
2. Corrupt or illegal practice Parliament can also make provision with
3. Crime respect to all matters relating to elections
4. Non-residence to the state legislatures.
(a)  I, III and IV (b)  I, II and III Which of the above is/are true?
(c)  II, III and IV (d)  I, II, III and IV (a)  1 only (b)  2 only
Solution: (d) (c)  Both 1 and 2 (d)  None
Article 326 states that “elections to the House Solution: (c)
of the People and to the Legislative Assemblies The Constitution (Article 324) provides for an
of States to be on the basis of adult suffrage independent Election Commission in order to
that is to say every person who is a citizen ensure free and fair elections in the country.
of India and who is not less than eighteen There is to be only one general electoral roll
years of age on such date as may be fixed in for every territorial constituency for election
that behalf by of under any law made by the to the Parliament and the state legislatures.
appropriate Legislature and is not otherwise Thus, the Constitution has abolished the
disqualified under this Constitution or any system of communal representation and
law made by the appropriate Legislature on separate electorates which led to the partition
the ground of non-residence, unsoundness of the country. The state legislatures can also
of mind, crime or corrupt or illegal practice make provision with respect to all matters
shall be entitled to be registered as a voter at relating to elections to the state legislatures
any such election. including the preparation of electoral rolls
Further Representation of the People Act, and all other matters necessary for securing
1950 says their due constitution. But, they can make
1. No person to be registered in more than provision for only those matters which are not
one constituency. -No person shall be covered by the Parliament. In other words,
entitled to be registered in the electoral they can only supplement the parliamentary
roll for more than one constituency. law and cannot override it.
2. No person to be registered more than once Q.1521 With reference to electoral rolls, consider the
in any constituency. - No person shall be following statements
entitled to be registered in the electoral 1. The electoral roll is a list of all people
roll for any constituency more than once. in the constituency who are registered to
Q.1519 The Electorate means: vote in Indian Elections
(a) those people who do not have the right to 2. Only those people with their names on the
elect their representatives electoral roll are allowed to vote

P.428 For Civil Services Preliminary Examination

05-Indian Polity_Q1507-2004.indd 428 8/7/2018 7:49:57 PM


3. The electoral roll is revised once every Q.1523 The Election Commission registers political
5 years parties for the purpose of elections and grants
Which of the above statements is/are correct? them recognition as national or state parties
(a)  1 and 3 Only (b)  1 and 2 Only on the basis of their
(c)  2 and 3 Only (d)  All 1. Poll performance
Solution: (b) 2. Cadre strength
The electoral roll is a list of all people in the 3. Financial backing
constituency who are registered to vote in 4. Number of elections contested
Indian Elections. Only those people with their Select the correct answer using the codes
names on the electoral roll are allowed to vote. below.
The electoral roll is normally revised every year (a)  1, 2 and 3 only (b)  1 only
to add the names of those who are to turn 18 on (c)  2 and 3 only (d)  1, 2 and 4 only
the 1st January of that year or have moved into a Solution: (b)
constituency and to remove the names of those Q.1524 A state political party can be derecognized by
who have died or moved out of a constituency. the Election Commission of India when
If you are eligible to vote and are not on the 1. It fails to submit its poll expenses
electoral roll, you can apply to the Electoral 2. It does not reveal information under the
Registration Officer of the constituency, who RTI Act
will update the register. The updating of the 3. It fails to win any seat in state elections.
Electoral Roll only stops during an election Choose the correct answer using the codes
campaign, after the nominations for candidates below.
have closed. (a)  1 and 2 only (b)  2 and 3 only
(c)  1 only (d)  1 and 3 only
(70) POLITICAL PARTIES Solution: (c)
Election Commission of India (ECI) has
Q.1522 The ‘Left’, ‘Right’ and ‘Centre’ classification suspended recognition of the P. A. Sangma-
of political parties in India is done largely on led National People’s Party (NPP) which
the basis of was a recognized state party in Meghalaya.
(a)  Strength of political party cadres ECI decision in this regard comes in line
(b) Conventional sitting position in the with Supreme Court’s judgement in the
legislature case of Common Cause Vs Union of India
(c)  Ideology of the political parties and others. In this judgement SC had made
(d)  Past electoral performance compulsion for all political parties to file
Solution: (c) their election expenditure statement within
Learning: On the basis of ideologies, the 75 days of assembly elections and 90 days of
political scientists have placed the radical Lok Sabha elections. The de-recognition of a
parties on the left and the liberal parties in party does not mean its banned.
the centre and reactionary and conservative
Q.1525 Which of the following criteria must be
parties on the right.
fulfilled by a political party to be recognized
In India, the CPI and CPIM are the
as a national party?
examples of leftist parties, the Congress of
1. If it is recognized as a state party in four
centrist parties and the BJP is an example of
states.
rightist.
2. If it secures 6% of valid votes polled in any
The ideology of party affects their
four or more states at a general election
manifesto and working style when they
to the Lok Sabha or to the legislative
come to power. For example, TMC is
assembly.
farmer oriented and not so business friendly
3. If it wins four seats in the Lok Sabha from
supporting the socialist ideology. On the
any state or states.
other hand, Indian National Congress takes
more nuanced pro-capitalist positions.

Indian Polity Question Bank P.429

05-Indian Polity_Q1507-2004.indd 429 8/7/2018 7:49:57 PM


Choose the correct answer using the codes least one seat in the Lok Sabha for every
below: 25 seats or any fraction thereof allotted to
(a)  Either 1 or ( 2 and 3 together) that State.
(b)  2 alone would be sufficient. Which of the statements is/are correct?
(c)  1 and 3. (a)  1 Only (b)  2 Only
(d) All of them must be achieved together to (c)  Both 1 and 2 (d)  Neither 1 nor 2
be recognized as a national Solution: (c)
Solution: (a) A political party shall be treated as a
Conditions for recognition as a National recognised political party in a State, if and
party: only if either the conditions satisfy:
A political party shall be treated as a recognised (a) A political party should secure at least
National party, if, and only if, either 6% of the total valid votes polled during
1. the candidates set up by it, in any general election to a State Legislative
four or more States, at the last general Assembly and should, in addition, win
election to the House of the People, or at least two seats in that Assembly, or
to the Legislative Assembly of the State the party should win at least 3% of the
concerned, have secured not less than total number of seats or three seats in
six percent of the total valid votes polled the Legislative Assembly, whichever
in their respective States at that general is more;
election; (Or)
2. in addition, it has returned at least four (b) A political party should secure at least
members to the House of the People at the 6% of the total valid votes polled in a
aforesaid last general election from any State during a general election to Lok
State or States; Sabha and win at least one seat in the
or Lok Sabha from that State, or the party
3. its candidates have been elected to the should win at least one seat in the Lok
House of the People, at the last general Sabha for every 25 seats or any fraction
election to that House, from at least 2% thereof allotted to that State (or, at
of the total number of parliamentary least one member to the Legislative
constituencies in India, any fraction Assembly of that State for every 30
exceeding one half being counted as one; members of that Assembly or any
and fraction of that number.)
4. the said candidates have been elected to Q.1527 With regard to registration of political parties
that House from not less than three State in India, consider the following statements:
Q.1526 With regard to the necessary conditions for 1. All political parties must register
a party to be recognized as a State party. themselves with the Election Commission.
Consider the following statements: 2. Not all parties that are registered with the
1. A political party should secure at least Election Commission are recognized by it.
6% of the total valid votes polled during 3. The registered parties will get unique
general election to a State Legislative symbols.
Assembly and should, in addition, win at 4. The registered parties are entitled for free
least two seats in that Assembly or the broadcasting or telecasting facilities.
party should win at least 3% of the total Select the correct answers using the codes
number of seats or three seats in the below
Legislative Assembly, whichever is more. (a)  2 and 4 (b)  1 and 3
2. A political party should secure at least 6% (c)  1, 3 and 4 (d)  1, 2, 3 and 4
of the total valid votes polled in a State Solution: (d)
during a general election to Lok Sabha Every party in the country has to register
and win at least one seat in the Lok Sabha with the Election Commission. While the
from that State, or the party should win at

P.430 For Civil Services Preliminary Examination

05-Indian Polity_Q1507-2004.indd 430 8/7/2018 7:49:57 PM


Commission treats all parties equally, it offers Q.1529 A registered un-recognized (by the Election
some special facilities to large and established Commission of India) political party is
parties. These parties are given a unique NOT entitled to which of the following
symbol–only the official candidates of that privileges?
party can use that election symbol. Parties 1. Access to electoral rolls
that get this privilege and some other special 2. Allocation of party symbol
facilities are ‘recognised’ by the Election 3. Provision of time for political broadcasts
Commission for this purpose. That is why on the state-owned television and radio
these parties are called, ‘recognised political stations.
parties’. The Election Commission has laid Choose the correct answer using the codes
down detailed criteria of the proportion of below:
votes and seats that a party must get in order (a)  1 and 2 (b)  1 and 3
to be a recognised party. (c) Only 3 (d) all
‘Recognised parties’ get preference in the Solution: (d)
matter of allotment of free symbols. Further, All these benefits go only to registered
registered political parties, in course of time, recognized parties. Even among these parties
can get recognition as ‘State Party’ or National there are certain privileges for national and
Party’ subject to the fulfilment of the conditions state parties
prescribed by the Commission in the Election Q.1530 Lack of “Internal democracy” within political
Symbols (Reservation and Allotment) Order, parties implies
1968, as amended from time to time. 1. Concentration of power at the top in the
Recognised ‘State’ and ‘National’ party
parties need only one proposer for filing the 2. Provincial decentralization of the party
nomination and are also entitled for two sets Which of the above is/are correct?
of electoral rolls free of cost and broadcast/ (a)  1 only (b)  2 only
telecast facilities over Akashvani (AIR)/ (c)  Both 1 and 2 (d)  None
Doordarshan during general elections. Solution: (a)
Q.1528 Consider the following statements: • Justification: All over the world there
1. The Election Commission treats all parties is a tendency in political parties towards
equally. No special facility is offered for the concentration of power in one or
large and established parties. few leaders at the top. Parties do not
2. “Recognized parties” are allotted a keep membership registers, do not hold
unique election symbol by the Election organisational meetings, and do not
Commission. conduct internal elections regularly.
3. A political party that has formed • Ordinary members of the party do not get
government in a state cannot be recognized sufficient information on what happens
as a national party. inside the party.
Choose the correct answer using the codes • They do not have the means or the
below: connections needed to influence the
(a)  1 and 2 (b)  2 and 3 decisions. As a result the leaders assume
(c)  1 and 3 (d)  2 only greater power to make decisions in the
Solution: (d) name of the party.
The Election commission offers special • Since one or few leaders exercise
facilities for large, established and thus paramount power in the party, those who
recognized parties. They are rebate on land, disagree with the leadership find it difficult
tax exemption, a publicity slot on all-India to continue in the party. More than loyalty
radio, a unique election symbol etc. to party principles and policies, personal
A party that has government in a state can loyalty to the leader becomes more
also be recognized as a national party like important.
BSP if it fulfils the specified criteria.

Indian Polity Question Bank P.431

05-Indian Polity_Q1507-2004.indd 431 8/7/2018 7:49:58 PM


(71) ELECTORAL SYSTEM legislature in proportion to the percentage of
votes that it gets. In other electoral systems,
Q.1531 Consider the following statements with especially PR systems, voters are often asked
reference to proportional representation to choose a party and the representatives
system of election: are elected on the basis of party lists. As a
1. More than one representative may be result, there is no one representative who
elected from one constituency represents and is responsible for one locality.
2. Candidate who wins the election may not In constituency based system like the FPTP,
get majority votes. the voters know who their own representative
3. Voter votes for the candidate rather than is and can hold him or her accountable.
the party in this system. Q.1532 Which of the following is\are the features of
4. Under the system the entire country may a proportional representation (PR) system of
be considered as a single constituency. elections?
5. Every party gets seats in the legislature in 1. The entire country can be considered as a
proportion to the percentage of votes that single constituency.
it gets. 2. A Voter can vote for the party rather than
Select the correct answer using the codes the candidate.
below: 3. A party generally gets more seats in
(a)  2, 3 and 4 only (b)  1, 4 and 5 only proportion to the votes it receives.
(c)  1, 2, 3 and 5 only (d)  1, 2, 3, 4 and 5 Select the correct answer using the codes
Solution: (b) below.
Each party fills its quota of seats by (a)  1 and 2 only (b)  3 only
picking those many of its nominees from a (c)  1 and 3 only (d)  1, 2 and 3
preference list that has been declared before Solution: (a)
the elections. This system of elections is
Q.1533 Out of the several kinds of proportional
called the Proportional Representation (PR)
representation system, which of the following
system. In this system a party gets the same
has been adopted in India?
proportion of seats as its proportion of votes.
1. List System
In the PR system there could be two
2. Single Transferrable Vote system
variations.
3. Mixed member proportional
In some countries, like Israel or
representation
Netherlands, the entire country is treated as
Choose the correct answer using the codes
one constituency and seats are allocated to
below.
each party according to its share of votes in
(a)  1 and 2 only (b)  2 and 3 only
the national election.
(c)  1 and 3 only (d)  1 only
The other method is when the country
Solution: (d)
is divided into several multi-member
The system of proportional representation
constituencies as in Argentina and Portugal.
aims at removing the defects of territorial
Each party prepares a list of candidates for
representation. Under this system, all
each constituency, depending on how many
sections of the people get representation in
have to be elected from that constituency. In
proportion to their number. Even the smallest
both these variations, voters exercise their
section of the population gets its due share of
preference for a party and not a candidate.
representation in the legislature.
The seats in a constituency are distributed
There are two kinds of proportional
on the basis of votes polled by a party. Thus,
representation, namely, single transferable
representatives from a constituency, would
vote system and list system. In India, the first
and do belong to different parties
kind is adopted for the election of members
Voter votes for the party rather than the
to the Rajya Sabha and state legislative
candidate (which is followed in First past the
council and for electing the President and the
Vote system). Every party gets seats in the
Vice-President.

P.432 For Civil Services Preliminary Examination

05-Indian Polity_Q1507-2004.indd 432 8/7/2018 7:49:58 PM


Though some members of the Constituent 2. Anyone can form a party or contest
Assembly had advocated the system of elections: Universal Adult Franchise
proportional representation for the election of 3. Reservation of seats for the SCs and the
members to the Lok Sabha, the Constitution STs: Open Political Competition
has not adopted the system due to two Choose the correct matches from the codes
reasons. below.
1. Difficulty for the voters to understand the (a)  2 and 3 (b)  All except 2
system (which is complicated) due to low (c)  All except 3 (d)  Only 1
literacy scale in the country. Solution: (d)
2. Unsuitability to the parliamentary Anyone can form a party or contest elections:
government due to the tendency of the Open political competition
system to multiply political parties leading Reservation of seats for the SCs and the
to instability in government. STs: Representation of weaker sections
Q.1534 Consider the following statements. Q.1537 Which of the following electoral systems
Assertion (A): The simple majority system of have been adopted for various elections in
territorial representation does not represent India ?
the whole electorate. 1. System of direct elections on the basis of
Reasons (R): It does not secure due adult suffrage.
representation to minorities or other small 2. System of proportional representation by
groups. means of the single transferable vote.
In the context of the statements above, 3. List system of proportional representation.
which of these is true? 4. Cumulative system of indirect elections.
(a) A and R both are true, and R is the correct Choose the correct answer from the codes
explanation for A. given below:
(b) A and R both are true, and R is the NOT (a)  1 and 2 (b)  1 and 3
the correct explanation for A. (c)  1, 2 and 3 (d)  2, 3 and 4
(c)  A is correct, R is incorrect. Solution: (a)
(d)  A and R both are incorrect List system is a method of voting for several
Solution: (a) electoral candidates, usually members of the
Q.1535 Consider the following statements about same political party, with one mark of the
the proportional representation system of ballot. It is used to elect the parliaments of
election. many western European countries, including
1. The entire country may be a single Switzerland, Italy, the Benelux countries,
constituency in this system. and Germany. Electors vote for one of
2. Candidate who wins the election gets several lists of candidates, usually prepared
majority of votes. by the political parties. Each party is granted
3. A party may get more seats than votes in seats in proportion to the number of popular
the legislature. votes it receives. There are several rules for
Choose the correct answer using the codes computing the number of seats awarded to
below. a party, the best known being the―d‘Hondt
(a)  1 and 2 only (b)  2 and 3 only rule and the―largest-remainder rule. Seats
(c)  1 and 3 only (d)  All of the above are usually awarded to candidates in the order
Solution: (a) in which their names appear on the lists.
The following explains the essential Although ordinarily the list system forces
differences between both systems. the voters to cast their votes for parties rather
than for individual candidates, a number
Q.1536 Consider these matches of the features of
of variations on the system permit voter
our electoral system with the principles they
preferences for individuals to be taken into
reflect.
account. The Swiss system, one of the most
1. Each constituency has roughly the same
extreme variations, is marked by panachage,
population – “One vote one value”

Indian Polity Question Bank P.433

05-Indian Polity_Q1507-2004.indd 433 8/7/2018 7:49:58 PM


the ability of the voter to mix candidates from Select the correct answer using the codes
several party lists if he so desires. below.
Q.1538 In India, the system of Proportional (a)  1 and 2 only (b)  3 only
representation is followed in the election for (c)  1 only (d)  1, 2 and 3
the Solution: (a)
1. Rajya Sabha Justification: Statement 2: The Single
2. President Transferable Vote System (STV) is followed
3. Vice-President for Rajya Sabha elections. Every State has a
4. State Legislative Council specific quota of seats in the Rajya Sabha.
Choose the correct answer using the codes The members are elected by the respective
below: State legislative assemblies. The voters are
(a)  1 and 2 only (b)  1, 2 and 3 only the MLAs in that State.
(c)  All of the above (d)  3 and 4 only • Every voter is required to rank candidates
Solution: (c) according to her or his preference. To be
In India, we have adopted PR system on a declared the winner, a candidate must
limited scale for indirect elections. secure a minimum quota of votes, which
The Constitution prescribes a third and is determined by a formula.
complex variation of the PR system for the • If after the counting of all first preference
election of President, Vice President, and for votes, required number of candidates fail to
the election to the Rajya Sabha and Vidhan fulfil the quota, the candidate who secured
Parishads. the lowest votes of first preference is
eliminated and his/her votes are transferred
Q.1539 The system of proportional representation
to those who are mentioned as second
aims at removing the defects of territorial
preference on those ballot papers. This
representation, where all sections of the
process continues till the required numbers
people get representation in proportion to
of candidates are declared elected.
their number. What types of proportional
Statement 3: MLCs are chosen in the
representation are followed in India?
following manner:
1. List system
• One-third are elected by members of
2. Single transferable vote system
local bodies such as Corporations,
3. Mixed member system
Municipalities, Gram Sabhas, Gram
Select the correct answer using the codes
Panchayats, Panchayat Samitis and Zila
below.
Parishads.
(a)  1 and 2 only (b)  2 only
• One-third are elected by members of
(c)  1 and 3 only (d)  3 only
Legislative Assemblies of the State from
Solution: (b)
among the persons who are not members
Learning: The simple majority system of
of the Assembly.
representation does not represent the whole
• One-twelfth are elected by persons who
electorate. On the other hand, even the
are graduates of 3 years’ standing residing
smallest section of the population gets its due
in that state.
share of representation in the legislature in
• One-twelfth are elected by persons
the PR system.
engaged for at least three years in teaching
There are two kinds of proportional
in educational institutions within the
representation, namely, single transferable
state not lower than secondary schools,
vote system and list system.
including colleges and universities.
Q.1540 The Single Transferable Vote system (STV) • One-sixth are nominated by the
in India is followed for the elections of Governor from persons having
1. President knowledge or practical experience in
2. Rajya Sabha MPs fields such as literature, science, arts, the
3. Members of Legislative Council (MLCs) co-operative movement and social service.

P.434 For Civil Services Preliminary Examination

05-Indian Polity_Q1507-2004.indd 434 8/7/2018 7:49:58 PM


Q.1541 Consider the following statements about the All posts whether that of a MP, MLA, Speaker,
First-Past-the-Post (FPTP) system followed Vice-President, President etc. are concerned
in India. with legislative bodies/business in India. The
1. A party which does not get the majority of FPTP is used for electing MPs. PR system is
votes casted in the elections cannot win. used for electing President. Nomination is
2. A party gets as much percentage of seats in used by President for nominating MPs to Lok
the legislature as the percentage of overall Sabha and Rajya Sabha.
votes that it has garnered in the elections.
3. Only one MP or MLA can represent
an entire parliamentary or assembly
(72) DELIMITATION OF
constituency respectively. CONSTITUENCIES
Choose the correct answer using the codes
below: Q.1543 Consider the following about the Delimitation
(a)  1 and 2 (b)  2 and 3 Commission of India.
(c)  1 and 3 (d)  Only 3 1. It is a statutory body.
Solution: (d) 2. It redraws the boundaries of both assembly
The winning candidate need not secure a and Lok Sabha constituencies.
majority of the votes. This method is called 3. Its orders cannot be challenged in a court
the First-Past-the-Post (FPTP) system. In the of law.
electoral race, the candidate who is ahead of 4. The Lok Sabha cannot modify its orders.
others, who crosses the winning post first of 5. It is setup every 5 years.
all, is the winner. This method is also called Select the correct answer using the codes
the Plurality System. below.
This is the method of election prescribed (a)  1, 2 and 5 only (b)  3, 4 and 5 only
by the Constitution. (c)  1, 2, 3 and 4 only (d)  1, 4 and 5 only
In the Lok Sabha elections of 1984, the Solution: (c)
Congress party came to power winning 415 Justification: Statement 1: It is established
of the 543 Lok Sabha seats – more than by Government of India under the provisions
80% of the seats. Such a victory was never of the Delimitation Commission Act.
achieved by any party in the Lok Sabha. Statement 2: The main task of the
The Congress party got 48% of the votes. commission is to redraw the boundaries
This means that only 48% of those who of the various assembly and Lok Sabha
voted, voted in favour of the candidates put constituencies based on a recent census.
up by the Congress party, but the party still The representation from each state is not
managed to win more than 80% of the seats changed during this exercise. However, the
in the Lok Sabha. numbers of SC and ST seats in a state are
changed in accordance with the census.
Q.1542 Choosing people for constitutional positions Statement 3 and 4: The Commission is
in legislative bodies in India can NOT happen a powerful body whose orders cannot be
by which of the following systems? challenged in a court of law. These orders
1. First-Past-the-Post (FPTP) election come into force on a date to be specified
system by the President of India in this behalf. The
2. Proportional Representation election orders are laid before the Lok Sabha and
System the respective State Legislative Assemblies.
3. Nomination However, modifications are not permitted.
4. Competitive Examinations Statement 5: Delimitation commissions
Choose the correct answer using the codes have been set up four times in the past
below. – In 1952, 1963, 1973 and 2002 under
(a)  1 and 4 only (b)  2 and 3 only Delimitation Commission acts of 1952, 1962,
(c)  3 and 4 only (d)  4 only 1972 and 2002.
Solution: (d)

Indian Polity Question Bank P.435

05-Indian Polity_Q1507-2004.indd 435 8/7/2018 7:49:58 PM


The present delimitation of parliamentary taken? This decision is taken by an
constituencies has been done on the basis of independent body called the Delimitation
2001 census figures under the provisions of Commission
Delimitation Act, 2002. A quota of constituencies to be reserved in
However, the Constitution of India was each State is fixed depending on the proportion
specifically amended in 2002 not to have of SC or ST in that State. After drawing the
delimitation of constituencies till the first boundaries, the Delimitation Commission
census after 2026. looks at the composition of population in
Thus, the present Constituencies carved out each constituency. Those constituencies that
on the basis of 2001 census shall continue to have the highest proportion of Scheduled
be in operation till the first census after 2026. Tribe population are reserved for ST.
Reference: http://eci.nic.in/delim/AboutDel In the case of Scheduled Castes, the
.pdf Delimitation Commission looks at two
Q.1544 Consider the following about the Delimitation things. It picks constituencies that have higher
Commission. proportion of Scheduled Caste population.
1. It is appointed by the President of India. But it also spreads these constituencies in
2. It works in collaboration with the Election different regions of the State. This is done
Commission of India. because the Scheduled Caste population
3. Its orders cannot be challenged in a court is generally spread evenly throughout the
of law. country.
4. Only the Parliament can modify the orders These reserved constituencies can be
of the Commission. rotated each time the Delimitation exercise is
Select the correct answer using the codes undertaken.
below. Q.1546 The decision of which constituencies to
(a)  1 and 2 only (b)  1, 3 and 4 only reserve; and the basis on which it is to be
(c)  2 only (d)  1, 2 and 3 only reserved for SCs and STs is taken by which
Solution: (d) body?
Q.1545 Consider the following statements with (a) Parliament
reference to Delimitation Commission of (b)  The Council of Ministers
India: (c)  Delimitation Commission of India
1. Delimitation Commission decides on (d)  The Election Commission of India
reservation of constituencies for scheduled Solution: (c)
castes/scheduled tribes both in house of A quota of constituencies to be reserved in
the people and Legislative Assemblies. each State is fixed depending on the proportion
2. Delimitation Commission decides on of SC or ST in that State. After drawing the
boundaries of constituencies all over the boundaries, the Delimitation Commission
country. looks at the composition of population in
3. Delimitation Commission is an each constituency. Those constituencies that
independent body. have the highest proportion of Scheduled
4. Delimitation Commission’s orders have Tribe population are reserved for ST.
the force of law and cannot be called in In the case of Scheduled Castes, the
question before any court. Delimitation Commission looks at two
Select the correct answer using the codes things. It picks constituencies that have higher
below: proportion of Scheduled Caste population. But
(a)  1 and 2 only (b)  2 and 3 only it also spreads these constituencies in different
(c)  1, 2 and 3 only (d)  1, 2, 3 and 4 regions of the State. This is done because
Solution: (d) the Scheduled Caste population is generally
Who decides which constituency is to be spread evenly throughout the country. These
reserved? On what basis is this decision reserved constituencies can be rotated each
time the Delimitation exercise is undertaken:

P.436 For Civil Services Preliminary Examination

05-Indian Polity_Q1507-2004.indd 436 8/7/2018 7:49:58 PM


Q.1547 The Delimitation Act, 2002 act and divided into different areas for purposes of
Representation of the People Act, 1950 were elections. These areas are called electoral
last amended to take into account constituencies. The voters who live in an area
(a) Increased population since 2011 Census elect one representative.
(b) Rotation of new constituencies between Each Parliamentary constituency has
male and female candidates within it several assembly constituencies.
(c) India Bangladesh land boundary The same principle applies for Panchayat and
agreement Municipal elections. Each village or town
(d) Establishment of State Election is divided into several ‘wards’ that are like
Commissions in states where they didn’t constituencies. Each ward elects one member
exist earlier of the village or the urban local body. Currently,
Solution: (c) in the Lok Sabha, 84 seats are reserved for the
Justification: section 11 of the Delimitation Scheduled Castes and 47 for the Scheduled
Act, 2002 and section 9 of the Representation Tribes (as on 1 September 2012). This number
of the People Act, 1950 were amended is in proportion to their share in the total
recently. population. Thus the reserved seats for SC and
It will enable Election Commission to ST do not take away the legitimate share of
carry out limited delimitation of Assembly and any other social group. Seats are not reserved
Parliamentary Constituencies in the Cooch for OBC’s & Minorities in Parliament.
Behar District of West Bengal consequent Q.1549 Consider the following statements:
upon the exchange of 51 Bangladeshi enclaves 1. The number of parliamentary constituencies
and 111 Indian enclaves respectively between in a state cannot be lesser than the number
India and Bangladesh. of assembly constituencies.
Learning: Under Article 82 of the 2. A parliamentary constituency can span
Constitution, the Parliament by law enacts a across more than one district in a state of
Delimitation Act after every census. India.
• After coming into force commencement Which of these is/are true?
of the Act, the Central Government (a)  1 only (b)  2 only
constitutes a Delimitation Commission. (c)  Both 1 ad 2 (d)  None of the above
• The present delimitation of constituencies Solution: (b)
has been done on the basis of 2001 See the diagram in NCERT book for Gulbarga
census figures under the provisions of Lok Sabha Constituency and the Gulbarga
Delimitation Act, 2002. District in Karnataka.
Q.1548 India is divided into number of constituency Q.1550 Consider the following statements:
for electing it’s representatives. Consider the 1. Members of the SC and ST community can
following statements: fight elections only from constituencies
1. Each Parliamentary constituency has reserved for them.
within it several assembly constituencies 2. In the Gram Panchayat elections, women can
2. Each village or town is divided into several stand in election only from constituencies
‘wards’ that are like constituencies. which are reserved for women.
3. Some Parliamentary constituencies are Which of these is/are true?
reserved for people who belong to the (a)  1 only (b)  2 only
Scheduled Castes (SC), Scheduled Tribes (c)  Both 1 ad 2 (d)  None of the above
(ST) and Other Backward Class (OBC) Solution: (d)
Which of the statements is/are correct? Constituencies are of two types – general and
(a)  1 Only (b)  1 and 3 reserved. In a general constituency any one
(c)  1 and 2 (d)  1, 2 and 3 can stand and fight elections. In a reserved
Solution: (c) constituency – in any election, local, general
In our country we follow an area based etc. – candidates belonging to the certain
system of representation. The country is community can only stand for the election.

Indian Polity Question Bank P.437

05-Indian Polity_Q1507-2004.indd 437 8/7/2018 7:49:58 PM


(73) MODEL CODE OF CONDUCT Choose the correct answer using the codes
below:
Q.1551 The ‘Model Code of Conduct (MCC)’ has (a)  1 and 2 (b)  2 and 3
been issued by the (c)  1 and 3 (d)  All of the above
(a)  Supreme Court of India Solution: (b)
(b)  Election Commission of India MCC is not a law. It is an informal agreement
(c)  President of India between the political parties.
(d) Parliament Q.1553 The Model Code of Conduct for guidance
Solution: (b) of candidates and political parties comes
Explanation & Learning: MCC is not a immediately into effect
legally binding document. Hence, SC cannot (a) After the Election Commission of India
issue or enforce it. Option (a) is wrong. announces the election schedule in a
What is it? major press conference
It is a set of guidelines laid down by the (b) After the Election Commission of India
Election Commission to govern the conduct issues a specific order enforcing the
of political parties and candidates in the run- Model code of conduct
up to an election. (c) Automatically exactly one month before
What is the need for such a code of the election dates
conduct? (d) After the deadline for nominating
1. It is intended to provide a level playing candidates ends
field for all political parties, to keep the Solution: (a)
campaign fair and healthy, avoid clashes The Commission normally announces
and conflicts between parties, and ensure the schedule of elections in a major press
peace and order. Its main aim is to ensure conference a few weeks before the formal
that the ruling party, either at the Centre process is set in motion. The Model Code
or in the states, does not misuse its official of Conduct for guidance of candidates and
position to gain an unfair advantage in political parties comes immediately into
an election. effect after such announcement.
2. The Model Code of Conduct comes into The formal process for the elections starts
force the moment an election is announced with the Notification or Notifications calling
and remains in force till the results are upon the electorate to elect Members of a
declared. House. As soon as Notifications are issued,
3. For example, one provision of the MCC candidates can start filing their nominations
says, “No party or candidate shall include in the constituencies from where they wish
in any activity which may aggravate to contest.
existing differences or create mutual These are scrutinized by the Returning
hatred or cause tension between different Officer of the constituency concerned after
castes and communities, religious or the last date for the same is over after about
linguistic” a week. The validly nominated candidates
Q.1552 Consider the following statements about can withdraw from the contest within
the Model Code of Conduct (MCC) with 2 days from the date of scrutiny. Contesting
reference to elections in India. candidates get at least two weeks for political
1. It has been enacted by the Parliament campaign before the actual date of poll.
to ensure a level playing field between Q.1554 Which of the following is NOT a part of the
candidates/political parties. “Model Code of Conduct for the Guidance of
2. The Election Commission of India Political Parties and Candidates”, formulated
enforces the MCC. by the Election Commission of India?
3. The ECI can take disciplinary action (a) No political party can criticise the other
against the candidates/political parties political party or its candidates after the
which violate the MCC. notification of the election schedule.

P.438 For Civil Services Preliminary Examination

05-Indian Polity_Q1507-2004.indd 438 8/7/2018 7:49:58 PM


(b) There shall be no appeal to caste or laws or the Model Code of Conduct
communal feelings for securing votes. (MCC)?
(c) The Ministers shall not use government 1. A minister flags off a new train in his
resources for furthering the interest of constituency a week before polling
their political parties solely. day.
(d) Issue of advertisement at the cost of 2. A candidate promises that she will get a
public exchequer in media to further new train for her constituency if she is
the interest of a particular political party elected.
should be avoided during elections. 3. Supporters of a candidate take the voters
Solution: (a) to a temple and make them take an oath
Justification: It provides for the following that they will vote for him.
salient points: 4. The supporters of a candidate distribute
1. No party or candidate shall indulge in any blankets in slums in return for a promise
activity which may aggravate existing for vote.
differences or create mutual hatred or Choose the correct options from the codes
cause tension between different castes below:
and communities, religious or linguistic. (a)  All of the above (b)  1, 3 and 4
So, (b) is correct. (c)  3 only (d)  1 and 4 only
2. Criticism of other political parties, when Solution: (b)
made, shall be confined to their policies The option 2 is not wrong as it can be a part
and programmes, past record and work. of the Election Manifesto. It is a part of the
Parties and candidates shall refrain from policy and to-do list of the candidate if he is
criticism of all aspects of private life, elected. It is a fair practice.
not connected with the public activities The other three are unfair. The options 1
of the leaders or workers of other and 3 are violations of the MCC. The option
parties. Criticism of other parties or their 4 is clearly bribing the voters.
workers based on unverified allegations Q.1556 Which of the following activities are
or distortions shall be avoided. So, the prohibited by the Model Code of Conduct
option (a) is incorrect. as soon as the election are notified by the
3. The Ministers shall not combine their Election Commission of India?
official visit with electioneering work 1. Using any place of worship for election
and shall not also make use of official propaganda.
machinery or personnel during the 2. Using government vehicles, officials and
electioneering work money for elections.
4. Government transport including official 3. Once elections are announced, Ministers
air-crafts, vehicles, machinery and shall not lay foundation stones of any
personnel shall not be used for furtherance projects, take any big policy decisions or
of the interest of the party in power. So, make any promises of providing public
the option (c) is also correct. facilities.
5. Issue of advertisement at the cost of Choose the correct answer using the codes
public exchequer in the newspapers and below:
other media and the misuse of official (a)  1 and 2 (b)  2 and 3
mass media during the election period (c)  1 and 3 (d)  All of the above
for partisan coverage of political news Solution: (d)
and publicity regarding achievements Explained in previous questions.
with a view to furthering the prospects of
Q.1557 Which of the following are prohibited by
the party in power shall be scrupulously
the election laws of India with reference to
avoided. So, the option (d) is also correct.
elections?
Q.1555 Which of the following would be counted 1. Appealing to voters in the name of caste
as unfair practices under the Indian election or religion

Indian Polity Question Bank P.439

05-Indian Polity_Q1507-2004.indd 439 8/7/2018 7:49:58 PM


2. Use government resources for election (c) The Election Commission of India
campaign thinks it to be in the best-interest of the
3. Bribe or threaten voters nation.
Choose the correct answer using the codes (d) There is no decisive winner in a general
below: election
(a)  1 and 2 (b)  2 and 3 Solution: (b)
(c)  1 and 3 (d)  All of the above Explanation & Learning: In most cases
Solution: (d) these elections occur when the incumbent
Self-explanatory as these disturb the level died or resigned, but they also occur when
playing field between political parties and the incumbent becomes ineligible to continue
candidates. In addition to this mala fide in office (because of a recall, ennoblement,
activities are regulated by the Model Code criminal conviction, or failure to maintain
of conduct. a minimum attendance). Less commonly,
these elections have been called when a
constituency election is invalidated by voting
(74) ELECTIONS irregularities.
Q.1558 Which of these may NOT be a good reason Q.1560 Consider the following statements:
to say that elections in a certain country are 1. The state legislatures can also enact
democratic? laws relating to the elections to the
(a)  If it has very large voter base. state legislatures in so far that they are
(b) If an independent and powerful body consistent with the provisions enacted by
supervises elections. the Parliament.
(c) If all candidates have a fair chance of 2. The election petitions concerning the
competing and winning in elections. Parliament can only be entertained by the
(d) If the losing parties peacefully accept the Supreme Court.
electoral verdict. Which of these is/are true?
Solution: (a) (a)  Only 1 (b)  Only 2
Explanation: There may be confusion (c)  Both (d)  None of the above
between option (a) and (c). Option (a) is more Solution: (a)
appropriate because merely a large voter base The high courts and other authorities as
does not ensure successful and democratic provided by the Parliament can also entertain
elections. For e.g. Iraq may have a large voter election petitions
base, many agencies alleged that its 2003
Q.1561 Given below are some of the offices in the
elections were not democratic. Malpractices,
Indian polity. For which of the following
rigging etc. can take place in countries with
offices provisions regarding the appointment/
large voter base. In such cases, elections will
elections is/are not mentioned in the
not be fair and democratic.
constitution of India?
Options (c) and (d) show the political
1. Deputy Chairman of Rajya Sabha.
situation in the country. For e.g. if in
2. Solicitor general of India.
Pakistan, the incumbent government does
3. Lok Sabha Secretary
not have any chances of losing (as it controls
4. Advocate General
state institutions), the elections may not be
Choose the correct answer using the codes
considered democratic!
below:
To ensure the above option (b) is necessary.
(a)  1 and 4 only (b)  2 and 3 only
Q.1559 A ‘by-election’ is held when (c)  1 and 3 only (d)  2, 3 and 4
(a) A party candidate during elections Solution: (b)
withdraws or dies There is no provision for the Lok Sabha
(b) Vacancy caused by death or resignation secretary specifically in the constitution.
of a legislator is to be filled between It only talks about the secretarial staff via
general elections Article 98.

P.440 For Civil Services Preliminary Examination

05-Indian Polity_Q1507-2004.indd 440 8/7/2018 7:49:58 PM


Article 98 (1) depicts that “Each House of before the election and given to everyone.
Parliament shall have a separate secretarial This list is officially called the Electoral
staff: Provided that nothing in this clause shall Roll and is commonly known as the
be construed as preventing the creation of Voters’ List.
posts common to both Houses of Parliament.” 2. A complete revision of the Electoral Roll
Constitution has no provision for solicitor takes place every 5 years.
general of India. 3. Anyone who can be a voter can also
Q.1562 With regard to elections consider the become a candidate in elections and
following statements the candidate’s minimum age should be
1. Some non-democratic countries also hold 18 years.
elections. Select the correct answer using the codes
2. Sometimes election is held only for one given below
constituency to fill the vacancy caused by (a)  1 and 3 (b)  1 Only
death or resignation of a member. This is (c) 2 Only (d) None
called a mid- election. Solution: (b)
3. In India we follow a system of proportional In a democratic election, the list of those who
representation for the election of members are eligible to vote is prepared much before
of Lok Sabha. the election and given to everyone. This list
4. Each constituency should have a roughly is officially called the Electoral Roll and is
equal population living within it. commonly known as the Voters’ List.
Which of the statements given above is/are Anyone who can be a voter can also
correct? become a candidate in elections. The only
(a)  2 and 4 (b)  1 and 4 difference is that in order to be a candidate
(c)  1, 2 and 3 (d)  1, 2, 3 and 4 the minimum age is 25 years, while it is only
Solution: (b) 18 years for being a voter. There are some
Elections can be held in many ways. All other restrictions on criminals etc. but these
democratic countries hold elections. But apply in very extreme cases
most non-democratic countries also hold Which of these is NOT a good reason to
some kind of elections. say that Indian elections are democratic?
Sometimes election is held only for one (a) India has the largest number of voters in
constituency to fill the vacancy caused by the world.
death or resignation of a member. This is (b) India’s Election Commission is very
called a ‘by-election’. powerful.
In India we follow a system of territorial (c) In India, everyone above the age of
representation/area-based representation 18 has a right to vote.
for the election of members of Lok Sabha. (d) In India, the losing parties accept the
And in Rajya Sabha we follow proportional electoral verdict.
representation. Solution: (a)
One of the features of a democratic Even China has a very large number of voters
election is that every vote should have equal in its elections. But that does not make China
value. That is why our Constitution requires a democracy.
that each constituency should have a roughly The existence of a democracy is not
equal population living within it decided solely by the number of voters or
existence of voters. There are several other
Q.1563 India has a federal government, with elected
criteria such as: free and fair elections;
representatives at the central, state and local
decision-making power with the people;
levels. Which of the following statements
freedom of speech and expression; open
are incorrect with regard to elections held
political competition etc.
in India?
1. In a democratic election, the list of those Q.1564 Consider the following about Elections
who are eligible to vote is prepared much in India.

Indian Polity Question Bank P.441

05-Indian Polity_Q1507-2004.indd 441 8/7/2018 7:49:58 PM


1. The recommendation for elections is • The validly nominated candidates can
made by the Government of India. withdraw from the contest within 2 days
2. The announcement of elections is done by from the date of scrutiny.
the Election Commission of India. • Contesting candidates get at least 2 weeks
3. The model code of conduct is enforced by for political campaign before the actual
the Government of India. date of poll.
4. The election conducting machinery is not • On account of the vast magnitude of
controlled by the government. operations and the massive size of the
Choose the correct answer using the codes electorate, polling is held at least on
below. 3 days for the national elections.
(a)  1 and 2 only (b)  1, 3 and 4 only • A separate date for counting is fixed and
(c)  2 only (d)  1, 2 and 4 only the results declared for each constituency
Solution: (d) by the concerned Returning Officer.
The recommendation for election is made by • The Commission compiles the complete
the government and the notification for election list of Members elected and issues an
is issued by the Election Commission. After appropriate Notification for the due
the notification, the Election Commission Constitution of the House. With this, the
of India announces the schedule, and starts process of elections is complete and the
accepting nomination papers of candidates. President, in case of the Lok Sabha, and
After announcing the schedule, the model the Governors of the concerned States, in
code of conduct comes automatically in force case of Vidhan Sabhas, can then convene
that prohibits certain kind of political activities their respective Houses to hold their
to ensure a level playing field. sessions.
Q.1565 Arrange these election related activities in • The entire process takes between 5 to
the correct order from the earlier to the later. 8 weeks for the national elections, 4 to
1. Announcing election schedule 5 weeks for separate elections only for
2. Election Campaign Legislative Assemblies.
3. Filing nominations for elections Q.1566 The Election Commission of India (ECI)
Choose the correct answer from the codes compiles the complete list of members
below: who are elected in General elections. After
(a) 123 (b) 132 the elections are complete, who issues the
(c) 312 (d) 321 notification for the due constitution of the
Solution: (b) Lok Sabha?
• The Commission normally announces (a)  President of India
the schedule of elections in a major Press (b)  Former Council of Ministers
Conference a few weeks before the formal (c) Election Commission of India (ECI)
process is set in motion. The Model Code (d) Leader of the majority party/coalition
of Conduct for guidance of candidates and Solution: (c)
Political Parties immediately comes into Learning: The ECI issues an appropriate
effect after such announcement. notification for the due constitution of the
• The formal process for the elections starts House. With this, the process of elections is
with the Notification or Notifications complete and the President, in case of the Lok
calling upon the As soon as Notifications Sabha, can then convene the house to hold
are issued, Candidates can start filing its sessions. The Governors of the concerned
their nominations in the constituencies states, in case of State Assemblies, convene
from where they wish to contest. These the sessions
are scrutinised by the Returning Officer Q.1567 Consider the following
of the constituency concerned after the 1. The Model Code of Conduct (MCC)
last date for the same is over after about comes immediately into effect after the
a week. dissolution of Lok Sabha.

P.442 For Civil Services Preliminary Examination

05-Indian Polity_Q1507-2004.indd 442 8/7/2018 7:49:58 PM


2. After the general elections are Hence, if the number of candidates
complete, President of India issues the is very large, very few will be able to secure
notification for the due constitution of the one-sixth of votes. Others will lose the deposit.
Lok Sabha. Q.1569 Who among the following can vote in the
Which of the above is/are correct? elections to Lok Sabha, Rajya Sabha as well
(a)  1 only (b)  2 only as State Legislative Council?
(c)  Both 1 and 2 (d)  None (a)  Registered voters of the country
Solution: (d) (b) Elected members of State Legislative
Justification: Statement 1: The Commission Assembly
normally announces the schedule of elections (c)  President and Attorney-general
in a major press conference a few weeks (d)  Chief Minister of a state
before the formal process is set in motion. Solution: (b)
• The Model Code of Conduct comes Voters cannot choose members of Rajya
immediately into effect after such Sabha and State Legislative Council. They
announcement. are indirectly elected by members of the state
• The formal process for the elections starts LA (almost all in case of RS, and 1/3rd in case
with the Notification or Notifications of SLC).
calling upon the electorate to elect Chief Minister of a state cannot vote for
Members of a House. Rajya Sabha and State Legislative Council, if
Statement 2: Refer previous explanation. he is a member of State Legislative Council,
Q.1568 If in an election to a State Legislature and not State Legislative Assembly.
Assembly the candidate who is declared Attorney General has right to attend
elected loses his deposit, it means that proceeding of parliament but he/she has no
(a)  the polling was very poor right to vote.
(b)  the election was for a multi-member President is an integral part of Parliament
constituency S/he addresses the joint seating of Parliament.
(c) the elected candidate’s victory over his Only the Elected members choose 1/3rd of
nearest rival was very marginal the SLC members, as well as choose Rajya
(d)  a very large number of candidates Sabha members, as well as vote as registered
contested the election voters for Lok Sabha elections.
Solution: (d) Q.1570 Consider the following with reference to
The deposit made by a candidate shall be election outcomes in India.
returned if the following conditions are 1. Majority of votes by a political party
satisfied: (i) the candidate is not shown necessarily mean majority of seats.
in the list of contesting candidates, that is 2. Majority of seats garnered by a political
to say, either his nomination was rejected party necessarily mean majority of votes.
or after his nomination was accepted, he Which of the above is/are true?
withdrew his candidature and/or (ii) he dies (a)  1 only (b)  2 only
before the commencement of the poll and/or (c)  Both 1 and 2 (d)  None
(iii) he is elected and/or (iv) he is not elected Solution: (d)
but gets more than 1/6th of the total no. of A simple case of majority seats not meaning
valid votes polled by all the candidates at majority votes is Modi Government’s win
the election. by a majority of seats but only around 38%
Note: (1) If the candidate has polled votes. On the other hand, in Delhi, the Aam
exactly 1/6th of the total number of valid Admi Party (AAP) secured 54.4% of votes,
votes polled by all the candidates, the deposit and won nearly 95% seats. So majority of
will not be refunded. (2) if the candidate was seats does not mean necessarily majority
elected, the deposit will be refunded even if of votes. The opposite is also true. A party
he did not poll more than 1/6th of the total can get majority of votes, yet it may not
valid votes polled by all the candidates. get majority of seats, because it may not

Indian Polity Question Bank P.443

05-Indian Polity_Q1507-2004.indd 443 8/7/2018 7:49:58 PM


win by a huge margin in all electoral with that State Government / Union Territory
constituencies. Administration.
Q.1571 The declaration which outlines the future The Election Commission of India
programme and policy of a political party nominates officers of Government as
issued on the eve of a general election is Observers (General Observers and Election
called: Expenditure Observers) for Parliamentary
(a) white paper (b) manifesto and assembly constituencies. They perform
(c) yellow paper (d) mandate such functions as are entrusted to them by
Solution: (b) the Commission. They report directly to the
Commission.
Q.1572 Which of the following is correct regarding
booth capturing? Q.1574 Elections are being conducted almost every
1. It has been defined in the Constitution year in India. Elaborate machinery is required
after the 61st amendment. for election management. Who acts as the
2. It includes the seizure of a polling District returning Officer in the Districts in
booth to prevent the orderly conduct of these elections?
elections. (a)  District Collector
3. It is also committed when any elector is (b)  Divisional Commissioner as the Chief
threatened and prevented from going to electoral Officer is in charge of the
the polling station to cast his vote. Districts under him.
4. It has been declared a cognizable offence (c)  An Officer appointed by the District
punishable by imprisonment. Magistrate
(a)  2, 3, and 4 (b)  1, 2 and 3 (d)  An Officer other than the District
(c)  2 and 3 (d)  1, 2, 3 and 4 Magistrate appointed by the ECI.
Solution: (a) Solution: (a)
Self explanatory The Election Commission is assisted by
deputy election commissioners. They are
drawn from the civil service and appointed
Election Officers by the commission with tenure system. They
Q.1573 Which of the following government officials are assisted, in turn, by the secretaries, Joint
are the parts of the machinery of Election Secretaries, deputy secretaries and Under
Commission of India that conducts and Secretaries posted in the secretariat of the
supervises elections in India? commission.
1. Observers At the state level, the Election Commission
2. Presiding officer is assisted by the chief electoral officer who is
3. Returning Officer appointed by the chief election commissioner
4. Chief Electoral Officer in consultation with the state government.
Choose the correct answer from the codes Below this, at the district level, the collector
given below. acts as the district returning officer. He appoints
(a)  1 and 4 only (b)  2 and 4 only a returning officer for every constituency in
(c)  1 and 3 only (d)  All of the above the district and presiding officer for every
Solution: (d) polling booth in the constituency.
The Chief Electoral Officer of a State/Union Q.1575 The Returning Officer of a Parliamentary or
Territory is authorized to supervise the assembly constituency is responsible for the
election work in the state/Union Territory conduct of elections in the Parliamentary or
subject to the overall superintendence, assembly constituency concerned. How is
direction and control of the Election she appointed?
Commission. The Election Commission of (a) The Election Commission of India (ECI)
India nominates or designates an Officer of nominates or designates an officer of
the Government of the State / Union Territory the Government in consultation with the
as the Chief Electoral Officer in consultation State government

P.444 For Civil Services Preliminary Examination

05-Indian Polity_Q1507-2004.indd 444 8/7/2018 7:49:58 PM


(b) The District Magistrate nominates from (b) “Exit Poll” and “Opinion Poll” are one
amongst the officers of the Constituency and the same
in consultation with the ECI (c) “Exit Poll” is a device through which
(c)  The Chief Electoral Officer (CEO) results of voting can be most exactly
appoints her based on the recommendation predicted
of the District Magistrate (d) “Exit Poll” is an administrative device
(d) The State Government appoints her from made recently by the Chief Election
amongst the polling officers within the Commissioner to prevent impersonation.
constituency in consultation with the ECI Solution: (a)
Solution: (a) An election exit-poll is a poll of voters taken
Learning: This is the hierarchy of the immediately after they have exited the polling
election machinery. stations. Unlike an opinion poll, which asks
1. The Chief Electoral Officer of a State/ whom the voter plans to vote for or some
Union Territory is authorized to similar formulation, an exit poll asks whom
supervise the election work in the state/ the voter actually voted for. A similar poll
Union Territory subject to the overall conducted before actual voters have voted
superintendence, direction and control of is called an entrance poll. Pollsters, mostly
the Election Commission. private companies working for newspapers
2. Subject to the superintendence, direction or broadcasters, use to conduct the exit polls
and control of the Chief Electoral Officer, to gain an early indication as to how an
the District Election Officer supervises election has turned out, as in many elections
the election work of a district. the actual result may take hours or even days
3. Next lies Returning officer (RO) and to count.
Presiding officer. There was a widespread controversy
4. The Presiding Officer with the assistance during the Indian general election, 2014
of polling officers conducts the poll at a when the Election Commission of India
polling station. barred media organisations from displaying
5. The District Election Officer appoints the exit poll results until the votes had been
Presiding Officers and the Polling Officers. counted. This was followed by a strong
Q.1576 Who is a Returning Officer? [Teachers’ protest from the media which caused the
Exam, 1993] Election Commission to withdraw its
(a)  An officer who returns the unaccepted statement and that the exit polls can be
applications for allotment of land shown at 6:30 pm on 12th may after the last
(b)  An officer who is sent back to his parent vote is cast.
department Q.1578 Which one of the following statements
(c)  An officer who is head of the State correctly differentiates between ‘Exit Poll’
Assembly Secretariat and ‘Opinion Poll’?
(d)  An officer who is responsible for conduct 1. ‘Exit Polls’ are used by the ECI to prevent
of election in a constituency and declares impersonation and forgery in the electoral
the results process, whereas opinion polls are used
Solution: (d) before the elections to gauge popular mood.
2. The final result of elections is derived
Polls from ‘Exit Polls’, whereas ‘opinion polls’
do not carry administrative value apart
Q.1577 Which one of the following statements from allowing general public to express
regarding “Exit Poll” is correct? its opinion.
(a) “Exit Poll” is a term used to denote a Which of the above is/are correct?
post-election survey of voters regarding (a)  1 only (b)  2 only
the candidate in whose favour they had (c)  Both 1 and 2 (d)  None
exercised their franchise Solution: (d)

Indian Polity Question Bank P.445

05-Indian Polity_Q1507-2004.indd 445 8/7/2018 7:49:58 PM


Justification: Statement 1: An election exit 5. The above mandates are in the spirit of
poll is a poll of voters taken immediately Article 19.
after they have exited the polling stations. Which of the statements is/are correct?
Unlike an opinion poll, which asks for whom (a)  1, 2 and 5 (b)  1, 2 and 3
the voter plans to vote, or some similar (c) 1, 2, 3 and 4 (d) 1, 2, 3, 4 and 5
formulation, an exit poll asks for whom the Solution: (a)
voter actually voted. Every person who wishes to contest an
Statement 2: However, every voter may election has to fill a ‘nomination form’ and
not reveal his correct preferences. So, result give some money as ‘security deposit’.
is based only on counting of votes in the Recently, a new system of declaration
ballot box or via the EVM. has been introduced on direction from the
Opinion polls are usually designed to Supreme Court. Every candidate has to make
represent the opinions of a population by a legal declaration, giving full details of:
conducting a series of questions and then • Serious criminal cases pending against the
extrapolating generalities in ratio or within candidate;
confidence intervals. • assets and liabilities of the candidate and
Q.1579 Which of the following correctly points the his or her family
difference between Exit polls and Opinion • Education qualifications of the candidate.
Polls? This information has to be made public.
1. Exit polls are not banned, but opinion This provides an opportunity to the voters
polls are banned in India. to make their decision on the basis of the
2. Exit polls are taken after the elector votes information provided by the candidates.
and exits from a particular polling booth; Right to Information is a part of
opinion polls are taken before the election. fundamental rights under Article 19(1) of the
3. Exit polls are conducted by the Election Constitution. Article 19(1) says that every
Commission of India, whereas opinion citizen has freedom of speech and expression.
polls are conducted by private media In 1976, the Supreme Court said in the case
agencies. of Raj Narain v. State of UP, 1975 AIR
Choose the correct answer from the codes 865, that people cannot speak or express
given below. themselves unless they know. Therefore, right
(a)  1 and 2 only (b)  2 and 3 only to information is embedded in article 19.
(c)  1 and 3 only (d)  2 only Q.1581 The order/direction of making mandatory
Solution: (d) for every candidate (contesting elections) to
file an affidavit giving details of his property
and criminal cases pending against him was
(75) ELECTORAL REFORMS given by?
Q.1580 On the basis of landmark judgments (a)  Election Commission of India
passed by the Supreme Court, the Election (b)  Supreme Court
Commission of India came up with the (c) Parliament
following mandates: (d)  The Prime Minister’s office (PMO)
1. Every candidate has to make a legal Solution: (b)
declaration, giving full details of serious The ECI only implements whatever directives
criminal cases pending against the re framed. It does not have the power to issue
candidate such powerful directives. This directive came
2. Details of the assets and liabilities of the out in a Supreme Court judgement.
candidate and his or her family Q.1582 Which of the following are legally mandatory
3. Education qualifications was not for political parties in or political candidates
mandatory fighting elections in India?
4. The information furnished by the 1. Giving a certain percentage of election
candidates has to be confidential. tickets to women candidates

P.446 For Civil Services Preliminary Examination

05-Indian Polity_Q1507-2004.indd 446 8/7/2018 7:49:58 PM


2. A record of the adherence of a political Select the correct answer using the codes
party to its own Constitution below.
3. Submit an affidavit giving details of his (a)  1 and 3 only (b)  2 and 4 only
property and criminal cases pending (c)  1, 2 and 3 only (d)  2, 3 and 4 only
against him to the concerned authority Solution: (c)
Select the correct answer using the codes Justification: The Voter Verified Paper
below. Audit Trail is attached to electronic voting
(a)  1 and 2 only (b)  2 only machines.
(c)  3 only (d)  1 and 3 only This slip contains:
Solution: (c) 1. the poll symbol and
Justification: These are some of the 2. serial number and name of the candidate
compulsory provisions: only.
• The Constitution was amended to prevent It allows the voter to verify his/her choice.
elected MLAs and MPs from changing After being visible to the voter from a
parties. glass case in the VVPAT for seven seconds,
• The Supreme Court passed an order the ballot slip will be cut and dropped into
to reduce the influence of money and the drop box in the VVPAT machine and a
criminals. Now, it is mandatory for every beep will be heard. VVPAT machines can be
candidate who contests elections to file an accessed by polling officers only.
affidavit giving details of his property and Q.1584 Consider the following about Election
criminal cases pending against him. Commission of India – Electronic Voting
• The Election Commission passed an order Machines (ECI-EVMs).
making it necessary for political parties to 1. ECI procures most Completely-Built
hold their organisational elections and file Units (CBUs) of EVMs from abroad to
their income tax returns reduce security exposure to domestic
Besides these, many suggestions are often clients.
made to reform political parties: 2. The EVMs use ‘date and time stamping
• A law should be made to regulate the of each and every key press’ making them
internal affairs of political parties. It tamper proof.
should be made compulsory for political 3. Most EVMs are re-programmable
parties to maintain a register of its allowing much needed flexibility
members, to follow its own constitution, to incorporate new features without
to have an independent authority, to act as incurring new production expenditures.
a judge in case of party disputes, to hold Select the correct answer using the codes
open elections to the highest posts. below.
• It should be made mandatory for political (a)  2 only (b)  1 and 3 only
parties to give a minimum number (c)  1 only (d)  2 and 3 only
of tickets, about one-third, to women Solution: (a)
candidates. Similarly, there should be a Justification: Recently, there have been some
quota for women in the decision making queries and allegations about the security
bodies of the party. features of Electronic Voting Machines
Q.1583 When a voter presses a button in the (EVMs). Political parties have questioned on
Electronic Voting Machine (EVM), a paper the tamper proof nature of EVMs. However,
slip is printed through the Voter Verifiable ECI published a clarification note on this.
Paper Audit Trail (VVPAT) Machine. This Statement 1: Contrary to misinformation
slip contains the and as alleged by some, India do not use any
1. Name and caste of the Voter EVMs produced abroad. EVMs are produced
2. Poll symbol of the voted candidate indigenously by PSUs in India. However, the
3. Name of the voted candidate chips used in EVMs are sourced from abroad
4. Address where the EVM is installed because we don’t have the capability of

Indian Polity Question Bank P.447

05-Indian Polity_Q1507-2004.indd 447 8/7/2018 7:49:58 PM


producing semi-conductor microchips within 5. In this regard special Camps will be
the country. However, the software used in organized, Voter Facilitation Centres,
these chips is written domestically. e-Seva centres and Citizen Service
Statement 2: The ECI-EVMs use some Centres. While Booth Level Officers will
of the most sophisticated technological conduct door-to-door surveys to collect
features like one time programmable (OTP) the details.
microcontrollers, dynamic coding of key Q.1586 Reforming political parties is one of the
codes, date and time stamping of each aspects of good governance. Which of the
and every key press, advanced encryption following is/are a step in this direction?
technology and EVM-tracking software to 1. The Parliament enacted anti-defection
handle EVM logistics, among others to make law
the machine 100% tamper proof. 2. On Supreme Courts orders, it’s now
Statement 3: Since, software is based mandatory for every candidate who
on OTP the program cannot be altered, contests elections to file an Affidavit
re-written or Re-read. Thus, making EVM giving details of his property and
tamper proof. If anyone make, attempt, the criminal cases pending against him.
machine will become inoperative. 3. The Election Commission passed an order
Q.1585 Election Commission of India (ECI) had making it necessary for political parties to
launched National Electoral Roll Purification hold their organizational elections and file
and Authentication Programme (NERPAP). their income tax returns.
Under the programme 4. The political parties do not come under
(a) Voter ID card of voters will be linked the ambit of Right to Information Act.
with Aadhaar data. Select the correct answers using the codes
(b) Voter ID card of voters will be re-issued below
after redundancy check and verification. (a)  1 and 4 (b)  1, 2 and 4
(c) Voter ID card of voters will be stored (c)  1, 2 and 3 (d)  1, 2, 3 and 4
online and new cards will be allotted Solution: (a)
online. Q.1587 State funding of elections takes place in
(d)  None of the above [IAS 1997]
Solution: (a) (a)  USA and Canada
1. Objective is to bring out a totally error- (b)  Britain and Switzerland
free and authenticated electoral roll (c)  France and Italy
throughout the country. (d)  Germany and Austria
2. For the authentication purpose, Electoral Solution: (d)
Photo Identity Card (EPIC) data of
electors will be linked with Aadhaar
data. (76) ANTI-DEFECTION LAW
3. It also focuses to improve the image Q.1588 The Tenth Schedule — popularly known
quality of electors along with sorting as the Anti-Defection Act was added to the
issues like corrections of errors. Facility Constitution through which of the following
to link Aadhaar number will be provided amendments?
to electors through SMS, E-mail, mobile (a)  51st Amendment
application and National Voters Service (b)  52nd Amendment
Portal using web services through ECI (c)  53rd Amendment
website. (d)  54th Amendment
4. Electors also can link their Aadhaar Solution: (b)
number by making a call at 1950 to state • What is the Anti-Defection Law?
call centres. Under NERPAP, collection The Tenth Schedule—popularly known
and feeding of Aadhaar will also be done as the Anti-Defection Act—was included
by Electoral Registration Officer. in the Constitution in 1985 by the Rajiv

P.448 For Civil Services Preliminary Examination

05-Indian Polity_Q1507-2004.indd 448 8/7/2018 7:49:58 PM


Gandhi ministry and sets the provisions party merges with another; if a new political
for disqualification of elected members party is formed by some of the elected
on the grounds of defection to another members of one party; if he or she or other
political party. members of the party have not accepted the
The law was added via the 52nd merger between the two parties and opted to
Amendment Act, 1985, soon after the Rajiv function as a separate group from the time of
government came to power with a thumping such a merger.
majority in the wake of the assassination of • What are the powers of a party whip
Prime Minister Indira Gandhi. The Congress under the Constitution in case of a
had won 401 seats in the Lok Sabha. defection?
• What are the grounds for disqualification The whip upholds the party directives in the
under the Anti-Defection Law’s Articles House as the authorised voice of the party.
102(2) and 191(2)? On defection of elected members of his party,
(a) If an elected member voluntarily gives the whip can send a petition on the alleged
up his membership of a political party; defection to the Chairman or the Speaker of
(b) If he votes or abstains from voting in such a House for their disqualification. He can also
House contrary to any direction issued by expel the members from the party. But this
his political party or anyone authorised to does not necessarily mean that the members
do so, without obtaining prior permission. so expelled lose their seats in the House. They
As a pre-condition for his disqualification, continue to hang on to their seats as long as the
his abstention from voting should not be Chairman or the Speaker of a House gives a
condoned by his party or the authorised final decision on their disqualification from the
person within 15 days of such incident. House after a proper enquiry on the basis of the
• What were the loopholes? petition filed by the party whip.
As per the 1985 Act, a ‘defection’ by one-third • What are the options before a
of the elected members of a political party was disqualified elected member?
considered a ‘merger’. Such defections were The members so disqualified can stand for
not actionable against. The Dinesh Goswami elections from any political party for a seat in
Committee on Electoral Reforms, the Law the same House. But he, naturally cannot get
Commission in its report on “Reform of a ticket from his former party.
Electoral Laws” and the National Commission • Who is the deciding authority?
to Review the Working of the Constitution The decision on questions as to
(NCRWC) all recommended the deletion of the disqualification on ground of defection is
Tenth Schedule provision regarding exemption referred to the Chairman or the Speaker
from disqualification in case of a split. of such House, and his decision is final.
Finally the 91st Constitutional All proceedings in relation to any question
Amendment Act, 2003, changed this. So on disqualification of a member of a
now at least two-thirds of the members of House under this Schedule are deemed
a party have to be in favour of a “merger” to be proceedings in Parliament or in the
for it to have validity in the eyes of the law. Legislature of a state. No court has any
“The merger of the original political party jurisdiction.
or a member of a House shall be deemed to Q.1589 Clear constitutional recognition to the
have taken place if, and only if, not less than existence of political parties in India for the
two-thirds of the members of the legislature first time was given by
party concerned have agreed to such merger,” (a) The establishment of Election
states the Tenth Schedule. Commission of India
• Under which circumstances is a split in (b) The Anti-defection provisions in the
a party not considered a ‘defection’? Tenth schedule of the Constitution
A split in a political party will not be (c) The establishment of the Inter-State
considered a defection if an entire political Council of India

Indian Polity Question Bank P.449

05-Indian Polity_Q1507-2004.indd 449 8/7/2018 7:49:58 PM


(d) The 73rd and 74th amendment to the • All such rules must be placed before
Constitution of India the House for thirty days. The House
Solution: (b) may approve or modify or disapprove
The 52nd Amendment Act of 1985 provided them.
for the disqualification of the members of • Further, he may direct that any wilful
Parliament and the state legislatures on the contravention by any member of such
ground of defection from one political party rules may be dealt with in the same
to another. For this purpose, it made changes manner as a breach of privilege of the
in four Articles of the Constitution and added House.
a new Schedule (the Tenth Schedule) to the • According to the rules made so, the
Constitution. This act is often referred to as presiding officer can take up a defection
the ‘anti-defection law’. These provisions case only when he receives a complaint
facilitate democratic realignment of parties in from a member of the House.
the legislature by way of merger of parties. It • Before taking the final decision, he must
also reduces corruption at the political level give the member (against whom the
as well as non-developmental expenditure complaint has been made) a chance to
incurred on irregular elections. And, for the submit his explanation. He may also refer
first time, it gives, a clear-cut constitutional the matter to the committee of privileges
recognition to the existence of political for inquiry. Hence, defection has no
parties. immediate and automatic effect.
Q.1590 Any question regarding disqualification Q.1591 Consider a case where a MP has voted
arising out of defection is to be decided by the against his political party in the Parliament.
(a)  President of India Which among the following authorities will
(b)  Presiding officer of the House decide on his disqualification pertaining to
(c)  Chief Justice of India anti defection law?
(d)  Chief Election Commissioner (a)  Presiding Officer of that House of the
Solution: (b) Parliament
Any question regarding disqualification (b)  President of India
arising out of defection is to be decided (c)  Election Commission of India
by the presiding officer of the House. (d)  Parliamentary Committee on Anti
Originally, the act provided that the decision defection
of the presiding officer is final and cannot be Solution: (a)
questioned in any court. However, in Kihoto Q.1592 The question of disqualification under the
Hollohan v. Zachillhu, 1992 SCR (1) 686, Anti-defection provisions under Tenth
the Supreme Court declared this provision as Schedule of the constitution is decided by the
unconstitutional on the ground that it seeks (a) Chairman in the case of Rajya Sabha and
to take away the jurisdiction of the Supreme Speaker in the case of Lok Sabha
Court and the high courts. It held that the (b) Supreme Court in case of Parliament and
presiding officer, while deciding a question concerned High courts in case of State
under the Tenth Schedule, function as a legislative assemblies
tribunal. Hence, his decision like that of any (c) President of India in all cases based on the
other tribunal is subject to judicial review recommendations of the Union cabinet
on the grounds of mala fides, perversity, etc. (d)  Election Commission of India
But, the court rejected the contention that Solution: (a)
the vesting of adjudicatory powers in the Explained in the previous questions.
presiding officer is by itself invalid on the
Q.1593 Which of the following are the circumstances
ground of political bias.
under which an elected Member of
• The presiding officer of a House is also
Parliament may be disqualified on the ground
empowered to make rules to give effect to
of defection?
the provisions of the Tenth Schedule.

P.450 For Civil Services Preliminary Examination

05-Indian Polity_Q1507-2004.indd 450 8/7/2018 7:49:59 PM


1. If he voluntarily gives up his membership • If a member, after being elected as the
of a political party presiding officer of the House, voluntarily
2. If he votes or abstains from voting gives up the membership of his party or
contrary to any direction issued by his re-joins it after he ceases to hold that office.
political party without prior permission of This exemption has been provided in view
the political party of the dignity and impartiality of this office.
3. If he speaks against the political party Q.1595 Consider the following statements
4. If he joins a political party other than the 1. At present a ‘defection’ by one-third of
party on whose ticket he contested and got the elected members of a political party is
elected considered a ‘merger’
Select the correct answer from the codes 2. A split in a political party will not be
given below: considered a defection if an entire political
(a)  1, 2 and 3 (b)  1, 2 and 4 party merges with another
(c)  1, 3 and 4 (d)  2, 3 and 4 Which of the above statements is/are correct?
Solution: (b) (a)  1 Only (b)  2 Only
A MP can speak against his political party. (c) Both (d) None
This forms part of his/her Parliamentary Solution: (b)
privileges as well as Freedom of Speech and
Q.1596 Which of the following amendments to
Expression (Article 19(a)). If this were not
the Constitution curbed Prime Minister’s
the case, then Parliamentary democracy will
discretion to appoint any number of ministers
not be a reality in India post enactment of
in his Cabinet?
anti-defection law.
(a)  90th Amendment
Q.1594 Disqualification on ground of defection does (b)  91st Amendment
not apply in cases of (c)  92nd Amendment
1. Political Party mergers (d)  93ed Amendment
2. Political party splits Solution: (b)
3. A candidate quitting political party Article 72 of the Constitution prescribes that
Choose the correct answer from the codes the total number of Ministers, including the
given below. Prime Minister, in the Council of Ministers
(a)  1 and 2 only (b)  2 and 3 only shall not exceed 15% of the number of
(c)  1 and 3 only (d)  2 only members of the House of the People.
Solution: (a) Prior to 1 January 2004 (effective date
A member of a House belonging to any of 91st Amendment of the Constitution) the
political party becomes disqualified for Prime Minister had discretion to appoint any
being a member of the House, (a) if he number in his council of ministers. But the
voluntarily gives up his membership of such Constitution (Ninety-first Amendment) Act in
political party; or (b)  if he votes or abstains 2003 made a drastic change in curbing such
from voting in such House contrary to any power of the Prime Minister.
direction issued by his political party without This Amendment added Clause (1A) in
obtaining prior permission of such party and this Article which made a specific provision
such act has not been condoned by the party that, the total number of Ministers, including
within 15 days. The above disqualification on Prime Minister, in no case can exceed 15%
the ground of defection does not apply in the of the total number of Lok Sabha members.
following two cases:
Q.1597 The 91st Amendment Act of 2003 is
• If a member goes out of his party as
significant because
a result of a merger of the party with
1. It gave constitutional recognition to the
another party. A merger takes place when
system of political parties.
two-thirds of the members of the party
2. It regulated the size of council of ministers
have agreed to such merger.
based on the strength of Lok Sabha.

Indian Polity Question Bank P.451

05-Indian Polity_Q1507-2004.indd 451 8/7/2018 7:49:59 PM


Which of the above is/are correct? Usually interest groups seek to promote
(a)  1 only (b)  2 only the interests of a particular section or group
(c)  Both 1 and 2 (d)  None of society. Trade unions, business associations
Solution: (c) and professional (lawyers, doctors, teachers,
Justification: Statement 1: Till this etc.) bodies are some examples of this type.
amendment was introduced, the Constitution They are sectional because they represent
did not refer to the existence of political a section of society: workers, employees,
parties, even though the entire system of business persons, industrialists, followers of
governance is based on the party system. a religion, caste group, etc. Their principal
Through this amendment, there is a clear concern is the betterment and well-being
constitutional recognition of political parties of their members, not society in general.
for the first time. The amendment brought On balance, however, pressure groups and
in the Tenth Schedule provides that an MP/ movements have deepened democracy. Putting
MLA/MLC forfeits his/her membership pressure on the rulers is not an unhealthy
of the legislature if he/she defects from activity in a democracy as long as everyone
the party on whose symbol he/she was gets this opportunity.
elected. While interest groups and movements
Statement 2: As per the amendment, the do not directly engage in party politics,
total number of ministers, including the they seek to exert influence on political
Prime Minister, in the Council of Ministers parties. Most of the movement groups take
shall not exceed 15% of the total strength of a political stance without being a party. They
the Lok Sabha. have political ideology and political position
on major issues. The relationship between
political parties and pressure groups can
(77) PRESSURE GROUPS take different forms, some direct and others
Q.1598 Consider the following statements with very indirect.
regard to Pressure Groups. Q.1599 Consider the following statements about
1. They have political ideology and take pressure groups.
positions on political issues 1. They do not raise political issues.
2. Their decision making is more formal and 2. They do not seek to come into political
rigid. power.
3. Their principal concern is the betterment 3. They are not involved in political activities
and well-being of their members, not like mobilizing people etc.
society in general. Choose the correct answer using the codes
4. They have deepened democracy. below:
5. These groups are formed having (a)  1 and 2 (b)  2 and 3
uncommon objectives. (c)  1 and 3 (d)  2 only
Which of the statements is/are correct? Solution: (d)
(a)  1, 4 and 5 (b)  3, 4 and 5 Pressure groups are meant to raise political
(c)  1, 3 and 4 (d)  1, 2, 4 and 5 issues. A political issue is any issue that
Solution: (c) affects the power distribution in a society.
Pressure groups are organisations that attempt The means can be giving speeches,
to influence government policies. But unlike mobilizing people, meeting the political
political parties, pressure groups do not aim executive putting up their demands etc. For
to directly control or share political power. example, the Student Unions which fight
These organisations are formed when people for rights of the students raise essentially
with common occupation, interest, aspirations political issues.
or opinions come together in order to achieve But, these organizations do not seek for
a common objective. Their decision-making themselves a share in institutional political
is more informal and flexible . power (legislature, executive etc.).

P.452 For Civil Services Preliminary Examination

05-Indian Polity_Q1507-2004.indd 452 8/7/2018 7:49:59 PM


Q.1600 The pressure groups influence governance by (78) NATIONAL INTEGRATION
1. Fighting political elections with
community support
2. Lobbying for a change in the public policy OTHER ASPECTS
by exerting pressure on the government
Which of the above is/are correct? (79) LIST
(a)  1 only (b)  2 only
(c)  Both 1 and 2 (d)  None Q.1602 Seventh Schedule of the Constitution of India
Solution: (b) deals with
Justification: Statement 1: The pressure (a) Protection of the rights of tribals in hilly
groups are also called interest groups or and forest areas
vested groups. They are different from (b) Powers of the Executive branch of the
the political parties in that they neither government
contest elections nor try to capture political (c) Allowances of the President, Vice-
power. President and Supreme Court Judges
Statement 2: They are concerned with (d) Division of powers between the Union
specific programmes and issues and their and the States
activities are confined to the protection and Solution: (d)
promotion of the interests of their members Learning: The legislative section is divided
by influencing the government. into three lists: Union List, State List
The pressure groups influence the policy- and Concurrent List. Unlike the federal
making and policy-implementation in the governments of the United States, Switzerland
government through legal and legitimate or Australia, residual powers remain with the
methods like lobbying, correspondence, Union Government, as with the Canadian
publicity, propagandising, petitioning, public federal government The Union List or List-I is
debating, maintaining contacts with their a list of 100 items given in Seventh Schedule in
legislators and so forth. the Constitution of India on which Parliament
However, sometimes they resort to has exclusive power to legislate.
illegitimate and illegal methods like violence, Q.1603 Which of the following fall in the Concurrent
for example, LWE groups. List under the Seventh Schedule of the
Q.1601 A pressure group does NOT perform which Constitution of India?
of the following activities? 1. Forests
(a)  Contest for political power by fighting 2. Education
general elections 3. Trade and Commerce
(b)  Influencing public opinion by use of 4. Ports
media, propaganda and publishing Choose the correct answer using the codes
(c) Acting as an intermediary between the below:
Government and the public at large (a)  1 and 2 only (b)  2 and 3 only
(d) Lobbying, protests and organizing masses (c)  1, 2 and 3 only (d)  All of the above
Solution: (a) Solution: (a)
Learning: Pressure groups provide a Q.1604 In India, which of these matters comes under
means of popular participation in national the State List of the Seventh Schedule to the
politics between elections. They are Indian Constitution?
sometimes able to gather sufficient support (a) Education (b) Forests
to force government to amend or even (c) Irrigation (d) Banking
scrap legislation. Solution: (c)
For example, pressure groups like ABVP Learning: Union List includes subjects
frequently organize protests, blockades and of national importance such as defence
lobby with the Universities and governments of the country, foreign affairs, banking,
for making their point. communications and currency.

Indian Polity Question Bank P.453

05-Indian Polity_Q1507-2004.indd 453 8/7/2018 7:49:59 PM


State List contains subjects of State and laws or policies as far as the state list subjects
local importance such as trade, commerce are concerned.
and irrigation. Q.1607 Consider the following statements.
Concurrent List includes subjects Assertion (A): Despite Agriculture being a
of common interest to both the Union State Subject, the Centre can legislate on it
Government as well as the State Governments, even in ordinary circumstances.
such as education, forest, trade unions, Reason (R): Agriculture is a means of
marriage, adoption and succession. Both livelihood to a majority of population in
the Union as well as the State Governments India. The Parliament legislates in national
can make laws on the subjects mentioned in interest.
this list. In the context of the statements above,
Q.1605 Which of these subjects does NOT come which of these is true?
under the State List under Seventh Schedule (a) A and R both are true, and R is the correct
of the Constitution? explanation for A.
(a)  Public order (b) A and R both are true, and R is the NOT
(b)  Public health and sanitation the correct explanation for A.
(c) Relief of the disabled and unemployable (c)  A is incorrect, R is correct.
(d)  Banking, Insurance and Financial (d)  A and R both are incorrect.
corporations Solution: (c)
Solution: (d) Agriculture being a state subject, the primary
Learning: Police, Prisons, Local responsibility for increasing agricultural
government, Socio-economic planning, production and productivity, exploiting
Agriculture, including agricultural education untapped potential, and enhancing incomes
and research, Water, that is, water supplies, of the farming community, rests with state
irrigation and canals and Land are important governments. Their efforts are supplemented
subjects under the State list. by many centrally sponsored and central
Central List includes Posts and telegraphs; sector schemes (not legislation).
telephones, wireless, broadcasting and other Therefore, the Union Ministry of
like forms of communication, Currency, Agriculture supports the efforts to the state
coinage and legal tender; foreign exchange, to bring about greater coordination and
Reserve Bank of India, Inter-state trade and reduce regional imbalances
commerce, Trading corporations etc. Q.1608 Legislation with respect to the individual
Q.1606 Which of the following subjects fall in the rights of “Adoption and Succession” comes
State list under the seventh schedule of the under the
constitution? (a)  State List in the Seventh Schedule
1. Banking 2.  Education (b)  Union List in the Seventh Schedule
3. Forests 4.  Police (c) Concurrent List in the Seventh Schedule
5. Agriculture (d)  Residuary List
Choose the correct answer using the codes Solution: (c)
below: Q.1609 The responsibility of maintaining local law
(a)  All of the above (b)  4 and 5 only and order in the disturbed North-Eastern
(c)  2, 3, 4 and 5 (d)  1, 2 and 3 only regions of the country rests primarily with the
Solution: (b) (a)  Central Government
Education and forests come in the concurrent (b)  State Governments
list. (c) Tribal Councils established under Sixth
Banking is in the Union list. Schedule of Constitution
Rest fall in the state list as these matters (d)  Autonomous District Councils
need to be sensitive to local needs. No Solution: (b)
national uniformity is required whether in

P.454 For Civil Services Preliminary Examination

05-Indian Polity_Q1507-2004.indd 454 8/7/2018 7:49:59 PM


Learning: Law and order is a state subject. comes under law and order, which is in state
However the efforts of State Government are list. So, no directions can be forced on the
often supplemented by the Central Government. state governments.
Deployment of CAPF, Border security Match the following: [CDS 1993]
forces like Assam Rifles and ITBP are such A. Taxes on income 1. State list other than
examples. agricultural income
The state government can also request B. Estate duty in 2. Residuary power respect
Central assistance in case of major public of agricultural land
order problems such as insurgencies. C. Inter planetary 3. Concurrent List outer
Q.1610 Economic and Social Planning for tribals is a space travel
subject on which D.  Acquisition and 4. Union List
(a) The Parliament alone can make law but requisitioning of property
with the consent of State Legislatures ABCD
(b)  Only the Parliament can make law (a)  4 3 2 1 (b)  4 1 2 3
(c) Only State Legislatures can make law (c)  2 3 1 4 (d)  3 2 4 1
(d) Both the Parliament and State Solution: (b)
Legislatures can make law
Solution: (d) (80) TABLE OF PRECEDENCE
Justification: Some other important subjects
Q.1612 In the Table of Precedence
that come under the concurrent list are
1. Members of Parliament (MPs) are placed
1. population control and family planning
higher than Chief Ministers of States.
2. Social security and social insurance
2. Governors (inside their respective states)
3. employment and unemployment
are placed higher than former Presidents.
4. Welfare of labour
3. Chief Justice of India is placed higher
5. Education, including technical education,
than Union Cabinet Ministers.
medical education and universities
Choose the correct answer using the codes
6. Relief and rehabilitation of persons
below.
displaced from their original place of
(a)  1 and 2 only (b)  2 and 3 only
residence etc.
(c)  1 and 3 only (d)  All of the above
Both the Parliament and state legislature can
Solution: (b)
make laws on it. But, if a law made by State
The Table of Precedence is related to the rank
legislature contravenes a Central law, the
and order of the officials of the Union and
provision will be null and void.
State Governments. The present notification
Q.1611 Which of the following directions by the on this subject was issued on 26 July, 1979.
Central government (CG), if given, to the This notification superseded all the previous
state governments (SGs) would go against notifications and was also amended many
the Indian federal structure? times. The updated version of the Table,
1. The CG forces SGs to adopt a specific containing all the amendments made therein
policy in dealing with Naxalism. so far (as in 2013), is given below:
2. It decides to launch a defence missile from 1. President
a particular state, which the state is denying. 2. Vice-President
3. It forces the state to adopt the newly 3. Prime Minister
introduced Rs. 20 currency note. 4. Governors of states within their respective
Choose the correct answer using the codes states
below. 5. Former presidents
(a)  1 and 2 (b)  2 and 3 5A. Deputy Prime Minister
(c)  1 and 3 (d)  1 only 6. Chief Justice of India Speaker of
Solution: (d) Lok Sabha
Defence and currency are in the realm of the 7. Cabinet Ministers of the Union Chief
CG, hence directions can be given. Naxalism Ministers of States within their respective

Indian Polity Question Bank P.455

05-Indian Polity_Q1507-2004.indd 455 8/7/2018 7:49:59 PM


States Deputy Chairman, Planning 10.  Deputy Chairman, Rajya Sabha
Commission Former Prime Ministers Deputy Chief Ministers of States
Leaders of Opposition in Rajya Sabha and Deputy Speaker, Lok Sabha
Lok Sabha MPs are ranked at number 21 Ministers of State of the Union (and any
after all the ministers in the Union and other Minister in the Ministry of Defence for
states. defence matters)
Q.1613 Arrange the ranks of the following 11.  Attorney General of India, Cabinet Secretary,
functionaries from higher to lower in the Lieutenant Governors within their
order of precedence. respective Union Territories.
1. Governor of a state within the state Q.1615 The correct order with respect to the rank and
2. Speaker of Lok Sabha precedence of the persons named therein as
3. Former President approved by the President is
4. Chief Minister of a State outside a state (a) Prime Minister > Governors of States
Choose the correct order using the codes below. within their respective States > Former
(a) 3124 (b) 1324 Presidents > Chief Justice of India
(c) 2134 (d) 3241 (b) Prime Minister > Former Presidents >
Solution: (b) Chief Justice of India > Governors of
Q.1614 In the Table of Precedence, which of the States within their respective States >
following figures/authorities precede the Speaker of Lok Sabha
Chief Election Commissioner? (c) President > Chief Justice of India>
1. Holders of Bharat Ratna decoration Governors of States within their
2. Judges of Supreme Court respective States > Speaker of Lok Sabha
3. Attorney General of India > Former Presidents
4. Leader of Opposition, Lok Sabha. (d)  None of the above is correct.
Choose the correct answer using the codes Solution: (a)
below. Learning: The table of precedence has
(a)  1 and 2 only (b)  1, 2 and 4 only nothing to do with the actual rank and
(c)  2 and 3 only (d)  1 and 4 only authority in the government setup. It is more
Solution: (b) of symbolic value and used in ceremonies
From the 7th number, here is the table of and functions as a matter of protocol.
precedence. The Table of Precedence is Q.1616 Consider the following statements
related to the rank and order of the officials 1. Table of Precedence is only meant for
of the Union and State Governments. State and Ceremonial occasions and not
7. Cabinet Ministers of the Union for day to day businesses of government
Chief Ministers of States within their 2. Holders of Bharat Ratna take precedence
respective States, Former Prime Ministers, over Judges of the Supreme Court
Leaders of Opposition in Rajya Sabha and Which of the above is/are correct?
Lok Sabha (a)  1 Only (b)  2 Only
7A.  Holders of Bharat Ratna decoration (c) Both (d) None
8. Ambassadors Extraordinary and Solution: (c)
Plenipotentiary and High Commissioners Q.1617 Consider the following functionaries:
of Commonwealth countries accredited 1. Cabinet Secretary
to India Chief Ministers of States outside 2. Chief Election Commissioner
their respective States Governors of States 3. Union Cabinet Ministers
outside their respective States 4. Chief Justice of India
9. Judges of Supreme Court Their correct sequence, in the Order of
9A.  Chairperson, Union Public Precedence is
Service Commission Chief Election (a)  3, 4, 2, 1 (b)  4, 3, 1, 2
Commissioner (c)  4, 3, 2, 1 (d)  3, 4, 1, 2
Comptroller & Auditor General of India Solution: (c)

P.456 For Civil Services Preliminary Examination

05-Indian Polity_Q1507-2004.indd 456 8/7/2018 7:49:59 PM


Q.1618 Which one of the following is the correct National Flag by members of public, private
sequence in the descending order of organizations, educational institutions,
precedence in the warrant of precedence? etc. Part III of the Code relates to display
[IAS, 2004] of the National Flag by Central and State
(a) Attorney General of India: Judges of the governments and their organisations and
Supreme Court - Members of Parliament – agencies
Deputy Chairman of Rajya Sabha Q.1620 Consider the following with reference to the
(b) Judges of the Supreme Court: Deputy Flag code of India.
Chairman of Rajya Sabha – Attorney 1. When carried in a procession with other
General of India - Members of Parliament flags, the Indian flag must be on the left
(c) Attorney General of’ India: Deputy side of all.
Chairman of Rajya Sabha – Judges of the 2. When the flag is in a moving column,
Supreme Court - Members of Parliament persons present will stand at attention or
(d) Judges of the Supreme Court: Attorney salute as the flag passes them.
General of India – Deputy Chairman of Which of the above is/are correct?
Rajya Sabha – Members of Parliament (a)  1 only (b)  2 only
Solution: (b) (c)  Both 1 and 2 (d)  None
Solution: (b)
(81) NATIONAL SYMBOLS Justification: Statement 1: The flag, when
carried in a procession or parade or with
National Flag another flag or flags, should be on the
marching right or alone in the centre at the
Q.1619 Consider the following statements front. The flag may form a distinctive feature
1. The design of the national flag was of the unveiling of a statue, monument,
adopted by the Constituent Assembly of or plaque, but should never be used as the
India after India became republic covering for the object.
2. The display of the National Flag is solely When the Indian flag is flown on Indian
governed by the non-statutory instructions territory along with other national flags, the
issued by the government time to time general rule is that the Indian flag should be
Which of the above statements is/are correct? the starting point of all flags.
(a)  1 Only (b)  2 Only Statement 2: Moreover, a dignitary may
(c) Both (d) None take the salute without a head dress. The flag
Solution: (d) salutation should be followed by the playing
Statement 1: It was adopted on july 22 1947 of the national anthem.
Statement 2: Apart from non-statutory Learning: Khadi or hand-spun cloth is the
instructions issued by the Government from only material allowed to be used for the flag,
time to time, display of the National Flag is and flying a flag made of any other material
governed by the provisions of the Emblems is punishable by law with imprisonment up to
and Names (Prevention of Improper Use) three years, besides a fine.
Act, 1950 (No.12 of 1950) and the Prevention The Flag should never touch the ground
of Insults to National Honour Act, 1971 or water. Also, the Flag should not hold
(No. 69 of 1971). Flag Code of India, 2002 any objects other than flower petals before
is an attempt to bring together all such laws, unfurling.
conventions, practices and instructions for No lettering should be inscribed on the flag.
the guidance and benefit of all concerned. The flag cannot be used in uniforms.
For the sake of convenience, Flag Code Q.1621 Generally, the flag of India is flown at half-
of India, 2002, has been divided into three mast ‘All Over India’ in the event of the death
parts. The Part I of the Code contains general of a
description of the National Flag. Part II of 1. President and Vice-President
the Code is devoted to the display of the 2. Prime Minister

Indian Polity Question Bank P.457

05-Indian Polity_Q1507-2004.indd 457 8/7/2018 7:49:59 PM


3. Chief Justice of India National Song – Vande Mataram
4. Cabinet Ministers
Select the correct answer using the codes Q.1623 “Vande Mataram”, which eventually shaped
below. our National Song, is a poem composed by
(a)  1 and 2 only (b)  2, 3 and 4 only Bankim Chandra Chattopadhyay in 1870s,
(c)  1, 2 and 3 only (d)  1 and 4 only which he included in his novel
Solution: (a) (a)  Shaarda (b)  Young Bengal
Justification: It is only for the President, (c) Anandamath (d) Geetanjali
Vice-President and Prime Minister that it is Solution: (c)
flow at half-mast allover India. Learning: The first two verses of the song
For the Speaker of the Lok Sabha and the were adopted as the national song of India
Chief Justice of India, it is flown in Delhi and in 1937.
for a Union Cabinet Minister it is flown in • It played a vital role in the Indian
Delhi and the state capitals, from where he or independence movement, first sung in a
she came. political context by Rabindranath Tagore
For a Minister of State, it is flown only at the 1896 session of the Indian National
in Delhi. Congress.
For a Governor, Lt. Governor, or • It became a popular marching song for
Chief Minister of a state or union territory, it political activism and Indian freedom
is flown in the concerned state. movement in 1905.
• Spiritual Indian nationalist and
Q.1622 Consider the following about the protocol
philosopher Sri Aurobindo referred it as
followed while hoisting the National Flag.
“National Anthem of Bengal”.
1. The Flag should never touch the ground or
water. Q.1624 Consider the following about the National
2. Indian citizens can fly the flag even at Anthem and National Song.
night. 1. The National Anthem of India was first
3. The Flag should not hold any objects sung at the Calcutta Session of the Indian
other than flower petals before unfurling. National Congress.
4. No lettering should be inscribed on the flag. 2. The National Anthem was adopted at
5. The flag cannot be used in uniforms. the first Republic Day by the constituent
Select the correct answer using the codes assembly.
below. 3. The National Song was first sung at
(a)  2 and 4 only (b)  3, 4 and 5 only the 1930 Lahore Session of the Indian
(c)  1, 2 and 3 only (d)  1, 2, 3 and 4 only National Congress.
Solution: (d) Select the correct answer using the codes
Learning: The original flag code also below.
forbade use of the flag on uniforms, costumes (a)  1 only (b)  1 and 2 only
and other clothing. (c)  2 and 3 only (d)  1 and 3 only
In 2005, the Government of India amended Solution: (a)
the code to allow some forms of usage. The Justification: Statements 1 and 2: The song
amended code forbids usage in clothing Jana-gana-mana, composed originally
below the waist and on undergarments, and in Bengali by Rabindra Nath Tagore, was
forbids embroidering onto pillowcases, adopted in its Hindi version by the Constituent
handkerchiefs or other dress material. Assembly as the National Anthem of India on
Display and usage of the flag is governed 24 January 1950.
by the Flag Code of India, 2002 (successor to It was first sung on 27 December 1911 at
the Flag Code of India, i.e., the original flag the Calcutta Session of the Indian National
code); the Emblems and Names (Prevention of Congress. The complete song consists of
Improper Use)  Act, 1950; and the Prevention five stanzas. The first stanza contains the full
of Insults to National Honour Act, 1971. version of the National Anthem.

P.458 For Civil Services Preliminary Examination

05-Indian Polity_Q1507-2004.indd 458 8/7/2018 7:49:59 PM


Statement 3: The song Vande Mataram, (82) MINISTRIES AND DEPARTMENTS
composed in Sanskrit by Bankim Chandra
Chatterji, was a source of inspiration to the Q.1627 Consider the following statements about the
people in their struggle for freedom. It has an Prime Minister’s Office (PMO)?
equal status with Jana-Gana-Mana. The first 1. The PMO includes the anticorruption
political occasion when it was sung was the unit and the public wing dealing with
1896 session of the Indian National Congress. grievances
2. Monitors the actions taken by the Prime
Q.1625 The song Vande Mataram, composed in
Minister in earlier meetings.
Sanskrit by Bankim Chandra Chatterji, was
3. It generally handles all activities not
a source of inspiration to the people in their
assigned to any particular department.
struggle for freedom. It was first sung at
Select the correct answer using the codes
(a) 1929 Purna Swaraj declaration session at
below:
Lahore
(a)  1 and 2 only (b)  2 and 3 only
(b)  Calcutta during the mutiny of 1857
(c)  1 and 3 only (d)  All of the above
(c) 1896 session of the Indian National
Solution: (d)
Congress
The Prime Minister’s Office (PMO) consists
(d) Swadeshi movement 1905 after partition
of the immediate staff of the Prime Minister
of Bengal
of India, as well as multiple levels of support
Solution: (c)
staff reporting to the Prime Minister.
Learning: It has an equal status with
• The PMO provides secretarial assistance
Jana-Gana-Mana, our national anthem.
to the Prime Minister – virtual ‘think tank’
Jana-gana-mana was first sung in 1911 at
of the Prime Minister.
the Kolkata Session of the Indian National
• It is headed by the Principal Secretary to
Congress.
the Prime Minister.
The English translation of Vande Matram
• The PMO includes the anti-corruption
stanzas was rendered by Sri Aurobindo.
unit and the public wing dealing with
Q.1626 The song Vande Mataram, composed in grievances.
Sanskrit by Bankim Chandra Chatterji, was • The office houses the Prime Minister
a source of inspiration to the people in their and few selected officers of Indian Civil
struggle for freedom. Consider the following Service who work with him to manage
about it. and coordinate government and his office.
1. It has an equal status with Jana-Gana- • The Prime Minister through his office
Mana, the national anthem. coordinates with all ministers in the central
2. The first political occasion when it was union cabinet, minister of independent
sung was the 1929 session of the Indian charges and governors and ministers of
National Congress (INC). state government.
Which of the above is/are correct? • Monitors the actions taken by the Prime
(a)  1 only (b)  2 only Minister in earlier meetings.
(c)  Both 1 and 2 (d)  None • It generally handles all activities not
Solution: (a) assigned to any particular department.
Justification: It played a vital role in the • Its importance increases writ to cabinet
Indian independence movement, first sung too as Prime Minister is the coordinator of
in a political context by Rabindranath Tagore the Cabinet.
at the 1896 session of the Indian National
Q.1628 Consider the following about Prime
Congress.
Minister’s Office (PMO), India.
In 1907, Bhikaiji Cama (1861–1936)
1. It is headed by the Principal Secretary to
created the first version of India’s national
the Prime Minister.
flag (the Tiranga) in Stuttgart, Germany, in
2. The PMO provides secretarial assistance
1907. It had “Vande Mataram” written on it
to the Prime Minister.
in the middle band.

Indian Polity Question Bank P.459

05-Indian Polity_Q1507-2004.indd 459 8/7/2018 7:49:59 PM


3. The Prime Minister’s National Relief Q.1631 Which of the following is NOT one of the
Fund (Prime Minister National Relief departments of the Union Finance Ministry?
Fund) is operated directly from the PMO. (a)  Department of Expenditure
Select the correct answer using the codes (b)  Department of Economic Affairs
below. (c)  Department of Banking
(a)  2 only (b)  1 and 3 only (d) Department of Investment and Public
(c)  3 only (d)  1, 2 and 3 Asset Management
Solution: (d) Solution: (c)
Q.1629 If the Indian government wants to amend the Justification: The Union Finance Ministry
Companies Act 2013, which of the following of India comprises five departments.
ministry will be primarily concerned with (a)  Department of Economic Affairs
this amendment and its implementation? (b)  Department of Expenditure
(a)  Ministry of Law and Justice (c)  Department of Revenue
(b) Ministry of Heavy Industries and Public (d)  Department of Financial Services
Enterprises (e) Department of Investment and Public
(c)  Ministry of Corporate Affairs Asset Management
(d)  Ministry of Parliamentary Affairs Learning: The Ministry of Finance
Solution: (c) concerns itself with taxation, financial
The Ministry of Corporate Affairs is legislation, financial institutions, capital
primarily concerned with administration of markets, centre and state finances, and the
the Companies Act 2013, the Companies Union Budget.
Act 1956, the Limited Liability Partnership The following cadre controlling authority of
Act, 2008 & other allied Acts and rules & the Civil Services (including Indian Revenue
regulations framed there-under mainly for Service, Indian Economic Service, Indian Cost
regulating the functioning of the corporate Accounts Service and Indian Civil Accounts
sector in accordance with law. Service) are under the administration and
The Ministry is also responsible for supervision of the Finance Ministry.
administering the Competition Act, 2002 to Q.1632 Which of the following departments do
prevent practices having adverse effect on NOT belong to the Union Ministry of Home
competition, to promote and sustain competition Affairs?
in markets, to protect the interests of consumers 1. Department of Administrative Reforms
through the commission set up under the Act. 2. Department of Defence Production
Q.1630 Which of the following departments come 3. Department of States
under the Ministry of Information and 4. Department of Home
Technology? 5. Department of Ex-servicemen Welfare
1. Department of Posts. 6. Department of Official Language
2. Department of Telecommunications. Select the correct answer using the codes
3. Department of Information Technology. below.
Choose the correct answer using the codes (a)  1, 2, 4 and 5 only
below: (b)  1, 2 and 5 only
(a)  Only 1. (b)  Only 2 & 3. (c)  2, 3, 4 and 5 only
(c)  Only 1 & 3. (d)  All. (d)  1 and 6 only
Solution: (d) Solution: (b)
Explanation: The Ministry of Communication Justification: 2 and 5 are in Ministry of
and Information Technology contains three Defence. 1 is in the Ministry of Personnel,
departments: Public Grievances and Pensions. Departments
1. Department of Telecommunications under Ministry of Home Affairs are:
2. Department of Electronics and Information 1. Department of Internal Security (Aantarik
Technology Suraksha Vibhag)
3. Department of Posts 2. Department of States (Rajya Vibhag)

P.460 For Civil Services Preliminary Examination

05-Indian Polity_Q1507-2004.indd 460 8/7/2018 7:49:59 PM


3. Department of Official Language (Raj The Chairman of the State Planning board is
Bhasha Vibhag) (a) Minister of Planning, State Government
4. Department of Home (Grih Vibhag) (b)  Planning Secretary of the State
5. Department of Jammu and Kashmir (c)  Chief Secretary
Affairs (Jammu tatha Kashmir Vibhag) (d)  Chief Minister
6. Department of Border Management Solution: (d)
(Seema Prabandhan Vibhag) He is a member of the Inter-State Council
Q.1633 Consider the following about the Department and the National Development Council, both
of Personnel and Training (DoPT), headed by the prime minister.
Government of India. He acts as a vice-chairman of the
1. The Department deals with cases of concerned zonal council by rotation, holding
appointment to Board of Management of office for a period of one year at a time.
various Public Sector Undertakings. The CM is the chairman of the State
2. There is a mandatory provision for DoPT Planning Board.
to consult the Public Service Commissions He is the chief spokesman of the state
on all matters relating to methods of government.
recruitment and promotions. He is the crisis manager-in-chief at the
Which of the above is/are correct? political level during emergencies.
(a)  1 only (b)  2 only As a leader of the state, he meets
(c)  Both 1 and 2 (d)  None various sections of the people and receives
Solution: (c) memoranda from them regarding their
Justification: Statement 1: The Department problems, and so on.
has the direct responsibility of being the He is the political head of the services.
cadre controlling authority for the IAS and Q.1635 Which of the following functions have been
the three Secretariat Services in the Central assigned to the Ministry of Parliamentary
Secretariat. Affairs under the Government of India
• The Department also deal with cases (Allocation of Business) Rules, 1961?
of appointment to posts of Chairman, 1. Summoning and proroguing the two
Managing Director, full-time functional houses of Parliament
Director/Member of the Board of 2. Preparing answers to all questions raised
Management of various Public Sector in the Zero hour and question hour to
Undertakings/ Enterprises, Corporations, Ministers
Banks and financial institutions. 3. Allocation of Government time in
• It also deals with the assignment of Indian Parliament for discussion of motions
experts to various developing countries. given notice of by Members
Statement 2: The two organizations Select the correct answer using the codes below.
through which the Department ensures (a)  2 only (b)  1 and 3 only
recruitment of personnel for the Government (c)  1 only (d)  3 only
are the Union Public Service Commission Solution: (d)
(UPSC)  and the Staff Selection Commission Justification: Statement 1: It notifies the
(SSC). dates of summoning and prorogation of the
The former is constituted under a provision two Houses of Parliament, Dissolution of Lok
of the Constitution and is responsible for Sabha, President’s Address to Parliament.
conducting examinations for appointment to The actual summon and prorogation is done
the higher civil services and civil posts under by the President.
the Union Government; including recruitment Other functions are:
to the All India Services. 1. Advice to Ministries on procedural and
Q.1634 With India adopting a decentralized and other Parliamentary matters.
participatory model of planning, state 2. Planning and Coordination of legislative
planning boards have become more relevant. and other official business in both Houses.

Indian Polity Question Bank P.461

05-Indian Polity_Q1507-2004.indd 461 8/7/2018 7:49:59 PM


3. Liaison with Leaders and Whips of Ministry of Finance as larger implications of
various Parties and Groups represented in disinvestment for the economy, fiscal deficit,
Parliament. public asset management etc need to be taken
4. Lists of Members of Select and Joint into account
Committees on Bills. Statement 3: Department of States is
5. Appointment of Members of Parliament under Ministry of Home Affairs, which
on Committees and other bodies set up by should also be clear from the fact that Home
Government. Minister heads the zonal councils meant for
6. Implementation of assurances given by centre-state coordination.
Ministers in Parliament. Q.1638 Consider the following National
7. Matters connected with powers, Commissions/Central Bodies and the related
privileges and immunities of Members of Union Ministries.
Parliament. 1. Inter-State Council: Ministry of
8. Organisation of Youth Parliament Parliamentary Affairs
Competitions in Schools/Colleges 2. Central Information Commission:
throughout the country. Ministry of Personnel
Q.1636 Which of the following is NOT a function of 3. National Human Rights Commission:
the Ministry of Parliamentary Affairs? Ministry of Social Justice & Empowerment
(a) Planning and Coordination of legislative Select the correct answer using the codes
and other official business in both Houses below.
(b) Organisation of Youth Parliament (a)  1 only (b)  2 and 3 only
Competitions in Schools/Colleges (c)  2 only (d)  3 only
throughout the country Solution: (c)
(c) Appointment of Members of Parliament Justification: Statement 1 and 3: It falls
on Committees and other bodies set up under Ministry of Home affairs along with
by Government zonal councils. National Human Rights
(d)  None of the above Commission functions as an autonomous
Solution: (d) organization under Home Ministry. The NIA
Q.1637 Consider the following departments under also comes under MHA.
the Government of India. Statement 2: s
1. Department of Ex-Serviceman Welfare: National Commissions/Central Bodies and the Related
Ministry of Personnel and Public Ministries
Grievances
2. Department of Investment and Public SI. Commission / Body Falls Under
Asset Management: Ministry of Heavy No.
Industries and Public Enterprises 1. Central Information Ministry of Personnel
3. Department of States: Ministry of Commission
Parliamentary affairs 2. Finance Commission Ministry of France
4. Department of Bio-technology: Ministry
of Science and Technology 3. Union Public Service Ministry of Personnel
Select the correct matches using the codes Commission
below. 4. Inter-State Council Ministry of Home
(a)  1 and 2 only (b)  3 and 4 only Affairs
(c)  4 only (d)  1, 3 and 4 only 5. Staff Selection Ministry of Personnel
Solution: (c) Commission
Justification: Statement 1: Department of
6. National Commission Ministry of
Ex-Serviceman Welfare is under Ministry of
for SCs Social Justice &
Defence.
Empowerment
Statement 2: Department of Investment
and Public Asset Management is under

P.462 For Civil Services Preliminary Examination

05-Indian Polity_Q1507-2004.indd 462 8/7/2018 7:49:59 PM


7. National Commission Ministry of Tribal Justification: The option (b)  is verified by
for STs Affairs the following:
• In the SEZ board of approval, Secretary,
8. Central Vigilance Ministry of Personnel Department of Commerce is the Chairman.
Commission • There is an office of Development
9. Zonal Councils Ministry of Home Commissioner of Special Economic
Affairs Zones (SEZs).
10. Central Bureau of Ministry of Personnel • Director General of Foreign Trade is under
Investigation Ministry of Commerce and Industry.
Learning: For getting approval for setting
11. National Ministry of Home SEZs, The developer submits the proposal
Investigation Agency Affairs for establishment of SEZ to the concerned
12. Commissioner for Ministry of Minority State Government. The State Government
Linguistic Minorities Affairs has to forward the proposal with its
13. National Commission Ministry of recommendation within 45 days from the
for Protection of Women and Child date of receipt of such proposal to the Board
Child Rights Development of Approval.
The applicant also has the option to
14. National Commission Ministry of submit the proposal directly to the Board of
for Backward Classes Social Justice & Approval. The Board of Approval has been
Empowerment constituted by the Central Government in
15. Central Ministry of exercise of the powers conferred under the
Commissioner for Social Justice & SEZ Act. All the decisions are taken in the
Disabled Persons Empowerment Board of Approval by consensus. The Board
16. Central Social Ministry of of Approval has 19 Members including Secy.
Welfare Beard Women and Child from key ministries.
Development Q.1640 Many development and industrial projects such
17. North-Eastern Ministry of as erection of dams, mining, and construction
Council Development of the of industries or roads require diversion of
North-Eastern Region forest land. Consider the regulatory aspects
of it.
18. Central Administrative Ministry of Personnel 1. Any project proponent, government or
Tribunal private must apply for forest clearance
19. National Commission Ministry of Minority from Ministry of Environment, Forests
for Minorities Affairs and Climate Change (MoEFCC), before
20. National Human Ministry of Home the conversion of land takes place.
Rights Commission Affairs 2. The compensation for the lost forest
land is decided by the MoEFCC and the
21. National Commission Ministry of regulators.
for Women Women and Child Which of the above is/are correct?
Development (a)  1 only (b)  2 only
(c)  Both 1 and 2 (d)  None
Q.1639 Which of the following ministries/
Solution: (c)
departments has the key role in the
Learning: With a cover of 23% of
Governance of SEZs in India?
Geographical area of the country, forest
(a)  Ministry of External Affairs
in India comprise of a number of diverse
(b)  Department of Commerce
forest types and reserved areas designated as
(c)  Department of Economic Affairs
National Parks and Wildlife Sanctuaries.
(d) Ministry of Heavy Industries and
Statement 1: Compensatory Afforestation
Enterprises
Fund Management and Planning Authority
Solution: (b)

Indian Polity Question Bank P.463

05-Indian Polity_Q1507-2004.indd 463 8/7/2018 7:49:59 PM


(CAMPA) are meant to promote afforestation persistently by making use of such
and regeneration activities as a way of computer resource or a communication
compensating for forest land diverted to non- device,
forest uses. Q.1642 The Supreme Court has recently
Statement 2: This proposal is to be declared the Section 66A of the IT Act as
submitted through the concerned forest unconstitutional. Which of the following
department of the state government. If activities that could have been charged
clearance is given, then compensation for the under the law earlier can
lost forest land is also to be decided by the now be allowed based on the judgment?
ministry and the regulators. 1. A comment on Social networking website
that is viewed as offensive by some
(83) SOME IMPORTANT ACTS individuals or community
2. A false declaration in public (not on
Q.1641 You are a District Magistrate. You denied internet) about an incident or fact that has
allotting a huge tender to an influential local not occurred
businessman. To seek revenge, he is posting 3. A private e-mail message that causes
defamatory and offensive speeches against insult to an individual
you on the internet. Which legal recourse can Choose the correct answer using the codes
you take against him? below.
(a) Approach the State high court accusing (a)  1 only (b)  2 and 3 only
the businessman of abusing freedom of (c)  1 and 3 only (d)  1 and 2 only
speech given under Article 19 Solution: (a)
(b) File a case in local police station
Q.1643 With reference to the Supreme Court
under Section 66A of the Information
judgment on the Section 66A of Information
Technology Act
Technology Act, consider the following
(c) File a case with the local police station
statements
under the IPC Section 499/500
1. The concerned police officer of a police
(d)  Both (b)  and (c)
station may not arrest any person until
Solution: (d)
she/he has obtained prior approval of such
IPC Section 499/500 of IPC was in news
arrest from an officer, not below the rank
in 2017. The Centre has told the Supreme
of Inspector General of Police (IGP) in
Court that defamation should remain a penal
metropolitan cities
offence in India as the defamer may be too
2. Any information which he/she knows to
poor to compensate the victim. It has denied
be false, but for the purpose of causing
that criminal defamation had any chilling
annoyance, inconvenience is also an
effect on free speech. The IPC under Sections
offence under this act.
499/500 criminalizes defamatory speech.
Which of the statements given above is/are
This means that a person can be imprisoned
correct?
for a maximum period of two years, if found
(a)  1 only (b)  2 only
guilty. According to the Section 66A of the
(c) both (d) none
IT act, any person who sends, by means of
Solution: (c)
a computer resource or a communication
The Supreme Court has said that the State
device–
governments are advised that as regard to arrest
(a) any information that is grossly offensive
of any person in complaint registered under
or has menacing character or
section 66A of the Information Technology
(b) any information which he knows to be
Act, the concerned police officer of a police
false, but for the purpose of causing
station may not arrest any person until she/he
annoyance, inconvenience, danger,
has obtained prior approval of such arrest from
obstruction, insult, injury, criminal
an officer, not below the rank of Inspector
intimidation, enmity, hatred or ill will,
General of Police (IGP) in metropolitan

P.464 For Civil Services Preliminary Examination

05-Indian Polity_Q1507-2004.indd 464 8/7/2018 7:49:59 PM


cities or of an officer not below the rank of 2. The Act makes provisions for a non-
Deputy Commissioner of Police (DCP) or admitted child to be admitted to an age
Superintendent of Police (SP) at district level, appropriate class.
as the case may be. The Section 66A(b) of 3. Education in the Indian constitution is
IT act makes an offence of sending through a a concurrent issue and both centre and
computer resource or communication device states can legislate on the issue.
“any information which he knows to be false, Which of the above statements are true?
but for the purpose of causing annoyance, (a)  Only 1 (b)  Only 2 & 3
inconvenience, danger, obstruction, insult, (c)  Only 1 & 3 (d)  All
injury, criminal intimidation, enmity, hatred Solution: (d)
or ill will, persistently by making use of such Right to Education (RTE) Act
computer resource or a communication device. The Constitution (Eighty-sixth Amendment)
Q.1644 Consider the following statements about the Act, 2002 inserted Article 21-A in the
Prevention of Corruption Act, 1988 Constitution of India to provide free and
1. It covers only the Central government compulsory education of all children in
agencies/bodies and PSUs. the age group of six to fourteen years as a
2. The offences punishable under this act can Fundamental Right in such a manner as the
be tried by special Judges only. State may, by law, determine. The Right of
3. Police does not have powers to investigate Children to Free and Compulsory Education
offences under this particular Act. (RTE) Act, 2009, which represents the
Choose the correct answer using the codes consequential legislation envisaged under
below. Article 21A, means that every child has a
(a)  1 and 2 only (b)  1 and 3 only right to full time elementary education of
(c)  2 and 3 only (d)  2 only satisfactory and equitable quality in a formal
Solution: (d) school which satisfies certain essential norms
and standards. Article 21-A and the RTE Act
Q.1645 Consider the following provisions of the
came into effect on 1 April 2010.
Civil Nuclear Liability Act, 2010 that has
With this, India has moved forward
been in news for some time.
to a rights based framework that casts a
1. This Act limits the liability of the operator
legal obligation on the Central and State
in case of nuclear accident.
Governments to implement this fundamental
2. It secures the operator the right to recourse
child right as enshrined in the Article 21A
under certain circumstances.
of the Constitution, in accordance with the
3. It provides a mechanism to compensate
provisions of the RTE Act. It is seen as the
victims of nuclear damage.
most historic development in universalization
Which of the above provisions have been
of elementary education in the country. It
cause(s) of contention between India and other
implies that every child in the age group
nuclear supplier countries in recent times?
of 6 to 14 years has Right to elementary
Choose the correct answer using the codes
education. They are entitled for free and
below.
compulsory education.
(a)  1 and 2 only (b)  2 and 3 only
The RTE Act provides for the:
(c)  1 and 3 only (d)  All of the above
1. Right of children to free and compulsory
Solution: (a)
education till completion of elementary
education in a neighbourhood school.
The Right to Education, 2009 2. It clarifies that ‘compulsory education’
Q.1646 Consider the following statements with means obligation of the appropriate
reference to the Right of Children to Free and government to provide free elementary
Compulsory Education Act, 2009: education and ensure compulsory
1. According to the Act, every child in the admission, attendance and completion of
age group of 6 to 14 years has Right to elementary education to every child in the
elementary education.

Indian Polity Question Bank P.465

05-Indian Polity_Q1507-2004.indd 465 8/7/2018 7:49:59 PM


six to fourteen age group. ‘Free’ means Plan, teacher training on Autism and
that no child shall be liable to pay any therapeutic support.
kind of fee or charges or expenses which 2. The programme has been implemented in
may prevent him or her from pursuing and order to narrow down gender and social
completing elementary education. gaps in elementary education. Special
3. It makes provisions for a non-admitted efforts have been made to reach out to
child to be admitted to an age appropriate girls and children belonging to SC/ST and
class. Muslim minority communities.
4. It specifies the duties and responsibilities
of appropriate Governments, local Right to Information Act, 2005
authority and parents in providing free
and compulsory education, and sharing Q.1647 The Right to Information does NOT include
of financial and other responsibilities (a) Inspection of government documents on
between the Central and State request
Governments. (b)  Taking out files from a government office
5. It lays down the norms and standards to any place desired by the applicant
relating inter alia to Pupil Teacher Ratios (c)  Taking a photograph of files and
(PTRs), buildings and infrastructure, obtaining information in tapes
school-working days, teachers’ working (d) It does not include any of the above.
hours. Solution: (b)
6. It provides for rational deployment of Justification and Learning: A citizen has
teachers by ensuring that the specified a right to seek such information from a
pupil teacher ratio is maintained for each public authority which is held by the public
school, rather than just as an average authority or which is held under its control.
for the State or District or Block, thus 1. This right includes inspection of work,
ensuring that there is no urban-rural documents and records; taking notes,
imbalance in teacher postings. It also extracts or certified copied of documents
provides for prohibition of deployment or records; taking certified samples of
of teachers for non-educational work, material held by the public authority
other than decennial census, elections or held under the control of the public
to local authority, state legislatures and authority. So, the option (a) is correct.
parliament, and disaster relief. 2. The public authority under the RTI Act
7. It provides for appointment of is not supposed to create information; or
appropriately trained teachers, i.e. to interpret information; or to solve the
teachers with the requisite entry and problems raised by the applicants; or to
academic qualifications. furnish replies to hypothetical questions.
The Sarva Shiksha Abhiyan (SSA) is the Only such information can be had under
main vehicle for implementation of the RTE the Act which already exists with the
Act. It is one of the largest programmes of public authority.
its kind in the world. It is primarily funded 3. A citizen has a right to obtain
from central budget and it covers the whole information in the form of diskettes,
country. floppies, tapes, video cassettes or in any
1. Under SSA, special attention has been other electronic mode or through print-
given to urban deprived children, children out provided information is already
affected by periodic migration and stored in a computer or in any other
children living in remote and scattered device from which the information
habitations. Attempts have also been made may be transferred to diskettes. So, the
to reach out to children suffering from option (c) is correct.
autism. It involves their identification, 4. The information to the applicant shall
preparation of individualized Education ordinarily be provided in the form in

P.466 For Civil Services Preliminary Examination

05-Indian Polity_Q1507-2004.indd 466 8/7/2018 7:49:59 PM


which it is sought. However, if the supply The Act also allows states to enact their
of information sought in a particular own ESMA and choose the essential services
form would disproportionately divert the on which to enforce Esma.
resources of the public authority or may Statement 2: Although it is Central
cause harm to the safety or preservation of law, its execution can be done by the State
the records, supply of information in that governments and UT governments too. For
form may be denied. The Act gives the example, Delhi government has invoked the
right to information only to the citizens ESMA to declare a nurse’s strike illegal.
of India. It does not make provision for Recently the AP government declared IT
giving information to Corporations, to be an essential service, which allows it to
Associations, Companies etc. which are call off illegitimate strikes in the IT sector.
legal entities/persons, but not citizens. Q.1650 Consider the following about Essential
Q.1648 The Right to Information Act, 2005 empowers Services Maintenance Act (ESMA).
the citizens, promotes transparency and 1. It is a central law.
accountability in the working of the 2. Each state has a separate ESMA.
Government. Consider the following about it. 3. Both Central and State governments can
1. The Act is applicable only to constitutional enforce ESMA.
or statutory authorities. 4. The government cannot order private
2. Under the Act, a citizen can visit public operators to restore essential services.
office and take certified samples of Choose the correct answer using the codes
material held by the public authority. below.
3. The Act requires public authorities to (a)  1, 2 and 3 only (b)  1 and 4 only
publish certain types of information (c)  2, 3 and 4 only (d)  1 and 2 only
publicly on a suo motu basis. Solution: (d)
Select the correct answer using the codes below. It was established to ensure the delivery of
(a)  1 and 2 only (b)  2 and 3 only certain services, which if obstructed would
(c)  3 only (d)  1 only affect the normal life of the people. These
Solution: (b) include services like public transport (bus
As explained in previous question. services), health services (doctors and
hospitals).
Essential Services Maintenance Act Although it is a very powerful law, its
execution rests entirely on the discretion
(ESMA) of the State government. Each state in the
Q.1649 Consider the following about Essential union of India, hence has a separate state
Services Maintenance Act (ESMA). Essential Services Maintenance Act with
1. The Act can be used to call off worker’s slight variations from the central law in its
strikes both in public and private sector if provisions. This freedom is accorded by the
it disrupts essential services. central law itself.
2. Only the Central government can order
restoration of essential services in India. Forest Rights Act, 2006
Which of the above is/are correct? Q.1651 As per the Scheduled Tribes and Other
(a)  1 only (b)  2 only Traditional Forest Dwellers (Recognition of
(c)  Both 1 and 2 (d)  None Forest Rights) Act, 2006, which of these are
Solution: (a) the legal rights of tribals?
Justification: The Act includes a long list of 1. Conservation of forests and bio-diversity
“essential services” in its charter—ranging 2. Granting leases to non-tribals for
from post and telegraph, through railway, accessing core forests
airport and port operations—and it prohibits 3. Repeal environmentally sensitive projects
the key employees in these services from by local bodies
striking.

Indian Polity Question Bank P.467

05-Indian Polity_Q1507-2004.indd 467 8/7/2018 7:50:00 PM


4. Right to historical homestead, cultivable • Relief and development rights, that is
and grazing land to rehabilitation in case of illegal eviction
Select the correct answer using the codes or forced displacement; and to basic
below. amenities, subject to restrictions for forest
(a)  1, 2 and 4 only (b)  2 and 3 only protection
(c)  1 and 4 only (d)  All of the above • Forest management rights, that is, to
Solution: (c) protect forests and wildlife
Explanation and Learning: India’s Q.1652 The Scheduled Tribes and Other Traditional
forests are governed by two main laws, Forest Dwellers (Recognition of Forest
the Indian Forest Act, 1927 and the Wild Rights) Act, 2006, also called as the Forest
Life (Protection) Act, 1972. The former Rights Act (FRA), 2006 confers which of the
empowers the government to declare any following rights on tribals?
area to be a reserved forest, protected forest 1. Rights over forest land patches
or village forest. The latter allows any area to 2. Rights over minor produce from forests
be constituted as a “protected area”, namely 3. Rights over timber produce from forest
a national park, wildlife sanctuary, tiger 4. Rights over the wildlife in the forests
reserve or community conservation area. Choose the correct answer using the codes
Since times immemorial, the tribal below.
communities of India have had an integral (a)  1 and 2 only (b)  2, 3 and 4 only
and close knit relationship with the forests (c)  1 and 4 only (d)  All of the above
and have been dependent on the forests for Solution: (a)
livelihoods and existence.
Q.1653 Consider the following statements about
Under these laws, the rights of people
the Scheduled Tribes and Other Traditional
living in or depending on the area to be
Forest Dwellers (Recognition of Forest
declared as a forest or protected area are to
Rights) Act, 2006.
be “settled” by a “forest settlement officer.
1. This Act is meant to undo the historical
However, rights were rarely recognized
injustices meted out to forest dwelling
by the authorities and in the absence of
populations in not recognising their rights
real ownership of the land, the already
to land and resources.
marginalized local dwellers suffered. To
2. This Act recognises tribal’s right to
address this, FRA 2006 was enacted.
homestead cultivable and grazing land
Eligibility to get rights under the Act
and to non-timber forest produce.
is confined to those who “primarily reside
3. The Act prohibits diversion of forest land
in forests” and who depend on forests and
falling in the areas specified under the Act
forest land for a livelihood. Further, either
for any public utility facilities.
the claimant must be a member of the
4. The forest right related to conversion of
Scheduled Tribes scheduled in that area or
forest villages into revenue villages is to be
must have been residing in the forest for
adjudicated by the Gram Sabha and other
75 years.
sub- divisional and district committees.
Major provisions are:
Select the correct answer using the codes
• Title rights, i.e. ownership - to land that is
below.
being farmed by tribals or forest dwellers,
(a)  1 and 2 only (b)  1, 2 and 4 only
subject to a maximum of 4 hectares;
(c)  2, 3 and 4 only (d)  1, 3 and 4 only
ownership is only for land that is actually
Solution: (b)
being cultivated by the concerned family
as on that date, meaning that no new lands Q.1654 Consider the following statements about the
are granted Forest Rights Act 2006 concerning Scheduled
• Use rights, that is to minor forest produce Tribes and other Traditional forest dwellers.
(also including ownership), to grazing 1. It recognizes their right to homestead and
areas, to pastoralist routes, etc. cultivable land in the forest.

P.468 For Civil Services Preliminary Examination

05-Indian Polity_Q1507-2004.indd 468 8/7/2018 7:50:00 PM


2. It allots these land deprived communities Scheduled Castes and the Scheduled
one-tenth of an acre of land in the forest
land. Tribes (Prevention of Atrocities)
3. It recognizes the right of these
communities to conserve biodiversity and
Act, 1989
forests. Q.1656 Which of the following is NOT correct
4. It recognizes their right to timber as well regarding the Scheduled Castes and the
as non-timber produce from forests. Scheduled Tribes (Prevention of Atrocities)
5. Gram Sabha is the authority identifying Act, 1989?
the forest rights of these communities and (a) Based on its provisions, tribals can
individuals therein. demand re-possession of alienated lands.
Choose the correct answer using the codes (b) Communities committing atrocities against
below: tribals can be evicted from tribal areas.
(a)  1, 2 and 3 (b)  3, 4 and 5 (c) It does not extend to the State of Jammu
(c)  1, 4 and 5 (d)  1, 3, 4 and 5 & Kashmir.
Solution: (d) (d) It provides for Special Courts for the trial
Q.1655 As per the Scheduled Tribes and Other of atrocities against SCs/STs.
Traditional Forest Dwellers (Recognition of Solution: (b)
Forest Rights) Act, 2006, the responsibility Explanation: The Act penalises anyone
for recognition and vesting of forest rights who wrongfully occupies or cultivates any
and distribution of land rights rests with the land owned by, or allotted to a member of a
(a)  District Collector SC or a ST or gets the land allotted to him
(b)  State Government transferred. So, if they are dispossessed, land
(c)  Central Government can be reclaimed back. Option (a) is correct.
(d) Chief Wildlife Warden of the forest region At another level, the Act recognizes that
Solution: (b) crimes against Dalit and tribal women are
Learning: It seeks to recognize the forest of a specific kind and provides for special
rights and occupation in forest land of courts. So, option (d) is correct. The Act does
old forest dwellers. They chose rights on not apply to J&K. So, option (c) is correct.
ancestral lands and their habitat were not Learning: Adivasi activists refer to the
adequately recognized in the consolidation 1989 Act to defend their right to occupy land
of State forests during the colonial period as that was traditionally their since the
well as in independent India. ancient time. This Act merely confirms
1. Section 6(1) of the Act provides that the what has already been promised to tribal
gram Sabha, or village assembly, will people in the Constitution – that land
initially pass a resolution recommending belonging to tribal people cannot be sold
whose rights to which resources should to or bought by non-tribal people. In cases
be recognised (i.e. which lands belong where this has happened, the Constitution
to whom, how much land was under the guarantees the right of tribal people to
cultivation of each person. re-possess their land.
2. This resolution is then screened and
approved at the level of the subdivision Armed Forces Special Powers Act
(or taluka) and subsequently at the district
level. The screening committees consist (AFSPA) 1958
of three government officials (Forest, Q.1657 Armed Forces Special Powers Act (AFSPA)
Revenue and Tribal Welfare departments) is NOT under operation in which of these
and three elected members of the local states of India?
body at that level. (a)  Manipur (b)  Jammu & Kashmir
3. The state government finally recognizes (c) Nagaland (d) Tripura
and vests the forest rights. Solution: (d)

Indian Polity Question Bank P.469

05-Indian Polity_Q1507-2004.indd 469 8/7/2018 7:50:00 PM


Justification: It was withdrawn from Tripura 1. It aims to promote self-rule for the bulk of
in 2015. the tribal population in India.
On the other hand, the Armed Forces 2. It provides that Gram Sabha will be
Special Powers Act (AFSPA) has been responsible for the identification of
extended for another six months in Nagaland beneficiaries under poverty alleviation
(as of till march 2018). programmes.
In Manipur, where the clamour for the 3. It grants the Panchayats exclusive
repeal of AFSPA has been most vocal— authority to acquire land in the Scheduled
with Irom Sharmila on a hunger-strike since Areas for development projects.
2000 — the government, in 2004, withdrew 4. It empowers the Gram Sabha and the
the Act from certain assembly constituencies, Panchayats for recommending grant of
but NOT the entire state. concession for the exploitation of minor
In J&K, it has been in operation since 1990s. minerals by auction.
Q.1658 Consider the following about AFSPA. Select the correct answer using the codes
1. Only the Governor of a state can declare below.
an area to come under AFSPA. (a)  1, 2 and 4 only (b)  1 and 3 only
2. The Act makes local civilian (c)  1, 2 and 3 only (d)  2 and 4 only
administration accountable to the military. Solution: (a)
3. It can be withdrawn from an area only after Justification: Statement 1: It aims for village
the consent of the Central government and governance with participatory democracy
Indian armed forces. and to make the Gram Sabha a nucleus of all
4. AFSPA cannot be applicable in a state activities by their extension to scheduled areas.
during emergency. Statement 2: Apart from the statement
Choose the incorrect answer using the codes (as mentioned in the question above), every
below. Gram Sabha shall approve of the plans,
(a)  1 and 2 only (b)  3 only programmes and projects for social and
(c)  1 only (d)  1 and 4 only economic development before they are taken
Solution: (c) up for implementation by the Panchayat at
The AFSPA initially empowered only the the village level.
Governors of the States and the Administrators Statement 3: The Gram Sabha or the
of the Union Territories to declare areas in Panchayats at the appropriate level shall be
the concerned State or the Union Territory as consulted before making the acquisition of
‘disturbed’. But, since 1972 AFSPA can be land in the Scheduled Areas for development
enforced by the Central government too. projects and before resettling or rehabilitating
The Act gives the military special powers persons affected by such projects in the
over ‘disturbed areas’ where it can use force Scheduled Areas.
(without formal prosecution etc.) if it deems However, the actual planning and
necessary. implementation of the projects in the
The Act has received criticism from Scheduled Areas shall be coordinated at the
several sections for alleged concerns about state level.
human rights violations in the regions of its Statement 4: The recommendations of
enforcement alleged to have happened. the Gram Sabha or the Panchayats at the
appropriate level shall be mandatory for
grant of prospecting licence or mining lease
Panchayats (Extension to the for minor minerals in the Scheduled Areas.
Scheduled Areas) Act (PESA), 1996 The prior recommendation of the Gram
Sabha or the Panchayats at the appropriate
Q.1659 Which of the following is/are the major level shall be mandatory for grant of
features of the Provisions of the Panchayats concession for the exploitation of minor
(Extension to the Scheduled Areas) Act minerals by auction.
(PESA), 1996?

P.470 For Civil Services Preliminary Examination

05-Indian Polity_Q1507-2004.indd 470 8/7/2018 7:50:00 PM


Q.1660 Provisions of the Panchayats (Extension to Select the correct answer using the codes
the Scheduled Areas) Act, 1996, popularly below.
known as the PESA Act or the Extension Act (a)  1 and 2 only (b)  2 only
has which of the following objectives? (c)  1 and 3 only (d)  1, 2 and 3
1. To provide self-rule for the bulk of the Solution: (a)
tribal population Refer previous answer.
2. To evolve a suitable administrative
Q.1662 The provisions of Part IX of the constitution
framework consistent with traditional
relating to the Panchayats are not applicable
practices
to the Fifth Schedule areas. So, the
3. To safeguard and to preserve the traditions
Parliament has enacted the “Provisions of
and customs of tribal communities
the Panchayats (Extension to the Scheduled
Choose the correct answer using the codes
Areas) Act (PESA)”, 1996. Which of the
below.
following states is/are covered by the Act?
(a)  1 and 2 only (b)  2 and 3 only
1. Jharkhand 2. Rajasthan
(c)  1 and 3 only (d)  All of the above
3. Tamil Nadu 4.  West Bengal
Solution: (d)
5. Odisha 6. Gujarat
The objectives of the PESA Act are as
Select the correct answer using the codes
follows:
below.
• To extend the provisions of Part IX of the
(a)  2, 3, 4 and 6 only (b)  1, 2, 5 and 6 only
Constitution relating to the panchayats
(c)  1, 4 and 5 only (d)  1, 2, 3, 4, 5 and 6
to the scheduled areas with certain
Solution: (b)
modifications
Learning: At of 2018, ten states have Fifth
• To provide self-rule for the bulk of the
Schedule Areas. These are: Andhra Pradesh,
tribal population
Telangana, Chhattisgarh, Gujarat, Himachal
• To have village governance with
Pradesh, Jharkhand, Madhya Pradesh,
participatory democracy and to make the
Maharashtra, Odisha and Rajasthan.
Gram Sabha a nucleus of all activities
• To evolve a suitable administrative Q.1663 Under the Panchayats (Extension to
framework consistent with traditional Scheduled areas) PESA, 1996 Act, Gram
practices Sabha are given the authority to stop land
• To safeguard and to preserve the traditions alienation by
and customs of tribal communities (a)  State Legislatures
• To empower panchayats at the appropriate (b)  District Collector
levels with specific powers conducive to (c)  State Forest department
tribal requirements (d)  Central government
• To prevent panchayats at the higher level Solution: (a)
from assuming the powers and authority Panchayats (Extension to Scheduled Areas)
of panchayats at the lower level of the Act, 1996 or PESA is a law enacted by the
Gram Sabha Government of India to cover the “Scheduled
areas”, which are not covered in the 73rd
Q.1661 Which of the following is/are the objectives
amendment or Panchayati Raj Act of the
of the PESA Act, 1996?
Indian Constitution. It was enacted on 24
1. To prevent panchayats at the higher level
December 1996 to enable Gram Sabha to
from assuming the powers and authority
self-govern their natural resources. How
of panchayats at the lower level of the
much self-governance will be allowed
Gram Sabha
depends on the state legislative assemblies.
2. To provide self-rule for the bulk of the
For e.g. in some states like Jharkhand, Gram
tribal population
Sabha under scheduled areas do not have the
3. To allow for the flow of mainstream
power to stop land alienation, while in some
culture in remote tribal populations
other states they do.

Indian Polity Question Bank P.471

05-Indian Polity_Q1507-2004.indd 471 8/7/2018 7:50:00 PM


Q.1664 The Panchayats (Extension to Scheduled (84) SOME OF THE GOVERNMENT
Areas) Act, PESA Act, 1996 concerns
1. Ownership of minor forest produces for BODIES
the community
2. Power to Gram Sabha to stop land Constitutional Body
alienation Q.1665 The Union Cabinet has recently approved
3. Recommendation of Gram Sabha for the process, formation and functioning of
grant of mining lease for minor minerals the Goods and Services Tax (GST) Council.
Select the correct answer using the codes Consider the following about it.
below. 1. It will be a constitutional body.
(a)  1 and 2 only (b)  2 only 2. It will be a joint forum of the Centre and
(c)  1 and 3 only (d)  1, 2 and 3 the States chaired by the Prime Minister.
Solution: (d) 3. The Council will make recommendations
Justification: Statements 1 and 2: While to the Union and the States on GST rates.
endowing Panchayats in the Scheduled Select the correct answer using the codes
Areas with such powers and authority as below.
may be necessary to enable them to function (a)  1 and 2 only (b)  2 and 3 only
as institutions of self-government, a State (c)  1 and 3 only (d)  3 only
Legislature shall ensure that the Panchayats Solution: (c)
at the appropriate level and the Gram Sabha Justification: As per Article 279A of
are endowed specifically with: the amended Constitution, GST Council
1. the ownership of minor forest produce which will be a joint forum of the Centre
2. the power to prevent alienation of land and the States. It shall have the following
in the Scheduled Areas and to take composition:
appropriate Union Finance Minister (Chairperson).
3. action to restore any unlawfully alienated The Union Minister of State (MoS) in-
land of a Scheduled Tribe charge of Revenue of finance (Member)
4. the power to manage village markets The Minister In-charge of taxation or
Statement 3: The prior recommendation of finance or any other Minister nominated by
the Gram Sabha or the Panchayats at the each State Government (Members)
appropriate level shall be mandatory for grant Functions of GST Council
of concession for the exploitation of minor • As per Article 279A (4), the Council will
minerals by auction. make recommendations to the Union and
As per the PESA Act: the States on important issues related to
1. Acquisition of land requires consultation GST, like
with the concerned Gram Sabha • Goods and services that may be subjected
2. Actual planning and implementation of or exempted from GST.
the projects in the Scheduled Areas shall • Model GST Laws
be coordinated at the Panchayat level • Principles that govern Place of Supply,
3. For the exploitation of minor minerals threshold limits
prior recommendation of the Gram Sabha • GST rates including the floor rates with
or the Panchayat is needed. bands, special rates for raising additional
Choose the correct answer using the codes resources during natural disasters/
below. calamities, special provisions for certain
(a)  1 and 2 only (b)  2 and 3 only States, etc.
(c)  1 and 3 only (d)  All of the above
Solution: (d)
Non-Statutory
Q.1666 Consider the following about the Law
Commission of India (LCI).

P.472 For Civil Services Preliminary Examination

05-Indian Polity_Q1507-2004.indd 472 8/7/2018 7:50:00 PM


1. It is a non-statutory and non-constitutional National Water Policy was recommended to
body. all States for implementation.
2. It is headed by the incumbent Union Law Q.1668 Which among the following is/are member(s)
Secretary. of the National Water Resources Council
3. It is constituted by the Parliament from (NWRC)?
time to time. 1. Chief Ministers of all States
Select the correct answer using the codes 2. Union Minister of Water Resources
below. 3. Chairman, NITI Aayog
(a)  1 only (b)  2 and 3 only Select the correct answer using the codes
(c)  1 and 3 only (d)  1, 2 and 3 only below.
Solution: (a) (a)  1 and 2 only (b)  3 only
Justification: Statement 2: It is usually (c)  2 only (d)  1, 2 and 3
headed by a retired Supreme Court judge or Solution: (a)
former Chief Justice of a High Court.
Q.1669 Consider the following about the National
Statement 3: It is constituted by the Union
Authority, Chemical Weapons Convention
Government from time to time. The first
(NACWC).
commission was constituted in 1955 and
1. It was set up by a resolution of Cabinet
since then various commissions were re-
Secretariat.
constituted every three years.
2. It is chaired by the Prime Minister of
India.
Statutory Bodies 3. It is housed under the Ministry of
Q.1667 Consider the following about the National Chemicals and Fertilizers.
Water Resources Council (NWRC). Select the correct answer using the codes
1. The council is chaired by the Prime below.
Minister with Chief Minister of states as (a)  1 and 2 only (b)  1 only
its members. (c)  2 only (d)  2 and 3 only
2. The council has the authority to approve Solution: (b)
National Water Policy (NWP) which is Justification: It was set up in 1997 to fulfil
later considered by Cabinet. the obligations enunciated in the Chemical
Which of the above is/are correct? Weapons Convention (CWC).
(a)  1 only (b)  2 only • It prohibits the development, production,
(c)  Both 1 and 2 (d)  None execution, transfer, use and stockpiling of
Solution: (c) all chemical weapons by Member-States
Justification: Statement 1: The National in a non-discriminatory manner.
Water Resources Council (NWRC) was • The Parliament in 2000 enacted the CWC
set up by the Government of India in 1983. Act which came into force in 2015 to give
The Prime Minister is the Chairman and effect to the provisions of the Chemical
Union Minister of Water Resources, River Weapons Convention in India.
Development and Ganga Rejuvenation is the • In accordance with the provisions of
Vice-Chairman of the Council. this Act a high-level steering committee
The Union Minister of State for Water under the Chairmanship of the Cabinet
Resources, concerned Union Ministers/ Secretary with Secretary (Chemical and
Ministers of State, and Chief Ministers Petrochemicals), Foreign Secretary,
of all States & Lieutenant Governors/ Secretary, Defence Research and
Administrators of the Union Territories are Development, Defence Secretary and
the Members. Chairman, National Authority as its other
Statement 2: The draft National Water members, oversees the functioning of the
Policy (2012) was first considered by National Authority.
the National Water Resources Council • The National Authority is responsible for
(NWRC)  for adoption. Thereafter, the implementation of the CWC Act, liaison

Indian Polity Question Bank P.473

05-Indian Polity_Q1507-2004.indd 473 8/7/2018 7:50:00 PM


with Organisation for the Prohibition of external threats to the country, and oversees
Chemical Weapons (OPCW) and other state strategic issues.
parties, fulfilling of declaration obligation, The NSA of India also serves as the Prime
negotiating facility agreements etc. Minister’s Special Interlocutor on border
Q.1670 Which of the following statements with issues with China, and frequently accompanies
reference to the Office of the National the Prime Minister on Foreign State visits.
Security Advisor is INCORRECT? Brajesh Mishra was appointed the first
(a)  The office can be held only by officers National Security Adviser of India. The post
of the Indian Police Service (IPS) Cadre. was created in 1998 by the Government of
(b) NSA is the chief executive of the National Atal Bihari Vajpayee.
Security Council. Q.1672 Consider the following statements.
(c)  Brajesh Mishra was the first National 1. The Nuclear Command Authority is
Security Advisor. headed by the Minister of Home Affairs
(d)  None of the above and situated in Parliamentary premises.
Solution: (a) 2. National Security Advisor (NSA) is the
Justification and Learning: Option A: All chief executive of the National Security
the NSAs appointed since the inception of Council (NSC).
the post belong to the Indian Foreign Service 3. Cabinet Committee on Security is chaired
except M K Narayanan and the incumbent, by the Prime Minister.
Ajit Doval, who belong to the Indian Police Select the correct answer using the codes
Service. So, A is incorrect. below.
The National Security Advisor (NSA) is (a)  1 and 2 only (b)  2 and 3 only
the chief executive of the National Security (c)  3 only (d)  1, 2 and 3
Council (NSC), and the primary advisor to Solution: (b)
the Prime Minister of India on national and Justification: Statement 1: The Nuclear
international security. Command Authority comprises a Political
It is the National Security Advisor to Council and an Executive Council. The
whom intelligence agencies such as the Political Council is chaired by the Prime
Research and Analysis Wing and Intelligence Minister. It is the sole body which can
Bureau report, rather than directly to the authorise the use of nuclear weapons.
Prime Minister. Statement 2: The Executive Council is
Due to such vested powers, NSA is chaired by the National Security Advisor.
a prominent and powerful office in the Moreover, NSA is the chief executive of NSC.
bureaucracy. the NSC is an executive government agency
Q.1671 Consider the following statements about the tasked with advising the Prime Minister’s
post of National Security Advisor, India. office on matters of national security and
1. He is appointed by the Parliament. strategic interest.
2. He reports directly to the intelligence Statement 3: The Cabinet Committee on
agencies of India. Security (CCS) reviews the progress in the
3. He heads the National Security Council. operationalizing of India’s nuclear doctrine. It
Select the correct answer using the codes consists of Prime Minister, MHA (Ministry of
below. Home Affairs), MoEA (Ministry of external
(a)  1 and 2 only (b)  2 and 3 only affairs), Minister of Finance, Minister of
(c)  3 only (d)  None of the above Defence and Minister of Corporate affairs.
Solution: (d) Q.1673 Which of the following authorities have a
Justification: He is appointed by the Prime bearing on the decision of Nuclear retaliation
Minister, not the Parliament. So, 1 is wrong. to a first strike on India?
The National Security Adviser (NSA) 1. Nuclear Command Authority
is tasked with regularly advising the Prime 2. Indian Armed Forces
Minister on all matters relating to internal and 3. Parliament

P.474 For Civil Services Preliminary Examination

05-Indian Polity_Q1507-2004.indd 474 8/7/2018 7:50:00 PM


Choose the correct answer from the Justification and Learning: The SFC is a
codesgiven below. specially raised missile-handling unit of the
(a)  1 and 2 only (b)  2 and 3 only Indian Army. It forms part of India’s Nuclear
(c)  1 and 3 only (d)  1 only Command Authority (NCA).
Solution: (d) It was created in 2003, by an executive
Nuclear retaliatory attacks can only order of Cabinet Committee on Security
be authorised by the civilian political (CCS) headed by then Prime Minister Atal
leadership through the Nuclear Command Bihari Vajpayee.
Authority. The Nuclear Command It is responsible for the management and
Authority comprises a Political Council and administration of the country’s tactical and
an Executive Council. It provides inputs for strategic nuclear weapons stockpile.
decision making by the Nuclear Command It is headed by a Commander-in-Chief of
Authority and executes the directives given the rank of Air Marshal. So, D is wrong.
to it by the Political Council. It is out of the RTI-ambit to ensure
Q.1674 As per India’s nuclear doctrine (2003), nuclear national security.
weapons will only be used in retaliation Q.1676 Which of the following describes the
against a nuclear attack on Indian Territory ‘Mission’ of the Atomic Energy Regulatory
or on Indian forces anywhere. Nuclear Board (AERB) most appropriately?
retaliatory attacks can only be authorised by (a) Safeguarding public health and
(a)  Cabinet Committee on Security environment from hazards of nuclear
(b)  National Security Council energy
(c)  Nuclear Command Authority (b) Addressing energy security of the nations
(d)  Indian Armed Forces by establishing high end nuclear power
Solution: (c) stations
Learning: The Nuclear Command Authority (c) Supervising quality research in matters
comprises a Political Council and an Executive of fundamental nuclear energy to serve
Council. The Political Council is chaired by national interest
the Prime Minister. It is the sole body which (d) Oversee the technical operations of
can authorise the use of nuclear weapons. nuclear power centres in India
The Executive Council is chaired by the Solution: (a)
National Security Advisor. It provides inputs Explanation and Learning: The Mission
for decision making by the Nuclear Command of the Atomic Energy Regulatory Board
Authority and executes the directives given (AERB) is to ensure the use of ionising
to it by the Political Council. radiation and nuclear energy in India does not
The Cabinet Committee on Security (CCS) cause undue risk to the health of people and
reviews the progress in the operationalizing the environment.
of India’s nuclear doctrine including the It was established in 1983 by the
existing command and control structures, the President of India. The AERB is supported
state of readiness etc. in its functions by a number of committees.
Q.1675 Which of the following statements is Members of all the AERB committees are
INCORRECT with reference to the Strategic recognized experts with long experience in
Forces Command (SFC)? the relevant fields. The regulatory authority
(a)  It is a unit of the Indian Army. of AERB is derived from the rules and
(b) It was created by an executive order of notifications promulgated under the Atomic
Cabinet Committee on Security (CCS). Energy Act, 1962 and the Environmental
(c) It forms part of India’s Nuclear Command (Protection) Act, 1986.
Authority (NCA). Q.1677 The functions of the Atomic Energy
(d) It is headed by the Prime Minister of Commission is/are

India. 1. To undertake prospecting and extraction
Solution: (d) of atomic minerals in India

Indian Polity Question Bank P.475

05-Indian Polity_Q1507-2004.indd 475 8/7/2018 7:50:00 PM


2. To organise research in atomic energy and Atomic Energy is ex-officio Chairman of the
train scientists in the country Commission. The other Members of the AEC
Which of the above is/are correct? are appointed for each calendar year on the
(a)  1 only (b)  2 only recommendation of the Chairman, AEC and
(c)  Both 1 and 2 (d)  None after approval by the Prime Minster.
Solution: (c) Q.1679 The Central Electricity Authority (CEA)
Justification: The Atomic Energy 1. is a Statutory Body.
Commission (AEC) was set up in 1948 to 2. has the generating stations need not
look after atomic energy activities in the compulsorily follow the CEA’s technical
country. The functions of the Atomic Energy standards specified for construction of
Commission are electrical plants and electric lines.
• to organise research in atomic scientists in 3. Is headed by the Union Home Minister.
the country; Which of the above statements are true?
• to train, atomic scientists in the country; (a)  Only 1. (b)  Only 1 & 2.
• to promote nuclear research in (c)  Only 1 & 3. (d)  All
commission’s own laboratories as well as Solution: (b)
in India; • The Central Electricity Authority of
• to undertake prospecting of atomic India (CEA) is a statutory organization
minerals in India and to extract such constituted under section 3(1) of
minerals for use on industrial scale. Electricity Supply Act 1948, which has
Learning: The Secretary to the been superseded by section 70(1) of the
Government of India in the Department of Electricity Act 2003.
Atomic Energy is ex officio Chairman of the • The CEA advises the government on
Commission. The other Members of the AEC matters relating to the National Electricity
are appointed for each calendar year on the Policy and formulates short-term and
recommendation of the Chairman, AEC and perspective plans for the development of
after approval by the Prime Minster. electricity systems. Under the Electricity
Q.1678 Consider the following about the governance Act 2003, CEA prescribes the standards on
of Atomic Energy in India. matters such as construction of electrical
1. The Department of Atomic Energy (DAE) plants, electric lines and connectivity
is under the direct charge of the Prime to the grid, installation and operation of
Minister. meters and safety and grid standards.
2. Atomic Energy Commission (AEC) of • The CEA is also responsible for
India is the governing body of the DAE. concurrence of hydro power development
3. Cabinet Secretary is the ex-officio schemes of central, state and private
Chairman of the AEC. sectors taking into consideration the
Select the correct answer using the codes factors which will result in efficient
below. development of the river and its tributaries
(a)  1 and 2 only (b)  2 only for power generation, consistent with the
(c)  2 and 3 only (d)  1, 2 and 3 requirement of drinking water, irrigation,
Solution: (a) navigation and flood control.
Justification and Learning: Statement 1: • Preparation of technical standards for
The Department of Atomic Energy (DAE) construction of electrical plants, electric
came into being in 1954 under the direct lines and connectivity to the grid is the
charge of the Prime Minister through a responsibility of CEA as per section
Presidential Order. 73 (b) of the Electricity Act, 2003.
Statement 2: Explained in previous However as per section 7 of this Act, a
answer generating company may establish, operate
Statement 3: The Secretary to the and maintain a generating station if it complies
Government of India in the Department of with the technical standards only relating to

P.476 For Civil Services Preliminary Examination

05-Indian Polity_Q1507-2004.indd 476 8/7/2018 7:50:00 PM


connectivity to the grid as given in clause (b) Ministers. The body meets twice an year. So,
of section 73. This implies that generating the option (b) is incorrect.
stations need not follow compulsory the CEA Not so important facts like this are
technical standards specified for construction often asked by UPSC. You should also pay
of electrical plants and electric lines. attention to these details.
• The CEA plays a lead role in promoting Learning: The mandate of the National
the integrated operations of the regional Council of Senior Citizens is to advise
power grids and the evolution of a national the Central and State Governments on the
grid. The eastern, western and north- entire gamut of issues related to the welfare
eastern grids have been integrated and of senior citizens and enhancement of their
are operating in a synchronous mode. The quality of life, with special reference to the
eastern grid is connected to the northern following:-
as well as southern grid through High- • Policies, programmes and legislative
Voltage Direct Current (HVDC) back measures.
to back links. The western grid is also • Promotion of physical and financial
connected to the northern and southern security, health and independent and
grid through similar arrangements. The productive living.
CEA facilitates exchange of power within • Awareness generation and community
the country from surplus to deficit regions mobilization.
and with neighbouring countries for Q.1681 Consider the following about National
mutual benefits. Population Commission (NPC) of India.
• The CEA advises central government, 1. It is chaired by the Prime Minister of
state governments and regulatory India.
commissions on all technical matters 2. It conducts the decennial demographic
relating to generation, transmission and Census in India.
distribution of electricity. It also advises Which of the above is/are correct?
state governments, licensees or generating (a)  1 only (b)  2 only
companies on matters which enable them (c)  Both 1 and 2 (d)  None
to operate and maintain the electricity Solution: (a)
system under their ownership or control Justification: Statement 1: It is chaired by
in an improved manner. the prime minister with the Deputy Chairman
Q.1680 Which of the following statements is Planning Commission as vice chairman.
INCORRECT about National Council of Chief Ministers of all states, ministers of
Senior Citizens which is chaired by the the related central ministries, secretaries of the
Minister of Social Justice & Empowerment? concerned departments, eminent physicians,
(a) The council was originally known as demographers and the representatives of the
National Council for Older Persons civil society are members of the commission.
(NCOP). Statement 2: It is conducted by the
(b) The Council is a permanent body that Registrar General and Census Commissioner
meets on a daily basis. of India.
(c) It is an advisory body to both Central and Learning: The commission has the
State governments. mandate
(d)  None of the above 1. to review, monitor and give direction for
Solution: (b) implementation of the National Population
Justification: You can solve this question by Policy with the view to achieve the goals
elimination. set in the Population Policy
If the body is chaired by a person like the 2. promote synergy between health,
Minister of Social Justice & Empowerment, educational environmental and
it is NOT possible for the body to meet on developmental programmes so as to
a daily basis given the busy schedule of the hasten population stabilization

Indian Polity Question Bank P.477

05-Indian Polity_Q1507-2004.indd 477 8/7/2018 7:50:00 PM


3. promote inter-sectoral coordination in telecommunications business in India.
planning and implementation of the Its main purpose is to deliver a fair and
programmes through different sectors and transparent environment for fair competition
agencies in Centre and the States. in telecom market.
4. develop a vigorous peoples programme to It came into existence by the Act of the
support this national effort. Parliament in 1997. It was established in wake
Q.1682 The Government has recently decided to of entry of private sector in telecom industries
establish the National Tribal Advisory Council after Government had constituted the National
(NTAC). Consider the following about it. Telecom Policy (NTP) to attract domestic and
1. It will be a constitutional body setup Foreign Direct Investment (FDI) investment
under the compulsory provisions of the in the telecommunication sector.
Fifth Schedule of the Constitution. Statement 2: TRAI also fixes or revises
2. It will be chaired by the Prime Minister. the tariffs for telecom services in India.
3. It will approve the release of tribal welfare TRAI also has punitive mandate.
funds to both Fifth and Sixth Schedule Recently it had recommended Department of
areas. Telecom (DoT) to impose combined penalty
Select the correct answer using the codes of Rs. 3,050 crore on Bharti Airtel, Vodafone
below. and Idea cellular.
(a)  1 and 2 only (b)  2 only Q.1684 Consider the following statements regarding
(c)  1 and 3 only (d)  None of the above the Telecom Regulatory Authority of India
Solution: (b) (TRAI).
Justification: Government has recently 1. It is a statutory body.
(2015) decided to set up a National Tribal 2. The Head of the authority should either be
Advisory Council for effecting monitoring a retired Supreme Court Judge or a retired
and implementation of various tribal welfare High Court Chief Justice.
schemes. Which of the above statements are true?
Statement 1: The NTAC is different from (a)  Only 1. (b)  Only 2.
a TAC that is setup under Fifth Schedule (c) Both. (d) None.
provisions. As per Para 4(1) of the Fifth Solution: (a)
Schedule of the Constitution, there shall be Explanation: The Telecom Regulatory
TAC in each State having Scheduled Areas Authority of India (TRAI) was established
therein and , if the President should directs, with effect from 20th February 1997 by
also in any State having Scheduled Tribes but an Act of Parliament, called the Telecom
non- Scheduled Areas Therein Regulatory Authority of India Act, 1997, to
Statement 2: The council is chaired by the regulate telecom services, including fixation/
Prime Minister and will meet once or twice revision of tariffs for telecom services
in a year. which were earlier vested in the Central
Statement 3: It monitors and does not Government. The TRAI is administered
approve funds for tribal welfare. through a Secretariat headed by a secretary.
Q.1683 Telecom Regulatory Authority of India Q.1685 Consider the following about Cyber appellate
(TRAI) is empowered to tribunal, India.
1. Regulate telecom market to ensure fair 1. It has been established under the
competition Information Technology Act, 2000.
2. Fix tariffs for telecom services in India 2. It is not bound by the Civil Procedure
Which of the above is/are correct? Code and guided by the principles of
(a)  1 only (b)  2 only natural justice.
(c)  Both 1 and 2 (d)  None Which of the above is/are correct?
Solution: (c) (a)  1 only (b)  2 only
Justification: Statement 1: The TRAI (c)  Both 1 and 2 (d)  None
is an independent regulator of the Solution: (c)

P.478 For Civil Services Preliminary Examination

05-Indian Polity_Q1507-2004.indd 478 8/7/2018 7:50:00 PM


Justification: Statement 1: It is the first and but at the same time the Tribunal is guided by
the only Cyber Appellate Tribunal in the the principles of natural justice.
country which has been established by the The Cyber Appellate Tribunal has powers
Central Government. to regulate its own procedure including
Statement 2: The Cyber Appellate the place at which it has its sittings. Every
Tribunal has powers to regulate its own proceeding before the Cyber Appellate
procedure including the place at which it Tribunal shall be deemed to be a judicial
has its sittings. Every proceeding before the proceeding within the meaning of sections
Cyber Appellate Tribunal shall be deemed to 193 and 228, and for the purposes of section
be a judicial proceeding. 196 of the Indian Penal Code and the Cyber
It has the same powers as are vested in a Appellate Tribunal shall be deemed to be a
civil court under the Code of Civil Procedure. civil court for the purposes of section 195
Learning: The selection of the Chairperson and Chapter XXVI of the Code of Criminal
and Members of the Tribunal is made by the Procedure, 1973.
Central Government in consultation with the Q.1687 Which of the following will be the main
Chief Justice of India. agency handling the establishment of National
The Presiding Officer of a Cyber Appellate Cyber Coordination Centre (NCCC)?
shall hold office for a term of five years from (a)  The Cyber Society of India (CySI)
the date on which he enters upon his office (b)  Ministry of Home Affairs (MHA)
or until he attains the age of sixty five years, (c) Indian Computer Emergency Response
whichever is earlier. Team (CERT-In)
Q.1686 Consider the following statements regarding (d)  National Informatics Centre (NIC)
the Cyber Appellate Tribunal: Solution: (c)
1. It has been established under the Learning: It is a cyber security and
provisions of the Information Technology e-surveillance agency in India.
Act, 2000. • Some of the components of NCCC include
2. It is the first and the only Cyber Appellate the cyber-crime prevention strategy, cyber-
Tribunal in the country. crime investigation training, review of out
3. The Cyber Appellate Tribunal has powers dated laws, etc.
to regulate its own procedure including • Indian and U.S. intelligence agencies
the place at which it has its sittings. are also working together to curb misuse
Which of the above statements are true? of social media platforms in the virtual
(a)  Only 1. (b)  Only 2 & 3. world by terror groups.
(c)  Only 1 & 3. (d)  All. • Some have expressed concern that the
Solution: (d) body could encroach on Indian citizens’
Explanation: Cyber Appellate Tribunal privacy and civil-liberties, given the lack
(CAT) has been established under the of explicit privacy laws in the country.
Information Technology Act under the • This Centre will have top experts from
aegis of Controller of Certifying Authorities the field and it will be run like similar
(CCA). The first and the only Cyber Appellate organisation in other countries such as the
Tribunal in the country has been established US, the UK, France, Germany, etc.
by the Central Government in accordance with Q.1688 Consider the following statements regarding
the provisions contained under Section 48(1) the Indian Computer Emergency Response
of the Information Technology Act, 2000. Team (CERT-In):
The Cyber Appellate Tribunal has, for the 1. It is the Government organization
purposes of discharging its functions under under Ministry of Communications and
the IT Act, the same powers as are vested in a Information Technology.
civil court under the Code of Civil Procedure, 2. It is a nodal agency that deals with
1908. However, the procedure was laid down cyber security threats like hacking and
by the Code of Civil Procedure, 1908 applies phishing.

Indian Polity Question Bank P.479

05-Indian Polity_Q1507-2004.indd 479 8/7/2018 7:50:00 PM


Which of the above statements are true? information security practices, procedures,
(a)  Only 1. (b)  Only 2. prevention, response and reporting of
(c) Both. (d) None. cyber incidents
Solution: (c) Q.1690 The mandate of Competition Commission
Explanation: Indian Computer Emergency of India (CCI) is to oversee which type of
Response Team under Ministry of practices?
Communications and Information 1. Whether mergers and acquisitions are
Technology. It is a nodal agency that deals leading to monopoly in the markets
with cyber security threats like hacking and 2. Whether firms are dealing with consumer
phishing. It strengthens security-related grievances on a regular basis
defence of the Indian Internet domain. In 3. Whether commercial banks have adhered
March 2014, CERT reported a critical flaw in to the lending limits prescribed by RBI
Android Jelly Bean’s Virtual Private Network Select the correct answer using the codes
(VPN) implementation. below.
Q.1689 Indian Computer Emergency Response (a)  1 and 2 only (b)  1 only
Team (CERT-In) has been designated under (c)  2 and 3 only (d)  1 and 3 only
the Information Technology (Amendment) Solution: (b)
Act 2008 to serve as the national agency to Justification: The CCI was established to
perform which of the following functions? eliminate practices that adversely affect
1. Approving the commercial plans of competition in different industries and
Internet Service providers (ISPs) protect interests of consumers and ensure
2. Forecast and provide alerts of cyber freedom of trade.
security incidents So, if a merger is leading to unfair
3. Regulating the mergers and acquisitions monopolistic tendencies, CCI may fine such
of Internet Service Providers to maintain establishment and prevent the merger. So,
service quality 1 is correct.
Select the correct answer using the codes Unfair practices can happen even when
below. firms collude and set prices and output to
(a)  1 and 2 only (b)  2 and 3 only maintain their monopoly in the market,
(c)  1 only (d)  2 only which may hurt the consumer.
Solution: (d) But, it is not concerned with the regular
Justification: Statement 1: It has not role in administrative processes of the company,
overseeing the commercial or the financial neither does it regulate the firms for ensuring
aspects of ISPs in India. So, 1 is incorrect. all laws, rules and regulations are adhered.
Statement 3: Overseeing the mergers of So, the statements 2 and 3 are incorrect.
companies in India comes under the jurisdiction It is statutory body established under The
of the Competition Commission of India (CCI). Competition Act of 2002. It was established
So, the statement 3 is also incorrect. in 2003 and became fully functional in 2009.
Learning: It performs the following Q.1691 Consider the following about the Indian
functions in the domain of cyber security: Council for Cultural Relations (ICCR).
• Collection, analysis and dissemination of 1. It was founded before India’s independence.
information on cyber incidents 2. It aims to promote cultural exchange with
• Forecast and alerts of cyber security other countries and peoples.
incidents 3. It funds research in premier cultural
• Emergency measures for handling cyber institutions of India.
security incidents Select the correct answer using the codes
• Coordination of cyber incident response below.
activities (a)  2 only (b)  1 and 3 only
• Issue guidelines, advisories, vulnerability (c)  1 and 2 only (d)  None of the above
notes and whitepapers relating to Solution: (a)

P.480 For Civil Services Preliminary Examination

05-Indian Polity_Q1507-2004.indd 480 8/7/2018 7:50:00 PM


Justification: Statement 1: It was founded Choose the correct answer using the codes
in 1950 by Maulana Abul Kalam Azad, below.
independent India’s first Education (a)  1 and 2 (b)  2 and 3
Minister. (c)  1 and 3 (d)  All of the above
Statement 2: Its objectives are to Solution: (d)
actively participate in the formulation and Chairperson of Central Board of Film
implementation of policies and programmes Certification (CBFC) Leela Samson has
pertaining to India’s external cultural relations; resigned.
to foster and strengthen cultural relations and She resigned after Film Certification
mutual understanding between India and other Appellate Tribunal (FCAT) cleared the
countries; to promote cultural exchanges with controversial film ‘Messenger of God’
other countries and people; and to develop featuring Dera Saccha Sauda chief Gurmeet
relations with nations. Ram Rahim Singh in leading role.
Q.1692 With reference to the National Archives The Censor Board headed by Ms. Samson
of India (NAI), consider the following had earlier denied the clearance certificate
statements to the film saying that the movie depicts
1. It functions as an Attached Office of the Gurmeet Ram Rahim Singh as a ‘God’.
Department of Culture under the Ministry Film Certification Appellate Tribunal
of Culture, Government of India (FCAT)
2. It was set-up in 1891 in Calcutta and • The FCAT is a statutory body. It has been
at present it is located next to Writer’s constituted under the provisions of the
Building serving as treasure house of Cinematograph Act, 1952.
precious imperial records • It comes under the aegis of Ministry of
Which of the above statements is/are correct? Information and Broadcasting.
(a)  1 Only (b)  2 Only • Function: It hears the appeals filed as
(c) Both (d) None per provisions of Cinematograph Act.
Solution: (a) Any applicant can file an appeal before
The National Archives of India (NAI) is the tribunal for a certificate in respect of
a repository of the non-current records of a film who is aggrieved by an order of
the Government of India and holds them the Central Board of Film Certification
in trust for the use of administrators and (CBFC).
scholars. Originally established as the Q.1694 Indian Renewable Energy Development
Imperial Record Department in 1891, in Agency Limited (IREDA) provides financial
Calcutta, then capital of British India, the support to
NAI is situated at the intersection of Janpath 1. Generate electricity by new sources
and Rajpath, in Delhi. It functions as an 2. Conserving energy through energy
Attached Office of the Department of Culture efficiency
under the Ministry of Culture, Government Which of the above is/are correct?
of India. (a)  1 only (b)  2 only
Q.1693 Consider the following statements about Film (c)  Both 1 and 2 (d)  None
Certification Appellate Tribunal (FCAT). Solution: (c)
1. It is a statutory body. Explanation and Learning: The IREDA
2. It comes under the aegis of Ministry of is a Public Limited Government Company
Information and Broadcasting. established as a Non-Banking Financial
3. Any applicant can file an appeal before Institution in 1987 under the administrative
the tribunal for a certificate in respect of control of Ministry of New and Renewable
a film who is aggrieved by an order of Energy (MNRE) to promote, develop an
the Central Board of Film Certification extending financial assistance for renewable
(CBFC). energy and energy efficiency/conservation
projects with the motto: “ENERGY FOR

Indian Polity Question Bank P.481

05-Indian Polity_Q1507-2004.indd 481 8/7/2018 7:50:00 PM


EVER”. Its objectives are: Q.1696 Consider the following about Advertising
• To give financial support to specific Standards Council of India (ASCI)
projects and schemes for generating 1. It is a self-regulatory voluntary
electricity and/or energy through new and organization.
renewable sources and conserving energy 2. Any person can complain to the ASCI if
through energy efficiency. an advertisement released in India seems
• To increase IREDA’s share in the objectionable.
renewable energy sector by way of 3. Advertisements which violate ASCI code
innovative financing. cannot be permitted on TV.
• To strive to be competitive institution Select the correct answer using the codes
through customer satisfaction. below.
• To maintain its position as a leading (a)  1 and 2 only (b)  2 and 3 only
organization to provide efficient and (c)  1 only (d)  1, 2 and 3
effective financing in renewable energy and Solution: (d)
energy efficiency/conservation projects. Justification: Statement 1: Like other
• Improvement in the efficiency of services countries around the world, India too has
provided to customers through continual a self-regulatory organization (SRO) for
improvement of systems, processes and advertising content – The Advertising
resources. Standards Council of India, ASCI founded in
Q.1695 Consider the following about the Animal 1985.
Welfare Board of India. The three main constituents of advertising
1. It is an advisory body established by an industry, viz. advertisers, advertising
executive order. agencies and media came together to form
2. It is chaired by the Prime Minister of this independent NGO. The aim of ASCI is to
India. maintain and enhance the public’s confidence
3. The Board provides financial assistance in advertising.
to recognised Animal Welfare Statement 2: There is no other non
Organisations(AWOs). governmental body in India which regulates
Select the correct answer using the codes the advertising content that is released in
below. India. If an ad that is released in India seems
(a)  1 and 2 only (b)  2 and 3 only objectionable, a person can write to ASCI
(c)  3 only (d)  None of the above with their complaint.
Solution: (c) This complaint will be deliberated on by the
Justification: Statement 1: The Animal Consumer Complaints Council (CCC)  after
Welfare Board of India is a statutory providing due process to advertiser to defend
advisory body on Animal Welfare Laws and the ad against the complaint and depending
promotes animal welfare in the country. It on whether the ad is in alignment with the
was established in 1962 under the Prevention ASCI code and law of the land.
of Cruelty to Animals Act, 1960. So, 1 is Statement 3: In 2007, the Government of
incorrect. India amended the Cable TV Network Rules’
Statement 2: National Board for Wildlife Advertising Code by which ads which violate
is chaired by the Prime Minister, not the ASCI code cannot be permitted on TV.
welfare board. Q.1697 With reference to the Press Trust of India
Statement 3: It works to ensure that (PTI), consider the following statements
animal welfare laws in the country are 1. It is a news agency owned by the
followed and provides grants to Animal Government of India and operated under
Welfare Organisations. The Board oversees the management of the Ministry of
Animal Welfare Organisations (AWOs) by Information and Broadcasting
granting recognition to them if they meet its 2. Press Trust of India is the only news
guidelines. agency in South Asia which operates its

P.482 For Civil Services Preliminary Examination

05-Indian Polity_Q1507-2004.indd 482 8/7/2018 7:50:00 PM


own communication satellite, an INSAT, Statement 2: The Council has its own
to broadcast news and information. funds for performance of its functions under
Which of the above is/are correct? the Act that comprises of the fee collected
(a)  1 Only (b)  2 Only by it from newspapers, other receipts and
(c) Both (d) None also Grants in-Aid by from the Central
Solution: (b) Government.
Press Trust of India (PTI) is the largest news Statement 3: This is because the council
agency in India.[1] It is headquartered in New is a quasi-judicial authority and independent
Delhi and is a non-profit cooperative among from the government. Moreover, it’s a
more than 450 Indian newspapers and has 984 self-regulatory body of the Press and not a
full-time employees, as on January 8, 2014. government tribunal. This is why its orders
It employs over 400 journalists and 500 part- can’t be questioned except by way of a writ.
time correspondents located in most of the Learning: It comprises of a Chairman
district headquarters in the country. A few and 28 members. While the Chairman has,
correspondents are based in major capitals and by convention, been a sitting or retired judge
important business centres around the world. of the Supreme Court of India, of the 28
It took over the operations of the Associated members, 20 represent various segments of
Press from Reuters soon after India’s the Press and eight overseeing the readers’
independence on 15 August 1947. It provides interest, are representatives of the two Houses
news coverage and information of the region of Parliament (Three in Lok Sabha and Two
in both English and Hindi. Its corporate office in Rajya Sabha) and premier literary and legal
is located at Sansad Marg, New Delhi and bodies of the country, i.e., UGC, BCI etc.
registered office in D N Road, Mumbai. The Council discharges its functions
Q.1698 Consider the following about the Press primarily through adjudications on complaint
Council of India. cases received by it, either against the Press
1. It is a statutory quasi-judicial authority for violation of journalistic ethics or by the
autonomous from the government. Press for interference with its freedom.
2. It can collect fees from newspapers to Q.1699 Consider the following about the Press
fund the Council. Council of India.
3. The decisions of the Council are final 1. It is a quasi-judicial body.
and cannot be questioned in any court 2. It is headed by a retired judge of Supreme
of law except by way of writ under the Court by convention.
constitution. 3. It adjudicates the complaints made by the
Select the correct answer using the codes below. Indian Press.
(a)  1 only (b)  2 and 3 only 4. Citizens can complain to this authority if
(c)  2 only (d)  1, 2 and 3 the Press violates standard guidelines and
Solution: (d) engages in unethical reporting.
Justification and Learning: Statement 1: Select the correct answer using the codes
It was first set up in 1966 by the Parliament below.
on the recommendations of the First Press (a)  1, 2 and 4 only (b)  2 and 3 only
Commission. (c)  1, 3 and 4 only (d)  1, 2, 3 and 4
It has the twin objective of preserving the Solution: (d)
freedom of the press and maintaining and Q.1700 Central Board of Film Certification (CBFC)
improving the standards of newspapers and regulates the public exhibition of films in
the news agencies in India exercising equal India and has been often in news. Consider
quasi-judicial functions over the authorities the following about it.
as well the press person. 1. It is a statutory body under Ministry of
It functions under the Press Council Act Information and Broadcasting.
1978. It is a statutory, quasi-judicial body 2. Its board is appointed by the Press Trust
which acts as a watchdog of the press. Council of India.

Indian Polity Question Bank P.483

05-Indian Polity_Q1507-2004.indd 483 8/7/2018 7:50:00 PM


3. Films can be publicly exhibited in India and capacity-building among developing
only after they have been certified by the countries on international economic issues.
Central Board of Film Certification. The focus of the work programme of RIS
Select the correct answer using the codes is to promote South-South Cooperation and
below. assist developing countries in multilateral
(a)  1 and 2 only (b)  2 and 3 only negotiations in various forums. And RIS is
(c)  1 and 3 only (d)  1, 2 and 3 engaged in the Track II process of several
Solution: (c) regional initiatives. The RIS is providing
Leaning: The CBFC regulates the public analytical support to the Government of
exhibition of films under the provisions of India in the negotiations for concluding
the Cinematograph Act 1952. The Board, comprehensive economic cooperation
consists of non-official members and a agreements with partner countries. Through
Chairman (all of whom are appointed by its intensive network of policy think tanks,
Central Government) and functions with RIS seeks to strengthen policy coherence
headquarters at Mumbai. So, 2 is incorrect. on international economic issues. RIS is
1. It has nine Regional offices, one each at mandated to function as an advisory body
Mumbai, Kolkata, Chennai, Bangalore, to the Government of India on matters
Thiruvananthapuram, Hyderabad, New pertaining to multilateral economic and
Delhi, Cuttack and Guwahati. social issues, including regional and sub-
2. The Regional Offices are assisted in the regional cooperation arrangements and
examination of films by Advisory Panels. innovation policies. The RIS functions in
3. The members of the panels are nominated close association with various governmental
by Central Government by drawing bodies, research institutions, academicians,
people from different walks of life for a policy-makers, and business and industry
period of 2 years. circles in India and abroad.
Q.1701 Consider the following statements regarding The RIS has a consultative status with
the Research and Information System (RIS) United Nations Conference on Trade and
for Developing Countries: Development (UNCTAD), NAM and the
1. It is an autonomous think-tank under the WTO and has conducted policy research and
Ministry of External Affairs, Government other activities in collaboration with agencies
of India. including UN ESCAP, UNCTAD, UNU,
2. The focus of the work programme of RIS is Group of 77, SAARC Secretariat, Asian
to promote South- South Cooperation and Development Bank (ADB), The World Bank
assist developing countries in multilateral and the South Centre.
negotiations in various forums. Q.1702 Consider the following about the All
3. It is an advisory body to the Government India Council of Sports constituted by the
of India. government.
Which of the above statements is/are true? 1. President of India will be its ex officio
(a)  Only 1. (b)  Only 2 & 3. chairman.
(c) All. (d) None. 2. It will financially support all states to
Solution: c. establish a council of sports.
Explanation: Research and Information 3. Its recommendations will not be binding
System for Developing Countries (RIS), a on the government.
New Delhi based autonomous think-tank 4. It will adjudicate all complaints, disputes
under the Ministry of External Affairs, and malpractices associated with sports.
Government of India, is an organization that Choose the correct answer using the codes
specializes in policy research on international below.
economic issues and development (a)  1, 3 and 4 only (b)  3 only
cooperation. The RIS is envisioned as a (c)  1 and 2 only (d)  All of the above
forum for fostering effective policy dialogue Solution: (b)

P.484 For Civil Services Preliminary Examination

05-Indian Polity_Q1507-2004.indd 484 8/7/2018 7:50:00 PM


Union Government has constituted All India Choose the correct answer using the codes
Council of Sports (AICS), as an advisory below.
body to deliberate on matters relating to the (a)  1 and 2 only (b)  1 and 3 only
development and promotion of sports in the (c)  2 and 3 only (d)  1 only
country. Solution: (d)
Government will duly consider the advice The Board of Control for Cricket in India
rendered by this Council, but it will not be (BCCI) is the national governing body for
binding and obligatory on the government. cricket in India. The board was formed in
• Composition: It will be headed by a December 1928 as a society, registered
president in the rank of Minister of State under the Tamil Nadu Societies Registration
(MoS). It will include 4 Members of Act. It is a consortium of state cricket
Parliament (MPs), coaches, experts, sports associations and the state associations select
personalities, administrators and Director their representatives who in turn elect the
General (DG) of Sports Authority of India BCCI officials. The BCCI’s constitution
(SAI). provides for annual elections at its Annual
• Appointments: President and members, General Meeting (AGM) for all posts, with a
other than ex-officio members will be bar on re-election of an incumbent president
appointed by the Union Government. beyond two consecutive years, “provided
• Meeting: It will meet from time to time that the General Body may in its discretion
i.e. at least once in a quarter of year to re-elect the same person as president for the
deliberate on matters related sports and third consecutive year” The BCCI has been
games in the country. known to use its power to influence certain
• Functions: Popularize sports amongst the ICC decisions. These included scheduling,
youth and increase its outreach to rural player suspension and ICC appointments.
and tribal areas along with North East and As India is a large market in terms of
Jammu and Kashmir and areas affected by international cricket revenue, the BCCI’s
Left Wing Extremism and Maoists. opinions carry weight within the ICC’s
• Promote inclusiveness in sports with decision making process.
special focus on women, tribal and Q.1704 Consider the following about National
differently-abled. Mineral Exploration Trust (MET).
• Prevent fraud of age and sexual harassment 1. It grants mining leases for major minerals
of women in sports along with drug abuse. to after obtaining sanction from the State
• Bring transparency, professionalism government.
and good governance in functioning of 2. It carries out detailed exploration for
National Sports Federations (NFS). minerals using the funds accrued to it.
• Promote sports sciences and sports medicine 3. It invests in basic infrastructural
and augment the sports infrastructure and development and implements welfare
ensure its proper utilization. schemes in mining affected belts.
• Tackle the issues arising malpractices in Select the correct answer using the codes below.
competitive sports especially match fixing (a)  1 only (b)  2 only
and early identification of sports talent (c)  1 and 3 only (d)  1 and 2 only
and nurturing the identified talent. Solution: (b)
Q.1703 Consider the following about the Board of Justification: Statement 1: Major minerals
Control for Cricket in India (BCCI). are administered by the Central government.
1. It is an autonomous society not controlled Moreover, this body doesn’t give sanction for
by the Government of India. mining leases. So, 1 is wrong.
2. It is subordinate to International Cricket Statement 2: The Mines and Minerals
Council. (Development & Regulation) Amendment
3. It comes under the purview of the RTI Act (MMRDA), 2015, mandated the setting
Act. up of Mineral Exploration Trust.

Indian Polity Question Bank P.485

05-Indian Polity_Q1507-2004.indd 485 8/7/2018 7:50:00 PM


It is a non-profit body by the Central • The Economic Enforcement is economic
Government for the purposes of regional intelligence and law enforcement agency
and detailed exploration of minerals using responsible for enforcing economic laws
the funds accrued to it and in such manner as and fighting economic crime in India.
prescribed by the Central Government. • It functions under the aegis of the
Statement 3: It is done by District Mineral Department of Revenue, Union Ministry
Foundation. So, the statement 3 is wrong. of Finance. So, the option (b) is incorrect.
Q.1705 Consider the following about the Directorate Q.1707 A Council for Trade Development and
of Enforcement. Promotion was constituted in 2015 to ensure
1. It is housed under the Ministry of 1. Administration of export promotion
Corporate Affairs. schemes
2. It enforces the Prevention of Money 2. Cooperation between the Centre and the
Laundering Act. State for export promotion
3. It keeps track of foreign exchange Which of the above is/are correct?
movement in and out of India. (a)  1 only (b)  2 only
Select the correct answer using the codes (c)  Both 1 and 2 (d)  None
below. Solution: (b)
(a)  1 and 2 only (b)  2 and 3 only Justification: Statement 2: The government
(c)  1 only (d)  2 only has set up a Trade Facilitation Council
Solution: (b) with representations from states and Union
Justification: Statement 1: Directorate territories to mainstream them in the process
of Enforcement is a specialized financial of international trade and assist them with
investigation agency under the Department their infrastructural needs.
of Revenue, Ministry of Finance. So, 1 is Statement 1: It is not an executive body.
wrong. It will make the states active partners in
Statements 2 and 3: It enforces the Foreign boosting India’s exports.
Exchange Management Act, 1999 (FEMA) Q.1708 Consider the following about Insolvency
and Prevention of Money Laundering Act and Bankruptcy Board of India established
(PMLA), 2002. recently.
The Act and Rules notified there under 1. It is an executive body established by the
impose obligation on banking companies, Competition Commission of India (CCI).
financial institutions and intermediaries to 2. It can make model bye-laws to be adopted
verify identity of clients, maintain records and by insolvency professional agencies
furnish information working in India.
Q.1706 Which of the following statements about 3. It is chaired by Union Minister of
the Enforcement Directorate (ED) is Corporate Affairs.
INCORRECT? Select the correct answer using the codes
(a) ED is appointed on the approval of the below.
Appointments Committee of the Cabinet (a)  1 and 3 only (b)  2 only
(ACC)  headed by Prime Minister. (c)  3 only (d)  None of the above
(b) It functions under the aegis of the Central Solution: (b)
Bureau of Investigation (CBI). Background: In India, the legal and
(c) It is responsible for tracking black money institutional machinery for dealing with
and hawala trade cases. debt default has not been in line with global
(d)  None of the above standards.
Solution: (b) • The recovery action by creditors, either
Learning: The Union Government has through the Contract Act or through
appointed Senior IPS officer Karnal Singh special laws such as the Recovery of Debts
as the Director of Enforcement Directorate Due to Banks and Financial Institutions
(ED). Act, 1993 and the Securitisation and

P.486 For Civil Services Preliminary Examination

05-Indian Polity_Q1507-2004.indd 486 8/7/2018 7:50:01 PM


Reconstruction of Financial Assets and 1. It is the highest policy making body on
Enforcement of Security Interest Act, payment systems in the country.
2002, has not had desired outcomes. 2. It is the apex body responsible for printing
• So, this board established in accordance and circulating currency throughout India.
with the provisions of The Insolvency and Which of the above is/are correct?
Bankruptcy Code, 2016 is considered as (a)  1 only (b)  2 only
the biggest economic reform next only to (c)  Both 1 and 2 (d)  None
GST. Solution: (a)
Justification: Statement 2: The objective of Justification: The Central Bank of any
the new law is to promote entrepreneurship, country is usually the driving force in
availability of credit, and balance the interests the development of national payment
of all stakeholders by consolidating and systems. The Reserve Bank of India as the
amending the laws relating to reorganization central bank of India has been playing this
and insolvency developmental role and has taken several
For this it regulates the conduct of initiatives for Safe, Secure, Sound, Efficient,
insolvency professional agencies working in Accessible and Authorised payment systems
India. in the country.
Statement 3: It is distinguished public Statement 1: The Board for Regulation
servant who serves as Chairperson of the and Supervision of Payment and Settlement
Insolvency and Bankruptcy Board of India. Systems (BPSS), a sub-committee of the
Q.1709 The new Governor of the Reserve Bank of Central Board of the Reserve Bank of
India was appointed by the India is the highest policy making body on
(a)  Prime Minister’s Office payment systems in the country. The BPSS
(b)  Cabinet Secretary is empowered for authorising, prescribing
(c) Financial Sector Regulatory policies and setting standards for regulating
Appointments Search Committee and supervising all the payment and
(FSRASC) settlement systems in the country.
(d) Financial Stability and Development Statement 2: This function is managed by
Council (FSDC) RBI as a whole alongwith the help of other
Solution: (a) commercial banks, designated banks and
Learning: The Appointment Committee Security Printing and Minting Corporation of
of Cabinet(ACC) recommended the India. So, 2 is wrong.
appointment of Urjit Patel based on the Learning: The Department of Payment
recommendation of the Financial Sector and Settlement Systems of the Reserve Bank
Regulatory Appointments Search Committee of India serves as the Secretariat to the Board
(FSRASC). and executes its directions.
• The PMO appoints the RBI Governor. In India, the payment and settlement
• The latter is only an advisory body headed systems are regulated by the Payment and
by the Cabinet Secretary. Settlement Systems Act, 2007 (PSS Act).
• Some of the recent changes made to the The PSS Act as well as the Payment and
RBI Act is the inclusion of a Monetary Settlement System Regulations, 2008 framed
Policy Committee which will have there under came into effect from 2008. In
representation both from government and terms of Section 4 of the PSS Act, no person
RBI. other than the Reserve Bank of India (RBI)
• The veto power of the Governor regarding can commence or operate a payment system
key monetary policy decisions has been in India unless authorised by RBI.
revoked. The Reserve Bank has since authorised
Consider the following with reference to the payment system operators of pre-paid
Board for Regulation and Supervision of payment instruments, card schemes, cross-
Payment and Settlement Systems (BPSS). border in-bound money transfers, Automated

Indian Polity Question Bank P.487

05-Indian Polity_Q1507-2004.indd 487 8/7/2018 7:50:01 PM


Teller Machine (ATM) networks and Ministry of Surface Transport for budgetary
centralised clearing arrangements. allocation only otherwise it is under
Q.1710 Which of the following statements about ministry of Defence for administrative
the Securities Appellate Tribunal (SAT) is control, and officers from Indian Army
INCORRECT? Corps of Engineers, who are posted to
(a)  It is a statutory body. GREF on Extra Regimental Employment
(b) Retired judges of Supreme Court or High (ERE) tenure.
Courts are barred from its membership. Q.1712 The Border Roads Organization (BRO)
(c) SAT is not bound by procedure laid down functions under the
by Code of Civil Procedure. (a)  Ministry of Defence
(d)  Its presiding officer and members are (b)  Ministry of Home Affairs
appointed by the Central government. (c) Ministry of Road Transport and
Solution: (b) Highways
Justification: SAT comprises one Presiding (d) Department of Border Management
Officer who is a sitting/retired Judge of the Solution: (a)
Supreme Court or a sitting/retired Chief Learning: The Border Roads Organisation
Justice of a High Court; or a sitting or retired (BRO) develops and maintains road
Judge of a High Court who has completed not networks in India’s border areas and friendly
less than seven years of service as a Judge in neighbouring countries. Ministry of Defence
a High Court. is the parent organization of BRO.
It also contains two members who are It was established in 1960 for accelerating
persons of ability, integrity and standing economic development and strengthening
and have shown capacity in dealing with defence preparedness through rapid and
problems relating to securities market coordinated improvement of strategically
and have qualification and experience important roads along the northern and north-
of corporate law, securities law, finance, eastern boundary.
economics or accountancy. The BRO also undertakes snow clearance
Q.1711 With reference to Border Roads Organisation in high altitude areas.
(BRO), consider the following statements Q.1713 Consider the following about the Strategic
1. BRO has operations in Andaman and Forces Command (SFC).
Nicobar also 1. It forms part of the National Command
2. BRO was formed after 1962 India-China Authority (NCA).
war 2. It is exempt from Right to Information
3. BRO works under the Ministry of Defence (RTI) disclosures.
Which of the above is/are incorrect? 3. It authorizes military attacks on other
(a)  1 and 2 Only (b)  2 and 3 Only sovereign nations.
(c)  1 and 3 Only (d)  All Select the correct answer using the codes
Solution: (d) below.
The BRO has operations in 22 states, (a)  1 and 2 only (b)  2 only
including the Andaman and Nicobar Islands, (c)  1 and 3 only (d)  2 and 3 only
a union territory. Solution: (a)
The BRO was formed on 7 May 1960 as Justification: Statement 1: The SFC
the Border Roads Development Board with is responsible for the management and
Indian Prime Minister Jawaharlal Nehru as administration of the country’s tactical and
its chairman. strategic nuclear weapons stockpile. The SFC
The BRO consists of Border Roads manages and administers all strategic forces by
Development Board and the General exercising complete command and control over
Reserve Engineer Force (GREF). The nuclear assets, and producing all contingency
GREF—the execution force—under the plans as needed to fulfil the required tasks.

P.488 For Civil Services Preliminary Examination

05-Indian Polity_Q1507-2004.indd 488 8/7/2018 7:50:01 PM


Statement 2: Union Government has Select the correct answer using the codes
recently issued a notification mentioning below.
SFC has been brought under sub-section (2) (a)  1 and 2 only (b)  1 and 3 only
of Section 24 of the RTI. The Section 24 of (c)  2 only (d)  1, 2 and 3
the RTI exempts security and intelligence Solution: (a)
establishments from its purview. Justification: Statement 1: First established
Statement 3: Ultimate approval must come in November 1945 as an advisory body and
from the Cabinet and then the President; not later on in 1987 as given statutory status by an
the NSC or SFC. So, the statement 3 is wrong. Act of Parliament, the AICTE is responsible
Q.1714 Consider the following statements about the for proper planning and coordinated
‘Wildlife institute of India’. development of the technical education and
1. It is an autonomous body. management education system in India.
2. It is under the Ministry of Environment The AICTE accredits postgraduate and
and Forests. graduate programs under specific categories
3. It has recently launched an initiative for at Indian institutions as per its charter.
allowing PSU and Companies to adopt UGC, a statutory body, provides
endangered species. recognition to universities in India,
4. It has proposed that wildlife conservation and disburses funds to such recognised
efforts spending be counted under universities and colleges.
Corporate Social responsibility (CSR). Statement 3: In a 2013 judgement the
Choose the correct answer using the codes Supreme Court said “as per provisions
below. of the AICTE Act and University Grants
(a)  1 and 2 only (b)  1, 3 and 4 only Commission (UGC) Act, the council
(c)  2, 3 and 4 only (d)  All of the above (AICTE) has no authority which empowers it
Solution: (d) to issue or enforce any sanctions on colleges
The Wildlife Institute of India (WII) is an affiliated with the universities as its role is to
autonomous institution under the Ministry provide guidance and recommendations.
of Environment and Forests, Government Q.1716 The ‘X’ is recognized as International
of India. WII carries out wildlife research in living experiment in human unity, endorsed
areas of study like Biodiversity, Endangered by the UNESCO and supported by the
Species, Wildlife Policy, Wildlife Government of India. The ‘X’ foundation
Management, Wildlife Forensics, Spatial is also an autonomous body under the
Modelling, Eco-development, and Climate Ministry of Human Resource Development.
Change. launches initiative for allowing PSU The X is?
and Companies to adopt endangered species (a) Auroville (b) Mahabalipuram
such as Great Indian bustard, Gharial, Lesser (c)  Daman and Diu (d)  Ladakh
Florican, Snow Leopard etc. The spending Solution: (a)
will be counted under CSR in Companies Learning: The Auroville (City of Dawn)
Act 2013. This is because Budget 2015 has is an experimental township in Viluppuram
reduced its funding by 25%. district in Tamil Nadu with some parts in the
Q.1715 Consider the following with reference to the Union Territory of Puducherry.
University Grants Commission (UGC) and The Auroville was meant to be a
All India Council for Technical Education universal town where men and women of
(AICTE). all countries are able to live in peace and
1. Both of them are statutory bodies. progressive harmony and, above of the all
2. Both of them are under the Ministry of creeds, politics and all nationalities.
Human Resource Development (MHRD). The purpose of Auroville is to realize
3. Both of them have the authority to enforce human unity. The Auroville Foundation is a
any sanctions on colleges affiliated with Statutory Body.
the universities regulated by them.

Indian Polity Question Bank P.489

05-Indian Polity_Q1507-2004.indd 489 8/7/2018 7:50:01 PM


Q.1717 Consider the following about the Unique These enforceable rights were incorporated
Identification Authority of India (UIDAI). in the Part III of the Constitution.
1. The Unique Identification project was The non-justifiable rights were
conceived by then Planning Commission incorporated as a directive to the state to
to give unique identity number for each take all measures to provide those rights to
resident. individuals without any guarantee.
2. The authority is chaired by the Prime Statement 2: There was no such provision,
Minister of India. in fact even the word privy purse was not
3. Union Cabinet Ministers nominated by mentioned in the report.
the President of India are some of the ex- Q.1719 The Prime Minister’s High Level Committee,
officio members of the UIDAI. popularly known as Sachar Committee was
Select the correct answer using the codes setup with a focus on
below. (a) Curbing violence and discrimination
(a)  1 only (b)  2 and 3 only against Dalits
(c)  1 and 3 only (d)  1, 2 and 3 (b) Blockades in social and economic
Solution: (a) development of Women in India and
Justification: UIDAI was constituted by the means to empower them
Government of India vide an order in 2009. (c) Social, economic, and educational
Statement 1: After the idea was conceived conditions of the Muslim community of
by the PC, the concept of unique identification India
was first discussed and worked upon in 2006, (d) Creating social harmony between the
when administrative approval for the project people of North-eastern India and the
as entitled Unique ID for Below Poverty Line rest of India
(BPL) families was given by the Department Solution: (c)
of Information Technology. Learning: The committee was headed by
Statement 3: No Union Ministers are ex- former Chief Justice of the Delhi High Court,
officio members of the authority. The UIDAI Rajinder Sachar and had six other members.
tries to use expertise of market leaders and • The report was the first of its kind to
researchers. reveal the economic and social lag of the
Muslim community compared with other
(85) COMMITTEES & COMMISSIONS communities.
• An issue highlighted was that while
Committee Muslims constitute 14% of the Indian
population, they only comprise 2.5% of
Q.1718 The Sapru committee report that was the Indian bureaucracy.
published in 1945 advocated that • The Sachar Committee concluded that
1. Fundamental Rights must be included in the conditions facing Indian Muslims
the Constitution of India. was below that of Scheduled Castes and
2. Princely states should be integrated in the Scheduled Tribes.
Indian Union without giving them privy • In November 2013, the Gujarat
purse. government contended before the
Which of the above is/are correct? Supreme Court that the Rajinder Sachar
(a)  1 only (b)  2 only Committee was “unconstitutional,” and
(c)  Both 1 and 2 (d)  None that it only sought to help Muslims.
Solution: (a) • It has strongly criticized the manner
Justification: Statement 1: This committee in which the PMO set up the Sachar
divided fundamental rights into two parts viz. Committee in 2005 to survey the socio-
Justifiable Rights and Non-justifiable rights. economic conditions of Muslims, while
The Justifiable rights were those enforceable “ignoring” other religious minorities.
by a court of law.

P.490 For Civil Services Preliminary Examination

05-Indian Polity_Q1507-2004.indd 490 8/7/2018 7:50:01 PM


Q.1720 With reference to justice Verma committee imprisonment or fine or both under Section
report on amendments to criminal law, 509 of the IPC. This section should be
consider the following statements repealed. The Committee has suggested that
1. Any non-consensual penetration of a use of words, acts or gestures that create
sexual nature, even within marriages, an unwelcome threat of a sexual nature
should be included in the definition of should be termed as sexual assault and be
rape. punishable for one year imprisonment or
2. Non-penetrative forms of sexual contact fine or both.
should be regarded as sexual assault. Police reforms: The Committee has
3. The Committee has suggested that use recommended certain steps to reform the
of words, acts or gestures that create an police. These include establishment of State
unwelcome threat of a sexual nature Security Commissions to ensure that state
should be termed as sexual assault and governments do not exercise influence on
be punishable for 1 year imprisonment or the state police. Such Commissions should
fine or both. be headed by the Chief Minister or the Home
4. Establishment of State Security Minister of the state. The Commission would
Commissions to ensure that state lay down broad policy guidelines so that the
governments do not exercise influence on Police acts according to the law. A Police
the state police Establishment Board should be established to
Which of the statements given above is/are decide all transfers, postings and promotions
correct? of officers. Director General of Police and
(a)  1, 2 and 3 Only (b)  1, 2 and 4 Only Inspector General of Police should have a
(c)  1, 3 and 4 Only (d)  1, 2, 3 and 4 minimum tenure of 2 years.
Solution: (d) Q.1721 In 2014, Supreme court of India setup a
Sexual assault: Currently, “assault or use Committee under N. R. Madhava Menon to
of criminal force to a woman with the intent frame guidelines for
to outrage her modesty” is punishable under (a) Allowing new mining operations in left
Section 354 of the Indian Penal Code (IPC) wing extremism affected areas
with two years’ imprisonment. The term (b) Government advertisements in print and
outraging the modesty of a woman is not media
defined in the IPC. Thus, where penetration (c) Regulation of economic activities in Eco-
cannot be proved, the offence is categorized Sensitive Zones (ESZs)
as defined under Section 354 of the IPC. (d) Improving the socio-economic status of
The Committee recommended that non- the Transgender
penetrative forms of sexual contact should Solution: (b)
be regarded as sexual assault. The offence Learning: To keep politics away from such
of sexual assault should be defined so as to ads, the committee emphasized that only
include all forms of non-consensual non- the pictures and names of the President,
penetrative touching of a sexual nature. The the Prime Minister, Governor and Chief
sexual nature of an act should be determined Ministers should be published.
on the basis of the circumstances. Sexual 1. The committee also endorsed the
gratification as a motive for the act should not suggestions of the Election Commission
be prerequisite for proving the offence. The that there must be severe restrictions on
offence should be punishable with 5 years such advertisements six months prior to
of imprisonment, or fine, or both. Use of election.
criminal force to disrobe a woman should be 2. Apart from this, the committee also
punishable with 3 to 7 years of imprisonment. recommended that a deadline should be
Verbal sexual assault: At present, use fixed to prohibit their publication and the
of words or gestures to “insult a woman’s poll panel should be authorised for the
modesty” is punishable with one year of purpose.

Indian Polity Question Bank P.491

05-Indian Polity_Q1507-2004.indd 491 8/7/2018 7:50:01 PM


3. The committee also recommended (c) Burmese Enclaves in the north-eastern
that there should only be a single frontier of India
advertisement, preferably by Information (d) North-west frontier of India post-Soviet
and Broadcasting Ministry, in respect of invasion
commemorative advertisements, which Solution: (b)
are given on birth and death anniversary Learning: The Radcliffe Line was published
of an important personality on 17 August 1947 as a boundary demarcation
Q.1722 The Union Information and Broadcasting line between India and Pakistan.
(I&B) Ministry has recently constituted a Pakistan included the provinces of West
three-member Committee headed by the Punjab, Sind, Baluchistan, East Bengal,
former Chief Election Commissioner (CEC), North-Western Frontier Province and the
B. B. Tandon, to district of Sylhet of erstwhile Assam.
(a) Frame rules and guidelines to stop paid The help of a referendum was taken
news during elections to establish the total geographical area of
(b) Set an upper limit for election Pakistan. The referendum in the North-
expenditures in remote rural areas Western Frontier Province and Sylhet was in
(c) See that guidelines set for government favour of Pakistan.
advertisements by the Supreme Court are Q.1724 Consider the following well-known
followed commissions and the subject they dealt with.
(d) Decide on the suitability of candidates 1. Punchhi A. Illegal Mining
with criminal charges filing nomination Commission
papers 2. Parthasarathi B. Tax Reforms
Solution: (c) Shome Committee
Learning: In 2015, Supreme Court in its 3. Hota Committee C. Civil Services
order had issued Guidelines on Regulating Reforms
Government Advertisements based on 4. Shah Commission: D. Centre-State
recommendations of Prof. N. R. Madhava Relation
Menon Committee. Select the correct answer using the codes below.
In its order, the Supreme Court had (a)  1-A, 2-C, 3-B, 4-D
barred publication of photos of leaders in (b)  1-D, 2-B, 3-C, 4-A
government advertisements except those of (c)  1-D, 2-B, 3-A, 4-C
the President, Prime Minister and the Chief (d)  1-C, 2-A, 3-D, 4-B
Justice of India. Solution: (b)
SC had also directed the Union Justification: Statement 1: It was chaired
Government to constitute a three-member by former Chief Justice of India on in 2007
committee to regulate the issue of public to look into the new issues of Centre-State
advertisements. relations keeping in view the changes that
In 2016, the Supreme Court modified its have taken place in the polity and economy
earlier 2015 order and allowed photographs of India since the Sarkaria Commission.
of Chief Ministers, Governors and ministers Statement 2: It was setup in 2012 for
to be carried in public advertisements. drawing up the final guidelines on General
Anti-Avoidance Rule (GAAR) and mainly
Commissions to bring about tax clarity and address the
concerns of foreign investors.
Q.1723 The boundary commission headed by Statement 3: The Committee examined
Radcliff demarcated the whole gamut of the Civil Service reform
(a) Provincial areas of Bihar and Assam post covering the higher Civil Service. It was
Bengal Partition 1905 to recommend making the Civil Services:
(b) Boundaries between the two Dominions Responsive and citizen-friendly; Transparent;
of India and Pakistan Accountable; and Ethical.

P.492 For Civil Services Preliminary Examination

05-Indian Polity_Q1507-2004.indd 492 8/7/2018 7:50:01 PM


Statement 4: look into the illegal mining objected to by many retired officers. So, (b)
of iron ore and manganese in the country. It is correct.
reported that the illegal mining of Iron and One-Rank-One-Pension (OROP) scheme
Manganese ores in Odisha caused a loss of for central government staffers, para-military
around Rupees sixty thousand crore to state’s as well as armed forces personnel was
exchequer and the state government should announced by the Seventh Pay Commission.
recover this money as soon as possible. So, (a)  is also correct.
Q.1725 The Mandal Commission Report 1980 was Its recommendations are usually followed
related to up by state governments as they hike their
(a) Reservation for other backward classes staff wage structure. However, it is not
(b) Linguistic reorganization of Indian states binding on the states.
(c)  Design of India’s education policy Q.1727 The Second Administrative Reforms
(d) Development of Panchsheel principles Commission did NOT deal specifically with
Solution: (a) (a)  Public Order
Learning: The plan to set up another (b)  Disaster Management
commission was taken by the Morarji Desai (c)  Ethics in Governance
government in 1978 as per the mandate of the (d)  It dealt with all of the above.
under article 340 of the constitution. Solution: (d)
The commission estimated that 54% of Learning: The Second Administrative
the total population (excluding SCs and Reforms Commission (ARC) submitted the
STs), belonging to 3,743 different castes and following reports.
communities, were ‘backward’. • Right to Information: Master Key to Good
In 1980, the commission’s report affirmed Governance (First report)
the affirmative action practice under Indian • Unlocking human capital: Entitlements
law whereby members of lower castes and Governance–a Case Study relating to
(known as Other Backward Classes (OBC), NREGA (Second Report)
Scheduled Castes (SC) and Scheduled Tribes • Crisis Management From Despair to
(ST)) were given exclusive access to a Hope (Third report)
certain portion of government Jobs and slots • Ethics in Governance (Fourth Report)
in public universities, and recommended • Public order (Fifth Report)
changes to these quotas, increasing them by • Local Governance (Sixth Report)
27% to 50%. • Capacity Building for Conflict Resolution
Q.1726 Which of the following statements about the (Seventh Report)
Pay Commissions setup by the Government • Combating Terrorism Protecting by
of India is INCORRECT? Righteousness (Eight Report)
(a) Its recommendations cover the work and • Social Capital–A Shared Destiny (Ninth
pay structure of both civil and military Report)
divisions of the Government of India. • Refurbishing of Personal Administration-
(b) A serving government employee can be Scaling new Heights (Tenth Report)
its member. • Promoting e-governance: The smart way
(c)  It first submits its report to the Finance Forward (Eleventh Report)
Minister who then places it before the • Citizen Centric Administration–The Heart
Cabinet. of Governance (Twelfth Report)
(d) Its recommendations are binding on the • Organisational Structure of Government
State governments, but not on the Central of India (Thirteenth Report)
government. • Strengthening Financial management
Solution: (d) System (Fourteenth Report)
Justification: Secretary, Ministry of • State and District Administration
Petroleum and Natural Gas was a member of (Fifteenth Report)
the 7th Pay Commission. However, this was

Indian Polity Question Bank P.493

05-Indian Polity_Q1507-2004.indd 493 8/7/2018 7:50:01 PM


(86) NATIONAL POLICIES Q.1730 With reference to the National Water Policy
of India, consider the following statements
Q.1728 With reference to National Youth Policy, 1. National Water Policy is formulated by
2012, consider the following statements. the Ministry of Water Resources of
1. It proposes to change the target age group 2. National Water Board is the agency which
from 16–35 years to 13–35 years to prepares plans for the implementation of
accommodate more number of youths water policies
2. The policy divides youths into target 3. Before 2012 water policy was determined
groups including those of transgender, by the NWP 2002
gay and lesbian youth Which of the above is/are correct?
3. The Policy proposes monitorable (a)  1 Only (b)  1 and 2 Only
indicators such as Youth Work Index and (c)  2 and 3 Only (d)  All
Youth Amenities Index among others Solution: (b)
Which of the above statements is/are correct? National Water Policy 2012 governs water
(a)  1 and 2 Only (b)  2 and 3 Only related policies in India.
(c)  1 and 3 Only (d)  All The main emphasis of National Water
Solution: (b) Policy 2012 is to treat water as economic
Shrinking the target age group defining youth good which the ministry claims to promote
from the existing 13-35 years to 16-30 years its conservation and efficient use.[4] This
and inclusion of groups — transgender, gay provision intended for the privatization of
and lesbians along with those infected with water-delivery services is being criticized
HIV/AIDS and Tuberculosis who suffer from from various quarters.[5] The policy also does
social or moral stigma — in a new category away with the priorities for water allocation
are some of the salient features of the draft mentioned in 1987 and 2002 versions of
National Youth Policy (NYP) 2012. the policy. The policy was adopted with a
The NYP, which is a step forward from disapproval from many states.
the earlier such polices formulated in 1988 The other major features are
and 2003 with an aim to empower youth by 1. To ensure access to a minimum quantity
bringing holistic development, will remain of portable water for essential health and
effective till 2022 once implemented by hygiene to all citizens, available within
Ministry of Youth Affairs and Sports. easy reach of the household
Q.1729 With reference to the Comprehensive 2. To curtail subsidy to agricultural
National Sports Policy 2007, consider the electricity users
following statements 3. Setting up of Water Regulatory Authority
1. It aims to address the national goal of 4. To keep aside a portion of the river flow
emerging as a global, yet inclusive, to meet the ecological needs and to
economic power in the near future through ensure that the low and high flow releases
means of sports correspond in time closely to the natural
2. The ‘Sports’ is listed in the Union List flow regime.
under the the Seventh Schedule of the 5. To give statutory powers to Water Users
Constitution of India Associations to maintain the distribution
Which of the above is/are correct? system
(a)  1 Only (b)  2 Only 6. Project benefited families to bear part of
(c) Both (d) None the cost of resettlement & rehabilitation of
Solution: (a) project affected families
Sports is part of the State List. Union 7. To remove the large disparity between
government is trying since twelfth Lok stipulations for water supply in urban areas
Sabha to bring it into the Concurrent List to and in rural areas
formulate a comprehensive sports policy 8. To support a National Water Framework
Law

P.494 For Civil Services Preliminary Examination

05-Indian Polity_Q1507-2004.indd 494 8/7/2018 7:50:01 PM


Q.1731 Consider the following about the National Statement 2: The policy emphasizes on
Data Sharing and Accessibility Policy making available baseline geo-scientific
(NDSAP). data of world standards in the public
1. It aims to facilitate access to government domain, quality research in a public-private
owned data in machine readable form. partnership, special initiatives for search of
2. It envisages proactive and open access to deep-seated and concealed deposits (similar
the data generated by various Government to UNCOVER initiative of Australia), quick
entities. aerogeophysical surveys of the country, and
Which of the above is/are correct? creation of a dedicated geoscience database
(a)  1 only (b)  2 only etc.
(c)  Both 1 and 2 (d)  None Q.1733 Consider the following with reference to
Solution: (c) the National Pharmaceutical Pricing Policy
Justification: The objective of this policy is (NPPP).
to facilitate access to Government of India 1. Pharmaceutical companies cannot
owned shareable data (along with its usage revise the cost of drugs under the policy
information) in machine readable form without permission from the National
through a wide area network all over the Pharmaceutical Pricing Authority.
country in a periodically updatable manner, 2. It is applicable to all drugs on the national
within the framework of various related list on life-saving medicines.
policies, acts and rules of Government of 3. Drug pricing in the policy is fixed based
India, thereby permitting a wider accessibility on a formula.
and usage by public. Which of the above is/are correct?
The Department of Science and Technology (a)  1 and 2 only (b)  2 and 3 only
is serving the nodal functions of coordination (c)  1 and 3 only (d)  All of the above
and monitoring of policy through close Solution: (c)
collaboration with all Central Ministries and The Supreme Court recently directed the
the Department of Electronics and Information government to have a re-look at the drug
Technology by creating data.gov.in through pricing policy to help make life-saving
National Informatics Centre (NIC). medicines affordable for the common man.
Q.1732 The National Mineral Exploration Policy • The government had approved the
(NMEP) aims at National Pharmaceutical Pricing Policy
1. Encouraging private sector exploration on (NPPP) in 2012.
revenue sharing basis with the government • This policy at bringing 348 essential
2. Creation of a dedicated geo-science drugs under price control and also lead
database of national minerals to reduction in prices. With this, the
Which of the above is/are correct? Government would control prices of 348
(a)  1 only (b)  2 only essential drugs.
(c)  Both 1 and 2 (d)  None • The policy debars the companies from
Solution: (c) using the Wholesale Price Index (WPI)
Justification: Statement 1: The Ministry of to increase the prices of the essential
Mines will carry out auctioning of identified medicines on their own each year. Thus,
exploration blocks for exploration by private the companies had to seek approval from
sector on revenue sharing basis in case their the National Pharmaceutical Pricing
exploration leads to auctionable resources. Authority whenever they wanted to
The revenue will be borne by the successful increase the prices of the items covered
bidder of those auctionable blocks. under the Drug Price Control Order.
If the explorer agencies do not discover • It covers only 348 drugs covered under
any auctionable resources, their exploration National list of life saving medicines.
expenditure will be reimbursed on normative
cost basis.

Indian Polity Question Bank P.495

05-Indian Polity_Q1507-2004.indd 495 8/7/2018 7:50:01 PM


(87) MISCELLANEOUS Q.1736 January 26 was specifically chosen as the ‘date
of commencement’ of the Constitution because
Q.1734 India ceased to be a British dominion on (a) It was on this day in 1930 that Purna
(a) The passage of the Indian Independence Swaraj day was celebrated.
Act, 1947 (b) The members of the Constituent
(b)  August 15, 1947 assembly appended their signatures to
(c)  January 26, 1950 the Constitution of India on this date.
(d) Day of swearing in of Independent (c) The Government of India Act 1935 was
India’s first Governor-General enacted on this date, which became the
Solution: (c) bedrock of the Indian constitution.
Justification and Learning: Till the passage (d) The transfer of power from the British
of the Indian Independence Act, 1947, India Crown to native Indian government took
was a dependency (colony) of the British place on this date.
Empire. From August 15, 1947 to January Solution: (d)
26, 1950, India’s political status was that of Learning: It was chosen because of its
a dominion in the British Commonwealth of historical importance. It was on this day in
Nations. 1930 that Purna Swaraj day was celebrated,
India ceased to be a British dominion following the resolution of the Lahore
on January 26, 1950, by declaring herself Session (December 1929) of the INC. With
a sovereign republic. However, Pakistan the commencement of the Constitution, the
continued to be a British Dominion until Indian Independence Act of 1947 and the
1956. Government of India Act of 1935, with all
India also decided to become a member enactments amending or supplementing the
of the commonwealth, however this did not latter Act, were repealed. Option (b) is wrong
affect India’s independence. as it happened on 24th January 1950.
Q.1735 Consider the following statements. Q.1737 Why 15th August was chosen as the
1. The British sovereign appointed the last Independence Date of India?
Governor General of free India. A. It was on this date that Indian Independence
2. The constituent assembly was composed Bill was introduced in the British House
on a British given formula. of Commons and was passed.
3. India joining commonwealth nations after B. Lord Mountbatten chose the date as he
independence meant that India continued considered this date to be lucky.
to be a dominion state of the British. C. It was on this date that the Lahore Session
Choose the correct answer using the codes of the Congress passed the “Purna Swaraj”
below. resolution.
(a)  1 and 2 only (b)  1and 3 only D. On this date, India’s Constituent Assembly
(c)  2 and 3 only (d)  All of the above passed the “Objectives Resolution”
Solution: (a) granting freedom to India.
The Constituent Assembly was constituted Solution: (b)
in November 1946 under the scheme Option A: Indian Independence Bill was
formulated by the Cabinet Mission Plan. The introduced in the British House of Commons
total strength of the Constituent Assembly on July 4, 1947 and passed within a fortnight.
was to be 389. Of these, 296 seats were to Option B: The date was chosen by
be allotted to British India and 93 seats to the Lord Mountbatten himself because he had
Princely States. Out of 296 seats allotted to considered this date to be lucky. It was on
the British India, 292 members were to be this day during the World War II, that the
drawn from the eleven governors’ provinces Japanese Army surrendered to the allies.
and four from the four chief commissioners’ Option C: In 1929, when Jawaharlal
provinces, one from each. Nehru as Congress President gave the call
for ‘Purrna Swaraj’ or total independence

P.496 For Civil Services Preliminary Examination

05-Indian Polity_Q1507-2004.indd 496 8/7/2018 7:50:01 PM


from British colonial rule, January 26 was criticism of these privileges since integration
chosen as the Independence Day. Later it was and consolidation was the primary aim.
declared as the Republic Day. • Yet, hereditary privileges were not
Q.1738 The Government of India is celebrating 21 consonant with the principles of equality
April every year as ‘Civil Services Day’. The and social and economic justice laid down
21st of April has been chosen as date because in the Constitution of India. Nehru had
(a)  Sardar Vallabhbhai Patel addressed expressed his dissatisfaction over the
the first batch of Indian Administrative matter time and again.
Services (IAS) officers on this date. • Following the 1967 elections, Indira
(b) Indian Covenanted Civil Services were Gandhi supported the demand that the
created by the British on this date. government should abolish privy purses.
(c)  First batch of Civil servants of Morarji Desai, however, called the move
Independent India graduated on this date. morally wrong and amounting to a ‘breach
(d) Pandit Nehru chose this date based on of faith with the princes’.
the first International meeting of Civil • The government tried to bring a
Servants. Constitutional amendment in 1970, but
Solution: (a) it was not passed in Rajya Sabha. It then
Learning: On the occasion, civil servants issued an ordinance which was struck down
are awarded by the Prime Minister for their by the Supreme Court. Indira Gandhi made
excellent work done in the field of public this into a major election issue in 1971 and
administration. got a lot of public support.
It is celebrated as an occasion for the civil • Following its massive victory in the 1971
servants to rededicate themselves to the cause election, the Constitution was amended
of citizens and renew their commitment to to remove legal obstacles for abolition of
public service and excellence in work. ‘privy purse’
First such function was held in 2006. Q.1740 Which of these best characterizes a “nation-
Q.1739 Abolition of privy purse was a major issue state”?
in post-independent India. This privy purse (a)  A nation with fascist authority
allowed (b)  Any political entity
(a)  British government to appropriate a (c) A legal or political entity which is also a
fraction of India’s revenues for certain cultural and social entity
years after independence to maintain (d) Anybody that has a constitution
financial continuity governing it
(b)  Local administration to abolish Solution: (c)
zamindari from erstwhile permanent Justification: Option A: Nation-states can be
settlement regions fascists or democratic or monarchical; there
(c)  Government to confiscate private land is no set-rule.
based on the principle of ‘eminent domain’ Option B: Even a political party of its
(d)  Princely state rulers to retain certain affiliates are political entities. So, B is wrong.
private property and grant in heredity Option C: A nation-state is a mix of
Solution: (d) several ethnicities.
Learning: The integration was preceded by an • A nation refers only to a socio-cultural
assurance that after the dissolution of princely entity, a union of people sharing who can
rule, the then rulers’ families would be allowed identify culturally and linguistically. This
to retain certain private property, and given a concept does not necessarily consider
grant in heredity or government allowance, formal political unions.
measured on the basis of the extent, revenue • A state as referred to a legal/political
and potential of the merging state. entity that is comprised of the following:
This grant was called the Privy Purse. • a permanent population
At the time of accession, there was little • a defined territory

Indian Polity Question Bank P.497

05-Indian Polity_Q1507-2004.indd 497 8/7/2018 7:50:01 PM



• a government government, but also seeks to restrain these

• the capacity to enter into relations with powers. Constitutionalism envisages checks
other states. and balances and puts power of legislature
The combination of both legal and political and executive under some restrain, otherwise
entity is called a ‘Nation-state’. freedom of people would be jeopardized,
Q.1741 What differentiates a ‘nation-state’ from a leading an authoritarian, oppressive
‘state’? government. Therefore, to preserve basic
(a) A nation-state does not have a centralized freedom of individual and to maintain the
authority as a state. dignity and personality, a Constitution should
(b) A state is a mix of several ethnicities and be permeated with Constitutionalism; that is,
communities unlike a nation-state. it should have in-built restrictions on powers
(c) A nation-state does not have a constitution Q.1743 ‘Balkanization’ is a term that is often used in
unlike a state. political discourse. It means
(d) State is only a legal/political entity, (a) Fragmentation of a state into smaller
whereas nation-state is a legal/political units that can be hostile with one another
entity as well as a cultural/social entity (b) Excessive centralization of authority
Solution: (d) away from satellite states of a nation
Justification: USSR was a nation-state. (c) A stage in the development of certain
Authority was centralized in it. So, the option feudal states
(a) is wrong. (d) Segregation within a nation that may
A nation-state is a mix of several ethnicities, emerge from economic displacements
a state need NOT be. So, the option (b) is Solution: (a)
wrong. Justification: The option (d) is known as
Both nation-states (e.g. USA, Canada) ‘social or national polarization’ and is a
and states (e.g. India) can have constitution. general term. Hence not appropriate here.
So, the option (c) is also wrong. Option (c) is known as feudal
Q.1742 Which of the following statements are fragmentation, hence incorrect.
correct? Option (b) may be a cause of balkanization,
1. Constitutionalism implies Governance by but is not known as balkanization. Hence, it
Law. is incorrect.
2. Constitutionalism implies only liberalism. Leaning: The term refers to the division
3. Constitutionalism implies protection of of the Balkan peninsula, formerly ruled
individual liberty. almost entirely by the Ottoman Empire, into
4. Constitutionalism implies Government by a number of smaller states between 1817 and
enlightened people. 1912. In India, the term is often employed
Choose the correct answer from the codes to discuss the implications of extreme
given below regionalism in the NE and some other parts
(a)  1, 2 and 3 (b)  1 and 3 of the country.
(c)  1, 2, 3 and 4 (d)  2, 3 and 4 Q.1744 Which of the following does NOT correctly
Solution: (b) differentiate between the terms ‘State’ and
Constitutionalism means authority of ‘Government’?
government derives from and is limited by a (a) Government can change with elections,
body of fundamental law. In simple words State does not.
it is willingness to obey the Constitution. (b)  State is one part of the government.
A country may have a Constitution, but not (c) Government represents the body of
necessarily ‘constitutionalism.’ For example, persons who implements the will of the
a country where dictator’s word is law state.
can be said to have a constitution, but not (d) None of the above correctly differentiates
constitutionalism. A Constitution does not between state and government.
merely confer powers to various organs of the Solution: (b)

P.498 For Civil Services Preliminary Examination

05-Indian Polity_Q1507-2004.indd 498 8/7/2018 7:50:01 PM


Explanation and Learning: When we use decisions and run the government. The
the word State, it is different from the term monarch may have a small group of people
‘government’. to discuss matters with but the final decision-
A ‘Government’ is responsible for making power remains with the monarch.
administering and enforcing laws. The Unlike in a democracy, kings and queens do
government can change with elections. For not have to explain their actions or defend the
example, the NDA/UPA governments. decisions they take.
The State on the other hand refers to Q.1747 The concept of ‘Ram Rajya’ as given by
a political institution that represents a Mahatma Gandhi was
sovereign people who occupy a definite 1. A society governed by a non-secular
territory. We can, thus, speak of the Indian Hindu constitution
State, the Nepali State etc. 2. A centralized government that promotes a
The Indian State has a democratic form balance of socialism and capitalism
of government. The government (or the 3. A stateless society where every individual
executive) is one part of the State. The State is powerful
refers to more than just the government and 4. An open economy that trades intensively
cannot be used interchangeably with it. with the outside world
Q.1745 The term ‘Law’ used in the phrase “Rule of Select the correct answer using the codes
law’ refers to below.
(a)  Positive law (a)  1, 2 and 4 only (b)  2 only
(b)  Natural law (c)  3 only (d)  1 and 4 only
(c)  Conventions of the Constitution Solution: (c)
(d)  Common law Learning: As per Gandhi, Ram-Rajya
Solution: (a) would be a state less society consisting of
‘Rule of Law’ is derived from the French self-sufiicing, self-sufficing, self-governing
phrase ‘La Principe de Legality’ (the principle village communities.
of legality) which refers to a government • In fact, Gandhi s ideal state was a non-
based on principles of law and not of men. In violent democratic state where social life
a broader sense Rule of Law means that Law would remain self-regulated.
is supreme and is above every individual. • In Ram-Rajya everyone is his own ruler.
Here laws are the one formed by the society Manual work gives an opportunity to all
as whole. Hence answer is positive law. who wish to take part in the government
Positive laws are human-made laws that and the well-being of the state.
oblige or specify an action. It also describes • Here, the powers are to be decentralized
the establishment of specific rights for an and equality is to prevail in every sphere
individual or group. of life.
Q.1746 In a monarchical form of government, the • Every individual is to be given fullest
final decision vests with freedom to devote himself to social
(a)  The Monarch service according to his capacity.
(b)  The elected Council of Ministers • Gandhi was opposed to heavy transport,
(c)  The elected Parliament courts, lawyers, the modem system of
(d) It varies from case to case, but in matters medicine and big cities.
of national importance, monarch’s • According to Gandhi, self-control, self-
opinion reigns. discipline, self-awareness and self-respect
Solution: (a) are the true marks of Ram-Rajya.
Explanation and Learning: In a democracy Q.1748 A ‘Welfare state’ is a State that
the elected government has to explain its 1. Cannot be run wholly on the principles of
actions and defend its decisions to the people. capitalism
However, in a monarchy, the monarch 2. The public responsibility in the state
(king or queen) has the power to make applies for those who are unable to avail

Indian Polity Question Bank P.499

05-Indian Polity_Q1507-2004.indd 499 8/7/2018 7:50:01 PM


themselves of the minimal provisions for (a)  I and II (b)  I, II and IV
a good life (c)  II and IV (d)  I, II, III and IV
3. Is necessarily democratic. Solution: (d)
Select the correct answer using the codes Q.1752 ‘Bicameralism’ can exist in which of the
below: following systems of governance?
(a)  1 and 2 only (b)  2 and 3 only (a)  Federal System
(c)  1 and 3 only (d)  All of the above (b)  Parliamentary System
Solution: (a) (c)  Unitary Government
A welfare state is a concept of government (d)  All of the above
in which the state plays a key role in the Solution: (d)
protection and promotion of the economic Justification: Bicameralism is the practice
and social well-being of its citizens. It of having two legislative or parliamentary
is based on the principles of equality of chambers.
opportunity, equitable distribution of 1. Some countries, such as, Argentina,
wealth, and public responsibility for those Australia, Austria, Belgium, Bosnia and
unable to avail themselves of the minimal Herzegovina, Brazil, Canada, Germany,
provisions for a good life. The general term India, Malaysia, Mexico, Pakistan, Russia,
may cover a variety of forms of economic Switzerland, Nigeria, and the United
and social organization. The sociologist States, link their bicameral systems to
T.H. Marshall identified the modern their federal political structure.
welfare state as a distinctive combination 2. Many bicameral countries like the
of democracy, welfare, and capitalism Netherlands, the Philippines, the Czech
(not fully). Even though Kautilya’s and Republic, the Republic of Ireland and
Ashoka’s state were not democratic, they Romania are examples of bicameral
were welfare states. systems existing in unitary states.
Q.1749 The Welfare State is essentially a synthesis of 3. In many countries, the upper house generally
(a)  Liberalism and Socialism focuses on scrutinizing and possibly vetoing
(b)  Socialism and Communism the decisions of the lower house.
(c)  Communism and Idealism 4. In a few countries, bicameralism involves
(d)  Individualism and Fascism the juxtaposition of democratic and
Solution: (a) aristocratic elements. Hence, all options
Refer previous explanation. are correct.
Q.1750 Which one of the following statement is Power sharing is the very spirit of democracy.
correct? In this context, which of the following
A welfare state envisages statements are correct?
(a)  a fully egalitarian society 1. Power sharing increases conflict among
(b) state ownership of the means of different communities
production 2. Power sharing increases the possibility of
(c)  abolition of private property arbitrariness
(d) a system which combines right to 3. Power sharing delays decision making
personal property with state intervention process
for social security and providing help to 4 Power sharing increases instability
the needy and indigent and divisiveness
Solution: (d) 5. Power sharing undermines the unity of a
Refer previous explanation. country
Select the correct answer using the codes
Q.1751 The main sources of law in India are
below
1. The Constitution
(a)  1, 2 and 5 Only (b)  2 and 3 Only
2. Statutes
(c)  2, 3 and 4 Only (d)  None
3. Customary law
Solution: (d)
4. Judicial decisions of superior courts

P.500 For Civil Services Preliminary Examination

05-Indian Polity_Q1507-2004.indd 500 8/7/2018 7:50:01 PM


Q.1753 Power is shared among different organs of belongs to me is the right to an honourable
government. Such a separation of powers livelihood, no better than that enjoyed by
ensures that none of the organs can exercise millions of others. The rest of my wealth
unlimited power. Which the following belongs to the community and must be used
system of power sharing is called checks and for the welfare of the community.”  Gandhi
balances along with his followers, after their release
1. Horizontal division of powers from prison, formulated a “simple” and a
2. Separation of powers “practical” formula where the concept of
3. Vertical division of powers Trusteeship was explained.
4. Federal division of powers Q.1755 Which of the following can be associated
Which of the above answers is/are correct? with ‘Fabianism’?
(a)  1 Only (b)  1 and 3 1. Value is the creation of society and not of
(c)  1 and 4 (d)  4 Only labour
Solution: (a) 2. Land should cease to be personal property
Power is shared among different organs of of the landlord
government, such as the legislature, executive 3. The changes in the society should be
and judiciary. This horizontal distribution brought by revolution
of power because it allows different organs 4. The key industries may be owned
of government placed at the same level to privately
exercise different powers. Such a separation Select the correct answer from the codes
ensures that none of the organs can exercise given below:
unlimited power. Each organ checks the (a)  1 and 2 (b)  2 and 3
others. This results in a balance of power (c)  2 and 4 (d)  1, 2, 3 and 4
among various institutions. Solution: (a)
Q.1754 Which one of the following statements about ‘Fabianism’ believes in gradual changes in
Trusteeship’s is not correct? society not the sudden changes.
(a) ‘Trusteeship’ provides a means of Q.1756 Which one of the following reflects the true
transforming the capitalist order of nature of the Fascist State as propounded by
society into an egalitarian one its theorist?
(b) It is based on the faith that human nature (a) The Fascist State is an organization
is never beyond redemption with purely material aims, including
(c) It does not recognize any right of private the creation of peaceful conditions of
ownership of property, except so far as it national life
may be permitted by society for its own (b) The Fascist State is a night watchman
welfare guaranteeing personal safety to its
(d) It excludes legislative regulation of the citizens
ownership and use of wealth. (c) The Fascist State is a purely political
Solution: (d) creation, concerned with the moral and
Trusteeship is a socio-economic philosophy material interests of man
that was propounded by Gandhiji. It provides (d) The Fascist State is a spiritual and moral
a means by which the wealthy people would entity, absolute in nature.
be the trustees of trusts that looked after the Solution: (d)
welfare of the people in general. Gandhi Fascism is based on glorification of the state
believed that the rich people could be and the total subordination of the individual
persuaded to part with their wealth to help to it. The state is defined as an organic whole
the poor. Putting it in Gandhiji’s words, into which individuals must be absorbed
“Supposing I have come by a fair amount of for their own and the state’s benefit. This
wealth–either by way of legacy, or by means “total state” is absolute in its methods and
of trade and industry – I must know that unlimited by law in its control and direction
all that wealth does not belong to me; what of its citizens.

Indian Polity Question Bank P.501

05-Indian Polity_Q1507-2004.indd 501 8/7/2018 7:50:01 PM


Q.1757 In India, the Inner Line Permit (ILP) system Q.1759 Consider the following countries which
is operational in practice Universal Adult Franchise.
(a)  All states with International borders 1. New Zealand
(b) All designated Scheduled Areas under 2. India
the Constitution 3. Britain
(c)  All North-Eastern States 4. United States of America
(d)  None of the above 5. Sri Lanka
Solution: (d) Choose the correct chronological order in
Justification: This permit is presently which Universal Adult Franchise was granted
operational in only three north-eastern in these nations.
states viz. Arunachal Pradesh, Nagaland and (a) 13245 (b) 21345
Mizoram. So, D is the answer. (c) 13524 (d) 13452
Manipur was earlier regulated by Inner Solution: (c)
Line permit system (ILP), which was later
abolished. The system of ILP is a special pass The years when the Universal Adult
or permit or quasi visa that is required by Franchise was granted in the countries
Indian citizens to enter protected/restricted given below.
area of state for a limited period. 1893 New Zealand
It was introduced by then British India 1917 Russia
government to protect indigenous tribal people
from encroachment into their areas by outsiders. 1918 Germany
Later, they used it as an instrument to 1919 Netherlands
advance the commercial interest. 1928 Britain
Q.1758 Which of the following have a bearing on the 1931 Sri Lanka
governance of North-eastern States?
1. Part IX of the Constitution 1934 Turkey
2. Part IXA of the Constitution 1944 France
3. Sixth Schedule 1945 Japan
Select the correct answer using the codes
below. 1950 India
(a)  1 and 2 only (b)  3 only 1951 Argentina
(c)  1 and 3 only (d)  1, 2 and 3 1952 Greece
Solution: (d)
1955 Malaysia
Justification: The Status of North Eastern
States with regard to Self-Governing 1962 Australia
Institution can be seen under Parts IX and IXA 1965 US
of the Constitution and Special Constitutional
1978 Spain
Provisions (Article 371 series).
• Part IX of the Constitution deals with the 1994 South Africa
Panchayats that can be setup in these areas.
• Part IXA of the Constitution deals with Q.1760 Which of the following correctly
the Municipalities. differentiates between “Demand Polity” and
• Sixth Schedule of the Constitution “Command Polity”, terms coined by Rudolph
contains special provisions as to the and Rudolph (1987)?
Administration of Tribal Areas in the (a) Former refers to a democratic system;
States of Assam, Meghalaya, Tripura and latter refers to a despotic system.
Mizoram. (b) Former system concerns short-term
• It also envisages an autonomous District populist gains, whereas latter concerns
Council for the tribal areas in these states. long-term growth for public welfare

P.502 For Civil Services Preliminary Examination

05-Indian Polity_Q1507-2004.indd 502 8/7/2018 7:50:01 PM


(c) Former system is based on popular • Public interest: Refers to the collective
sovereignty; latter is based on charismatic common good within society, to which is
rule. the main objective of public interest.
(d) Former system stresses on governance • Public policy: The means by which new
rather than just rules; latter emphasizes public policy is enacted, and introduced.
on rules rather than governance Not limited in participation of the public
Solution: (b) but encouraged involvement.
Justification: Rudolph and Rudolph have • Services to citizens: Providing and
attempted to find corre­lates between the upholding the moral and ethical standard
nature of polity of the state and economic in regards to meeting the needs of citizens
characteristics. through institutions and bureaucracies.
In a democratic country, there are two When this movement swept public
competing sovereignties: state sovereignty administration, people started seeing
and voter/citizen sovereignty. administration as a change maker rather than
The demand polity refers to the situation merely a bureaucratic tool.
when the government’s policies and Q.1762 Politics–Administration dichotomy implies
programmes are for the short-term benefits that
of people. (a) Bureaucracy should be politically active
The allocations and welfare activities are and allowed to participate in elections.
carried out keeping elections in mind. (b) Policy making and policy execution are
The model of command polity refers to separate activities.
that political set-up in which allocations and (c) Government should not venture out in
distributions are oriented to the long-term provision of public goods.
goals and sustainable benefits to people. (d) A welfare state cannot provide good
• The role of the state in command politics governance.
is monopolistic and oligopolistic. Solution: (b)
The command politics relates to the Justification: The politics-administration
postponement theory, which believes dichotomy is an important concept in the field
in investment for ensuring future of public administration because it deals with
development at the cost of the present the policy-makers role as an administrator
needs and necessities of the people. and the balancing act that is the relationship
• In the existing world, both of these polities between politics and administration.
are found in a mixed form to varying There are two views on this concept.
degrees. • Those who support it say that politics and
Q.1761 The Philosophy of ‘New Public administration should be distinct, claim
Administration’ vouches that it will ensure an efficient, effective
(a) privatization of all government activities and neutral bureaucracy. A politically
(b)  the state of Laissez faire motivated bureaucracy will not augur well
(c) inculcation of values and equity in for administration. So, option A is wrong.
administration However, the extent of such distinction
(d) outsourcing of major governmental had not been clearly stated.
activities • The other group who argue that the two
Solution: (c) disciplines should not be separated rest
Justification and Learning: New Public their thesis on the interconnection between
Administration theory deals with the politics and administration. To them, politics
following issues: and administration are complementary.
• Democratic citizenship: Refers directly to Q.1763 New Public Management philosophy stands
the belief in creating a government where for
the “common man” has a voice in politics. 1. Complete privatization of commercial
government undertakings

Indian Polity Question Bank P.503

05-Indian Polity_Q1507-2004.indd 503 8/7/2018 7:50:01 PM


2. Decentralizing working units • To move the focus of the department from
3. Establishing short-term labour contracts process-orientation to result-orientation and
instead of long-term agreements • To provide an objective and fair basis
Select the correct answer using the codes to evaluate department’s overall
below. performance at the end of the year.
(a)  1 and 2 only (b)  2 and 3 only This document contains not only the agreed
(c)  1 and 3 only (d)  1, 2 and 3 objectives, policies, programs and projects
Solution: (b) but also success indicators and targets to
Justification: Statement 1: The New Public measure progress in implementing them.
Management (NPM) emphasizes the concept Q.1765 ‘Social audit’ is increasingly being seen as an
that ideas used in the private sector must important tool to improve accountability in
be successful in the public sector. But, it governance. Which of these can be assigned
does not advocate complete privatization of the responsibility for the social audit of a
governmental functions. government scheme?
Statement 2: This point suggest that it (a)  Comptroller General of India only
is more appropriate to shift from a unified (b) Gram Sabha or Gram Panchayat only
management system to a decentralized (c) A community group or professional
system in which managers gain flexibility institution
and are not limited to agency restrictions. (d)  Independent Evaluation Office (IEO)
Statement 3: It focuses on the necessity to only
establish short-term labour contracts, develop Solution: (c)
corporate plans, performance agreements and Learning: Social audit means the process
mission statements. in which people collectively monitor and
Q.1764 Consider the following statements about the evaluate the planning and implementation of
Results Framework Document (RFD). a programme or scheme.
1. Under the Performance Monitoring • For example, a social audit was conducted
and Evaluation System (PMES) each by Society for Social Audit Accountability
department is required to prepare an and Transparency (SSAAT) in two
RFD. districts, viz. Khammam and Chittoor of
2. An RFD provides a summary of the Andhra Pradesh during 2012-2013.
most important results that a department/ • Encouraged by the outcome , the Ministry
ministry expects to achieve during the of Human Resource Development
financial year. (MHRD) has issued detailed guidelines
Which of the above is/are correct? for conducting of social audit under ‘Mid-
(a)  1 only (b)  2 only Day Meal’ Scheme.
(c)  Both 1 and 2 (d)  None • The states/UTs were advised to select
Solution: (c) the eminent institute available in their
Justification: Statement 1: The Prime respective states for this purpose.
Minister approved the outline of a • Similarly, the Gram Sabha is entitled for the
“Performance Monitoring and Evaluation social audit of fund usage under Mahatma
System (PMES) for Government Gandhi National Rural Employment
Departments” in 2009. Guarantee Act  (MGNREGA) as a
A Results-Framework Document (RFD) mandatory provision.
is essentially a record of understanding • Therefore, social audit is comprehensive
between a Minister representing the people’s and involves stakeholders other than
mandate, and the Secretary of a Department government.
responsible for implementing this mandate. Q.1766 Which of the following has/have a bearing on
Statement 2: This document has two main Manual Scavenging in India?
purposes. 1. Articles 17 and 23 of the Constitution
2. Protection of Civil Rights Act, 1955

P.504 For Civil Services Preliminary Examination

05-Indian Polity_Q1507-2004.indd 504 8/7/2018 7:50:01 PM


3. Protection of Human Rights Act, 1993 are exposed to subhuman conditions of work
4. Eleventh Schedule and face serious health hazards. They are
5. Twelfth Schedule constantly exposed to infections that affect
Select the correct answer using the codes their eyes, skin, respiratory and gastro-
below. intestinal systems. They get very low wages
(a)  4 and 5 only (b)  1, 3, 4 and 5 only for the work they perform.
(c)  1 and 3 only (d)  All of the above Q.1767 Which of the following statements is
Solution: (d) INCORRECT?
Explanation: Article 17 provides for (a) Universal adult franchise ensures
the abolition of untouchablity. Manual political equality.
scavenging is a major reason behind (b) Fundamental rights ensure opportunities
untouchability. Article 23 provides for for economic non-discrimination.
“Prohibition of traffic in human beings (c) Directive Principles of State Policy
and forced labour”. Manual scavenging is provide moral guidance to the state.
considered as forced labour. So, statement 1 (d) Fundamental Duties ensure that local
is correct. self-government runs smoothly in
Protection of Civil Rights Act, 1955; villages and towns.
Protection of Human Rights Act,1993; Solution: (d)
Schedule Castes and the Schedule Tribes Justification: Option A: One man, one vote,
Prevention of Atrocities Act, 1989; and one value is the principle behind this.
Schedule Castes and the Schedule Tribes Option B: For e.g. in ordinary cases, no
Prevention of Atrocities Rules, 1995, consider one can be discriminated in matters of public
practices such as manual scavenging to be a employment based on caste, creed or religion
violation of human rights and have provisions etc.
to deal with the same. So, statement 2 and 3 Option C: For example, the Directive
is also correct. Principles of State Policies (DPSP) ensure
Statements 4 and 5 are also correct as that all should have the right to work; at
• ELEVENTH SCHEDULE [ARTICLE living wages; income inequality should be
243G], minimized; women should get equal wages
1.  Entry 23: Health and sanitation, for equal work as men do etc.
including hospitals, primary health Option D: There is no mention of the
centres and dispensaries. term self-government or Panchayati Raj
2.  Entry 27: Welfare of the weaker Institutions (PRIs) in the Fundamental Duties
sections and in particular of the (FDs), they are a moral compass mainly for
Scheduled Castes and the Scheduled the citizens.
Tribes.
Q.1768 Consider the following statements with
• TWELFTH SCHEDULE [Article 243W]
regard to the Executives of India:
1. Entry 6: Public health and conservancy
1. Functionaries who take day-to-day
sanitation and solid waste management.
decisions but do not exercise supreme
Learning: Manual scavenging refers to the
power on behalf of the people are
practice of removing human and animal
collectively known as the executive.
waste/excreta using brooms, tin plates and
2. The temporary executives have lesser
baskets from dry latrines and carrying it on
power than the permanent executives.
the head to disposal grounds some distance
3. Secretary of Ministry of State and Director
away. There are 13 lakh persons from Dalit
General of Police are part of temporary
communities who continue to be employed
Executive.
in this job in this country and who work
4. The temporary executives are experts in
in the 96 lakh private and community dry
the matters of their ministry.
latrines managed by municipalities. They
Which of the statements is/are correct?
Solution: (a)

Indian Polity Question Bank P.505

05-Indian Polity_Q1507-2004.indd 505 8/7/2018 7:50:01 PM


At different levels of any government we find Article 32 and 226 give judiciary the
functionaries who take day-to-day decisions power to correct excesses that have led to
but do not exercise supreme power on behalf the violation of the fundamental rights of a
of the people. All those functionaries are citizen by the state. So, 2 is correct.
collectively known as the executive. Thus, Tribunals are established under Article
when we talk about ‘the government’, we 323 of the Constitution for granting relief
usually mean the executive. In a democratic and settling disputes regarding taxation,
country, two categories make up the executive. competitive practices, service matters etc.
That the first one which is elected by the people So, the statement 3 is correct.
for a specific period, is called the political Q.1770 Which of the following institutions can make
executive (temporary executives). Political changes to an existing law of the country?
leaders who take the big decisions fall in this 1. The Supreme Court
category. In the second category, people are 2. The President
appointed on a long-term basis. This is called 3. The Prime Minister
the permanent executive or civil services.They Choose the correct answer using the codes
remain in office even when the ruling party below:
changes.These officers work under political (a)  None of the above (b)  1 and 2 only
executive and assist them in carrying out the (c)  1 only (d)  3 only
day-to-day administration. Political executive Solution: (a)
have more power than the non-political The SC can declare a law void, not change it.
executive In a democracy, the will of the The Parliament can only change an existing
people is regarded as supreme. The minister is law, not the President or Prime Minister. The
elected by the people and thus empowered to President can utmost delay the changing of
exercise the will of the people on their behalf. the law or resend it to the Parliament for
She is finally answerable to the people for reconsideration.
all the consequences of her decision. That is
Q.1771 Which of the following have a bearing on
why the minister takes all the final decisions.
the decisions taken or orders given by the
The minister decides the overall framework
Central Government?
and objectives in which decisions on policy
1. Supreme Court
should be made. The minister is not, and is
2. High Courts
not expected to be, an expert in the matters of
3. Cabinet Secretary
her ministry. The minister takes the advice of
Choose the correct answer using the codes
experts on all technical matters.
below:
Q.1769 What is/are the remedies available to a (a)  1 and 2 only (b)  2 and 3 only
common citizen against the excesses of the (c)  1 and 3 only (d)  All of the above
executive branch of Government? Solution: (d)
1. Get the executive order declared void A decision is taken by the Central cabinet.
after a Judicial Review The Cabinet Secretary is present in all the
2. Appeal to the High Court under Article meetings of the Cabinet and advises and
226 of the Constitution assists the cabinet in making decisions.
3. Approach tribunals made for the specific After a decision is taken, the Supreme
purpose Court or High Courts have the power to
Select the correct answer using the codes reverse or repeal and order or decision of the
below. government if it is illegal, unconstitutional or
(a)  1and 2 only (b)  1 and 3 only mala fide or arbitrary.
(c)  2 only (d)  1, 2 and 3
Q.1772 Consider the following statements.
Solution: (d)
1. The Central and state governments are
Justification: Article 13 gives Judiciary
empowered to prohibit employment
the power to review legislative acts and
of contract labour in any activity in an
administrative rules. So, 1 is correct.
establishment.

P.506 For Civil Services Preliminary Examination

05-Indian Polity_Q1507-2004.indd 506 8/7/2018 7:50:02 PM


2. Only the State governments are process of law”. So, the option (d) will be the
empowered to fix and revise minimum correct answer.
wages for labour in various industries. Q.1774 Apart from the Supreme Court of India,
Which of the above is/are correct? which of the following is/are authorities
(a)  1 only (b)  2 only empowered to interpret the constitution?
(c)  Both 1 and 2 (d)  None 1. Chief Justice, High Court
Solution: (a) 2. Speaker, Lok Sabha
Justification: Statement 1: As per the 3. Chairman, Rajya Sabha
Contract Labour (Regulation and Abolition) Choose the correct answer using the codes
Act, 1970: below.
• The Central and state governments are (a)  1 and 2 only (b)  2 and 3 only
empowered to prohibit employment (c)  1 only (d)  All of the above
of contract labour in any activity in an Solution: (d)
establishment. Speaker is the final interpreter of the
• The Act contains specific provisions to provisions of (a) the Constitution of India,
ensure payment of wages and certain (b) the Rules of Procedure and Conduct
amenities to be provided by the contractor of Business of Lok Sabha and (c) the
to the contract labour. parliamentary precedents, within the House.
Statement 2: Under The Minimum Wages Powers of Chairman, Rajya Sabha are the
Act, 1948 both Central and state governments same. Moreover, they derive their powers
are “Appropriate Governments” for fixation/ and duties from three sources, that is, the
revision of minimum rates of wages for Constitution of India, the Rules of Procedure
employments covered by the Schedule to the and Conduct of Business of Lok Sabha
Act. and Parliamentary Conventions (residuary
powers that are unwritten or unspecified in
Constitutional Provisions the Rules).
Q.1773 Which of the following terms has/have NOT Q.1775 As per the Constitution, the Union executive
been defined in the constitution? does NOT include
(a)  Union Territory (a)  President of India
(b) Securities (b)  Vice President of India
(c)  Agricultural Income (c)  Council of Ministers
(d)  Procedure Established by Law (d)  Advocate general of India
Solution: (d) Solution: (d)
Justification: Article 366 contains the Justification: It should be Attorney General
definitions of various terms used in different of India. Advocate General is a part of the
provisions of the constitution. These are Stat executive. So, the answer is (d).
mentioned below: The Articles 52 to 78 in Part V of the
1. Agricultural Income means agricultural Constitution deal with the Union Executives.
income as defined for the purposes of the The President is the head of the Indian
enactments relating to Indian income-tax. State. He is the first citizen of India and acts
2. Securities include stock. as the symbol of unity, integrity and solidarity
3. Union Territory means any Union territory of the nation.
specified in the First Schedule and includes Q.1776 The Union Executives consist of the
any other territory comprised within the 1. President
territory of India but not specified in that 2. Vice-President
Schedule. 3. Prime Minister
Since the term “procedure established by 4. Council of Ministers
law” is not defined, the courts have taken 5. Attorney-General of India
enough liberty to interpret it to mean “due

Indian Polity Question Bank P.507

05-Indian Polity_Q1507-2004.indd 507 8/7/2018 7:50:02 PM


6. Central Administrative Tribunal Justification: Statement 1: Two Anglo-
7. Central Vigilance Commission Indian members can be nominated to the
Select the correct answer using the codes Lok Sabha, if they are not being adequately
below. represented.
(a)  1, 2, 3, 4 and 5 only Statement 2: In some states Governor may
(b)  1, 2, 5, 6 and 7 only appoint 1 member to represent minorities,
(c)  3 and 4 only e.g. the Anglo-Indian community, if he finds
(d)  1, 2, 3, 4, 5, 6 and 7 that minority inadequately represented in the
Solution: (a) House.
Justification: Statement 6: The Central Statement 3: The president nominates 12
Administrative Tribunal (CAT) is the tribunal members to the Rajya Sabha from people
wielding judicial powers and cannot be said who have special knowledge or practical
to be a part of the Union executives. experience in art, literature, science and
Statement 7: The Central Vigilance social service.
Commission (CVC) acts as a quasi-judicial Statement 4: One-sixth of its members
body that enquires into cases of corruption in are nominated by the Governor from persons
the Union government. So, it is also not the having knowledge or practical experience in
part of the Union Executives. fields such as literature, science, arts, the co-
The Articles 52-78 (Part V) of the operative movement and social service.
Constitution deal with the Union executive. Q.1779 Which of these bodies is subject to
President heads the Union executive. dissolution?
Q.1777 Consider the following statements about the 1. Lok Sabha
executive branch of government in India. 2. Rajya Sabha
1. It is concerned with implementing laws 3. State Legislative Assembly
made by the Parliament. 4. State Legislative Council
2. It can make laws too for short periods. Choose the correct answer using the codes
3. The whole Council of Ministers is a part below:
of the executive. (a)  1, 2 and 3
Choose the correct answer using the codes (b)  All of the above
below: (c)  1 and 3 only
(a)  1 and 2 (b)  2 and 3 (d)  2 and 4 only
(c)  1 and 3 (d)  All of the above Solution: (c)
Solution: (d) Rajya Sabha and State Legislative Council
The executive can make laws via the (SLC) are permanent chambers. They cannot
ordinance route as mentioned in the Article be dissolved. Members retire every 2 years in
123 of the constitution. Rajya Sabha and SLC.
Q.1778 Which of these bodies has a provision for Q.1780 The presiding officer (Chairman or Speaker)
nomination of certain members? of which of the following is NOT elected by
1. Lok Sabha its members?
2. State Legislative Assembly 1. Rajya Sabha
3. Rajya Sabha 2. Lok Sabha
4. State Legislative Council 3. State Legislative assembly
Select the correct answer using the codes 4. State Legislative Council
below. Choose the correct answer using the codes
(a)  1, 2 and 3 only below:
(b)  1 and 4 only (a)  1, 3 and 4 (b)  Only 1
(c)  3 and 4 only (c)  1 and 4 (d)  None of the above
(d)  1, 2, 3 and 4 only Solution: (b)
Solution: (d) The Vice-President presides over the Raja
Sabha. Rest all Presiding officers are elected.

P.508 For Civil Services Preliminary Examination

05-Indian Polity_Q1507-2004.indd 508 8/7/2018 7:50:02 PM


Q.1781 Which of the following is fixed by the Government, representing the Union of India
Constitution of India? and the State governments. Later, a third
1. Allowances of the President and Governor tier of federalism was added in the form of
2. Maximum Size of Council of Ministers Panchayats and Municipalities.
3. Manner of election of the Sarpanch of the All States in the Indian Union do not have
Gram Panchayat. identical powers. Some States enjoy a special
Choose the correct answer using the codes status. Jammu and Kashmir has its own
below. Constitution. Many provisions of the Indian
(a)  1 and 2 only (b)  2 and 3 only Constitution are not applicable to this State
(c)  1 and 3 only (d)  All of the above without the approval of the State Assembly.
Solution: (a) Indians who are not permanent residents of
• The 2nd schedule of the constitution fixes this State cannot buy land or house here.
the allowances, privileges, emoluments Similar special provisions exist for some
of President, Governor, Chief Justice of other States of India as well. There are some
India (CJI) and other functionaries. It can units of the Indian Union which enjoy very
be revised only after amending this part of little power.
the constitution. These are not subject to These are areas which are too small to
the annual vote of the Parliament. become an independent State but which
• 91st amendment fixed the maximum could not be merged with any of the existing
strength of the Council of Ministers at States. These areas, like Chandigarh, or
15% of the strength of the Lok Sabha. Lakshadweep or the capital city of Delhi, are
• While the manner of election of the called Union Territories. These territories do
Panchs is fixed by constitution in form not have the powers of a State. The Central
of adult franchise, the state legislature Government has special powers in running
decides on the manner of election of the these areas.
Sarpanch. Q.1783 Which of these are Constitutional Provisions?
Q.1782 Consider the following statements: 1. Duties of prime minister to furnish
1. The Concurrent List contains forest, required information to the president
education and agriculture as its subjects. 2. Power of president to consult Supreme
2. All states in the Indian Union do not have Court
identical powers. 3. Special provisions with respect to Delhi
3. The Union territories do not have the Select the correct answer using the codes
powers of a State below.
4. The Constitution originally provided for a (a)  1 and 2 only (b)  2 only
two-tier system of government, the Union (c)  1 and 3 only (d)  1, 2 and 3
Government and the State Government Solution: (d)
only. Justification: Statement 1: It is dealt by
Which of the statements is/are correct? Article 78.
(a)  1 and 4 (b)  1, 2 and 4 In respect to the furnishing of information to
(c)  2, 3 and 4 (d)  1, 2, 3 and 4 the President, the duties of the Prime Minister
Solution: (c) will be:
The Concurrent List includes subjects (a)  to communicate to the President all
of common interest to both the Union decisions of the council of Ministers
Government as well as the State relating to the administration of the
Governments, such as education, forest, trade affairs of the union and proposals for
unions, marriage, adoption and succession. legislation;
The Constitution originally provided for (b) to furnish such information relating to
a two-tier system of government, the Union the administration of the affairs of the
Government or what we call the Central Union and proposals for legislation as
the President may call for; and

Indian Polity Question Bank P.509

05-Indian Polity_Q1507-2004.indd 509 8/7/2018 7:50:02 PM


(c) To submit for the consideration of the (c)  Both (a) and (b)
Council of Ministers any matter on which (d) None
a decision has been taken by a Minister Solution: (b)
but which has not been considered by the Justification and Learning: It has the
Council if the President requires. relation with Article 246. (see http://mowr.
Statement 2: Article 143: Advisory gov.in), Article 262 (inter-state river water
jurisdiction disputes) and Seventh Schedule (division of
Statement 3: Under article 239 AA, it powers between Centre and States on water
provides for the legislative assembly of resources). In case of disputes relating to
National Capital Territory of Delhi has waters, Article 262 provides:
legislative and executive powers with the 1. Parliament may by law provide for the
exceptions of land, police etc. adjudication of any dispute or complaint
Also, certain categories of Bills require with respect to the use, distribution or
the prior approval of the Central Government control of the waters of, or in, any inter-
for introduction in the legislative assembly. State river or river valley.
Some Bills, passed by the legislative assembly 2. Notwithstanding anything in this
of the Union Territory of Puducherry and Constitution, Parliament may, by law
National Capital Territory of Delhi are provide that neither the Supreme Court
required to be reserved for consideration and nor any other court shall exercise
assent of the President. jurisdiction in respect of any such dispute
Q.1784 The Constitution does NOT contain any or complaint as is referred to in the clause.
specific procedure for the selection and Q.1786 As you flip through the pages of the
appointment of the constitution, you are likely to find specific
1. Chief Minister 2. Governor provisions for the following in which order?
3. Prime Minister 4. President 1. Legislative Powers of the President
Choose the correct answer using the codes 2. Comptroller and Auditor General of India
below. 3. High courts
(a)  1, 2 and 3 (b)  All of the above 4. Citizenship
(c)  1 and 2 only (d)  3 and 4 only Select the correct answer using the codes
Solution: (a) below.
• The Constitution does not contain any (a) 4312 (b) 1234
specific procedure for the selection (c) 4123 (d) 1324
and appointment of the Chief Minister. Solution: (c)
Article 164 only says that the Chief Justification: If you remember some major
Minister shall be appointed by the articles of the Constitution, you can arrive at
governor. However, this does not imply the answer. Or if you know broadly which
that the governor is free to appoint any parts of the Constitution deal with which
one as the Chief Minister. In accordance subjects, you can arrive at the correct answer.
with the convections of the parliamentary Q.1787 Which of the following find mention in the
system of government, the governor has constitution?
to appoint the leader of the majority 1. All Parliamentary Committees
party in the state legislative assembly as 2. Cabinet Committees
the Chief Minister. 3. Committees established by President to
• The same goes for the Prime Minister. report on the condition of SCs and STs
• Governor‘s appointment and selection too Select the correct answer using the codes
is not detailed in the constitution. below.
Q.1785 Water finds a mention in the Constitution of (a)  1 and 2 only (b)  3 only
India in the (c)  2 and 3 only (d)  1, 2 and 3
(a)  Directive Principles of State Policy Solution: (b)
(b)  Seventh Schedule

P.510 For Civil Services Preliminary Examination

05-Indian Polity_Q1507-2004.indd 510 8/7/2018 7:50:02 PM


Justification: Statement 1: They are 3. The Fundamental Duties are known
established based on the rules of Parliament as instrument of instructions in Indian
by the speaker. administration.
Statement 2: These are constituted based 4. The President does not enjoy constitutional
on the Rules of Business of the government of discretion.
India. They are extra-constitutional in nature. Which of the statements given above is/are
Statement 3: The Constitution requires the correct?
president to appoint a commission to report (a)  2 and 3 Only
on the administration of the scheduled areas (b)  1 and 2 Only
and the welfare of the scheduled tribes in the (c)  1, 2 and 4 Only
states. He can appoint such a commission at (d)  1, 2, 3 and 4 Only
any time but compulsorily after ten years of Solution: (c)
the commencement of the Constitution. Only Lok Sabha can initiate, reject or amend
Q.1788 Consider the following: money bills. If it is a money bill, the Rajya
1. Governor has the power to recommend Sabha can either approve the bill or suggest
the dismissal of the State government changes but cannot reject it. If it takes no
and suspension or dissolution of State action within 14 days the bill is deemed to
assembly. have been passed.
2. The Lok Sabha consists of only elected Amendments to the bill, suggested by
representatives. Rajya Sabha, may or may not be accepted
3. The Vice President belongs to the Upper by the Lok Sabha. The Constitutional
House of the Parliament. Amendment Bills can only be ratified i.e., it
4. The 73rd and 74th amendments have cannot be rejected or returned.
created uniformity in the structures The President does not enjoy constitutional
of Panchayati Raj and Nagarpalika discretion. S/he enjoys only situational
institutions across the country. discretion like appointment of Prime
Which of the above statements is/are correct? Minister, dismissal of council of ministers.
(a)  1 and 4 Only (b)  1 and 3 Only Directive Principles of State Policy are
(c)  2 , 3 and 4 Only (d)  1, 2, 3 and 4 known as instrument of instructions in Indian
Solution: (a) administration.
The Governor has the power to recommend Q.1790 Consider the following statements:
the dismissal of the State government and 1. It is the constitutional obligation of the
suspension or dissolution of State assembly. Prime Minister to communicate to the
The Lok Sabha consists of elected President all decisions of the Council of
representatives and also the President can Ministers relating to the administration of
nominate two members to the Lok Sabha the affairs of the Union and proposals for
from the Anglo-Indian Community. legislation.
The Vice President should not be member 2. The Governor has more discretionary
of either House of the Parliament or State powers compared to the President.
Legislature. 3. The writ jurisdiction of the Supreme
The 73rd and 74th amendments have Court is narrower compared to the High
created uniformity in the structures of Court.
Panchayati Raj and Nagarpalika institutions Which of the above statements is/are correct?
across the country. (a)  1 Only (b)  2 and 3Only
Q.1789 Consider the following statements. (c)  1 and 3Only (d)  1, 2 and 3
1. The Rajya Sabha cannot initiate, reject or Solution: (d)
amend the money bills. The Prime Minister acts as a link between
2. The President cannot send back an the Council of Ministers on the one hand
amendment bill for reconsideration of the and the President as well as the Parliament
Parliament. on the other. It is this role of the Prime

Indian Polity Question Bank P.511

05-Indian Polity_Q1507-2004.indd 511 8/7/2018 7:50:02 PM


Minister which led Pt. Nehru to describe him Statement 3: Union Public Service
as ‘the linchpin of Government’. It is also Commission to conduct examinations for
the constitutional obligation of the Prime recruitment to all-India services and higher
Minister to communicate to the President all Central services and to advise the President
decisions of the Council of Ministers relating on disciplinary matters.
to the administration of the affairs of the State Public Service Commissions
Union and proposals for legislation. in every state conduct examinations for
At the State level, a similar parliamentary recruitment to state services and advice the
executive exists, though with some variations. governor on disciplinary matters.
The most important variation is that there Q.1792 Which of these bodies find mention in the
is a Governor of the State appointed by Constitution of India?
the President (on the advice of the Central 1. District Planning Committee and
Government). Though the Chief Minister, Metropolitan Planning Committee
like the Prime Minister is the leader of the 2. Inter-State Trade and Commerce
majority party in the Assembly, the Governor Commission
has more discretionary powers. 3. National Backward Classes Commission
The writ jurisdiction of the Supreme Court 4. Official Language Commission and
is narrower compared to the High Court since Official Language Committee of
writ jurisdiction of High Court involves not Parliament
only Fundamental Rights but also ordinary Select the correct answer using the codes
legal right. below.
Q.1791 Which of these constitutional bodies (a)  1 and 2 only (b)  3 and 4 only
are functionally autonomous from the (c)  1, 2, 3 and 4 (d)  None of the above
government? Solution: (c)
1. Election Commission of India Justification: Statement 2: While there is
2. Comptroller and Auditor-General of India a provision to establish the commission, it
3. State Public Service Commissions has not been established till date. Even the
Select the correct answer using the codes Punchhi commission recommended the
below. same.
(a)  1 only (b)  2 and 3 only Statement 4: As defined in the Article 344
(c)  1 and 3 only (d)  1, 2 and 3 of the Constitution, it shall be the duty of the
Solution: (d) Commission to make recommendations to the
Justification: The Indian Constitution not President as to:
only provides for the legislative, executive • Progressive use of the Hindi language for
and judicial organs of the government the official purposes of the Union;
(Central and state) but also establishes certain • Restrictions on the use of the English
independent bodies. They are envisaged language for all or any of the official
by the Constitution as the bulwarks of the purposes of the Union;
democratic system of Government in India. • Language to be used for all or any of the
Statement 1: Election Commission purposes mentioned in Article 348;
ensures free and fair elections to the • Form of numerals to be used for any one
Parliament, the state legislatures, the office or more specified purposes of the Union;
of President of India and the office of Vice- Learning: These are some of the Articles
president of India. Related to Constitutional Bodies (which are
Statement 2: Comptroller and Auditor- not very popular or widely known)
General of India audits the accounts of the • Art. 165. Advocate-General of the State
Central and state governments. He acts as • Art. 243 I. State Finance Commission
the guardian of public purse and comments • Art. 243 K. State Election Commission
on the legality and propriety of government • Art. 243ZD. District Planning Committee
expenditure.

P.512 For Civil Services Preliminary Examination

05-Indian Polity_Q1507-2004.indd 512 8/7/2018 7:50:02 PM


• Art. 243ZE. Metropolitan Planning On the other hand, the other National
Committee commissions like the National Commission
• Art. 307. Inter-State Trade and Commerce for Women (1992), the National Commission
Commission for Minorities (1993), the National
• Art. 324. Election Commission Commission for Backward Classes (1993),
• Art. 339. Scheduled Areas and Scheduled the National Human Rights Commission
Tribes Commission (1993) and the National Commission for
• Art. 340. Backward Classes Commission Protection of Child Rights (2007) are
• Art. 344. Official Language Commission statutory bodies in the sense that they are
and Official Language Committee of established by acts of the Parliament.
Parliament Q.1795 Which of these is/are Constitutional bodies?
• Art. 350B. Special Officer for Linguistic 1. National Commission for STs
Minorities 2. National Commission for Minorities
Q.1793 Which of the following bodies do NOT find a 3. National Commission for Protection of
mention in the constitution of India? Child Rights
1. Cabinet 4. National Human Rights Commission
2. Parliamentary Committees Choose the correct answer using the codes
3. NITI Ayog below.
4. Prime Minister’s Office (PMO) (a)  1 only (b)  1 and 4 only
Choose the correct answer using the codes (c)  All of the above (d)  2, 3 and 4 only
below. Solution: (a)
(a)  1 and 4 only (b)  2 and 3 only The National Commission for Scheduled
(c)  3 and 4 only (d)  All of the above tribes(STs) is a constitutional body in the
Solution: (c) sense that it is directly established by Article
The NITI Ayog replaces Planning 338A of the Constitution.
Commission as another executive body. It is On the other hand, the other national
not even a statutory body. The Prime Minister commissions like the National Commission
Office is also an executive body set up under for
an order of cabinet. Cabinet however finds Women (1992), the National Commission
mention under the constitution. Also, the for Minorities (1993), the National
constitution mentioned about the important Commission for Backward Classes (1993),
parliamentary committee like PAC. the National Human Rights Commission
Q.1794 Which of these bodies is/are extra- (1993) and the National Commission
constitutional statutory bodies? for Protection of Child Rights (2007) are
1. National Commission for Scheduled statutory bodies in the sense that they are
Castes (SCs) established by acts of the Parliament.
2. National Commission for Women Q.1796 Who among the following can be removed
3. National Commission for Protection of in same manner as that of a Supreme Court
Child Rights Judge?
4. National Commission for Backward 1. Chairman of UPSC
Classes 2. Chief Election Commissioner
Select the correct answer using the codes 3. Chairman of CVC
below. 4. Attorney General of India
(a)  1, 2 and 3 only (b)  2, 3 and 4 only Choose the correct option from the codes
(c)  2 and 3 only (d)  1 and 4 only below:
Solution: (b) (a)  Only 2 (b)  1 and 2
Justification: The National Commission for (c)  2 and 3 (d)  1, 3 and 4
Scheduled Castes (SCs) is a constitutional Solution: (a)
body in the sense that it is directly established The Chairmans of UPSC and CVC can be
by Article 338 of the Constitution. removed by the President after a due inquiry

Indian Polity Question Bank P.513

05-Indian Polity_Q1507-2004.indd 513 8/7/2018 7:50:02 PM


by the SC. The Attorney general holds office Select the correct answer using the codes
during the pleasure of the President. below.
Q.1797 Which of the following are NOT eligible for (a)  1 and 2 only (b)  2 only
reappointment to the office? (c)  1 and 3 only (d)  1, 2 and 3
1. Comptroller and Auditor General of India Solution: (d)
2. Chief Information Commissioner Q.1800 Recommendations made by which of the
3. Chairman of UPSC following bodies is NOT binding on the
4. Chief Election Commissioner Government of India?
Choose the correct answer using the codes 1. Finance Commission of India
below 2. Central Vigilance Commission
(a)  1, 3 and 4 only (b)  All of the above 3. NITI Aayog
(c)  1 and 2 only (d)  2, 3 and 4 only Select the correct answer using the codes
Solution: (b) below.
Q.1798 Which of the following constitutional offices (a)  1 and 2 only (b)  2 and 3 only
cannot be held by the same individual for (c)  1 and 3 only (d)  1, 2 and 3
successive terms or more than one term? Solution: (d)
1. Office of the President of India Q.1801 Which of the following bodies are associated
2. Office of the Comptroller and Auditor with ‘Economic Planning’ in India?
General of India 1. NITI Ayog
3. Office of the Chairman of Rajya Sabha 2. The Parliament
4. Office of the Governor 3. Zila Panchayats
5. Office of the Chief Election Commissioner 4. State Planning departments
Choose the correct answer using the codes Select the correct answer using the codes
below. below:
(a)  1 and 2 only (b)  1, 3 and 5 only (a)  1 and 2 only (b)  2, 3 and 4 only
(c)  2 and 5 only (d)  2, 3, 4 and 5 only (c)  1 and 3 only (d)  All of the above
Solution: (c) Solution: (b)
India has had the first President running for The NITI Ayog is not concerned with
successive terms. It can be as many numbers economic planning, it is concerned with
of times. In the USA, however, there is a economic policy. That is the major difference
ban on more than two terms for a President. between this and Planning Commission.
CAG is an important constitutional office Parliament used to approve the five year
maintaining the financial accountability of plans and even now the annual budget
the executive to the legislature. A promise (an economic plan) is approved by the
of successive terms to a pliant bureaucrat Parliament. Zila panchayats consolidate the
can erode the autonomy of the institution. plans received by the lower tiers and forward
Same is the case with Election Commission it to the District Planning Committee.
of India. More than one term may erode Q.1802 Which of the following is/are statutory
the independence of the CEC. Governor bodies?
and Chairman, RS do not carry such 1. National Commission for Minorities
responsibilities. Hence, they can run for 2. National Commission for Scheduled
successive terms. Castes (SCs)
Q.1799 After their tenure, who among the following 3. National Commission for Protection of
is/are NOT eligible for further employment Child Rights
under the Central or a state government? 4. National Commission for Backward
1. Central Vigilance Commissioner Classes
2. Chief Information Commissioner 5. National Human Rights Commission
3. Chairman, National Human Rights Select the correct answer using the codes
Commission below.

P.514 For Civil Services Preliminary Examination

05-Indian Polity_Q1507-2004.indd 514 8/7/2018 7:50:02 PM


(a)  1, 3, 4 and 5 only (b)  2, 3 and 5 only Option B: The Nagalim is a proposed
(c)  1, 2, 3 and 4 only (d)  4 and 5 only independent country for the Naga people.
Solution: (a) The National Socialist Council of Nagaland 
(NSCN) group is leading these secessionist
Sectarian Ideologies – Communalism, movements.
Option C: The National Liberation Front
Regionalism, Casteism etc. of Tripura (or NLFT) is a Tripuri nationalist
Q.1803 Regionalism is a country-wide phenomenon organisation which seeks for Tripura to secede
which manifests itself in India in which of from India and establish an independent
these forms? Tripura state. It has actively participated in
(a)  Inter-state boundary disputes the Tripura Rebellion.
(b)  Secessionist movements Q.1805 Which of the following can be classified as
(c)  ‘Sons of the soil theory’ “communal” activities?
(d)  All of the above 1. When religion is expressed in politics in
Solution: (d) purely exclusive and partisan terms
Learning: It is the preference for a region or 2. When one religion tries dominating the
state in preference to the country as a whole. other because of chauvinism
There is also sub-regionalism, that is, love 3. When the demands of one religious group
for a particular region in preference to the are formed in opposition to another and
state of which the region forms a part. when state power is used to establish
Regionalism is a subsidiary process domination of one religious group over
of political integration in India. It is a the rest.
manifestation of those residual elements Choose the correct answer using the codes
which do not find expression in the national below:
polity and national culture. (a)  1 and 2 (b)  2 and 3
Option A: Inter-state boundary disputes (c)  1 and 3 (d)  All of the above
(like Chandigarh and Belgaum) and river- Solution: (d)
water disputes (like Cauvery, Krishna, Ravi- Having several religions with their own
Beas and so on) also show this. approach to living is fine in a multi-cultural
Option B: Demand of the people of nation. The problem begins when religion is
certain states for secession from the Indian seen as the basis of the nation. The example
Union (like Khalistan, Dravida Nadu, Mizos, of Northern Ireland shows the dangers
Nagas and so on) have shown this. of such an approach to nationalism. The
Option C: ‘Sons of the soil theory’ problem becomes more acute when religion
advocates preference to local people in is expressed in politics in exclusive and
government jobs, private jobs, permits and partisan terms, when one religion and its
so on. Their slogans are Assam for Assamese, followers are pitted against another. This
Maharashtra for Maharashtrians and so on. happens when beliefs of one religion are
Q.1804 Secessionist movements by the resident presented as superior to those of other
citizens have been NOT been orchestrated or religions, when the demands of one religious
active in which of these North-eastern states? group are formed in opposition to another
(a) Assam (b) Nagaland and when state power is used to establish
(c)  Tripura (d)  Arunachal Pradesh domination of one religious group over the
Solution: (d) rest. This manner of using religion in politics
Justification: Option A: The militant is communal politics.
organisation United Liberation Front of Communal politics is based on the
Assam (ULFA) demands a separate country idea that religion is the principal basis of
for the indigenous people of Assam. The social community. Communalism involves
Government of India had banned the ULFA thinking along the following lines. The
in 1990 and has officially labelled it as a followers of a particular religion must
terrorist group.
Indian Polity Question Bank P.515

05-Indian Polity_Q1507-2004.indd 515 8/7/2018 7:50:02 PM


belong to one community. Their fundamental Choose the correct answer using the codes
interests are the same. Any difference that below.
they may have is irrelevant or trivial for (a)  1 and 2 only
community life . (b)  1, 2 and 3 only
In its extreme form communalism leads to (c)  2, 3 and 4 only
the belief that people belonging to different (d)  All of the above
religions cannot live as equal citizens Solution: (d)
within one nation. Either, one of them has Partly due to their efforts and partly due to
to dominate the rest or they have to form other socio-economic changes, castes and
different nations. caste system in modern India have undergone
Q.1806 Which of the following ideologies can be great changes. With economic development,
counted as ‘Communal’ ideologies? large scale URBANISATION, growth of
1. The interest of religion X is different from literacy and education, OCCUPATIONAL
that of religion Y. MOBILITY and the weakening of the
2. The interests of religion X are antagonistic position of landlords in the villages, the
to that of religion Y. old notions of CASTE HIERARCHY are
3. The interests of religion X cannot be breaking down. The landlords often belonged
achieved unless the other religion Y is to upper castes and sought to perpetuate the
subordinated. discrimination meted out to the lower castes.
Choose the correct answer using the codes Since they were powerful in the villages, the
below: domination over the lower castes was also
(a)  1 and 2 only (b)  1 and 3 only adhered to by other villagers.
(c)  2 and 3 only (d)  All of the above Now, most of the times, in urban areas it
Solution: (d) does not matter much who is walking along
All are communal ideologies. next to us on a street or eating at the next table
In everyday language, the word in a restaurant. The Constitution of India
‘communalism ‘refers to aggressive prohibited any caste-based discrimination
chauvinism based on religious identity. and laid the foundations of policies to reverse
Chauvinism itself is an attitude that sees the injustices of the caste system.
one’s own group as the only legitimate Caste based discrimination is one of
or worthy group, with other groups being the menaces prevailing in Indian society.
seen– by definition – as inferior, illegitimate However, the Constitution provides some
and opposed. Thus, to simplify further, safeguards. Which of the following in the
communalism is an aggressive political constitution contain provisions that mandate
ideology linked to religion. This is a a non-discriminatory (caste)  society and the
peculiarly Indian, or perhaps South Asian, upliftment of the discriminated castes?
meaning that is different from the sense of 1. Directive principles of State Policy
the ordinary English word. In the English 2. Fundamental Rights
language, ‘communal’ means something 3. Preamble
related to a community or collectively as Choose the correct answer using the codes
different from an individual. below:
Which of the following factors could (a)  1 and 2 only (b)  1 and 3 only
possibly have contributed to the weakening (c)  2 and 3 only (d)  All of the above
of caste hierarchies in India? Solution: (d)
1. Urbanization Preamble clearly says that the Indian
2. Growth of literacy and education constitution promotes the ideal of social
3. Occupational mobility justice.
4. Weakening of the position of landlords in In Article 46 inside Directive Principles
villages of State Policies (DPSP) provides for the
welfare of the backward classes.

P.516 For Civil Services Preliminary Examination

05-Indian Polity_Q1507-2004.indd 516 8/7/2018 7:50:02 PM


The Articles 14-17 inside Fundamental Panchayat is concerned with the affairs of
Rights provide for non-discrimination against the Khap it represents. It is not affiliated with
any particular caste. the democratically elected local assemblies
Q.1807 The “Red Corridor” in India sometimes seen that are also termed Panchayat.
in news is related to A Khap panchayat has not any official
(a)  Biodiversity hotspots government recognition or authority, but can
(b) High non-Communicable Disease Burden exert significant social influence within the
(c)  Left Wing Extremism community they represent
(d)  Avalanches and landslides The Khap panchayts frequently deliberate
Solution: (c) on social issues to attempt to combat social
Learning: It is a region in the east of India problems like female abortions, alcohol abuse,
that experiences considerable Naxalite– dowry, rape etc.
Maoist insurgency. The Supreme Court has declared illegal
The 106 districts that span 10 States ‘Khap panchayats’ which often decree
—Bihar, Jharkhand, Andhra Pradesh, or encourage honour killings or other
Maharashtra, Odisha, Telangana, West institutionalised atrocities against boys and
Bengal, Madhya Pradesh, Uttar Pradesh girls of different castes and religions who
and Chhattisgarh — are described as those wish to get married or have married
affected by Left Wing Extremism (LWE)  and
constitute the ‘Red Corridor.’ Good Governance
The Union government has recently
Q.1809 Which of the following is NOT related to
decided to reduce the number of Maoist-
Good Governance?
affected districts by about one fifth of its
(a) Rule of Law and a citizen-friendly
previous extent. The considerations on which
government
the government has examined the districts
(b)  Higher spending on defence sector
with LWE features are:
(c) Increased transparency and accountability
• An assessment of the kind of logistical
of the government
and other support provided to armed
(d)  Greater use of E-governance
Maoist cadres by their sympathisers and
Solution: (b)
“over ground workers”, and
Learning: Good governance must be ‘pro-
• The kind of positive changes brought
people’ and ‘pro-active’. Good governance
about by development work that these
is putting people at the centre of the
districts have seen.
development process. It is about improving
Q.1808 A Khap Panchayat is government processes, brining greater
(a) a sub-body formed under Gram accountability, transparency and rule of law
Panchayat. in governance.
(b)  an assembly of elders of a group of It is a shift away from conventional
related clan. governance that focuses on rules and
(c)  a community organization affiliated procedures more than people. India is the only
with the formally elected government country in the world that officially celebrates
bodies. Christmas as ‘Good Governance Day’
(d)  affiliated with the democratically elected
Q.1810 The Second Administrative Reforms
local assemblies wielding local authority.
Commission (ARC) describes which of these
Solution: (b)
as “the master key to good governance”?
Learning: The Khap is a clan or a group
(a)  Building social capital
of related clan, mainly among the Jats of
(b)  Establishing e-governance
Western UP and Haryana. A Khap Panchayat
(c)  Inculcating ethics in Governance
is an assembly of Khap elders.
(d)  Right to information (RTI)
It is unaffiliated with the formally elected
Solution: (d)
government bodies. A Khap

Indian Polity Question Bank P.517

05-Indian Polity_Q1507-2004.indd 517 8/7/2018 7:50:02 PM


Learning: Good governance has four (b) To make service-delivery more citizen-
elements- transparency, accountability, friendly
predictability and participation. (c) To increase the penetration of
• Transparency refers to availability of Government in remote areas
information to the general public and (d) To provide all government services free
clarity about functioning of governmental of cost of citizens
institutions. Solution: (b)
• The RTI opens up government’s records Learning: Citizen’s Charter is a document
to public scrutiny, thereby arming citizens which represents a systematic effort to focus
with a vital tool to inform them about on the commitment of the Organisation
what the government does and how towards its Citizens in respects of Standard
effectively, thus making the government of Services, Information and Grievance
more accountable. Redress etc. The Citizen’s Charter is not
• The access to information can empower legally enforceable and, therefore, is non-
the poor and the weaker sections of justiciable.
society to demand and get information However, it is a tool for facilitating the
about public policies. delivery of services to citizens with specified
• Without good governance, no amount standards, quality and time frame etc. with
of developmental schemes can bring commitments from the Organisation and its
improvements in the quality of life of the clients.
citizens. A good Citizen’s Charter should have the
• It has been rightly been seen as the key to following components:
strengthening participatory democracy and • Vision and Mission Statement of the
ushering in people centred governance. Organisation
Q.1811 Which of the following laws/rules/ • Details of Business transacted by the
regulations/committees/bodies deal with Organisation
citizen’s grievances and corruption in the • Details of ‘Citizens’ or ‘Clients’
administration? • Statement of services including standards,
1. The Public Accounts Committee quality, time frame etc. provided to each
2. Central Vigilance Commission Citizen/ Client group separately and how/
3. National Consumer Disputes Redressal where to get the services
Commission (NCDRC) • Details of Grievance Redress Mechanism
Choose the correct answer using the codes and how to access it
below: • Expectations from the ‘Citizens’ or
(a)  1 and 2 (b)  2 and 3 ‘Clients’
(c)  1 and 3 (d)  All of the above • Additional commitments such as
Solution: (d) compensation in the event of failure of
The PAC looks into irregularities in public service delivery.
finances as brought to it by the CAG reports. Q.1813 ‘Sevottam’ is related to
The CVC looks into complaint against (a) Citizen’s charter in public organizations
higher officers. (b) Grievance redressal programmes within
The NCDRC looks into the complaints by the most vulnerable communities
the citizens in case of their exploitation by (c) Reform of Public Sector Banks (PSBs)
either public or private companies (d)  Providing support to first generation
entrepreneurs under ‘Make in India’
Citizen’s charter programmes
Solution: (a)
Q.1812 What is the objective of a “Citizen Charter”? Learning: The Department of Administrative
(a) 
To increase the coordination between Reforms and Public Grievances (DARPG),
different departments of the Government Ministry of Personnel, Public Grievances
that deal with citizens

P.518 For Civil Services Preliminary Examination

05-Indian Polity_Q1507-2004.indd 518 8/7/2018 7:50:02 PM


and Pensions has taken steps to put in place Statement 2: This should be clear.
Sevottam Compliant Citizen’s Charter and Statement 3: Government often holds
Grievance Redress Mechanism. press briefings, where media persons hold the
The Sevottam framework was designed government accountable by posing it various
by DARPG in 2006 as an assessment questions.
improvement framework for public service Statement 4: This is mainly via elections,
delivery. but even otherwise a citizen has the right to
ask government questions either through RTI
Accountability or directly by visiting a government official.
Q.1814 Which of the following are is/are ways
in which the government can be made
Media
accountable to the people in general? Q.1816 Mass Media is considered as the fourth pillar
1. Media of democracy because of its independence
2. Social movements from the other three organs. Which of the
3. Voting following may possibly undermine the ability
Choose the correct answer using the codes of media to be unbiased, fair and represent
below: pluralistic points of view?
(a)  1 and 2 (b)  2 and 3 1. Ownership by large and important
(c)  1 and 3 (d)  All of the above corporate houses.
Solution: (d) 2. Strict censorship imposed by the government
Accountability of the government is basically 3. Separation of ownership and editorial
answerability for its deeds. If the activities board of media houses
of the government can be communicated Choose the correct answer using the codes
and questioned by the people, it shows that below:
there is a sense of answerability. Finally it (a)  1 and 2 (b)  2 and 3
is up to the people to decide the fate of the (c)  1 and 3 (d)  All of the above
government by voting. Solution: (a)
All these are ways in which the government Large bourses who control the money flow
can be made accountable. You can also see to media can prevent unbiased reporting from
several rallies, marches and protests that keep media. They further may have connection
the government of the day at its toe. with the political class and this may
Q.1815 The Government of India can be held encourage selective and distorted reporting.
answerable to which of the following? The option 3 is in fact a way to make
1. Higher Judiciary the functioning of media transparent and
2. Lok Sabha unbiased.
3. Media Q.1817 Consider the following about the Fourth
4. Citizens of India Estate of democracy in India.
Select the correct answer using the codes 1. The fourth estate has been established by
below. the constitution.
(a)  1 and 3 only (b)  2, 3 and 4 only 2. Some of the rights of fourth estate are
(c)  4 only (d)  1, 2, 3 and 4 protected by the constitution.
Solution: (d) Which of the above is/are correct?
Justification: Statement 1: You would (a)  1 only (b)  2 only
have seen the courts often reprimanding (c)  Both 1 and 2 (d)  None
the government for its lack of action in key Solution: (b)
areas such as road accidents. The Judiciary Justification: “Fourth Estate” most
often asks the government vital questions on commonly refers to the news media. Other
governance and asks its officials to submit three estates are legislature, executive and
their reply on time. judiciary.

Indian Polity Question Bank P.519

05-Indian Polity_Q1507-2004.indd 519 8/7/2018 7:50:02 PM


Statement 1: It is autonomous of the Consumer Rights
government.
Statement 2: Article 19, as interpreted by Q.1819 Provisions for consumer protection have been
the Supreme Court, provides them the right promulgated by/in which of the following?
against censorship and thus gives them right 1. United Nations
to free speech. 2. Indian constitution
The term fourth estate makes implicit 3. Indian laws
reference to the earlier division of the three Choose the correct answer using the coded
estates of the realm: the clergy, the nobility, below:
and the commoners. (a)  1 and 2 (b)  2 and 3
(c)  1 and 3 (d)  All of the above
Q.1818 Consider the following statements about the
Solution: (c)
role of media in elections.
In 1985 United Nations adopted the UN
1. All recognised National and State parties
Guidelines for Consumer Protection. This
are allowed free access to the state
was a tool for nations to adopt measures
owned electronic media scale for their
to protect consumers and for consumer
campaigns for a certain duration during
advocacy groups to press their governments
elections.
to do so. At the international level, this
2. Media persons are given special passes
has become the foundation for consumer
to enter polling stations to cover the poll
movement.
process and the counting halls during the
Indian constitution does not have any
actual counting of votes.
provision. The Laws, such as, the Consumer
Which of the above is/are correct?
protection act, 1986 does exist.
(a)  1 only (b)  2 only
(c)  Both 1 and 2 (d)  None Q.1820 Which of the following rights are parts of
Solution: (c) consumer rights in India?
Justification: Statement 1: By Election 1. The right to be protected from all kind of
Commission, all recognized National and hazardous goods and services
State parties have been allowed free access 2. The right to be fully informed about the
to the state owned electronic media–AIR and performance and quality of all goods and
Doordarshan–on an extensive scale for their services
campaigns during elections. 3. The right to complete consumer education
The total free time allocated extends over Choose the correct answer using the coded
122 hours on the state owned Television and below:
Radio channels. This is allocated equitably (a)  1 and 2 (b)  2 and 3
by combining a base limit and additional (c)  1 and 3 (d)  All of the above
time linked to poll performance of the party Solution: (d)
in recent election. (See http://www.consumerrights.org.in/cgi-
Statement 2: After the polling has finished, sys/suspendedpage.cgi)
the votes are counted under the supervision of Q.1821 Consumer dispute redressal forums, under
Returning Officers and Observers appointed the Consumer Protection Act,1986, have
by the Election Commission. been established, inter alia, at
In order to bring as much transparency as 1. District level
possible to the electoral process, the media 2. Block level
are encouraged and provided with facilities 3. Village level
to cover the election (including the actual Choose the correct answer using the coded
counting of votes), although subject to below:
maintaining the secrecy of the vote. (a)  1 and 2 (b)  2 and 3
(c)  1 and 3 (d)  Only 1
Solution: (d)

P.520 For Civil Services Preliminary Examination

05-Indian Polity_Q1507-2004.indd 520 8/7/2018 7:50:02 PM


They are at Central, State and district levels Explanation: The PHC usually serves a
with specifications for filing cases regarding group of villages and a population of 3,000-
to certain goods and services. 5,000. The PHC supervises sub-centres
Q.1822 Consider the following statements. which usually exist at village level. So, the
1. Grahak Suvidha Kendra (GSK) help option (b) is wrong.
redress consumer complaint against a Community Health Centre is for a large
trader based in another State. group of villages usually serving a larger
2. Prime Minister chairs the Central population of around 15-20,000 people. It is
Consumer Protection Council. not dedicated to a village. It supervises 3-5
3. A statutory Consumer Forum Network PHCs. So, the option (c) is also wrong.
has been established in India at district, Sub-divisional hospitals exist at the sub-
state and Central levels. division level, which is one level lower than
Select the correct answer using the codes the District Hospital. So, the option (d) is also
below. wrong.
(a)  1 only (b)  1 and 3 only Learning: The Public Health Service (PHS)
(c)  2 and 3 only (d)  1 and 2 only is a chain of health centres and hospitals run
Solution: (b) by the government. They are linked together
Justification: Statement 2: Here the Minister so that they cover both rural and urban areas
of Consumer Affairs, Food and Public and can also provide treatment to all kinds of
Distribution is the Chairman, and the Minister problems – from common illnesses to special
of State, Consumer Affairs, Food and Public services.
Distribution is the Vice-Chairman. At the village level there are health
Corresponding ministers at the State level centres (sub-centres) where there is usually
are also its members. a nurse and a village health worker. They are
Statement 1: It will help redress consumer trained in dealing with common illnesses and
complaint against a trader based in another work under the supervision of doctors at the
State (for instance, if one is claiming a refund Primary Health Centre (PHC).
which the trader has refused and which the Such a centre covers many villages in a
consumer believes he/she is entitled to). rural area. At the district level is the District
It can ask counterpart centre in the State Hospital that also supervises all the health
of the trader/ service provider to contact the centres.
trader on consumer’s behalf and try to find a Therefore the hierarchy of Indian PHS is
solution to his/her complaint. as follows.
Statement 3: Under the provisions of the Sub-centre < PHC < CHC < Sub-
Consumer Protection (CP) Act, 1986 there is divisional Hospital < District Hospital
a three tier quasi-judicial mechanism at the Q.1824 ‘Kayakalp’ from the Ministry of Health and
District, State and National levels to provide Family Welfare (MoHFW) relates to
simple and speedy resolution to consumer (a)  Health sector awards
disputes. (b) Public Hospital modernization guidelines
(c) Revamping rural Primary Health centres
Public Health (PHCs)
(d)  None of the above
Q.1823 At the village level there are health centres Solution: (a)
where there is usually a nurse and a village Explanation and Learning: The ‘Kayakalp’
health worker. It is known as has been launched to promote cleanliness,
(a)  Sub-centre Health hygiene and infection control practices in
(b)  Primary Heath Centre (PHC) public health facilities.
(c)  Community Health Centre(CHC) Under this initiative, public healthcare
(d)  Sub-divisional hospital facilities shall be appraised and such public
Solution: (a) healthcare facilities that show exemplary

Indian Polity Question Bank P.521

05-Indian Polity_Q1507-2004.indd 521 8/7/2018 7:50:02 PM


performance meeting standards of protocols of • Mental Health Act, 1987
cleanliness, hygiene and infection control will • Food Safety and Standards Regulations
receive awards and commendation. Under this • The Protection of Women From Domestic
initiative, the numbers of awards are as under: Violence Act, 2005
• Best two District Hospitals in each state • The Marriage Laws (Amendment) Bill,
(Best District hospital in small states) 2010
• Best two Community Health Centres/Sub • The Prohibition Of Sexual Harassment Of
District Hospitals (CHC/ SDH) (limited to Women At Workplace Bill, 2010
one in smaller states). • Food Safety and Standards (Prohibition and
• One Primary Health Centre (PHC)  in Restrictions on Sales) Regulations, 2011
every district • Food Safety and Standards (Contaminants,
Q.1825 The role of government in maintaining public Toxins and Residues) Regulations, 2011
health is mandated/suggested in which of the Q.1826 Which of these laws in India have a bearing
following? on health?
1. Constitution of India. 1. Narcotic Drugs and Psychotropic
2. Government policies Substances Act, 1985
3. Parliamentary enactments 2. The Prevention of Food Adulteration Act,
Choose the correct answer using the code 1954
below: 3. Bio-Medical Waste (Management and
(a)  1 and 2 (b)  2 and 3 Handling)  Rules, 1998
(c)  1 and 3 (d)  All of the above 4. Consumer Protection Act, 1986
Solution: (d) Select the correct answer using the codes
The Constitution of India prescribes in below.
Directive principles of State policies that (a)  1 and 2 only (b)  3 and 4 only
it shall be the state’s endeavour to raise the (c)  1, 2 and 3 only (d)  1, 2, 3 and 4
level of nutrition and standard of living and Solution: (d)
to improve public health. Justification: Statement 1: The Act prohibits
The National health policy 2002 prescribes a person to produce/manufacture/cultivate,
the role of government in public health. possess, sell, purchase, transport, store, and/
Several laws related to health in India are: or consume any narcotic drug or psychotropic
• The Medical Termination of Pregnancy substance which has a direct bearing on
Act and Rules personal and social health.
• The Pre-Natal Diagnostic Techniques Statement 2: Food adulteration, such as
(PNDT) Act and Rules mixing of ammonia in milk, significantly risks
• Acts in Disability public health and is thus regulated by the Act.
• Insecticides Act and Rules Statement 3: Bio-medical wastes emitted
• Maternity Benefit Act and Rules by hospitals and other such institutions are
• Narcotic Drugs and Psychotropic regulated by this act, which could become a
Substances Act and Rules local menace by leaching into groundwater,
• The Prevention of Food Adulteration Act, spread microorganisms etc.
1954 Statement 4: Suppose a wrong medicine
• Drugs and Cosmetics Act, 1940 was prescribed by a hospital to patient,
• Bio-Medical Waste (Management and consuming which causes immense damage
Handling)  Rules, 1998 to the patient. The patient, as a consumer, can
• The Pharmacy Act, 1948 raise this issue in the consumer court and get
• The Transplantation of Human Organs adequate compensation.
Act and Rules
• Environmental Acts and Rules
• Consumer Protection Act and Medical
Profession

P.522 For Civil Services Preliminary Examination

05-Indian Polity_Q1507-2004.indd 522 8/7/2018 7:50:02 PM


PREVIOUS YEARS QUESTION Decorum is the maintaining proper behaviour.
Interpellation is the formal right of a
PAPERS Parliament to submit formal question to the
government.
2000 Crossing the floor depicts to vote against
Q.1827 Which one of the following statements about the party lines.
a Money Bill is not correct? Yielding the floor means the speaker
(a) A Money Bill can be tabled in either giving part of his or her speaking time
House of Parliament to another speaker. While this practice is
(b)  The Speaker of Lok Sabha is the final allowed in some legislative bodies, it is not
authority to decide whether a Bill is a allowed in deliberative assemblies, unless
Money Bill or not specifically authorized in the rules.
(c) The Rajya Sabha must return a Money Q.1830 Consider the following statements about the
Bill passed by the Lok Sabha and send it Attorney General of India
for consideration within 14 days 1. He is appointed by the President of India
(d) The President cannot return a Money Bill 2. He must have the same qualifications as
to the Lok Sabha for reconsideration are required for a judge of the Supreme
Solution: (a) Court
Explaination: Money bill can be introduced 3. He must be a member of either House of
only in LS (not in RS) that too on the Parliament
recommendation of the President. 4. He can be removed by impeachment by
Q.1828 The 73rd Constitution Amendment Act, 1992 Parliament
refers to the. Which of these statements are correct?
(a) generation of gainful employment for (a)  1 and 2 (b)  1 and 3
the unemployed and the under employed (c)  2, 3 and 4 (d)  3 and 4
men and women in rural areas Solution: (a)
(b)  generation of employment for the Under Article 76, the impeachment procedure
able bodied adults who are in need of the Attorney-General is not provided.
and desirous of work during the lean He is appointed by the president and s/he
agricultural season shall hold office during the pleasure of the
(c) laying the foundation for strong and vibrant President. President decides his remuneration
Panchayati Raj institutions in the country and service condition. He must be qualified
(d)  guarantee of right to life, liberty and to be appointed as judge of supreme court
security of person, equality before and must not be a member of either House
law and equal protection without of Parliament. He is the chief law officer of
discrimination union and is a member of central executive.
Solution: (c) Q.1831 The primary function of the Finance
Explanation: 73rd Amendment Act added Commission in India is to
Eleventh Schedule to the Constitution and (a)  distribution of revenue between the
Part IX, which provides for the Panchayati Centre and the States
Raj System. (b)  prepare the Annual Budget
Q.1829 The Speaker can ask a member of the House (c)  advise the President on financial matters
to stop speaking and let another member (d) allocate funds to various ministries of the
speak. This phenomenon is known as Union and State Governments
(a) decorum Solution: (a)
(b)  crossing the floor According to Article 280 it shall be the duty
(c) interpellation of the Commission to make recommendations
(d)  yielding the floor to the President as to
Solution: (d) 1. the distribution between the Union and the
States of the net proceeds of taxes which

Indian Polity Question Bank P.523

05-Indian Polity_Q1507-2004.indd 523 8/7/2018 7:50:02 PM


are to be, or may be, divided between (a) he obtains permission from the principal
them under this Chapter and the allocation of his college
between the States of the respective shares (b)  he is a member of a political party
of such proceeds (c)  his name figures in the voters’ list
2. the principles which should govern the (d)  he files a declaration owing allegiance to
grants in aid of the revenues of the States the Constitution of India
out of the Consolidated Fund of India Solution: (c)
3. any other matter referred to the Name in the voters list is the important
Commission by the President in the condition
interests of sound finance Q.1836 Match List I with List II and select the correct
4. The Commission shall determine their answer using the codes given below the lists
procedure and shall have such powers
in the performance of their functions as List-I List-II
Parliament may by law confer on them. (Local bodies) (States as in 1999)
Q.1832 The Parliament can make any law for the A. Zila Parishads at the sub- 1. Andhra Pradesh
whole or any part India for implementing divisional level
international treaties B. Mandal Praja Parishad 2. Assam
(a)  with the consent of all the State
(b) with the consent of the majority of States C. Tribal Councils 3. Mizoram
(c) with the consent of the States concerned D. Absence of Village 4. Meghalaya
(d)  without the consent of any State Panchayats
Solution: (d)
Codes:
It is the sole prerogative of Parliament under
(a)  A-2, B-1, C-4, D-3
Article 253 of the Constitution.
(b)  A-1, B-2, C-4, D-3
Q.1833 The state which has the largest number of (c)  A-3, B-2, C-1, D-4
seats reserved for the Scheduled Tribes in the (d)  A-2, B-1, C-3, D-4
Lok Sabha is Solution: (d)
(a) Bihar (b) Gujarat
Q.1837 The following news item appeared in a
(c)  Uttar Pradesh (d)  Madhya Pradesh
national daily dated 1 December 1999:
Solution: (d)
“...Parliament today rejected a Bill to
In Madhya Pradesh, the largest number of
grant women the right to vote and stand for
seats are reserved for scheduled tribe in the
office in parliamentary elections, by a margin
Lok Sabha.
of 32 to 30. The National Assembly was split
Q.1834 Consider the following functionaries between liberal pro-government and Shiite
1. Cabinet Secretary Muslim deputies who were in favour of
2. Chief Election Commissioner women’s rights, while the opposition camp
3. Union Cabinet Minister grouped Sunni Muslim fundamentalists and
4. Chief Justice of India tribal MPs. A total of 64 MPs and Ministers
Their correct sequence in the Order of were present, of whom two abstained.”
Precedence is: The Parliament referred to in this
(a)  3, 4, 2, 1 (b)  4, 3, 1, 2 quotation is that of:
(c)  4, 3, 2, 1 (d)  3, 4, 1, 2 (a) Kuwait (b) Iran
Solution: (c) (c) Bahrain (d) Saudi Arabia
Chief Justice of India < Union Cabinet Solution: (a)
Minister < Chief Election Commissioner and Kuwait’s all-male Parliament rejected granting
Cabinet Secretary women full political rights. The vote was 32
Q.1835 A college student desires to get elected to the to 30 with two abstentions. With 64 members
Municipal Council of his city. The validity present, the bill needed 33 votes to pass.
of his nomination would depend on the
important condition, among others, that

P.524 For Civil Services Preliminary Examination

05-Indian Polity_Q1507-2004.indd 524 8/7/2018 7:50:03 PM


2001 Solution: (d)
Seventh schedule: It lists the distribution of
Q.1838 Which Article of the Constitution provides powers between the Union and the states.
that it shall be the endeavour of every state Eighth schedule: It contains the languages
to provide adequate facility for instruction listed in the Constitution.
in the mother tongue at the primary stage of Eleventh schedule: subject matter of
education? Panchayat.
(a)  Article 349 (b)  Article 350 Q.1841 Match List I with List II and select the correct
(c)  Article 350-A (d)  Article 351 answer using the codes given below the lists.
Solution: (a)
These are some of the directives outside the List-I List-II
DPSP of the Constitution. (Amendments to the (Descriptions)
Constitution)
Q.1839 Which one of the following duties is not
performed by Comptroller and Auditor A. The Constitution 1. Establishment of
general of India? (69th Amendment) state level Rent
(a) To audit and report on all expenditure Act, 1991 Tribunals
from the Consolidated Fund of India B. The Constitution 2. No reservations for
(b) To audit and report on all expenditure (75th Amendment) Scheduled Castes
from the Contingency Funds and Public Act. 199 in Panchayats in
Accounts Arunachal Pradesh
(c) To audit and report on all trading,
C. The Constitution 3. Constitution of
manufacturing, profit and loss accounts
(80th Amendment) Panchayats in
(d) To control the receipt and issue of public
Act, 2000 Villages or at other
money, and to ensure that the public
local level
revenue is lodged in the exchequer.
Solution: (d) D. The Constitution 4. Accepting the
As per provisions under Article 149, the (83rd Amendment) recommend-
Comptroller and Auditor General shall Act, 2000 ations of the
perform such duties and exercise such powers Tenth Finance
in relation to the accounts of the Union and Commission
of the States and of any other authority or 5. According to the
body as may be prescribed by or under any Constitutional status
law made by Parliament and, until provision of National Capital
in that behalf is so made, shall perform such Territory to Delhi
duties and exercise such powers in relation to
the accounts of the Union and of the States Codes:
as were conferred on or exercisable by the (a)  A-5, B-1, C-4, D-2
Auditor General of India immediately before (b)  A-1, B-5, C-3, D-4
the commencement of this Constitution in (c)  A-5, B-1, C-3, D-4
relation to the accounts of the Dominion of (d)  A-1, B-5, C-4, D-2
India and of the Provinces respectively. Solution: (a)
Q.1840 Which one of the following statements Q.1842 If a new state of the Indian Union is to be
correctly describes the Fourth Schedule of created, which one of the following schedules
the Constitution of India? of the Constitution must be amended?
(a) It lists the distribution of powers between (a) First (b) Second
the Union and the states. (c) Third (d) Fifth
(b) It contains the languages listed in the Solution: (a)
Constitution. First schedule contains names of the States
(c)  subject matter of panchayat. and Union Territories, that’s why it should be
(d) It allocates seats in the Council of State. amended, if a new state is created.

Indian Polity Question Bank P.525

05-Indian Polity_Q1507-2004.indd 525 8/7/2018 7:50:03 PM


Q.1843 Match List-I with List-II and select the The State government does not have control
correct answer. over its local bodies in matters of Citizens’
using the codes given below the lists: [2001] grievances.
List-I List-II The 73rd amendment act says that state
(Articles of the (Contents) legislature can provide for the collection of
Constitution) taxes by the panchayat, allocate net proceeds
of taxes etc. It shows state government has
A. Article 54 1. Election of the President control over finances.
of India State government appoints Panchayat
B. Article 75 2. Appointment of the Prime secretary who is the executive officer at
Minister Panchayat level. Hence it has control over
C. Article 155 3. Appointment of the personnel matters.
Governor of a State 73rd amendment act prescribes 29 subjects
that can be entrusted to the panchayats. This
D. Article 164 4. Appointment of the Chief devolution is also the prerogative of the state
Minister and COMs of the legislature. Hence state government has
state control over legislations too.
5. Composition of Legislative Q.1846 Consider the following statements regarding
Assemblies the political parties in India
Codes: 1. The Representation of the People Act,
(a)  A-1, B-2, C-3, D-4 1951 provides for the registration of
(b)  A-1, B-2, C-4, D-5 political parties
(c)  A-2, B-1, C-3, D-5 2. Registration of political parties is carried
(d)  A-2, B-1, C-4, D-3 out by the Election Commission
Solution: (a) 3. A national level political party is one
which is recognized in four or more states
Q.1844 In what way does the Indian Parliament
4. During the 1999 general elections, there
exercise control over the administration?
were six national and 48 state level parties
(a)  Through Parliamentary Committees
recognised by the Election commission
(b) Through Consultative Committees in
Which of these statements are correct?
various ministries
(a)  1, 2 and 4 (b)  1 and 3
(c) By making the administrators send
(c)  2 and 4 (d)  1, 2, 3 and 4
periodic reports
Solution: (d)
(d) By compelling the executive to issue writs
Match List I with List II and select the correct
Solution: (a)
answer using the codes given below the lists
Parliamentary Committees are formed to
dispose of the large volume of work in time List-I List-II
but with detailed scrutiny. Their appointment, (Publisher) (Publication)
terms of office as well as functions etc. are A. Ministry of Industry 1. Report on Currency and
regulated by provisions under Article 118(1). Finance
These are of two kinds Standing Committees
B. Central Statistical 2. Economic Survey
and Ad hoc Committees.
Organisation
Q.1845 In which one of the following areas does the
C. Reserve Bank of India 3. Wholesale Price Index
State Government not have control over its
local bodies? D. Ministry of Finance 4. National Accounts
(a)  Citizens’ grievances Statistics
(b)  Financial matters Codes:
(c) Legislation (a)  A-4, B-3, C-2, D-1
(d)  Personnel matters (b)  A-3, B-4, C-1, D-2
Solution: (a)

P.526 For Civil Services Preliminary Examination

05-Indian Polity_Q1507-2004.indd 526 8/7/2018 7:50:03 PM


(c)  A-4, B-3, C-1, D-2 Prime Minister as its head, all Union Cabinet
(d)  A-3, B-4, C-2, D-1 ministers, the CMs of all states, CMs/
Solution: (b) Administrators of all UTs and the members
Q.1847 The Supreme Court of India tenders advice to of the Planning Commission.
the President on a matter of law or fact. There is no constitutional provision
(a)  on its own initiative regarding the accountability of the planning
(b)  only if he seeks such advice commission the parliament.
(c) only if the matter relates to the Which one of the following amendments
Fundamental Rights of citizens to the Indian Constitution empowers the
(d) only if the issue poses a threat to the President to send back any matter for
unity and integrity of the country reconsideration by the Council of Ministers?
Solution: (b) (a) 39th (b) 40th
As per the provisions under Article 143, the (c) 42nd (d) 44th
president can refer matter to the Supreme court. Solution: (d)
The supreme court has to tender the advice if • Before the 42nd amendment, Article
the question pertains to the pre constitutional 74(1) stated that “there shall be a Council
treaties and agreements. The advice tendered by of Ministers with the Prime Minister at the
the supreme court isadvisory not compulsory. head to aid and advise the President in the
It allows union government to have expert exercise of his functions”. However, there
opinion of the legal matters. was a slight ambiguity whether the advice
of the Council of Ministers is binding on
the President.
2002 • The Forty-second Amendment of the
Q.1848 With reference to Indian polity, which one of 42nd Constitutional Amendment (1976)
the following statements is correct? made it explicit that the President shall,
(a) Planning Commission is accountable to “act in accordance with such advice”.
the Parliament. The amendment went into effect from
(b) President can make ordinance only when 3 January 1977.
either of the two Houses of Parliament is • The 44th Amendment (1978) however
not in session. added that the President can send the
(c) The minimum age prescribed for advice back for reconsideration once. But
appointment as a Judge of the Supreme if the Council of Ministers sends the same
Court is 40 years. advice again to the President then the
(d) National Development Council is President must accept it. The amendment
constituted of Union Finance Minister went into effect from 20 June 1979.
and the Chief Ministers of all the States. Q.1849 The term of the Lok Sabha
Solution: (b) (a) cannot be extended under any
President can issue an ordinance only when circumstances
both or either houses of parliament are not in (b) can be extended by six months at a time
session (Article 123). It is because a bill has (c) can be extended by one year at a time
to be passed by both the house of parliament during the proclamation of emergency
to become law. (d) can be extended for two years at a time
No minimum age is prescribed for during the proclamation of emergency
appointment as a judge of the Supreme Court Solution: (c)
in the Constitution. The age of a Judge of the While a proclamation of National Emergency
SC shall be determined by such authority and is in operation, the life of the Lok Sabha may
in such manner as parliament may by law be extended beyond its normal term (five
provide. years) by a law of Parliament for one year at
Erstwhile the National Development a time (for any length of time with periodic
Council (NDC) is composed of the approval of Parliament). However, this

Indian Polity Question Bank P.527

05-Indian Polity_Q1507-2004.indd 527 8/7/2018 7:50:03 PM


extension cannot continue beyond a period of C. Article 30 (1) 3. All minorities whether based
six months after the emergency has ceased to on religion or language
operate. For example, the term of the Fifth shall have to establish and
Lok Sabha (1971–1977) was extended two administer educational
times by one year at a time. Similarly, the institutions of their choice.
Parliament may extend the normal tenure
of a state legislative assembly (five years) D. Article 31 (1) 4. No citizen shall be denied
by one year each time (for any length of admission into any
time)  during a national emergency, This also educational institution
subject to a maximum period of six months maintained by the State
after the Emergency has ceased to operate. or receiving State aid on
grounds of religion, race,
Q.1850 The 93rd Constitution Amendment deals caste, language or any of
with the them.
(a) continuation of reservation for backward
classes in government employment Codes:
(b) free and compulsory education for (a)  A-2, B-4, C-3, D-1
all children between the age of 6 and (b)  A-3, B-1, C-2, D-4
14 years (c)  A-2, B-1, C-3, D-4
(c) reservation of 30% posts for women in (d)  A-3, B-4, C-2, D-1
government recruitments Solution: (a)
(d) allocation of more number of Q.1851 Consider the following statements with
parliamentary seats for recently created reference to India.
States 1. The Chief Election Commission and other
Solution: (b) Election Commissioners enjoy equal
According to 93rd Amendment, every Child powers but receive unequal salaries
of the age group of 6-14 years shall have right 2. The Chief Election Commissioner is
to free and compulsory Education. No child entitled to the same salary as is provided
is liable to pay any kind of fee/capitation fee/ to a judge of the Supreme Court
charges. A collection of capitation fee invites 3. The Chief Election Commissioner shall
a fine up to 10 times the amount collected. It not be removed from his office except
is enforced by the Right to Education (RTE) in like manner and on like grounds as a
Act, 2009. judge of the Supreme Court
Match List I (Articles of Indian Constitution) 4. The term of office of the Election
with List II (Provisions) and select the correct Commissioner is five years from the date he
answer using the codes given below the lists assumes his office or till the day he attains
the age of 62 years, whichever is earlier
List-I List-II Which of these statements are correct?
(Articles (Provisions) (a)  1 and 2 (b)  2 and 3
of Indian (c)  l and 4 (d)  2 and 4
Constitution) Solution: (b)
A. Article 16 (2) 1. No person shall be deprived The Chief Election Commissioner and other
of his property save by the Election Commissioners enjoy equal powers
authority of law. and salaries. The term of office of the Election
B. Article 29 (2) 2. No person can be Commissioner is six years or till he attains
discriminated against the age of 65 years or whichever is earlier.
in the matter of public Q.1852 The purpose of the inclusion of Directive
appointments on the grounds Principles of the State Policy in the Indian
of race, religious or caste. Constitution is to establish:
(a)  political democracy
(b)  social democracy

P.528 For Civil Services Preliminary Examination

05-Indian Polity_Q1507-2004.indd 528 8/7/2018 7:50:03 PM


(c)  Gandhian Concept of democracy (c)  Contingency Fund of India
(d)  Social and economic democracy (d)  Contingency Fund of the State
Solution: (d) Solution: (b)
The purpose of Directive Principles of The salaries and allowances of the Judges of
State Policy is to establish the social and the HC are charged to the Consolidated Fund
economic democracy. Political democracy is of the state but their pensions are payable as
established by the Fundamental Rights. Charged Expenditure /Article 112(3).
Q.1853 Which one of the following Articles of the Q.1857 In the Indian Constitution, the Right to
Directive Principles of the State Policy deals Equality is granted by five Articles. They are.
with the promotion of international peace and (a)  Article 16 to Article 20
security? (b)  Article 15 to Article 19
(a) 51 (b) 48 A (c)  Article 14 to Article 18
(c) 43 A (d) 41 (d)  Article 13 to Article 17
Solution: (a) Solution: (c)
Article 51 states that the state shall endeavour There are six groups of Fundamental Rights:
to promote international peace and security A.  Right to Equality (Articles 14–18)
Q.1854 In the case of election to the Lok Sabha, the B.  Right to Freedom (Articles19–22)
amount of Security deposited for general C. Right against Exploitation (Articles 23
category candidates and SC/ST category and 24)
candidates respectively is D. Right to Freedom of Religion (Articles
(a)  Rs. 5,000 and Rs. 2,500 25 and 28),
(b)  Rs. 10,000 and Rs. 2,500 E. Cultural and Educational Rights (29 and
(c)  Rs. 10,000 and Rs. 5,000 30)
(d)  Rs. 15,000 and Rs. 7,500 F. Right to Constitutional remedies (Articles
Solution: (c) 32).
As per the given options the right answer Q.1858 Which one of the following rights was
is the option (c). But for general candidates described by Dr. B.R. Ambedkar as the ‘heart
the amount of security deposit is Rs. 20,000. and soul’ of the Constitution?
In case of elections of state legislatures, the (a)  Right to Freedom of Religion
amount of security deposit is as given in (b)  Right to Property
option (a) above. (c)  Right to Equality
Q.1855 The Consultative Committee of Members of (d)  Right to Constitutional Remedies
Parliament for Railway Zones is constituted Solution: (d)
by the. Right to Constitutional Remedies under
(a)  President of India article 32 is a Fundamental Right. It was
(b)  Ministry of Railways called the ‘Heart and Soul’ of Indian
(c)  Ministry of Parliament Affairs Constitution by B. R. Ambedkar.
(d)  Ministry of Transport Q.1859 Which one of the following authorities
Solution: (c) recommends the principles governing the
The Main purpose of these Committees is grants-in-aid of the revenues to the states out
to provide a forum for informal discussions of the Consolidated Fund of India?
between the Government and Members of (a)  Finance Commission
Parliament on policies and programmes of (b)  Inter-State Council
the Government and the manner of their (c)  Union Ministry of Finance
implementation. (d)  Public Accounts Committee
Q.1856 The salaries and allowances of the Judges of Solution: (a)
the High Court are charged to the The finance commission will be governed by
(a)  Consolidated Fund of India the principles to allocate Grants-in-Aid of the
(b)  Consolidated Fund of the State revenues of the states out of the consolidated
fund of India.

Indian Polity Question Bank P.529

05-Indian Polity_Q1507-2004.indd 529 8/7/2018 7:50:03 PM


2003 Q.1863 Under which Article of the Indian
Constitution did the President make a
Q.1860 The Ninth Schedule to the Indian Constitution reference to the Supreme Court to seek the
was added by Court’s opinion on the Constitutional validity
(a)  First Amendment of the Election Commission’s decision on
(b)  Eighth Amendment deferring the Gujarat Assembly Elections (in
(c)  Ninth Amendment the year 2002) ?
(d)  Forty Second Amendment (a)  Article 142 (b)  Article 143
Solution: (a) (c)  Article 144 (d)  Article 145
Ninth Schedule was added by First Solution: (b)
Amendment Act of 1951, which relates to
Q.1864 Which one of the following Articles of the
laws enacted ny state legislature in Land
Indian Constitution provides that ‘It shall be the
Reforms.
duty of the Union to protect every State against
Q.1861 Consider the following statements external aggression and internal disturbance’?
1. The joint sitting of the two houses of the (a)  Article 215 (b)  Article 275
Parliament in India is sanctioned under (c)  Article 325 (d)  Article 355
Article 108 of the Constitution Solution: (d)
2. The first joint sitting of Lok Sabha and According to the Article 355, it shall be the
Rajya Sabha was held in the year 1961 duty of the Union to protect every State
3. The second joint sitting of the two Houses against external aggression and internal
of Indian Parliament was held to pass the disturbance and to ensure that the government
Banking Service Commission (Repeal) of every State is carried on in accordance
Bill with the provisions of this Constitution.
Which of these statements is correct?
Q.1865 Match List I with List II and select the correct
(a)  1 and 2 (b)  2 and 3
answer:
(c)  1 and 3 (d)  1, 2 and 3
Solution: (d) List-I (Items in the List-II (Countries
So far, three joint sittings have been held. Indian Constitution) from which it was
They are derived)
1. on Dowry Prohibition Bill, 1961 A. Directive Principles of 1. Australia
2. on Banking Service Commission (Repeal) State Policy
Bill, 1978 B. Fundamental Rights 2. Canada
3. on Prevention of Terrorism Bill, 2002.
The presiding officer of joint sitting is C. Concurrent List in 3. Ireland
Speaker of the Lok Sabha. The joint sitting is Union-State Relations
governed by the rules and procedure of Lok D. India as a Union of 4. United Kingdom
Sabha. Hence Lok Sabha has an edge over States with greater
Rajya Sabha in the matter of joint-sitting. powers to the Union 5. United States of
Q.1862 Under which Article of the Indian America
Constitution did the President give his asset Codes:
to the ordinance on electoral reforms when (a)  A-5, B-4, C-1, D-2
it was sent back to him by the Union Cabinet (b)  A-3, B-5, C-2, D-1
without making any changes (in the year of (c)  A-5, B-4, C-2, D-1
2002) ? (d)  A-3, B-5, C-1, D-2
(a)  Article 121 (b)  Article 122 Solution: (d)
(c)  Article 123 (d)  Article 124 Q.1866 Which one of the following Bills must
Solution: (c) be passed by each House of the Indian
President can issue the ordinance under Parliamentary separately by special majority?
Article 123. (a)  Ordinary Bill
(b)  Money Bill

P.530 For Civil Services Preliminary Examination

05-Indian Polity_Q1507-2004.indd 530 8/7/2018 7:50:03 PM


(c)  Finance Bill In Rajya Sabha, 12 members are nominated
(d)  Constitution Amendment Bill by the President from the persons who have
Solution: (d) special knowledge in art, science, literature
According to Article 368 an amendment of and social service. In Lok Sabha, two
this Constitution may be initiated only by the members are nominated by the President
introduction of a Bill for the purpose in either from the Anglo-Indian community (Article
House of Parliament, and when the Bill is 331). A nominated member can vote only in
passed in each House by a majority of the total the Vice-Presidential elections.
membership of that House present and voting, Q.1870 As per Indian Protocol, who among the
it shall be presented to the President who following ranks highest in the order of
shall give his assent to the Bill and thereupon precedence?
the Constitution shall stand amended in (a)  Deputy Prime Minister
accordance with the terms of the Bill. (b)  Former Presidents
Q.1867 Which of the following Constitutional (c)  Governor of a State within his State
Amendments are related to raising the (d)  Speaker of the Lok Sabha
number of Members of Lok Sabha to be Solution: (c)
elected from the States? Q.1871 Consider the following statements:
(a)  6th and 22nd (b)  13th and 38th The function(s) of the Finance commission
(c)  7th and 31st (d)  11th and 42nd is/are
Solution: (c) 1. to allow the withdrawal of the money out
The seventh Amendment Act of 1956 of the Consolidated Fund of India
provides for composition of the House of 2. to allocate between the States the shares
the People and readjustment after every of proceeds of taxes
census. the 31st Amendment Act of 1973 3. to consider applications for grants-in-aid
provides for raising the upper limit for the from States
representation of states in the Lok Sabha 4. to supervise and report on whether the
from 500 to 525 and reducing the upper limit Union and State governments are levying
for the representation of UTs from 25 to 20. taxes in accordance with the budgetary
Q.1868 Which one of the following schedules of the provisions
Indian Constitution lists the names of states Which of these statements is/are correct?
and specifies their territories? (a)  Only 1 (b)  2 and 3
(a) First (b) Second (c)  3 and 4 (d)  1, 2 and 4
(c)  Third (d)  Fourth Solution: (b)
Solution: (a) Article 280 of the Constitution of India
Q.1869 Which one of the following statements is provides for a Finance Commission as a
correct? quasi-judicial body. It is constituted by the
(a) Only the Rajya Sabha and not the Lok President of India every fifth year or at such
Sabha can have nominated members earlier time as he considers necessary. The
(b) There is a constitutional provision for commission makes recommendations to
nominating two members belonging the president with regard to the distribution
to the Anglo-Indian community to the of the proceeds of taxes between the union
Rajya Sabha and the states. The principles should
(c) There is no constitutional bar for a govern the grants-in-aid to be given to the
nominated member to be appointed as a states. Any other matter referred to the
Union minister Commission by the President in the interest
(d) A nominated member can vote both in of sound finance
the Presidential and Vice Presidential Q.1872 Consider the following statements:
elections In the electoral college for Presidential
Solution: (c) Election in India,

Indian Polity Question Bank P.531

05-Indian Polity_Q1507-2004.indd 531 8/7/2018 7:50:03 PM


1. the value of the vote of an elected Member Committee on Parliamentary Affairs, which,
of Legislative Assembly equals State inter alia recommends prorogation of both
Population ×100 divided by Number of the Houses of the Parliament, Government’s
Elected MLAs of the State stand on Private Members’ Bills and
2. the value of the vote of an elected Member Resolutions. Option 3 is not correct.
of Parliament equals to total value of the The parliamentary committees work
votes of all elected MLA’s divided by total under supervision of the presiding officer of
number of elected MP’s the house.
3. there were more than 5000 members in Q.1874 Survey of India is under the ministry of:
the latest elections. (a) Defence
Which of these statements is/are correct? (b)  Environment and Forests
(a)  1 and 2 (b)  Only 2 (c)  Home Affairs
(c)  1 and 3 (d)  Only 3 (d)  Science and Technology
Solution: (b) Solution: (d)
The value of a MP’s vote is calculated by Survey of India, The National Survey and
dividing the total value of all MLAs’ votes Mapping Organization of the country under
by the number of MPs. the Department of Science and Technology,
Q.1873 Consider the following statements: is the oldest scientific department of the
1. While members of the Rajya Sabha are Government of India. It was set up in 1767.
associated with Committees on Public Q.1875 Which one of the following statements is
Accounts and Public Undertakings, NOT correct?
members of Committee on Estimates are (a) The Press Council of India is an
drawn entirely from the Lok Sabha autonomous quasi-judicial body
2. The Ministry of Parliamentary Affairs established under an Act of the Parliament
works under the overall direction of (b) The Press Information Bureau provides
Cabinet Committee on Parliamentary accreditation to media person so as to
Affairs have easy access to information from
3. The Minister of Parliamentary Affairs government sources
nominates Members of Parliament (c) Among all the states of India,
on Committees, Councils, Board Maharashtra publishes the largest number
and Commissions etc. set up by the of newspaper
Government of India in the various (d) Press Trust of India is the largest news
ministries. agency in the country
Which of these statements are correct? Solution: (c)
(a)  1 and 2 (b)  2 and 3 Madhya Pradesh publishes largest number of
(c)  1 and 3 (d)  1, 2 and 3 newspapers.
Solution: (a)
Q.1876 The power to enlarge the jurisdiction of
The PAC is formed every year with a strength
the Supreme Court of India with respect to
of not more than 22 members of which 15
any matter included in the Union List of
are from Lok Sabha, the lower house of the
Legislative Powers rests with:
Parliament, and 7 from Rajya Sabha, the
(a)  The President of India
upper house of the Parliament. The term of
(b)  The Chief Justice of India
office of the members is one year.
(c)  The Parliament
The Estimates Committee, constituted
(d) The Union Ministry of Law, Justice and
for the first time in 1950, is a Parliamentary
Company Affairs
Committee consisting of 30 Members,
Solution: (c)
elected every year by the Lok Sabha from
amongst its Members. Q.1877 Which one of the following High Courts has
The Ministry of Parliamentary Affairs the Territorial Jurisdiction over Andaman and
renders secretarial assistance to the Cabinet Nicobar Islands?

P.532 For Civil Services Preliminary Examination

05-Indian Polity_Q1507-2004.indd 532 8/7/2018 7:50:03 PM


(a)  Andhra Pradesh (b)  Kolkata • Article 257 in the Constitution states that
(c) Chennai (d) Orissa the executive power of every State shall be
Solution: (b) so exercised as not to impede or prejudice
The U.T. of Andaman and Nicobar Islands the exercise of the executive power of the
comes under the jurisdiction of high court of Union, and the executive power of the
Calcutta under its extended jurisdiction Act Union shall extend to the giving of such
1953. directions to a State as may appear to the
Government of India to be necessary for
2004 that purpose.
• Article 258 says that the power of the
Q.1878 Which one of the following statements Union to confer powers on the States in
correctly describes the Fourth Schedule of certain cases;
the Constitution of India? • Article 355 says that the duty of the
(a) It contains the scheme of the distribution Union to protect States against external
of powers between the Union and the aggression and internal disturbance;
States • Article 358 says that the suspension
(b) It contains the languages listed in the of provisions of Article 19 during
Constitution emergencies.
(c) It contains the provisions regarding the
Q.1881 Match List I (Articles of the Constitution of
administration of tribal areas
India) with List II (Provision) and select the
(d) It allocates seats in the Council of States
correct answer using the codes given below
Solution: (d)
the lists:
Q.1879 With reference to Indian Parliament, which
one of the following is not correct? List I List II
(a) The Appropriation Bill must be passed A. Article 14 1. The State shall not
by both the Houses of Parliament before discriminate against any
it can be enacted into law citizen on grounds only of
(b) No money shall be withdrawn from religion, race, caste, sex place
the Consolidated Fund of India except of birth or any of term.
under the appropriation made by the
B. Article 15 2. The State shall not deny
Appropriation Act
to any person equality
(c) Finance Bill is required for proposing new
before the law or the equal
taxes but no additional Bill/Act is required
protection of law within the
for making changes in the rates of taxes
territory of India.
which are already under operation.
(d) No Money Bill can be introduced except C. Article 16 3. ‘Untouchability’ is Abolished
on the recommendation of the President and its practice in any form is
Solution: (a) forbidden.
Appropriation Bill is a money bill. In case D. Article 17 4. 
There shall be equality
of money bill, RS has only recommendatory of opportunities for all
power and need not to be passed by Rajya citizens in matters relating to
Sabha. employment or appointment
Q.1880 Which one of the following Articles of the to any office under the State
Constitution of India says that the executive Codes:
power of every State shall be so exercised as (a)  A-2, B-4, C-1, D-3
not to impede or prejudice the exercise of the (b)  A-3, B-1, C-4, D-2
executive power of the Union? (c)  A-2, B-1, C-4, D-3
(a)  Article 257 (b)  Article 258 (d)  A-3, B-4, C-1, D-2
(c)  Article 355 (d)  Article 358 Solution: (c)
Solution: (a)

Indian Polity Question Bank P.533

05-Indian Polity_Q1507-2004.indd 533 8/7/2018 7:50:03 PM


Q.1882 Which Article of the Constitution of India House and until immediately before the
says, ‘No child below the age of fourteen first meeting of the House.
years shall the employed to work in any Which of the statements given above are
factory or mine or engaged in any other correct?
hazardous employment’? (a)  1 and 2 (b)  2 and 3
(a)  Article 24 (b)  Article 45 (c)  1 and 3 (d)  1, 2 and 3
(c)  Article 330 (d)  Article 368 Solution: (b)
Solution: (a) The options 2 and 3 are correct. But as to
Article 24 of the constitution states that, no option 1 there are special provisions. The
child below the age of fourteen years shall be Speaker shall determine the time when a
employed to work in any factory or mine or sitting of the House shall be adjourned sine
engaged in any other hazardous employment. die or to a particular day, or to an hour or part
Q.1883 Consider the following tasks: of the same day: provided that the Speaker
1. Superintendence, direction and conduct of may, if he thinks fit, call a sitting of the House
free and fair elections before the date or time to which it has been
2. Preparation of electoral rolls for all adjourned or at any time after the House
elections to the Parliament, state has been adjourned sine die. It is not the
Legislatures and the Office of the President.
President and the Vice-President Q.1885 Which one of the following statements is not
3. Giving recognition to political, parties correct?
and allotting election symbols to political (a)  In the Lok Sabha, a no-confidence motion
parties and individuals contesting the has to set out the grounds on which it is
election. based
4. Proclamation of final verdict in case of (b)  In the case of a no-confidence motion in
election disputes Lok Sabha, no conditions of admissibility
Which of the above are the functions of the have been laid down in the Rules
Election Commission of India? (c)  A motion of no-confidence once admitted,
(a)  1, 2 and 3 (b)  2, 3 and 4 has to be taken up within ten days of the
(c)  1 and 3 (d)  1, 2 and 4 leave being granted
Solution: (a) (d)  Rajya Sabha is not empowered to
The High Court (and not the Election entertain a motion of no-confidence
Commission) is the final authority to give a Solution: (a)
final verdict in case of election disputes. In In case of a No-confidence motion, there is no
the alternative special election benches may need to set out the grounds on which it is based.
be constituted in high courts and earmarked The no-confidence motion is introduced only
exclusively for the disposal of election in the Lok Sabha by the opposition and needs
petitions and disputes a support of not less than 50 members of
Q.1884 Consider the following statements. Lok Sabha for its introduction. Rule 198 of
1. The Speaker of Lok Sabha has the power the Lok Sabha specifies the procedure for a
to adjourn the House sine die but, on motion of no-confidence. Any member may
prorogation, it is only the President who give a written notice. The speaker shall read
can summon the House the motion of no-confidence in the House and
2. Unless sooner dissolved or there is an shall ask all those persons to rise who favours
extension of the term, there is an automatic that the motion be taken up. If there are 50
dissolution of the Lok Sabha by efflux of MPs in favour, the speaker allots a date for
time, at the end of the period of five years, discussing the motion.
even if no formal order of dissolution is Q.1886 The resolution for removing the Vice-
issued by the President President of India can be moved in the
3. The Speaker of Lok Sabha continues in (a)  Lok Sabha alone
office even after the dissolution of the (b)  Either House of Parliament

P.534 For Civil Services Preliminary Examination

05-Indian Polity_Q1507-2004.indd 534 8/7/2018 7:50:03 PM


(c)  Joint Sitting of Parliament Article 267. Disbursements from Public
(d)  Rajya Sabha alone Accounts of India are not subject to a Vote
Solution: (d) of Parliament.
The Article 67(b) in the Constitution of India But recently railway budget has been
states, a Vice President may be removed merged with the union budget.
from his office by a resolution of the council Q.1889 The Archaeological Survey of India is an
of States passed by a majority of all the then attached office of the Department/Ministry of
members of the council and agreed to by the (a) Culture
House of the People; but no resolution for the (b) Tourism
purpose of this clause shall be moved unless (c)  Science and Technology
at least fourteen days’ notice has been given (d)  Human Resource Development
of the intention to move the resolution. Solution: (a)
Q.1887 With reference to the Constitution of India, The Archaeological Survey of India
which one of the following pairs is not established in 1861 is a department of
correctly matched? the Government of India attached to the
(a)  Forests: Concurrent List Ministry of Culture. ASI is responsible for
(b)  Stock Exchange: Concurrent List archaeological studies and the preservation
(c)  Post Office Savings Bank: Union List of archaeological heritage of the country
(d)  Public Health: State List in accordance with the various acts of the
Solution: (b) Indian Parliament.
Stock Exchanges are listed in the Seventh Q.1890 Assertion (a): The Central Rural Sanitation
Schedule (Article 246) List I–Union List, Item Programme was launched in 1986 to improve
number 90 that reads, taxes other than stamp the quality of life of rural people in India.
duties on transactions in stock exchanges Reason (R): Rural sanitation is a subject
and futures markets. Forests-Concurrent in the Concurrent List in the Constitution of
List, 17A, Post Office Savings Bank–Union India.
List 3, Public health and sanitation; hospitals In the context of above two statements,
and dispensaries–State List 6. which one of the following is correct?
Q.1888 With reference to Indian public finance, (a) Both A and R are individually true and R is
consider the following statements the correct explanation of A.
1. Disbursements from Public Accounts (b) Both A and R are individually true but R
of India are subject to the Vote of the is not the correct explanation of A.
Parliament (c)  A is true but R is false.
2. The Indian Constitution provides for the (d)  A is false but R is true
establishment of a Consolidated Fund, a Solution: (c)
Public Account and a Contingency Fund Rural sanitation is not a subject in the
for each State Concurrent List. Public Health and Sanitation
3. Appropriations and disbursements under comes under the State List.
the Railway Budget are subject to the Q.1891 Consider the following statements:
same form of parliamentary control as 1. The highest criminal court of the district is
other appropriations and disbursements the Court of District and Session Judge
Which of the statements given above are 2. The District Judge are appointed by the
correct? Governor in consultation with the High
(a)  1 and 2 (b)  2 and 3 Courts
(c)  1 and 3 (d)  1, 2 and 3 3. A person to be eligible for appointment
Solution: (b) as a District Judge should be an advocate
The Indian Constitution provides for the or a pleader of seven years’ standing or
establishment of a Consolidated Fund and more, or an officer in judicial service of
a Public Account under Article 266 and the Union or the State
a Contingency Fund for each State under

Indian Polity Question Bank P.535

05-Indian Polity_Q1507-2004.indd 535 8/7/2018 7:50:03 PM


4. When the sessions’ judge awards a death Indian order of precedence, Judges of the
sentence, it must be confirmed by the Supreme Court – Rank 9
High Court before it is carried out • Deputy Chairman of Rajya Sabha – Rank
Which of the statements given above are 10
correct? • Attorney General of India – Rank 11
(a)  1 and 2 (b)  2, 3 and 4 • Members of Parliament – Rank 21
(c)  3 and 4 (d)  1, 2, 3 and 4
Solution: (d) 2005
These provisions are given under Article
Q.1894 Consider the following statements:
233-235 in the chapter of Subordinate Courts
1. Part IX of the Constitution of India
in the Constitution of India.
provisions for Panchyats and was inserted
Q.1892 According to the National Human Rights by the Constitution (Amendment) Act,
Commission Act, 1993, who amongst the 1992.
following can be its Chairman? 2. Part IX A of the Constitution of India
(a) Any serving Judge of the Supreme Court contains provisions for Municipalities and
(b) Any serving Judge of the High Court the Article 243Q envisages two types of
(c) Only a retired Chief Justice of India Municipalities a Municipal Council and a
(d) Only a retired Chief Justice of a High Municipal Corporation for every State.
Court Which of the statements given above is/are
Solution: (c) correct?
According to NHRC Act 1993, only a retired (a)  Only 1 (b)  Only 2
Chief Justice of India can become chairman (c)  Both 1 and 2 (d)  Neither 1 nor 2
of NHRC, appointed by President on the Solution: (a)
recommendation of a committee comprising Part IX and Eleventh Schedule were added
of Prime Minister, Speaker of Lok Sabha, by 73rd Constitutional Amendment Act, 1992
Home Minister, Leader of Opposition of both which contain provisions for Panchayats
Houses of Parliament and Deputy Chairman Part IX A and Twelfth Schedule were
of Rajya Sabha added by 74th Constitutional amendment
Q.1893 Which one of the following is the correct act, 1992 which contain provisions for
sequence in the descending order of Municipalities but Article 243 Q envisages
precedence in the warrant of precedence? three types of municipalities: Nagar
(a) Attorney General of India–Judges of Panchayats for a transitional area, Municipal
the Supreme Court–Members of the Council for smaller urban areas and Municipal
Parliament–Deputy Chairman of Rajya Corporation for larger urban areas.
Sabha Q.1895 Consider the following statements:
(b) Judges of the Supreme Court–Deputy 1. Article 371 A to 371 I were inserted in
Chairman of Rajya Sabha–Attorney the Constitution of India to meet regional
General of India–Members of the demands of Nagaland, Assam, Manipur,
Parliament Andhra Pradesh, Sikkim, Mizoram,
(c) Attorney General of India–Deputy Arunachal Pradesh and Goa.
Chairman of Rajya Sabha–Judges of the 2. Constitution of India and the United
Supreme Court– Members of Parliament States of America envisage a dual policy
(d) Judges of the Supreme Court–Attorney (The Union and the States) but a single
General of India–Deputy Chairman of citizenship.
Rajya Sabha– Members of Parliament 3. A naturalized citizen of India can never be
Solution: (b) deprived of his citizenship.
President comes first, Vice-President second, Which of the statements given above is/are
Prime Minister third and Governors of states correct?
with in their respective State comes fourth (a)  1, 2 and 3 (b)  1 and 3
in the Warrant of Precedence. According to (c)  3 only (d)  1 only

P.536 For Civil Services Preliminary Examination

05-Indian Polity_Q1507-2004.indd 536 8/7/2018 7:50:03 PM


Solution: (d) of the Ministry. None of the given options
Article 371 A to I deals with special provisions is correct.
to -Nagaland, Assam, Manipur, Andhra Q.1898 Who among the following was the chairman
Pradesh, Sikkim, Mizoram, Arunachal Pradesh, of the Union Constitution Committee of the
Goa and Karnataka. Karnataka is added via Constituent Assembly?
98th constitutional amendment act, 2013. (a)  B.R. Ambedkar
The Constitution of India envisages a (b)  J. B. Kripalani
single policy for both Union and the States. (c)  Jawaharlal Nehru
A naturalized citizen is one who acquires (d)  Alladi Krishnaswami Ayyar
citizenship either by Naturalization or Solution: (c)
by Registration. They can be deprived of
Q.1899 Consider the following:
citizenship if they acquired citizenship by
1. Disputes with mobile cellular companies
using fraudulent means.
2. Motor accident cases
Q.1896 Consider the following statements: 3. Pension cases
1. The Constitution of India has 40 parts. For which of the above are Lok Adalats held?
2. There are 390 Articles in the Constitution (a)  1 only (b)  1 and 2
of India in all. (c)  2 only (d)  1, 2 and 3
3. Ninth, Tenth, Eleventh and Twelfth Solution: (d)
Schedules were added to the Constitution The Lok Adalats are being held in all the
of India by the Constitution (Amendment) three areas.
Acts.
Q.1900 Consider the following statements
Which of the statements given above is/are
1. The Parliament cannot enlarge the
correct?
jurisdiction of the Supreme Court of
(a)  1 and 2 (b)  2 only
India as its jurisdiction is limited to that
(c)  3 only (d)  1, 2 and 3
conferred by the Constitution.
Solution: (c)
2. The officers and servants of the Supreme
The Constitution of India has 25 parts, 12
Court and High Courts are appointed
schedules and more than 444 articles at
by the concerned Chief Justice and the
present. In the original constitution, there
administrative expenses are charged on
were 22 parts, 8 schedules and 395 articles
the Consolidated fund of India.
• Ninth Schedule—1st Constitutional
Which of the statements given above is/are
Amendment Act, 1951.
correct?
• Tenth Schedule—52nd Constitutional
(a)  1 only (b)  2 only
Amendment Act, 1985.
(c)  Both 1 and 2 (d)  Neither 1 nor 2
• Eleventh Schedule—73rd Constitutional
Solution: (b)
Amendment Act, 1992.
The Statement (1) is not correct as according
• Twelfth Schedule—74th Constitutional
to Article 138(1) of the Constitution,
Amendment Act, 1992.
Parliament can enlarge the jurisdiction
Q.1897 Under which one of the Ministries of the and powers of the Supreme Court writ any
Government of India does the Food and of the matters in the Union List. Whereas
Nutrition Board work? Supereme Court’s writ jurisdiction to any
(a)  Ministry of Agriculture other matter can be enlarged by a special
(b) Ministry of Health and Family Welfare agreement between Government of India and
(c) Ministry of Human Resource government of the concerned State.
Development
Q.1901 Consider the following statements:
(d)  Ministry of Rural Development
1. Article 301 pertains to the Right to
(*) Food and Nutrition Board works
Property.
under Ministry of Women and Child
2. Right to Property is a legal right but not a
Development. It is a technical support
Fundamental Right.
wing under Child Development Bureau

Indian Polity Question Bank P.537

05-Indian Polity_Q1507-2004.indd 537 8/7/2018 7:50:03 PM


3. Article 300 A was inserted in the Solution: (a)
Constitutional Amendment. Statement 1 is correct as per provisions under
Which of the statement given above is/are Article 249.
correct? Statement 2 is incorrect as resolutions
(a)  2 only (b)  2 and 3 approving the proclamation of Emergency
(c)  1 and 3 (d)  1, 2 and 3 are passed by both Houses of Parliament (not
Solution: (a) only Lok Sabha) by special majority.
Article 301 pertains to Freedom of Trade, Q.1905 Consider the following statements:
Commerce and Intercourse. In the original 1. There is no provision in the Constitution
constitution right to property was a of India to encourage equal pay for equal
Fundamental Right under Article 19(1) (f). work for both men and women.
But 44th Amendment Act, 1978 omitted sub 2. The Constitution of India does not define
clause f, and inserted Article 300A to make backward classes.
right to property a legal right. The Government Which of the statement(s) given above is/are
at that time was Janta Party government. correct?
(a)  1 only (b)  2 only
2006 (c)  Both 1 and 2 (d)  Neither I nor 2
Solution: (b)
Q.1902 Which one among the following commission
Statement 1 is incorrect as equal pay for
was set up in pursuance of a definite provision
equal work for both men and women is
under an Article of the Constitution of India?
provided under Article 39d in Directive
(a)  University Grants Commission
Principles of the State Policy under Part IV
(b) National Human Rights Commission
of the Constitution.
(c)  Election Commission
(d)  Central Vigilance Commission Q.1906 Assertion (A): In India, every State has a
Solution: (c) High Court in its territory.
The superintendence, direction and control Reason (R): The Constitution of India
of elections are to be vested in an Election provides a High Court in each State.
Commission under Article 324. Codes:
(a) Both ‘A’ and ‘R’ are individually true and
Q.1903 Which one of the following subjects is under
‘R’ is the correct explanation of’ A’.
the Union List in the Seventh Schedule of the
(b) Both’ A’ and ‘R’ are individually true but’
Constitution of India ?
R’ is not the correct explanation of’ A’.
(a) Regulation of labour and safety in mines
(c)  ‘A’ is true but ‘R’ is false.
and oilfields
(d)  ‘A’ is false but ‘R’ is true.
(b) Agriculture
Solution: (d)
(c) Fisheries
Article 214 of the Constitution says that
(d)  Public Health
there shall be a High Court for each State.
Solution: (a)
Therefore, the reason (R) is correct. But for
Q.1904 Consider the following statements: 29 states and seven union territories we have
1. The Rajya Sabha alone has the power to only 24 High Courts.
declare that it would be in national interest Who was the Chief Justice of India
for the Parliament to legislate with respect when Public Interest Litigation (PIL) was
to a matter in the State List. introduced to the Indian judicial system?
2. Resolutions approving the proclamation (a)  M. Hidayatullah (b)  A. M. Ahmadi
of Emergency are passed only by the Lok (c)  A. S. Anand (d)  P. N. Bhagwati
Sabha. Solution: (d)
Which of the statement(s) given above is/are PN Bhagwati was CJI during July 1985–Dec
correct? 1986. During his tenure as as Chief Justice
(a)  1 only (b)  2 only India, the PIL was introduced to the Indian
(c)  Both 1 and 2 (d)  Neither 1 nor 2 judicial system.

P.538 For Civil Services Preliminary Examination

05-Indian Polity_Q1507-2004.indd 538 8/7/2018 7:50:03 PM


Q.1907 Consider the following statements: number of members in the Lok Sabha and the
1. A person who has held office as a total number of members of the Legislative
permanent Judge of a High Court cannot Assembly of that State, respectively?
plead or act in any court or before any (a) 91st (b) 93rd
authority in India except of the Supreme (c) 95th (d) 97th
Court. Solution: (a)
2. A person is not qualified for appointment Q.1910 Assertion (a): The Council of Ministers in
as a Judge of a High Court in India unless the Union of India is collectively responsible
he has for at least five years held a judicial both to the Lok Sabha and Rajya Sabha.
office in the territory of India. Reason (R): The Members of both
Which of the statement(s) given above is/are the Lok Sabha and the Rajya Sabha are
correct? eligible to be the Ministers of the Union
(a)  1 only (b)  2 only Government.
(c)  Both 1 and 2 (d)  Neither 1 nor 2 (a) Both A are R are true and R is the correct
Solution: (d) explanation of A
Statement 1 is incorrect because after (b) Both A and R are true but R is not a
retirement a permanent judge of High Court correctexplanation of A
shall not plead or act in a Court or before any (c)  A is true but R is false
authority in India, except the SC and a HC (d)  A is false but R is true
other than the HC in which he had held his Solution: (d)
office (Article 220). Assertion is false, because the Council of
Statement 2 is incorrect as according Ministers in the Union of India is collectively
to Article 217, a person is not qualified for responsible to the Lok Sabha only.
appointment as a judge of a High Court in
Q.1911 Who was the Speaker of the First Lok Sabha?
India unless he has for at least ten years held
(a)  Hukum Singh (b)  G.V. Mavalankar
a judicial office in the territory of India.
(c)  K. M. Munshi (d)  U.N. Dhebar
Q.1908 Consider the following statements: Solution: (b)
1. Free and compulsory education to the • G. V. Mavalankar (1952-56)
children of 6-14 years age-group by the • Hukum Singh (1962-67)
State by the seventy-sixth Amendment to K. M. Munshi and U. N. Dhebar were never
the Constitution of India. the Speakers of the Lok Sabha.
2. Sarva Shiksha Abhiyan seeks to provide
Q.1912 Consider the following statements in respect
computer education even in rural areas.
of financial emergency under Article 360 of
3. Education was included in the Concurrent
the Constitution of India:
List by the Forty-second Amendment,
1. A proclamation of financial emergency
1976 to the Constitution of India’.
issued shall cease to operate at the
Which of the statements given above are
expiration of two months, unless before
correct?
the expiration of that period it has been
(a)  1, 2 and 3 (b)  1 and 2
approved by the resolutions of both
(c)  2 and 3 (d)  1 and 3
Houses of Parliament.
Solution: (c)
2. If any proclamation of financial emergency
Statement 1 is incorrect as this provision was
is in operation, it is competent for the
added by 86th Amendment Act (not by 76th).
President of India to issue directions for
the reduction of salaries and allowances
2007 of all or any class of persons serving in
Q.1909 Which of the following Constitution connection with the affairs of the Union
Amendment Acts seeks that the size of the but excluding the Judges of Supreme
Councils of Ministers at the Centre and in a Court and the High Courts.
State must not exceed 15 per cent of the total

Indian Polity Question Bank P.539

05-Indian Polity_Q1507-2004.indd 539 8/7/2018 7:50:03 PM


Which of the statements given above is/are 2. After retirement from the office, a
correct? permanent judge of a High Court cannot
(a)  1 only (b)  2 only plead or act in any court or before any
(c)  Both 1 and 2 (d)  Neither 1 nor 2 authority in India.
Solution: (a) Which of the statements given above is/are
Statement 1 is correct as under Article 360, any correct?
Proclamation of Financial Emergency issued (a)  1 only (b)  2 only
shall cease to operate at the expiration of two (c)  Both 1 and 2 (d)  Neither 1 nor 2
months, unless before the expiration of that Solution: (a)
period it has been approved by the resolutions Statement 2 is incorrect because after
of both Houses of Parliament. If approved by retirement a permanent judge of High Court
both Houses, then it operates for 6 months. shall not plead or act in a Court or before any
Statement 2 is incorrect as it is excluding authority in India, except the SC and a HC
Judges of SC and High Courts; but under the other than the HC in which he had held his
provisions of effects of article 360, Judges of office (Article 220)
SC and HCs are included.
Q.1913 Consider the following statements: 2008
1. The Chairman of the Committee on Public
Q.1916 Which of the following is/are included in the
Accounts is appointed by the Speaker of
Directive Principles of the State Policy?
the Lok Sabha.
1. Prohibition of traffic in human beings and
2. The Committee on Public Accounts
forced labour
comprises Members of Lok Sabha,
2. Prohibition of consumption except for
Members of Rajya Sabha and few eminent
medicinal purposes of intoxicating drinks
persons of industry and trade.
and of other drugs which are injurious to
Which of the statements given above is/are
health
correct?
Select the correct answer using the code
(a)  1 only (b)  2 only
given below:
(c)  Both 1 and 2 (d)  Neither 1 nor 2
Code:
Solution: (a)
(a)  1 only (b)  2 only
statement 2 is incorrect as Public Accounts
(c)  Both 1 and 2 (d)  Neither 1 nor 2
Committee consists of 22 members: 15 from
Solution: (b)
Lok Sabha and 7 from Rajya Sabha.
Statement 1 is incorrect as it is a Fundamental
Q.1914 Consider the following statements: Right under article 23 of Part III of the
1. The nation-wide scheme of the National constitution.
Child Labour Projects (NCLP) is run by Statement 2 corresponds to Directive
the Union Ministry of Social Justice and Principles of State Policy under Article 47
Empowerment. under Part IV of the Constitution.
2. Gurupadswamy Committee dealt with the
Q.1917 Which Schedule of the Constitution of
issues of child labour.
India contains the special provisions for the
Which of the statements given above is/are
administration and control of Scheduled
correct?
Areas in several States?
(a)  1 only (b)  2 only
(a) Third (b) Fifth
(c)  Both 1 and 2 (d)  Neither 1 nor 2
(c) Seventh (d) Ninth
Solution: (b)
Solution: (b)
NCLP is run by the Union Ministry of Labour
Fifth schedule says about the control and
and Employment.
administration of scheduled areas in states other
Q.1915 Consider the following statements: than Tripura, Assam, Meghalaya and Mizoram.
1. The mode of removal of a Judge of a High Sixth Schedule says about the
Court in India is same as that of removal administration and control of tribal areas in
of a Judge of the Supreme Court.

P.540 For Civil Services Preliminary Examination

05-Indian Polity_Q1507-2004.indd 540 8/7/2018 7:50:03 PM


the state of Assam, Meghalaya, Mizoram and Solution: (b)
Tripura. Statement 2 is correct as per provisions given
Q.1918 Under which one of the following Constitution under Article 173.
Amendment Acts, four languages were added Statement 1 is incorrect as according to
to the list of languages under the Eighth article 170, the legislative assembly of each
Schedule of the Constitution of India, thereby state shall consist of not more than 500 and not
raising their number to 22? less than 60 members chosen by direct election
(a) Constitution (Ninetieth Amendment) Act from territorial constituencies in the state.
(b)  Constitution (Ninety-first Amendment)
Act 2009
(c) Constitution (Ninety-second Amendment)
Q.1921 With reference to the Union Government,
Act
consider the following statements
(d) Constitution (Ninety-third Amendment)
1. The Constitution of India provides that all
Act
Cabinet Ministers shall be compulsorily
Solution: (c)
be a sitting members of Lok Sabha only.
92nd Amendment Act 2003 has added Bodo,
2. The Union Cabinet Secretariat operates
Santhali, Maithali and Dogri languages in the
under the direction of the Ministry of
8th Schedule of the Constitution. Originally
Parliamentary Affairs.
there were 14 languages in the 8th Schedule.
Which of the statements given above is/are
• 21st Amendment – Sindhi
correct?
• 71st Amendment – Konkani, Manipuri
(a)  1 only (b)  2 only
and Nepali languages.
(c)  Both 1 and 2 (d)  Neither 1 nor 2
Q.1919 Who among the following have held the Solution: (d)
office of the Vice-President of India? Statement 1 is incorrect as members of RS
1. Mohammad Hidayatullah can become cabinet ministers.
2. Fakhruddin Ali Ahmed Statement 2 is incorrect as Cabinet
3. Neelam Sanjiva Reddy secretariat is under the direct charge of the
4. Shankar Dayal Sharma Prime Minister.
Select the correct answer using the code
Q.1922 Which one of the following Constitutional
given below:
Amendments states that the total number of
Codes:
Ministers, including the Prime Minister, in
(a)  1, 2, 3 and 4 (b)  1 and 4 only
the Council of Minister shall not exceed 15%
(c)  2 and 3 only (d)  3 and 4 only
of the total number of members of the House
Solution: (b)
of the People?
Mohd. Hidayatullah (1979-84);
(a) 90th (b) 91st
Shankar Dayal Sharma (1987-92)
(c) 92nd (d) 93rd
Q.1920 Consider the following statements: The Solution: (b)
Constitution of India provides that: The above provision has been added by 91st
1. the Legislative Assembly of each State Constitutional Amendment Act 2003.
shall consist of not more than 450
Q.1923 With reference to Union Government,
members chosen by direct election from
consider the following statements:
territorial constituencies in the State
1. The Ministries and Departments of the
2. a person shall not be qualified to be chosen
Government of India are created by
to fill a seat in the Legislative Assembly of
the Prime Minister on the advice of the
a State if he/ she is less than 25 years of
Cabinet Secretary.
age
2. Each of the ministries is assigned to a
Which of the statements given above is/are
Minister by the President of India on the
correct?
advice of the Prime Minister.
(a)  1 only (b)  2 only
(c)  Both 1 and 2 (d)  Neither 1 nor 2

Indian Polity Question Bank P.541

05-Indian Polity_Q1507-2004.indd 541 8/7/2018 7:50:03 PM


Which of the statements given above is/are 2. Number of Ministries at the Centre at
correct? present is 36.
(a)  1 only (b)  2 only Which of the statements given above is/are
(c)  Both 1 and 2 (d)  Neither 1 nor 2 correct?
Solution: (b) (a)  1 only (b)  2 only
According to Article 70, the Prime Minister (c)  Both 1 and 2 (d)  Neither 1 nor 2
shall be appointed by the President and the Solution: (a)
other Ministers shall be appointed by the The number of ministries at the Centre
President on the advice of the Prime Minister. can vary based on factors such as volume
Statement 1: Under Government of India of work, importance attached to different
rules, it is President of India who creates sectors, changes of orientation of policy etc.
ministries not Prime Minister. On 15 Aug, 1947, the number of ministries at
Q.1924 Consider the following statements: the centre was 18.
1. The Advocate General of a State in India As of march 2018 there are 52 ministries.
is appointed by the President of India upon Q.1927 If a Panchayat is dissolved, elections are to
the recommendation of the Governor of be held within:
the concerned State. (a)  1 month (b)  3 months
2. As provided in Civil Procedure Code, (c)  6 months (d)  1 year
High Courts have original, appellate and Solution: (c)
advisory jurisdiction at the State level. Duration of Panchayats is five year. Fresh
Which of the statements given above is/are election to constitute a Panchayat shall be
correct? completed before the expiry of its term; or in
(a)  1 only (b)  2 only case of dissolution before the expiry of a period
(c)  Both 1 and 2 (d)  Neither 1 nor 2 of 6 months from the date of its dissolution.
Solution: (d) Q.1928 In India, the first Municipal Corporation was
Statement 1 is incorrect as Advocate General set up in which one among the following?
of the state is appointed by the governor of (a) Calcutta (b) Madras
the State. He holds office during the pleasure (c) Bombay (d) Delhi
of governor. Solution: (b)
Statement 2 is incorrect as High Courts In 1688, the first Municipal Corporation of
have Original, Appellate and Writ jurisdiction India was set up in Madras.
(not advisory jurisdiction).
Q.1929 With reference to Lok Adalats, consider the
Q.1925 Consider the following statements: following statements:
1. Central Administrative Tribunal (CAT) 1. An award made by a Lok Adalat is
was set up during the Prime Ministership deemed to be a decree of a civil court and
of Lal Bahadur Shastri. no appeal lies against there to any court.
2. The Members of CAT are drawn from 2. Matrimonial/Family disputes are not
both judicial and administrative streams. covered under Lok Adalat.
Which of the statements given above is/are Which of the statements given above is/are
correct? correct?
(a)  1 only (b)  2 only (a)  1 only (b)  2 only
(c)  Both 1 and 2 (d)  Neither 1 nor 2 (c)  Both 1 and 2 (d)  Neither 1 nor 2
Solution: (b) Solution: (a)
Statement 1 is incorrect as CAT was set up in When statutory recognition had been given
1985 during the prime ministership of Rajiv to Lok Adalat, it was specifically provided
Gandhi. that the award passed by the Lok Adalat
Q.1926 With reference to Union Government, formulating the terms of compromise will
consider the following statements: have the force of decree of a court which can
1. Number of Ministries at the Centre on be executed as a civil court decree.
15th August 1947 was 18.

P.542 For Civil Services Preliminary Examination

05-Indian Polity_Q1507-2004.indd 542 8/7/2018 7:50:03 PM


2010 as a representative of the interests of the
consumers in general.
Q.1930 With reference to the Constitution of India, Solution: (c)
consider the following The District Forum entertains the complaints
1. Fundamental Rights where the value of goods or services does not
2. Fundamental Duties exceed rupees twenty lakhs.
3. Directive Principles of the State Policy
Q.1933 With reference to Lok Adalats, which of the
Which of the above provisions of the
following statements is correct?
Constitution of India is/are fulfilled by the
(a) Lok Adalats have the jurisdiction to settle
National Social Assistance Programme
matters at pre-litigating stage and not
launched by the government of India?
those matters pending before any court
(a)  1 only (b)  3 only
(b) Lok Adalats can deal with matters which
(c)  1 and 3 only (d)  1, 2 and 3
are civil and not criminal in nature.
Solution: (b)
(c) Every Lok Adalat consists of either
The National Social Assistance Programme
serving or retired judicial officers only
(NSAP) which came into effect from 15th
and not any other person.
August, 1995 represents a significant step
(d) None of the statements given above is
towards the fulfilment of the Directive
correct.
Principles in Article 41 (Right to Work, to
Solution: (d)
Education and to Public Assistance in certain
Cases that are pending in regular courts can
cases) of the Constitution.
be transferred to a Lok Adalat if both the
Q.1931 Consider the following statements. parties agree. These are usually presided over
The Supreme Court of India tenders advice to by retired judges, social Activists, or other
the President of India on matters of law or fact members of the legal profession. Lok Adalats
1. on its own initiative (on any matter of can deal with any matter falling within the
larger public interest). jurisdiction of civil, criminal etc.
2. if he seeks such an advice.
Q.1934 With reference to the United Nations
3. only if the matters relate to the
Convention on the Rights of the Child,
Fundamental Rights of the citizens.
consider the following:
Which of the statements given above is/are
1. The Rights of Development
correct ?
2. The Right to Expression
(a)  1 only (b)  2 only
3. The Right to Recreation
(c)  3 only (d)  1and 2
Which of the above is/are the Rights of the
Solution: (b)
child?
Article 143 discusses the Powers of President
(a)  1 only (b)  1 and 3
to consult with Supreme Court.
(c)  2 and 3 (d)  1, 2 and 3
Q.1932 With reference to the Consumer Disputes Solution: (d)
Redressal at district level in India, which one The Convention on the Rights of the Child
of the following statements is not correct ? (adopted on Nov. 20, 1989) is the first legally
(a) A State Government can establish more binding international instrument to incorporate
than one District Forum in a district if it the full range of human rights i.e. civil, cultural,
deems fit. economic, political and social rights.
(b) One of the members of the District
Forum shall be a woman 2011
(c) The District Forum entertains the
complaints where the value of goods or Q.1935 Under the Constitution of India, which one
services does not exceed rupees fifty lakhs. of the following is not a fundamental duty ?
(d) A complaint in relation to any goods sold (a)  To vote in public elections
or any service provided may be filed with (b)  To develop the scientific temper
a District Forum by the State Government (c)  To safeguard public property

Indian Polity Question Bank P.543

05-Indian Polity_Q1507-2004.indd 543 8/7/2018 7:50:03 PM


(d) To abide by the Constitution and respect for implementing Government sponsored
its ideals schemes in metropolitan area
Solution: (a) Q.1939 In India, if a religious sect/community is
To vote in public elections is not a given the status of a national minority, what
fundamental duty. special advantages it is entitled to?
Q.1936 The authorization for the withdrawal of funds 1. It can establish and administer exclusive
from the Consolidated Fund of India must educational institutions.
come from 2. The President of India automatically
(a)  The President of India nominates a representative of the
(b)  The Parliament of India community to Lok Sabha.
(c)  The Prime Minister of India 3. It can derive benefits from the Prime
(d)  The Union Finance Minister Minister’s 15-Point Programme.
Solution: (b) Which of the statements given above is/are
Parliament shall have power to authorise correct?
by law the withdrawal of moneys from the (a)  1 only (b)  2 and 3
Consolidated Fund of India for the purposes (c)  1 and 3 (d)  1, 2 and 3
for which the said grants are made Solution: (c)
Q.1937 The Constitution (Seventy-Third Amendment) Minority education institution has the power
Act, 1992, which aims at promoting the to reserve only upto 50% seats for students
Panchayati Raj Institutions in the country, belonging to its own community. (Supreme
provides for which of the following ? Court Judgement on St. Stephen’S College v.
1. Constitution of District Planning University Of Delhi, 1992 SCC 558).
Committees. Q.1940 India is home to lakhs of person with
2. State Election Commissions to conduct all disabilities. What are the benefits available to
panchayat elections. them under the law?
3. Establishment of State Finance 1. Free schooling till the age of 18 years in
Commission. government-run schools.
Select the correct answer using the codes 2. Preferential allotment of land for setting
given below: up business.
(a)  1 only (b)  1 and 2 only 3. Ramps in public buildings.
(c)  2 and 3 only (d)  1, 2 and 3 Which of the statements given above is/are
Solution: (c) correct?
District planning committee comes under (a)  1 only (b)  2 and 3
74th Amendment not in 73rd Amendment. (c)  1 and 3 (d)  1, 2 and 3
Q.1938 Consider the following statements: In India, Solution: (d)
a Metropolitan Planning Committee: Disable students come under the Right to
1. is constituted under the provisions of the Education Act and it provides for free and
Constitution of India. compulsory education to ‘children’ between
2. prepares the draft development plans for the ages six and 14 years, though under
metropolitan area. the Persons with Disability (PWD) Act,
3. has the sole responsibility for a child refers to a person up to the age of
implementing Government sponsored 18 years. So statement 1 is correct. Please
schemes in the metropolitan area. note that Persons with Disability (PWD)
Which of the statements given above is/are Act provides for ramps in public building;
correct? adaptation of toilets for wheel chair users;
(a)  1 and 2 only (b)  2 only Braille symbols and auditory signals in
(c)  1 and 3 only (d)  1, 2 and 3 elevators or lifts; ramps in hospitals, primary
Solution: (a) health centres and other medical care and
Article 243ZE: Committee on Metropolitan rehabilitation institutions. The same Act also
Planning does not have a sole responsibility states that appropriate Governments and

P.544 For Civil Services Preliminary Examination

05-Indian Polity_Q1507-2004.indd 544 8/7/2018 7:50:04 PM


local authorities shall by notification frame Q.1943 Which of the following special powers have
schemes in favour of persons with disabilities, been conferred on the Rajya Sabha by the
for the preferential allotment of land at Constitution of India?
concessional rates. (a) To change the existing territory of a State
Q.1941 Consider the following: and to change the name of a State
1. Right to education. (b) To pass a resolution empowering the
2. Right to equal access to public service. Parliament to make laws in the State List
3. Right to food. and to create one or more All India Services
Which of the above is/are Human Right/ (c) To amend the election procedure of the
Human Rights under “Universal Declaration President and to determine the pension of
of Human Rights’’ ? the President after his/her retirement
(a)  1 only (b)  1 and 2 (d) To determine the functions of the
(c)  3 only (d)  1, 2 and 3 Election Commission and to determine
Solution: (d) the number of Election Commissioners
Article 25 says everyone has the right Solution: (b)
to a standard of living adequate for the The Constitution of India empowering the
health and wellbeing of himself and of his Parliament to make laws in the State List and
family, including food, clothing, housing to create one or more All India Services.
and medical care and necessary social Q.1944 Which of the following provisions of the
services, and the right to security in the Constitution of India have a bearing on
event of unemployment, sickness, disability, Education?
widowhood, old age or other lack of livelihood 1. Directive Principles of State Policy
in circumstances beyond his control. Article 2. Rural and Urban Local Bodies
21(2) says that everyone has the right of 3. Fifth Schedule
equal access to public service in his country. 4. Sixth Schedule
Article 26(1) says that everyone has the right 5. Seventh Schedule
to education. Education shall be free, at least Select the correct answer using the codes
in the elementary and fundamental stages. given below.
(a)  1 and 2 only (b)  3, 4 and 5 only
2012 (c)  1, 2 and 5 only (d)  1, 2, 3, 4 and 5
Solution: (d)
Q.1942 Consider the following provisions under
the Directive Principles of State Policy as Q.1945 According to the Constitution of India, it is the
enshrined in the Constitution of India: duty of the President of India to cause to be laid
1. Securing for citizens of India a uniform before the Parliament which of the following?
civil code 1. The Recommendations of the Union
2. Organizing village Panchayats Finance Commission
3. Promoting cottage industries in rural 2. The Report of the Public Accounts
areas Committee
4. Securing for all the workers reasonable 3. The Report of the Comptroller and
leisure and cultural opportunities Auditor General
Which of the above are the Gandhian 4. The Report of the National Commission
Principles that are reflected in the Directive for Scheduled Castes
Principles of State Policy? Select the correct answer using the codes
(a)  1, 2 and 4 only (b)  2 and 3 only given below.
(c)  1, 3 and 4 only (d)  1, 2, 3 and 4 (a)  1 only (b)  2 and 4 only
Solution: (b) (c)  1, 3 and 4 only (d)  1, 2, 3 and 4
Organizing village Panchayats and promoting Solution: (c)
cottage industries in rural areas are the It is not the duty of the President of India to
Gandhian principles that are reflected in the cause to be laid report of public Accounts
Directive Principles of State Policy. Committee before the Parliament.

Indian Polity Question Bank P.545

05-Indian Polity_Q1507-2004.indd 545 8/7/2018 7:50:04 PM


Q.1946 Which of the following is/are among the (iv) The discussion an adjournment motion
Fundamental Duties of citizens laid down in should last for not less than two hours
the Indian Constitution? and thirty minutes
1. To preserve the rich heritage of our Q.1949 Consider the following statements:
composite culture 1. Union Territories are not represented in
2. To protect the weaker sections from social the Rajya Sabha.
injustice 2. It is within the purview of the Chief
3. To develop the scientific temper and spirit Election Commissioner to adjudicate the
of inquiry election disputes.
4. To strive towards excellence in all spheres 3. According to the Constitution of India, the
of individual and collective activity Parliament consists of the Lok Sabha and
Select the correct answer using the codes the Rajya Sabha only.
given below: Which of the statements given above is/are
(a)  1 and 2 only (b)  2 only correct?
(c)  1, 3 and 4 only (d)  1, 2, 3 and 4 (a)  1 only (b)  2 and 3
Solution: (c) (c)  1 and 3 (d)  None
Q.1947 The distribution of powers between the Centre Solution: (d)
and the States in the Indian Constitution is None of the above statements are correct
based on the scheme provided in the (i) Union Territories (Delhi and Pondicherry)
(a)  Morley-Minto Reforms, 1909 are represented in the Rajya Sabha.
(b)  Montagu-Chelmsford Act, 1919 (ii) It is not within the purview of the Chief
(c)  Government of India Act, 1935 Election Commissioner to adjudicate
(d)  Indian Independence Act, 1947 election disputes. It is the Supreme
Solution: (c) Court and High Court which look into
Distribution of power between the Centre the disputes.
and the States in the Indian Constitution is (iii) According to the Constitution of India, the
based on the Government of India Act. 1935. Parliament consists of the Lok Sabha the
Q.1948 In the Parliament of India, the purpose of an Rajya Sabha and the President of India.
adjournment motion is Q.1950 Regarding the office of the Lok Sabha
(a)  to allow a discussion on a definite matter Speaker, consider the following statements:
of urgent public importance 1. He/She holds the office during the
(b)  to let opposition members collect pleasure of the President.
information from the ministers 2. He/She need not be a member of the House
(c)  to allow a reduction of specific amount in at the time of his/her election but has to
demand for grant become a member of the House within six
(d) to postpone the proceedings to check the months from the date of his/her election.
inappropriate or violent behaviour on the 3. If he/she intends to resign, the letter of his/
part of some members her resignation has to be addressed to the
Solution: (a) Deputy Speaker.
Adjournment motion Which of the statements given above is /are
(i) It is introduced in the Parliament to draw correct?
attention of the house to a definite matter (a)  1 and 2 only (b)  3 only
of urgent public importance and needed (c)  1, 2 and 3 (d)  None
to support of 50 members to be admitted. Solution: (b)
(ii) It is regarded as an extraordinary device, If the Lok Sabha Speaker wants to resign,
because it interrupts the normal business the letter of his/her resignation has to be
of the house. addressed to the Deputy Speaker.
(iii) It involves an element of censure against Q.1951 Which of the following can be said to
the government and Rajya Sabha is not be essentially the parts of ‘Inclusive
permitted to make use this device. Governance’?

P.546 For Civil Services Preliminary Examination

05-Indian Polity_Q1507-2004.indd 546 8/7/2018 7:50:04 PM


1. Permitting the Non-Banking Financial become a member of one of the Houses
Companies to do banking within six months
2. Establishing effective District Planning (b) needs not necessarily be a member of
Committees in all the districts one of the Houses of the Parliament but
3. Increasing the government spending on must become a member of the Lok Sabha
public health within six months
4. Strengthening the Mid-day Meal Scheme (c) must be a member of one of the Houses
Select the correct answer using the codes of the Parliament
given below: (d) must be a member of the Lok Sabha
(a)  1 and 2 only (b)  3 and 4 only Solution: (a)
(c)  2, 3 and 4 only (d)  1, 2, 3 and 4 Article 75(1) of the Indian Constitution
Solution: (c) provides that the Prime Minister shall be
Q.1952 In India, other than ensuring that public appointed by the President. The Constitution
funds are used efficiently and for intended permits a person to be appointed Prime
purpose, what is the importance of the office Minister without his\her being a member of
of the Comptroller and Auditor General either House of the Parliament at the time of
(CAG)? appointment. Before expiry of this time, he
1. CAG exercises exchequer control on has to become a member of either the Rajya
behalf of the Parliament when the Sabha or the Lok Sabha.
President of India declares national Q.1954 A deadlock between the Lok Sabha and the
emergency/financial emergency. Rajya Sabha calls for a joint sitting of the
2. CAG reports on the execution of projects Parliament during the passage of
or programmes by the ministries are 1. Ordinary Legislation
discussed by the Public Accounts 2. Money Bill
Committee. 3. Constitution Amendment Bill
3. Information from CAG reports can be Select the correct answer using the codes
used by investigating agencies to frame given below:
charges against those who have violated (a)  1 only (b)  2 and 3 only
the law while managing public finances. (c)  1 and 3 only (d)  1, 2 and 3
4. While dealing with the audit and Solution: (a)
accounting of government companies, Q.1955 In the areas covered under the Panchayat
CAG has certain judicial powers for (Extension to the Scheduled Areas) Act,
prosecuting those who violate the law. 1996, what is the role/ power of Gram Sabha?
Which of the statements given above is/are 1. Gram Sabha has the power to prevent
correct? alienation of land in the Scheduled Areas.
(a)  1, 3 and 4 only (b)  2 only 2. Gram Sabha has the ownership of minor
(c)  2 and 3 only (d)  1, 2, 3 and 4 forest produce.
Solution: (c) 3. Recommendation of Gram Sabha is
Only 2nd and 3rd are correct statements. required for granting prospecting licence
(2) CAG reports on execution of projects or mining lease for any mineral in the
or programmes by the ministries are discussed Scheduled Areas.
by the Public Accounts Committee. Which of the statements given above is/are
(3) Information from CAG reports can correct?
be used by investigating agencies to press (a)  1 only (b)  1 and 2 only
charges against those who have violated the (c)  2 and 3 only (d)  1, 2 and 3
law while managing public finance. Solution: (d)
Q.1953 The Prime Minister of India, at the time of According to Panchayat Extension to the
his/her appointment Scheduled Areas) Act 1996, Gram Sabha has
(a) needs not necessarily be a member of one the power to prevent alienation of land in the
of the Houses of the Parliament but must Scheduled Areas, has the ownership of minor

Indian Polity Question Bank P.547

05-Indian Polity_Q1507-2004.indd 547 8/7/2018 7:50:04 PM


forest produce and the recommendation 4. All appointments of officers and staffs of
of Gram Sabha is required for granting the Supreme Court of India are made by
prospecting licence or mining lease for any the Government only after consulting the
mineral in the Scheduled Areas. Chief Justice of India.
Q.1956 Which of the following are included in the Which of the statements given above is/are
original jurisdiction of the Supreme Court? correct?
1. A dispute between the Government of (a)  1 and 3 (b)  3 and 4
India and one or more States (c)  4 only (d)  1, 2, 3 and 4
2. A dispute regarding elections to either Solution: (a)
House of the Parliament or that of Statement 1: Judges are appointed by the
Legislature of a State president of india in consultation with the
3. A dispute between the Government of chief justice of india. Chief justice has
India and a Union Territory to consult four most senior judges of the
4. A dispute between two or more States supreme court.
Select the correct answer using the codes Statement 2: Judges of Supreme Court
given below: including chief justice of india can be removed
(a)  1 and 2 (b)  2 and 3 by the president on the recommendation
(c)  1 and 4 (d)  3 and 4 of the Parliament. The parliament address
Solution: (c) must be supported by a special majority of
The Original Jurisdiction of the Supreme each House of Parliament (i.e., a majority
Court includes dispute between the of the total membership of that House and
Government of India and one or more States, a majority of not less than two-thirds of the
and dispute between two and more States. members of that House present and voting).
The Judges Enquiry Act (1968) regulates the
Q.1957 With reference to the Delimitation Commission,
procedure for removal.
consider the following statements:
Statement 3: It’s true.
1. The orders of the Delimitation
Statement 4: Officer and staff of Supreme
Commission cannot be challenged in a
Court are appointed by the Chief justice of
Court of Law.
india.
2. When the orders of the Delimitation
Commission are laid before the Lok Q.1959 With reference to consumers’ rights/
Sabha or State Legislative Assembly, they privileges under the provisions of law in
cannot effect any modifications in the India, which of the following statements is/
orders. are correct?
Which of the statements given above is/are 1. Consumers are empowered to take
correct? samples for food testing.
(a)  1 only (b)  2 only 2. When a consumer files a complaint in any
(c)  Both 1 and 2 (d)  Neither 1 nor 2 consumer forum, no fee is required to be
Solution: (c) paid.
Both are correct. Hence the option (c) is right. 3. In case of death of a consumer, his/
her legal heir can file a complaint in the
Q.1958 What is the provision to safeguard the
consumer forum on his/her behalf.
autonomy of the Supreme Court of India
Select the correct answer using the codes
1. While appointing the Supreme Court
given below:
Judges, the President of India has to
(a)  1 only (b)  2 and 3
consult the Chief Justice of India.
(c)  1 and 3 (d)  1, 2 and 3
2. The Supreme Court Judges can be
Solution: (c)
removed by the Chief Justice of India
Consumers are empowered to take samples
only.
for food testing. In case of death of a consumer
3. The salaries of the Judges are charged on
his/her legal heir can file a complaint in the
the Consolidated Fund of India to which
consumer forum on his /her behalf.
the legislature does not have to vote.

P.548 For Civil Services Preliminary Examination

05-Indian Polity_Q1507-2004.indd 548 8/7/2018 7:50:04 PM


2013 1. National Development Council
2. Planning Commission
Q.1960 According to the Constitution of India, which 3. Zonal Councils
of the following are fundamental for the Select the correct answer using the codes
governance of the country? given below.
(a)  Fundamental Rights (a)  1 and 2 only (b)  2 only
(b)  Fundamental Duties (c)  1 and 3 only (d)  1, 2 and 3
(c)  Directive Principles of State Policy Solution: (d)
(d) Fundamental Rights and Fundamental National Development council is not
Duties a constitutional body. It is an extra-
Solution: (c) constitutional body.
Directive Principles of State Policy are Planning Commission is a non-
guidelines to the central and State government constitutional and non-statutory body. It was
of India to be kept in mind while framing laws created by the Government of India in 1950
and policies. The DPSPs aim to create social by a resolution.
and economic conditions under which the The zonal councils have been recognized
citizens can lead a good life. They also aim in the 74th Constitutional Amendment Act of
to establish social and economic democracy 1992.
through a welfare state. They act as a check on
Q.1963 Consider the following statements:
the government. It is a yardstick in the hands
1. National Development Council is an
of the people to measure the performance of
organ of the Planning Commission.
the government. It shall be the duty of the state
2. The Economic and Social Planning is kept
to apply these principles in making laws.
in the Concurrent List in the Constitution
Q.1961 Consider the following statements: of India.
1. An amendment to the Constitution of 3. The Constitution of India prescribes that
India can be initiated by an introduction Panchayats should be assigned the task
of a bill in the Lok Sabha only. of preparation of plans for economic
2. If such an amendment seeks to make development and social justice.
changes in the federal character of the Which of the statements given above is/are
Constitution, the amendment also requires correct?
to be ratified by the legislature of all the (a)  1 only (b)  2 and 3 only
States of India. (c)  1 and 3 only (d)  1, 2 and 3
Which of the statements given above is/are Solution: (b)
correct? Out of 52 items on the concurrent list,
(a)  1 only (b)  2 only Economic and Social Planning is placed
(c)  Both 1 and 2 (d)  Neither 1 nor 2 under Article 40 of the Constitution.
Solution: (d) Directive Principles of State Policy lays
Statement 1: it can be introduced by an MP in down that the State shall take steps to
either of the two houses. organize village panchayats and endow them
Statement 2: An amendment to the with such powers and authority as may be
constitution of India is introduced as a bill necessary to enable them to function as units
in the Parliament. It then must be approved of self government. Planning for economic
by both the houses of Parliament. The development and social justice is one such
amendments must then be ratified by the power given to village panchayats.
legislatures of at least one half of the states
Q.1964 What will follow if a Money Bill is
(not all the states). Once all these stages are
substantially amended by the Rajya Sabha?
complete the amendment is bound to receive
(a) The Lok Sabha may still proceed
the assent of the President of India.
with the Bill, accepting or not
Q.1962 Which of the following bodies does not/do accepting the recommendations of the
not find mention in the Constitution? Rajya Sabha

Indian Polity Question Bank P.549

05-Indian Polity_Q1507-2004.indd 549 8/7/2018 7:50:04 PM


(b) The Lok Sabha cannot consider the Which of the statements given above is/are
bill  further correct?
(c) The Lok Sabha may send the Bill to the (a)  1 only (b)  2 and 4
Rajya Sabha for reconsideration (c)  1, 2 and 3 (d)  1 and 3 only
(d) The President may call a joint sitting for Solution: (c)
passing the Bill The Attorney General of India has a post
Solution: (a) parallel to any minister in Parliament. He can
When a money bill returns to the Lok take part in the proceedings of either house.
Sabha with amendments made by the Rajya He can be a member of any committee of
Sabha, it is open to Lok Sabha to accept or Parliament. He has the right to speak in the
to reject any or all of the recommendations. Parliament but he has no right to vote.
When the Lok Sabha chooses to accept or Q.1967 The Parliament can make any law for
decline the money bill with or without the whole or any part of India for implementing
recommendation, the money bill is deemed international treaties
passed in both houses. (a)  with the consent of all the States
Q.1965 Which one of the following statements is (b) with the consent of the majority of States
correct? (c) with the consent of the States concerned
(a) In India, the same person cannot be (d)  without the consent of any State
appointed as Governor for two or more Solution: (d)
States at the same time Parliament has exclusive power to make law
(b) The Judges of the High Court of the with respect to any of the matters enumerated
States in India are appointed by the with the Union List. According to entry no
Governor of the State just as the Judges 14 in the Union List it reads “entering into
of Supreme Court are appointed by the treaties and agreements with foreign countries
President and implementing of treaties, agreement and
(c) No procedure has been laid down in the convention with foreign countries”.
Constitution of India for the removal of a Q.1968 In the context of India, which of the following
Governor from his/her post principles is/are implied institutionally in the
(d) In the case of a Union Territory having parliamentary government?
a legislative setup, the Chief Minister 1. Members of the Cabinet are Members of
is appointed by the Lt. Governor on the the Parliament.
basis of majority support 2. Ministers hold the office till they enjoy
Solution: (d) confidence in the Parliament.
A lieutenant Governor is in charge of a 3. Cabinet is headed by the Head of the State.
Union Territory whereas a Governor is in Select the correct answer using the codes
charge of a State. The rank of Lt. Governor is given below.
present only in the states of Delhi, Andaman (a)  1 and 2 only (b)  3 only
and Nicobar Islands and Puducherry. So (c)  2 and 3 only (d)  1, 2 and 3
in the case of a Union Territory specified Solution: (a)
where there is a legislative setup, the Chief Minister/ministers can be removed by issuing
Minister is appointed by the Lieutenant no confidence motion in the parliament.
Governor. All cabinet members are mandated by the
Q.1966 Consider the following statements: Attorney constitution to be the member of either house
General of India can of the parliament of India. Cabinet is headed
1. take part in the proceedings of the Lok by the cabinet secretary not by the Head of
Sabha the State.
2. be a member of a committee of the Lok Q.1969 Consider the following statements:
Sabha 1. The Council of Ministers in the Centre
3. speak in the Lok Sabha shall be collectively responsible to the
4. vote in the Lok Sabha Parliament.

P.550 For Civil Services Preliminary Examination

05-Indian Polity_Q1507-2004.indd 550 8/7/2018 7:50:04 PM


2. The Union Ministers shall hold the office Solution: (d)
during the pleasure of the President of Section C of the Forest Dwellers Act
India. provides a transparent three step procedure
3. The Prime Minister shall communicate for deciding on who gets rights.
to the President about the proposals for • the Gram Sabha makes a recommendation,
legislation. i.e., who has been cultivating land for
Which of the statements given above is/are how long, which minor forest produce
correct? is collected; etc. The Gram Sabha plays
(a)  1 only (b)  2 and 3 only this role because it is a public body where
(c)  1 and 3 only (d)  1, 2 and 3 all people participate and hence is fully
Solution: (c) democratic and transparent.
The Council of Ministers shall be collectively • The Gram Sabha’s recommendation
responsible to the parliament; the Prime goes through two stages of screening
minister shall communicate to the president committees—the Taluka and the District
about the proposals for legislation but the levels.
union. If a President were to dismiss the Council Q.1972 The Government enacted the Panchayat
of Ministers on his or her own initiative, it Extension to Scheduled Areas (PESA) Act
might trigger a constitutional crisis. Thus, in in 1996. Which one of the following is not
practice, the Council of Ministers cannot be identified as its objective?]
dismissed as long as it commands the support (a)  To provide self-governance
of a majority in the Lok Sabha. (b)  To recognize traditional rights
Q.1970 Consider the following statements: (c) To create autonomous regions in tribal
1. The Chairman and the Deputy Chairman areas
of the Rajya Sabha are not the members (d) To free tribal people from exploitation
of that House. Solution: (d)
2. While the nominated members of the two PESA Act does not identify the freedom
Houses of the Parliament have no voting of tribal people from exploitation as its
right in the presidential election, they objectives, but it automatically becomes a
have the right to vote in the election of the by-product of its objectives.
Vice President. Q.1973 Consider the following statements: The
Which of the statements given above is/are parliamentary Committee on public accounts
correct? 1. consists of not more than 25 Members of
(a)  1 only (b)  2 only the Lok Sabha
(c)  Both 1 and 2 (d)  Neither 1 nor 2 2. scrutinizes appropriation and finance
Solution: (b) accounts of the Government
The nominated members of the Rajya Sabha 3. examines the report of the Comptroller
have the right to vote in the election of the and Auditor General of India.
Vice President so far none from them has Which of the statements given above is/are
been inducted in the Council of Ministers. correct?
Q.1971 Under the Scheduled Tribes and Other (a)  1 only (b)  2 and 3 only
Traditional Forest Dwellers (Recognition (c)  3 only (d)  1, 2 and 3
of Forest Rights) Act, 2006 who shall Solution: (b)
be the authority to initiate the process The committee consists of 15 members of
for determining the nature and extent of Lok Sabha not 25 members. The function
individual or community forest rights or both? of the committee is to examine the accounts
(a)  State Forest Department showing the appropriation of the sums granted
(b) District Collector/Deputy Commissioner by Parliament to meet the expenditure of the
(c)  Tahsildar/Block Development Officer/ government of India and such other accounts
Mandal Revenue Officer laid before the House as the committee
(d)  Gram Sabha may think fit. Apart from the Reports of the

Indian Polity Question Bank P.551

05-Indian Polity_Q1507-2004.indd 551 8/7/2018 7:50:04 PM


Comptroller and Auditor General of India on which the citizens can lead a good life. They
Appropriation also aim to establish social and economic
Accounts of the Union Government, the democracy through a welfare state.
Committee also examines the various Audit
Reports of the Comptroller and Auditor General 2014
on revenue receipts, expenditure by various
Ministries/Departments of Government and Q.1976 Which one of the following Schedules of
accounts of autonomous bodies. the Constitution of India contains provisions
regarding anti-defection?
Q.1974 With reference to National Legal Services
(a)  Second Schedule (b)  Fifth Schedule
Authority, consider the following statements:
(c)  Eighth Schedule (d)  Tenth Schedule
1. Its objective is to provide free and
Solution: (d)
competent legal services to the weaker
The 52nd Amendment act of 1985, added the
sections of the society on the basis of
Tenth Schedule to the Constitution. This is
equal opportunity.
often referred to as anti-defection law.
2. It issues guidelines for the State Legal
Services Authorities to implement Q.1977 In the Constitution of India, promotion of
the legal programmes and schemes international peace and security is included
throughout the country. in the
Which of the statements given above is/are (a)  Preamble to the Constitution
correct? (b)  Directive Principles of State Policy
(a)  1 only (b)  2 only (c)  Fundamental Duties
(c)  Both 1 and 2 (d)  Neither 1 nor 2 (d)  Ninth Schedule
Solution: (c) Solution: (b)
The National Legal Services Authority Promotion of international peace and security
(NALSA) has been constituted under the is included in the Directive Principles of State
Legal Services Authorities Act, 1987 to as Article 51 of constitution that mentions to
provide free Legal Services to the weaker promote international peace and security and
sections of the society and to organize Lok maintain just an honourable relation between
Adalats for amicable settlement of disputes. nations between nations; to foster respect
In every state, State Legal Services for international law and treaty obligations,
Authority has been constituted to give effect and to encourage settlements of international
to the policies and directions of the NALSA disputes by arbitration.
and to give free legal services to the people Q.1978 Consider the following statements:
and conduct Lok Adalats in the State. 1. The President shall make rules for the
Q.1975 ‘Economic Justice’ as one of the objectives more convenient transaction of the
of the Indian Constitution has been business of the Government of India, and
provided in for the allocation among Ministers of the
(a) the Preamble and the Fundamental said business.
Rights 2. All executive actions of the Government
(b) the Preamble and the Directive Principles of India shall be expressed to be taken in
of State Policy the name of the Prime Minister.
(c) the Fundamental Rights and the Directive Which of the statements given above is/are
Principles of State Policy correct?
(d)  None of the above (a)  1 only (b)  2 only
Solution: (b) (c)  Both 1 and 2 (d)  Neither 1 nor 2
The Preamble to the Constitution of India Solution: (a)
in its introductory statement says- “Justice- Clause (3) of Article 77 “Conduct of
Social, Economic and Political” and the Business of the Government of India” of the
Directive Principles of state policies aim to Constitution of India lays down as follows:
create social and economic condition under “The President shall make rules for the more

P.552 For Civil Services Preliminary Examination

05-Indian Polity_Q1507-2004.indd 552 8/7/2018 7:50:04 PM


convenient transaction of the business of the 4. The Union Ministry of Urban Development
Government of India, and for the allocation 5. The Parliament
among Ministers of the said business. The Select the correct answer using the code
Constitution of India mentions that, “All given below.
executive action of the Government of India (a)  1, 2 and 5 only (b)  1, 3 and 4 only
shall be expressed to be taken in the name (c)  2 and 5 only (d)  1, 2, 3, 4 and 5
of the President.” Therefore, only option (a) Solution: (c)
is correct. Finance commission is involved in
Q.1979 The power of the Supreme Court of India to distribution of taxes and grants. It is not
decide disputes between the Centre and the involved in planning. So by just eliminating all
States falls under its options involving statement 1, we get answer
(a)  advisory jurisdiction (c) correct. Now, the National Development
(b)  appellate jurisdiction Council (NDC) has been abolished.
(c)  original jurisdiction Q.1983 Which of the following is/are the function/
(d)  writ jurisdiction functions of the Cabinet Secretariat?
Solution: (c) 1. Preparation of agenda for Cabinet
Q.1980 The power to increase the number of judges Meetings
in the Supreme Court of India is vested in 2. Secretarial assistance to Cabinet
(a)  the President of India Committees
(b)  the Parliament 3. Allocation of financial resources to the
(c)  the Chief Justice of India Ministries
(d)  the Law Commission Select the correct answer using the code
Solution: (b) given below.
It is the Parliament which has the power to (a)  1 only (b)  2 and 3 only
increase the number of judges in the Supreme (c)  1 and 2 only (d)  1, 2 and 3
Court of India. Parliament increased the Solution: (c)
number of judges from the original eight in The functions of the Cabinet Secretariat are
1950 to eleven in 1956, fourteen in 1960, preparation of agenda for Cabinet Meetings &
eighteen in 1978, twenty-six in 1986 and Secretarial assistance to Cabinet Committees.
thirty-one in 2008. However Allocation of financial resources to
the Ministries as per the provisions in budget
Q.1981 Which one of the following is the largest
is prepared by finance ministry.
Committee of the Parliament?
(a)  The Committee on Public Accounts Q.1984 Consider the following statements.
(b)  The Committee on Estimates A Constitutional Government is one which
(c) The Committee on Public Undertakings 1. places effective restrictions on individual
(d)  The Committee on Petitions liberty in the interest of State Authority
Solution: (b) 2. places effective restrictions on the
The largest Committee is the committee of Authority of the State in the interest of
Estimates, given its 30 members individual liberty
• Public Accounts 22 Which of the statements given above is/are
• Estimates 30 correct?
• Public Undertakings 22 (a)  1 only (b)  2 only
• Petitions Lok Sabha (15), Rajya Sabha(10) (c)  Both 1 and 2 (d)  Neither 1 nor 2
Solution: (c)
Q.1982 Which of the following are associated with
A constitutional Government needs to
‘Planning’ in India
balance between individual liberty vis-à-vis
1. The Finance Commission
State Authority.
2. The National Development Council
3. The Union Ministry of Rural Q.1985 The fundamental object of Panchayati
Development Raj system is to ensure which among the
following?

Indian Polity Question Bank P.553

05-Indian Polity_Q1507-2004.indd 553 8/7/2018 7:50:04 PM


1. People’s participation in development Solution: (a)
2. Political accountability The Fifth Schedule of the Constitution of
3. Democratic decentralization India deals with administration and control
4. Financial mobilization of Scheduled areas and Scheduled Tribes in
Select the correct answer using the code these areas.
given below. The Sixth Schedule to the Constitution
(a)  1, 2 and 3 only (b)  2 and 4 only of India contains provisions concerning the
(c)  1 and 3 only (d)  1, 2, 3 and 4 administration of tribal areas in the States of
Solution: (a) Assam, Meghalaya, Tripura and Mizoram.
The fundamental object of Panchayati Raj Q.1987 Who/Which of the following is the custodian
system is to ensure people’s participation in of the Constitution of India?
development, political accountability, and (a)  The President of India
democratic decentralization. (b)  The Prime Minister of India
Financial mobilization is the social (c)  The Lok Sabha Secretariat
objective. (d)  The Supreme court of India
Q.1986 Which of the following are the discretionary Solution: (d)
powers given to the Governor of a State? The Supreme Court of India is the apex court
1. Sending a report to the President of India in India. As stated by the Indian Constitution,
for imposing the President’s rule the function of the Supreme Court of India
2. Appointing the Ministers is that of a custodian of the Constitution, a
3. Reserving certain bills passed by the court established by the authority of a federal
State Legislature for consideration of the government, and the uppermost court of
President of India appeal.
4. Making the rules to conduct the business Q.1988 Consider the following statements:
of the State Government 1 The Legislative Council of a State in
Select the correct answer using the code India can be larger in size than half of the
given below. Legislative Assembly of that particular
(a)  1 and 2 only (b)  1 and 3 only State
(c)  2, 3 and 4 only (d)  1, 2, 3 and 4 2. The Governor of a State nominates the
Solution: (b) Chairman of Legislative Council of that
The Governor has Constitutional discretion in particular State.
cases of Reservation of bill for consideration Which of the statements given above is/are
of the President and Recommendation of correct?
the imposition of President’s rule.Therefore, (a)  1 only (b)  2 only
Statements 1 and 3 are definitely right. (c)  Both 1 and 2 (d)  Neither 1 nor 2
Moreover Governor only appoints those Solution: (d)
persons as ministers who are recommended Statement 1: The maximum strength of the
by the Chief Minister. So Governor doesn’t legislative council is fixed at one third of the
have “Discretion” in appointment of the total strength of the legislative assembly and
minister.Therefore the Statement 2 is wrong. the minimum strength is fixed at 40
Statement 2: The chairman of the
2015 legislative council is elected by the council
The provisions in Fifth Sixth Schedules in itself from amongst its members.
the Constitution of India are made in order to Q.1989 “To uphold and protect the Sovereignty,
(a) protect the interests of Scheduled Tribes Unity and Integrity of India” is a provision
(b) determine the boundaries between States made in the
(c) determine the powers, authority and (a)  Preamble of the Constitution
responsibilities of Panchayats (b)  Directive Principles of State Policy
(d) protect the interests of all the border (c)  Fundamental Rights
States (d)  Fundamental Duties

P.554 For Civil Services Preliminary Examination

05-Indian Polity_Q1507-2004.indd 554 8/7/2018 7:50:04 PM


Solution: (d) the country, making it the duty of the State
The idea behind incorporation of to apply these principles in making laws to
fundamental duties was to remind the establish a just society in the count.
citizens of the country that they have certain Q.1992 With reference to the Cabinet Mission, which
obligations towards the country and society. of the following statements is/are correct?
The fundamental duties were added to 1. It recommended government a federal
the constitution on the recommendations government.
of the Swaran Singh Committee. There 2. It enlarged the powers of the Indian
were ten fundamental duties at the time of Courts.
incorporation but the eleventh was inserted 3. It provided for more Indians in the ICS.
by the 86th Amendment in 2002. To uphold Select the correct answer using the code
and protect the sovereignty, unity and given below.
integrity of India is one of them. (a)  1 only (b)  2 and 3
Q.1990 The ideal of Welfare State’ in the Indian (c)  1 and 3 (d)  None
Constitution is enshrined in its Solution: (a)
(a) Preamble Britain’s Prime Minister Clement Attlee was
(b)  Directive Principles of State Policy announced the dispatch of the Mission of
(c)  Fundamental Rights three Cabinet Ministers Pethick Lawrence,
(d)  Seventh Schedule Sir Stafford Cripps, and A. V. Alexander.
Solution: (b) According to the proposals of the Cabinet
Directive Principles of State Policy (DPSPs) Mission announced, the country could have a
aim to create social and economic conditions Federal government consisting of the Indian
under which the citizens can lead a good provinces and princely states.
life. They also aim to establish social and Q.1993 With reference to the Union Government,
economic democracy through a welfare state. consider the following statements:
The Directive Principles of State Policy is 1. The Department of Revenue is responsible
guidelines/principles given to the central for the preparation of Union Budget that is
and state governments of India, to be kept in presented to the Parliament.
mind while framing laws and policies. 2. No amount can be withdrawn from the
Q.1991 Consider the following statements regarding Consolidated Fund of India without the
the Directive Principles of State Policy: authorization from the Parliament of
1. The Principles spell out the socio- India.
economic democracy in the country. 3. All the disbursements made from Public
2. The provisions contained in these Account also need the authorization from
Principles are not enforceable by any the Parliament of India.
court. Which of the statements given above is / are
Which of the statements given above is / are correct?
correct? (a)  1 and 2 only (b)  2 and 3 only
(a)  1 only (b)  2 only (c)  2 only (d)  1, 2 and 3
(c)  Both 1 and 2 (d)  Neither Solution: (c)
Solution: (c) All revenues received by the Government
The directive principles ensure that the by way of taxes like Income Tax, Central
State shall strive to promote the welfare of Excise, Customs and other receipts flowing
the people by promoting a social order in to the Government in connection with
which social, economic and political justice the conduct of Government businesses,
is informed in all institutions of life. The i.e., Non-Tax Revenues are credited into
provisions of The Directive Principles of the Consolidated Fund constituted under
State Policy are not enforceable by any Article 266(1) of the Constitution of India.
court, but the principles laid down therein are No amount can be withdrawn from the Fund
considered fundamental in the governance of without authorization from the Parliament.

Indian Polity Question Bank P.555

05-Indian Polity_Q1507-2004.indd 555 8/7/2018 7:50:04 PM


Q.1994 There is a Parliamentary System of Q.1997 Consider the following statements
Government in India because the 1. The Executive Power of the Union of
(a) Lok Sabha is elected directly by the India is vested in the Prime Minister.
people 2. The Prime Minister is the ex officio
(b) Parliament can amend the Constitution Chairman of the Civil Services Board.
(c)  Rajya Sabha cannot be dissolved Which of the statements given above is / are
(d) Council of Ministers is responsible to correct?
the Lok Sabha (a)  1 only (b)  2 only
Solution: (d) (c)  Both 1 and 2 (d)  Neither 1 nor 2
The executive in a Parliamentary system Solution: (d)
is responsible to the legislature for all its The Executive Powers of the Union of India
actions. The ministers are answerable to the are vested in the President. The Cabinet
parliament and responsible to the Lok Sabha. Secretary (and not the Prime Minister)
The Council of Ministers remains in office as Q.1998 The fundamental object of Panchayati
long as they enjoy the support and confidence Raj system is to ensure which among the
of the Lok Sabha. following ?
Q.1995 Consider the following statements: 1 People’s participation in development
1. The Rajya Sabha has no power either to 2 Political accountability
reject or to amend a Money Bill. 3 Democratic decentralization
2. The Rajya Sabha cannot vote on the 4 Financial mobilization
Demands for Grants. Select the correct answer using the code
3. The Rajya Sabha cannot discuss the given below.
Annual Financial Statement. (a)  1, 2 and 3 only (b)  2 and 4 only
Which of the statements given above is / are (c)  1 and 3 only (d)  1, 2, 3 and 4
correct? Solution: (a)
(a)  1 only (b)  1 and 2 only The fundamental object to Panchayati Raj
(c)  2 and 3 only (d)  1, 2 and 3 system is to ensure people’s participation in
Solution: (b) development, political accountability and
A Money Bill cannot be introduced in Rajya democratic decentralization
Sabha. Rajya Sabha has no power either to
reject or amend a Money Bill. It can only 2016
make recommendations on the Money Bill.
It has no power to vote on the Demands for Q.1999 Consider the following statements
Grants 1. The minimum age prescribed for any
person to be a member of Panchayat is
Q.1996 When a bill is referred to a joint sitting of
25 years.
both the Houses of the Parliament, it has to
2. A Panchayat reconstituted after premature
be passed by
dissolution continues only for the
(a) a simple majority of members present
remainder period.
and voting
Which of the statements given above is/are
(b) three-fourths majority of members
correct?
present and voting
(a)  1 only (b)  2 only
(c)  two-thirds majority of the Houses
(c)  Both 1 and 2 (d)  Neither 1 nor 2
(d)  absolute majority of the Houses
Solution: (b)
Solution: (a)
As of now, there is no general power with the
In India, if a bill has been rejected by any
state government to dissolve the Panchayats
house of the parliament and if more than
prior to expiry of the prescribed term of five
six months have elapsed, the President
years. The Panchayats could be dissolved
may summon a joint session for purpose of
before the expiry of five years only in
passing the bill. The bill is passed by a simple
accordance with the law, if any, promulgated
majority of a joint sitting.

P.556 For Civil Services Preliminary Examination

05-Indian Polity_Q1507-2004.indd 556 8/7/2018 7:50:04 PM


by the state. In case of dissolution of a Select the correct answer using the code
Panchayat, a fresh election was to be given below.
completed within a period of six months (a)  1 and 2 only (b)  2 and 3 only
from the date of dissolution and the newly (c)  1 and 3 only (d)  1, 2 and 3
elected members should hold office only for Solution: (a)
the remaining part of the term and not for the Prime minister chairs the meetings of the
full term of five years.  NGRBA.
Q.2000 The Parliament of India acquires the power Q.2003 Consider the following statements:
to legislate on any item in the State List in the 1. The Chief Secretary in a State is appointed
national interest if a resolution to that effect by the Governor of that State.
is passes by the 2. The Chief Secretary in a State has a fixed
(a) Lok Sabha by a simple majority of its tenure
total membership Which of the statements given above is/are
(b) Lok Sabha by a majority of not less than correct?
two-thirds of its total membership (a)  1 only (b)  2 only
(c) Rajya Sabha by a simple majority of its (c)  Both 1 and 2 (d)  Neither 1 nor 2
total membership Solution: (d)
(d) Rajya Sabha by a majority of not less Not appointed by the governor but by the
than two-thirds of its members present CM. Chief secretary has no fixed tenure.
and voting Q.2004 With reference to the ‘Gram Nyayalaya Act’,
Solution: (d) which of the following statements is/are
Q.2001 Which of the following statements is/ar correct?
correct? 1. As per the Act, Gram Nyayalayas can hear
1. A bill pending in the Lok Sabha lapses on only civil cases and not criminal cases
its prorogation. 2. The Act allows local social activists as
2. A bill pending in the Rajya Sabha, which mediators/reconciliators.
has not been passed by the Lok Sabha, Select the correct answer using the code
shall not lapse on dissolution of the given below:
Lok Sabha. (a)  1 only (b)  2 only
Select the correct answer using the code (c)  Both 1 and 2 (d)  Neither 1 nor 2
given below: Solution: (b)
(a)  1 only (b)  2 only The Gram Nyayalayas have both civil and
(c)  Both 1 and 2 (d)  Neither 1 nor 2 criminal jurisdiction.
Solution: (b) Social Workers at the village level having
Prorogation of a session does not affect the the integrity and necessary qualifications can
bills or any other business pending before the be appointed as mediators.
House.
Q.2002 Which of the following are the key features
of ‘National Ganga River Basin Authority
(NGRBA)’?
1. River basin is the unit of planning and
management.
2. It spearheads the river conservation efforts
at the national level.
3. One of the Chief Ministers of the State
through which the Ganga flows becomes
the Chairman of NGRBA on rotation
basis.

Indian Polity Question Bank P.557

05-Indian Polity_Q1507-2004.indd 557 8/7/2018 7:50:04 PM


INDIAN SOCIETY 2. Hindi Language has been made the
national language of the country.
Q.2005 Consider the following statements: 3. The State intervenes in the matters of
1. Every social difference leads to social religion in order to ensure equality within
divisions. religious communities.
2. Socially homogenous societies do not 4. Enactment of Mahatma Gandhi National
have economic differences. Rural Employment Guarantee Act
3. Multi-cultural countries necessarily have (MGNREGA)
social divisions. Select the correct answer using the codes
Choose the correct answer using the codes below
below: (a)  1 and 4 (b)  3 and 4
(a)  1 and 2 (b)  2 and 3 (c)  1, 2 and 4 (d)  1, 2, 3 and 4
(c)  1 only (d)  None of the above Solution: (b)
Solution: (d) There are various kinds of social differences
Social differences unite and well as divide. that can take the form of social divisions and
It depends on how a society reacts to it. If inequalities. These are social differences
there is mutual respect and recognition of the based on gender, religion, caste, wealth.
uniqueness of each social class, it leads to Women’s reservation Bill has not yet been
greater cohesion and harmony. passed by the parliament. Hence it cannot
Multi-cultural countries may or may be considered as a step to reduce social
not have social divisions. In societies difference. Our Constitution did not give
where social differences tend to match the status of national language to any one
with economic or other stark differences, language. Hindi was identified as the official
experience social divisions. For e.g. blacks language only and not National language.
in America were socially different as well Hindi as a national language will only
as economically very weak. And hence were create more social difference in the society.
discriminated against. Constitution allows the state to intervene
Q.2006 Social Marginalisation is considered to be a in thematters of religion in order to ensure
bane to Indian Society. Which of following equality within religious communities. For
are regarded as the basis for marginalisation? example, it bans untouchability. Enactment of
1. Language 2. Customs Mahatma Gandhi National Rural Employment
3. Religion 4.  Social Status Guarantee Act (MGNREGA) would not
Which of the statements given above is/are only reduce the economic inequality but also
correct? social inequality
(a)  1 and 3 (b)  1 and 4
(c)  1, 3 and 4 (d)  1, 2, 3 and 4 Tribes
Solution: (d)
Marginalisation can be because group of Q.2008 Consider the following statements
people or communities who speak a different 1. Adivasis are not a homogeneous population
language, follow different customs, low 2. Adivasi societies are also most distinctive
social status or belong to a different religious because there is often very little hierarchy
group from the majority community. Also among them.
Economic, social, cultural and political Which of the statements above is/are correct?
factors work together to make certain groups (a)  1 Only (b)  2 Only
in society feel marginalised. (c) Both (d) None
Solution: (c)
Q.2007 Which of the following are the measures
taken by Government to reduce social Q.2009 In the Indian context the term De-notified
differences in the society? tribes’ refers to
1. The government has provided for 33% (a)  tribes which are aboriginals
reservation for women in the Parliament. (b)  nomadic tribes

P.558 For Civil Services Preliminary Examination

06-Indian Polity_Q2005-ENDD.indd 558 8/7/2018 7:51:31 PM


(c)  tribes practising shifting cultivation Statement 2: It allows tribals to use
(d) tribes which were earlier classified as minor forest produce and safeguard forest
criminal tribes resources.
Solution: (d) Statement 3: The concerned rights have
Denotified Tribes (DNTs), also known as been covered in last few tests.
vimukta jat, are the tribes that were originally Statement 4: Autonomous regions have
listed under the Criminal Tribes Act of been covered in sixth schedule.
1871, as Criminal Tribes and “addicted to Fifth schedule has special provisions for
the systematic commission of non-bailable tribal administration.
offences.” Once a tribe became “notified” Q.2011 Tribal groups are at different stages of social,
as criminal, all its members were required economic and educational development.
to register with the local magistrate, failing Particularly Vulnerable Tribal Groups
which they would be charged with a crime (PVTGs) being one of them. Which of the
under the Indian Penal Code. The Criminal following is NOT a necessary characteristic
Tribes Act of 1952 repealed the notification, of a PVTG?
i.e. ‘de-notified’ the tribal communities. (a)  Stagnant or declining population
This act, however, was replaced by a series (b)  Pre-agriculture level of technology
of Habitual Offenders Acts that asked police (c)  Absence of social hierarchies
to investigate a suspect’s criminal tendencies (d)  Extremely low literacy
and whether his occupation is “conducive to Solution: (c)
settled way of life.” The denotified tribes were Learning: Tribal communities live,
reclassified as habitual offenders in 1959. in various ecological and geo-climatic
The creation of these categories should conditions ranging from plains and forests to
be seen in the context of colonialism. The hills and inaccessible areas.
British authorities listed them separately by While some tribal communities have
creating a category of castes or tribes labelled adopted a mainstream way of life, at the
as criminal. other end of the spectrum, there are certain
The name “Criminal Tribes” is itself a Scheduled Tribes, 75 in number known
misnomer as no definition of tribe denotes as Particularly Vulnerable Tribal Groups
occupation, but they were identified as tribes (PVTGs), who are characterised by:
doing their primary occupation. The first (a)  pre-agriculture level of technology;
Census was in 1871 and at that time there (b)  stagnant or declining population;
was neither consensus nor any definition of (c)  extremely low literacy
“tribe”. The terms “tribe” and “caste” were (d)  subsistence level of economy.
used interchangeably for these communities.
Q.2012 These are Particularly Vulnerable Tribal
Q.2010 Tribal rights in India are granted and Groups (PTG) residing in the Andaman &
protected by Nicobar Islands (A&N). The government
1. Fundamental Rights had launched a specific policy notification in
2. Forest Rights Act 2015 for protection of these tribes. It can be
3. The Panchayats (Extension to the which of the following?
Scheduled Areas) Act (PESA) (a) Koyas (b) Malmis
4. Fifth and Sixth Schedule of the (c)  Shom pens (d)  Aminidivis
Constitution Solution: (c)
Select the correct answer using the codes Learning: What makes the Shom pens
below. distinct from the four other PVTGs of
(a)  1, 3 and 4 only (b)  2 and 3 only Andaman and Nicobar Islands—Jarawas,
(c)  1 and 4 only (d)  1, 2, 3 and 4 Great Andamanese, Onges and Sentinelese—
Solution: (d) is that they are the only tribe in the region
Justification: Statement 1: Article 14, 15, with Mongoloid features. The other PVTGs
16, 19, 25-30 protect tribal rights. have Negroid features.

Indian Polity Question Bank P.559

06-Indian Polity_Q2005-ENDD.indd 559 8/7/2018 7:51:31 PM


Different groups of Shom pens have read more here: http://tribal.nic.in/Contents.
developed different levels of symbiotic aspx?mo=7&li=65)
relationship—particularly a barter system (About Jarawas recently in news:
with the Great Nicobarese who are coastal http://pib.nic.in/newsite/PrintRelease.
dwellers and categorised as a Scheduled Tribe aspx?relid=142204)
and others who have settled on the island. Q.2014 Consider the following matches of tribes
• Pandanus (a tropical plant found in with the state they are found in.
the islands), whose fruits resemble the 1. Baiga: TamilNadu
woody pineapple, is the staple food of the 2. Bhot: Kerala
Shompens. 3. Chakma: Tripura
• Marriage by capturing women from Select the correct answer using the codes
different groups and sub-groups is one of below.
the customs of the Shompen society. (a)  1 and 3 only (b)  3 only
Q.2013 Consider the following matches of (c)  2 only (d)  None of the above
Particularly Vulnerable Tribal Groups Solution: (b)
(PTGS) with the State/UT they are found in. Justification: Statement 1: They are found
1. Birhor: Bihar mainly in MP, UP, Chhattisgarh, Jharkhand.
2. Totos: Rajasthan In a bid to undo historical injustice meted
3. Shom pens and Jarawa: Andaman and out to primitive tribal communities living
Nicobar Islands in central India, the government of Madhya
Select the correct answer using the codes Pradesh has for the first time recognised the
below. habitat rights of seven villages in Dindori
(a)  1 and 2 only (b)  1 and 3 only district, mostly inhabited by the Baigas. In a
(c)  2 and 3 only (d)  1, 2 and 3 meeting held in village Rajni Sarai in 2016,
Solution: (b) the district administration told the villagers
Justification: Statement 1: A forest-dwelling they are free to access all their ancestral
society of about 4,000 people in 2001, who rights over land and forests.
live in central and eastern India in the states Statement 2: are groups of ethno-
of Orissa, Chhattisgarh, West Bengal, and linguistically related Tibetan people living in
Jharkhand with the greatest concentration the Trans-Himalayan region of the SAARC
in the Ranchi, Hazaribagh, and Singhbhum countries. In India, they are found mainly in
districts of Jharkhand. Several recent reports HP, Uttarakhand, Uttar Pradesh, West Bengal,
have indicated that the population may have Sikkim, Arunachal Pradesh and Tripura.
dropped to 4,000 people. Statement 3: It is an ethnic group
Statement 2: The Toto is a primitive and concentrated in the Chittagong Hill Tracts of
isolated tribal group residing only in a small Bangladesh. Today, the geographic distribution
enclave called Totopara in the Jalpaiguri of of Chakmas is spread across Bangladesh and
West Bengal, India. Totopara is located at the parts of northeastern India in Tripura mainly.
foot of the Himalayas just to the south of the Q.2015 Consider the following about Saharia tribes.
borderline between Bhutan and West Bengal 1. They are migratory tribes and originally
(on the western bank of Torsa river). Read belong to West Asia.
more here: 2. They can be found living in Central India.
http://wbnorthbengaldev.gov.in/HtmlPage/ 3. They do not follow any religion and
toto.aspx) worship only natural forces.
Statement 3: The Shom pen live a nomadic Select the correct answer using the codes
life moving from one habitation to another in below.
search of fruit and hunt that is till now found (a)  1 and 2 only (b)  2 and 3 only
abundantly and sufficiently in the thick rain (c)  3 only (d)  2 only
forests of the Great Nicobar Island. You can Solution: (d)

P.560 For Civil Services Preliminary Examination

06-Indian Polity_Q2005-ENDD.indd 560 8/7/2018 7:51:31 PM


Justification: Statements 1 and 2: Sahariya and is sought-after for construction, although
is an indigenous Mundas peaking tribe in not well suited to planning and polishing. The
the Madhya Pradesh province of India. The wood is especially suitable for constructing
Saharias are mainly found in the districts frames for doors and windows. The dry leaves
of Morena, Sheopur, Bhind, Gwalior, of sal are a major source for the production of
Datia, Shivpuri, Vidisha and Guna districts leaf plates called as patravali and leaf bowls
of Madhya Pradesh and Baran district of in northern and eastern India. The leaves are
Rajasthan. They are a Particularly Vulnerable also used fresh to serve ready-made paan
Tribal Groups (PTGS). (betel nut preparations) and small snacks
Statement 3: Some sources indicate that such as boiled black grams, gol gappa, etc.
they take part in Hindu Festivals, and worship The used leaves/plates are readily eaten by
Hindu Gods and Goddesses. goats and cattle that roam the streets freely.
Q.2016 The Khonds ate fruits and roots collected The tree has therefore protected northern
from the forest and cooked food with the oil India from a flood of Styrofoam and plastic
they extracted from the seeds of the Sal and plates that would have caused tremendous
Mahuwa. Consider the following statements. pollution. In South India, fresh plantain and
1. The flowers of Mahuwa are used to prepare banana leaves are used instead.
alcoholic drink in central parts of India Sal tree resin, ṛla in Sanskrit, is used as
2. The dry leaves of sal are a major source an astringent in Ayurvedic medicine. It is also
for the production of leaf plates called as burned as incense in Hindu ceremonies, and
Patravali and leaf bowls in northern and sal seeds and fruit are a source of lamp oil
eastern India and vegetable fat. The Shorea Robusta seed
Which of the above statements is/are correct? oil is extracted from the seeds and used as
(a)  1 Only (b)  2 Only cooking oil after refining.
(c) Both (d) None Q.2017 Consider the following statements about a
Solution: (c) tribal group.
The mahuwa flower is edible and is a food 1. They live mainly in Odisha,
item for tribals. They are used to make syrup 2. Their native language is Kui, a Dravidian
for medicinal purposes. language.
They are also fermented to produce the 3. They generally settle in higher altitudes
alcoholic drink mahuwa, a country liquor. due to their economic demands
Tribals of Bastar in Chhattisgarh and Orissa, To which tribal group do the above refer to?
Santhals of Santhal Paraganas (Jharkhand), (a) Kols (b) Gaddis
Koya tribals of North-East Andhra Pradesh (c) Badags (d) Khonds
and tribals of North Maharashtra consider Solution: (d)
the tree and the mahuwa drink as part Explanation: Kols are found in Madhya
of their cultural heritage. Mahuwa is an Pradesh. So, option (a)  is wrong.
essential drink for tribal men and women Gaddis are found in Himachal Pradesh.
during celebrations. The main ingredients So, option (b) is also wrong. Badagas are
used for making it are chhowa gud (granular found in Nilgiri, Tamil Nadu. So, the option
molasses) and dried mahuwa flowers. (c) is also wrong.
Mahuwa flowers are also used to Learning: One sub-group of Kondhas
manufacture jam, which is being made by is the Dongria Kondhas. Some Dongaria
tribal cooperatives in the Gadchiroli district Khonds live in Niyamgiri Hill located in
of Maharashtra. Kalahandi district of Orissa. Niyamgiri is
Sal is one of the most important sources of the sacred mountain of this community.
hardwood timber in India, with hard, coarse- Recently SC had banned Vedanta mining
grained wood that is light in colour when project based on the community’s resolutions
freshly cut, but becomes dark brown with against the project, under Articles 29-30 of
exposure. The wood is resinous and durable, the constitution.

Indian Polity Question Bank P.561

06-Indian Polity_Q2005-ENDD.indd 561 8/7/2018 7:51:31 PM


Dongaria Khonds have a subsistence had banned tourists from using this road.
economy based on foraging, hunting & These tribes are
gathering but they now primarily depend on a (a) Biyons (b) Mithoras
subsistence agriculture i.e. shifting cultivation (c) Nigors (d) Jarawas
or slash and burn cultivation or Podu. Solution: (d)
The Dongria family is often nuclear, Justification: Among the mentioned options,
although extended families are found. only the Jarawas live in Andaman, thus the
Q.2018 Which of the following tribes doesn‘t belong answer could have been easily eliminated.
to Arunachal Pradesh? India’s Supreme Court finally banned
1. Apatani 2.  Mishmi tourists from traveling along the Andaman
3. Wanchos 4.  Bhil Nicobar Trunk Road, a controversial highway
5. Kokna that was used for more a decade to conduct
Choose the correct answer using the codes dehumanizing “human safaris” on the Andaman
below and Nicobar Islands Union Territory of India.
(a)  4 and 5 Only (b)  3 and 4 Only The Court also ordered the closure of two
(c)  4 Only (d)  2, 4 and 5 Only major tourist attractions on the island, both
Solution: (a) of which are located inside the buffer zone:
http://arunachalpradesh.gov.in/aptribes.htm the limestone caves and the mud volcano on
http://www.indianetzone.com/9/kokna_tribe. Baratang Island.
html Q.2021 Consider the following matches of recently
Q.2019 Consider the following about the Karbi tribe. included tribes, in the Scheduled Tribes list,
1. The Karbis are known for their matrilineal with the state they are found in.
society. 1. Darlong: Tripura
2. These tribes largely follow Animism. 2. Kurivikkaran: Bihar
3. Some members of the tribe live in areas 3. Dhanuhar: Chattisgarh
administered under the Sixth Schedule of Which of the above is/are correct matches?
the constitution. (a)  1 and 3 only (b)  2 only
Select the correct answer using the codes (c)  2 and 3 only (d)  1 and 2 only
below. Solution: (a)
(a)  1 and 2 only (b)  2 and 3 only Justification: It was simple to eliminate
(c)  1 and 3 only (d)  1, 2 and 3 if you knew a tribe named “Kuruvikaran”
Solution: (b) should ideally belong to a Southern State.
Justification: Statement 1: The Karbis are Bihar is an eastern State, so 2 can’t be correct,
the patrilineal society and is composed of five which leaves the only answer as option (a).
major clans which are further sub-divided It is not important to know the details
into sub-clans. about the individual tribes.
Statement 2: Animism is the worldview Learning: First list of Scheduled Tribes in
that non-human entities—such as animals, relation to a State or Union Territory is to be
plants, and inanimate objects—possess a issued by a notified Order of the President after
spiritual essence having consultation with the State Government
Statement 3: The Karbis are the principal concerned. Any subsequent inclusion in or
tribal community in the Karbi Anglong exclusion from the list of Scheduled Tribes can
district of Assam, a district administered as be effected through an Act of Parliament as
per the provisions of the Sixth Schedule of the envisaged under Clause (2) of Article 342.
Constitution of India, having an autonomous Tribes of following states are recently
district of their own since 1951. included
• Assam: Boro, Boro Kachari, Bodo, Bodo
Q.2020 The biggest threat to these tribes had come
Kachar Karbi (Mikir)
from the Great Andaman Trunk Road
• Jharkhand: Bhogta, Deshwari, Ganjhu,
passing through their forest homeland.
Dautalbandi (Dwalbandi)
Sometime back in 2013, the Supreme Court

P.562 For Civil Services Preliminary Examination

06-Indian Polity_Q2005-ENDD.indd 562 8/7/2018 7:51:31 PM


Patbandi, Raut, Maajhia, Khairi (Kheri) the North Cachar Hills & Karbi- Anglong
Puran district.
• Tamil Nadu: Malayali Gounderxi, Statement 3: The Konyak are a Naga people,
Narikoravan, Kurivikkaran and are recognised among other Naga by
• Chattisgarh: Bhuinya, Bhuiyan, their tattoos, which they have all over their
Bhuyaniv) Dhanuhar / face and hands; facial tattoos were earned
Dhanuwarv) Kisanvi) Saunra, Saonravii) for taking an enemy’s head. They are called
Dhangad the land of Angh. They have the largest
• Tripura: Darlong population among the Nagas.
• Pondicherry: Irular (including Villi and Statement 4: The Mishmi or Deng people
Vettaikaran of Tibet and Arunachal Pradesh are an ethnic
Q.2022 Consider the following matches of tribes with group comprising mainly three tribes: Idu
the state they are found predominantly in. Mishmi (Idu Lhoba); Digaro tribe (Taraon,
1. Reang: Uttarakhand Darang Deng), and Miju Mishmi (Kaman
2. Dimasa: Assam Deng). The Mishmis occupy the northeastern
3. Konyak: Nagaland tip of the central Arunachal Pradesh in Upper
4. Mishmi: Meghalaya and Lower Dibang Valley, Lohit and Anjaw
Select the correct matches using the codes Districts/Medog County.
below.
(a)  1 and 2 only (b)  1, 3 and 4 only Women
(c)  3 only (d)  2 and 3 only Consider the following statements:
Solution: (d) 1. No Women had the right to vote in
Justification: Statement 1: Reang are one of elections in India before Independence.
the 21 scheduled tribes of the Indian state of 2. The first Woman suffrage movement
Tripura. in India dates back to the immediate
• The correct nomenclature for this ethnic aftermath of the Revolt of 1857.
group is actually Bru. Which of these is/are true?
• The Bru can be found mainly in the North (a)  1 only (b)  2 only
Tripura, Dhalai and the South Tripura (c)  Both 1 and 2 (d)  None
districts of Tripura state in India. Solution:(d)
• However, they may also be found Women suffrage movement dates back to the
in Mizoram, Assam, Manipur and First World War days (not the 1857 revolt).
Bangladesh. They speak the Reang dialect The Women’s Indian Association (WIA)  was
of Kokborok language which is of Tibeto- founded in 1917. It sought votes for women
Burmese origin and is locally referred to as and the right to hold legislative office on the
Kau Bru. same basis as men. These positions were
Statement 2: The Dimasa Kacharis: The endorsed by the main political groupings, the
Kacharis are the most widely spread tribe in Indian National Congress and the All-India
northeast India. They are said to be the earliest Muslim League. British and Indian feminists
inhabitants of the Brahmaputra Valley. combined in 1918 to publish a magazine Stri
• The Kacharis belong to the Indo- Dharma that featured international news
Mongoloid (Kirata)  group which include from a feminist perspective. In 1919 in the
the Bodos and their allied tribes. Montagu–Chelmsford Reforms, the British
• They have prominent Mongoloid features set up provincial legislatures which had the
with high cheek bones, slit eyes and a power to grant women’s suffrage. Madras in
slight growth of hair in the body and scant 1921 granted votes to wealthy and educated
beard. women, under the same terms that applied to
• They call themselves Bodo or Bodo-fisa men. The other provinces followed, but not
in the Brahmaputra valley and Dimasa or the princely states (which did not have votes
Dima-fisa or ‘sons of the great river’ in for men either).

Indian Polity Question Bank P.563

06-Indian Polity_Q2005-ENDD.indd 563 8/7/2018 7:51:31 PM


Q.2023 Women related laws concerning which of the refers to a set of social and behavioural
following exist in India? norms that, within a specific culture, are
1. Domestic violence widely considered to be socially appropriate
2. Sexual harassment at workplace for individuals of a specific sex. Socially
3. Maternity benefits accepted gender roles differ widely between
Choose the correct answer using the codes different cultures. Proponents of gender role
below: theory assert that observed gender differences
(a)  1 and 2 (b)  2 and 3 in behaviour and personality characteristics
(c)  1 and 3 (d)  All of the above are, at least in part, socially constructed,
Solution:(d) and therefore, the product of socialization
The Prevention of Sexual harassment at experiences; this contrasts with other models
Workplace Act, 2013 was recently enacted. of gender that assert that gender differences
Domestic violence act is in place since 2005. are “essential” to biological sex. Gender
Q.2024 The dignity of women is protected or secured differences exist in almost all societies.
by which of the following in the Indian With differences in the norms adopted, this
constitution? suggests that gender differences are, at least
1. Fundamental Rights partly, influenced by culture.
2. Fundamental Duties Q.2026 Women in India enjoy reservation in which
3. Directive Principles of State Policy of the following constitutional/statutory/
Select the correct answer using the codes executive bodies?
below 1. National Commission for Women
(a)  Only 2 (b)  1 and 2 2. National Commission for Protection of
(c)  1 and 3 (d)  All of the above Child Rights
Solution: (d)  Article 51A says that it is the 3. Child Welfare Committees
fundamental duty of every individual to 4. Zila panchayat
renounce practices derogatory to the dignity Choose the correct answer using the codes
of women. Also, the Fundamental rights and below:
DPSP contain a number of provisions which (a)  All of the above (b)  1, 3 and 4
promote a society where dignity of every (c)  1 and 4 only (d)  2 and 3 only
individual is respected, including women. Solution:(b)
The word fraternity means the dignity of the National Commission for Women composition
individual (including women) and the unity (a) A Chairperson, committed to the cause of
and integrity of the nation as a whole. women, to be nominated by the Central
The fundamental rights already ensure Government.
the dignity of the individual by not granting (b) Five Members to be nominated by the
special privileges to any section of society by Central Government from amongst persons
ensuring civic equality. of ability, integrity and standing who have
Q.2025 Gender divisions usually refer to: had experience in law or legislation, trade
(a)  Unequal child sex ratio unionism, management of an industry
(b) Unequal roles assigned by the society to potential of women, women’s voluntary
men and women organisations (including women activist),
(c) Biological difference between men and administration, economic development,
women health, education or social welfare;
(d) Absence of voting rights for women in Provided that at least one Member each
democracies shall be from amongst persons belonging
Solution: (b) to the Scheduled Castes and Scheduled
Gender divisions are based on gender roles. Tribes respectively;
A gender role is a theoretical construct in (c) A Member-Secretary to be nominated by
the social sciences and humanities that the Central Government

P.564 For Civil Services Preliminary Examination

06-Indian Polity_Q2005-ENDD.indd 564 8/7/2018 7:51:31 PM


National Commission for protection of Child grievances and advise the Government on all
rights does not have any reservation for policy matters affecting women.
women. Q.2028 The Self-Employed Women’s Association of
India (SEWA)  is a/an
Child Welfare Committees (a) Body formed by the Government of India
As per the provisions of the Juvenile Justice for the welfare of unorganized women
(Care and Protection of Children) Act 2000 workers
(amended in 2006) State governments (b)  Sub-agency of the United Nations
are required to establish a CWC or two in Population Fund (UNPF)
ever district. Each CWC should consist (c)  Trade Union for poor and self-employed
of achairperson and four members. The women workers in India.
chairperson should be a person well versed in (d) Self-help group registered as a society
child welfare issues and at least one member of that provides lending and borrowing
the board should be a woman. The CWC has services to any women in India
the same powers as a metropolitan magistrate Solution: (c)
or a judicial magistrate of the first class. A Learning: It was founded in 1972 by the
child can be brought before the committee (or noted Gandhian and civil rights leader
a member of the committee if necessary) by Dr Ela Bhatt. SEWA’s main goals are to
a police officer, any public servant, childline organise women workers for full employment.
personnel, any social worker or public spirited Full employment means employment
citizen, or by the child himself/herself. whereby workers obtain work security,
income security, food security and social
Q.2027 Consider the following statements.
security (at least health care, child care and
1. Rashtriya Mahila Kosh (RMK) provides
shelter). SEWA organises women to ensure
loans to NGO-MFIs which on-lend to
that every family obtains full employment. It
women Self Help Groups (SHGs).
follows the principles of satya (truth), ahimsa
2. National Commission for Women (NCW)
(non-violence), sarvadharma (integrating all
has all the powers of a civil court while
faiths, all people)  and khadi (propagation of
hearing cases of harassment of women
local employment and self-reliance).
and ordering appropriate punishments.
Which of the above is/are correct? Q.2029 One of SEWA’s (voluntary organization)
(a)  1 only (b)  2 only main goals is to
(c)  Both 1 and 2 (d)  None (a) Organise women workers for full
Solution: (a) employment
Justification: Statement 1: Established in (b) Ensure the fulfilment of reproductive
1993 is a national level organization as an rights of children and mothers
autonomous body under the aegis of the (c) Reduce cases of domestic violence
Ministry of Women and Child Development, against women
the operating model currently followed by (d)  Providing accessible and affordable
RMK is that of a facilitating agency. credit to rural Self-Help Groups
RMK extends micro-credit to the women Solution: (a)
in the informal sector through a client Learning: SEWA is a trade union registered
friendly, without collateral and in a hassle- in 1972. It is an organisation of poor, self-
free manner for income generation activities. employed women workers.
Statement 2: It is an advisory body, • These are women who earn a living
not a court. It can attend to grievances but through their own labour or small
only recommend action, and not order businesses. They do not obtain regular
punishments. salaried employment with welfare benefits
It can review the Constitutional and Legal like workers in the organised sector.
safeguards for women; recommend remedial • Of the female labour force in India, more
legislative measures; facilitate redressal of than 94% are in the unorganised sector.

Indian Polity Question Bank P.565

06-Indian Polity_Q2005-ENDD.indd 565 8/7/2018 7:51:31 PM


However their work is not counted and (b) A is correct, but R is not an appropriate
hence remains invisible. In fact, women explanation of A.
workers themselves remain uncounted, (c) A is correct, but R is incorrect.
undercounted and invisible. (d)  Both A and R are incorrect.
• SEWA’s main goals are to organise Solution: (d)
women workers for full employment. Justification: Recent trends in India towards
Full employment means employment women reservation (also other than the ambit
whereby workers obtain work security, of the question):
income security, food security and social • 33% in PRIs mandated by constitution at
security (at least health care, child care least however in some states it has been
and shelter). extended to 50% - e.g. Bihar, Uttarakhand,
• Gandhian thinking is the guiding force for Odisha, MP. So, both A and R are wrong.
SEWA’s poor, self-employed members in • 33% reservation bill for women in
organising for social change: satya (truth), legislatures was introduced in Lok Sabha
ahimsa (non-violence), sarvadharma in 2008
(integrating all faiths, all people) and • Gujarat government recently introduced
khadi (propagation of local employment 33% reservation for women in the police
and self-reliance). force. Bihar followed with 50% reservation
Q.2030 Consider the following statements about for women in government jobs.
Self-Help Groups (SHGs). • Companies Act 2013: Woman member in
1. There is no strict need for a formal the board of directors
registration of a SHG. • The Government of India planning to
2. This system eliminates the need for introduce 50% compulsory women
collateral and is closely related to that of reservation in local bodies.
solidarity lending, widely used by micro Q.2032 Supreme Court has recently given a ruling
finance institutions. over the growing cases of acid attacks
3. Forming self-help group is a fundamental on women. Consider the following with
right just as forming a cooperative is. reference to it.
Choose the correct answer using the coded 1. State will be responsible to provide
below: compensation to the victim.
(a)  1 and 2 (b)  2 and 3 2. Victim will be provided free 
(c)  1 and 3 (d)  All of the above reconstruction surgery.
Solution: (a) 3. Criminal action can be taken on the
Forming a SHG is not a fundamental right, hospitals that discriminate with such
forming a cooperative is. victims.
(Refer to http://en.wikipedia.org/wiki/ 4. State will provide special privileges for
Self-help_group_(finance) the livelihood of the victim.
Q.2031 Consider the following statements. Which of the above is NOT correct?
Assertion (A): States cannot reserve by (a)  1 and 3 only (b)  3 only
legislation more than 33% seats for women (c)  4 only (d)  2 and 4 only
in local bodies established under the 73rd and Solution: (c)
74th constitutional amendment. Differently abled persons
Reason (R): The 73rd and 74th Q.2033 Consider the following about the provisions
constitutional amendment put an upper for the differently abled persons in India.
ceiling of 33% on women reservations in 1. The constitution places a duty on the state
local bodies. to provide a lawyer to any citizen who is
In the context of the above, which of these unable to engage on due to disability.
is correct? 2. No disabled person can be compelled
(a) A is correct, and R is an appropriate to pay any taxes for the promotion and
explanation of A. maintenance of any particular religion.

P.566 For Civil Services Preliminary Examination

06-Indian Polity_Q2005-ENDD.indd 566 8/7/2018 7:51:31 PM


3. Legally, physical disability disentitles a India Campaign (Sugamya Bharat Abhiyan),
person from inheriting ancestral property. as a nation-wide campaign for achieving
4. Tax laws provide for tax liability universal accessibility for PwDs.
deduction in respect of the expenditure The campaign targets three separate
incurred on the medical treatment of verticals for achieving universal accessibility
handicapped dependants. namely the built up environment,
Select the correct answer using the codes transportation eco-system and information &
below. communication eco-system.
(a)  2 and 3 only (b)  1, 2 and 4 only Q.2035 Consider the following about the Rights of
(c)  3 and 4 only (d)  1, 2, 3 and 4 Persons with Disabilities Act, 2016.
Solution: (b) 1. It has increased the quota of reservation
Justification: Statement 1: Article 22 of the for the disabled in government jobs.
constitution says that the right to be defended 2. It gives the right to free education for
is a fundamental right. Article 39A (DPSP) every child of certain age group with
places the duty on the state as mentioned in benchmark disability.
statement 1. Which of the above is/are correct?
Statement 2: This is a fundamental tenet of (a)  1 only (b)  2 only
secularism which is applicable to all without (c)  Both 1 and 2 (d)  None
discrimination. Solution: (c)
Statement 3: Under the Hindu Succession Justification: It comprehensively covers a
Act, 1956 which applies to Hindus it has been whole spectrum of problems from physical
specifically provided that physical disability disabilities to mental illness and multiple
or physical deformity would not disentitle a disabilities under it.
person from inheriting ancestral property. • It raises number of disabilities from 7
Statement 4: Section 80 DD of IT Act to 21.
provides for a deduction in respect of the • It also raises reservation in government
expenditure incurred by an individual jobs for persons with benchmark
or Hindu Undivided Family resident in disabilities from present 3% to 4% and in
India on the medical treatment (including higher education from 3% to 5% per cent.
nursing) training and rehabilitation etc. of • It includes mental illness, autism, and
handicapped dependants. many such others as well under the
Q.2034 Swavlamban and Sugamya Bharat Abhiyaan definition of disability.
are specifically aimed at the development and • The Act aims to bring Indian laws in line
welfare of with the United Nations Convention on
(a)  Manual Scavengers Rights of Persons with Disabilities, to
(b)  Slum dwellers which India became a signatory in 2007.
(c)  Differently abled People • Once it comes into effect, every child with
(d)  Widow senior citizens benchmark disability between the age
Solution: (c) group of 6 and 18 years will have the right
Learning: Swavlamban, a National Action to free education.
Plan for Skill Training of Persons with
Disabilities was launched last year. International Issues
The Department of Empowerment of
Persons with Disabilities in collaboration with Q.2036 There is no world government. However,
NSDC, proposes to set an ambitious target of some international organizations do lay
skilling 5 lakh persons with disability in next down certain norms; perform certain
3 years. The Action plan is aimed at skilling functions and put restrictions on the conduct
25 lakh persons with disabilities by end 2022. of several sovereign governments. Which of
DEPeD, Ministry of Social Justice and these would qualify in the abovementioned
Empowerment, has formulated the Accessible category?

Indian Polity Question Bank P.567

06-Indian Polity_Q2005-ENDD.indd 567 8/7/2018 7:51:31 PM


1. International Monetary Fund Choose the correct answer using the codes
2. United Nations Security Council below:
3. International Union for the Conservation (a)  2 and 4 only (b)  1 and 3 only
of Nature (IUCN) (c)  1, 3 and 4 only (d)  All of the above
Choose the correct answer using the codes Solution: (c)
below: Nations can usually see that there are some
(a)  1 and 2 (b)  2 and 3 things they must do together. There are issues
(c)  1 and 3 (d)  All of the above that are so challenging that they can only be
Solution: (a) dealt with when everyone works together.
Who makes laws and rules to govern the seas Disease is an example. Some diseases can
that do not fall within the boundaries of any only be eradicated if everyone in the world
one country? Or who takes steps to control cooperates in inoculating or vaccinating their
environmental degradation that threatens all populations. Or take global warming and its
the countries together. The United Nations effects. As atmospheric temperatures rise
(UN)has evolved many Conventions on because of the spread of certain chemicals
these questions that are now binding on called chlorofluorocarbons (CFCs), there
most countries of the world. The UN is a is a danger that sea levels will also rise,
global association of nations of the world thereby submerging many coastal areas of
to help cooperation in international law, the world including huge cities. Of course,
security, economic development and social each country can try to find its own solution
equity. The UN Secretary General is its chief to the effects of global warming. But in the
administrative officer. end a more effective approach is to stop the
What happens when a country attacks warming itself. This requires at least all of the
another country in an unjust manner? The major industrial powers to cooperate.
UN Security Council, an organ of the UN, Q.2038 Which among the following statements is NOT
is responsible for maintaining peace and correct about the contemporary world order?
security among countries. It can put together 1. There is an absence of world government,
an international army and take action against which could regulate a State’s behaviour.
the wrongdoer. 2. States, which violate international law, are
Who lends money to governments when severely punished by the UN.
they need it? The International Monetary 3. The UNSC operates on democratic
Fund (IMF)  does so. The World Bank also principles of equity and fairness.
gives loans to the governments. Before Choose the correct answer using the codes
lending they ask the concerned government below:
to show all its accounts and direct it to make (a)  1 and 2 only (b)  2 and 3 only
changes in its economic policy. (c)  1 only (d)  All of the above
The International Union for the Solution: (b)
Conservation of Nature (IUCN) is only an UN is not the world government. It is only
agreement between the nations. It does not an international organization with hardly any
perform any such functions except record formal authority.
keeping and sending periodic warnings and UNSC has 5 permanent members that
advisories. yield veto power. So it does not work on the
Q.2037 International organizations and cooperation principle of equity.
is helpful because some challenges transcend States cannot be punished severely.
national boundaries. For which of these Occasional sanctions may be put that too only
challenges, International cooperation is a must? when there is a gross case of unauthorized
1. Preventing Cyber Crimes military intervention; genocide etc. There are
2. Tackling Population explosion other bodies like WTO which may take action
3. Tackling Climate Change against its members in case of violation of
4. Tackling epidemics WTO rules

P.568 For Civil Services Preliminary Examination

06-Indian Polity_Q2005-ENDD.indd 568 8/7/2018 7:51:31 PM


Q.2039 In bilateral agreements Confidence Building (c) Recognition of high lands near
measures (CBMs) mean that two countries ecological hotspots
(a) have agreed for a total cease fire across (d) South China Sea dispute
the international borders Solution: (d)
(b) have agreed to improve the relations Learning: It was used by China for its claims
between the armed personnel of both of the major part of the South China Sea.
countries across the borders • The ‘nine-dash line’ (see image below)
(c) have agreed to exchange information stretches hundreds of kilometres south
on defence matters between them on a and east of its southerly Hainan Island,
regular basis covering the strategic Paracel and Spratly
(d) have agreed to halt down military island chains.
modernization drives across the border • China claims this citing 2,000 years of
Solution: (c) history when the two island chains were
In a slew of measures, India and China have regarded as its integral parts.
agreed for some CBMs across the borders. It • The Permanent Court of Arbitration
includes sharing information on the location (PCA) has ruled that China’s claims of
of troops; military modernization drives; historical rights over South China Sea
information exchange in case of violation of (SCS) have no legal basis.
ceasefire; strengthening the communication • China has boycotted the hearings at the
between the major generals of both the Permanent Court of Arbitration, saying
armies etc. it does not have jurisdiction to decide on
Q.2040 What does the term ‘strategic partnership’ the matter.
mean? Q.2042 Which of these Non-governmental
(a) Strategic partnership between two organizations does NOT primarily work
nations is equivalent of joining towards safeguarding human rights?
an alliance and becoming allies in (a) Amnesty International
international affairs. (b) International Committee of the Red Cross
(b) It is a long-term interaction/commitment (c) UN Watch
between two countries based on political, (d) ENGO
economic, social and historical factors. Solution: (d)
(c) Strategic partners are obstructed with Justification: Option B: It directs and
least restrictions in the economic sphere coordinates international relief and works
like tariffs, investments and duties. to promote and strengthen humanitarian law
(d) It is a long term mutual agreement between and universal humanitarian principles.
two nations to support each other militarily Option C: N Watch is a non-governmental
in the outbreak of a war or aggression. organization based in Geneva whose mandate
Solution: (b) is to monitor the performance of the United
A strategic partnership is a long-term Nations by the yardstick of its own Charter.
interaction between two countries based on Option D: The Environmental Non-
political, economic, social and historical Governmental Organization (ENGO) is
factors. Such a partnership manifests itself a generic term for NGO in the field of
in a variety of relationships. India has signed environmentalism. Examples of ENGOs
“strategic partnerships” with more than 30 include the WWF, Greenpeace, Conservation
countries. You can read this beautiful article International, The Nature Conservancy and
for a better understanding. the Environmental Investigation Agency is
Q.2041 The ‘nine-dash’ line is often heard in news an environmental organization.
with reference to the Q.2043 Which of the following developments could
(a) Border skirmishes between India and be causal factors behind World War-I?
Pakistan 1. Pan-Slav movement
(b) Israel-Palestine conflict and settlement 2. Race for colonies amongst European
empires
Indian Polity Question Bank P.569

06-Indian Polity_Q2005-ENDD.indd 569 8/7/2018 7:51:31 PM


3. Truman Doctrine Select the correct (b) Differentiating between the Aggressor
answer using the codes below states and invaded states
(a)  1 and 2 only (b)  2 and 3 only (c) Former colonial occupations of Western
(c)  1 and 3 only (d)  All of the above powers in the Southern Hemisphere
Solution: (a) (d) Diplomatic groupings formed within the
Justification: Truman doctrine is the USA cold Asian nations
war policy of communism containment. It is Solution: (a)
not related to the World War I. So, 3 is incorrect. Learning: The North mostly covers the
Race for colonies and European West and the First World, along with much
domination took the following forms of the Second World, while the South largely
competition to control declining Ottoman corresponds with the Third World.
Empire e.g. Secret pacts like Sykes Picot • While the North may be defined as the
Agreement 1916 entered b/w britain and richer, more developed region and the
Russia for dividing Ottoman Empire etc. So, South as the poorer, less developed
the point 2 is correct. region, many more factors differentiate
Pan–Slavism movement: All Slavic between the two global areas. 95% of the
countries wanted to Unite, like unification of North has enough food and shelter.
Germany, Italy, but Austria resisted it. It was • Similarly, 95% of the North has a
a major factor behind WW-I. So, the option functioning educational system. In the
1 is correct. South, on the other hand, only 5% of the
Q.2044 Strict interpretation of which of these population has enough food and shelter.
ideologies forms the core value of the Islamic • It “lacks appropriate technology, it has
radical and terrorist groups working in the no political stability, the economies are
Middle-East? disarticulated, and their foreign exchange
(a)  Sufism (b)  Fascism earnings depend on primary product
(c) Wahabism (d) Libertarianism exports”.
Solution: (c) • As nations become economically
Learning: It is an austere form of Islam developed, they may become part of
that insists on a literal interpretation of the the “North”, regardless of geographical
Koran. Strict Wahhabis believe that all those location, while any othermic republic.
who don’t practice their form of Islam are nations which do not qualify for
heathens and enemies. “developed” status are in effect deemed to
• For over 2 centuries, Wahhabism has been be part of the “South”.
Saudi Arabia’s dominant faith. Q.2046 Consider the following list of persecuted
• The religious curriculum in Saudi Arabia communities and match them with their
teaches you that people are basically two respective regions.
sides: Salafis (Wahhabis), who are the 1. Falun Gong: China
winners as the chosen ones, who will go 2. Gafatar: Indonesia
to heaven, and the rest. 3. Mandaeans: Thailand
• Wahhabism’s explosive growth began in Select the correct answer using the codes
the 1970s from Saudi. below.
• A Huffington post says that, “you Can’t (a)  1 and 2 only (b)  2 and 3 only
Understand ISIS If You Don’t Know the (c)  1 only (d)  1, 2 and 3
History of Wahhabism in Saudi Arabia.” Solution: (a)
Q.2045 The term North–South divide in the context Justification: Statement 1: The persecution
of International affairs is related to of Falun Gong refers to the campaign initiated
(a) A socio-economic and political divide in 1999 by the Chinese Communist Party to
between the Developed and Developing eliminate the spiritual practice of Falun Gong
World in China.

P.570 For Civil Services Preliminary Examination

06-Indian Polity_Q2005-ENDD.indd 570 8/7/2018 7:51:31 PM


It is estimated that 65,000 Falun Gong Choose the correct answer using the codes
practitioners were killed for their organs below.
from 2000 to 2008. (a)  1 and 4 only (b)  1 and 3 only
Statement 2: Human Rights Watch (c)  2 and 3 only (d)  None of the above
remains concerned that the Indonesian Solution: (d)
government is persecuting members on the • The deal puts strict limits on Iran’s
basis that Gafatar is a religious organization nuclear activities for at least a decade and
and is violating their basic rights to freedom calls for stringent U.N. oversight, with
of expression, association, and religion. world powers hoping this will make any
Statement 3: Owing to the rise of Arab dash to make an atomic bomb virtually
nationalism, Mandaeans (in Iraq, Iran, Syria, impossible.
Jordan) were Arabised at an accelerated rate. • In return, Iran will get sanctions relief
Moreover, it is estimated that around 90% although the measures can “snap back”
of Iraqi Mandaeans were either killed or have into place if there are any violations.
fled after the American-led invasion in Iraq. • The international arms embargo against
Q.2047 In which of the following ways an Iran will remain for 5 years but deliveries
International Institution will tend to limit the would be possible with special permission
sovereignty of a State of the U.N. Security Council. Iran has
1. When the institution imposes accepted allowing the U.N. atomic
conditionality on loans availed by the state watchdog tightly-controlled “managed
2. When the state accedes to a binding treaty access” to military bases.
or convention of the institution • Iran will slash by around two-thirds
Which of the above is/are correct? the number of centrifuges from around
(a)  1 only (b)  2 only 19,000 to 6,104.
(c)  Both 1 and 2 (d)  None • The deal caps uranium enrichment at
Solution: (c) 3.67% and limits the stockpile to 300 kg,
Justification: Statement 1: The IMF loan all for 15 years.
to India in 1991 is a classic case of this • Iran will be required to ship spent fuel out
statement. of the country forever, as well as allow
IMF imposed several conditions on India inspectors from the IAEA inspectors
such as devaluing rupee, opening up the certain access in perpetuity. Heightened
economy etc which limited India’s sovereign inspections, including tracking uranium
space. mining and monitoring the production and
Statement 2: For e.g. many developed storage of centrifuges, will last for up to
nations acceded to the Kyoto protocol and 20 years.
decided to implement GHG reductions, Q.2049 The “collective self-defence” doctrine
which limited the authority of governments presently being pushed by the Japanese
to design industrial programs that generate government means
GHGs. (a) Japan is not in favour of independent
Q.2048 As per the terms arrived at in the Iran armies for all nations
Nuclear Deal (b) Japan may wish to send troops abroad to
1. Iran will not enrich any Uranium. rescue allies under attack
2. Iran will be subject to oversight from US (c) Japan will intervene constructively in
Nuclear authorities. all international negotiations that have
3. All previous nuclear installations will be significant geo-strategic implications
shutdown in a period of time. (d) Japan will make military conscription
4. Iran will not be able to share nuclear compulsory for its citizens in larger
technology with any nation. national interest
Solution: (b)

Indian Polity Question Bank P.571

06-Indian Polity_Q2005-ENDD.indd 571 8/7/2018 7:51:31 PM


The security legislation proposed by Japanese 3. Strengthen the global governance of
Prime Minister Shinzo Abe’s administration migration by bringing the International
moved a step closer to becoming law when Organization for Migration into the
Parliament’s lower house approved it. The UN system.
bills, which seek to rewrite the country’s post- 4. Expand the opportunities for refugees to
War pacifist security policy, are now before relocate to other countries
the upper house. Over a period of 7 decades, Select the correct answer using the codes
Japan’s security policy, shaped under a below.
war-renouncing Constitution following (a)  1, 2 and 3 only (b)  1, 3 and 4 only
the misadventures of the imperial regime, (c)  2 and 4 only (d)  1, 2, 3 and 4
has been focussed on selfdefence. But the Solution: (b)
present bills seek to replace the selfdefence Justification: The declaration expresses the
doctrine with “collective self-defence”, that political will of world leaders to save lives,
would allow Japan to send troops abroad to protect rights and share responsibility on a
rescue allies under attack. This big shift in global scale.
approach makes the legislation controversial It, inter alia, includes (apart from
and unpopular. For reasons refer statements 1, 3 and 4 above):
this article: • Protect the human rights of all refugees
and migrants, regardless of status.
International Declarations • Prevent and respond to sexual and gender-
based violence.
Q.2050 The Cartagena Declaration, 1984, is related • Support those countries rescuing,
to the receiving and hosting large numbers of
(a)  Protection of refugees refugees and migrants.
(b)  Educational rights for minorities • Work towards ending the practice of
(c)  Livelihood for migrant workers detaining children for the purposes of
(d) Crèche facilities for working women determining their migration status.
Solution: (a) • Strongly condemn xenophobia against
Learning: The Cartagena Declaration on refugees and migrants and support a
Refugees was adopted by the Colloquium global campaign to counter it.
on the International Protection of Refugees • Find new homes for all refugees identified
in Central America, Mexico and Panama by UNHCR as needing resettlement
in 1984.
Q.2052 The “1961 Vienna convention” that was seen
The declaration is a non-binding agreement
in news sometime back deals with
but has been incorporated in refugee law in
(a) Reduction of persistent organic pollutants
various countries. Its principles are based on
(POPs)
the 1951 UN Refugee Convention and the
(b)  Protocol to handle refugees
1967 Protocol.
(c)  Diplomatic Immunity
The Declaration reaffirms the importance
(d)  Nuclear Stockpile
of the right to asylum, the principle of non-
Solution: (c)
refoulement and the importance of finding
Learning: The Vienna Convention and the
durable solutions.
Convention on Consular Relations, 1963
Q.2051 The New York Declaration for Refugees and have been ratified by 187 countries, including
Migrants include commitments to India. This means it is a law under the Indian
1. Ensure that all refugee and migrant legal framework and cannot be violated.
children are receiving education within a Article 31 of the 1961 Vienna Conventions
few months of arrival says diplomats “shall enjoy immunity from
2. Put sanctions against countries the criminal jurisdiction of the receiving state
hosting large numbers of refugees and and also from its civil and administrative
migrants. jurisdiction.

P.572 For Civil Services Preliminary Examination

06-Indian Polity_Q2005-ENDD.indd 572 8/7/2018 7:51:31 PM


The Convention was meant to enable (d)  International cooperation for better
diplomats to carry out their duties without operation of international flights
obstruction in the Cold War environment. Solution: (a)
However, as per legal scholars, the number Beijing Protocol: This Protocol has brought
of abuses affecting fundamental rights of out new principal offences in hijacking. This
citizens in very small; and thus its occasional protocol is combined with ancillary offences,
abuse is largely offset by the international enlarged the scope of “hijacking”, expanded
need of the convention. jurisdiction and strengthened extradition and
Using its provisions to save diplomats mutual assistance regimes in case hijacking.
facing charges of heinous crimes such as The government has recently given nod
enslavement and rape cannot be justified to for introduction of the comprehensive
ethically. Provisions on criminal liability of Anti-Hijacking Bill, 2014 in the Parliament.
diplomats should be given thought to. The Cabinet has also given its approval to
Q.2053 What was the purpose behind the Vienna ratify Beijing Protocol, 2010 of the UN body
Convention 1961? International Civil Aviation Organisation
1. To enable nations to keep embassies in all (ICAO) to which India is a signatory.
nations without obstruction Q.2055 India has acceded to the Customs Convention
2. To enable diplomats to carry out their on International Transport of Goods under
duties without obstruction in the Cold cover of TIR Carnets (TIR Convention)
War environment and necessary procedures for ratification.
Which of the above is/are true? What benefits will India derive from this
(a)  1 only (b)  2 only ratification?
(c)  Both 1 and 2 (d)  None 1. It will eliminate the custom duties Indian
Solution: (b) goods need to pay to members of the
Explanation: Even if both nations are convention.
signatory of the Vienna conventions, the host 2. It will obviate the need for inspection
nation can deny keeping and maintaining of goods at intermediate borders due to
of embassy to a foreign nation. Thus, reciprocal recognition of Customs controls.
statement 1 is wrong. Which of the above is/are correct?
Learning: The Vienna Convention classifies (a)  1 only (b)  2 only
diplomats according to their posting in (c)  Both 1 and 2 (d)  None
the embassy, consular or international Solution:(b)
organisations such as the UN. A nation has Justification: It is an international transit
only one embassy per foreign country, usually system under the auspices of the United
in the capital, but may have multiple consulate Nations Economic Commission for Europe
offices, generally in locations where many of (UNECE).
its citizens live or visit. Diplomats posted in • It facilitates the seamless movement of
an embassy get immunity, along with his or goods within and amongst the Parties to
her family members. While diplomats posted the Convention.
in consulates too get immunity, they can be • At present there are 70 parties to the
prosecuted in case of serious crimes, that Convention, including 69 states and the
is, when a warrant is issued. Besides, their European Union.
families don’t share that immunity. • It covers customs transit by road and
Q.2054 The Beijing protocol concerns other modes of transport (e.g., rail, inland
(a)  Hijacking of planes waterway, maritime transport), as long as
(b) International Aviation operational safety at least one part of the total transport is
standards made by road.
(c) Safety standards on International airports • It also serves as a Customs declaration,
and hence it precludes the need to file

Indian Polity Question Bank P.573

06-Indian Polity_Q2005-ENDD.indd 573 8/7/2018 7:51:31 PM


multiple declarations satisfying national the two countries, but they were determined
laws of the different transiting countries. to get Khrushchev to remove the missiles
• By joining the convention, Indian traders and nuclear weapons from Cuba. Kennedy
will get access to fast, easy, reliable ordered American warships to intercept any
and hassle free international system for Soviet ships heading to Cuba as a way of
movement of goods by road or multi- warning the USSR of his seriousness. A clash
modal means across the territories of seemed imminent in what came to be known
other contracting parties. as the Cuban Missile Crisis.
• It will avoid clearances at Border Q.2057 Which of the following factors did NOT
Crossing Points and ports that may often contribute/propel the Vietnam War?
be congested, as under the convention (a)  Anti-communist agenda
customs clearance can take place at (b)  Fight against colonialism
internal Customs locations. (c) North Korean acquisition of Vietnam
(d)  Coup in Vietnam government
Cold War Solution: (c)
Justification: It started as a fight against
Q.2056 Which of the f ollowing are cited as possible
French colonial capture. So, (b) is a factor.
reasons for the Cuban Missile Crisis,
Japan has acquired some territories of Indo-
1962?
China by force, not North Korea. In fact, the
1. Spread of capitalism in Cuba
latter was a supporter of communist forces in
2. Impending USA led invasion of Cuba
Vietnam. So, (c) is incorrect. USA’s cold war
3. Installation of nuclear missiles in Cuba
policy of containment of communism was
Choose the correct answer using the codes
a major factor in fuelling the Vietnam War.
below:
So, (a) is a factor. Ousting and assassination
(a)  1 and 2 (b)  2 and 3
of Ngô Đình Diệm also insued political
(c)  1 and 3 (d)  All of the above
instability in Vietnam, destabilized the region
Solution: (b)
and fuelled the war further. So, (d) was a
In 1961, the leaders of the Union of Soviet
factor
Socialist Republics (USSR) were worried
that the United States of America (USA) Q.2058 Which of the following could have been the
would invade communist-ruled Cuba and possible effects and consequences of the
overthrow Fidel Castro, the president of the Cold war between USA and USSR?
small island nation off the coast of the United 1. Attempts by both superpowers to promote
States. Cuba was an ally of the Soviet Union their respective political and economic
and received both diplomatic and financial ideologies
aid from it. Nikita Khrushchev, the leader of 2. Nuclear arms proliferation in the world
the Soviet Union, decided to convert Cuba 3. Formation of military alliances across the
into a Russian base. globe
In 1962, he placed nuclear missiles in Choose the correct answer using the codes
Cuba. The installation of these weapons put below:
the US, for the first time, under fire from (a)  1 and 2 (b)  2 and 3
close range and nearly doubled the number (c)  1 and 3 (d)  All of the above
of bases or cities in the American mainland Solution:(d)
which could be threatened by the USSR. The Cold War was not simply a matter of
Three weeks after the Soviet Union had power rivalries, of military alliances, and of
placed the nuclear weapons in Cuba, the the balance of power.
Americans became aware of it. These were accompanied by a real
The US President, John F. Kennedy, and ideological conflict as well, a difference
his advisers were reluctant to do anything that over the best and the most appropriate way
might lead to full-scale nuclear war between of organising political, economic, and social
life all over the world.

P.574 For Civil Services Preliminary Examination

06-Indian Polity_Q2005-ENDD.indd 574 8/7/2018 7:51:32 PM


The western alliance, headed by the US, Q.2060 In the power blocs formed by the superpowers,
represented the ideology of liberal democracy why were the smaller states important to the
and capitalism while the eastern alliance, superpowers?
headed by the Soviet Union, was committed 1. For getting mineral resources
to the ideology of socialism and communism. 2. For getting locations to spy on the other
The two superpowers and the countries superpower
in the rival blocs led by the superpowers 3. For military bases
were expected to behave as rational and 4. Ideological spread
responsible actors. They were to be rational Choose the correct answer using the codes
and responsible in the sense that they below:
understood the risks in fighting wars that (a)  1 and 4 (b)  2 and 3
might involve the two superpowers. When (c)  1, 3 and 4 (d)  All of the above
two superpowers and the blocs led by them Solution: (d)
are in a deterrence relationship, fighting wars Why the superpowers needed any allies at
will be massively destructive. all? After all, with their nuclear weapons and
Q.2059 What were the possible advantages that regular armies, they were so powerful that
accrued to the smaller states that became the combined power of most of the smaller
part of the power blocs formed by USA and states in Asia and Africa, and even in Europe,
USSR during the Cold war? was no match to that of the superpowers.
1. Economic aid from superpowers Yet, the smaller states were helpful for the
2. Military support superpowers in gaining access to
3. Greater voting rights in International (i) vital resources, such as oil and minerals,
bodies (ii) territory, from where the superpowers
Choose the correct answer using the codes could launch their weapons and troops,
below: (iii) locations from where they could spy on
(a)  1 and 2 (b)  2 and 3 each other, and
(c)  1 and 3 (d)  All of the above (iv) economic support, in that many small
Solution: (a) allies together could help pay for
The two superpowers were keen on expanding military expenses.
their spheres of influence in different parts They were also important for ideological
of the world. In a world sharply divided reasons. The loyalty of allies suggested that
between the two alliance systems, a state the superpowers were winning the war of
was supposed to remain tied to its protective ideas as well.
superpower to limit the influence of the other Q.2061 Which of the following significant events/
superpower and its allies. actions were associated with the Cold war in
The smaller states in the alliances used the that period?
link to the superpowers for their own purposes. 1. Truman Doctrine
They got the promise of protection, weapons, 2. Berlin Blockade
and economic aid against their local rivals, 3. American intervention in Vietnam
mostly regional neighbours with whom they 4. Soviet intervention in Hungary
had rivalries. The alliance systems led by the Choose the correct answer using the codes
two superpowers, therefore, threatened to below:
divide the entire world into two camps. This (a)  1 and 4 (b)  2 and 3
division happened first in Europe. (c)  1, 3 and 4 (d)  All of the above
Most countries of western Europe sided Solution: (d)
with the US and those of eastern Europe Q.2062 Consider the following statements.
joined the Soviet camp. That is why these 1. The cold war international order was
were also called the “western” and the unipolar in nature with United States
“eastern” alliances. (US) as the sole superpower.

Indian Polity Question Bank P.575

06-Indian Polity_Q2005-ENDD.indd 575 8/7/2018 7:51:32 PM


2. The US approach to the cold war was industrialized countries like Brazil, India and
originally set out in the Truman Doctrine China now more commonly referred to as
which advocated disarmament and part of BRIC.
demilitarization. Q.2064 In context of World History, what was
Which of the above is/are correct? referred to as “Iron Curtain”?
(a)  1 only (b)  2 only (a) Economic growth of iron ore rich and
(c)  Both 1 and 2 (d)  None iron ore poor nations
Solution: (d) (b) Development divide between colonized
Justification: Statement 1: The world order and colonizer nations
was bi-polar with erstwhile soviet union and (c) Contrast in the progress of Industrial
USA. The cold war was a rivalry between revolution in the East and West
them for domination. (d)  Boundary that divided communist and
Statement 2: Before the US began its capitalist states in Europe
expansionist policies during the cold war, its Solution: (d)
international policy was isolationist. Learning: The term Iron Curtain refers to
It meant that the US would not intervene the boundary that divided Europe in the west
in the internal affairs of a country normally and the Soviet Union and its Communist one-
unless USA is threatened or destabilised. party states in the east.
However, with Soviet adventurism, Truman The division began at the end of World
pledged to: War Two in 1945 and lasted until the fall of
• Support Free people who are resisting the USSR in 1989.
subjugation by armed minorities or by It was a political, military, and ideological
outside pressures barrier erected by the Soviet Union; a fall out
• Commitment to a policy of containing of the cold war.
communism not just in Europe but
Q.2065 During the Cold War, the idea of a New
throughout the world.
International Economic Order (NIEO) as
Q.2063 The term Third World, as known in history, proposed by the UNCTAD was
refers to 1. Giving the least developed countries
(a) Disease stricken regions in tropical (LDCs) greater access to their own
countries natural resources.
(b) Underworld organizations operating in 2. Greater western market access for LDCs
the Middle East 3. Providing the LDCs with a greater role in
(c) Colonies of European nations in Africa the International institutions
(d) Countries that were not aligned with Choose the correct answer using the codes
either USA or Soviet Union below:
Solution: (d) (a)  1 and 2 only (b)  2 and 3 only
Learning: The term has been subject to (c)  3 only (d)  All of the above
evolutionary interpretations. Solution: (d)
Third World countries were labelled during Economic development was also vital for the
the Cold War to reference those nations that independence of the new countries. Without
were not aligned with either the United States sustained development, a country could not
or the Soviet Union. be truly free. It would remain dependent on
Later, once cold war ended, it came to the richer countries including the colonial
include many countries with colonial pasts powers from which political freedom had been
in Africa, Latin America, Oceania and Asia achieved. The idea of a New International
which were poor and underdeveloped. It Economic Order (NIEO) originated with this
became a stereotype to refer to poor countries realisation. The United Nations Conference
as “third world countries”. on Trade and Development (UNCTAD)
However, recently the “Third World” brought out a report in 1972 entitled Towards
term is also often taken to include newly a New Trade Policy for Development.

P.576 For Civil Services Preliminary Examination

06-Indian Polity_Q2005-ENDD.indd 576 8/7/2018 7:51:32 PM


The report proposed a reform of the global USSR President Mikhail Gorbachev and the
trading system so as to: US President George Bush (Senior) on the
(i) give the LDCs control over their natural reduction and limitation of strategic offensive
resources exploited by the developed arms in Moscow on 31 July 1991.
Western countries, STRATEGIC ARMS REDUCTION
(ii) obtain access to Western markets so that TREATY II (START-II) treaty signed by
the LDCs could sell their products and, the Russian President Boris Yeltsin and the
therefore, make trade more beneficial US President George Bush (Senior) on the
for the poorer countries, reduction and limitation of strategic offensive
(iii) reduce the cost of technology from the arms in Moscow on 3 January 1993.
Western countries, and Q.2067 The USA has had a hegemonic position
(iv) provide the LDCs with a greater role in the international sphere since the end
in international economic institutions. of Cold War especially by exercising its
Gradually, the nature of nonalignment military influence. However, its hegemonic
changed to give greater importance to power is subject to some constraints. These
economic issues. may be
Q.2066 Several arms control treaties were signed 1. American Mass Media and Civil Society
during the Cold War. Which of these were 2. The moderating influence of NATO
from amongst those? 3. Permanent members of the UNSC
1. Limited Test Ban treaty Choose the correct answer using the codes
2. Nuclear Non-Proliferation treaty below:
3. Strategic Arms Reduction Treaty (a)  1 and 2 only (b)  2 and 3 only
Choose the correct answer using the codes (c)  3 only (d)  All of the above
below: Solution: (d)
(a)  1 and 2 only (b)  2 and 3 only The first constraint is the institutional
(c)  1 and 3 only (d)  All of the above architecture of the American state itself.
Solution: (d) A system of division of powers between
LIMITED TEST BAN TREATY (LTBT) the three branches of government places
banned nuclear weapon tests in the significant brakes upon the unrestrained and
atmosphere, in outer space and under water. immoderate exercise of America’s military
Signed by the US, UK and USSR in Moscow power by the executive branch.
on 5 August 1963. Entered into force on The second constraint on American power
10 October 1963. is also domestic in nature, and stems from the
NUCLEAR NON-PROLIFERATION open nature of American society.
TREATY (NPT) allows only the nuclear Although the American mass media
weapon states to have nuclear weapons and may from time to time impose or promote
stops others from aquiring them. For the a particular perspective on domestic public
purposes of the NPT, a nuclear weapon state opinion in the US, there is nevertheless a
is one which has manufactured and exploded deep scepticism regarding the purposes
a nuclear weapon or other nuclear explosive and methods of government in American
device prior to 1 January 1967. So there are political culture. This factor, in the long run,
five nuclear weapon states: US, USSR (later is a huge constraint on US military action
Russia), Britain, France and China. Signed in overseas.
Washington, London, and Moscow on 1 July However, it is the third constraint on the
1968. Entered into force on 5 March 1970. US that is perhaps the most important.
Extended indefinitely in 1995. There is only one organisation in the
STRATEGIC ARMS REDUCTION international system that could possibly
TREATY I (START-I) treaty signed by the moderate the exercise of American power

Indian Polity Question Bank P.577

06-Indian Polity_Q2005-ENDD.indd 577 8/7/2018 7:51:32 PM


today, and that is the North Atlantic Treaty different political systems and interests
Organisation (NATO). The US obviously has joined it. This made the movement less
an enormous interest in keeping the alliance homogeneous and also made it more difficult
of democracies that follow the market to define in very neat and precise terms:
economies alive and therefore it is possible what did it really stand for? Increasingly,
that its allies in the NATO will be able to NAM was easier to define in terms of what
moderate the exercise of US hegemony. it was not. It was not about being a member
of an alliance. The policy of staying away
Non-Alignment Movement from alliances should not be considered
isolationism or neutrality.
Q.2068 India followed the policy of Non-alignment Non-alignment is not isolationism since
post-independence in response to isolationism means remaining aloof from
(a)  Economic backwardness of India world affairs. Isolationism sums up the foreign
(b) Impending wars with South Asian policy of the US from the American War of
neighbours Independence in 1787 up to the beginning of
(c)  Cold war rivalry the First World War. In comparison, the non-
(d) Former agreements between South Asian aligned countries, including India, played an
neighbours active role in mediating between the two rival
Solution: (c) alliances in the cause of peace and stability.
Justification: A case can be made for options Their strength was based on their unity and
(a) and (b), but these are not as strong as option their resolve to remain non-aligned despite
(c), which is the most appropriate option. the attempt by the two superpowers to bring
Option (a) because, policy of non-alignment them into their alliances.
helped us maintain an independent foreign Non-alignment is also not neutrality.
policy and stop neo-colonialism, which could Neutrality refers principally to a policy of
have again sapped India’s resources. staying out of war. States practising neutrality
Option (b) because joining a cold war are not required to help end a war. They do
alliance may have further aggravated rivalry not get involved in wars and do not take any
with neighbours. position on the appropriateness or morality
But, the option (c) is most appropriate of a war. Non-aligned states, including India,
as it was the cold war and in this context, were actually involved in wars for various
Jawaharlal Nehru observed: “We propose to reasons. They also worked to prevent war
keep away from the power politics of groups, between others and tried to end wars that had
aligned against one another, which have led in broken out.
the past to world wars and which may again Q.2070 Critics allege India’s non-alignment as
lead to disasters on an even vaster scale. unprincipled during the Cold War. Which of
Q.2069 The Non-alignment Movement believed in the following arguments would strengthen
which of the following principles? the proposition?
1. Isolationism from International conflicts 1. India refused to take a firm stand on
2. Neutrality in International affairs crucial international issues like Russian
3. Equal opportunities for the economic intervention of Afghanistan.
development of recently decolonized 2. The Treaty of Friendship in 1971 with the
nations USSR for 20 years made India virtually a
Choose the correct answer using the codes member of the Soviet Alliance system.
below: Choose the answer using the codes below.
(a)  1 and 2 only (b)  2 and 3 only (a)  1 only
(c)  3 only (d)  All of the above (b)  2 only
Solution: (c) (c)  Both 1 and 2
As non-alignment grew into a popular (d)  None strengthens the proposition
international movement, countries of various Solution: (c)

P.578 For Civil Services Preliminary Examination

06-Indian Polity_Q2005-ENDD.indd 578 8/7/2018 7:51:32 PM


India’s policy of non-alignment was criticised 1. A stagnant soviet economy
on a number of counts. Here we may refer to 2. Authoritarian Soviet government
only two criticisms: 3. Coup in the communist party
First, India’s non-alignment was said to Choose the correct answer using the codes
be ‘unprincipled’. In the name of pursuing below:
its national interest, India, it was said, (a)  3 only
often refused to take a firm stand on crucial (b)  1 and 2 only
international issues. (c)  1 only
Second, it is suggested that India was (d) None of the options could be denied as
inconsistent and took contradictory postures. possible reasons.
Having criticised others for joining Solutions: (d)
alliances, India signed the Treaty of Friendship There was no unemployment. State ownership
in August 1971 with the USSR for 20 years. was the dominant form of ownership: land
This was regarded, particularly by outside and productive assets were owned and
observers, as virtually joining the Soviet controlled by the Soviet state.
alliance system. The Indian government’s view The Soviet system, however, became
was that India needed diplomatic and possibly very bureaucratic and authoritarian, making
military support during the Bangladesh crisis life very difficult for its citizens. Lack of
and that in any case the treaty did not stop democracy and the absence of freedom of
India from having good relations with other speech stifled people who often expressed
countries including the US. their dissent in jokes and cartoons. Most of
Q.2071 Bandung Conference (1955) is related to the institutions of the Soviet state needed
which of the following? reform: the one-party system represented by
(a) Paving the way forward to Non-aligned the Communist Party of the Soviet Union
Movement had tight control over all institutions and
(b) New World Information and was unaccountable to the people. The party
Communication Order refused to recognise the urge of people in
(c) Five Principles of Peaceful Coexistence the fifteen different republics that formed
– India and China the Soviet Union to manage their own affairs
(d) Agreement between the sunshine countries including their cultural affairs. Although,
completely or partly between the tropic of on paper, Russia was only one of the fifteen
Cancer and the tropic of Capricorn republics that together constituted the USSR,
Solution: (a) in reality Russia dominated everything, and
Learning: It was the first large-scale Afro– people from other regions felt neglected and
Asian Conference of Afro-Asian nations often suppressed.
most of which were newly independent. The In the arms race, the Soviet Union
conference’s stated aims were to promote Afro- managed to match the US from time to time,
Asian economic and cultural cooperation and to but at great cost. The Soviet Union lagged
oppose colonialism or neo-colonialism by any behind the West in technology, infrastructure
nation. The conference was an important step (e.g. transport, power), and most importantly,
toward the Non-Aligned Movement. in fulfilling the political or economic
The Non-Aligned Movement (NAM) is a aspirations of citizens. The Soviet invasion
group of states which are not formally aligned of Afghanistan in 1979 weakened the system
with or against any major power bloc. even further. Though wages continued to
grow, productivity and technology fell
considerably behind that of theWest. This
Soviet Union and Disintegration led to shortages in all consumer goods. Food
Q.2072 Which of these could NOT have been the imports increased every year. The Soviet
possible reasons for the breakdown of Soviet economy was faltering in the late 1970s and
Union? became stagnant.

Indian Polity Question Bank P.579

06-Indian Polity_Q2005-ENDD.indd 579 8/7/2018 7:51:32 PM


Q.2073 While reforms led by Soviet leader Gorbachev Q.2074 The consequences of the breakdown of
were aimed at reforming the Soviet Union, Soviet Union had global proportions. Which
they also lead to the breakdown of the Soviet of the following could have been the possible
Union. What such reforms were introduced consequences?
by Gorbachev after 1985? 1. End of cold war confrontations
1. Freedom to the media 2. Emergence of new countries in Europe
2. Making Soviet Communist party more 3. Shift in the balance of power in the world
democratic Choose the correct answer using the codes
3. Allowing small private industries to operate below:
Choose the correct answer using the codes (a)  1 and 2 only (b)  2 and 3 only
below: (c)  1 and 3 only (d)  All of the above
(a)  1 and 2 only (b)  2 and 3 only Solution: (d)
(c)  1 and 3 only (d)  All of the above The collapse of the second world of the
Solution: (d) Soviet Union and the socialist systems in
The disintegration of Soviet Union started eastern Europe had profound consequences
under the leadership of Mikhail Gorbachev. for world politics.
A discussion of his policies and their First of all, it meant the end of Cold
shortcomings would lead us to the break-up War confrontations. The ideological
of the soviet Union. dispute over whether the socialist system
Gorbachev came to power in 1985. He would beat the capitalist system was not
was determined to transform and revitalize an issue any more. Since this dispute had
the Union and for this purpose intended engaged the military of the two blocs,
the modernizing and streamlining the had triggered a massive arms race and
communist party. Gorbachev wanted to accumulation of nuclear weapons, and
make communism practiced in Soviet more had led to the existence of military blocs,
humane and democratic. the end of the confrontation demanded an
Reforms by Gorbachev–perestroika end to this arms race and a possible new
(economic restructuring)  and Glasnost peace. Second, power relations in world
(openness): politics changed and, therefore, the relative
1. He made foreign affairs a priority, took influence of ideas and institutions also
initiatives for better relations with china; changed. The end of the Cold War left open
and pushed Soviet withdrawal from only two possibilities: either the remaining
Afghanistan. superpower would dominate and create a
2. Freedom was given to media to criticize unipolar system, or different countries or
the government. groups of countries could become important
3. For the first time, parties other than the players in the international system, thereby
Communist Party could participate in bringing in a multipolar system where no
elections. one power could dominate. As it turned
4. In economic affairs government out, the US became the sole superpower.
grip was loosened. Small scale Third, the end of the Soviet bloc meant
private entrepreneurs were allowed. the emergence of many new countries. All
Computerization was started which these countries had their own independent
reduced the manual labor. aspirations and choices. Some of them,
5. Workers were given the right to strike for especially the Baltic and east European
better wages and conditions. states, wanted to join the European Union
6. Gorbachev also encouraged foreign and become part of the North Atlantic Treaty
investment in Soviet enterprises. Organisation (NATO). Thus, the international
7. Political changes were initiated that led system saw many new players emerge, each
to a move towards democracy within the with its own identity, interests, and economic
Communist party. and political difficulties.

P.580 For Civil Services Preliminary Examination

06-Indian Polity_Q2005-ENDD.indd 580 8/7/2018 7:51:32 PM


Q.2075 The model of transition in Russia, Central Military Operations
Asia and east Europe that was influenced
by the World Bank and the IMF after the Q.2076 In 2003, the US launched its invasion of
breakdown of the Soviet Union came to be Iraq under the codename ‘Operation Iraqi
known as ‘shock therapy’. It included Freedom‘. It was intended to
1. Collective farms were replaced by private 1. Prevent Iraq from developing Weapons of
farming and capitalism in agriculture. Mass Destruction (WMD)
2. Privatisation of state assets. 2. Contain the anti-democratic uprising
3. Break-up of the existing trade alliances in Iraq
among the countries of the Soviet bloc 3. Protect Iraq from the attacks by the
Choose the correct answer using the codes neighbouring Gulf States
below: Choose the correct answer using the codes
(a)  1 and 2 only (b)  2 and 3 only below:
(c)  1 and 3 only (d)  All of the above (a)  1 and 2 only (b)  2 and 3 only
Solution: (d) (c)  1 only (d)  All of the above
The collapse of communism was followed in Solution: (c)
most of these countries by a painful process On 19 March 2003, the US launched
of transition from an authoritarian socialist its invasion of Iraq under the codename
system to a democratic capitalist system. The ‘Operation Iraqi Freedom’. More than forty
model of transition in Russia, Central Asia other countries joined in the US-led ‘coalition
and east Europe that was influenced by the of the willing’ after the UN refused to give
World Bank and the IMF came to be known its mandate to the invasion. The ostensible
as ‘shock therapy’. purpose of the invasionwas to prevent Iraq
Each of these countries was required to from developing weapons of mass destruction
make a total shift to a capitalist economy, (WMD). Since no evidence of WMD has
which meant rooting out completely any been unearthed in Iraq, it is speculated
structures evolved during the Soviet period. that the invasion was motivated by other
Above all, it meant that private ownership was objectives, such as controlling Iraqi oilfields
to be the dominant pattern of ownership of and installing a regime friendly to the US.
property. Although the government of Saddam
Privatisation of state assets and corporate Hussein fell swiftly, the US has not been
ownership patterns were to be immediately able to pacify‘ Iraq. Instead, a full-fledged
brought in. Collective farms were to be insurgency against US occupation was
replaced by private farming and capitalism in ignited in Iraq.
agriculture. Q.2077 Operation Enduring Freedom (OEF)  is
Shock therapy also involved a drastic (a) The Global War on Terrorism by the
change in the external orientation of these Government of the United States of
economies. Development was now envisaged America
through more trade, and thus a sudden and (b) 2003 Invasion of Iraq by the USA
complete switch to free trade was considered (c) Bombings of Afghanistan and Sudan
essential. bases by American forces
The free trade regime and foreign direct (d) The Gulf War operations leading to the
investment (FDI) were to be the main engines buildup of troops in Saudi Arabia
of change. This also involved openness to Solution: (a)
foreign investment, financial opening up or Learning: The U.S. government used
deregulation, and currency convertibility. the term “Operation Enduring Freedom -
Finally, the transition also involved a Afghanistan” to officially describe the War
break-up of the existing trade alliances in Afghanistan, from the period between
among the countries of the Soviet bloc. October 2001 and December 2014. Continued
operations in Afghanistan by the United

Indian Polity Question Bank P.581

06-Indian Polity_Q2005-ENDD.indd 581 8/7/2018 7:51:32 PM


States’ military forces, both non-combat and 1985 and made concrete proposals for nuclear
combat, now occur under the name Operation disarmament. By not signing the Nuclear
Freedom’s Sentinel. Non-Proliferation Treaty (NPT) of 1968 and
Q.2078 “Operation Decisive Storm” and “Operation the Comprehensive Test Ban Treaty (CTBT)
Restoring Hope” were of 1996, India has kept its nuclear options
1. Started with the Arab Spring in Tunisia open. India opposes NPT and CTBT due to
2. Led by the NATO to tackle extremist their discriminatory and hegemonic nature.
forces in Syria and Tunisia They perpetuate an international system in
Which of the above is/are correct? which only five nations (USA, Russia, China,
(a)  1 only (b)  2 only UK and France) can legitimately possess
(c)  Both 1 and 2 (d)  None nuclear weapons.
Solution: (d) Q.2080 Gujral Doctrine of India advocates
Justification: The Saudi Arabian-led 1. Engaging more proactively with South
military intervention in Yemen is known as Asian neighbours
“Operation Decisive Storm”. 2. Complete disarmament by major nuclear
The new Operation Restoring Hope of powers
the Saudi Arabia in Yemen begins with 3. Suspension of the veto system in the
the following goals: continuing to protect United Nations Security Council
civilians, continuing to fight terrorism and Select the correct answer using the codes
continuing to facilitate the evacuation of below.
foreign nationals and to intensify relief and (a)  2 only (b)  1 and 3 only
medical assistance to the Yemeni people. (c)  1 only (d)  1, 2 and 3
So, both statements are wrong. Solution: (c)
Justification: The Gujral Doctrine is
India’s Foreign Policy a milestone in India’s foreign policy
propounded and initiated in 1996 by I.K.
Q.2079 The foreign policy of India advocates Gujral, the then Foreign Minister.
(a) Stopping the arms race leading to The doctrine is a five-point roadmap to
disarmament in both conventional and guide the conduct of India’s foreign relations
nuclear weapons with its immediate neighbours. These five
(b) Stopping the arms race in conventional principles are as follows:
weapons but retaining nuclear weapons • With the neighbours like Bangladesh,
(c) No particular stand on disarmament and Bhutan, Maldives, Nepal and Sri Lanka,
nuclear weapons stockpile India should not ask for reciprocity, but
(d) Development of the arms and nuclear given to them what it can in good faith.
weapons industry only to an extent of • No South Asian country should allow its
generating gainful employment territory to be used against the interest of
Solution: (a) another country of the region.
The foreign policy of India is opposed to • No country should interfere in the internal
arms race and advocates disarmament, both affairs of another country.
conventional and nuclear. This is aimed • All South Asian countries should respect
at promoting world peace and security each other’s territorial integrity and
by reducing or ending tensions between sovereignty.
power blocs and to accelerate economic • All South Asian countries should settle all
development of the country by preventing the their disputes through peaceful bilateral
unproductive expenditure on the manufacture negotiations.
of arms. India has been using the UNO The doctrine advocates that India, being
platform to check the arms race and to achieve the biggest country in South Asia, should
disarmament. India took the initiative of extend unilateral concessions to the smaller
holding a six-nation summit at New Delhi in neighbours.

P.582 For Civil Services Preliminary Examination

06-Indian Polity_Q2005-ENDD.indd 582 8/7/2018 7:51:32 PM


Q.2081 Which of these are core components of According to the Cold Start Doctrine, battle
Panchsheel? Groups will be well forward from existing
1. Non-Alignment garrisons. India’s elite strike forces will no
2. Anti-Racialism longer sit idle waiting for the opportune
3. Anti-Colonialism moment, giving Pakistan the luxury of time.
4. Neighbourhood First Policy Cold Start Doctrine was developed as the
5. No First Use policy limitations of the earlier doctrine – Sundarji
Select the correct answer using the codes Doctrine – were exposed after the attack on
below. the Indian Parliament.
(a)  1, 3 and 4 only (b)  2 and 5 only Q.2083 Which of the following is the primary
(c)  1, 2, 3, 4 and 5 (d)  None of the above objective of India’s foreign policy?
Solution: (d) (a) Neutrality in all matters of foreign
Justification: Panchsheel implies the five relations and policies to avoid strategic
principles of conduct in international alignment
relations. It was embodied in the Preamble (b) Promote an external environment
of the Indo-China Treaty on Tibet, signed conducive to domestic inclusive growth
in 1954. and development
The five principles were: (c) Increasing its political weight in the
• mutual respect for each other’s territorial World by controlling neighbour republics
integrity and sovereignty; (d) Advance territorial rights and claims to
• non-aggression; continuously increase India’s territory
• non-interference in each other’s internal Solution: (b)
affairs; Learning: India’s foreign policy seeks to
• equality and mutual benefit; and safeguard the country’s enlightened self-
• peaceful co-existence. interest.
India explained it as based on the concept The primary objective of India’s foreign
of universalism as against the concept of the policy is to promote and maintain a peaceful
balance of power”. and stable external environment in which
Panchsheel became very popular and the domestic tasks of inclusive economic
many countries of the world like Burma, development and poverty alleviation can
Yugoslavia, Indonesia and so on adopted progress rapidly and without obstacles.
it. Panchsheel and non-alignment are the Given the high priority attached by the
greatest contributions of India to the theory Government of India to socioeconomic
and practice of international relations. development, India has a vital stake in a
Q.2082 The Cold Start doctrine of India is intended to supportive external environment both in our
1. Build military alliances with major region and globally.
military powers to sabotage defence India, therefore, seeks a peaceful periphery
bases of Pakistan and that of terrorist and works for good neighbourly relations in
groups in Afghanistan its extended neighbourhood.
2. Allow India’s conventional forces Q.2084 Which of the following statement is
to perform holding attacks in order INCORRECT?
to prevent a nuclear retaliation from (a) India is not a signatory to the 1951
Pakistan in case of a conflict Refugee Convention.
Which of the above is/are correct? (b) India has a National refugee policy and
(a)  1 only (b)  2 only National Refugee protection law
(c)  Both 1 and 2 (d)  None (c) India is a member of the United Nations
Solution: (b) High Commissioner for Refugees
Justification: Cold Start is a military doctrine (UNHCR) executive committee.
developed by the Indian Armed Forces to put (d) All of the above
to use in case of a war with Pakistan. Solution: (b)

Indian Polity Question Bank P.583

06-Indian Polity_Q2005-ENDD.indd 583 8/7/2018 7:51:32 PM


Explanation: The Indian government has • As a result of different activities under
signed neither the Refugee Convention nor this programme, there is now a visible and
its Protocol, and Indian law does not offer any growing awareness among other countries
special language or provisions for refugees. about the competence of India as a provider
So, option (a) is a correct statement, and can’t of technical know-how and expertise as
be the answer. This also means that (d) can’t well as training opportunities, consultancy
be the answer. However, India is a member services and feasibility studies.
of the UNHCR executive committee. This • These programmes have generated immense
makes option (b) as the answer. goodwill and substantive cooperation
Learning: A possible reason for India not among the developing countries.
signing the Refugee convention may be that a Q.2086 What is India’s official position with respect
protection framework will encourage refugee to the Gilgit-Baltistan region?
inflow in India putting pressure on resources. (a) It views the region as an integral part of
South-Asian borders are also very porous and Afghanistan.
any conflict can result in a mass movement (b) It views it as an autonomous state where
of people. a referendum for independence should be
However, India does protect refugees. conducted as soon as possible.
The Indian refugee protection framework (c) It assigns legitimacy to Chinese plans of
is based on executive decisions; judicial development in this Pakistani region as
pronouncements; and international a part of its China-Pakistan Economic
conventions (1984 Convention against Corridor.
torture which prevents forcible repatriation of (d) It views it as part of Indian Territory
refugees). For example, under the RTA, even illegally occupied by Pakistan.
refugees can claim right to education. They Solution: (d)
are also entitled to public health facilities; Justification: Option B: India has flatly
justice systems etc. rejected this move claiming that Gilgit-
Q.2085 The Indian Technical and Economic Baltistan is an integral part of its territory (in
Cooperation (ITEC)  Programme assists J&K).
(a) Indian Universities for Higher education The Parliament has already passed a
(b) Developing countries both financially resolution declaring the entire Jammu and
and non-financially Kashmir as an integral part of India.
(c) Linguistic and religious Minorities all Learning: It is located in the northern
across the World Pakistan. It borders China in the North,
(d) All multi-national corporations operating Afghanistan in the west, Tajikistan in the
in India North West and Kashmir in the south east
Solution: (b) (see map).
Learning: It is a major plank of India’s • India sees it as part of Indian Territory
South-South foreign policy. illegally occupied by Pakistan.
Instituted in 1964, it is a bilateral • The China-Pakistan Economic Corridor
programme of assistance of the Government (CPEC)  also passes through this region.
of India. • Pakistan is planning to declare the
• Under ITEC and its sister programme strategic Gilgit-Baltistan region as its fifth
SCAAP (Special Commonwealth African Province, a move that may raise concerns
Assistance Programme), 161 countries in in India as it borders the disputed Pakistan-
Asia, Africa, East Europe, Latin America occupied Kashmir.
etc. are invited to share in the Indian Q.2087 Consider the following statements.
developmental experience acquired Assertion (A): India does not participate in
over 6 decades of India’s existence as a United Nations Peacekeeping operations.
free nation.

P.584 For Civil Services Preliminary Examination

06-Indian Polity_Q2005-ENDD.indd 584 8/7/2018 7:51:32 PM


Reason (R): India contributed to the Viet Nam) plus China, Japan, and Korea.
development of the policy of non-alignment It excludes India. So, 2 is incorrect.
(NAM) in international relations. Statement 3: You can see APEC member
In the context of the above, which of these states here. China has consistently blocked
is correct? India’s bid to APEC membership.
(a) A is correct, and R is an appropriate Q.2089 India is building emergency Strategic
explanation of A. Petroleum Reserves (SPRs) storages in
(b) A is correct, but R is not an appropriate underground caverns in southern India. Why
explanation of A. is it important for India to build such reserves?
(c)  A is correct, but R is incorrect. 1. India imports most of its oil from the
(d)  A is incorrect, but R is correct. Middle East which is a politically
Solution: (d) unstable region.
Justification: India has been the largest 2. A SPR guards against crude price
troop contributor to UN missions since its volatility in international markets.
inception. So, A is wrong. Which of the above is/are correct?
Recently Indian Peacekeepers were lauded (a)  1 only (b)  2 only
by the UN for their efforts in preventing a (c)  Both 1 and 2 (d)  None
carnage in the South Sudan conflict. Solution: (c)
The policy of NAM does not mean non- Justification: Statements 1 and 2: India
interventionist approach in international imports around 80% of its crude oil
affairs. Rather, the goal of peace building is requirement from the international market.
at the core of India’s foreign policy. Most of these imports come from Middle
Any member of UN can support and East which is unstable region.
undertake peacekeeping operations. India • Any price aberrations in crude oil prices
became a member of the UNO in 1945 have a direct impact on the Indian economy
itself. Since then, it has been supporting the especially upon the current account and
activities and programmes of UNO. inflation amounting to strategic risks.
Q.2088 India is a member country of which of the • Thus, it is necessary for India to have
following organizations? SPRs which will act as a valuable buffer
1. East Asia Summit against short term oil supply disruptions
2. ASEAN + 3 and global price shocks.
3. Asia-Pacific Economic Cooperation Learning: India will store more than 5
(APEC) million tonnes of crude oil in these SPRs
Select the correct answer using the codes enough to cover almost 2 weeks of demand.
below. They are located are in Visakhapatnam,
(a)  1 only (b)  1 and 3 only Mangalore and Padur.
(c)  2 and 3 only (d)  1 and 2 only • The erstwhile Planning Commission in
Solution: (a) its Integrated Energy Policy, 2006 also
Justification: Statement 1: 16 countries in had recommended to maintain a reserve
the East Asian, Southeast Asian, South Asian equivalent to 90 days of oil imports for
regions (including India), USA and RUSSIA strategic-cum-buffer stock purposes.
are its members. Trade is an important focus Q.2090 The “Shangri-La Dialogue”, an annual Asia
for the summit. Security Summit, is organized by
Statement 2: ASEAN+3 includes the (a)  Ministry of External Affairs, India
10 members of the Association of Southeast (b)  Shanghai Cooperation Organization
Asian Nations (Brunei Darussalam, (SCO)
Cambodia, Indonesia, Lao People’s (c)  International Institute for Strategic
Democratic Republic, Malaysia, Myanmar, Studies and the Singaporean Government
the Philippines, Singapore, Thailand, and (d)  United Nations Security Council (UNSC)
Solution: (c)

Indian Polity Question Bank P.585

06-Indian Polity_Q2005-ENDD.indd 585 8/7/2018 7:51:32 PM


Learning: It is the Raisina Dialogue Solution: (d)
that is organized jointly by Ministry of The Sri Lankan problem involves people
External Affairs and the Observer Research of Indian origin, and there is considerable
Foundation (ORF), an independent think pressure interests of the Tamils in Sri Lanka.
tank based in India. The government of India has from time to
time tried to negotiate with the Sri Lankan
Bilateral Relations government on the Tamil question.
But in 1987, the government of India for
(1) Indo-Sri Lanka the first time got directly involved in the Sri
Lankan Tamil question. India signed an accord
Q.2091 The “13th Amendment to Sri Lankan with Sri Lanka and sent troops to stabilise
Constitution” is sometimes seen in news. relations between the Sri Lankan government
Which major issue it is related to? and the Tamils. Eventually, the Indian Army
(a) Sharing of heritage island territory of got into a fight with the Liberation Tigers
Sri Lanka with India of Tamil Eelam (LTTE). The presence of
(b) Power sharing arrangement with the Indian troops was also not liked much by the
Indian government in Palk Bay Sri Lankans. They saw this as an attempt by
(c)  Political rights of Tamils in Sri Lanka India to interfere in the internal affairs of Sri
(d) Sri Lankan Military cooperation with Lanka. In 1989, the Indian Peace Keeping
South Asian neighbours Force (IPKF) pulled out of Sri Lanka without
Solution: (c) attaining its objective.
Learning: In 1987, Indo-Sri Lanka Accord
was signed between Indian Prime Minister (2) Indo-Pakistan
Rajiv Gandhi and Sri Lankan President J. R.
Jayewardene which stated the devolution of Q.2093 The conflicts between India and Pakistan in
powers to the Sri Lankan provinces (Tamil). 1948 and 1965 were possibly rooted in the
• The amendment aims at creating 1. Status of Kashmir
provincial councils in Sri lanka and 2. Status of Bangladesh
enable Sinhalese and Tamil as national 3. Control of the Siachen glacier
languages while preserving English as the Choose the correct answer using the codes
link language. below:
• However there are practical problems (a)  1 and 2 only (b)  2 and 3 only
in devolving land, police and financial (c)  1 and 3 only (d)  All of the above
powers to the provinces and the Solution: (c)
Government has stressed that the structure Soon after the partition, the two countries
that is implemented should be acceptable got embroiled in a conflict over the fate of
to all parts of the country. Kashmir.
The Pakistani government claimed that
Q.2092 In what ways had the Indian government
Kashmir belonged to it. Wars between India
involved itself in the problem of Sri Lankan
and Pakistan in 1947-1948 and 1965 failed to
Tamils in Sri Lanka since the origin of the
settle the matter. The 1947-1948 war resulted
problem?
in the division of the province into Pakistan-
1. Deploying Indian Peace Keeping forces
occupied Kashmir and the Indian province
in SL
of Jammu and Kashmir divided by the Line
2. Pushing SL diplomatically on the 13th
of Control. In 1971, India won a decisive
amendment to its constitution
war against Pakistan but the Kashmir issue
3. Taking up housing projects in war torn
remained unsettled.
areas
India’s conflict with Pakistan is also over
Choose the correct answer using the codes
strategic issues like the control of the Siachen
below:
glacier and over acquisition of arms. The
(a)  1 and 2 only (b)  2 and 3 only
arms race between the two countries assumed
(c)  1 and 3 only (d)  All of the above

P.586 For Civil Services Preliminary Examination

06-Indian Polity_Q2005-ENDD.indd 586 8/7/2018 7:51:32 PM


a new character with both states acquiring • India also shows the 1914 document
nuclear weapons and missiles to deliver which says that the creek is navigable
such arms against each other in the 1990s. during high tides and says that pillars
In 1998, India conducted nuclear explosion which were installed in 1924 were along
in Pokaran. Pakistan responded within a the midcourse.
few days by carrying out nuclear tests in the • Pakistan counters India’s claim by saying
Chagai Hills. Since then India and Pakistan that the creek is not a non-tidal river and
seem to have built a military relationship in thalweg doctrine can be used in case of
which the possibility of a direct and full-scale non-tidal rivers only.
war has declined. • Another important point is that Sir
Q.2094 India and Pakistan are not in agreement over Creek frequently changes its course and
the demarcation line in Sir Creek in the Rann this creates difficulty in demarcation of
of Kutch. It is important for both the nations boundaries.
because of Q.2096 Consider the following about Indus Waters
(a) the control of sea resources in the area Treaty (IWT), the water-distribution treaty
adjoining Sir Creek between India and Pakistan. Under the Treaty
(b) its implications for the Indus water treaty 1. The waters of Eastern Rivers are allocated
(c) the possibilities of effective coordination to India.
over containing marine piracy 2. India is under obligation not to use any
(d) the need to contain terrorism across the waters of the Western Rivers.
sea lanes of communication 3. Construction of water storage structures
Solution: (a) are banned on Western Rivers.
Q.2095 The Thalweg Doctrine sometimes seen in Select the correct answer using the codes
news is related to below.
(a) Track Two Dialogues between the North (a)  1 and 2 only (b)  1 only
and South (c)  1 and 3 only (d)  3 only
(b) Offensive Defence Nuclear Strategy Solution: (b)
(c)  India’s Look East Policy Justification: The treaty was brokered by the
(d) Sir Creek dispute between India and World Bank (erstwhile IBRD).
Pakistan • The treaty deals with the sharing of water
Solution: (d) of six rivers—Beas, Ravi, Sutlej, Indus,
Learning: Sir Creek is a tidal estuary which Chenab and Jhelum.
exist on the border of India and Pakistan • As per treaty, control over three eastern
(Gujarat state and Sind Province). The land rivers—Ravi, Beas and Sutlej was given
boundary between the countries upto Western to India. While control over three western
Terminus were fixed by UN Tribunal, rivers—Indus, Jhelum and Chenab was
but it left the boundary from thereon not given to Pakistan.
demarcated. This has become a conflicting • It allows India to use only 20% of the
issue between the two countries. water of Indus River, which flows through
• Pakistan says that boundary lines to the it first, for irrigation, power generation
eastern flank of creek (i.e. creek part of and transport.
Pakistan) and shows the agreement signed • Most disagreements and disputes have
in 1914 between Government of Sindh and been settled via legal procedures, provided
RaoMaharaj of Kutch to buttress its point. for within the framework of the treaty.
• On the other hand India also used the same • The treaty has survived India-Pakistan
document and ‘thalweg doctrine’ to claim wars of 1965, 1971 and the 1999 Kargil
that boundary lies in between the creek. standoff besides Kashmir insurgency
Thalweg doctrine says that boundary line since 1990. It is most successful water
must be midway through a navigable treaty in world.
channel.

Indian Polity Question Bank P.587

06-Indian Polity_Q2005-ENDD.indd 587 8/7/2018 7:51:32 PM


Q.2097 Consider the following statements about (3) Indo-Nepal
agreements between India and Pakistan.
1. The Simla Agreement sought to reverse Q.2099 Nepal and India enjoy a very special
the consequences of the 1999 Kargil war. relationship that has very few parallels in
2. The Lahore Declaration concerns the the world due to the Indo-Nepal Treaty of
responsibility of both nations’ leadership friendship 1950. It includes
towards avoiding nuclear race. 1. Porous borders
Which of the above is/are correct? 2. Work permit for Nepali nationals in India
(a)  1 only (b)  2 only 3. Allowing Nepali nations to acquire land
(c)  Both 1 and 2 (d)  None in India
Solution: (b) Choose the correct answer using the codes
Justification: Statement 1: It sought to below:
reverse the consequences of the 1971 war (a)  1 and 2 only (b)  2 and 3 only
(i.e. to bring about withdrawals of troops and (c)  1 and 3 only (d)  All of the above
an exchange of Prisoner of Wars). Solution: (d)
It was a comprehensive blue print for Q.2100 Consider the following about the 1950 India-
good neighbourly relations between India Nepal Treaty of Peace and Friendship.
and Pakistan. 1. It allows free movement of people and
Statement 2: Under the terms of the goods across the India-Nepal border.
treaty, a mutual understanding was reached 2. It prohibits Nepal from entering into
towards the development of atomic arsenals any military exercise with nations other
and to avoid accidental and unauthorised than India.
operational use of nuclear weapons. Which of the above is/are correct?
The Lahore Declaration signalled a major (a)  1 only (b)  2 only
breakthrough in overcoming the historically (c)  Both 1 and 2 (d)  None
strained bilateral relations in the aftermath of Solution: (a)
the 1998 publicly performed atomic tests. Justification: Statement 1: The treaty
Q.2098 The Coalition Support Fund (CSF) provides for:
programme to Pakistan given by the US is • Open border between the two countries
provided for • Allows Nepali nationals to work in India
(a) Developing tribal and backward areas in without a work permit
Pakistan • Allows them to apply for Indian
(b) Combating insurgency in neighbouring government jobs except for IAS, IPS
Afghanistan and IFS
(c) Improving defence bases of Pakistani • Allows Nepali nationals to open bank a/c
Army and own property in India
(d) Implementing several socioeconomic Statement 2: Nepal and China will hold
programs of USA running in Pakistan their first ever joint military exercise named
Solution: (b) ‘Sagarmatha Friendship-2017’ from April
Pakistan received $336 million from 17 to April 26, 2017. The 10 day long joint
the United States for its ongoing role exercise will focus on combating terror and
in combating a Taliban insurgency in disaster management.
neighbouring Afghanistan. Regular payments • Although, Nepal conducts joint exercises
to Pakistan under the Coalition Support with other countries like India and United
Fund (CSF) programme began in 2001 States, this is the first time it is holding a
when Pakistan joined the US-led coalition joint exercise with China.
in Afghanistan as a “frontline ally.” Pakistan • Even though the bilateral military
provides use of its air bases and other facilities engagement between Nepal and China
in exchange for the reimbursements. does not violate India-Nepal treaty of

P.588 For Civil Services Preliminary Examination

06-Indian Polity_Q2005-ENDD.indd 588 8/7/2018 7:51:32 PM


peace and friendship (1950), the exercise After India regained its independence from
does appear unconventional as Nepal has Britain, and China expelled the foreign powers,
conducted exercises with India earlier. there was hope that both would come together
India has agreed to provide fund of Rs 1500 to shape the future of the developing world
crore to Nepal’s Pancheshwar multi-purpose and of Asia particularly. For a brief while, the
project on Mahakali river. The major benefits slogan of ‘Hindi-Chini bhaibhai’ was popular.
to India from this project will be However military conflict over a border
Q.2101 The containment of Kheri floods in Uttar dispute between the two countries marred that
Pradesh hope. Soon after independence, both states
2. Procuring additional electricity produced were involved in differences arising from the
by the project Chinese takeover of Tibet in 1950 and the final
3. Irrigation of the Terai regions in India settlement of the Sino-Indian border. China
Choose the correct answer using the codes and India were involved in a border conflict
below. in 1962 over competing territorial claims
(a)  1 and 2 (b)  2 and 3 principally in Arunachal Pradesh and in the
(c)  1 and 3 (d)  All of the above Aksai Chin region of Ladakh.
Solution: (d) Q.2103 Consider the following statements about the
India will provide fund of Rs. 1500 crore to Maritime Silk Road (MSR) project.
Nepal’s Pancheshwar multi-purpose project 1. It has been initiated by China.
on Mahakali river for countering the Kheri 2. It involves building infrastructure in
floods in Uttar Pradesh. strategically significant countries, inter
Pancheshwar Multi-purpose Project alia, in Africa and Europe.
• It is located at Narayan Ghat of Dharchula 3. No ports will be built in Pakistan or
on Mahakali River (also known as Kali Bangladesh under MSR.
Ganga in Uttarakhand) in Nepal. Choose the correct answer using the codes
• After completion dam will be the second below.
largest dam in the world with a capacity of (a)  1 and 2 only (b)  2 and 3 only
6720 Mega Watt (MW) power production. (c)  1 and 3 only (d)  All of the above
• In case of power sharing, India has reached Solution: (a)
into an agreement with Nepal to procure • The 21st century MSR project is an
additional electricity produced by project. initiative by China to resurrect the ancient
• This project will regulate and control the maritime Silk Road. It is perceived to
free flow of Karnali and Mohana rivers be an attempt by China to ameliorate
which cause floods in Kheri, Pilibhit of relations with South and Southeast Asia-
Uttar Pradesh and other Terai districts of in this case the focus is on maritime trade
northern India. security.
• The project will prioritize construction
(4) Indo-China ports and infrastructure, industrial parks
in strategically significant countries in
Q.2102 Which of the following factors could have
Southeast Asia, the Mideast, Africa,
been responsible for the India-China conflict
Europe and the Indian Ocean region,
of 1962?
which includes Bangladesh and Sri Lanka.
1. Chinese takeover of Tibet in 1950
• China has already begun building of
2. Competing territorial claims in the
port projects in Gwadar (Pakistan),
Ladakh region
Hambantota (Sri Lanka)  and Chittagong
3. Chinese support to Pakistani army
(Bangladesh) respectively.
Choose the correct answer using the codes
• China has announced a USD 40 billion
below:
Silk Road fund which became operational
(a)  1 and 2 only (b)  2 and 3 only
in February 2015 for attracting small
(c)  1 and 3 only (d)  All of the above
countries in the projects.
Solution: (a)

Indian Polity Question Bank P.589

06-Indian Polity_Q2005-ENDD.indd 589 8/7/2018 7:51:32 PM


Q.2104 How is China-Pakistan Economic Corridor communication, which extend from the
(CPEC) important to China? Chinese mainland to Port Sudan.
1. The project will shorten the route • The sea lines run through several major
for China’s energy imports from the maritime choke points such as the Strait
Middle East. of Mandeb, the Strait of Malacca, the
2. It will strengthen Chinese presence in the Strait of Hormuz, and the Lombok Strait
Arabian Ocean. as well as other strategic maritime centres
Which of the above is/are correct? in Pakistan, Sri Lanka, Bangladesh, the
(a)  1 only (b)  2 only Maldives, and Somalia.
(c)  Both 1 and 2 (d)  None • The Chinese government insists that
Solution: (c) China’s burgeoning naval strategy is
Justification: Statement 1: The Straits of entirely peaceful in nature and is only for
Malacca provide China with its shortest the protection of regional trade interests.
maritime access to Europe, Africa, and the
Middle East. (5) Indo - US
• Approximately 80% pass of its Middle
Q.2106 The National Defence Authorization Act
Eastern energy imports also pass through
(NDAA), 2017 of the United States is
the Straits of Malacca.
significant because
• In addition to vulnerabilities faced in
1. It gives India’s “Major Defence Partner”
the Straits of Malacca region, China
status a statutory backing.
is heavily dependent upon sea-routes
2. It authorizes USA’s Asia-Pivot Policy.
that pass through the South China Sea,
Which of the above is/are correct?
near the disputed Spratly Islands and
(a)  1 only (b)  2 only
Paracel Islands.
(c)  Both 1 and 2 (d)  None
• The CPEC project will allow Chinese
Solution: (a)
energy imports to circumvent these
Justification: Statement 1: India has been
contentious areas and find a new artery
designated as a major defence partner of
in the west, and thereby decrease the
the United States. It puts India on a par with
possibility of confrontation between the
the closest allies and partners of the US. So
United States and China.
far, US has bestowed this status upon the
Statement 2: CPEC will connect Xinjiang (in
North Atlantic Treaty Organisation (NATO)
China) with Pakistan’s Gwadar Port (located
countries and the US treaty allies such as
at the shores of Arabian Ocean) through
Japan, South Korea, Australia, New Zealand
Pakistan-occupied Kashmir. Growing
and Philippines.
Chinese presence in western neighbourhood
Statement 2: There is no such provision.
is a strategic concern for India.
It is a strategic foreign policy matter of the
Q.2105 The String of Pearls theory, often seen in United States.
news, concerns Learning: Under the NDAA 2017, India
(a) Chinese military and commercial will be included in conventional arms
facilities in the Indian Ocean transfer policy, defence services, or related
(b)  Oil pipelines being laid around the technology, inform the review of requests to
North-South transport corridor export defence articles under the Arms Export
(c) Valuable mineral and stone exploration Control Act. Significance The designation
licensing regime in ocean beds of this status is unique to India, a non-NATO
(d) Network of diplomatic think tanks being ally of US. It institutionalizes the progress
established across the world made by US to facilitate defence trade and
Solution: (a) technology sharing with India to a level at
Learning: It refers to the network of par with its closest allies and partners. It will
Chinese military and commercial facilities facilitate US to transfer of advanced defence
and relationships along its sea lines of technology to India.

P.590 For Civil Services Preliminary Examination

06-Indian Polity_Q2005-ENDD.indd 590 8/7/2018 7:51:32 PM


Q.2107 It is said that the US-India Defense Indian Navy Joint Exercises
Technology and Partnership Act that • Varuna naval exercise: France and India.
proposes to amend the ‘Arms Export • SLINEX-Sri Lanka India
Control Action’ is of immense strategic • INDRA - India and Russia
significance to India. What can be the • Exercise Malabar: United States and
possible reason(s)? India.
1. It seeks to formalise India’s status as a • Simbex: Indian Navy with Republic of
major partner of equal status as US’s Singapore Navy
NATO allies and closest partners. • IBSAMAR with the Brazil and South
2. It will elevate India’s status in export of African navies.
defence articles from the United States • KONKAN: A bilateral Naval Exercise
to India. between Indian Navy and Royal Navy of
Which of the above is/are correct? Britain
(a)  1 only (b)  2 only Indian Army Joint Exercises
(c)  Both 1 and 2 (d)  None • Mitra Shakti Exercise: India and Sri Lanka.
Solution: (c) • Hand-in-Hand: India-China Joint Military
Learning: Defence trade between the US Training Exercise
and India is one of the strongest areas of the • Exercise Shakti: India and France Armies
bilateral economic relationship. Over the past • Exercise Nomadic Elephant: Indian Army
10 years it has risen from $300 million to exercises with the Mongolian Army.
over $14 billion. The act has been introduced • Exercise Yuddh Abhyas is a series of joint
by Congressman George Holding, Co-Chair exercises between the Indian and United
of the House India Caucus. It will help States Armies since 2005.
• Cement the India-US relations and lay • Surya Kiran: India Nepal Joint Military
a foundation for future cooperation exercise
and growth. • Lamitye: India and Seychelles
• Shorten the time required for the • Prabal Dostyk: India Kazhakhstan Joint
notification of sale or export of defence Exercise
articles from the United States to India.
• Encourage more joint contingency International Organisations
planning and require the US government
review and assess India’s ability to execute Q.2109 The Bretton Woods institutions were
military operations of mutual interest. established with which of the following
objectives?
1. Finance post-world war II reconstruction
Joint Military Exercises 2. To deal with external surpluses and
Q.2108 These are some of the joint military deficits of member nations
exercises India holds with several countries. 3. To ensure full employment in developed
Consider the matches of exercises with the countries
partner nations. Select the correct answer using the codes
1. Indradhanush: India-UK below.
2. Varuna: India-Sri Lanka (a)  1 and 2 only (b)  2 and 3 only
3. Exercise Malabar: India-USA (c)  1 only (d)  All of the above
4. Surya Kiran: India-Nepal Solution: (a)
Select the correct answer using the codes Justification and Learning: at the UN
below. monetary and financial Conference held
(a)  1 and 3 only (b)  2 and 4 only in 1944, at Bretton Woods, USA, it was
(c)  1, 3 and 4 only (d)  1, 2, 3 and 4 decided to establish IMF and International
Solution: (c) Bank for reconstruction and Development
Justification: These are frequently in news. (World Bank). IMF would deal with external

Indian Polity Question Bank P.591

06-Indian Polity_Q2005-ENDD.indd 591 8/7/2018 7:51:32 PM


surpluses and deficits of member nations. on the supervision and management of the
The IBRD would setup to finance post-war international monetary and financial system,
reconstruction. So, 1 and 2 is correct. The including on responses to unfolding events
goal of full employment is dealt with national that may disrupt the system.
governments. So, the option 3 is wrong. Statement 2: Original aims of IMF:
Q.2110 Which of the following statements about the • promote international monetary
League of Nations is INCORRECT? cooperation;
(a) It was the first international organisation • facilitate the expansion and balanced
whose principal mission was to maintain growth of international trade;
world peace. • promote exchange stability;
(b) It aimed at collective security and • assist in the establishment of a multilateral
disarmament. system of payments and
(c) USA did not join the league. • make resources available (with adequate
(d) Its membership was open only to safeguards) to members experiencing
industrially developed nations. balance of payments difficulties.
Solution: (d) Statement 3: Biggest borrowers from IMF
Justification: The need for maintaining (amounts outstanding as of 2015 first
world peace was visibly felt after WW-I. quarter): Portugal, Greece, Ireland, Ukraine.
Hence, the League of Nations (LoN) came Q.2112 Which of the following represents India at
into existence in 1920 as a result of the Paris the IMF?
Peace Conference that ended the First World (a) An Executive Director nominated by the
War. So, (a) is correct. government
Its primary goals, as stated in its Covenant, (b)  Union Finance Secretary
included preventing wars through collective (c) Head of the largest Public Sector Bank
security and disarmament and settling (PSB)
international disputes through negotiation (d)  Finance Minister
and arbitration. So, the option (b) is correct. Solution: (a)
Many developing nations like Ecuador, Learning: Finance Minister is the ex-officio
Paraguay joined the league. So, (d) is Governor on the Board of Governors of the
incorrect. It was dissolved in 1946 to form IMF.
United Nations. The RBI Governor is the Alternate
Governor at the IMF.
(1) International monetary fund India is represented at the IMF by an
(IMF) Executive Director, who also represents three
other countries as well, viz. Bangladesh, Sri
Q.2111 Consider the following statements with
Lanka and Bhutan.
reference to the International Monetary Fund
(IMF). Q.2113 Which of these are recent reforms in the
1. It is governed by the International Monetary International Monetary Fund (IMF)?
and Financial Committee. (IMFC) 1. Voting quotas of some emerging
2. One of the aims of IMF is to promote economies has been increased.
exchange stability. 2. IMF’s Executive Board will now consist
3. India is the biggest borrower from IMF. entirely of elected
Select the correct answer using the codes Executive Directors.
below. Which of the above is/are correct?
(a)  1 and 2 only (b)  2 and 3 only (a)  1 only (b)  2 only
(c)  1 and 3 only (d)  2 only (c)  Both 1 and 2 (d)  None
Solution: (d) Solution: (c)
Justification: Statement 1: The IMFC advises Justification: Statement 1: India’s voting
and reports to the IMF Board of Governors rights increased by 0.3% from the current
2.3% to 2.6%. China’s voting rights increased

P.592 For Civil Services Preliminary Examination

06-Indian Polity_Q2005-ENDD.indd 592 8/7/2018 7:51:33 PM


by 2.2% from current 3.8% to 6 %. These Q.2115 The SDR is an international reserve asset,
reforms shifted more than 6% of the quota created by the IMF in 1969 to supplement its
shares to emerging and developing countries member countries’ official reserves. Consider
from the US and European countries. Russia the following about Special Drawing
and Brazil also have gained from the reforms. Rights (SDRs).
Statement 2: Currently, US, Japan, 1. SDRs cannot be exchanged for freely
France, Germany, Italy, United Kingdom, usable currencies.
Canada and Saudi Arabia are among the top 2. Value of a SDR is fixed on an annual basis
ten members of the IMF. and revised on the discretion of IMF.
But, the member countries with the Which of the above is/are correct?
5 largest quotas appointed the Executive (a)  1 only (b)  2 only
Director, which will be made to an entirely (c)  Both 1 and 2 (d)  None
elected system now. So, 2 is correct. Solution: (d)
Learning: The reforms were agreed upon by Justification: Statement 1: It was created as
the 188 members of the IMF in 2010 in the a supplementary international reserve asset.
aftermath of the global financial meltdown. SDRs can be exchanged for freely usable
However, there implementations were currencies. So, 1 is wrong.
delayed due to the time taken by the US The value of the SDR is currently based
Congress to approve the changes. on a basket of four major currencies: the U.S.
Q.2114 The legitimacy of Special Drawing dollar, euro, the Japanese yen, and pound
Rights (SDR) being used for international sterling. The basket will be expanded to
transactions comes from include the Chinese renminbi (RMB) or yuan
1. IMF authorising its use as the fifth currency, effective 1 October 2016.
2. Nations willing to use it Statement 2: Its value is determined daily
3. Nearly constant value of the SDR based on the currency values of its constituent
Choose the correct answer using the codes currencies.
below. The SDR is neither a currency, nor a claim
(a)  1 and 3 only (b)  2 and 3 only on the IMF. Rather, it is a potential claim on
(c)  1 and 2 only (d)  All of the above the freely usable currencies of IMF members.
Solution: (d) Q.2116 A first-of-its-kind South Asia Training
The SDR is an international reserve asset, and Technical Assistance Centre
created by the IMF in 1969 to supplement its (SARTTAC) was recently opened in India
member countries’ official reserves. Its value by International Monetary Fund (IMF) for
is based on a basket of five key international 1. Socio-economic capacity building in
currencies, and SDRs can be exchanged for South Asia
freely usable currencies. 2. Addressing immediate liquidity crunch in
The SDR is neither a currency, nor a claim foreign exchange
on the IMF. Rather, it is a potential claim on Which of the above is/are correct?
the freely usable currencies of IMF members. (a)  1 only (b)  2 only
Holders of SDRs can obtain these currencies (c)  Both 1 and 2 (d)  None
in exchange for their SDRs in two ways: Solution: (a)
first, through the arrangement of voluntary Justification: It will work to support local
exchanges between members; and second, member countries of South Asia viz. India,
by the IMF designating members with strong Bangladesh, Bhutan, Maldives, Nepal and
external positions to purchase SDRs from Sri Lanka to build human and institutional
members with weak external positions. capacity and implement policies for growth
In addition to its role as a supplementary and poverty reduction.
reserve asset, the SDR serves as the unit • The SARTTAC is financed mainly by
of account of the IMF and some other its six member South Asia countries
international organizations. (mentioned above) with additional support

Indian Polity Question Bank P.593

06-Indian Polity_Q2005-ENDD.indd 593 8/7/2018 7:51:33 PM


from Australia, South Korea, European (d) High growth rates sustained in the East
Union and United Kingdom. Asian economies due to heavy focus on
• It will allow the IMF to meet more of the industries instead of agriculture.
high demand for technical assistance and Solution: (a)
training from the region. It is expected to The East Asian miracle was all about the
become the focal point for the delivery right mixture of state and market forces. The
of IMF capacity development services to WB had drawn many conclusions based on
South Asia. this right set of mixture. Such a model could
• The opening of SARTTAC is part of the be replicated in the whole world and served
MoU (between IMF and India and other as an important lesson for the world. With
nations) and marks a major milestone in the right set of interventions by the state, the
the partnership between the IMF and its economy was able to sustain high growth
member countries in the region rates by reviving demand as and when
required. Not only this, but the opening of
(2) World Bank industries and regulating competition in the
markets ensured fair-play.
Q.2117 How is the World Bank Group (WBG)
President selected? Q.2119 Consider the following about the International
(a) Nominated by the largest shareholder Bank for Reconstruction and Development
of WBG subject to confirmation by the (IBRD).
Board of Governors 1. It was created to help Europe rebuild after
(b) Elected on a majority basis by the the Second World War.
shareholder member states of the WBG 2. It is owned jointly by the United States
(c) Elected by the United Nations General and European Union.
Assembly (UNGA) on a two-thirds 3. It supports long-term human and social
majority basis development that private creditors may
(d) Appointed by the UN Secretary General not finance.
subject to approval by the Board of 4. It provides advisory services in public
Governors debt and asset management to help
Solution: (a) governments.
Learning: Traditionally, the Bank President Select the correct answer using the codes
has always been a U.S. citizen nominated below.
by the President of the United States, (a)  1 and 2 only (b)  1 and 3 only
the largest shareholder in the bank. The (c)  3 and 4 only (d)  1, 3 and 4 only
nominee is subject to confirmation by the Solution: (d)
Board of Governors, to serve for a five-year, Justification: Statement 1: It was created
renewable term. The president is responsible in 1944 to help Europe rebuild after World
for chairing the meetings of the Boards of War II. Today, IBRD provides loans and
Directors and for overall management of the other assistance primarily to middle income
World Bank Group. countries.
Statement 2: IBRD is owned by the
Q.2118 The East Asian Miracle‘ (World Bank
governments of its 189 member countries,
Reports) refers to which of the following:
not by US and EU alone.
(a) High growth rates sustained in the East
Statement 3: IBRD is the original World
Asian economies due to appropriate state
Bank institution. It works closely with the rest
intervention.
of the World Bank Group to help developing
(b) High growth rates sustained in the East
countries reduce poverty, promote economic
Asian economies due to success of
growth, and build prosperity.
market led growth.
Specifically, IBRD (apart from
(c) High growth rates sustained in the East
statement 3, which is correct):
Asian economies due to a completely
centralized planning process.

P.594 For Civil Services Preliminary Examination

06-Indian Polity_Q2005-ENDD.indd 594 8/7/2018 7:51:33 PM


• Preserves borrowers’ financial strength by developing countries to support economic
providing support in times of crisis, when growth, reduce poverty, and improve
poor people are most adversely affected people’s lives.
• Promotes key policy and institutional • The International Centre for Settlement
reforms (such as safety net or anti- of Investment Disputes (ICSID)  provides
corruption reforms) international facilities for conciliation and
Statement 4: The IBRD’s financial products arbitration of investment disputes.
and services help countries build resilience
to shocks by facilitating access to products (3) WTO
that mitigate the negative impact of currency,
Q.2121 Consider the following about the “Annual
interest rate, and commodity price volatility,
Public Forum”, WTO’s flagship event.
natural disasters and extreme weather.
1. It is the publisher of the “World Trade
Q Source: Improvisation: Page 40:Indian
Statistical Review” since 1995.
Economic Development: Class XIth NCERT
2. It provides a common platform for heads
Q.2120 Which of these institutions form part of the of state as well as NGOs to discuss
World Bank Group (WBG)? major trade and developmental issues.
1. International Finance Corporation (IFC) Which of the above is/are correct?
2. International Monetary Fund (IMF) (a)  1 only (b)  2 only
3. Multilateral Investment Guarantee (c)  Both 1 and 2 (d)  None
Agency (MIGA) Solution: (b)
4. United Nations Economic and Social Justification: Statement 1: The WTO
Council (UNESCO) has launched a new annual statistical
Select the correct answer using the codes publication—the “World Trade Statistical
below. Review”—in 2016. This new publication
(a)  1, 2 and 3 only provides insights into how world trade has
(b)  1 and 3 only evolved in recent years by analysing the
(c)  2 and 4 only latest trade statistics within an economic
(d)  1, 2 and 4 only context. So, 1 is wrong.
Solution: (b) Statement 2: It provides a unique
Justification: The World Bank Group platform for heads of state and leading
consists of five organizations: IBRD, IDA, global businesspeople, academics and
IFC, MIGA and ICSID. In contrast, ‘World non-governmental organisations to come
Bank’ consists of only two organizations: together and discuss some of the major
IBRD and IDA. trade and development issues of the day.
• The International Bank for Reconstruction Over 1,500 participants attend the Forum
and Development (IBRD) lends to each year.
governments of middle-income and
Q.2122 Preferential Market Access (PMA) policy
creditworthy low-income countries.
adopted by India in the National Electronics
• The International Development
Policy 2012 violates international trade
Association (IDA) provides interest-free
norms because:
loans—called credits—and grants to
1. Foreign Suppliers are not treated at par
governments of the poorest countries.
with domestic suppliers
• The International Finance Corporation
2. India will stop importing sensitive
(IFC) is the largest global development
electronic equipment for security- related
institution focused exclusively on the
reasons
private sector.
3. WTO does not allow any discrimination
• The Multilateral Investment Guarantee
between foreign goods from different
Agency (MIGA) was created in 1988 to
countries
promote foreign direct investment into

Indian Polity Question Bank P.595

06-Indian Polity_Q2005-ENDD.indd 595 8/7/2018 7:51:33 PM


Select the correct answer using the codes 3. India has not ratified the TFA because
given below the TFA is discriminatory in nature and
(a)  1 and 2 (b)  1 and 3 benefits the developed nations more than
(c)  2 only (d)  Only 1 the developing nations.
Solution: (d) Choose the correct statements from the codes
Q.2123 Green Box subsidies under WTO are allowed below:
because they are considered to be (a)  Only 1 (b)  2 and 3
(a)  Minimally trade distorting. (c)  All of the above (d)  1 and 2
(b) Higher among developed and relatively Solution: (d)
lower for developing countries. http://www.wto.org/english/tratop_e/
(c) They are confined to agriculture sector. tradfa_e/tradfa_e.htm
(d) They are related to GATS provision. http://www.dnaindia.com/money/report-
Solution: (a) why-is-india-not-agreeing-to-the-wto-trade-
Justification: In WTO terminology, facilitation-agreement-2005181
subsidies in general are identified by “boxes” Q.2125 Which among the following are components
which are given the colours of traffic lights: of multi-lateral trading principles as laid out
green (permitted), amber (slow down—i.e. by the World Trade Organization (WTO)?
be reduced), red (forbidden). 1. National Treatment
• The Agriculture Agreement has no 2. Reciprocity
red box, although domestic support 3. Protectionism
exceeding the reduction commitment 4. Transparency in trade regulations
levels in the amber box is prohibited; Select the correct answer using the codes
and there is a blue box for subsidies below.
that are tied to programmes that limit (a)  1, 2 and 4 only
production. (b)  3 and 4 only
• There are also exemptions for developing (c)  1 and 2 only
countries (sometimes called an S&D (d)  2, 3 and 4 only
box). Solution: (a)
• In order to qualify, green box subsidies Justification: Statement 1: Non-
must not distort trade, or at most cause discrimination is the pillar behind this rule.
minimal distortion. So, (a) is correct. It has two major components: the Most
• They have to be government-funded (not Favoured Nations (MFN) rule, and the
by charging consumers higher prices) and national treatment policy.
must not involve price support. The MFN rule requires that a WTO
• They also include environmental member must apply the same conditions on
protection and regional development all trade with other WTO members.
programmes. So, the option (c) is wrong. National treatment means that imported
Q.2124 Consider the following statements about the goods should be treated no less favourably
Trade facilitation Agreement signed between than domestically produced goods.
the WTO member nations at Bali Ministerial Statement 2: For a nation to negotiate, it
conference: is necessary that the gain from doing so be
1. The Trade Facilitation Agreement (TFA) greater than the gain available from unilateral
contains provisions for faster and more liberalization; reciprocal concessions intend
efficient customs procedures in the to ensure that such gains will materialise
context of trade in goods. Statement 3: This is the opposite of
2. It also contains provisions for technical openness that WTO advocates.
assistance and capacity building for trade Statement 4: The WTO members are
facilitation and customs compliance required to publish their trade regulations, to
issues. maintain institutions allowing for the review
of administrative decisions affecting trade, to

P.596 For Civil Services Preliminary Examination

06-Indian Polity_Q2005-ENDD.indd 596 8/7/2018 7:51:33 PM


respond to requests for information by other or elimination in tariffs, particularly on
members, and to notify changes in trade exportable goodsof interest to developing
policies to the WTO. countries.
Q.2126 “Uruguay Round” is often heard in the The WTO considers the NAMA
context of which of these international negotiations important because NAMA
organizations? products account for almost 90% of the
(a)  World Bank Group world’s merchandise exports.
(b) United Nations Framework Convention
on Climate Change (UNFCCC) (4) UN System
(c)  World Trade Organization (WTO) Q.2128 Consider the following statements.
(d) Food and Agriculture Organization Assertion (A): The United Nations does not
(FAO) possess any authority to recognize either a
Solution: (c) State or a Government.
Learning: The negotiations at Uruguay led Reason (R): The United Nations is neither
to the creation of WTO from GATT. a State nor a Government.
So, this Round brought about the biggest In the context of the above, which of these
reform of the world’s trading system since is correct?
GATT was created at the end of the Second (a) A is correct, and R is an appropriate
World War. explanation of A.
Within only 2 years, member nations of (b) A is correct, but R is not an appropriate
WTO agreed on a package of cuts in import explanation of A.
duties on agricultural products. (c)  A is correct, but R is incorrect.
The dispute resolution mechanism, that (d)  Both A and R are incorrect.
stands at the core of WTO, was strengthened Solution: (a)
by this round. Justification: States are admitted to
Q.2127 NAMA-11 (Nama-11) group of countries is membership in the United Nations by
often mentioned in the news in context of? decision of the General Assembly upon the
(a) Missile Technology Control Regime recommendation of the Security Council.
(b)  International Energy Agency The recognition of a new State or
(c)  World Economic Forum Government is an act that only other States
(d)  World Trade Organization and Governments may grant or withhold.
Solution: (d) As an organization of independent States,
Learning: It is a group of eleven developing the UN can only admit a new State to its
countries working toward strengthening membership or accept the credentials of the
Non-agricultural market access (NAMA) representatives of a new Government.
in WTO. Q.2129 Which of the following does NOT come
The group has two main objectives: within the ambit of the United Nations (UN)?
• Supporting flexibilities for developing (a) Maintain international peace and security
countries (b)  Protect Human Rights
• Maintaining the balance between NAMA (c)  Upholding International Law
and other areas under negotiation. (d)  All come in its ambit.
Member countries of NAMA-11 are Solution: (d)
Argentina, Bolivarian Republic of Venezuela, Justification: Due to the powers vested
Brazil, Egypt, India, Indonesia, Namibia, in its Charter and its unique international
Philippines, South Africa and Tunisia. character, the United Nations can take action
The Non-Agricultural Market Access on the issues confronting humanity in the 21st
(NAMA)  negotiations of the World century, such as peace and security, climate
Trade Organization are based on the Doha change, sustainable development, human
Declaration of 2001 that calls for a reduction rights, disarmament, terrorism, humanitarian

Indian Polity Question Bank P.597

06-Indian Polity_Q2005-ENDD.indd 597 8/7/2018 7:51:33 PM


and health emergencies, gender equality, • All Members shall give the United Nations
governance, food production, and more. every type of assistance in any action it takes
Option (a): The Security Council has in accordance with the present Charter, and
primary responsibility, under the United shall refrain from giving assistance to any
Nations Charter, for the maintenance of state against which the United Nations is
international peace and security. taking preventive or enforcement action.
Option (c): The UN Charter, in its • The Organization shall ensure that states
Preamble, set an objective: “to establish which are not Members of the United
conditions under which justice and respect Nations act in accordance with these
for the obligations arising from treaties and Principles so far as may be necessary for
other sources of international law can be the maintenance of international peace
maintained”. and security.
Option (b): Bodies like UNHCR achieve • Nothing contained in the present Charter
the objective apart from the general shall authorize the United Nations to
agreements and resolutions passed by UN. intervene in matters which are essentially
Q.2130 Which of these is/are NOT principle(s) of within the domestic jurisdiction of any
United Nations? state or shall require the Members to
1. Some sovereigns are more equal than submit such matters to settlement under
others. the present Charter; but this principle
2. Member nations have rights but no shall not prejudice the application of
obligations. enforcement measures under Chapter Vll.
3. All members shall settle their international Q.2131 Which of the following are NOT the principal
disputes by peaceful means. organs of the United Nations (UN)?
4. All members shall settle their national 1. International Court of Justice
disputes by peaceful means. 2. Economic and Social Council
Choose the correct answer using the codes 3. WTO
below. Choose the correct answer using the codes
(a)  1 only (b)  2 and 3 only below:
(c)  3 and 4 only (d)  1, 2 and 4 only (a)  1 and 2 only (b)  2 and 3 only
Solution: (d) (c)  3 only (d)  1 and 3 only
The Organization and its Members, in pursuit Solution: (c)
of the Purposes stated in Article 1, shall act Explanation for this and the next three
in accordance with the following Principles. questions are together.
• The Organization is based on the principle
of the sovereign equality of all its Members. (A) UN General Assembly (UNGA)
• All Members, in order to ensure to all
Q.2132 Consider the following statements about the
of them the rights and benefits resulting
manner in which decisions are made in the
from membership, shall fulfill in good
UN General Assembly (UNGA).
faith the obligations assumed by them in
1. Each member state has one vote.
accordance with the present Charter.
2. All decisions are passed by a two-thirds
• All Members shall settle their international
majority.
disputes by peaceful means in such a
3. Its decisions are not binding on all UN
manner that international peace and
members.
security, and justice, are not endangered.
Choose the correct answer using the codes
• All Members shall refrain in their
below:
international relations from the threat or
(a)  1 only (b)  2 and 3 only
use of force against the territorial integrity
(c)  1 and 3 only (d)  All of the above
or political independence of any state, or
Solution: (c)
in any other manner inconsistent with the
Explanation above.
Purposes of the United Nations.

P.598 For Civil Services Preliminary Examination

06-Indian Polity_Q2005-ENDD.indd 598 8/7/2018 7:51:33 PM


Q.2133 Voting shares in the United Nations General Justification: Statements 1 and 3: Most
Assembly are distributed between members of these operations are established and
on the basis of implemented by the United Nations itself,
(a) Population with troops serving under UN operational
(b)  Economic Size control.
(c)  Contribution to UN In these cases, peacekeepers remain
(d)  All members have equal vote members of their respective armed forces,
Solution: (d) and do not constitute an independent “UN
Learning: Voting in the General army,” as the UN does not have such a force.
Assembly on important questions, namely, In cases where direct UN involvement is
recommendations on peace and security, not considered appropriate or feasible, the
budgetary concerns and the election, Council authorizes regional organizations
admission, suspension or expulsion of such as the NATO or coalitions of willing
members – is by a two-thirds majority of countries to undertake peacekeeping or
those present and voting. peace-enforcement tasks.
Other questions are decided by a Statement 2: The UN Charter gives the
straightforward majority. Each member UNSC the power and responsibility to take
country has one vote. collective action to maintain international
Apart from approval of budgetary matters, peace and security. For this reason, the
including adoption of a scale of assessment, international community usually looks to the
Assembly resolutions are not binding on the UNSC to authorize peacekeeping operations
members. through Chapter VI authorizations.
The Assembly may make Q.2135 Which United Nations Organ is charged with
recommendations on any matters within the maintenance of international peace and
the scope of the UN, except matters of security as well as recommending admission
peace and security under Security Council of new members to the United Nations (UN)?
consideration. (a)  UN Secretariat
The one state, one vote power structure (b)  UN Trusteeship Council
potentially allows states comprising just (c)  International Court of Justice
5 percent of the world population to pass a (d)  UN Security Council
resolution by a two-thirds vote. Solution: (d)
Explanation & Learning: Under the
(B) United Nations Security Council Charter, the Security Council has primary
(UNSC) responsibility for the maintenance of
international peace and security. It has
Q.2134 Consider the following statements about 15 Members, and each Member has one vote.
United Nations (UN) peacekeeping Under the Charter, all Member States are
operations. obligated to comply with Council decisions.
1. UN maintains an independent The Security Council takes the lead
peacekeeping army funded by in determining the existence of a threat
contributions from member states. to the peace or act of aggression. It calls
2. UN Security Council authorizes upon the parties to a dispute to settle it by
peacekeeping operations. peaceful means and recommends methods of
3. Regional organizations can be authorized adjustment or terms of settlement. In some
to undertake peacekeeping operations. cases, the Security Council can resort to
Select the correct answer using the codes imposing sanctions or even authorize the use
below. of force to maintain or restore international
(a)  1 and 2 only (b)  2 and 3 only peace and security. The Security Council
(c)  2 only (d)  1 and 3 only also recommends to the General Assembly
Solution: (b) the appointment of the Secretary-General

Indian Polity Question Bank P.599

06-Indian Polity_Q2005-ENDD.indd 599 8/7/2018 7:51:33 PM


and the admission of new Members to the • A whole new set of challenges confronts
United Nations. And, together with the the world (genocide, civil war, ethnic
General Assembly, it elects the judges of the conflict, terrorism, nuclear proliferation,
International Court of Justice. climate change, environmental
Q.2136 Consider the following statements about degradation, epidemics).
the composition and working of the United Q.2138 Which of the following are NOT the
Nations Security Council (UNSC). permanent members of the United Nations
1. Its non-permanent members are elected Security Council (UNSC)?
by the UNGA for a term of 2 years. 1. Russia
2. Its decision is binding on all UN members. 2. Japan
3. Any expansion in the composition of the 3. Germany
UNSC will need the approval of both the Choose the correct answer using the codes
UNSC and UNGA. below:
Choose the correct answer using the codes (a)  1 and 2 (b)  2 and 3
below: (c)  1 and 3 (d)  All of the above
(a)  1 only (b)  2 and 3 only Solution: (b)
(c)  1 and 3 only (d)  All of the above The fifteen-member Security Council of the
Solution: (d) UN takes such crucial decisions. The Council
Explanation above. has five permanent members – US, Russia,
Q.2137 Apart from the changed power relations in UK, France and China. Ten other members
the World, which of the following arguments are elected by the General Assembly for
would force a revisit of the composition of 2-year terms. The real power is with five
the UNSC? permanent members.
1. The economies of Asia are growing at an Q.2139 Peacekeeping operations of the United
unprecedented rate. Nations are authorized by
2. A whole new set of challenges confronts (a) United Nations General Assembly
the world like climate change, ethnic (UNGA)
conflicts. (b) United Nations Human Rights Council
3. Democratic global governance is only (UNHRC)
a logical extension of the spread of (c) United Nations Security Council (UNSC)
democracy across the globe. (d) A group of nations in the UN that
Choose the correct answer using the codes are stakeholders in the peacekeeping
below: operations
(a)  1 and 2 only (b)  2 and 3 only Solution: (c)
(c)  1 and 3 only (d)  All of the above Learning: The UN Charter gives the UNSC
Solution: (d) the power and responsibility to take collective
After the Cold War, the realities of the action to maintain international peace and
globe are different that force a revisit of the security. For this reason, the international
composition of the UNSC. Here are some of community usually looks to the UNSC to
the changes that have occurred that revisit: authorize peacekeeping operations through
• The Soviet Union has collapsed. Chapter VI authorizations.
• The US is the strongest power. Most of these operations are established
• China is fast emerging as a great power, and implemented by the United Nations itself,
and India also is growing rapidly. with troops serving under UN operational
• The economies of Asia are growing at an control.
unprecedented rate. In these cases, peacekeepers remain
• Many new countries have joined the UN members of their respective armed forces,
(as they became independent from the and do not constitute an independent “UN
Soviet Union or former communist states army,” as the UN does not have such a force.
in Eastern Europe).

P.600 For Civil Services Preliminary Examination

06-Indian Polity_Q2005-ENDD.indd 600 8/7/2018 7:51:33 PM


Q.2140 Consider the following about the United “substantive” resolution, as well as decide
Nations Security Council (UNSC). which issues fall under “substantive” title.
1. It is the only UN body with the This de facto control over the UN Security
authority to issue binding resolutions to Council by the five governments is seen by
member states. critics, since its creation in 1945, as the most
2. All decisions of the UNSC must be undemocratic character of the UN.
approved by the UN General Assembly The veto is exercised when any permanent
to be enforced. member—the so-called “P5”—casts a
3. It recommends new states for admission “negative” vote on a “substantive” draft
as member states of the United resolution. Abstention or absence from the
Nations (UN). vote by a permanent member does not prevent
Which of the above is/are correct? a draft resolution from being adopted.
(a)  1 and 2 only (b)  2 only Q.2142 The non-permanent members of the United
(c)  1 and 3 only (d)  None of the above Nations Security Council (UNSC)  are
Solution: (c) (a) Elected for two-year terms by the General
Justification: Decisions that are vetoed by Assembly
the P-5 cannot be submitted for the review of (b) Nominated by the permanent members
the UNGA. Also, the UNSC is the authority of UNSC
under the UN charter to take action in (c) Appointed by the UN Secretary General
situations threatening international peace and (d) Selected on rotation basis from the
security, e.g. Syria crisis. So, 2 is incorrect. members of the General Assembly
Decisions taken under Chapter VII, such Solution: (a)
as economic sanctions or other sections like Learning: The Council is composed of
international security, are binding on UN 15  Members: five permanent members:
members; the Security Council is the only China, France, Russian Federation, the
UN body with the authority to issue binding United Kingdom, and the United States, and
resolutions. The UNGA can only make ten non-permanent members.
recommendations in this regard. So, 1 is More than 60 United Nations Member
correct. States have never been Members of the
Q.2141 Apart from the Permanent members of the Security Council. A State which is a Member
UNSC, who/which among the following of the United Nations but not of the Security
have the right to use veto power to stall key Council may participate, without a vote, in
decisions in the United Nations (UN)? its discussions when the Council considers
1. Founding members of the UN that that country’s interests are affected. Both
2. UN Secretary General Members and non-members of the United
3. Countries that contribute substantially to Nations, if they are parties to a dispute being
the UN Budget considered by the Council, may be invited
Select the correct answer using the codes to take part, without a vote, in the Council’s
below. discussions; the Council sets the conditions
(a)  1 and 2 only for participation by a non-member State.
(b)  1 and 3 only Q.2143 The Presidency of the Security Council of
(c)  2 only United Nations is
(d)  None of the above (a) Fixed by a vote of the General Assembly
Solution: (d) for a period of 2 years
Justification: Power of veto” refers to (b) Decided by majority vote amongst the
the veto power wielded solely by the five permanent members for a given period
permanent members of the United Nations of the term
Security Council (China, France, Russia, (c) Rotated every month among all the
United Kingdom, and United States), members and non-members of the UNSC
enabling them to prevent the adoption of any (d)  None of the above

Indian Polity Question Bank P.601

06-Indian Polity_Q2005-ENDD.indd 601 8/7/2018 7:51:33 PM


Solution: (d) Reason (R): The office of Secretary-
Justification and Learning: The president is General was created few decades after the
the head of the delegation from the Security establishment of the UN.
Council member state that holds the rotating In the context of the above, which of these
presidency. is correct?
The rotation takes place in alphabetical (a) A is correct, and R is an appropriate
order of the member states’ official United explanation of A.
Nations names in English. (b) A is correct, but R is not an appropriate
Non-members are not entitled for the explanation of A.
Presidency, so the option (c) is incorrect. (c)  A is correct, but R is incorrect.
(d)  Both A and R are incorrect.
(C) Secretary-General Solution: (d)
Justification: Both statements are wrong.
Q.2144 How is the UN Secretary-General selected?
The Charter describes the Secretary-
(a) Elected on a majority basis by the UN
General as “chief administrative officer”
General Assembly (UNGA)
of the Organization, who shall act in that
(b) Appointed by the UN Security Council
capacity and perform “such other functions
(UNSC) based on the discretion of the
as are entrusted” to him or her by the Security
P-5
Council, General Assembly, Economic and
(c) Appointed by the General Assembly on
Social Council and other United Nations
the recommendations of the UNSC
organs.
(d) Nominated by a body of former UN
The Charter also empowers the Secretary-
Secretary Generals
General to “bring to the attention of the
Solution: (c)
Security Council any matter which in his
Learning: The Secretary-General of the
opinion may threaten the maintenance of
United Nations (UNSG) is the head of
international peace and security”.
the United Nations Secretariat, one of the
These guidelines both define the powers
principal organs of the United Nations The
of the office and grant it considerable scope
Secretary-General also acts as the de facto
for action.
spokesperson and leader of the United
Nations. Responsibilities of the Secretary-
General are further outlined in Articles 98
(D) International Court of Justice
through 100, which states that they shall Q.2146 Consider the following about the role of
act as the officer “in all meetings of the International Court of Justice (ICJ).
General Assembly, of the Security Council, 1. Only States are eligible to appear before
of the Economic and Social Council and the Court, no private entity can move
the Trusteeship Council, and shall perform the ICJ.
other functions as are entrusted to him 2. It provides individuals and groups with
by these organs”. They are responsible, legal counselling and helps them deal
according to Article 99, for making an with the authorities of any State in major
annual report to the General Assembly as human rights violation situations.
well as notifying the Security Council on 3. It is the final appellate court for decisions
matters which “in their opinion may threaten passed by major international tribunals.
the maintenance of international peace Select the correct answer using the codes
and security”. below.
Q.2145 Consider the following about the office of the (a)  1 only (b)  2 and 3 only
Secretary-General of United Nations. (c)  1 and 3 only (d)  3 only
Assertion (A): Even though he is the Solution: (a)
leader of the UN, the office does not find a Justification: Statements 1 and 2: The Court
mention in the United Nations (UN) Charter. has no jurisdiction to deal with applications

P.602 For Civil Services Preliminary Examination

06-Indian Polity_Q2005-ENDD.indd 602 8/7/2018 7:51:33 PM


from individuals, non-governmental 3. Veto in the UNSC is not applicable for
organizations, corporations or any other the appointment of these judges.
private entity. It cannot provide them with Choose the correct answer using the codes
legal counselling or help them in their below:
dealings with the authorities of any State (a)  1 only (b)  2 and 3 only
whatever. Only States are eligible to appear (c)  3 only (d)  None of the above
before the Court in contentious cases. At Solution: (d)
present, this basically means the 192 United Explanation above
Nations Member States. Q.2148 The International Court of Justice (ICJ) is the
Statement 3: The Court is not a supreme principal judicial organ of the United Nations
court to which national courts can turn; (UN). Consider the following about it.
it does not act as a court of last resort for 1. The ICJ judges are elected for terms of
individuals. Nor is it an appeal court for any office of nine years by the United Nations
international tribunal. It can, however, rule General Assembly and the Security
on the validity of arbitral awards. Council.
Learning: However, a State may take up 2. The Court can take suo motu cognizance
the case of one of its nationals and invoke of disputes between nation-states.
against another State the wrongs which its 3. The seat of the Court is in the Hague.
national claims to have suffered at the hands 4. ICJ gives advisory opinions on legal
of the latter; the dispute then becomes one questions at the request of the organs of
between States. the United Nations.
• The Court can only hear a dispute when Select the correct answer using the codes
requested to do so by one or more States. below.
It cannot deal with a dispute of its own (a)  1 and 3 only (b)  1, 3 and 4 only
motion. It is not permitted, under its (c)  2 and 4 only (d)  1, 2, 3 and 4
Statute, to investigate and rule on acts of Solution:(b)
sovereign States as it chooses. Explanation above
• The States concerned must also have
Q.2149 China’s Supreme Court has decided to
access to the Court and have accepted
set up its own International Maritime
its jurisdiction, in other words they must
Judicial Centre (IMJC) to handle territorial
consent to the Court’s considering the
disputes and protect its sea rights. However,
dispute in question.
International maritime disputes between
• This is a fundamental principle governing
countries are usually brought before the
the settlement of international disputes,
(a)  International Court of Justice (ICJ)
States being sovereign and free to choose
(b) International maritime organisation
the methods of resolving their disputes.
(IMO)
• Judgments delivered by the Court (or by
(c) United Nations Security Council (UNSC)
one of its Chambers) in disputes between
(d) International Chamber of Shipping
States are binding upon the parties
(ICM)
concerned.
Solution: (a)
• Judgments are final and without appeal. If
Learning: Presently China is locked in
either of the parties challenges their scope
disputes with its neighbours over claims in the
or meaning, it has the option to request an
resource-rich South China Sea. The tensions
interpretation.
have risen recently over China’s aggressive
Q.2147 Consider the following statements about the land reclamation continues to build artificial
appointment of judges to the International islands, airport runways and facilities on
Court of Justice, Hague? disputed reefs. This move will help China to
1. They are appointed for a term of five bolster its claims in the disputed South and
years. East China seas and also help it become a
2. They are appointed solely by the UNSC. maritime power.

Indian Polity Question Bank P.603

06-Indian Polity_Q2005-ENDD.indd 603 8/7/2018 7:51:33 PM


Q.2150 Consider the following about the International (E) Economic and Social Council
Court of Justice (ICJ).
1. It provides advisory opinions on legal Q.2151 Consider the following about the United
questions submitted to it by the UN Nations Economic and Social Commission
General Assembly. for Asia and the Pacific (ESCAP).
2. UN Security Council can enforce its 1. It is the first agency to have its
rulings. international headquarters in India.
3. No two judges may be nationals of the 2. It coordinates the initiatives of major
same country in the ICJ. international NGOs operating in Asia-
4. Its judges are appointed for 5-year terms Pacific.
by the UN Security Council. 3. India had acceded to ESCAP membership
Select the correct answer using the codes even before its independence.
below. Select the correct answer using the codes
(a)  1, 2 and 3 only (b)  3 and 4 only below.
(c)  1, 2 and 4 only (d)  1, 2, 3 and 4 (a)  1 only (b)  2 and 3 only
Solution: (a) (c)  3 only (d)  1, 2 and 3
Justification: Statement 1: ICJ is the primary Solution: (c)
judicial branch of the United Nations (UN) in Justification: Statements 1 and 2: It was
the Hague. It settles legal disputes submitted established in 1947 with its headquarters
to it by states and provides advisory opinions in Bangkok, Thailand, to assist in post-war
on legal questions submitted to it by duly economic reconstruction.
authorized international branches, agencies, The ESCAP works to overcome some of
and the UN General Assembly. the region’s greatest challenges by providing
Statements 2: Chapter XIV of the United results oriented projects, technical assistance
Nations Charter authorizes the UN Security and capacity building to member States in the
Council to enforce Court rulings. However, following areas:
such enforcement is subject to the veto Macroeconomic Policy, Poverty
power of the five permanent members of the Reduction and Financing for Development,
Council, which the United States used in the Trade and Investment, Transport etc.
Nicaragua case. In addition, ESCAP gives stronger
Statement 3 and 4: The ICJ is composed participation to the smaller and often left
of fifteen judges elected to Nine Years’ out voices of the region, the least developed
terms by the UN General Assembly and the countries, the small island States and
UN Security Council from a list of people landlocked States.
nominated by the national groups in the Statement 3: India became its member on
Permanent Court of Arbitration. According 28 March 1947, Pakistan on 30 September
to Article 9, the membership of the Court 1947. So, 3 is correct.
is supposed to represent the “main forms of
civilization and of the principal legal systems (F) UN Specialized Agencies
of the world”. Essentially, those have been Q.2152 International Maritime Organization (IMO)
meant the Common Law, Civil and Socialist is mandated to regulate
Law (now post-communist law). (a) Hydrocarbon extraction from disputed
• There is an informal understanding that territories
the seats will be distributed by geographic (b) Resolving jurisdiction issues related to
region maritime claims
• The five permanent members of the (c)  Shipping in international waters
United Nations Security Council (France, (d) Climate related parameters in ocean
Russia, China, the United Kingdom, and currents, tides and waves
the United States) always have a judge on Solution: (c)
the Court (except for 1967-1985 China).

P.604 For Civil Services Preliminary Examination

06-Indian Polity_Q2005-ENDD.indd 604 8/7/2018 7:51:33 PM


Learning: It is a specialised agency of the Statement 2: India hasn’t joined the
United Nations responsible for regulating accord, and this has disappointed IATA. But,
shipping. it is hope that India will join soon as it has
The IMO’s primary purpose is to develop a large fleet of flight operators and has led
and maintain a comprehensive regulatory the Paris and Kigali agreements from the
framework for shipping and its remit today forefront.
includes safety, environmental concerns, legal Q.2154 Consider the following about United Nations
matters, technical co-operation, maritime Human Rights Council (UNHRC).
security and the efficiency of shipping. IMO 1. It is an inter-governmental body.
is governed by an assembly of members and 2. It had replaced the former United Nations
is financially administered by a council of Commission on Human Rights.
members elected from the assembly. Which of the above is/are correct?
Q.2153 Consider the following about Carbon (a)  1 only (b)  2 only
Offsetting and Reduction Scheme for (c)  Both 1 and 2 (d)  None
International Aviation (CORSIA)  developed Solution: (c)
by the International Civil Aviation Justification: Statement 1: The Council
Organization (ICAO). was created by the United Nations General
1. ICAO decided to develop CORSIA Assembly in 2006. It is an inter-governmental
because aviation emissions were not body responsible for strengthening the
covered under Paris Agreement. promotion and protection of human rights
2. India led the negotiation committee that around the globe and for addressing
drafted CORSIA and recently ratified situations of human rights violations and
the scheme that will be operational make recommendations on them. It has the
from 2018. ability to discuss all thematic human rights
Which of the above is/are correct? issues and situations that require its attention
(a)  1 only (b)  2 only throughout the year. It meets at the UN Office
(c)  Both 1 and 2 (d)  None at Geneva.
Solution: (d) Statement 2: The Council is made up of
Justification: Aviation (domestic and 47 United Nations Member States which are
international) accounts for approximately elected by the UN General Assembly. The
2 per cent of global CO2 emissions produced Human Rights Council replaced the former
by human activity. This led ICAO to develop United Nations Commission onHuman
CORSIA. Rights.
The CORSIA calls for international Q.2155 Consider the following with reference to the
aviation to address and offset its emissions United Nations Economic and Social Council
through the reduction of emissions elsewhere (UNESCO).
(outside of the international aviation sector), 1. Members are appointed on rotation basis
involving the concept of “emissions units”. by the United Nations
This is similar to sale of carbon trading units. Secretary General.
Statement 1: Emissions from domestic 2. Seats on the Council are based on
aviation are considered under the UNFCCC. geographical representation.
Therefore, Parties to the UNFCCC have the 3. International Labour Organization (ILO)
possibility of addressing these emissions in is one of its sub-agencies.
their Nationally Determined Contributions Select the correct answer using the codes
(NDCs) under the Paris Agreement. below.
Statement 1 is wrong. (a)  2 only (b)  1 and 3 only
Agreement on the CORSIA contributes (c)  2 and 3 only (d)  1, 2 and 3
to the level of ambition set by the Paris Solution: (a)
Agreement. Justification: ILO is an autonomous
organization working with the United Nations

Indian Polity Question Bank P.605

06-Indian Polity_Q2005-ENDD.indd 605 8/7/2018 7:51:33 PM


coordinating with other specialized agencies assists in the development and coordination of
of the UN and UNECO. So, the Statement 3 worldwide technical standards.
is wrong. Q.2158 The WHO Framework Convention on
The Council has 54 member states out of Tobacco Control (WHO FCTC)  provides for
the 193 UN member states, which are elected 1. Banning of tobacco products in all
by the United Nations General Assembly countries party to the convention
for overlapping three-year terms. Seats 2. Eliminating illicit trade of tobacco products
on the Council are based on geographical 3. Alternative livelihoods to tobacco
representation with 18 allocated to African farmers
states, 13 to Asian states, 8 to East European Choose the correct answer using the codes
states, 13 to Latin American and Caribbean below.
states and 13 to West European and other states. (a)  1 and 2 only (b)  3 only
Q.2156 Consider the following statement: “Its (c)  2 only (d)  1 and 3 only
purpose is to contribute to peace and security Solution: (c)
by promoting international collaboration WNTD is one of eight official global public
through education, science, and culture in health campaigns marked by the WHO.
order to further universal respect for justice, The FCTC, one of the most quickly
the rule of law, and human rights along with ratified treaties in United Nations history,
fundamental freedom proclaimed in the is a supranational agreement that seeks “to
United Nations Charter.” protect present and future generations from
Which of the following United Nations the devastating health, social, environmental
body has the above statement as its main and economic consequences of tobacco
purpose? consumption and exposure to tobacco smoke”
(a) UNICEF (b) UNHCR by enacting a set of universal standards
(c) UNESCO (d) UNDP stating the dangers of tobacco and limiting its
Solution: (c) use in all forms worldwide.
Q.2157 Consider the following about International Even though it talks about regulating
Telecommunications Union (ITU). the use of tobacco, it does not provide for
1. It is a United Nations specialized agency a tobacco ban; neither about alternative
for information and communication livelihoods for farmers.
technologies. Q.2159 Which of the following components are
2. It is unique in having membership of only included in WHO’s definition of a ‘Healthy
private companies and agencies. City‘?
3. It coordinates the shared global use of the 1. Meets the ‘Basic Needs’ of ‘All’ its
radio spectrum. inhabitants.
Select the correct answer using the codes 2. Involves the ‘Community’ in local
below. government.
(a)  1 and 3 only (b)  2 only 3. Upgrading ‘Energy’ use and alternative
(c)  3 only (d)  1, 2 and 3 ‘Transport’ systems.
Solution: (a) Choose the correct answer from the codes
Justification: Statement 2: ITU currently has below:
a membership of 193 countries and almost 800 (a)  1 and 2 (b)  2 and 3
private-sector entities and academic institutions. (c)  None of the above (d)  All of the above
It is unique among UN agencies in having both Solution: (a)
public and private sector membership. World Health Organisation (WHO) suggests
Statement 3: The ITU coordinates the shared that, among other things, a ‘healthy city‘
global use of the radio spectrum, promotes must have:
international cooperation in assigning satellite • A ‘Clean’ and ‘Safe’ environment.
orbits, works to improve telecommunication • Meets the ‘Basic Needs’ of ‘All’ its
infrastructure in the developing world, and inhabitants.

P.606 For Civil Services Preliminary Examination

06-Indian Polity_Q2005-ENDD.indd 606 8/7/2018 7:51:33 PM


• Involves the ‘Community’ in local Which of the above is/are correct?
government. (a)  1 only (b)  2 only
• Provides easily accessible ‘Health’ service (c)  Both 1 and 2 (d)  None
Solution: (a)
(G) Funds & Programmes Justification: There are other partners, apart
from WHO and UNICEF. However, there
Q.2160 Consider the following about UNICEF.
is no legal commitment, so statement 2 is
1. It is one of the members of the United
wrong.
Nations Development Group.
Learning: India is among nine countries that
2. UNICEF relies on contributions only
will be part of the network. Other countries
from private donors.
are Bangladesh, A Cote d’Ivoire, Ethiopia,
3. UNICEF started working in India even
Ghana, Malawi, Nigeria, Tanzania and
before our First Five Year Plan (FYP)
Uganda.
became operational.
• Under this network, countries will work
Select the correct answer using the codes
to improve the quality of health care
below.
facilities to mothers and babies.
(a)  2 only (b)  1 and 3 only
• They will strengthen national efforts
(c)  2 and 3 only (d)  1, 2 and 3
to end preventable deaths of pregnant
Solution: (b)
women and newborns by 2030.
Justification: Statement 1: It is a United
• Under it, counties will build and strengthen
Nations (UN) program headquartered in
their national institutions, identify quality
New York City that provides humanitarian
of care focal points at all levels of the
and developmental assistance to children
health system.
and mothers in developing countries. It is
• They will also accelerate and sustain
one of the members of the United Nations
the implementation of quality-of-care
Development Group and its executive
improvement packages for mothers,
committee.
newborns and children.
Statement 2: UNICEF relies on
• They will also strengthen capacity and
contributions from governments and private
motivation of health professional to
donor. Governments contribute two-thirds
plan and manage quality improvement,
of the organization’s resources. However,
improve data collection and increase
UNICEF is funded entirely by voluntary (not
access to medicines, equipment, clean
compulsory) contributions.
water and supplies.
Statement 3: The organisation began
its work in India in 1949 with three staff Q.2162 Consider the following statements about
members and established an office in Delhi the United Nations High Commissioner for
three years later. Currently, it advocates for Refugees (UNHCR).
the rights of India’s children in 16 states. 1. It is a member of the United Nations
Development Group (UNDP).
Q.2161 India has joined ‘Network for Improving
2. It can protect and support refugees even
Quality of Care for Maternal, New-born
at the request of a national government.
and Child Health’, a global health network
Which of the above is/are correct?
focused on improving the quality of care
(a)  1 only (b)  2 only
for new mothers and babies. Consider the
(c)  Both 1 and 2 (d)  None
following about it.
Solution: (c)
1. The network is supported by WHO and
Justification and Learning: It is a United
United Nations International Children’s
Nations agency mandated to protect and
Fund (UNICEF).
support refugees.
2. The network membership legally
• It is a member of the United Nations
commits India to match global healthcare
Development Group. Initially it was
service norms to improve health security
established to help people displaced by
in India.

Indian Polity Question Bank P.607

06-Indian Polity_Q2005-ENDD.indd 607 8/7/2018 7:51:33 PM


World War II. But later it became the Justification: The New Urban Agenda was
principal agency that has helped displaced adopted at the UN Conference on Housing
persons all over world. and Sustainable Urban Development (also
• It is mandated to protect and support known as “Habitat III” conference) held
refugees at the request of a government in Quito, Ecuador. So, 1 is correct and 2 is
or the UN itself and assists in their wrong.
voluntary repatriation, local integration or It sets the global vision of sustainable
resettlement to a third country. urbanization for the next 20 years. It is a set
Q.2163 International Fund for Agricultural of 175 commitments that countries need to
Development (IFAD)  is a/an adhere to tackle challenges of urbanization.
(a) Specialized agency of the United Nations It is considered as roadmap for building
(UN) cities that can serve as engines of prosperity
(b) Fund established by the World Bank for and centres of social and cultural well-being
developing countries while protecting the environment. It provides
(c) Voluntary organization working in guidance for achieving the Sustainable
developing countries Development Goals and provides the
(d) Agreement under WTO Peace Clause underpinning for actions to address climate
Solution: (a) change.
Learning: The International Fund for So, 3 is wrong as there is no commitment
Agricultural Development (IFAD), a by developed countries to fund urbanization
specialized agency of the United Nations, in LDCs.
was established as an international financial Q.2165 The organization United Nations (UN)
institution in 1977 as one of the major Women works under the overall supervision
outcomes of the 1974 World Food Conference. and aegis of
The conference was organized in response (a)  UN General Assembly
to the food crises of the early 1970s that (b)  UN Economic and Social Council
primarily affected the Sahelian countries (c)  Commission on Status of Women
of Africa. It resolved that “an International (d)  All of above
Fund for Agricultural Development should Solution: (d)
be established immediately to finance Learning: In 2010, the United Nations
agricultural development projects primarily General Assembly created UN Women, the
for food production in the developing United Nations Entity for Gender Equality
countries.” IFAD is dedicated to eradicating and the Empowerment of Women. In doing
rural poverty in developing countries. so, UN Member States took an historic step
Q.2164 Consider the following about “Habitat III” in accelerating the Organization’s goals on
conference. gender equality and the empowerment of
1. It is also known as UN Conference women.
on Housing and Sustainable Urban According to UN General Assembly
Development. resolution, which established UN Women,
2. The 2016 conference was held at the organization is governed by a multi-tier
New Delhi. intergovernmental governance structure as
3. The “New Urban Agenda” adopted at the follows:
conference commits development financing “(a)… the General Assembly, the
by developed countries for urbanization of Economic and Social Council and the
least developed countries (LDCs). Commission on the Status of Women shall
Select the correct answer using the codes constitute the multi-tiered intergovernmental
below. governance structure for the normative
(a)  1 and 2 only (b)  2 and 3 only support functions and shall provide normative
(c)  1 only (d)  1, 2 and 3 policy guidance to the Entity;”
Solution: (c)

P.608 For Civil Services Preliminary Examination

06-Indian Polity_Q2005-ENDD.indd 608 8/7/2018 7:51:33 PM


Q.2166 Consider the following statements about vaccines to protect the people, especially
United Nations Development Programme children, against deadly infectious diseases.
(UNDP). Statement 2: It was established in 1997
1. The status of UNDP is that of an executive on the initiatives of the United Nations
board within the United Nations General Development Programme (UNDP).
Assembly. Its work is exclusively on vaccine
2. UNDP is funded entirely by voluntary development and introduction specifically for
contributions from member nations. people in developing countries, with a focus
3. UNDP supports national democratic on neglected diseases affecting these regions.
transitions by providing policy advice Statement 3: In 2007 with the approval of
and technical support. Union Cabinet, India joined IVI. Since then
Select the correct answer using the codes India is a long-term collaborator and stake-
below. holder of IVI (paying almost fifty thousand
(a)  1 and 2 only (b)  2 only dollar annually to IVI).
(c)  3 only (d)  1, 2 and 3 Q.2168 The stated objectives of the World Food
Solution: (d) Programme (WFP) is/are
Learning: Statement 1: It also provides 1. To scientifically develop and finance
expert advice, training and grants support genetically improved varieties of crops
to developing countries, with increasing across the world to meet food security
emphasis on assistance to the least developed challenges
countries. 2. To reduce under-nutrition and break the
Statement 2: The organization operates in intergenerational cycle of hunger so that
around 177 countries, where it works with the need for food aid itself is eliminated
local governments to meet development in the long run
challenges and develop local capacity. Which of the above is/are correct?
Additionally, the UNDP works internationally (a)  1 only (b)  2 only
to help countries achieve the SDGs. (c)  Both 1 and 2 (d)  None
Statement 3: It works here by improving Solution: (b)
institutional and individual capacity within Justification and Learning: It is one of the
countries, educating populations about world’s largest humanitarian organizations
and advocating for democratic reforms, addressing hunger and promoting food
promoting negotiation and dialogue, and security. The WFP works to help people who
sharing successful experiences from other cannot produce or obtain enough food for
countries and locations. themselves and their families.
Q.2167 Consider the following about International The WFP strives with the ultimate goal
Vaccine Institute (IVI). in mind of eliminating the need for food aid
1. It is a non-profit intergovernmental itself.
organization. The objectives that the WFP hopes to
2. It was founded by the Global Alliance for achieve are to:
Genomics and Health (GAGH). • Save lives and protect livelihoods in
3. India, as a collaborator, contributes an emergencies
annual financial grant to IVI. • Support food security and nutrition and
Select the correct answer using the codes (re)build livelihoods in fragile settings
below. and following emergencies
(a)  1 and 2 only (b)  1 and 3 only • Reduce risk and enable people,
(c)  3 only (d)  1, 2 and 3 only communities and countries to meet their
Solution: (b) own food and nutrition needs
Justification: Statement 1: IVI is an • Reduce under nutrition and break
international non-profit organization devoted to the intergenerational cycle of hunger
developing and introducing new and improved WFP food aid is also directed to fight

Indian Polity Question Bank P.609

06-Indian Polity_Q2005-ENDD.indd 609 8/7/2018 7:51:33 PM


micronutrient deficiencies, reduce child is a legally binding instrument under
mortality, improve maternal health, and international law.
combat disease, including HIV and AIDS. 2. The International Convention on the
Food-for-work programmes help promote Elimination of All Forms of Racial
environmental and economic stability and Discrimination outlaws hate speech.
agricultural production. Which of the above is/are correct?
Q.2169 Consider the following about the Global (a)  1 only (b)  2 only
Alliance for Vaccines and Immunization (c)  Both 1 and 2 (d)  None
(GAVI). Solution: (b)
1. The alliance is exclusive of major Justification: Statement 1: it is not a legally
international institutions like the UNICEF binding instrument under international
and WHO. law, however it does represent the dynamic
2. The alliance raises new resources development of international legal norms.
for immunization to channel them to It works towards eliminating human
developing country health systems. rights violations against the planet’s 370
Which of the above is/are correct? million indigenous people and assisting
(a)  1 only (b)  2 only them in combating discrimination and
(c)  Both 1 and 2 (d)  None marginalisation.
Solution: (b) India is a signatory to the convention.
Justification: Statement 1: The Global Statement 2: This Convention requires its
Alliance for Vaccines and Immunization parties to outlaw hate speech and criminalize
(GAVI) is an international coalition of membership in racist organizations.
partners. It includes national governments, The Convention also includes an
UNICEF, WHO, and the World Bank; individual complaints mechanism, effectively
philanthropic institutions, such as the Bill and making it enforceable against its parties.
Melinda Gates Children’s Vaccine Program India is a signatory, but does not recognize
and the Rockefeller Foundation; the private the competence of the Convention Committee
sector, represented by the International to hear complaints from individuals about
Federation of Pharmaceutical Manufacturers violations of the rights protected by the
Associations (IFPMA); and research and Convention.
public health institutions. Q.2171 International Covenant on Economic, Social
Statement 2: The Global Fund for and Cultural Rights does NOT recognize
Children’s Vaccines is a financially which human right?
independent mechanism designed by the (a)  Right to work opportunity
GAVI partners to raise new resources for (b) Right to social security and insurance
immunization and swiftly channel them to (c)  Right to adequate housing
developing country health systems. (d)  Right against indulgement in war by
The Global Fund makes its funding the State
decisions based on the recommendations Solution: (d)
of the GAVI Board. Since the partners Explanation & Learning: ICESCR is a
of the Alliance provide direction and multilateral treaty adopted by the UN General
support, administrative costs are kept low -- Assembly in 1966, coming into force since
approximately 98% of Global Fund resources 1976. It commits its parties to work toward
go directly to countries. the granting of economic, social, and cultural
rights (ESCR) to the Non-Self-Governing and
(H) UN Conventions Trust Territories and individuals, including
labour rights and the right to health, the right
Q.2170 Consider the following statements.
to education, and the right to an adequate
1. United Nations Declaration on the
standard of living.
Rights of Indigenous Peoples (UNDRIP)

P.610 For Civil Services Preliminary Examination

06-Indian Polity_Q2005-ENDD.indd 610 8/7/2018 7:51:33 PM


As of 2015, the Covenant has 164 parties. Learning: The Law of the Sea Convention
Six countries, including the United States, defines the rights and responsibilities of
have signed but not ratified the Covenant. The nations with respect to their use of the
Covenant is monitored by the UN Committee world’s oceans, establishing guidelines
on Economic, Social and Cultural Rights for businesses, the environment, and the
Universal Declaration of Human Rights. management of marine natural resources.
Q.2172 Consider the following about the International Internal waters: Covers all water and
Bill of Human Rights. waterways on the landward side of the
1. It is deemed as a part of international law. baseline. The coastal state is free to set laws,
2. It is also known as Universal Declaration regulate use, and use any resource. Foreign
of Human Rights. vessels have no right of passage within
Which of the above is/are correct? internal waters.
(a)  1 only (b)  2 only Territorial waters: Out to 12 nautical
(c)  Both 1 and 2 (d)  None miles from the baseline, the coastal state is
Solution: (a) free to set laws, regulate use, and use any
Justification: Statements 1 and 2: The resource.
Universal Declaration of Human Rights Exclusive economic zones (EEZs): These
consists of thirty articles which have been extend from the edge of the territorial sea
elaborated in subsequent international out to 200 nautical miles from the baseline.
treaties, economic transfers, regional human Within this area, the coastal nation has sole
rights instruments, national constitutions, exploitation rights over all natural resources.
and other laws. In casual use, the term may include the
• The International Bill of Human Rights territorial sea and even the continental shelf.
consists of the Universal Declaration of Continental shelf: The continental shelf
Human Rights, the International Covenant is defined as the natural prolongation of the
on Economic, Social and Cultural Rights, land territory to the continental margin’s
and the International Covenant on Civil outer edge from coastal baseline.
and Political Rights and its two Optional Coastal states have the right to harvest
Protocols. mineral and non-living material in the subsoil
• In 1966, the General Assembly adopted of its continental shelf, to the exclusion of
the two detailed Covenants, which others. Coastal states also have exclusive
complete the International Bill of Human control over living resources “attached” to
Rights. the continental shelf, but not to creatures
• In 1976, after the Covenants had been living in the water column beyond the
ratified by a sufficient number of exclusive economic zone.
individual nations, the Bill took on the Q.2174 Consider the following about the International
force of international law. Seabed Authority (ISA).
1. It is an autonomous international
(1) UNCLOS – United Nations organization established under the United
Convention on the Law of the Sea Nations Convention on the Law of the
Sea (UNCLOS).
Q.2173 As per the United Nations Convention on the
2. It has a bearing on the administration of
Law of the Sea (UNCLOS), a state has the
resources in the international sea waters.
right to resource extraction in
Which of the above is/are correct?
(a)  Territorial Sea only
(a)  1 only (b)  2 only
(b) Territorial Sea and Exclusive Economic
(c)  Both 1 and 2 (d)  None
Zone (EEZ) only
Solution: (c)
(c)  Internal waters only
Justification: Statement 1: It was established
(d) Continental shelf, Territorial Sea and
in 1982, and it has obtained observer status to
Exclusive Economic Zone (EEZ)
the United Nations in 1996.
Solution: (d)

Indian Polity Question Bank P.611

06-Indian Polity_Q2005-ENDD.indd 611 8/7/2018 7:51:34 PM


Statement 2: It regulates mining and 1. It is described as the International bill
related activities in the international seabed of rights for women focussing on the
beyond national jurisdiction, an area that economic and social rights of women.
includes most of the world’s oceans. 2. India has not signed the convention and
For e.g. the ISA recently cleared India’s thus not legally bound to enforce its
programmes of mining Polymetallic nodules provisions.
from the ocean. Currently, the Authority 3. Under the convention, states must report
has 167 members and the European Union, periodically about the progress on women
composed of all parties to the Law of the rights to the UN Economic and Social
Sea Convention. Council.
Q.2175 The International Tribunal for the Law of the Select the correct answer using the codes
Sea (ITLOS) can settle disputes relating to below.
1. Use of resources in International waters (a)  2 only (b)  1 and 3 only
2. Accident involving two national vessels (c)  1 only (d)  1, 2 and 3 only
in International waters Solution: (c)
Which of the above is/are correct? Justification: Statement 1: It was adopted
(a)  1 only (b)  2 only in 1979 by the United Nations General
(c)  Both 1 and 2 (d)  None Assembly. It focuses on non-discrimination,
Solution: (c) sex stereotypes, and sex trafficking.
The Italian government has approached Statement 2: India has both signed and
the International Tribunal for the Law of ratified this convention.
the Sea (ITLOS), one of the four forums Statement 3: Under article 18 of the
available for international disputes, in the convention, CEDAW states must report to the
Italian Marines case. committee of CEDAW on the progress they
The International Tribunal for the Law have made in implementing the convention
of the Sea (ITLOS) is an intergovernmental within their state. It is not UNECOSOC.
organization created by the mandate of the Q.2177 Consider the following statements.
Third United Nations Conference on the 1. Capital punishment or Death Sentence
Law of the Sea. It was established by the is impermissible under the Universal
United Nations Convention on the Law of Declaration of Human Rights (UDHR).
the Sea, signed at Montego Bay, Jamaica, on 2. Revocation of appointment on grounds
December 10, 1982. of pregnancy amounts to violation of
The Convention entered into force on women’s rights under Convention on the
November 16, 1994, and established an Elimination of all forms of Discrimination
international framework for law over “all against Woman (CEDAW).
ocean space, its uses and resources”. The Which of the above is/are correct?
tribunal is based in Hamburg, Germany. The (a)  1 only (b)  2 only
Convention also established the International (c)  Both 1 and 2 (d)  None
Seabed Authority, with responsibility for Solution: (c)
the regulation of seabed mining beyond the Justification: Statement 1: The United
limits of national jurisdiction that is beyond Nations adopted without dissent the Universal
the limits of the territorial sea, the contiguous Declaration of Human Rights (UDHR). The
zone and the continental shelf. Declaration proclaims the right of every
individual to protection from deprivation of
Convention on the Elimination life. It states that no one shall be subjected
of All Forms of Discrimination to cruel or degrading punishment. The death
against Women (CEDAW) penalty violates both of these fundamental
rights.
Q.2176 Consider the following about the Convention Statement 2: It provides that, “States
on the Elimination of all Forms of Parties shall ensure to women appropriate
Discrimination against Women (CEDAW).

P.612 For Civil Services Preliminary Examination

06-Indian Polity_Q2005-ENDD.indd 612 8/7/2018 7:51:34 PM


services in connection with pregnancy, It is the first and only internationally
confinement and the post-natal period, legally binding framework set up to address
granting free services where necessary, as the problem of desertification.
well as adequate nutrition during pregnancy The Convention’s 195 parties work
and lactation”. together to improve the living conditions
It also provides for special protection for for people in drylands, to maintain and
women from harmful types of work during restore land and soil productivity, and to
pregnancy and with the provision of paid mitigate the effects of drought. The UNCCD
maternity leave. is particularly committed to a bottom-up
approach, encouraging the participation of
UNCCD—United Nations Convention local people in combating desertification and
to Combat Desertification: 1994 land degradation. The UNCCD secretariat
facilitates cooperation between developed
Q.2178 Consider the following statements with
and developing countries, particularly
reference to the United Nations Convention
around knowledge and technology transfer
to Combat Desertification:
for sustainable land management.
1. It is the sole legally binding international
agreement linking environment and (I) Other Related Organisations &
development to sustainable land
management.
Issues
2. It is the first and only internationally Q.2179 Consider the following with reference
legally binding framework set up to to the United Nations International Law
address the problem of desertification. Commission (UNILC).
3. Only countries having deserts are 1. It was established by the United Nations
members to this convention. General Assembly (UNGA).
Which of the above statements is/are true? 2. The commission has been exempted from
(a)  Only 1 & 2 (b)  Only 2 & 3. the responsibility of developing laws on
(c)  Only 3. (d)  All. protection of the atmosphere.
Explanation: Desertification, along with 3. It is the only UN organization to not have
climate change and the loss of biodiversity, any participation from India or Indian
were identified as the greatest challenges to citizens.
sustainable development during the 1992 Rio Select the correct answer using the codes
Earth Summit. below.
Established in 1994, UNCCD is the sole (a)  1 only (b)  3 only
legally binding international agreement (c)  2 and 3 only (d)  1 and 2 only
linking environment and development Solution: (a)
to sustainable land management. The Justification: Statement 1: The International
Convention addresses specifically the arid, Law Commission was established by the
semi-arid and dry sub-humid areas, known General Assembly, in 1947, to “initiate
as the drylands, where some of the most studies and make recommendations for the
vulnerable ecosystems and peoples can be purpose of ... encouraging the progressive
found. In the 10-Year Strategy of the UNCCD development of international law and its
(2008-2018) that was adopted in 2007, codification.”
Parties to the Convention further specified Statement 2: It deals with a variety
their goals: “to forge a global partnership of subjects including Protection of the
to reverse and prevent desertification/land environment in relation to armed conflicts;
degradation and to mitigate the effects of Protection of the atmosphere and Crimes
drought in affected areas in order to support against humanity.
poverty reduction and environmental Statement 3: Aniruddha Rajput, a young
sustainability”. Indian lawyer, has been elected to the UN
International Law Commission.

Indian Polity Question Bank P.613

06-Indian Polity_Q2005-ENDD.indd 613 8/7/2018 7:51:34 PM


He is among 34 individuals elected by Which of the above is/are correct?
the UN General Assembly as members of (a)  1 and 2 Only (b)  2 and 3 Only
the International Law Commission that is (c)  1 and 3 Only (d)  All
tasked with the progressive development of Solution: (c)
international law and its codification. It was established by the resolution of United
The five-year term of the newly elected Nations General Assembly in December 1966
members commenced from 2017. “to promote the progressive harmonization
Q.2180 Consider the following with reference to the and unification of international trade law”.
International Criminal Court (ICC) UNCITRAL carries out its work at annual
1. United Nations Security Council and sessions held alternately in New York City
individual states have been authorized to and Vienna.
refer investigations to the Court. Q.2182 United Nations Disengagement Observer
2. ICC has the jurisdiction to prosecute Force (UNDOF)  maintains the ceasefire
individuals for crimes of genocide. between
Which of the above is/are correct? (a)  Syria and Israel
(a)  1 only (b)  2 only (b)  Jordan and Egypt
(c)  Both 1 and 2 (d)  None (c)  Kuwait and Iraq
Solution: (c) (d)  Saudi Arabia and Kuwait
Justification: ICC is an intergovernmental Solution: (a)
organization and international tribunal which Learning: The UNDOF was established
can prosecute individuals for the international in 1974 by United Nations Security Council
crimes of genocide, crimes against humanity, (UNSC) following the agreed disengagement
and war crimes. of Syrian and Israeli forces in the Golan
The ICC is intended to complement Heights. Since then, UNDOF has remained
existing national judicial systems and it in the area to maintain the ceasefire between
may therefore only exercise its jurisdiction Syrian and Israeli forces and to supervise
when certain conditions are met, such the implementation of the disengagement
as when national courts are unwilling or agreement.
unable to prosecute criminals or when Major General Jai Shanker Menon of India
the United Nations Security Council or has been appointed as the Head of Mission
individual states refer investigations to and Force Commander of the United Nations
the Court. Disengagement Observer Force (UNDOF).
Governed by an international treaty called Q.2183 The Nobel Peace Prize laureate who is also
the Rome Statute, the ICC is the world’s first the youngest United Nations (UN) Messenger
permanent international criminal court. of Peace is
Q.2181 With reference to the United Nations (a)  Antonio Guterres
Commission on International Trade Law (b)  Malala Yousafzai
(UNCITRAL), consider the following (c)  Ban Ki-Moon
statements (d)  Kailash Satyarthi
1. It is the core legal body of the United Solution: (b)
Nations system in the field of international Learning: United Nations Messengers of
trade law Peace are distinguished individuals, carefully
2. At present India is not its member yet selected from the fields of art, literature,
3. The Commission member States are science, entertainment, sports or other fields
elected by the General Assembly. of public life, who have agreed to help focus
Membership is structured so as to be worldwide attention on the work of the
representative of the world’s various United Nations.
geographic regions and its principal UN Secretary-General Antonio Guterres
economic and legal systems. has appointed Nobel laureate Malala
Yousafzai (at 19, youngest such messenger)

P.614 For Civil Services Preliminary Examination

06-Indian Polity_Q2005-ENDD.indd 614 8/7/2018 7:51:34 PM


as a UN Messenger of Peace to promote girls Which of the above statements is/are correct?
education. (a)  2, 3, and 4 Only
It is the highest honour given by the (b)  1, 2, 3 and 4
United Nations. The messengers are initially (c)  3 and 4 Only
chosen for a period of 3 years. (d)  2 and 3 Only
They mainly promote the work of the UN Solution: (a)
agency they are ambassador for. The amount of water recommended by
Q.2184 The United Nations Development Group the United Nations for drinking, washing,
(UNDG) was created by the Secretary- cooking and maintaining proper hygiene is a
General of the UN to improve the minimum of 50 litres per person per day.
effectiveness of UN development activities Q.2186 Consider the following about the United
at the country level. It consists of which of Nations Commission on Sustainable
these organizations? Development (CSD).
1. World Health Organization 1. It was established by the Conference of
2. United Nations Children’s Fund Parties (COP) to the UNFCCC.
3. International Labour Organization 2. Its primary duty is to enforce the binding
4. United Nations Women agreements signed under the UNFCCC.
Select the correct answer using the codes 3. It meets every ten years to chart the
below. course of ‘sustainable development’
(a)  2 and 4 only (b)  1 and 4 only in both developed and developing
(c)  2 and 3 only (d)  1, 2, 3 and 4 countries.
Solution: (d) Select the correct answer using the codes
Learning: Created in 1997, UNDG brings below.
together 32 UN agencies and groups, plus five (a)  1 only (b)  2 and 3 only
observers working on various development (c)  3 only (d)  None of the above
issues. Correct Answer: (d)
At the global level, the UNDG serves as Statement 1: It was established by the
a high-level forum for joint policy formation UN General Assembly in December 1992
and decision-making. to ensure effective follow-up of United
The UNDG meets three to four times a Nations Conference on Environment and
year under the chairmanship of the UNDG Development (UNCED), also known as the
Chair. Earth Summit.
Q.2185 Consider the following statements about At the United Nations Conference on
water conservation and United Nations. Sustainable Development (Rio 20), Member
1. The amount of water recommended by States agreed to establish a high level
the United Nations for drinking, washing, political forum that replaced the Commission
cooking and maintaining proper hygiene on Sustainable Development. The body no
is a minimum of 100 liters per person per longer exists.
day. Statement 2: The Commission was
2. Year 2003 was observed as the responsible for reviewing progress in
International Year of Freshwater to make the implementation of Agenda 21 and
people aware of this dwindling natural the Rio Declaration on Environment and
resource. Development; as well as providing policy
3. UN has declared 2013 as the UN guidance to follow up the Johannesburg
International Year of Water Cooperation Plan of Implementation (JPOI) at the local,
4. World Water Day is held annually national, regional and international levels.
on 22 March as a means of focusing Statement 3: The CSD met annually in
attention on the importance of freshwater New York, in two-year cycles, opening its
and advocating for the sustainable sessions to broad participation from both
management of freshwater resources. governmental and non-governmental actors.

Indian Polity Question Bank P.615

06-Indian Polity_Q2005-ENDD.indd 615 8/7/2018 7:51:34 PM


QSource: https://sustainabledevelopment. 2. Under the framework, UN Agencies
un.org/intergovernmental/csd provide support for national
Q.2187 Consider the following about the High Level development priorities as listed in
Political Forum on Sustainable Development. the 12th 5 Year Plan.
1. It was a part of the agreement following Which of the above is/are correct?
the Rio+20 Conference. (a)  1 only (b)  2 only
2. It is a United Nations central platform (c)  Both 1 and 2 (d)  None
for the follow-up and review of the 2030 Solution: (b)
Sustainable Development Goals (SDGs). Justification: Statement 1: It is a document
Which of the above is/are correct? prepared in partnership with the Planning
(a)  1 only (b)  2 only Commission and Line Ministries of
(c)  Both 1 and 2 (d)  None the Government of India, Civil Society
Solution: (c) Organisations and the United Nations in
Justification: HLPF is the most inclusive and India. But, it is signed with the government.
participatory forum at the United Nations, So, 1 is wrong.
bringing all States Members of the United Statement 2: It guides the collective
Nations and States members of specialized work of the United Nations in India. Under
agencies together. the UNDAF, along with the national
The HLPF was part of the agreement development priorities of the 12th Five Year
following the United Nations Conference on Plan the UN bodies also help to implement the
Sustainable Development or Rio+20 with the internationally agreed goals like the MDGs.
aim to strengthen Sustainable Development Under the current UNDAF, collective
governance at the United Nations it has the engagement by the UN will focus on nine
mandate to: priority states: Assam, Bihar, Chhattisgarh,
• provide political leadership and Jharkhand, Madhya Pradesh, Maharashtra,
recommendations for sustainable Odisha, Rajasthan and Uttar Pradesh. Other
development, focus areas are J&K and NE-India.
• follow-up and review progress in Q.2189 The former United Nations Secretary-
implementing sustainable development General’s UNiTE initiative aims to
commitments, (a)  End Violence against Women
The forum meets: (b)  Bring developing and developed
• every four years at the level of Heads of countries together
State and Government under the auspices (c) Achieve ceasefire in conflict-prone
of the General Assembly regions in the Middle-east
• every year under the auspices of the (d) Raise agricultural productivity in Least
Economic and Social Council—for eight Developed Countries (LDCs) to attain
days, including a three-day ministerial food security for the world
segment HLPF in 2016 was the first since Solution: (a)
the adoption of the 2030 Agenda and Learning: The global vision of the UNiTE
the SDGs. campaign is a world free from violence
Q.2188 Consider the following about the United against all women and girls.
Nations Development Action Framework Some of its goals are:
(UNDAF)  2013-17, India. • Adoption and enforcement of national
1. It is a programme document signed laws to address and punish all forms of
between all major voluntary bodies in violence against women and girls, in line
India and the United with international human rights standards.
Nations Country Team (UNCT) • Establishment of data collection and
mandating specific measurable and analysis systems on the prevalence of
binding targets. various forms of violence against women
and girls.

P.616 For Civil Services Preliminary Examination

06-Indian Polity_Q2005-ENDD.indd 616 8/7/2018 7:51:34 PM


• Systematic efforts to address sexual The yardstick for Democratic world
violence in conflict situations and to governance is whether each of the countries
protect women and girls from rape as a has free and equal say in the decisions
tactic of war and full implementation of that affect them. It depends more on the
related laws and policies. International institutional architecture in the
Q.2190 The Millennium Ecosystem Assessment UN, than in democracy being brought in the
(MEA) was called for by the nations of the world.
(a) Intergovernmental Panel on Climate UNSC does not need the approval of UNGA
Change (IPCC) even though the non-permanent members are
(b)  United Nations Secretary-General elected by UNGA. The permanent members
(c) Global Biodiversity and Ecosystems wield a lot of power in UNSC.
Organization (GBEO) Q.2192 Consider the following statements.
(d)  None of the above 1. The United Nations Democracy Fund
Solution: (b) (UNDEF)  finances the functioning of
Learning: Kofi Annan was the UN Sec-Gen legislatures in Third World transitioning
in 2000. Democratic countries.
Initiated in 2001, the objective of the MA 2. The United Nations Educational,
was to assess the consequences of ecosystem Scientific and Cultural Organization
change for human well-being and the (UNESCO) is the only UN agency with
scientific basis for action needed to enhance a mandate in higher education.
the conservation and sustainable use of those Which of the above is/are correct?
systems and their contribution to human (a)  1 only (b)  2 only
well-being. (c)  Both 1 and 2 (d)  None
The MA has involved the work of Solution: (b)
thousands of experts worldwide. Justification: Statement 1: It was built to
Their findings, contained in five technical support projects that strengthen the voice
volumes and six synthesis reports, provide of civil society, promote human rights, and
a state-of-the-art scientific appraisal of encourage the participation of all groups in
the condition and trends in the world’s democratic processes.
ecosystems and the services they provide The large majority of UNDEF funds go
(such as clean water, food, forest products, to local civil society organizations -- both in
flood control, and natural resources) and the the transition and the consolidation phases of
options to restore, conserve or enhance the democratization.
sustainable use of ecosystems. Statement 2: UNESCO is the only UN agency
Q.2191 Consider the following statements: with a mandate in higher education. It fosters
1. Democratic world governance is not innovation to meet education and workforce
necessary even if all the nations in the needs. It examines ways of increasing higher
world turn democratic. education opportunities for young people from
2. The UNSC can order an armed military vulnerable and disadvantaged groups.
intervention in a country only after the Q.2193 Consider the following statements.
approval of the United Nations General 1. The members of the General Assembly
Assembly (UNGA). are automatically the members of all
3. The non-permanent members of the other principal organs and specialised
UNSC are elected by the UNGA for a 2 agencies of the UN.
year term. 2. The UN Secretary-General can veto the
Choose the correct answer using the codes decision of the UN General assembly
below: when the decision goes against
(a)  1 and 2 (b)  2 and 3 international public interest.
(c)  1 and 3 (d)  All of the above 3. One veto can stall a Security Council
Solution: (c) resolution.

Indian Polity Question Bank P.617

06-Indian Polity_Q2005-ENDD.indd 617 8/7/2018 7:51:34 PM


Which of the above are correct? Statement 4: It was first established in
(a)  1 and 2 only (b)  2 and 3 only 1988 by two United Nations organizations,
(c)  1 and 3 only (d)  3 only the World Meteorological Organization
Solution: (d) (WMO) and the United Nations Environment
UN sec-Gen does not have any decision Programme (UNEP), and later endorsed by
making powers with respect to the decisions the United Nations General Assembly.
taken by the UNGA and UNSC. He is more
of an executive officer of the UN who (5) Trade Blocks
assists the UN in its functioning. WHO,
UNCTAD, UNECOSOC are organs of the (A) ASEAN
UN which have lesser members than the
UNGA. So membership of the UNGA does Q.2195 With reference to a grouping of countries
not automatically qualify countries for their called as ASEAN, Consider the following:
membership of the other agencies and organs. 1. India is not a founding member of
ASEAN.
Q.2194 Which of the following bodies/agencies of 2. It includes Japan and South Korea also.
the United Nations (UN) has/have won the 3. It sponsors an exclusive ASEAN bank
Nobel Peace Prize? dedicated to finance member nations.
1. United Nations Children’s Fund 4. The ASEAN region is economically a
(UNICEF) single market.
2. The Office of the United Nations High Select the correct answer using the codes
Commissioner for Refugees (UNHCR) below.
3. The Organization for the Prohibition of (a)  1 and 4 only (b)  1 and 3 only
Chemical Weapons (OPCW) (c)  2 and 3 only (d)  1 only
4. Intergovernmental Panel on Climate Solution:(d)
Change (IPCC) The ASEAN Free Trade Area (AFTA) which
Select the correct answer using the codes was established on 28 January 1992 includes
below. a Common Effective Preferential Tariff
(a)  1, 2 and 4 only (b)  2 and 3 only (CEPT) to promote the free flow of goods
(c)  1 and 3 only (d)  1 and 2 only between member states. When the AFTA
Solution: (a) agreement was originally signed, ASEAN
Justification: Solve such questions by had only six members (Brunei, Indonesia,
elimination. Malaysia, the Philippines, Singapore, and
OPCW is not a UN agency, so B and C are Thailand). Vietnam joined in 1995, Laos
incorrect. IPCC works under the auspices of and Burma in 1997, and Cambodia in 1999.
the UN and has won the Noble Peace Prize in The newcomers have not fully met AFTA’s
2007. So, the only answer can be A. obligations, but they are officially considered
Statement 1: Reasons can be seen here part of the AFTA as they were required to
http://www.un.org/en/sections/nobel-peace- sign the agreement upon entry into ASEAN,
prize/united-nations-children%E2%80%99s- and were given longer time frames in which
fund-unicef/index.html to meet AFTA’s tariff reduction obligations.
Statement 3: The OPCW was established The next steps are to create a:
to oversee the implementation of the • single market and production base
Chemical Weapons Convention – the full • competitive economic region
global ban on these kinds of weapons. The • region of equitable economic development
Nobel Committee stated: “The conventions • region fully integrated into the global
and the work of the OPCW have defined the economy
use of chemical weapons as a taboo under Since 2007, ASEAN countries have gradually
international law.” lowered their import duties with member
nations. The target is zero import duties by
2016.

P.618 For Civil Services Preliminary Examination

06-Indian Polity_Q2005-ENDD.indd 618 8/7/2018 7:51:34 PM


Q.2196 Free Trade Agreement (FTA) between India (d) Never taking recourse to the judicial
and ASEAN can potentially lead to: route to resolve commercial conflicts
1. Increased employment opportunities in between nations
both nations Solution: (c)
2. Violation of WTO global trading norms Learning: It is a working process or style
3. Higher Foreign Direct Investment that is informal and personal. Policymakers
(FDI) from ASEAN into India constantly utilize compromise, consensus,
Select the correct answer using the codes and consultation in the informal decision-
given below making process. Above all it prioritizes a
(a)  1 and 3 (b)  1 and 2 consensus-based, non-conflictual way of
(c)  2 and 3 (d)  Only 1 addressing problems. Quiet diplomacy allows
Solution: (d) ASEAN leaders to communicate without
Q.2197 East Asia Summit (EAS) includes which of bringing the discussions into the public view.
these nations as members? Members avoid embarrassment that may lead
1. India to further conflict.
2. United States Q.2199 Which of the following countries is/are NOT
3. Russia members of ASEAN?
4. Saudi Arabia 1. Cambodia
Select the correct answer using the codes 2. China
below. 3. Myanmar
(a)  2 and 3 only (b)  1 and 4 only 4. Bangladesh
(c)  1, 2 and 3 only (d)  1, 2, 3 and 4 5. Thailand
Solution: (c) Select the correct answer using the codes
Learning: List can be found here https:// below.
en.wikipedia.org/wiki/East_Asia_ (a)  2 only (b)  3, 4 and 5 only
Summit#Member_Nations (c)  1, 2 and 3 only (d)  2 and 4 only
• United States and Russia were admitted as Solution: (d)
member in 2011. Justification: It is a political and economic
• EAS shares a close relation with ASEAN organisation of ten Southeast Asian countries.
and associated organizations. It was formed in 1967 by Indonesia, Malaysia,
• ASEAN Plus Three (APT) is a forum the Philippines, Singapore, and Thailand.
that functions as a coordinator of co- Since then, membership has expanded to
operation between the Association of include Brunei, Cambodia, Laos, Myanmar
Southeast Asian Nations and the three (Burma), and Vietnam. Its aims include
East Asia nations of China, Japan, and accelerating economic growth, social
South Korea. progress, and socio-cultural evolution among
• ASEAN plus Six further includes India, its members, alongside protection of regional
Australia and New Zealand. stability as well as providing a mechanism
Q.2198 It has been said that the merits of the ASEAN for member countries to resolve differences
Way might “be usefully applied to global peacefully. ASEAN shares land borders with
conflict management”. What is called as the India, China, Bangladesh, East Timor, and
‘ASEAN Way’? Papua New Guinea, and maritime borders
(a) It reflects the spiritual life style of with India, China, and Australia.
ASEAN members countries.
(b) Involving third party countries in (B) Regional Trade Blocks – Africa
bilateral negotiations Q.2200 Consider the following statements about
(c) It is a form of interaction among ASEAN South African Customs Union
members that is informal, cooperative 1. It is the oldest customs union of the world
and respects cultural norms. 2. Lesotho, Swaziland, South Africa,
Botswana, Namibia are its members

Indian Polity Question Bank P.619

06-Indian Polity_Q2005-ENDD.indd 619 8/7/2018 7:51:34 PM


3. India and SACU have signed Preferential Learning: It is a sub-regional bloc. Its full
trade Agreement and it came into members are Argentina, Brazil, Paraguay,
operational in 2012 Uruguay and Venezuela.
Which of the above statements is/are correct? Its associate countries are Bolivia, Chile,
(a)  1 and 3 Only (b)  1 Only Peru, Colombia, Ecuador and Suriname.
(c)  3 Only (d)  1 and 2 Only Observer countries are New Zealand and
Solution: (d) Mexico.
India and SACU are yet to sign the treaty. Its purpose is to promote free trade and
The Southern African Customs Union the fluid movement of goods, people, and
(SACU) consists of Botswana, Lesotho, currency, as it is a full customs Union.
Namibia, South Africa, and Swaziland. The Q.2202 MERCOSUR as a sub-regional bloc excludes
SACU Secretariat is located in Windhoek, which of these countries?
Namibia. SACU was established in 1910, (a) Argentina (b) Mexico
making it the world’s oldest Customs Union. (c) Brazil (d) Peru
Historically SACU was administered by Solution: (b)
South Africa, through the 1910 and 1969 Justification: Eliminate Mexico, because
Agreements. The customs union collected MERCOSUR is a South-American trading
duties on local production and customs duties bloc; Mexico is a North American country.
on members’ imports from outside SACU, So, B is the answer.
and the resulting revenue was allocated to It was established in 1991. Its full members
member countries in quarterly installments are Argentina, Brazil, Paraguay and Uruguay.
utilizing a revenue-sharing formula. Its associate countries are Bolivia, Chile, Peru,
Negotiations to reform the 1969 Agreement Colombia, Ecuador and Suriname.
started in 1994, and a new agreement was Mercosur suspended Venezuela sometime
signed in 2002. The new arrangement was back from it for violating the bloc’s
ratified by SACU Heads of State. democratic principles and failing to meet its
The Economic structure of the Union basic standards.
links the Member states by a single tariff Learning: Brazil urges India to broaden
and no customs duties between them. The MERCOSUR presence and to expand its
Member States form a single customs commercial and multilateral footprints in
territory in which tariffs and other barriers South America.
are eliminated on substantially all the trade India and the Mercosur bloc have stepped
between the Member States for products up efforts to expand their preferential trade
originating in these countries; and there agreement (PTA) to make greater inroads
is a common external tariff that applies to into the other’s market.
nonmembers of SACU.
(D) AIIB – Asian Infrastructure
(C) Regional Trade Blocks – South Investment Bank
America & Carribean
Q.2203 How does the newly established Asian
Q.2201 Mercosur, also known as the “Common Infrastructure Investment Bank (AIIB) differ
Market of the South” is from the IMF and World Bank (WB)?
(a) An economic Union of Southern African 1. No nation holds veto power in the AIIB,
states unlike in IMF and WB.
(b) A free trade area among the Central 2. All member nations will have equal
Asian states decision-making power in AIIB, whereas
(c) An economic and political agreement in IMF and WB its rests on their economic
among some South American states size and global economic contributions.
(d) A private trade body that acts as a 3. Membership of the AIIB is open only to
representative of South Asia in WTO Asian nations, unlike in IMF and WB.
Solution: (c)

P.620 For Civil Services Preliminary Examination

06-Indian Polity_Q2005-ENDD.indd 620 8/7/2018 7:51:34 PM


Choose the correct answer using the codes • The AIIB was launched in Beijing last year
below. to spur investment in Asia in transportation,
(a)  1 and 2 only (b)  1 and 3 only energy, telecommunications and other
(c)  2 only (d)  None of the above infrastructure.
Solution: (d) • AIIB is regarded by some as a rival for
The AIIB will have authorised capital of USD the IMF, the World Bank and the Asian
100 billion, and Asian countries will contribute Development Bank (ADB), which the
to up to 75 per cent of the total capital. Each AIIB says are dominated by developed
member will be allocated a share of the quota countries like the United States and Japan.
based on their economic size (not contribution to • Presently there are 22 members including
authorized capital), according to the agreement. China.
China, India and Russia are the three largest • The authorised capital of AIIB will be
shareholders, taking a 30.34 per cent, 8.52 per $100 billion. AIIB’s headquarters is to be
cent, 6.66 per cent stake, respectively. Their located in Beijing.
voting shares are calculated at 26.06 per cent, • India is the second largest shareholder in
7.5 per cent and 5.92 per cent. The AIIB is the Bank after China.
designed to finance infrastructure construction • It is being called as a very positive
in the continent. In the IMF - Unlike the development in the sense that it opens up
General Assembly of the United Nations, more borrowing opportunities.
where each country has one vote, decision
making at the IMF was designed to reflect the (E) Regional Blocks in Which India Is
relative positions of its member countries in a Member
the global economy. The United States was
a leading force in the establishment of the (1) SAARC
World Bank in 1944 and remains the largest
Q.2205 South Asian Free Trade Area (SAFTA)
shareholder of the World Bank today. As the
includes
only World Bank shareholder that retains veto
1. Bhutan
power over changes in the Bank’s structure, the
2. Maldives
United States plays a unique role in influencing
3. Pakistan
and shaping development priorities.
4. Myamnar
Q.2204 Consider the following statements about 5. Afghanistan
AIIB. Select the correct answer using the codes
1. AIIB provides finances to infrastructure below.
projects in the Asia- Pacific region only. (a)  All except 4
2. India is the second largest shareholder in (b)  All except 5
the Bank after China. (c)  All except 1 and 2
3. AIIB and BRICS’ New Development (d)  All except 3 and 4
Bank have a joint funding facility for Solution: (a)
needy nations. Learning: SAFTA in 2004 created a free
Choose the correct answer using the codes trade area in Afghanistan, Bangladesh,
below. Bhutan, India, Maldives, Nepal, Pakistan and
(a)  1 and 2 only (b)  2 and 3 only Sri Lanka. The foreign ministers of the region
(c)  1 and 3 only (d)  All of the above signed a framework agreement on SAFTA to
Solution: (a) reduce customs duties of all traded goods to
The Asian Infrastructure Investment zero by the year 2016.
Bank (AIIB)  is an international financial The basic principles underlying SAFTA
institution proposed by China. The purpose are:
of the multilateral development bank is to • Overall reciprocity and mutuality of
provide finance to infrastructure projects in advantages
the Asia-Pacific region. • Negotiation of tariff reform

Indian Polity Question Bank P.621

06-Indian Polity_Q2005-ENDD.indd 621 8/7/2018 7:51:34 PM


• Recognition of the special needs of the help them to share the hydroelectric
Least Developed Contracting States etc. power generated in North East India to
Q.2206 Which of the following steps were taken by Bangladesh, Nepal and other countries.
India under the South Asian Free Trade Area • Excess electricity from India and Pakistan
(SAFTA) Agreement? can be supplied to electricity deficient
1. Reducing the items under the sensitive Afghanistan.
list • Offshore wind projects could be set up
2. It has brought down peak tariffs on in Sri Lanka’s coastal borders to power
several items. southern part of India and Sri Lanka.
3. It signed this agreement with Myanmar Q.2208 The SAARC Food Bank has been set up to
and Maldives too. supplement national efforts to provide food
Choose the correct answer using the security to the people of the SAARC region
following codes: during emergencies and food shortages. The
(a)  1 and 2 (b)  2 and 3 country with the largest share of food grains is
(c)  1 and 3 (d)  None of the above (a) Bangladesh (b) India
Solution: (a) (c) Pakistan (d) Sri Lanka
Refer to http://saarc-sec.org/ Solution: (b)
areaofcooperation/detail.php?activity_id=5 Learning: SAARC Food Bank has a reserve
Myanmar is a South-East Asian country. It of food grains maintained by each member
is not a part of SAFTA. country consisting of either wheat or rice, or
Q.2207 Recently the Union cabinet has agreed to a combination of both as assessed share of
sign SAARC Framework Agreement for the country. Presently India’s assessed share
Energy Cooperation. Which of the following of food grains is 3,06,400 lakh tonnes out
countries are NOT its included in it? of a total quantity of 4,86,000 lakh tonnes
1. Pakistan allocated for SAARC Food Bank.
2. Myanmar Q.2209 Which of these BRICS nations has an
3. Sri Lanka observer status at the South Asian Association
Choose the correct answer using the codes for Regional Cooperation (SAARC)?
below: (a) Brazil (b) Russia
(a)  1 and 2 (b)  2 and 3 (c) China (d) South Africa
(c)  1 and 3 (d)  Only 2 Solution: (c)
Solution: (d) Learning: States with observer status include
In 2000, the process of regional cooperation Australia, China, the European Union, Iran,
in energy sector began between SAARC Japan, Mauritius, Myanmar, South Korea and
Member States with the establishment of a the United States.
Technical Committee on Energy. Myanmar has expressed interest in
Key facts of SAARC Framework upgrading its status from an observer to
Agreement for Energy Cooperation a full member of SAARC. Russia has
(Electricity) applied for observer status membership of
• It will enable greater cooperation in the SAARC. Turkey applied for observer status
power sector among SAARC countries. membership of SAARC in 2012. South
• It is expected to improve the power Africa has participated in meetings.
availability in the entire SAARC region.
• It would facilitate integrated operation of (2) IORA
the regional power grid among member
Q.2210 Consider the following with reference to the
States.
Indian Ocean Rim Association (IORA).
Implications of this framework
1. It grants approvals for sourcing minerals
• This agreement will boost the
from Exclusive Economic Zones (EEZ) and
implementation of common power
continental shelves of the Indian Ocean.
grid between SAARC members. Thus

P.622 For Civil Services Preliminary Examination

06-Indian Polity_Q2005-ENDD.indd 622 8/7/2018 7:51:34 PM


2. It focuses on economic cooperation as 2. India has never hosted a Heart of Asia
well as combating marine piracy. conference.
3. Its coordinating Secretariat is located in Which of the above is/are correct?
India. (a)  1 only (b)  2 only
Select the correct answer using the codes (c)  Both 1 and 2 (d)  None
below. Solution: (d)
(a)  1 and 2 only (b)  2 only Justification: Statement 1: The Heart of Asia-
(c)  2 and 3 only (d)  1 and 3 only Istanbul Process was launched in 2011 and
Solution: (b) the participating countries include Pakistan,
Justification: Statement 1: The Indian Afghanistan, Azerbaijan, China, India, Iran,
Ocean Rim is rich in strategic and precious Kazakhstan, Kyrgyzstan, Russia, Saudi
minerals, metals and other natural resources, Arabia, Tajikistan, Turkey, Turkmenistan and
marine resources and energy, all of which can the United Arab Emirates.
be sourced from Exclusive Economic Zones It keeps Afghanistan at its centre focus.
(EEZ), continental shelves and the deep Statement 2: It provides a platform
seabed. for discussing key regional issues among
However, approvals are granted by participating States.
International Seabed Authority for minerals • The 6th Ministerial Conference (2016)
in international waters. States are free to of Heart of Asia summit was held in
extract resources within EEZ. So, 1 is Amritsar, India.
incorrect. • The Amritsar Declaration recognises
Statement 2: IOR-ARC, a regional terrorism as the biggest threat to peace
cooperation initiative of the Indian Ocean and security.
Rim countries, was established in Mauritius • The declaration states the urgency to
in 1997 with the aim of promoting economic respond to the nexus between drug
and technical cooperation. IOR-ARC is the menace and its financial support for
only pan-Indian ocean grouping. terrorist entities in Afghanistan.
Statement 3: Its coordinating Secretariat Q.2212 The Heart of Asia (HoA) conference
is located at Ebene, Mauritius. It is a regional essentially focuses on
forum, tripartite in nature that brings together (a) Economic integration of Asia with
representatives of Government, Business and Europe and Latin America
Academia, for promoting co-operation and (b) Regional cooperation with Afghanistan
closer interaction among them. to ensure its stability
Learning: Presently it has 19 members- (c) Developing the North-South Transport
Australia, Bangladesh, India, Indonesia, Iran, Corridor (NSTC)
Kenya, Malaysia, Madagascar, Mauritius, (d) Countering piracy in the Arabian and
Mozambique, Oman, Seychelles, Singapore, Indian Ocean
South Africa, Sri Lanka, Tanzania, Thailand, Solution: (b)
UAE and Yemen. Learning: HoA conference is a part of the
There are five Dialogue Partners namely Istanbul Process established 2011 which
China, Egypt, France, Japan and UK and two provides a platform to discuss an agenda of
Observers namely, Indian Ocean Research regional cooperation with Afghanistan at its
Group (IORG) and Indian Ocean Tourism centre.
Organisation (IOTO), Oman. • It seeks to provide a platform to discuss
regional issues, particularly encouraging
(3) Heart of Asia Conference security, political, and economic
Q.2211 Consider the following about Heart of Asia cooperation among Afghanistan and its
Conference. neighbours.
1. It is being conducted since the first non- • Its three main elements are (i) Political
alignment conference at Belgrade. Consultation involving Afghanistan

Indian Polity Question Bank P.623

06-Indian Polity_Q2005-ENDD.indd 623 8/7/2018 7:51:34 PM


and its near and extended neighbours. aims to facilitate closer contacts among
(ii) Achieving sustained incremental the people inhabiting these two major river
approach to implementation of Confidence basins.
Building Measures (CBMs) identified in India-Myanmar-Thailand Trilateral Highway
the Istanbul Process document. (iii) Seeks project and Hanoi Programme of Action
to contribute and bring greater coherence (HPA) are some of the key projects being
to work of various regional processes and monitored by MCG.
organisations, particularly as they relate to Q.2215 Mekong-Ganga Cooperation Project is
Afghanistan. (a) A security agreement between India and
• 14 participating countries: Russia, ASEAN
China, India, Pakistan, Afghanistan, Iran, (b) A bilateral investment agreement
Kazakhstan, Kyrgyz Republic, Tajikistan, involving India and Myanmar
Turkmenistan, Saudi Arabia, UAE, (c) A joint border demarcation exercise of
Azerbaijan and Turkey. East Asian countries and India
Q.2213 Apart from India and Pakistan, the member (d) Cooperation in tourism, culture,
states of the‘Heart of Asia’ conference education, and transportation linkages
include between India and some East Asian
1. China nations
2. Russia Solution: (d)
3. Saudi Arabia Learning: It was established in 2000 at
4. Iraq Vientiane at the First MGC Ministerial
Select the correct answer using the codes Meeting.
below. It comprises six member countries, namely
(a)  1 and 2 only (b)  2 and 4 only India, Thailand, Myanmar, Cambodia, Laos
(c)  1 and 3 only (d)  1, 2 and 3 only and Vietnam.
Solution: (d) They emphasised four areas of
cooperation, which are tourism, culture,
(4) Mekong-Ganga Cooperation (MCG) education, and transportation linkage in order
to be solid foundation for future trade and
Q.2214 Consider the following about Mekong-Ganga
investment cooperation in the region.
Cooperation (MCG).
In the latest meeting, ASEAN diplomats
1. It does not include any nation that is
demanded India to revamp their foreign
either a member of ASEAN Summit or
policy towards South-East Asia.
has a strategic partnership with China.
2. It only deals with power sharing and
inland waterways transport across the
(5) Multi- Sectoral Technical and
rivers Mekong and Ganga. Economic Cooperation (BIMSTEC)
Which of the above is/are correct? Q.2216 The Bay of Bengal Initiative for Multi-
(a)  1 only (b)  2 only Sectoral Technical and Economic
(c)  Both 1 and 2 (d)  None Cooperation (BIMSTEC) is an international
Solution: (d) organisation involving a group of countries
Justification: The Mekong-Ganga in South Asia and South East Asia. These are
Cooperation (MGC) is an initiative by six 1. Myanmar
countries – India and five ASEAN countries, 2. Sri Lanka
namely, Cambodia, Lao PDR, Myanmar, 3. Indonesia
Thailand and Vietnam for cooperation 4. Bhutan
in tourism, culture, education, as well as 5. Nepal
transport and communications. So, both 1 Select the correct answer using the codes
and 2 are wrong. below.
Both the Ganga and the Mekong are (a)  1, 2, 4 and 5 only
civilizational rivers, and the MGC initiative (b)  3 and 4 only

P.624 For Civil Services Preliminary Examination

06-Indian Polity_Q2005-ENDD.indd 624 8/7/2018 7:51:34 PM


(c)  1, 4 and 5 only It is not a FTA. IT is a sector-drive
(d)  1, 2 and 3 only organization focussing on sharing best
Solution: (a) practices and help each other in agriculture,
Justification: BIMSTEC headquarters is public health, poverty alleviation, counter-
situated in Dhaka, Bangladesh terrorism, environment, culture etc
A BIMSTEC Free Trade Area Framework
Agreement has been signed by member nations. (6) BBIN
Cooperation in BIMSTEC, starting with
Q.2218 Bangladesh, Bhutan, India, Nepal (BBIN)
six sectors—including trade, technology,
Initiative intended to facilitate
energy, transport, tourism and fisheries—
(a) Cross border movement of both
for sectoral cooperation in the late 1997, it
passengers and cargo vehicles
expanded to embrace nine more sectors—
(b) Better coordination between security
including agriculture, public health, poverty
forces of BBIN
alleviation, counter-terrorism, environment,
(c)  A Free trade zone in South Asia
culture, people to people contact and climate
(d) Registration of illegal migrants who have
change—in 2008.
overstayed in BBIN countries
Q.2217 Consider the following about the Bay of Solution: (a)
Bengal Initiative for Multi-Sectoral Technical Learning: BBIN initiative is a sub regional
and Economic Cooperation (BIMSTEC). architecture of countries in South Asia.
1. It was established through the Bangkok • India is in a bilateral motor vehicle
Declaration. agreement with Nepal and Bangladesh, but
2. Nepal and Bhutan are members of the a multilateral pact like BBIN would go a
BIMSTEC. long way in boosting trade in the region.
3. It is an implicit free trade agreement • Recently, Bhutan withdrew from BBIN,
among the member nations. and now India, Nepal and Bangladesh will
Select the correct answer using the codes have to decide whether to wait for Bhutan
below. to reconsider or to press ahead with a
(a)  1 and 2 only (b)  2 and 3 only truncated ‘BIN’ arrangement
(c)  3 only (d)  1, 2 and 3
Solution: (a) (7) BRICS
Justification: The Bay of Bengal Initiative
for Multi-Sectoral Technical and Economic Q.2219 In which of the following countries, not even
Cooperation (BIMSTEC) is a regional a single BRICS Summit has been held till
organization comprising seven Member date?
States lying in the littoral and adjacent 1. Brazil
areas of the Bay of Bengal constituting a 2. China
contiguous regional unity. 3. Russia
This sub-regional organization came 4. India
into being in 1997 through the Bangkok Select the correct answer using the codes
Declaration. It constitutes seven Member below.
States: five deriving from South Asia, (a)  1 and 4 only
including Bangladesh, Bhutan, India, Nepal, (b)  2 only
Sri Lanka, and two from Southeast Asia, (c)  2 and 3 only
including Myanmar and Thailand. (d) It has been held in all of the above at least
Initially, the economic bloc was formed once.
with four Member States with the acronym Solution (d)
‘BIST-EC’ (Bangladesh, India, Sri Lanka and Justification: 2011 Summit was held in
Thailand Economic Cooperation). China. It was the first summit to include
Later, Myanmar was included and it South Africa alongside the original BRIC
became BIMS-TEC. countries. So, 2 can’t be the answer.

Indian Polity Question Bank P.625

06-Indian Polity_Q2005-ENDD.indd 625 8/7/2018 7:51:34 PM


BRICS New Development Bank and Iran, Europe and Central Asia. It has been
BRICS Contingent Reserve Arrangement in construction for long. Option (d) is thus
agreements were signed in 2014 in Brazil wrong.
BRICS Summit. So, 1 can’t be the answer. Learning: The joint declaration seeks to
A summit was held in India in 2012, and is enhance co-investment of resources to support
happening in October 2016 (Goa). So, 4 can’t multilateral Research and Development
be the answer. (R&D) Projects in mutually agreed areas
2015 BRICS Summitt was held in Russia, by BRICS nations. BRICS nations mutually
Ufa. There was also a joint summit with have agreed on the collaboration for:
SCO-EEU. Q.2221 “Udaipur Declaration” adopted at the BRICS
Learning: The acronym BRIC was first meet concerns
used in 2001 by Goldman Sachs in their (a)  Coordinated Monetary Policies
Global Economics Paper, “The World Needs (b)  Disaster Risk Reduction
Better Economic BRICs” on the basis of (c)  Deforestation and land degradation
econometric analyses projecting that the (d)  Trade Facilitation Agreement
economies of Brazil, Russia, India and China Solution (b)
would individually and collectively occupy Learning: BRICS nations have made a
far greater economic space and would be clear move from relief-centric to a holistic
amongst the world’s largest economies in the approach to disasters with a greater emphasis
next 50 years or so. on prevention, mitigation and preparedness.
As a formal grouping, BRIC started after • BRICS nations will set up a dedicated Joint
the meeting of the Leaders of Russia, India Task Force for Disaster Risk Management
and China in St. Petersburg on the margins of for regular dialogue, exchange, mutual
G8 Outreach Summit in 2006. BRICS brings support and collaboration among them.
together five major emerging economies, • The meeting was a new milestone in
comprising 43% of the world population, collaboration and cooperation among
having 30% of the world GDP and 17% share BRICS countries in the field of disaster
in the world trade. management.
Q.2220 The Moscow Declaration recently signed in • ‘Flood Risk Management’ and
2015 by BRICS relates to ‘Forecasting of Extreme Weather Events
(a) Fighting piracy in Indian and Pacific in the context of Changing Climate‘were
Ocean the major themes discussed in the meet.
(b) Tackling challenges of terrorism in Q.2222 Fortaleza Declaration seen in news sometimes
Central and South Asia back led to the creation of
(c) Addressing common regional and global (a) Asian Infrastructure Investment Bank
socio-economic challenges by utilising (AIIB)
science, technology and innovation (b) BRICS New Development Bank (NDB)
(d) Improving the North-South Transport (c)  China Construction Bank
Corridor to improve access to energy (d) Asia-Pacific Investment Bank (APIB)
resources amidst BRICS nations Solution: (b)
Solution: (c) Learning: In 2014 at the sixth summit in
Explanation: Terrorism is generally the Fortaleza, Brazil the member countries
subject-matter of SCO negotiations and signed the Articles for the New Development
meetings. Option (b) is wrong. Bank with an Authorized Capital of USD 100
Fighting piracy is generally dealt by the billion.
IOR-ARC, ASEAN and other such littoral • The founders established the Bank
bodies. Option (a) is wrong. with a purpose of mobilizing resources
The International North–South Transport for infrastructure and sustainable
Corridor is the ship, rail, and road route development projects in BRICS and other
for moving freight between India, Russia,

P.626 For Civil Services Preliminary Examination

06-Indian Polity_Q2005-ENDD.indd 626 8/7/2018 7:51:34 PM


emerging economies and developing 2. Bank will be headquartered in Shanghai.
countries, complementing the existing 3. Bank’s first President will be Kundapur
efforts of multilateral and regional Vaman Kamath from India.
financial institutions for global growth 4. All member nations will have an equal
and development. say in Bank’s management.
• To fulfill its purpose, the Bank was envisaged 5. All member nations will contribute
to support public or private projects through equally to Bank’s start-up capital.
loans, guarantees, equity participation and Choose the correct answer using the codes
other financial instruments. below.
• It shall also cooperate with international (a)  1, 4 and 5 only
organizations and other financial entities, (b)  2 and 3 only
and provide technical assistance for (c)  1, 2, 3 and 4 only
projects to be supported by the Bank. (d)  All of the above
Q.2223 Consider the following statements about the Solution: (d)
New Development Bank (NDB). Agreement for establishing NDB was signed
1. It is the first Multi-lateral Development during the 6th BRICS Summit being held
Bank established by developing countries in Fortaleza, Brazil in April, 2014. It was
and emerging economies of BRICS. formally launched at the 7th BRICS summit
2. All member nations have an equal held in Ufa, Russia in July 2015.
shareholding in the NDB. Purpose: To fund infrastructure projects
3. It funds only projects related to sustainable in the emerging economies. It is seen as an
development and ecological conservation. alternative institute to west dominated World
Select the correct answer using the codes Bank and the International Monetary Fund
below. (IMF).
(a)  1 and 2 only (b)  3 only Capital: It will have initial capital of US
(c)  1 only (d)  1, 2 and 3 50 billion dollars and will be raised to US
Solution: (a) 100 billion dollars within the next couple of
Justification: Statement 1: NDB is the first years. Each member’s role: They will have
such Development Bank established by an equal say in the bank’s management,
BRICS – in accordance with the Fortaleza regardless of GDP size and contribute an
declaration signed in 2014. equal share in establishing a startup capital.
• The NDB members represent 42 percent President: Eminent banker Kundapur
of world population, 27 percent of the Vaman Kamath from India is President of
global surface area and accounting for Bank for the first five years i.e. till 2020.
over 20% of the Global GDP.
Statement 2: The five member nations – (8) IBSA
Brazil, Russia, India, China and South Africa Q.2225 The Brasilia declaration signed by the foreign
– have an equal shareholding in the NDB ministers of IBSA pressed for
Statement 3: The NDB was established (a) Greater security for women at workplaces
aiming to mobilize resources for infrastructure (b) Improved macro-economic policy
and sustainable development projects in coordination in South-South countries
BRICS and other emerging economies and (c) Urgent reforms in the United Nations,
developing countries, complementing the especially the Security Council
existing efforts of multilateral and regional (d) Climate change mitigation funding
financial institutions for global growth and support by developed countries
development. Solution: (c)
Q.2224 Consider the following about the BRICS Learning: It is one of the few important
New Development Bank. agreements signed by IBSA, hence the
1. Agreement for establishing NDB was question.
made at Fortaleza.

Indian Polity Question Bank P.627

06-Indian Polity_Q2005-ENDD.indd 627 8/7/2018 7:51:35 PM


In 2003, the Brasilia declaration was the land route, Sittwe provides these States
signed by the foreign ministers of India, access to Kolkata
Brazil and South Africa.
They agreed on the urgent need for (10) SCO
reforms in the United Nations, especially the
Q.2227 Full Shanghai Corporation Organization
Security Council.
(SCO) membership to India will have
The declaration was of the view that the
significant benefits in
United Nations Security Council should
1. Combating terrorism
reflect the current world scenario.
2. Opening up trade, energy sector and
It also touched upon the subjects of
strategic transit routes
international terrorism, trans-national crime
3. Resolving strictly bilateral land border
and illegal arms dealing, stating that such
disputes
threats to international peace must effectively
Choose the correct answer using the codes
tackled with respect for the sovereignty of
below.
States and for International Law
(a)  1 and 2 only
Learning: The IBSA Dialogue Forum
(b)  2 and 3 only
aims to promote South-South cooperation
(c)  1 and 3 only
and build consensus on issues of international
(d)  All of the above
importance. It also aims at increasing the
Solution (a)
trade opportunities among the three countries,
As Iran has observer status in the SCO, it will
as well as facilitate the trilateral exchange of
serve as a platform for India to boost trade
information, technologies and skills
through the Iranian ports of Bandar Abbas
and Chabahar. These ports are considered
(9) BCIM Corridor as India’s gateway to Central Asia through
Q.2226 The BCIM corridor is one of the major joint International North-South Transport Corridor
economic initiatives of India and China. The (INSTC). Thus, it will open up trade,
BCIM corridor will pass through energy sector and strategic transit routes
1. Kunming for India between Russia, Central Asia and
2. Mandalay China. SCO will provide platform for the
3. Xinjiang security grouping for India and Pakistan
4. Dhaka especially on issue to counter terrorism.
5. Kolkata It will also provide a valuable interface to
Choose the correct answer using the codes engage with security concern of Afghanistan
below. especially its members including India due
(a)  1, 2, 4 and 5 only to pullout international troops.SCO may
(b)  4 and 5 only also serve as guarantor for projects such as
(c)  1, 2, 3 and 5 only the Turkmenistan-Afghanistan-Pakistan-
(d)  1, 2, 3 and 4 only India (TAPI) and Iran-Pakistan-India (IPI)
Solution (a) pipelines, which are held by India due to
Xinjiang province is near the Northern border security concerns. Balance in International
of India with China. BCIM corridor passes Politics: In this politically polarised world,
from the eastern side.One of the strategic SCO will play an important role in counter-
factors driving the corridor is reduction of balancing India’s perceived tilt on security
reliance on the Straits of Malacca, militarily issues towards US and its allies. Thus, it
dominated by the U.S. Mizoram would be can help to maintain full balance of India’s
connected with Myanmar’s port of Sittwe, relations with the great powers globally.
through the Kaladan River, and the passage Q.2228 Consider the following about the Regional
will provide all the landlocked north-eastern Anti-terrorist Structure (RATS) of the
States access to the sea. Compared with Shanghai Corporation Organization (SCO).

P.628 For Civil Services Preliminary Examination

06-Indian Polity_Q2005-ENDD.indd 628 8/7/2018 7:51:35 PM


1. It is headquartered in Beijing, China. Europe and Central Asia. The route primarily
2. It is a permanent organ of the Shanghai involves moving freight from India, Iran,
Cooperation Organization. Azerbaijan and Russia via ship, rail and road.
3. It deals with extremism and separatism The objective of the corridor is to increase
too. trade connectivity between major cities such
4. All member-states contribute to as Mumbai, Moscow, Tehran, Baku, Bandar
intelligence gathering in RATS. Abbas, Astrakhan, Bandar Anzali and etc.
Choose the correct answer using the codes Dry runs of two routes were conducted
below. in 2014, the first was Mumbai to Baku via
(a)  1 and 4 only (b)  2 and 3 only Bandar Abbas and the second was Mumbai
(c)  2, 3 and 4 only (d)  All of the above to Astrakhan via Bandar Abbas, Tehran and
Solution: (c) Bandar Anzal.
RATS SCO, headquartered in Tashkent Q.2230 The southernmost and northernmost hubs
(Uzbekistan), is a permanent organ of the on the International North South Transport
Shanghai Cooperation Organization which Corridor (NSTC) respectively are
serves to promote cooperation of Member (a)  Kabul and Vladivostok
States against terrorism, separatism, and (b)  Mumbai and Moscow
extremism, which it refers to as “the three (c)  Colombo and Chabahar
evil forces.” RATS SCO includes 8 Member (d)  Jakarta and Azerbaijan
States (the Republic of Kazakhstan, the Solution: (b)
People’s Republic of China, the Kyrgyz Justification: Work by elimination. One
Republic, the Russian Federation, the node of NSTC begins from India, so only
Republic of Tajikistan, and the Republic option B can be correct.
of Uzbekistan, India and Pakistan), three • The major Northern end junctions are:
observers, and three dialogue partners. To Astrakhan, Moscow, Baku, Bandar Abbas,
ensure the safety of border areas, the border Tehran and Bandar Anzali.
defence cooperation mechanism under the • Mumbai is the only Northernmost hub or
framework of RATS SCO was established. junction on the NSTC.
RATS SCO has also collected and distributed • NSTC is the ship, rail, and road route for
to its Member States intelligence information moving freight between India, Russia,
regarding the use of the Internet by terrorist Iran, Europe and Central Asia.
groups active in the region to promote their • The primary objective of the NSTC
ideas. project is to reduce costs in terms of time
Q.2229 International North–SouthTransport Corridor and money over the traditional route
The International North–SouthTransport currently being used
Corridor will directly connect India with • It is 3,500 miles long and will be
which of these countries? integrated with the Ashgabat agreement
1. Russia on multimodal transport.
2. Europe
3. Central Asia (10) Ashgabat Agreement
4. Iran
Q.2231 The Union Cabinet has given its approval for
5. Azerbaijan
India to accede to the Ashgabat Agreement. It
Choose the correct answer using the codes
establishes international transport and transit
below.
corridor between
(a)  2, 3 and 5 only (b)  1, 3 and 4 only
(a) Central Asia and the Persian Gulf
(c)  1 and 4 only (d)  All of the above
countries
Solution: (d)
(b) South-East Asia to European Countries
The International North–South Transport
(c) West-African countries to South Asian
Corridor is the ship, rail, and road route for
countries
moving freight between India, Russia, Iran,

Indian Polity Question Bank P.629

06-Indian Polity_Q2005-ENDD.indd 629 8/7/2018 7:51:35 PM


(d) Oil producing Gulf countries and littoral up the time and reduce the costs of moving
states of Indian Ocean goods, vehicles, and people across borders.
Solution: (a) The six SASEC countries presently remain
Learning: It is a transit agreement established some of the least economically integrated
in year 2011. Its founding members are in the world. SASEC also assists member
Uzbekistan, Iran, Turkmenistan and Oman. countries in improving energy security by
Kazakhstan had joined the grouping in 2015. developing infrastructure and promoting
Accession to the Agreement would enable intraregional power trade to reduce costs and
India to: import dependence.
• Utilise this existing transport and
transit corridor to facilitate trade nand (12) Commonwealth
commercial interaction and ties with the
Q.2233 Consider the following about the
Eurasian region.
Commonwealth Association for Public
• Synchronise India’s efforts to implement
Administration and Management (CAPAM).
the International North South Transport
1. CAPAM is a research partner of the
Corridor (INSTC) for enhanced
United Nations General Assembly
connectivity.
(UNGA) and UN Economic and Social
• It will provide India an opportunity
Council (UNECOSOC).
for reorientation of the freight traffic
2. The Government of India is a member of
from the traditional sea route to land
CAPAM, which enables it to keep pace
transcontinental routes.
with the latest developments in the field
of public administration.
(12) SASEC Which of the above is/are correct?
Q.2232 The South Asia Sub-Regional Economic (a)  1 only (b)  2 only
Cooperation (SASEC) (c)  Both 1 and 2 (d)  None
Program aims to promote Solution: (b)
1. Cross-border power sharing Justification: Statement 1: With its
2. Cross-border rail and road links headquarters at Ottawa, Canada, is an
3. Develop joint military outfits to secure organization dedicated to strengthening
common market checkposts public management and consolidating
Select the correct answer using the codes democracy and good governance throughout
below. the Commonwealth.
(a)  1 only (b)  1 and 3 only It was formed in 1994 as a result of
(c)  1 and 2 only (d)  3 only decisions taken at the Commonwealth Heads
Solution: (c) of Government meeting.
Learning: The South Asia Sub-regional Statement 2: The Ministry of Personnel,
Economic Cooperation (SASEC) Program Public Grievances and Pensions, Government
brings together Bangladesh, Bhutan, of India became an institutional member of
India, the Maldives, Nepal, and Sri Lanka CAPAM in 1997.
in a project-based partnership that aims The membership allows it participation
to promote regional prosperity, improve in various programmes of CAPAM
economic opportunities, and build a better viz; International Innovations Awards
quality of life for the people of the sub- Programme, International Innovations
region. Cascading Programme etc.
SASEC seeks to strengthen multimodal
cross-border transport networks that boost (13) International Solar Alliance (ISA)
intraregional trade and open up trade
Q.2234 International Solar Alliance (ISA) is an
opportunities with East and South-East
alliance of solar resource-rich tropical
Asia. The program helps build modern and
countries lying fully or partially between two
effective customs administrations that speed

P.630 For Civil Services Preliminary Examination

06-Indian Polity_Q2005-ENDD.indd 630 8/7/2018 7:51:35 PM


tropics. Which of these is/are the objectives 1. It is a political Union.
of the ISA? 2. It is a sub-agency of NATO.
1. Promote standardisation in the use of 3. It has a common constitution.
equipments and processes for generating 4. There is a common flag and anthem.
solar electricity Select the correct answer using the codes
2. Boost global demands which will result below.
in further reduction in prices of solar (a)  1 and 4 only (b)  2 and 3 only
energy deployment (c)  1, 3 and 4 only (d)  4 only
3. Adopt mandatory Domestic Content Solution: (d)
Requirements for solar industries to Justification: Statement 1: It is an economic
generate employment and domestic Union, not a political Union, as all member
growth states are autonomous units.
Select the correct answer using the codes Statement 2: Some EU members are also
below. NATO members, but EU is not its sub-agency.
(a)  1 and 2 only (b)  2 only Statement 3: The Treaty establishing a
(c)  1 and 3 only (d)  1, 2 and 3 Constitution for Europe (TCE), (commonly
Solution: (a) referred to as the European Constitution or as
Justification: Statement 1: Standardisation the Constitutional Treaty), was an unratified
will make the manufacturing of equipments international treaty intended to create a
and other hardware cheaper. consolidated constitution for the European
Statement 2: It also envisages boosting Union (EU). It would have replaced the
R&D, particularly in areas of efficient storage existing European Union treaties with a
systems. single text. But it was rejected by the French
Statement 3: DCRs do not go well with and Dutch voters.
WTO norms, and can be challenged (as Statement 4: It has its own flag, anthem,
India’s Solar Mission DCR was challenged founding date, and currency. It also has some
by USA). It is not one of the objectives of form of a common foreign and security
the ISA. policy in its dealings with other nations.
Learning: More than 120 countries are Q.2236 Consider the following statements about the
geographically located in the tropics i.e. European Union.
between the Tropic of Cancer and the Tropic of 1. It has its own constitution and a
Capricorn, either fully or partially. These places Parliament.
get ample sunlight throughout the year, making 2. It is a free trade area.
solar energy easily available resources. 3. Its share of world trade is larger than that
These countries also happen to be ones of the USA.
where maximum growth in energy demand in 4. It has a common currency.
the coming years, but their current production Choose the correct answer using the codes
is woefully short of requirement. below:
Thus, the ISA will ensure that as these (a)  1 and 2 only (b)  2, 3 and 4 only
countries rapidly ramp up their electricity (c)  2 and 4 only (d)  3 and 4 only
production, predominantly by using clean Solution: (b)
solar energy and avoid fossil fuels. The European Union has evolved over time
from an economic union to an increasingly
(F) Other Interational Organisations political one. The EU has started to act more
– Alliances & Agreements as a nation state. While the attempts to have
a Constitution for the EU have failed, it has
(1) European Union its own flag, anthem, founding date, and
currency. It also has some form of a common
Q.2235 Consider the following statements about the
foreign and security policy in its dealings
European Union.
with other nations.

Indian Polity Question Bank P.631

06-Indian Polity_Q2005-ENDD.indd 631 8/7/2018 7:51:35 PM


The European Union has tried to expand Select the correct answer using the codes
areas of cooperation while acquiring new below.
members, especially from the erstwhile (a)  1 and 2 only (b)  3 and 4 only
Soviet bloc. The process has not proved easy, (c)  2, 3 and 4 only (d)  1 and 3 only
for people in many countries are not very Solution: (c)
enthusiastic in giving the EU powers that Learning: Each member state is party to the
were exercised by the government of their founding treaties of the union and thereby
country. There are also reservations about subject to the privileges and obligations of
including some new countries within the EU. membership.
Its share of world trade is three times Unlike members of most international
larger than that of the United States allowing organisations, the member states of the EU
it to be more assertive in trade disputes with are subjected to binding laws in exchange for
the US and China. Its economic power gives representation within the common legislative
it influence over its closest neighbours as and judicial institutions.
well as in Asia and Africa. It also functions The EU countries are: Austria, Belgium,
as an important bloc in international. Bulgaria, Croatia, Republic of Cyprus,
Q.2237 “Lisbon Treaty” was recently in news with Czech Republic, Denmark, Estonia, Finland,
reference to France, Germany, Greece, Hungary, Ireland,
(a) Brexit referendum for leaving European Italy, Latvia, Lithuania, Luxembourg, Malta,
Union (EU) Netherlands, Poland, Portugal, Romania,
(b) Model Foreign Account Tax Compliance Slovakia, Slovenia, Spain, Sweden and the UK.
Act (FATCA) to be adopted by G-20 Q.2239 Which of the following resulted in the
nations creation of the European Union?
(c) Modification of Iran’s nuclear programme (a)  Treaty of Portsmouth
under pressure from P5+1 nations (b)  Paris Declaration
(d)  Retrospective taxation on foreign (c)  Rome Declaration
investors in WTO member nations (d)  Maastricht treaty
Solution: (a) Solution: (d)
Learning: Under the Lisbon treaty, a Learning: Upon its entry into force in 1993,
member state wishing to leave the EU should it created the European Union and led to the
first notify the European Council its decision. creation of the single European currency,
This would set in motion a process by the euro. One of the obligations of the treaty
which the member and the EU leadership for the members was to keep “sound fiscal
will negotiate the terms of the departure and policies, with debt limited to 60% of GDP
reach an agreement in 2 years. and annual deficits no greater than 3% of
People of United Kingdom (UK) in a GDP”.
historic Brexit referendum voted in favour of • The treaty also created what was
leaving European Union (EU). commonly referred to as the pillar
The vote to leave the EU now has triggered structure of the European Union.
a two-year ‘withdrawal process’ which will • The treaty established the three pillars of
determine the future of UK’s relationship the European Union— one supranational
with the EU. pillar created from three European
Q.2238 Which of the following countries is/are Communities (which included the
member of the European Community (EC), the European
European Union? Coal and Steel Community and the
1. Switzerland European Atomic Energy Community),
2. Austria the Common Foreign and Security Policy
3. Czech Republic (CFSP) pillar, and the Justice and Home
4. Sweden Affairs (JHA) pillar.

P.632 For Civil Services Preliminary Examination

06-Indian Polity_Q2005-ENDD.indd 632 8/7/2018 7:51:35 PM


• The Maastricht Treaty has been amended and a board of the heads of national central
by the treaties of Amsterdam, Nice and banks. The principal task of the ECB is to
Lisbon. keep inflation under control. Though there
Q.2240 Consider the following about Euro zone and is no common representation, governance
European Union or fiscal policy for the currency union,
1. Euro zone is an Economic entity, and EU some co-operation does take place through
is a Politico-economic entity. the Eurogroup, which makes political
2. Euro zone has a common currency, EU decisions regarding the eurozone and the
doesn’t. euro. The Eurogroup is composed of the
3. Euro zone follows a common monetary finance ministers of eurozone states, but in
policy, EU doesn’t. emergencies, national leaders also form the
4. Both the EU and Euro zone have a single Eurogroup.
market.
5. EU has a supra-national Parliament, Euro (2) OECD
zone doesn’t. Q.2241 Decision-making in OECD is vested in the
Choose the correct answer using the codes OECD council which consists of
below. 1. One representative from each member
(a)  1, 2 and 5 only country
(b)  3 and 4 only 2. Elected representatives from five member
(c)  1, 2, 3 and 4 only countries by rotation
(d)  All of the above 3. A representative of European Commission
Solution: (a) 4. Secretary-general of UN
The question has been made again with Choose the correct answer from the codes
reference to the Greece crisis and its possible below.
exit from European Union. It is an important (a)  1 and 2 only (b)  2 and 3 only
topic for this year. The European Union (c)  1 and 3 only (d)  2 and 4 only
(EU) is a politico-economic union of 28 Solution: (c)
member states that are located primarily Decision-making power is vested in the
in Europe. The EU operates through a OECD Council. It is made up of one
system of supranational institutions and representative per member country, plus a
intergovernmental-negotiated decisions by representative of the European Commission.
the member states. The institutions are: the The Council meets regularly at the level
European Commission, the Council of the of permanent representatives to OECD
European Union, the European Council, the and decisions are taken by consensus.
Court of Justice of the European Union, the These meetings are chaired by the OECD
European Central Bank, the European Court Secretary-General. The Council also meets at
of Auditors, and the European Parliament. ministerial level once a year to discuss key
The European Parliament is elected issues and set priorities for OECD work. The
every five years by EU citizens. The EU work mandated by the Council is carried out
has developed a single market through a by the OECD Secretariat.
standardised system of laws that apply in all
Q.2242 Consider the following with reference to the
member states.
Organisation for Economic Co-operation and
The eurozone, officially called the euro
Development (OECD).
area, is a monetary union of 19 European
1. Its main mission is to enhance the
Union (EU) member states that have adopted
economic interests of developed countries
the euro (€) as their common currency and
exclusively.
sole legal tender.
2. It is a non-profit organization working
Monetary policy of the zone is the
an attached agency under the European
responsibility of the European Central Bank
Union.
(ECB) which is governed by a president

Indian Polity Question Bank P.633

06-Indian Polity_Q2005-ENDD.indd 633 8/7/2018 7:51:35 PM


3. India is an accession candidate to OECD. through faster customs procedures at borders;
Select the correct answer using the codes more favorable business climates behind
below. the border; and aligning regulations and
(a)  1 and 2 only standards across the region.
(b)  3 only For example, APEC’s initiative to
(c)  1, 2 and 3 only synchronize regulatory systems is a key step
(d)  None of the above to integrating the Asia-Pacific economy. A
Solution: (d) product can be more easily exported with
Justification: Statement 1: The mission of just one set of common standards across all
the Organisation for Economic Co-operation economies.
and Development (OECD) is to promote Statement 4 and 5: APEC operates as a
policies that will improve the economic and cooperative, multilateral economic and trade
social well-being of people around the world. forum. Member economies participate on the
The OECD provides a forum in which basis of open dialogue and respect for views
governments can work together to share of all participants.
experiences and seek solutions to common In APEC, all economies have an equal say
problems. and decision-making is reached by consensus.
Statement 2: It is separate from EU. There are no binding commitments or treaty
It is an inter-governmental organization obligations. Commitments are undertaken
headquartered in Paris. on a voluntary basis and capacity building
Statement 3: India is an enhanced projects help members implement APEC
engagement partner of OCED. It is neither a initiatives.
member non-accession candidate to the OECD. APEC’s structure is based on both a
“bottom-up” and “top-down” approach.
(3) Asia-Pacific Economic Cooperation Q.2244 Which of these nations is NOT a member
(APEC) of the Asia-Pacific Economic Cooperation
(APEC)?
Q.2243 Consider the following about Asia-Pacific
(a) China
Economic Cooperation (APEC).
(b) India
1. It consists of Pacific Rim economies.
(c)  United States of America
2. It promotes free trade throughout the Asia-
(d) Russia
Pacific region by promoting common
Solution: (b)
trade standards across the region.
Learning: It is a regional economic forum
3. It is an affiliate member of the G-8.
established in 1989 to leverage the growing
4. All economies have an equal say in
interdependence of the Asia-Pacific.
APEC and decision-making is reached by
APEC’s 21 member economies are
consensus.
Australia; Brunei Darussalam; Canada;
5. There are no binding commitments or
Chile; People’s Republic of China; Hong
treaty obligations other than those agreed
Kong, China; Indonesia; Japan; Republic
voluntarily by member economies on a
of Korea; Malaysia; Mexico; New Zealand;
case to case basis.
Papua New Guinea; Peru; The Philippines;
Select the correct answer using the codes
The Russian Federation; Singapore; Chinese
below.
Taipei; Thailand; United States of America;
(a)  1 and 2 only (b)  1, 2, 4 and 5 only
Vietnam.
(c)  3, 4 and 5 only (d)  1, 2 and 5 only
In APEC, all economies have an equal
Solution: (b)
say and decision-making is reached by
Statement 2: APEC ensures that goods,
consensus.
services, investment and people move easily
There are no binding commitments or
across borders. Members facilitate this trade
treaty obligations.

P.634 For Civil Services Preliminary Examination

06-Indian Polity_Q2005-ENDD.indd 634 8/7/2018 7:51:35 PM


(4) OPEC Q.2246 Consider the following about Organization of
the Petroleum Exporting Countries (OPEC).
Q.2245 Organization of Petroleum Exporting 1. It is an informal consortium of private oil
Countries (OPEC) is producers in the Gulf region.
(a) a joint Arab private firm that manages oil 2. Iran and Iraq have not been included in
exports from Middle-East OPEC.
(b) an oil cartel that jointly changes prices 3. Its headquarters is not located in West
and supply of oil Asia where some of the largest oil
(c) a conglomeration of Arab-American producers are situated.
companies with the exclusive right to 4. OPEC decides and administers the global
extract and export petroleum from Arab price of Brent Crude Oil.
nations Select the correct answer using the codes
(d) a political entity setup by all Arab oil below.
companies to supervise extraction and (a)  1 and 3 only (b)  3 only
production of oil and natural gas from (c)  2 and 4 only (d)  1, 2, 3 and 4
middle-east Solution: (b)
Solution: (b) Explained in previous answer
Explanation: OPEC is a cartel of countries.
Q.2247 The organization that works as a cartel among
So, it is incorrect to refer to it as a joint
the following is
private firm. Hence, option (a) is incorrect.
(a) International labour Organization (ILO)
Other options can be eliminated as it does
(b) North American Free Trade Area
not confirm to the definition of a cartel.
(NAFTA)
Learning: A cartel is a collection of
(c)  World Trade Organization (WTO)
businesses or countries that act together as a
(d) Organization of Petroleum Exporting
single producer and agree to influence prices
Countries (OPEC)
for certain goods and services by controlling
Solution: (d)
production and marketing. A cartel has less
Justification: A cartel is an association of
power than a monopoly. This is because
manufacturers or suppliers with the purpose
cartels do not control the entire market, as
of maintaining prices at a high level and
monopolists usually do.
restricting competition.
OPEC was formed when the international
NAFTA is a free trade bloc, so b is
oil market was largely dominated by a
incorrect.
group of multinational companies known
ILO and WTO are inter-governmental
as the ‘seven sisters’. The formation OPEC
organizations, so b and c are incorrect.
represented a collective act of sovereignty by
OPEC is a notorious cartel that has often
oil exporting nations, and marked a turning
spiked oil prices by cutting supplies, so d is
point in state control over natural resources.
correct.
OPEC’s mandate is to “coordinate and
Domestic cartel formation is usually
unify the petroleum policies” of its members
restricted by anti-competition laws, but
and to “ensure the stabilization of oil markets
international cartels cannot be regulated as
in order to secure an efficient, economic and
such.
regular supply of petroleum to consumers, a
steady income to producers, and a fair return
on capital for those investing in the petroleum
(5) Financial Action Task Force (FATF)
industry. AND G7
In 2014 OPEC comprised twelve members: Q.2248 Which of the following statements about
Algeria, Angola, Ecuador, Iran, Iraq, Kuwait, the Financial Action Task Force (FATF),
Libya, Nigeria, Qatar, Saudi Arabia, the established in 1989 by G-7, is INCORRECT?
United Arab Emirates, and Venezuela. (a) It is an inter governmental policy making
body.

Indian Polity Question Bank P.635

06-Indian Polity_Q2005-ENDD.indd 635 8/7/2018 7:51:35 PM


(b) Its mandate covers setting international Select the correct answer using the codes
standards for combating money below.
laundering and terrorist financing. (a)  1, 2 and 3 only
(c) It does not deal with proliferation of (b)  2, 3 and 4 only
weapons of mass destruction which are (c)  1 and 5 only
dealt by the UNSC and multi-lateral (d)  1 and 2 only
export control regimes. Solution: (a)
(d) It works to identify national-level Learning: It is a group of seven major
vulnerabilities with the aim of protecting advanced economies as reported by the
the international financial system from International Monetary Fund (IMF).
misuse. Collectively they represent more than
Solution: (c) 64% of the net global wealth; very high
Justification: Option B: The objectives of Human Development Index and 46% of the
the FATF are to set standards and promote global GDP.
effective implementation of legal, regulatory G7 countries include Canada, France,
and operational measures for combating Germany, Italy, Japan, UK and US. Russia
money laundering, terrorist financing and was expelled from the group G8 following its
other related threats to the integrity of the annexation of Crimea.
international financial system. Hiroshima Declaration (2016) reaffirms
• The FATF is therefore a “policy-making commitment of G-7 countries’ to seek a safer
body” which works to generate the world for all and create the conditions for a
necessary political will to bring about world without nuclear weapons to promote
national legislative and regulatory reforms international stability.
in these areas.
Option C: The FATF has developed a series (6) FSB & G20
of Recommendations that are recognised
Q.2250 The Financial Stability Board (FSB) that
as the international standard for combating
promotes international financial stability
of money laundering and the financing of
through enhanced information exchange and
terrorism and proliferation of weapons of
international cooperation in financial market
mass destruction
supervision was established by
Option D: The FATF monitors the progress of its
(a) The G-8 finance ministers and central
members in implementing necessary measures,
bank governors
reviews money laundering and terrorist
(b)  G-20 major economies
financing techniques and counter-measures,
(c) United Nations Economic and Social
and promotes the adoption and implementation
Council (UNECOSOC)
of appropriate measures globally.
(d) A resolution of all members of the UN
• In collaboration with other international
General Assembly
stakeholders, the FATF works to identify
Solution: (b)
national-level vulnerabilities with the aim
Learning: The Financial Stability Board
of protecting the international financial
(FSB) is an international body that monitors
system from misuse.
and makes recommendations about the global
• The FATF’s decision making body, the
financial system.
FATF Plenary, meets three times per year.
It was established after the 2009
Q.2249 Which of the following countries is/are NOT G-20 London summit in April 2009 as a
members of G-7? successor to the Financial Stability Forum
1. Russia (FSF). The Board includes all G-20 major
2. China economies, FSF members, and the European
3. Australia Commission. It is based in Basel, Switzerland
4. Japan
5. Italy

P.636 For Civil Services Preliminary Examination

06-Indian Polity_Q2005-ENDD.indd 636 8/7/2018 7:51:35 PM


Q.2251 The Financial Stability Board (FSB) is an • It seeks to address issues that go beyond the
international body that monitors and makes responsibilities of any one organization.
recommendations about the global financial • The G20 heads of government or heads
system. Consider the following about it. of state have periodically conferred at
1. It was established by G-20 as the successor summits since their initial meeting in
to the Financial Stability Forum (FSF). 2008, and the group also hosts separate
2. It has been made a member organ of the meetings of finance ministers and central
International Monetary and Financial bank governors.
Committee (IMFC).
Which of the above is/are correct? (7) G77
(a)  1 only (b)  2 only
Q.2253 Which of the following about G-77, a
(c)  Both 1 and 2 (d)  None
coalition of developing nations, is correct?
Solution: (a)
(a) India and China are the cofounders of
Justification: Statement 2: FSB will report
G-77.
any possible threats to the stability of the global
(b) It founded the Non-Alignment movement
financial system to the G20 finance ministers,
(NAM), 1961.
the IMF and central bank governors.
(c) Being an informal grouping, it is
The FSF was made an observer of the
officially not recognized by bodies of
IMFC in 1999. FSB is not a member organ
United Nations.
of the IMFC.
(d)  None of the above
Q.2252 Which of the following countries are Solution: (d)
member(s) of G-20? Justification: China was not a co-founder of
1. Australia G77. So, (a) is incorrect.
2. Saudi Arabia NAM was founded by a group of five
3. Canada nations India, Egypt, Indonesia, Ghana and
4. China Yogoslavia. So, (b) is incorrect.
5. Japan There are Chapters of the Group of 77
Select the correct answer using the codes in Rome (FAO), Vienna (UNIDO), Paris
below. (UNESCO), Nairobi (UNEP) and the Group
(a)  1, 3 and 5 only of 24 in Washington, D.C. (International
(b)  1, 2 and 4 only Monetary Fund and World Bank). So, (c) is
(c)  2, 3 and 5 only incorrect
(d)  1, 2, 3, 4 and 5 Learning: It was designed to promote
Solution: (d) its members’ collective economic interests
Justification: Use elimination. China is an (such as climate change) and create an
influential member of G-20. So, options A enhanced joint negotiating capacity in the
and C can be discarded right away. United Nations. There were 77 founding
There is no reason why Japan, being a members of the organization, but by 2013
major developed economy, should not be a the organization had since expanded to 134
member of G-20. This clearly leads to the member countries.
answer option d.
Learning: (8) MCAA
• G20 is an international forum for the
Consider the following with reference to
governments and central bank governors
the Multilateral Automatic Exchange of
from 20 major economies.
Financial Account Information (MCAA).
• It was founded in 1999 with the aim of
1. It is a multilateral convention on
studying, reviewing, and promoting high-
administrative assistance in taxation
level discussion of policy issues pertaining
matters.
to the promotion of international financial
stability.

Indian Polity Question Bank P.637

06-Indian Polity_Q2005-ENDD.indd 637 8/7/2018 7:51:35 PM


2. It was developed by the Organisation To be noted, even though India has not
for Economic Co-operation and signed NPT, due to signing Indo-US civil
Development (OECD). nuclear deal, we are supposed to allow for
3. India has not joined the convention. IAEA inspections.
Select the correct answer using the codes Q.2255 The International Atomic Energy Agency
below. (IAEA) reports to both the UN General
(a)  1 and 2 only (b)  2 and 3 only Assembly and the UN Security Council
(c)  1 only (d)  1, 2 and 3 because
Solution: (a) 1. IAEA was established by the UN Charter.
Justification: It sets up a system wherein bulk 2. It is under the direct control of the UN.
taxpayer information will be periodically be Which of the above is/are correct?
sent by source country of income to the (a)  1 only (b)  2 only
country of residence of the taxpayer. So, 1 is (c)  Both 1 and 2 (d)  None
correct. Solution: (d)
• This agreement obliges signatories Justification and Learning: Though
to exchange a wide range of financial established independently of the United
information among themselves Nations through its own international treaty,
periodically and automatically. the IAEA Statute, the IAEA reports to both
• Its main aim is to prevent international tax the United Nations General Assembly and
evasion and avoidance and help member Security Council. However, it is not under
countries to curb tax evasion and deal the direct control of the UN. So, both 1 and
with the problem of black money. 2 are wrong.
The IAEA has its headquarters in Vienna.
(G) IAEA – International Atomic The IAEA serves as an intergovernmental
Energy Agency forum for scientific and technical cooperation
Q.2254 The International Atomic Energy Agency in the peaceful use of nuclear technology and
(IAEA) was established in 1957. It has been nuclear power worldwide
in news frequently. Consider the following
statements about it. (H) Arctic Council
1. It is associated with the UN. Q.2256 Consider the following statements with
2. It seeks to prevent countries from using reference to the Arctic Council:
any form of nuclear energy. 1. Only states with territory in the Arctic can
3. Nations which have signed the Non- be members of the Council.
Proliferation Treaty (NPT) are required 2. Chairmanship of the Council rotates
to allow regular inspections by the IAEA. every two years amongst its members
Choose the correct answer using the codes 3. Observer status is open to non-Arctic
below: states approved by the Council at the
(a)  1 and 2 only (b)  2 and 3 only Ministerial Meetings.
(c)  1 and 3 only (d)  All of the above 4. Only permanent observers have voting
Solution: (c) rights in the Council.
The International Atomic Energy Agency Which of the above statements is/are true?
(IAEA) was established in 1957. It came (a)  Only 1. (b)  Only2, 3 & 4.
into being to implement US President Dwight (c)  Only 1 & 4. (d)  Only 1, 2 & 3.
Eisenhower’s ―Atoms for Peace proposal. It Answer: (d)
seeks to promote the peaceful use of nuclear Explanation: The Arctic Council is a high-
energy and to prevent its use for military level intergovernmental forum that addresses
purposes. IAEA teams regularly inspect issues faced by the Arctic governments and
nuclear facilities all over the world to ensure the indigenous people of the Arctic. It has
that civilian reactors are not being used for eight member countries: Canada, Denmark,
military purposes.

P.638 For Civil Services Preliminary Examination

06-Indian Polity_Q2005-ENDD.indd 638 8/7/2018 7:51:35 PM


Finland, Iceland, Norway, Russia, Sweden, Twelve non-arctic countries have been
and the United States. Only states with approved as Observers to the Arctic Council:
territory in the Arctic can be members of the France; Germany; The Netherlands;
Council. All eight countries are members Poland; Spain; United Kingdom; People’s
making the Arctic Council a circumpolar Republic of China; Italian Republic ; Japan;
forum. The Council also has permanent and Republic of Korea; Republic of Singapore;
ad hoc observer countries and “permanent Republic of India.
participants”. Chairmanship of the Council Member states are Russia, Canada,
rotates every two years. Finland, Norway, Sweden, USA, Denmark
Observer status is open to non-Arctic states and Iceland.
approved by the Council at the Ministerial
Meetings that occur once every two years. (I) Antarctica Treaty System
Permanent observers have no voting rights
Q.2258 Consider the following about the Antarctica
in the Council. As of May 2013, twelve
Treaty System.
non-Arctic states have Permanent Observer
1. It designates Antarctica as a scientific
status. Observer states receive invitations for
preserve.
most Council meetings. Their participation in
2. It bans military activity and related
projects and task forces within the Working
conflicts on that continent.
Groups is not always possible, but this poses
Which of the above is/are correct?
few problems as few Observer States want
(a)  1 only (b)  2 only
to participate at such a detailed level.Ad hoc
(c)  Both 1 and 2 (d)  None
observer states need to request permission
Solution: (c)
for their presence at each individual meeting;
Justification: Some important provisions of
such requests are routine and most of them
the Treaty:
are granted. There are six ad hoc members,
• Antarctica shall be used for peaceful
not including the European Union. Approved
purposes only (hence 2 is correct)
intergovernmental and interparliamentary
• Freedom of scientific investigation in
organizations (both global and regional) and
Antarctica and cooperation toward that
non-governmental organizations can also
end … shall continue.
obtain Observer Status
• Scientific observations and results from
Q.2257 The nations that have an Observer status in Antarctica shall be exchanged and made
the Arctic freely available.
Council is/are Among the signatories of the Treaty
1. India were seven countries - Argentina, Australia,
2. China Chile, France, New Zealand, Norway and
3. Russia the United Kingdom - with territorial claims,
4. Japan sometimes overlapping. Other countries do
Select the correct answer using the codes not recognize any claims.
below.
Q.2259 Sovereign rights over Antarctic territory is
(a)  1, 2 and 4 only (b)  2 and 3 only
exercised by
(c)  1 and 4 only (d)  3 and 4 only
1. USA
Solution: (a)
2. Chile
Justification: Observer status in the Arctic
3. Australia
Council is open to:
4. UK
• non-Arctic states
Select the correct answer using the codes
• inter-governmental and inter-
below.
parliamentary organizations, global and
(a)  1 and 3 only (b)  2 and 3 only
regional
(c)  1, 2, 3 and 4 (d)  None of the above
• non-governmental organizations.
Solution: (d)

Indian Polity Question Bank P.639

06-Indian Polity_Q2005-ENDD.indd 639 8/7/2018 7:51:35 PM


Justification: Antarctica is a global common Q.2261 Which of the following countries was
and so no one nation exercises sovereignty admitted as the latest member of North
over the territory. It is governed under 1959 Atlantic Treaty Organization (NATO)?
Antarctic Treaty, an international treaty, that (a) Montenegro (b) Canada
bars countries from owning or exploiting its (c)  Switzerland (d)  None of the above
land. Since its signing by 45 nations, claims Solution: (d)
of seven countries for territory in the region Learning: US Senate recently voted with
were suspended. near unanimity to approve Montenegro as
29th member of NATO. So far, 25 other
(J) NATO NATO members have ratified Montenegro’s
accession, a small country which is being seen
Q.2260 Consider the following about the North
as a geostrategic ally. However, Spain and
Atlantic Treaty Organization (NATO).
Netherlands are yet to ratify its membership.
1. NATO membership is open to any
So, d is the answer.
country that is willing to accept the terms
Learning: The NATO is an
of the military alliance.
intergovernmental military alliance based on
2. All decisions are taken by consensus in
the North Atlantic Treaty signed in 1949. It
NATO.
consists of 28 independent member countries
3. The Membership Action Plan (MAP) of
across North America and Europe.
NATO is a military code that prohibits
• It is based on a system of collective
attack on member countries by any of the
defence whereby its member states agree
NATO members.
to mutual defence in response to an attack
Select the correct answer using the codes
by any external party.
below.
• NATO’s headquarters are located in
(a)  1 only (b)  2 and 3 only
Brussels, Belgium. Its headquarters of
(c)  2 only (d)  None of the above
Allied Command Operations is near
Solution: (c)
Mons. The combined military spending of
Justification: Statement 1: NATO
all NATO members constitutes over 70%
membership is open to “any other European
of the global defence spending.
state in a position to further the principles of
this Treaty and to contribute to the security of
the North Atlantic area.”
(K) WIPO
Statement 2: A “NATO decision” is the Q.2262 Consider the following about the Patent
expression of the collective will of all 28 Cooperation Treaty (PCT) of the World
member countries since all decisions are Intellectual Property Organization (WIPO).
taken by consensus. 1. PCT makes it possible to seek patent
NATO is committed to the peaceful protection for an invention simultaneously
resolution of disputes. If diplomatic efforts in a large number of countries by filing a
fail, it has the military capacity needed to single “international” patent application.
undertake crisis-management operations. 2. PCT shifts the authority of granting
These are carried out under the Washington patents from national offices to the
Treaty - NATO’s founding treaty - or under WIPO.
a UN mandate, alone or in cooperation Which of the above is/are correct?
with other countries and international (a)  1 only (b)  2 only
organizations. (c)  Both 1 and 2 (d)  None
Statement 3: The Membership Action Solution: (a)
Plan (MAP) is a NATO programme of advice, Justification: Statement 1: The Patent
assistance and practical support tailored to Cooperation Treaty (PCT) assists applicants
the individual needs of countries wishing to in seeking patent protection internationally
join the Alliance. for their inventions, helps patent Offices with
their patent granting decisions, and facilitates

P.640 For Civil Services Preliminary Examination

06-Indian Polity_Q2005-ENDD.indd 640 8/7/2018 7:51:35 PM


public access to a wealth of technical (M) IAE – International Energy
information relating to those inventions. By Association
filing one international patent application
under the PCT, applicants can simultaneously Q.2264 Consider the following about the International
seek protection for an invention in a very Energy Association (IAE).
large number of countries. 1. It was established in the wake of the 1973
Statement 2: But, the granting of patents oil crisis after the OPEC cartelized a
remains under the control of the national or steep increase in oil price
regional patent Offices in what is called the 2. India is lone associate member of IAE.
“national phase”. 3. The flagship World Energy Outlook is
An international patent application, provided published by IEA.
that it complies with the minimum requirements Select the correct answer using the codes
for obtaining an international filing date, has below.
the effect of a national patent application (and (a)  1 only (b)  2 and 3 only
certain regional patent applications) in or for all (c)  3 only (d)  1, 2 and 3
PCT Contracting States. Solution: (d)
Justification: Statement 1: IEA is an inter-
(L) ADB governmental organization established in
1974 as per framework of the Organisation
Q.2263 Consider the following about the Asian
for Economic Co-operation and Development
Development Bank (ADB).
(OECD).
1. It was founded before World Bank (WB).
Statement 2: India, world’s 3rd largest
2. ADB assists its members by providing
energy consumer has joined International
loans and technical assistance to
Energy Association (IAE)  as associate
promote socio-economic development.
member recently. India will also now have a
3. All its members are from the Asia-Pacific
greater say in global energy issues.
region.
With India as a associate member, IEA
• Select the correct answer using the codes
now formally covers 70% of the world’s
below.
energy consumption, thus increasing its
(a)  1 only (b)  2 and 3 only
relevance.
(c)  2 only (d)  1 and 3 only
Statement 3: It also publishes Key World
Solution: (c)
Energy Statistics and the Monthly Oil Data
Justification: Statement 1: The Asian
Service.
Development Bank was conceived in the
early 1960s as a financial institution that Q.2265 Consider the following about International
would be Asian in character and foster Smart Grid Action Network (ISGAN).
economic growth and cooperation in one of 1. It is an agreement under International
the poorest regions in the world. Energy Agency (IEA).
Statement 2: ADB assists its members (for 2. India has recently applied for the
example, India in infrastructure projects), membership of ISGAN.
and partners, by providing loans, technical Which of the above is/are correct?
assistance, grants, and equity investments to (a)  1 only (b)  2 only
promote social and economic development. (c)  Both 1 and 2 (d)  None
Statement 3: ADB is composed of 67 Solution: (a)
members, 48 of which are from the Asia and Justification: Statements 1 and 2: ISGAN
Pacific region. So, 3 is wrong. is an agreement under International Energy
India became a member of the Asian Agency (IEA)  and consists of representatives
Development Bank (ADB) as a founding from 25 countries across the globe. India is
member in 1966. one of the founding Member of ISGAN and
Japan and the US represent the largest Joint Secretary (Distribution), Ministry of
shareholders. Power, is the member representative of India.

Indian Polity Question Bank P.641

06-Indian Polity_Q2005-ENDD.indd 641 8/7/2018 7:51:35 PM


Learning: ISGAN reports on progress • National Societies can take on additional
and projects being undertaken in the field of humanitarian tasks that are not directly
Smart Grids across the world would also be defined by international humanitarian
helpful in suitable deployment of Smart Grid law or the mandates of the international
technologies in India. Movement.
ISGAN creates a mechanism for • In many countries, they are tightly linked
multilateral government-to-government to the respective national health care
collaboration to advance the development system by providing emergency medical
and deployment of smarter electric grid services (e.g. blood donation camps).
technologies, practices and systems. Q.2267 Consider the following about The Hague
System for the International Registration of
(N) Others Industrial Designs.
Q.2266 The International Red Cross and Red 1. It allows registering industrial designs in
Crescent Movement is a many countries simultaneously by filing
(a) 
Left-Wing extremism movement in one single international application.
certain parts of China 2. If the national registration of a design in
(b) 
Major international humanitarian one country has been invalidated, it is
movement automatically terminated under this system.
(c) 
Biodiversity conservation protocol Which of the above is/are correct?
organization (a)  1 only (b)  2 only
(d) 
Group of War drug mercenaries from (c)  Both 1 and 2 (d)  None
Syria, Turkey and Libya Solution: (a)
Solution: (b) Justification: Statement 1: A business can
Learning: It is an international humanitarian register over 100 designs simultaneously,
movement with millions of volunteers, and save the trouble of filing for separate
many members and staff worldwide. It was applications in over many countries together.
founded to protect human life and health, to Statement 2: As per the relevant
ensure respect for all human beings, and to convention, “Even if the effects of an
prevent and alleviate human suffering. international registration have been
The movement consists of several distinct invalidated in a designated Contracting
organizations that are legally independent Party, such invalidation will not have any
from each other, but are united within the consequences on the validity of the effects
movement through common basic principles, of that international registration in any other
objectives, symbols, statutes and governing designated Contracting Party.”
organisations. The movement’s parts are: Learning: In a legal sense, an industrial
• The International Committee of the Red design constitutes the ornamental or aesthetic
Cross (ICRC) is a private humanitarian aspect of an article.
institution founded in 1863 in Geneva, In principle, the owner of a registered
Switzerland industrial design or of a design patent
• The International Federation of Red Cross has the right to prevent third parties from
and Red Crescent Societies (IFRC) was making, selling or importing articles bearing
founded in 1919. or embodying a design which is a copy,
• On an international level, the Federation or substantially a copy, of the protected
leads and organizes, in close cooperation design, when such acts are undertaken for
with the National Societies, relief commercial purposes.
assistance missions responding to large- Q.2268 Consider the following about International
scale emergencies. Finance Corporation (IFC).
• National Red Cross and Red Crescent 1. It is an arm of the International Monetary
Societies exist in nearly every country in Fund (IMF) administered by the General
the world. For instance it has operations in Plenary Council (GPC).
East Africa through its own offices.
P.642 For Civil Services Preliminary Examination

06-Indian Polity_Q2005-ENDD.indd 642 8/7/2018 7:51:35 PM


2. It focuses exclusively on investing in the • It represents the private-sector’s
private sector in developing countries. views to national governments and
3. India is founding member of IFC. intergovernmental bodies around the
4. The mandate of IFC bars it from investing world. It is not an inter-governmental
into long gestation infrastructure projects. body itself. So, 1 is wrong.
Select the correct answer using the codes • It was the first international organization
below. to be granted general consultative status
(a)  1 and 4 only with the United Nations Economic and
(b)  2, 3 and 4 only Social Council.
(c)  2 and 3 only Learning: ICC had elected founder and
(d)  1 and 3 only chairman of Bharti Enterprises, Sunil Bharti
Solution: (c) Mittal, as its chairman in 2016 session in
Justification: Statement 1, 2 and 3: Brazil. He is the third Indian business leader
International Finance Corporation (IFC), a to hold this position in ICC’s near 100 years
member of the World Bank Group, focuses history.
exclusively on investing in the private sector Q.2270 CARE International is a non-governmental
in developing countries. Established in 1956, organization committed to
IFC has 184 members. India is founding (a) Fighting poverty and empowering poor
member of IFC. (b)  Lobbying with governments to
Statement 4: IFC is an important strengthen democracy and its institutions
development partner for India with its (c) Reconstructing ancient International
operations of financing and advising the pathways
private sector in the country. (d) Stopping infiltration of arms in war
The IFC’s investments in India are spread prone zones
across important sectors like infrastructure, Solution: (a)
manufacturing, financial markets, agribusiness, Learning: They focus in empowering poor
SMEs and renewable energy. So, 4 is wrong. women because women have the power to
Learning: India represents IFC’s single- help whole families and entire communities
largest country exposure globally. IFC has escape poverty.
committed over US$15 billion in India since They also work towards strengthening
1958. capacity for self-help; providing
Keeping in alignment with the Country economic opportunity; delivering relief in
Partnership Strategy (CPS) of the World emergencies; influencing policy decisions at
Bank Group in India, IFC focuses on low- all levels and addressing discrimination in
income states in India. all its forms.
Q.2269 Consider the following about the International For e.g. CARE is providing long-term
Chambers of Commerce (ICC). help to thousands of women struggling to
1. It is an inter-governmental organization raise their families after years of sectarian
chaired by the member nations’ violence in Iraq.
Commerce Ministers in rotation. Q.2271 Consider the following statements about the
2. It has been granted general consultative Alliance for Financial inclusion.
status with the United Nations Economic 1. It organizes the Global Policy Forum
and Social Council. (GPF) which is a forum for government
Which of the above is/are correct? regulatory institutions with an interest in
(a)  1 only (b)  2 only promoting financial inclusion policy.
(c)  Both 1 and 2 (d)  None 2. It championed the Maya Declaration
Solution: (b) which is the first global and measurable
Justification: ICC is the world’s largest set of financial inclusion commitments
business organisation that was founded in by developing and emerging country
1919 headquartered in Paris, France. governments.

Indian Polity Question Bank P.643

06-Indian Polity_Q2005-ENDD.indd 643 8/7/2018 7:51:35 PM


Which of the above is/are correct? Q.2273 Consider the following about the International
(a)  1 only (b)  2 only Astronomical Union (IAU).
(c)  Both 1 and 2 (d)  None 1. It is an Intergovernmental organization.
Solution: (c) 2. It is an organ of United Nations.
Justification: It was founded as a Bill & 3. It approves all space missions that extend
Melinda Gates Foundation - funded project. beyond the orbit of Earth around the Sun.
Statement 1: GPF is organized by AFI as 4. It is the internationally recognized
the keystone event for its membership. authority for assigning designations to
AFI is led by its members and partners, celestial bodies.
central banks and other financial regulatory Choose the correct answer using the codes
institutions from developing countries. The below.
AFI Network includes members from more (a)  1 and 4 only (b)  1 and 2 only
than 80 countries. (c)  4 only (d)  2, 3 and 4 only
Statement 2: The Maya Declaration is a Solution: (c)
statement of common principles regarding It is a collection of professional astronomers, at
the development of financial inclusion the PhD level and beyond, active in professional
policy made by a group of developing nation research and education in astronomy. It acts
regulatory institutions during the AFI 2011 as the internationally recognized authority
Global Policy Forum held in Mexico. for assigning designations to celestial bodies
Commitments under the declaration (stars, planets, asteroids, etc.) and any surface
may include for example, enacting a law to features on them. The IAU is a member of the
regulate the use of electronic money. International Council for Science (ICSU). Its
Q.2272 PolarGAP is an ambitious international main objective is to promote and safeguard
mission to the science of astronomy in all its aspects
1. Capture new and critical data about the through international cooperation. The IAU
Earth’s global gravity field maintains friendly relations with organizations
2. Monitor how Earth’s ice fields are that include amateur astronomers in their
responding to global climate change membership
Which of the above is/are correct? Q.2274 Which of the following countries are
(a)  1 only (b)  2 only members of the European Organisation for
(c)  Both 1 and 2 (d)  None Nuclear Research (CERN)?
Solution: (c) 1. India
Justification: The PolarGAP project will 2. Pakistan
collect new gravity data and combine them 3. Russia
with datasets from other Antarctic missions 4. United Kingdom
to build the first accurately constrained global Select the correct answer using the codes
gravity model. below.
• This is essential as global gravity data (a)  1 and 4 only (b)  1, 2 and 3 only
provide unique information on mass (c)  4 only (d)  None of the above
distribution and transport in the Earth Solution: (c)
System, linked to processes and changes Justification: India recently joined CERN as
in the Solid Earth, hydrology, cryosphere, associate member (not full member).
oceans and atmosphere. CERN has 22 member states (including
• Two earth observing satellite missions UK), four associate member states (including
(GOCE and CryoSat 2) mounted by India and Pakistan) and three International
the European Space Agency (ESA) Organisations have observer status. Russia
revolutionised scientists’ ability to ‘map’ has observer status.
the Earth’s global gravity field and monitor India had ‘observer’ status till September
how Earth’s ice fields are responding to 2016, when the CERN Council adopted a
global change. resolution upgrading its position.

P.644 For Civil Services Preliminary Examination

06-Indian Polity_Q2005-ENDD.indd 644 8/7/2018 7:51:35 PM


Member states have special duties and In news: The Union Cabinet has given
privileges. They make a contribution to its approval to the signing of a MoU
the capital and operating costs of CERN’s between India and the African Asian Rural
programmes, and are represented in the Development Organization (AARDO) for
council, responsible for all important capacity building programmes in the field of
decisions about the organization and its rural development.
activities. Q.2276 Consider the following about the Permanent
Learning: CERN as an organisation is Court of Arbitration (PCA).
world’s largest nuclear and particle physics 1. It is a United Nations Agency.
laboratory. It is situated in North West 2. It is deemed as an international court by
suburbs of Geneva on France-Swiss Border. all United Nations member nations.
It was established in 1954. 3. It can resolve disputes between private
CERN operates the Large Hadron Collider parties arising out of international
(LHC) which is the world’s largest and most agreements.
powerful particle accelerator. It is associated Select the correct answer using the codes
with the discovery of the Higgs Boson which below.
is popularly known as the God particle. (a)  1 and 2 only (b)  3 only
Q.2275 Consider the following about African Asian (c)  2 and 3 only (d)  1 and 3 only
Rural Development Organization (AARDO). Solution: (b)
1. It is an autonomous inter-Governmental Justification: Statements 1 and 2: The PCA
organization headquartered in South is an international organization based in The
Africa. Hague and created by a treaty in 1899.
2. India is one of the Founder Members The PCA is not a court “in the traditional
of the Organization and is the largest sense”, but provides services of arbitral
contributor in terms of membership tribunal to resolve disputes. It does not even
contribution. have permanent judges. The organization is
Which of the above is/are correct? also not a United Nations agency. PCA is an
(a)  1 only (b)  2 only official United Nations Observer.
(c)  Both 1 and 2 (d)  None Statement 3: The PCA is a permanent
Solution: (b) bureaucracy that assists temporary tribunals
Justification: Statement 1 & 2: The to resolve disputes among states (and similar
AARDO, which has its headquarters in New entities), intergovernmental organizations,
Delhi, is an autonomous, inter-Governmental or even private parties arising out of
organization established in 1962 with the international agreements.
objective of promoting cooperation among The cases span a range of legal issues
the countries of the African - Asian Region involving territorial and maritime boundaries,
in the direction of eradicating thirst, hunger, sovereignty, human rights, international
illiteracy, disease and poverty in the region. investment, and international and regional
• India is one of the Founder Members of the trade.
Organization and is the largest contributor For e.g. the India-Bangladesh maritime
in terms of membership contribution apart dispute on EEZ was arbitrated by PCA.
from contributing by way of providing Q.2277 Consider the following about Transports
some fully paid training scholarships Internationaux Routiers (TIR).
for Human Resource Development 1. It aims to harmonize shipping trade
Programme under the Indian Technical between member nations.
& Economic Cooperation Programme 2. It was adopted under the auspices of the
(ITEC) to the Organization. United Nations Economic Commission
• AARDO currently has 31 countries of the for Europe (UNECE).
African - Asian Region under its fold. 3. India’s Union Cabinet has approved the
signing of the TIR Convention.

Indian Polity Question Bank P.645

06-Indian Polity_Q2005-ENDD.indd 645 8/7/2018 7:51:35 PM


4. The treaty prohibits trade transport to is conducive to the growth of Travel &
nations that are not members of the World Tourism. This means planning and investing
Trade Organization (WTO). in appropriate infrastructure and creating a
Select the correct answer using the codes tax regime which allows the private sector to
below. be competitive.
(a)  1 and 2 only (b)  2 and 3 only Q.2279 Consider the following statements about the
(c)  3 and 4 only (d)  1, 2, 3 and 4 International Renewable Energy Agency
Solution: (b) (IRENA).
Justification: Statement 1: It is a multilateral 1. It is an intergovernmental organization.
treaty concluded at Geneva in 1975 to 2. It has three governing bodies (Assembly,
simplify and harmonise the administrative Council and Executives).
formalities of international road transport. 3. India is a permanent member of it.
The TIR system has a globally accepted 4. It is an organ of United Nations.
electronic control system for integrated Which of the above is NOT correct about
transit operations. IRENA?
Statement 2: TIR is the only global (a)  1 and 4 only (b)  3 only
customs transit system that provides easy and (c)  2 and 4 only (d)  3 and 4 only
smooth movement of goods across borders Solution: (c)
in sealed compartments or containers under IRENA is an intergovernmental organization
customs control from the customs office of to promote adoption and sustainable use
departure to the customs office of destination. of renewable energy. It is the global hub
This will allow India to take full benefit for renewable energy cooperation and
of International North South Transportation information exchange within its member
Corridor or INSTC, which enables access to nations. Founded in 2009 and its statute
Eurasian region. entered into force on 8 July 2010. Headquarters
Statement 3: India will become the 71st is in Abu Dhabi. Two governing structures
signatory of the convention as it eyes seamless are present- IRENA Assembly which takes
trade connectivity with both Eurasian region decisions at macro level and gives policy
and Southeast Asia. guidance. IRENA Council is main governing
Statement 4: There is no such provision. body and is responsible for implementing
Q.2278 Consider the following statements about the various decisions of assembly. Members -
World Travel & Tourism Council (WTTC). In total it has 140 Members. India is one of
1. It is an inter-governmental body. the 33 Permanent and founder members of
2. It includes representation from the private IRENA. IRENA is not a UN Body.
sector too. Q.2280 Consider the following about the Organization
3. WTTC campaigns for governments for Islamic Cooperation (OIC).
to implement policies that ensure the 1. All West Asian states, except Iran, hold
business environment is conducive to the the membership of OIC.
growth of Travel & Tourism. 2. India has been blocked by OIC on the
Choose the correct answer from the codes “Kashmir Issue”.
below. Which of the above is/are correct?
(a)  1 and 2 only (b)  2 and 3 only (a)  1 only (b)  2 only
(c)  1 and 3 only (d)  All of the above (c)  Both 1 and 2 (d)  None
Solution: (b) Solution: (b)
WTTC has made it a priority to raise Justification: Statement 1: The OIC is an
awareness of the negative impact punitive international organization founded in 1969.
taxation has – particularly aviation tax - It consists of 57 member states including
on inbound and outbound tourism.WTTC Iran and Pakistan. Administrative centre
campaigns for governments to implement (headquarters) is in Jeddah, Saudi Arabia. So,
policies that ensure the business environment 1 is incorrect.

P.646 For Civil Services Preliminary Examination

06-Indian Polity_Q2005-ENDD.indd 646 8/7/2018 7:51:36 PM


Statement 2: In this organisation, India is Pakistan, Bangladesh, Malaysia among
a blocked country, though it has about 12% others. So, 3 is wrong.
of the world’s Muslim population. India has Q.2282 Which of these states is NOT a member of
been blocked by Pakistan from joining the the Gulf Cooperation Council (GCC)?
OIC over Kashmir issue. OIC regard parts of (a) Saudi Arabia (b) Iraq
Kashmir as “occupied by India”. (c) Iran (d) Kuwait
Learning: Its Objectives: Solution: (b)
(i) Raise the collective voice of the Learning: It is a political and economic
Muslim world. union consisting of all Arab states of the
(ii) Collectively work to safeguard and Persian Gulf, except for Iraq. Its member
protect the interests of the Muslim world in states are Bahrain, Kuwait, Oman, Qatar,
the spirit of promoting international peace Saudi Arabia, and the United Arab Emirates.
and harmony. All current member states are monarchies,
OIC has permanent delegations to United including three constitutional monarchies
Nations and the European Union. (Qatar, Kuwait, and Bahrain).
Q.2281 Consider the following with reference to the In order to reduce their dependence on
Islamic Military Alliance to Fight Terrorism oil in the future, the GCC states are pursuing
(IMAFT). unprecedented structural reform initiatives.
1. It was established by Saudi Arabia. Q.2283 The World Summit on the Information
2. It has been established in pursuance with Society Forum (WSIS) 2017 represents the
objectives and principles of the charter of world’s largest annual gathering of
the Organization of Islamic Cooperation (a) ‘Empowerment of Women’ community
(OIC). (b)  ‘Free Trade’ proponents
3. The organization has been boycotted by (c)  ‘ICT for development’ community
Pakistan. (d)  ‘Global Banking’ community
Select the correct answer using the codes Solution: (c)
below. Learning: The Forum provides an
(a)  1 and 2 only (b)  1 and 3 only opportunity for discussing about Knowledge
(c)  2 only (d)  1, 2 and 3 Societies.
Solution: (a) A knowledge society heavily uses
Justification: Statement 1: IMAFT is Information and Communication Technology
military coalition comprising 39 nations. It (ICT).
was established by Saudi Arabia in December The Forum serves as a key platform for
2015 and its command centre, headquarters is discussing the role of ICTs as a means of
based in Riyadh. implementation of the SDGs.
The coalition calls for all member states to
Q.2284 The Colombo Plan’ is a regional organisation
cooperate to combat terrorism in all its forms
of the Asia-Pacific which has its primary
and manifestations.
focus on
Statement 2: It is envisaged to serve as a
(a) Regulating mineral extraction from the
platform for security cooperation, including
Indian Ocean
provision of training, troops and equipment,
(b)  Human resources development
and involvement of religious scholars for
(c)  Fighting marine piracy
dealing with extremism.
(d) Creating a free trade and special
Member countries of this alliance will
economic zone across South and South-
coordinate and support military operations
East Asia
against terrorism in Iraq, Libya, Syria,
Solution: (b)
Afghanistan and Egypt.
Learning: It is one of the oldest regional
Statement 3: The coalition includes
inter-governmental organizations dates
countries like United Arab Emirates, Turkey,
far back as 1950, when the idea was first
Bahrain, Tunisia, Sudan, Egypt, Yemen,

Indian Polity Question Bank P.647

06-Indian Polity_Q2005-ENDD.indd 647 8/7/2018 7:51:36 PM


conceived to enhance economic and social 2. Granting permanent Intellectual Property
development of the countries of the Asia- Rights (IPR) to people with disability
Pacific region. across national borders
• It was established by Australia, Canada, Which of the above is/are correct?
India, Pakistan, New Zealand, Sri Lanka (a)  1 only (b)  2 only
and the United Kingdom and currently has (c)  Both 1 and 2 (d)  None
expanded to include 26 member countries Solution: (a)
including non-Commonwealth countries Justification: Statement 1: The Marrakesh
and countries belonging to regional Treaty addresses the “book famine” by
groupings such as ASEAN and SAARC. requiring its contracting parties to adopt
• The Colombo Plan is a partnership national law provisions that permit the
concept of self-help and mutual-help in reproduction, distribution and making
development aimed at socio-economic available of published works in accessible
progress of its member countries. formats through limitations and exceptions
• The Colombo Plan is not intended as an to the rights of copyright right holders.
integrated master plan to which national It also provides for the exchange of these
plans were expected to conform. accessible format works across borders by
• It is, instead, a framework for bi-lateral organizations that serve the people who are
arrangements involving foreign aid and blind, visually impaired, and print disabled.
technical assistance for the economic and It will harmonize limitations and exceptions
social development of the region. so that these organizations can operate across
Q.2285 Oxfam is an international confederation of borders.
18 organizations working in more than 90 Statement 2: There is no such provision
countries towards the goal of in the treaty. It is related to the facilitation of
(a) Containing Human Rights Violations access only.
(b)  Fighting poverty However, the Treaty is also designed to
(c)  Ecological conservation provide assurances to authors and publishers
(d) Promoting liberalization and (who may not be disabled) that system will
globalization not expose their published works to misuse or
Solution: (b) distribution to anyone other than the intended
Learning: Around the globe, Oxfam works beneficiaries.
to find practical, innovative ways for people Q.2287 Consider the following with reference to the
to lift themselves out of poverty and thrive. Payment Card Industry Security Standards
Oxfam works directly with communities and Council.
seeks to influence the powerful, to ensure 1. It is an intergovernmental organization
that poor people can improve their lives and regulating the payment cards industry.
livelihoods and have a say in decisions that 2. It designs and promotes standards for
affect them. Oxfam’s involvement in India the safety of cardholder data across the
began when money was granted in 1951 to globe.
fight famine in Bihar. Bihar at the time was Which of the above is/are correct?
one of the poorest and most populated states (a)  1 only (b)  2 only
in India. (c)  Both 1 and 2 (d)  None
Oxfam India was established in 2008 Solution: (b)
under the Companies Act, 2005 as a non Justification: Statement 1: The Council
profitable organization with its head office was founded in 2006 by American Express,
Q.2286 The Marrakesh Treaty facilitates Discover, JCB International, MasterCard and
1. Creation and transfer of specially adapted Visa Inc. They share equally in governance
books across national boundaries for use and execution of the Council’s work.
by visually impaired people It is not an intergovernmental organization.
So, 1 is wrong.

P.648 For Civil Services Preliminary Examination

06-Indian Polity_Q2005-ENDD.indd 648 8/7/2018 7:51:36 PM


Statement 2: The payment card industry Asia, Africa and Europe. OBOR initiative
consists of all the organizations which store, is part of China’s revived 21st century Silk
process and transmit cardholder data, most Road diplomacy that seeks to push it to take a
notably for debit cards and credit cards. bigger role in global affairs as a major global
• The security standards are developed power.
by the Payment Card Industry Security Latvia has become the first country in the
Standards Council which develops the Baltic Sea area to sign a Memorandum of
Payment Card Industry Data Security Understanding to link up with China’s One
Standards used throughout the industry. Belt and One Road (OBOR) Initiative.
• Individual card brands establish
compliance requirements that are used (O) Environment Related
by service providers and have their own
Q.2290 Consider the following about the
compliance programs.
Intergovernmental Panel on Climate Change
Q.2288 Consider the following about the Court of (IPCC).
Arbitration for Sport (CAS). 1. It is a scientific intergovernmental group
1. According to the Olympic Charter, all established by the UNFCCC.
disputes in connection with the Olympic 2. Membership of the IPCC is open to all
Games can only be submitted to CAS. member countries of the United Nations
2. Appeals against the orders of CAS (UN).
can be filed in the IOC disciplinary 3. It conducts authoritative research on
commission. climate related issues.
Which of the above is/are correct? 4. It monitors climate related parameters
(a)  1 only (b)  2 only such as Mean Sea Level (MSL).
(c)  Both 1 and 2 (d)  None 5. Its recommendations are binding on all
Solution: (a) members of the UNEP.
CAS is an international quasi-judicial body Select the correct answer using the codes
established to settle disputes related to sport below.
through arbitration. (a)  1, 4 and 5 only
Statement 1: According to rule 61 of the (b)  1, 2, 3 and 4 only
Olympic Charter, all disputes in connection (c)  2 only
with the Olympic Games can only be (d)  1, 2, 3, 4 and 5
submitted to CAS. Since 2016, an anti-doping Solution: (c)
division of CAS was given full authority to Justification: IPCC is the leading
judge doping cases at the Olympic Games. international body for the assessment of
Statement 2: It had replaced earlier climate change.
mechanism of the IOC disciplinary It was established by the United Nations
commission. So, 2 is incorrect. Environment Programme (UNEP) and
Q.2289 The Silk Road Economic Belt and the the World Meteorological Organization
Maritime Silk Road are parts of the (WMO) in 1988 to provide the world with a
(a)  North-South Transport Corridor clear scientific view on the current state of
(b)  One Belt and One Road (OBOR) knowledge in climate change and its potential
(c) India-Myanmar-Thailand Trilateral environmental and socioeconomic impacts.
Highway So, 1 is wrong.
(d)  Kaladan Multi-modal project The IPCC reviews and assesses the
Solution: (b) most recent scientific, technical and socio-
Learning: The One Belt and One Road economic information produced worldwide
(OBOR) initiative is China’s ambitious relevant to the understanding of climate
development strategy and framework that change. It does not conduct any research
aims to boost its connectivity and trade across nor does it monitor climate related data or
parameters. So, 3 and 4 are wrong.

Indian Polity Question Bank P.649

06-Indian Polity_Q2005-ENDD.indd 649 8/7/2018 7:51:36 PM


As an intergovernmental body, Based mostly in the developing world, it has
membership of the IPCC is open to all many regional, national or project offices
member countries of the United Nations in all continents and a head office in the
(UN) and WMO. Currently 195 countries are Netherlands.
Members of the IPCC. So, 2 is correct. Statement 2: Ramsar convention was
Q.2291 Consider the following about Conservation held in 1971. It was founded in 1954 as
International (CI). the International Wildfowl Inquiry and the
1. It is an inter-governmental organization organisation was focused on the protection
of the Conference of Parties (COP) based of waterbirds. Later, the name became
in Hague. International Waterfowl & Wetlands
2. Its work focuses on science, policy, Research Bureau (IWRB).
and partnership with businesses and Q.2293 Consider the following about International
communities with regard to climate Carbon Action Partnership (ICAP).
change and environment. 1. It was a brainchild of the former United
Which of the above is/are correct? Nations Secretary General Ban Ki-Moon.
(a)  1 only (b)  2 only 2. It helps developing countries build
(c)  Both 1 and 2 (d)  None capacity on designing and implementing
Solution: (b) carbon cap and trade systems.
Justification: Statement 1: It is an American Which of the above is/are correct?
non-profit environmental organization (a)  1 only (b)  2 only
headquartered in Arlington, Virginia. Its goal (c)  Both 1 and 2 (d)  None
is to protect nature as a source of food, fresh Solution: (b)
water, livelihoods and a stable climate. Learning: Statement 1: It was founded in 2007
It publishes the Ocean Health Index by more than 15 government representatives
(OHI). It was founded in 1987 with the aim as an international cooperative forum to
of analysing the problems most dangerous or implement emissions trading systems (ETS).
harmful to nature and building a foundation The rationale behind ICAP’s work is that
dedicated to solving these issues on a global linking carbon cap and trade systems would
scale. lead to economic, social and environmental
Statement 2: It is working in more than benefits
30 countries. It has helped establish 1,200 Statement 2: ICAP acts as a knowledge
protected areas across 78 countries and sharing hub, disseminating knowledge on
protected more than 730 million hectares of existing and planned ETS, as well as the
land, marine and coastal areas. general benefits and design aspects of an
Q.2292 Consider the following about Wetlands ETS.
International. ICAP also runs workshops and side
1. It is the only global not-for-profit events at the United Nations Climate Change
organisation dedicated to the conservation Conferences.
and restoration of wetlands. Q.2294 Consider the following about the Crop Trust,
2. The body was formed immediately after formerly known as the Global Crop Diversity
the Ramsar Convention. Trust.
Which of the above is/are correct? 1. It is an attached fund with the International
(a)  1 only (b)  2 only Food Policy Research Institute (IFPRI).
(c)  Both 1 and 2 (d)  None 2. It is a non-profit organization which
Solution: (a) works to preserve crop diversity in order
Justification: Statement 1: It is an to protect global food security.
independent, not-for-profit, global 3. It is a part of the funding strategy of the
organisation, supported by government and International Treaty on Plant Genetic
NGO membership from around the world. Resources for Food and Agriculture.

P.650 For Civil Services Preliminary Examination

06-Indian Polity_Q2005-ENDD.indd 650 8/7/2018 7:51:36 PM


Select the correct answer using the codes UNEP-WCMC has a mandate to facilitate
below. the delivery of the global indicators under
(a)  2 only (b)  2 and 3 only the CBD’s 2010 Biodiversity Target on the
(c)  3 only (d)  1 and 3 only rate of loss of biological diversity, and works
Solution: (b) alongside the CITES Secretariat producing a
Justification: Statement 1: The Crop Trust range of reports and databases.
is an established independent organization It also manages the World Database of
under international law. Protected Areas in collaboration with the
• The Crop Trust was founded in 2004 in IUCN World Commission on Protected
Rome, Italy by the Food and Agriculture Areas.
Organization and Bioversity International Q.2296 Consider the following about Alliance for
on behalf of the CGIAR international Zero Extinction (AZE).
agricultural research consortium. 1. It is an inter-governmental organization
• Throughout its 10-year history, the working towards ecological conservation.
Crop Trust has made significant leaps in 2. It assists party nations in integrating
ensuring the conservation and availability protection of AZE sites and species into
of crop diversity for food security world- National Biodiversity Strategies and
wide. Action Plans (NBSAP).
• Statement 2 and 3: In 2006, the Crop Which of the above is/are correct?
Trust entered into an agreement with the (a)  1 only (b)  2 only
governing body of the International Treaty (c)  Both 1 and 2 (d)  None
on Plant Genetic Resources for Food and Solution: (b)
Agriculture. Justification: Statement 1: The Alliance
• The agreement recognises the Crop Trust for Zero Extinction – a joint initiative of
as an “essential element” of the treaty’s around 93 non-governmental biodiversity
funding strategy in regards to the ex situ conservation organizations from 37 countries
conservation and availability of plant around the world, with frequent additions of
genetic resources for food and agriculture. new members.
• It also confirms the autonomy of the Crop Statement 2: Alliance for Zero Extinction
Trust as a scientific organization in raising (AZE) sites contain the entire population of
and disbursing funds. one or more species listed as Endangered or
Q.2295 The World Conservation Monitoring Centre Critically Endangered on the IUCN Red List
(WCMC) is an executive agency of of Threatened Species.
(a) Ministry of Environment, Forest Following identification, the Alliance
and Climate Change (MoEFCC), aims to act together to eliminate threats and
Government of India restore habitat at these sites to allow species
(b)  World Wild Life Fund (WWF) populations to rebound.
(c) International Union for Conservation of Q.2297 “Addis Ababa Action Agenda 2015” is a
Nature (IUCN) historic agreement reached by countries on
(d) United Nations Environment Programme (a)  Stopping tacit neo-colonization
(UNEP) (b) Global framework for financing
Solution: (d) Sustainable Development Goals (SDGs)
Learning: The activities of UNEP–WCMC (c) Tackling trans-national communicable
include biodiversity assessment, support diseases by establishing an International
to international conventions such as the Border Health Force (IBHF)
CBD and the CITES, capacity building and (d) Exchanging sensitive defence
management of both aspatial and spatial information between members of the
data on species and habitats of conservation United Nations (UN)
concern. Solution: (b)

Indian Polity Question Bank P.651

06-Indian Polity_Q2005-ENDD.indd 651 8/7/2018 7:51:36 PM


Justification: Domestic resource mobilization responsibility in transfers of conventional
is central to the agenda. In the outcome arms and dual-use goods and technologies
document, countries agreed to an array of Option D: It deals with nuclear fuel and
measures aimed at widening the revenue base, supplies.
improving tax collection, and combating tax Q.2299 The international group “New Agenda for
evasion and illicit financial flows. Countries Coalition (NAC)” sometimes seen in news
also reaffirmed their commitment to official (a) Promotes the interest of Least
development assistance, particularly for the Development Countries (LDCs) in
least developed countries, and pledged to getting greater export market share in
increase South-South cooperation. developed countries
Learning: There were agreements for (b) Vouches for climate change financing
international cooperation for financing of from developed countries for developing
specific areas where significant investments countries
are needed, such as in infrastructure for (c) Promotes the Non-Proliferation
energy, transport, water and sanitation, and Treaty (NPT) and pushes for nuclear
other areas to help realize the proposed disarmament worldwide
sustainable development goals. (d) Advocates global economic governance
reforms in the World Bank and IMF
(P) Weapon Ban Treaties Solution: (c)
Q.2298 Which of these groups help member Learning: NAC is a geographically
countries to identify those exports which dispersed group of middle power countries
need to be controlled so as not to contribute that promotes the NPT and pushes for nuclear
to the spread of chemical and biological disarmament worldwide.
weapons? It consists of Brazil, Egypt, Ireland,
(a)  Australia Group Mexico, New Zealand and South Africa.
(b) Missile Technology Control Regime The group was officially launched in
(MTCR) Dublin (Ireland)  in 1998 in response to the
(c)  Wassenaar Arrangement North-South divide that stymied talks on
(d)  Nuclear Suppliers Group (NSG) nuclear disarmament and non-proliferation
Solution: (a) within the framework of the NPT.
Justification: Option A: The Australia Q.2300 Consider the following about the way nuclear
Group, an informal group of countries (now explosions are monitored in the world
joined by the European Commission) was presently by the Comprehensive Nuclear Test
established in 1985 after the use of chemical Ban Treaty Organization (CTBTO).
weapons by Iraq in 1984. Assertion (A): CTBTO uses infrasonic
Its members are supposed to maintain sensors for monitoring atmospheric nuclear
export controls on a uniform list of several explosions.
chemical compounds. Reason (R): Infrasonic sensors measure
Option B: The Missile Technology micropressure changes in the atmosphere
Control Regime (MTCR) is a multilateral which are generated by the propagation of
export control regime. It is an informal and infrasonic waves.
voluntary partnership among countries to In the context of the above, which of these
prevent the proliferation of certain category is correct?
of missile and unmanned aerial vehicle (a) A is correct, and R is an appropriate
technology. explanation of A.
Option C: The Wassenaar Arrangement (b) A is correct, but R is not an appropriate
was established to contribute to regional explanation of A.
and international security and stability (c)  A is correct, but R is incorrect.
by promoting transparency and greater (d)  Both A and R are incorrect.
Solution: (a)

P.652 For Civil Services Preliminary Examination

06-Indian Polity_Q2005-ENDD.indd 652 8/7/2018 7:51:36 PM


Justification: Acoustic waves with very low Q.2302 Pelindaba Treaty aims to
frequencies are called infrasound. In fact, (a) To preventing nuclear proliferation and
these waves are below the frequency band preventing strategic minerals of Africa
audible to the human ear, which typically from being exported freely
ranges from 20 to 20,000 Hertz. (b) To evacuate refugees from war prone
Infrasound is produced by a variety of zones with minimal intervention
natural and man-made sources: exploding (c) To bring foreign direct investment in
volcanoes, earthquakes, meteors, storms and sensitive zones
auroras in the natural world; nuclear, mining (d) To increase official development
and large chemical explosions, as well as assistance without the need for legislative
aircraft and rocket launches in the man-made approval
arena. Solution: (a)
CTBTO has 60 IMS stations in 35 The Pelindaba Treaty, also known as the
countries around the world. These stations African Nuclear Weapon Free Zone Treaty,
catch the very low frequency waves aims at preventing nuclear proliferation and
created by atmospheric nuclear explosions. preventing strategic minerals of Africa from
Infrasound stations relay data continuously being exported freely.
in real time. Q.2303 The Hague Code of Conduct (HCOC) is a/an
Q.2301 Established in 1987, the voluntary Missile (a) Set of norms followed by every refugee-
Technology Control Regime (MTCR) aims protecting nation
to limit the spread of ballistic missiles and (b) Informal code related to proceedings in
other unmanned delivery systems that could the International Court of Justice (ICJ)
be used for chemical, biological, and nuclear (c) Epidemic related advisory issued by the
attacks. How is India joining the Missile World Health Organization (WHO)
Technology Control Regime (MTCR) (d) International Code of Conduct against
significant? Ballistic Missile proliferation
1. All member states would agree to transfer Solution: (d)
latest missile technologies to India with Learning: The HCOC is the result of
safeguards. international efforts to regulate access to
2. India will get access to nuclear equipment ballistic missiles which can potentially
and materials from member states. deliver weapons of mass destruction.
Which of the above is/are correct? The HCOC does not ban ballistic missiles,
(a)  1 only (b)  2 only but it does call for restraint in their production,
(c)  Both 1 and 2 (d)  None testing, and export. India has joined the The
Solution: (d) Hague Code of Conduct against Ballistic
Justification: Statement 1: Joining the Missile Proliferation by notifying HCoC
MTCR does not make getting missile Central Contact, Vienna, through diplomatic
technology easier. There are no special channels. India’s joining the Code signals
concessions for MTCR members. But India our readiness to further strengthen the global
hopes its MTCR membership will be one non-proliferation regimes.
more reason for the US to consider exporting Q.2304 Consider the following about Wassenaar
Category 1 UAVs, Reaper and Global Hawk, Arrangement.
which have been key to counter-terrorism 1. It concerns with transfers of conventional
efforts in Afghanistan, Pakistan, Somalia and arms and dual-use goods and technologies.
Yemen. 2. It is a sub-treaty under the Missile
Statement 2: It would happen if India is Technology Control Regime(MTCR).
admitted to NSG. MTCR does not deal with 3. India is a member of the Wassenaar
nuclear material, nor are all its members Arrangement.
nuclear powers 4. It does not extend to web-based systems.

Indian Polity Question Bank P.653

06-Indian Polity_Q2005-ENDD.indd 653 8/7/2018 7:51:36 PM


Select the correct answer using the codes Prohibition of the Development, Production,
below. Stockpiling and Use of Chemical Weapons
(a)  1, 2 and 4 only (b)  1 only and on their Destruction (CWC).
(c)  2 and 3 only (d)  1, 2, 3 and 4 This convention outlaws the production,
Solution: (b) stockpiling, and use of chemical weapons
Justification: Statement 1: It promotes and their precursors. 190 member-states have
transparency and greater responsibility in signed and ratified this convention including
transfers of conventional arms and dual-use India. Six states- Angola, Egypt, Israel,
goods and technologies, thus preventing Myanmar, North Korea and South Sudan are
destabilising accumulations. The aim is also still outside the CWC.
to prevent the acquisition of these items by Q.2306 Consider the following about the Organisation
terrorists. for the Prohibition of Chemical Weapons
Statement 2: Australia Group, Nuclear (OPCW).
Suppliers Group and Missile Technology 1. It encourages international cooperation in
Control Regime are different arrangements peaceful uses of chemistry.
than Wassenar. 2. It provides protection and assistance to
Statement 3: India and China are non- parties against chemical weapons.
participating states. 3. It had received the Noble Prize in
Statement 4: New technologies Chemistry for its decryption of complex
(amendment in 2013) placed under the chemical weapons.
export control regime include “intrusion Select the correct answer using the codes
software”—software designed to defeat a below.
computer or network’s protective measures (a)  1 only (b)  2 and 3 only
so as to extract data or information—as well (c)  1 and 2 only (d)  1, 2 and 3
as IP network surveillance systems. Solution: (c)
Q.2305 Consider the following about OPCW Justification: Statements 1 and 2: It has four
(Organisation for the Prohibition of Chemical broad aims:
Weapons). • to ensure a credible and transparent
1. It is an inter-governmental organization regime for verifying the destruction of
headquartered in Hague. chemical weapons and to prevent their
2. One of its mandates is to destroy the re-emergence, while protecting legitimate
global chemical weapons stockpile. national security and proprietary interests;
3. It operates directly under the aegis • to provide protection and assistance
of United Nations Security Council against chemical weapons;
(UNSC). • to encourage international cooperation in
4. India is not a signatory to the Convention peaceful uses of chemistry; and
on Chemical weapons • to bring about universal membership of
Choose the correct answer using the codes the OPCW by facilitating international
below. cooperation and national capacity
(a)  1 and 2 only (b)  3 and 4 only building.
(c)  2 and 4 only (d)  1 and 3 only Statement 3: It received the Noble Peace
Solution: (a) Prize in 2013 because it had, with the
Recently, Nobel peace prize winning Chemical Weapons Convention, “defined the
organisation OPCW (Organisation for the use of chemical weapons as a taboo under
Prohibition of Chemical Weapons) has international law”.
achieved major milestone of destroying Q.2307 The Organisation for the Prohibition of
90 percent of the global chemical weapons Chemical Weapons (OPCW) is
stockpile. (a) An intergovernmental organisation that
OPCW promotes, administers and verifies verifies the adherence to the Chemical
the adherence to the Convention on the Weapons Convention

P.654 For Civil Services Preliminary Examination

06-Indian Polity_Q2005-ENDD.indd 654 8/7/2018 7:51:36 PM


(b) An international non-profit body that regulations and be expected to track the
monitors global proliferation of chemical destination of exports to ensure they do
weapons not end up in the wrong hands. Ideally,
(c) A subsidiary organization of the UN that means limiting the inflow of deadly
reporting to the UNSC on matters related weapons into places like Syria.
to Chemical weapons Statement 2: The UN Office for
(d) An arm of the Nuclear Suppliers Group Disarmament Affairs claimed the treaty
(NSG) to prevent access of harmful would not interfere with domestic arms
chemical weapons in wrong hands commerce or the right to bear arms in its
Solution: (a) member states; ban the export of any type
Learning: The activities of the OPCW and of weapon; harm the legitimate right to
its core organisational structure are described self-defence; or undermine national arms
in the Chemical Weapons Convention (whose regulation standards already in place.
members are all in OPCW). Statement 3: India has stressed
The principal body is the conference of consistently that the ATT should ensure a
states parties, which normally is convened balance of obligations between exporting and
yearly, and in which all countries participate importing states.
and have equal voting rights. All States • However, the treaty that is annexed to the
Parties make contributions to the OPCW resolution is weak on terrorism and non-
budget, based on a modified UN scale of state actors and these concerns find no or
assessments. weak mention in the specific prohibitions
of the Treaty.
(Q) International Treaty • Further, India cannot accept that the
Treaty be used as an instrument in the
Q.2308 Consider the following about the landmark
hands of exporting states to take unilateral
Arms Trade Treaty (ATT).
force majeure measures against importing
1. The Treaty obligates member states
states parties without consequences.
to monitor arms exports and ensure
that weapons don’t cross existing arms Q.2309 With reference to the International Energy
embargoes. Charter, consider the following:
2. Signatories of the ATT are bound to dilute 1. It is a declaration of political intention
the right to bear arms by general public aiming at strengthening energy
inside their territory and gradually revoke cooperation between the signatory states.
the right. 2. It is legally binding on member states and
3. India did not sign the ATT because it involves annual financial commitment to
believed the treaty fails to make a real the corpus of the charter.
impact on illicit trafficking of arms and 3. India recently agreed to sign and ratify
its use by terrorists. the charter.
Select the correct answer using the codes Select the correct answer using the codes
below. below.
(a)  1 and 2 only (b)  2 and 3 only (a)  2 only (b)  1 and 3 only
(c)  1 and 3 only (d)  1, 2 and 3 (c)  1 only (d)  2 and 3 only
Solution: (c) Solution: (c)
Justification: Statement 1: The treaty Background: The Energy Charter Treaty
regulates the international trade in (ECT) is an international agreement which
conventional arms – from small arms to establishes a multilateral framework for
battle tanks, combat aircraft and warships – cross-border cooperation in the energy
and entered into force in 2014. industry. It strives to promote principles of
• Member states, with the assistance of openness of global energy markets and non-
the U.N., will put into place enforceable, discrimination to stimulate foreign direct
standardized arms import and export investments and global cross-border trade.

Indian Polity Question Bank P.655

06-Indian Polity_Q2005-ENDD.indd 655 8/7/2018 7:51:36 PM


Justification: Statement 1: In 2015, a major 3. Pakistan
milestone in modernisation and expansion 4. South Sudan
of the Energy Charter was reached, as 72 5. Sri Lanka
Countries plus the EU signed the International Select the correct answer using the codes
Energy Charter. below.
The International Energy Charter is a (a)  1, 3 and 4 only
political declaration about principles for (b)  1, 2, 4 and 5 only
international energy cooperation. (c)  3, 4 and 5 only
Statement 2: No such commitments are (d)  1 and 3 only
provided for, however it is intended as a first Solution: (a)
step towards accession to the legally binding Learning: A total of 191 states have joined
Energy Charter Treaty. the Treaty, though North Korea, which
Statement 3: The treaty covers all aspects acceded to the NPT in 1985 but never came
of commercial energy activities including into compliance, announced its withdrawal
trade, transit, investments and energy in 2003. Four UN member states have never
efficiency. India is not a signatory. joined the NPT: India, Israel, Pakistan and
Q.2310 India has not signed the Nuclear Non- South Sudan.
proliferation Treaty (NPT) of 1968 and the The treaty recognizes five states as
Comprehensive Test Ban Treaty (CTBT) of nuclear-weapon states: the United States,
1996 because Russia, the United Kingdom, France, and
(a) India did not have nuclear weapons when China (also the P-5) India and Pakistan have
the treaties were signed. publicly disclosed their nuclear weapon
(b) The treaties were discriminatory and programs, and Israel has a long-standing
hegemonistic in nature in India’s view. policy of deliberate ambiguity with regards
(c) The treaty demands sharing of crucial to its nuclear program.
and strategic information of a nation’s
nuclear facilities.
(d) The treaty demands complete control of the
concerned nuclear authority of the nation.
Solution: (b)
Justification: By not signing the Nuclear
Non-proliferation Treaty (NPT) of 1968 and
the Comprehensive Test Ban Treaty (CTBT)
of 1996, India has kept its nuclear options
open. India opposses NPT and CTBT due to
their discriminatory and hegemonistic nature.
They perpetuate an international system in
which only five nations (USA, Russia, China,
UK and France) can legitimately possess
nuclear weapons. India is already sharing
information regarding its nuclear facilities
with IAEA. So, this could not have been a
contention while signing the NPT and CTBT.
(c)  will be wrong thus.
Q.2311 Apart from India, which of the following UN
member states have NOT joined the Treaty
on the Non-Proliferation of Nuclear Weapons
(NPT)?
1. Israel
2. Nigeria

P.656 For Civil Services Preliminary Examination

06-Indian Polity_Q2005-ENDD.indd 656 8/7/2018 7:51:36 PM


References
1 Indian Polity (5th edition) – Laxmikanth http://www.pmindia.gov.in – Prime minister
2 Introduction to constitution of India (22 of India
edition) – D D Basu http://rajyasabha.nic.in – Rajya Sabha
3 Indian Constitution – P M Bakshi 5 https://legalpoint-india.blogspot.in
4 Official websites of 6 Press Information Bureau
http://eci.nic.in/eci/eci.html – Election 7 PRS india
commission of India 8 Newspaper articles of Indian express,
https://www.india.gov.in/ – National portal The Hindu, Times of India etc
of India 9 Official websites of ministries
http://parliamentofindia.nic.in/ – Parliament 10 Official website of international
of India organisations http://www.un.org/en/
https://presidentofindia.nic.in/ – President of index.html
India
http://vicepresidentofindia.nic.in/ – Vice
president of india
http://supremecourtofindia.nic.in/ – Supreme
court of India

Indian Polity Question Bank P.657

06-Indian Polity_Q2005-ENDD.indd 657 8/7/2018 7:51:36 PM

Potrebbero piacerti anche